67% found this document useful (3 votes)
3K views856 pages

Gate Architecture and Planning:: Comprehensive Question Bank (1991-2023)

Title: GATE Architecture and Planning: Comprehensive Question Bank (1991-2023) Author: Hemant Vilas Parulekar Description: This comprehensive question bank is an essential resource for GATE Architecture and Planning aspirants. Authored by Hemant Vilas Parulekar, the book covers a vast array of practice questions from the years 1991 to 2023, each meticulously solved and explained in detail. This collection is designed to help students thoroughly prepare for the GATE exam by providing them with

Uploaded by

Hemant Parulekar
Copyright
© © All Rights Reserved
We take content rights seriously. If you suspect this is your content, claim it here.
67% found this document useful (3 votes)
3K views856 pages

Gate Architecture and Planning:: Comprehensive Question Bank (1991-2023)

Title: GATE Architecture and Planning: Comprehensive Question Bank (1991-2023) Author: Hemant Vilas Parulekar Description: This comprehensive question bank is an essential resource for GATE Architecture and Planning aspirants. Authored by Hemant Vilas Parulekar, the book covers a vast array of practice questions from the years 1991 to 2023, each meticulously solved and explained in detail. This collection is designed to help students thoroughly prepare for the GATE exam by providing them with

Uploaded by

Hemant Parulekar
Copyright
© © All Rights Reserved
We take content rights seriously. If you suspect this is your content, claim it here.
You are on page 1/ 856

GATE

ARCHITECTURE
AND PLANNING:
COMPREHENSIVE QUESTION
BANK
Years 1991-2023
Practice Questions with Detailed Solutions

HEMANT VILAS PARULEKAR

INDI A SINGAPORE M A L AY S I A
Copyright © Hemant Vilas Parulekar 2024
All Rights Reserved.

ISBN 979-8-89277-646-2

This book has been published with all efforts taken to make the material error-free after the consent of the
author. However, the author and the publisher do not assume and hereby disclaim any liability to any party
for any loss, damage, or disruption caused by errors or omissions, whether such errors or omissions result
from negligence, accident, or any other cause.

While every effort has been made to avoid any mistake or omission, this publication is being sold on the
condition and understanding that neither the author nor the publishers or printers would be liable in any
manner to any person by reason of any mistake or omission in this publication or for any action taken or
omitted to be taken or advice rendered or accepted on the basis of this work. For any defect in printing or
binding the publishers will be liable only to replace the defective copy by another copy of this work then
available.
CONTENTS

1 GATE QUESTION PAPER 2023 1

2 GATE QUESTION PAPER 2022 37

3 GATE QUESTION PAPER 2021 91

4 GATE QUESTION PAPER 2020 133

5 GATE QUESTION PAPER 2019 167

6 GATE QUESTION PAPER 2018 205

7 GATE QUESTION PAPER 2017 242

8 GATE QUESTION PAPER 2016 272

9 GATE QUESTION PAPER 2015 294

10 GATE QUESTION PAPER 2014 320

11 GATE QUESTION PAPER 2013 340

12 GATE QUESTION PAPER 2012 365

13 GATE QUESTION PAPER 2011 385

14 GATE QUESTION PAPER 2010 410

15 GATE QUESTION PAPER 2009 435

16 GATE QUESTION PAPER 2008 460

17 GATE QUESTION PAPER 2007 487

18 GATE QUESTION PAPER 2006 520

19 GATE QUESTION PAPER 2005 553

20 GATE QUESTION PAPER 2004 575

21 GATE QUESTION PAPER 2003 603

22 GATE QUESTION PAPER 2002 641

23 GATE QUESTION PAPER 2001 655


24 GATE QUESTION PAPER 2000 671

25 GATE QUESTION PAPER 1999 675

26 GATE QUESTION PAPER 1998 690

27 GATE QUESTION PAPER 1997 717

28 GATE QUESTION PAPER 1996 743

29 GATE QUESTION PAPER 1995 772

30 GATE QUESTION PAPER 1994 795

31 GATE QUESTION PAPER 1993 821

32 GATE QUESTION PAPER 1992 829

33 GATE QUESTION PAPER 1991 841


GATE QUESTION PAPER 2023
General Aptitude (GA)

Q. 1 – Q. 5 carry one mark each.


Q.1 He did not manage to fix the car himself, so he _______ in the garage.
(A) got it fixed (B) getting it fixed (C) gets fixed (D) got fixed
Ans:- (A)
Q.2 Planting : Seed : : Raising : _____ (By word meaning)
(A) Child (B) Temperature (C) Height (D) Lift
Ans:- (A)
Q.3 A certain country has 504 universities and 25951 colleges. These are categorised into Grades I, II,
and III as shown in the given pie charts.
What is the percentage, correct to one decimal place, of higher education institutions (colleges and
universities) that fall into Grade III?

Universities Colleges
Grade Grade I Grade II Grade III Grade Grade I Grade II Grade III
Percentage 31% 62% 7% Percentage 9% 68% 23%
(A) 22.7 (B) 23.7 (C) 15.0 (D) 66.8
Ans:- (A)
Solution:- In order to calculate the percentage of Grade III institutions, we need to multiply the total
number of each type of institution by the percentage that falls into Grade III. Then add these two
amounts together and divide by the total number of institutions.
For universities, this is 504 * 0.07 = 35.28 (approx 35 universities).
For colleges, this is 25951 * 0.23 = 5972.73 (approx 5973 colleges).
So, there are approximately 6008 Grade III institutions.
The total number of institutions is 504 (universities) + 25951 (colleges) = 26455.
Therefore, the percentage of Grade III institutions is (6008 / 26455) * 100 = 22.7%.
So, the answer is (A) 22.7
2 GATE Architecture and Planning: Comprehensive Question Bank

Q.4 The minute-hand and second-hand of a clock cross each other _______ times between 09:15:00
AM and 09:45:00 AM on a day.
(A) 30 (B) 15 (C) 29 (D) 31
Ans:- (D)
Q.5 The symbols ○, *, ∆, and □ are to be filled, one in each box, as shown below.
The rules for filling in the four symbols are as follows.
1) Every row and every column must contain each of the four symbols.
2) Every 2×2 square delineated by bold lines must contain each of the four symbols.

Which symbol will occupy the box marked with ‘’ in the partially filled figure
(A) ○ (B) * (C) ∆ (D) □
Ans:- (A)
Q. 6 – Q. 10 Carry TWO marks each.
Q.6 In a recently held parent-teacher meeting, the teachers had very few complaints about Ravi. After
all, Ravi was a hardworking and kind student. Incidentally, almost all of Ravi’s friends at school
were hardworking and kind too. But the teachers drew attention to Ravi’s complete lack of interest
in sports. The teachers believed that, along with some of his friends who showed similar disinterest
in sports, Ravi needed to engage in some sports for his overall development.
Based only on the information provided above, which one of the following statements can be
logically inferred with certainty?
(A) All of Ravi’s friends are hardworking and kind.
(B) No one who is not a friend of Ravi is hardworking and kind.
(C) None of Ravi’s friends are interested in sports.
(D) Some of Ravi’s friends are hardworking and kind.
Ans:- (D)
Explanation:- Based on the information provided in the text, the statement that can be logically
inferred with certainty is:
(D) Some of Ravi’s friends are hardworking and kind.
The text does not provide enough information to confidently affirm options (A), (B), and (C).
Hemant Vilas Parulekar 3

Q.7 Consider the following inequalities 𝑝2 − 4𝑞 < 4


3𝑝 + 2𝑞 < 6
where 𝑝 and 𝑞 are positive integers.
The value of (𝑝 + 𝑞) is _______.
(A) 2 (B) 1 (C) 3 (D) 4
Ans:- (A)
Solution: Solving 𝑝2 − 4𝑞 < 4 for positive integers gives us possibilities like (p=1, q=0), (p=2, q=1).
Solving 3𝑝 + 2𝑞 < 6 for positive integers gives us possibilities like (p=1, q=1), (p=2, q=0), (p=1, q=2).
The only solution that satisfies both inequalities is (p=1, q=1), so the sum (𝑝 + 𝑞) = 1 + 1 = 2.
Q.8 Which one of the sentence sequences in the given options creates a coherent narrative?
(i) I could not bring myself to knock.
(ii) There was a murmur of unfamiliar voices coming from the big drawing room and the door was
firmly shut.
(iii) The passage was dark for a bit, but then it suddenly opened into a bright kitchen.
(iv) I decided I would rather wander down the passage.
(A) (iv), (i), (iii), (ii) (B) (iii), (i), (ii), (iv) (C) (ii), (i), (iv), (iii) (D) (i), (iii), (ii), (iv)
Ans:- (C)
Explanation:- The first sentence sets the scene by describing the drawing room and the closed door.
The second sentence shows that the narrator is hesitant to knock on the door. The third sentence shows
that the narrator decides to wander down the passage instead. The fourth sentence describes what the
narrator sees when they reach the end of the passage.
The other options are incorrect because they do not create a coherent narrative. For example, option
(A) starts with sentence (iv), but sentence (iv) only makes sense after the narrator has decided to
wander down the passage. Option (B) starts with sentence (iii), but sentence (iii) only makes sense after
the narrator has entered the passage. Option (D) starts with sentence (i), but sentence (i) only makes
sense after the narrator has seen the closed door.
Q.9 How many pairs of sets (S, T) are possible among the subsets of {1, 2, 3, 4, 5, 6} that satisfy the
condition that S is a subset of T?
(A) 729 (B) 728 (C) 664 (D) 665
Ans:- No option is correct
Solution:- The total number of subsets of a set with n elements is 2n. So, a set with 6 elements has 26 =
64 subsets.
For each subset S, there is exactly one corresponding subset T that is equal to S. For every other subset
T, S is a proper subset of T.
Therefore, the number of pairs (S, T) such that S is a subset of T is equal to the number of subsets (64)
plus the number of proper subsets for each subset.
The number of proper subsets of a subset with k elements is 2k - 1. Summing this over all k from 0 to 6
gives the total number of pairs (S, T) such that S is a proper subset of T.
So, the total is 64 + (20 - 1) + (21 - 1) + (22 - 1) + (23 - 1) + (24 - 1) + (25 - 1) + (26 - 1) = 64 + 0 + 1 + 3 +
7 + 15 + 31 + 63 = 185.
Adding these gives 64 (for the pairs where S = T) + 185 (for the pairs where S is a proper subset of T)
= 249 pairs. So, none of the provided options (A, B, C, D) is correct. The correct answer should be 249.
4 GATE Architecture and Planning: Comprehensive Question Bank

Q.10 An opaque pyramid (shown below), with a square base and


isosceles faces, is suspended in the path of a parallel beam of light,
such that its shadow is cast on a screen oriented perpendicular to
the direction of the light beam. The pyramid can be reoriented
in any direction within the light beam. Under these conditions,
which one of the shadows P, Q, R, and S is NOT possible?
(A) P (B) Q
(C) R (D) S

Ans:- (A)
PART A: Common FOR ALL CANDIDATES
Q.11 – Q.28 Each question carries one mark and negative marking of 1/3rd mark for each wrong
answer in MCQ. (Total marks = 18)
Q.11 The triad of secondary colours in the additive colour system is _________.
(A) Cyan, Magenta, Yellow (B) Red, Green, Blue
(C) Purple, Green, Orange (D) Magenta, Blue, Yellow
Ans:- (A)
Explanation: In the additive colour system, primary colours (Red, Green, Blue) are combined to create
secondary colours. Cyan is created by combining Green and Blue, Magenta is created by combining
Red and Blue, and Yellow is created by combining Red and Green.
Q.12 The criterion that is specifically mentioned in Special Conditions of Contract (SCC) is__________.
(A) Scope and performance of the work (B) Site mobilization advance
(C) Labour regulation (D) Arbitration and law
Ans:- (B)
Explanation: While all the options mentioned could potentially be included in a contract, site
mobilization advance is typically addressed in the Special Conditions of Contract (SCC). This refers to
the funds that are provided to the contractor to cover initial costs associated with preparing the site for
construction. Other details such as the scope of work, labour regulations, and arbitration procedures
are generally covered in the general conditions of the contract rather than the special conditions.
Q.13 The command employed in AutoCAD® to create a mesh from a line or curve that is swept along a
straight path (as shown in the figure below) is _________.

(A) TABSURF (B) REVSURF (C) RULESURF (D) EDGESURF


Ans:- (A)
Hemant Vilas Parulekar 5

Q.14 As per the Burra Charter (2013) ‘Cultural Significance’ means __________ for past, present, or
future generations.
(A) historic, aesthetic, scientific, social or spiritual value
(B) archaeological, architectural, environmental, cultural value
(C) natural, cultural, mixed, intangible heritage
(D) heritage value, authenticity, integrity
Ans:- (A)
Explanation:- The charter is a guide for the conservation and management of places of cultural
significance and is widely accepted and applied in Australia and globally.
Q.15 As per URDPFI (2015), the density range (in persons per Hectare) suggested for overall planning
approach for small towns in hill areas is ________.
(A) 20-30 (B) 45-70 (C) 100-125 (D) 125-150
Ans:- (B)
Explanation: Under table no. 5.1, guideline no. 5.3.1 of URDPFI 2015, the density range for small
towns in hill areas is mentioned.
Q.16 In ecology, the term ‘niche’ refers to ______.
(A) the ways in which species interact with biotic and abiotic factors of the environment
(B) only the abiotic factors such as temperature and rainfall
(C) the gradient change of physiochemical characteristics between two ecosystems
(D) the zone of junction or a transition area between two biomes
Ans:- (A)
Explanation:- An ecological niche refers to the role and position a species has in its environment,
including how it interacts with resources and other species. It involves various factors including the
physical factors like the conditions and resources it needs for survival and reproduction, as well as
interactions with other organisms including predation, competition and symbiosis.
Q.17 Lowry’s model of Metropolis (1964) includes two _____________ spatial interaction models
(A) Singly constrained (B) Doubly constrained
(C) Unconstrained (D) Triply constrained
Ans:- (A)
Q.18 Select the method(s) that involve(s) a pairwise comparison matrix for quantifying the weights of
decision criteria.
(A) Analytical hierarchy process (B) Exploratory factor analysis
(C) Latent class analysis (D) Multiple linear regression
Ans:- (A)
Explanation:- Analytical hierarchy process (AHP) is a structured technique for dealing with complex
decisions. It uses mathematics and psychology to improve the decision-making process when the
decision maker is not sure of the available alternatives. It does this by constructing a hierarchy of criteria
and alternatives, then making pairwise comparisons of each against all others, and finally calculating a
priority vector of the alternatives.
6 GATE Architecture and Planning: Comprehensive Question Bank

Q.19 Select the micro-organism which is NOT an enteric pathogen.


(A) Staphylococcus aureus (B) Vibrio cholerae
(C) Escherichia coli (D) Salmonella typhi
Ans:- (A)
Explanation: (A) Staphylococcus aureus - This is a bacterium that is often found on the skin and in
the nose of healthy people. It can cause a range of illnesses from minor skin infections, such as pimples
and boils, to life-threatening diseases, such as pneumonia, meningitis, and sepsis. It is not typically
classified as an enteric pathogen.
(B) Vibrio cholerae - This bacterium causes cholera, which is an acute diarrheal illness. The infection
is often mild or without symptoms, but can sometimes be severe and life-threatening.
(C) Escherichia coli (E. coli) - Most strains of E. coli are harmless and live in the intestines of healthy
humans and animals. However, some strains can cause diarrhea, urinary tract infections, respiratory
illness, and other illnesses. Some of these pathogenic strains are considered enteric pathogens.
(D) Salmonella typhi - This bacterium causes typhoid fever, a life-threatening illness. It is transmitted
through the ingestion of food or drink contaminated by the feces of an infected person. It’s considered
an enteric pathogen because it affects the intestinal tract.
Q.20 Select the publication by Ministry of Statistics and Programme Implementation (MoSPI) related
to Environmental Accounts as per UN-SEEA framework.
(A) EnviStats India 2022
(B) Energy Conservation Building Code 2017
(C) Eco Niwas Samhita 2018
(D) Climate Change 2022: Impacts, Adaptation and Vulnerability
Ans:- (A)
Explanation:- The Ministry of Statistics and Programme Implementation (MoSPI) publishes EnviStats
India, which includes environmental statistics as well as accounts as per the United Nations System
of Environmental-Economic Accounting (UN-SEEA) framework. This publication provides data
on various environmental and economic indicators, offering a comprehensive view of the interplay
between the economy and the environment.
The other options are not publications by the Ministry of Statistics and Programme Implementation
and are not directly related to the UN-SEEA framework:
(B) Energy Conservation Building Code 2017 - This is a building code related to energy conservation
published by the Bureau of Energy Efficiency (BEE), under the Ministry of Power, India. It’s not related
to environmental accounts or the UN-SEEA framework.
(C) Eco Niwas Samhita 2018 - This is a residential building code for energy efficiency, also under the
Ministry of Power in India. It’s not a publication of MoSPI or related to UN-SEEA.
(D) Climate Change 2022: Impacts, Adaptation and Vulnerability - This appears to reference a
report by the Intergovernmental Panel on Climate Change (IPCC), which is an international body for
assessing the science related to climate change. It’s not a publication of the MoSPI.
Hemant Vilas Parulekar 7

Q.21 Ebenezer Howard suggested the maximum population of ‘Garden City’ as________ persons.
(A) 10,000 (B) 22,000 (C) 32,000 (D) 58,000
Ans:- (C)
Explanation:- A Garden City is a town planned in an idealized way, with green spaces and self-
sufficient industries. The concept of Garden Cities was first proposed by Ebenezer Howard in his book
To-morrow: A Peaceful Path to Real Reform, published in 1898.
Howard believed that the Garden City would provide a solution to the problems of urban congestion
and rural poverty. He proposed that Garden Cities should be limited to a population of 32,000, which
he believed was the optimum size for a town to be both efficient and livable.
The Garden City would be surrounded by a green belt, which would prevent it from becoming too
large. The town would be self-sufficient, with its own industries, schools, and shops. Howard also
believed that the Garden City would be a more democratic and equitable place to live, with a strong
sense of community.
The Garden City movement was influential in the development of urban planning in the 20th century.
Many of the principles of Garden Cities can be seen in modern towns and cities around the world.
Q.22 In eighteenth century English gardens, __________was used to eliminate visual boundaries
between the garden and the landscape.
(A) Stroll garden (B) Sunken face (C) Topiary (D) Qanat
Ans:- (B)
Explanation::-
Term Stroll Garden Sunken Fence
Explanation This is a type of garden, especially The “ha-ha” or “sunken fence” was a land-
common in Japanese landscape scape design element used in 18th-century
architecture, where the arrangement English gardens to create a seamless visual
of plants, water features, and other transition from the garden to the natural
elements are designed to create an landscape. It served to keep livestock out
aesthetically pleasing scene. The while providing an uninterrupted view.
purpose is to create a journey or
experience for the person walking
through the garden, where they can
enjoy different views and elements as
they stroll along.
Image
8 GATE Architecture and Planning: Comprehensive Question Bank

Term Topiary Qanat


Explanation This is the practice of training and A qanat is an ancient system of
trimming live plants into specific underground tunnels that carry water
shapes. This is often done with shrubs from a source, usually a mountain, to
and trees. The plants are shaped into areas where it is needed for irrigation or
decorative, geometric, or fanciful drinking. This system was commonly
forms. Topiary gardens were especially used in the Middle East and North Africa.
popular in Europe from the Renaissance While not typically associated with English
until the 18th century. gardens, the qanat is a significant historical
method of landscape irrigation and water
supply.
Plant

Q. 23 - Q. 28 Numerical Answer Type (NAT) (NO negative marking)


Q.23 The figure below shows the spatial arrangement of rooms in a
building with access from the exterior, marked as ‘entry’. Identify
the appropriate diagram showing the access to rooms starting
from the entry.

(A) (B)

(C) (D)

Ans:- (B)
Hemant Vilas Parulekar 9

Q.24 In high-rise buildings, the method adopted to prevent ingress of smoke in an enclosed fire staircase
is ____________.
(A) Polarization (B) Pressurization (C) Perpetuation (D) Fumigation
Ans:- (B)
Explanation: Pressurization is a technique used to maintain a higher air pressure inside the fire
staircase compared to the surrounding areas. This positive pressure prevents the entry of smoke from
spreading into the staircase and helps to maintain a clear and smoke-free escape route for building
occupants during a fire emergency.
Pressurization systems typically involve the use of fans and air supply systems to introduce fresh air
into the fire staircase, creating a higher pressure zone. This higher pressure prevents the flow of smoke
into the staircase through gaps, cracks, and other openings.
It’s important to note that pressurization systems should be designed and installed according to relevant
fire safety codes and standards to ensure their effectiveness in preventing smoke ingress and facilitating
safe evacuation during a fire incident.
Q.25 Select the Act which stipulates prohibited area of 100 m around centrally protected monuments in
India.
(A) The Antiquities and Art Treasures Act, 1972 (B) The AMASR (Amendment and Validation) Act, 2010
(C) Urban Land (Ceiling and Regulation) Act, 1976 (D) Environment Protection Act, 1986
Ans:- (B)
Explanation: The AMASR Act, 2010, stands for “The Ancient Monuments and Archaeological Sites
and Remains (Amendment and Validation) Act, 2010”. This act was introduced to amend and validate
certain provisions of the existing Ancient Monuments and Archaeological Sites and Remains Act, 1958.
Under the AMASR Act, a prohibited area of 100 m is established around centrally protected
monuments. This means that any construction or development activities within the 100 m radius of
these monuments are prohibited without obtaining the necessary permissions and clearances from the
relevant authorities, primarily the Archaeological Survey of India (ASI).
The purpose of establishing the prohibited area is to ensure the protection, preservation, and
maintenance of the cultural heritage represented by these centrally protected monuments. It helps
prevent encroachments, unauthorized constructions, and other activities that could potentially harm
or degrade the monuments and their surroundings. By maintaining the integrity of the prohibited
area, the AMASR Act aims to safeguard the historical and cultural significance of these monuments for
future generations.
Q.26 Select the option(s) which include(s) a pair of ‘Gestalt’ principles.
(A) Proximity and Similarity (B) Continuity and closure
(C) Grain and Texture (D) Scale and Proportion
Ans:- (A) and (B)
Explanation: The Gestalt principles are a set of principles in psychology that describe how humans
perceive and organize visual information. They help explain how people perceive patterns, forms, and
structures in visual stimuli. Here is a brief explanation of the two Gestalt principles mentioned:
10 GATE Architecture and Planning: Comprehensive Question Bank

1. Continuity: This principle states that when people see a series of points or lines, they tend to perceive
them as continuous and flowing in a particular direction. It suggests that our brains naturally prefer
smooth and continuous paths rather than abrupt changes. It helps in perceiving lines or curves that
are partially obscured or interrupted as a continuous whole.
2. Closure: This principle states that when presented with an incomplete or partially obscured object,
people tend to mentally complete the missing parts and perceive the object as a whole. It suggests
that our brains have a tendency to fill in the gaps to perceive complete and meaningful shapes. This
principle helps us make sense of fragmented or ambiguous visual stimuli.
3. Proximity: This principle states that when elements are placed close to each other, they are perceived
as a group or related to each other. Objects or elements that are physically near each other are
perceived as being more related or belonging to the same group.
4. Similarity: This principle states that when elements share similar visual characteristics such as shape,
size, color, or texture, they are perceived as belonging to the same group or category. Elements that
have similar attributes are grouped together in our perception.
Q.27 Select the option(s) which is/are NOT considered as primary air pollutant(s).
(A) Suspended particulate matter (B) Oxides of Nitrogen
(C) Volatile organic compounds (D) Peroxyacetyl Nitrate
Ans:- (C) and (D)
Explanation: peroxyacetyl nitrate (D) is not directly emitted into the atmosphere as a primary
pollutant. It is formed as a secondary pollutant through chemical reactions involving nitrogen oxides
(NOx) and volatile organic compounds (VOCs) in the presence of sunlight. Peroxyacetyl nitrate is
known to contribute to photochemical smog and can have detrimental effects on respiratory health.
Q.28 Select the Biosphere Reserve(s) in India which is/are listed in the ‘Man and the Biosphere’ program
of UNESCO.
(A) Sunderban (B) Sena Oura (C) Majang Forest (D) Gulf of Mannar
Ans:- (A) and (D)
Explanation: The following biosphere reserves in India are listed in the ‘Man and the Biosphere’
program of UNESCO:
• Sunderban
• Gulf of Mannar
• Nilgiri
• Nanda Devi
• Pachmarhi
• Similipal
• Achanakmar-Amarkantak
• Great Nicobar
• Agasthyamala
• Kanchenjunga
• Panna
Hemant Vilas Parulekar 11

Q.29 – Q .49 Carry TWO marks Each


Q.29 Match the buildings in Group I with their dominant spatial pattern in Group II.
(A) P-1, Q-3, R-2, S-5 Group I Group II
(B) P-5, Q-3, R-1, S-4 (P) National Assembly Building, (1) Centralized organization
(C) P-3, Q-4, R-1, S-5 Capitol Complex, Dhaka
(Q) Secretariat Building, UNESCO (2) Clustered organization
(D) P-1, Q-4, R-2, S-3
Headquarters, Paris
(R) Fatehpur Sikri Palace Complex (3) Radial organization
(S) Shodhan House, Ahmedabad (4) Linear organization
(5) Grid organization

Ans:- (D)
Explanation:
• P - The National Assembly Building in Dhaka is a centralized building because it has a single,
dominant center. The circular plan of the building reinforces this sense of centralization.
• Q - The Secretariat Building in Paris is a linear building because it is arranged along a central
axis. The axis runs through the center of the building, and the other rooms and corridors are
arranged on either side of it.
• R - The Fatehpur Sikri Palace Complex is a radial building because it is organized around a
central courtyard. The courtyard is the focal point of the complex, and the other buildings and
gardens are arranged around it.
• S - The Shodhan House in Ahmedabad is a grid organization because it is made up of a series
of rectangular blocks that are arranged on a grid pattern. This grid pattern is evident in the floor
plan of the building, as well as in the exterior facades. The grid organization of the Shodhan
House allows for a variety of different spatial arrangements within the building. For example,
the blocks can be used to create different sized rooms, or they can be used to create open-plan
spaces. The grid organization also allows for a lot of natural light to enter the building, which is
important in a hot climate like Ahmedabad.
Q.30 Match the Parts of Residential Buildings in Group-I with their respective minimum width (in m)
in Group-II as per the National Building Code 2016
(A) P-2, Q-1, R-5, S-3 Group I Group II
(B) P-5, Q-3, R-4, S-1 (P) Habitable room (1) 1.0
(C) P-2, Q-3, R-5, S-4 (Q) Stair flight (2) 3.0
(R) Kitchen (3) 1.2
(D) P-5, Q-1, R-4, S-3
(S) Bathroom (4) 1.8
(5) 2.4
Ans:- (D)
12 GATE Architecture and Planning: Comprehensive Question Bank

Q.31 Match the following City Planning concepts in Group-I with their proponents in Group-II
(A) P-2, Q-4, R-5, S-3 Group I Group II
(B) P-3, Q-4, R-5, S-2 (P) Radiant City 1. Clarence Perry
(C) P-3, Q-2, R-1, S-4 (Q) Conservative surgery 2. Soria-Y-Mata
(R) Broadacre City 3. Le Corbusier
(D) P-1, Q-5, R-3, S-2
(S) Linear City 4. Patrick Geddes
5. Patrick Geddes

Ans:- (B)
Explanation: The following are the references for the city planning concepts and their proponents:
• Radiant City: Le Corbusier’s Radiant City was a utopian vision of a city that was organized
around a system of high-rise towers and green spaces. The concept was first proposed in Le
Corbusier’s book The City of Tomorrow (1922).
• Conservative surgery: Soria-Y-Mata’s Conservative surgery was a more incremental approach
to city planning that focused on preserving the existing fabric of the city while making
improvements. The concept was first proposed in Soria-Y-Mata’s book The Modern City (1929).
• Broadacre City: Frank Lloyd Wright’s Broadacre City was a vision of a decentralized city that
was organized around the automobile. The concept was first proposed in Wright’s book The
Disappearing City (1932).
• Linear City: Patrick Geddes’ Linear City was a proposal for a city that was organized along a
linear corridor. The concept was first proposed in Geddes’ book Cities in Evolution (1915).
The other options are incorrect because they do not match the city planning concepts and their
proponents. For example, option (A) matches the Radiant City with Clarence Perry, but the Radiant
City was actually proposed by Le Corbusier. Option (C) matches the Conservative surgery with Patrick
Geddes, but the Conservative surgery was actually proposed by Soria-Y-Mata. Option (D) matches the
Broadacre City with Frank Lloyd Wright, but the Broadacre City was actually proposed by Frank Lloyd
Wright.
Q.32 With reference to planning and design of housing, identify the correct statements
(P) Gross population density is higher than net population density
(Q) Gross population density is lower than net population density
(R) Net population density is directly proportional to area of the plot
(S) Net population density is inversely proportional to area of the plot
(A) Both Q and S are correct (B) Both Q and R are correct
(C) Both P and R are correct (D) Both P and S are correct
Ans:- (A)
Hemant Vilas Parulekar 13

Q.33 Match the Mission in Group I with thetheir objectives in Group II.
Group I Group II
(P) National Mission on Enhanced (1). Gain better understanding of climate science, impacts,
Energy Efficiency challenges by setting up climate research fund
(Q) National Mission on Sustainable (2). Weather insurance mechanism and afforestation of 6
Habitat million hectares of degraded forest land
(R) National Water Mission (3). Decrease energy consumption in large consuming
industries
(S) National Mission on Strategic (4). 20% improvement of water use efficiency through
Knowledge for Climate Change pricing
(5). Enforcement of automotive fuel economy standards
using pricing measures
(A) P-3, Q-5, R-4, S-1 (B) P-2, Q-5, R-4, S-3 (C) P-3, Q-4, R-5, S-1 (D) P-2, Q-5, R-3, S-4
Ans (A)
Explanation:-
• National Mission on Enhanced Energy Efficiency: This mission aims to reduce energy
consumption in India by 20% by 2022. One of its objectives is to decrease energy consumption
in large consuming industries.
• National Mission on Sustainable Habitat: This mission aims to make Indian cities more
sustainable and resilient to climate change. One of its objectives is to promote weather insurance
mechanism and afforestation of degraded forest land.
• National Water Mission: This mission aims to ensure water security for all by 2022. One of its
objectives is to improve water use efficiency by 20% through pricing measures.
• National Mission on Strategic Knowledge for Climate Change: This mission aims to improve
India’s understanding of climate change and its impacts. One of its objectives is to set up a
climate research fund to support research on climate change.
Q.34 Select the option(s) that is/are listed among the Sustainable Development Goals as articulated by
the United Nations.
(A) Globalization and Free Trade (B) Sustainable Cities and Communities
(C) Protection of Indigenous Culture and Architecture (D) Good Health and Well-being
Ans (B) and (D)
Explanation:-
• (B) Sustainable Cities and Communities is one of the 17 Sustainable Development Goals (SDGs)
adopted by the United Nations in 2015. The goal is to make cities and human settlements
inclusive, safe, resilient and sustainable.
• (D) Good Health and Well-being is another SDG that aims to ensure healthy lives and promote
well-being for all at all ages.
The other options, (A) Globalization and Free Trade and (C) Protection of Indigenous Culture and
Architecture, are not among the SDGs.
14 GATE Architecture and Planning: Comprehensive Question Bank

Q.35 Select the statement(s) that are TRUE regarding ‘Building Security Services’.
(A) ‘Radio Frequency Identification Device’ is used for electronic access control system.
(B) ‘Magnetic Loop Detector’ is used for fire detection system.
(C) ‘Infrared Sensor’ is used in public broadcasting system.
(D) ‘Iris Scan’ is a type of biometric access control system.
Ans (A) and (D)
Explanation:-
• (A) Radio Frequency Identification Device (RFID) is used for electronic access control system.
It is a technology that uses radio waves to identify and track objects. RFID tags can be attached
to people, vehicles, or other objects. When an RFID tag comes within range of an RFID reader,
the reader can read the tag’s unique identification number. This information can be used to
control access to a building or other facility.
• (D) Iris Scan is a type of biometric access control system. Biometric access control systems use
physical characteristics, such as fingerprints, iris scans, or facial recognition, to identify and
authenticate users. Iris scans are considered to be one of the most secure biometric identification
methods because they are unique to each individual.
The other statements are not true.
• (B) Magnetic Loop Detector is used for security applications, such as perimeter security and
intrusion detection. It is a type of passive sensor that detects changes in the magnetic field.
When a person or object moves through the magnetic field, it creates a disturbance that is
detected by the sensor.
• (C) Infrared Sensor is used in a variety of applications, including security, surveillance, and
medical imaging. It is a type of sensor that detects infrared radiation. Infrared radiation is
invisible to the human eye, but it can be detected by infrared sensors. This makes infrared
sensors useful for applications where it is important to see in the dark.
Q.36 Select the statement(s) that are TRUE regarding ‘Quality and Cost-Based Selection (QCBS)’ system
for tendering.
(A) Financial bid is opened before technical bid.
(B) Earnest Money Deposit (EMD) is submitted before the opening of technical bid.
(C) Technically qualified bidder with lowest financial bid is always awarded the job.
(D) A composite scoring system considering both the financial and technical bids is adopted for
awarding the job.
Ans:- (B) & (D)
Explanation:-
• Statement (A) is false because the technical bid is opened first, followed by the financial bid.
This is to ensure that the technically qualified bidders are evaluated before their financial bids
are considered.
• Statement (C) is false because the technically qualified bidder with the lowest financial bid is not
always awarded the job. The QCBS system takes into account both the technical and financial
bids, and the bidder with the highest composite score is awarded the job.
Hemant Vilas Parulekar 15

• Statement (B) is true because the EMD is submitted before the opening of the technical bid. This
is to ensure that the bidders are serious about their bids and that they will not withdraw their
bids after the technical bid has been opened.
• Statement (D) is true because the QCBS system uses a composite scoring system that considers
both the technical and financial bids. The weights assigned to the technical and financial bids
can be different, depending on the specific requirements of the tender.
Q.37 Design of septic tank requires consideration of space for the following item(s).
(A) Settling of incoming sewage (B) Storage of digested sludge
(C) Installation of liner to allow seepage of effluent (D) Digestion of the settled sludge
Ans:- (B) & (D)
Q.38 Select the place(s) which have adopted the “star pattern” of the French Garden in the design of its /
their urban form(s).
(A) Versailles (B) Washington D.C. (C) Islamabad (D) Jaipur
Ans:- (A) & (B)
Explanation:- The “star pattern” of the French Garden is a radiating pattern of streets and avenues
that is based on the design of the gardens at the Palace of Versailles. This pattern was first used in
Washington D.C., which was designed by Pierre L’Enfant in the late 18th century. The star pattern was
also used in the design of Islamabad, which was built in the mid-20th century.
Q.39 Select the parameter(s) required for determining peak rates of runoff using the Rational formula.
(A) Intensity of rainfall (B) Coefficient of runoff
(C) Velocity of flow (D) Hydraulic mean depth of flow
Ans:- (A) & (B)
Explanation:- Q = CIA
where:
• Q is the peak rate of runoff (in cubic feet per second)
• C is the coefficient of runoff (a dimensionless number)
• I is the intensity of rainfall (in inches per hour)
• A is the drainage area (in acres)
The velocity of flow and the hydraulic mean depth of flow are not required for determining peak rates
of runoff using the Rational formula.
Q.40 As per Solid Waste Management Rules 2016 (Ministry of Environment, Forest and Climate, Govt. of
India) select the correct statement(s).
(A) “dry waste” means waste other than bio-degradable waste and inert street sweepings.
(B) “combustible waste” means biodegradable, recyclable, reusable, hazardous solid waste having
maximum calorific value of 800 kcal/kg.
(C) “domestic hazardous waste” includes expired medicine, CFL bulbs, discarded
paint drums.
(D) “biodegradable waste” means any inorganic material that cannot be degraded by
micro-organisms into simpler stable compounds.
16 GATE Architecture and Planning: Comprehensive Question Bank

Ans:- (A) & (C)


Explanation:- The Solid Waste Management Rules 2016 defines combustible waste as “any solid waste,
which, when ignited, burns with ease and rapidity and is capable of producing heat.” This definition
does not include any reference to a maximum calorific value of 800 kcal/kg.
Therefore, the statements in options (B) and (D) are incorrect. The correct statements are:
• Dry waste means waste other than bio-degradable waste and inert street sweepings.
• Domestic hazardous waste includes expired medicine, CFL bulbs, discarded paint drums, and
other household items that can be harmful to human health or the environment.
Q.41 Select the correct statement(s) from the following.
(A) Introduction of automobiles led to urban sprawl
(B) Compact cities show relatively higher carbon emissions.
(C) Land use and transportation planning is inter-dependent on each other.
(D) Addition of a transport mode in an urban area does not change accessibility.
Ans:- (A) & (C)
Explanation: (A) The introduction of automobiles has contributed to urban sprawl, as increased car
ownership and reliance on private vehicles have led to the expansion of cities outward, resulting in low-
density developments and increased distances between residential, commercial, and industrial areas.
(C) Land use and transportation planning are interdependent on each other. The way land is used for
various purposes, such as housing, businesses, and amenities, affects transportation needs and patterns.
Similarly, transportation infrastructure and systems influence land use decisions and accessibility to
different areas within a city. Effective planning requires a coordinated approach to ensure that land use
and transportation systems support each other.
Q.42 Choose the correct statement(s) with regard to composting.
(A) It produces natural soil amendment and enhances the effectiveness of fertilizer
(B) Warm temperature of tropical regions is least suitable for composting.
(C) Composting is an aerobic thermophilic process.
(D) Windrow composting and in-vessel composting are two of the common composting methods.
Ans:- (C) & (D)
Explanation:- (A) Composting produces natural soil amendment by breaking down organic materials
into nutrient-rich compost. This compost can be used to improve soil quality and enhance the
effectiveness of fertilizers.
(C) Composting is an aerobic process, which means it occurs in the presence of oxygen. It is also
a thermophilic process, meaning it generates heat and requires elevated temperatures to promote
microbial activity and decomposition.
(D) Windrow composting and in-vessel composting are two common methods of composting.
Windrow composting involves piling organic materials in long rows, while in-vessel composting is
done in enclosed containers or vessels that provide controlled conditions for composting.
(B) The statement that the warm temperature of tropical regions is least suitable for composting is
incorrect. Composting can occur in various climates, including tropical regions, although the process
may need to be adapted to account for higher temperatures and moisture levels. Proper management
techniques can be employed to ensure successful composting even in warm climates.
Hemant Vilas Parulekar 17

Q.43 Select the item(s) that are NOT stipulated as obligatory function(s) of the urban local bodies as per
the 12th Schedule of the Indian Constitution.
(A) Urban poverty alleviation
(B) Promotion of cultural, educational and aesthetic aspects
(C) Special measures for disaster mitigation
(D) Prevention of cruelty to animals
Ans:- (C) & (D)
Explanation:- As per the 74th Amendment Act, 1992, the 12th Schedule of the Indian Constitution
outlines the functions and responsibilities of urban local bodies (municipalities) in India. The functions
mentioned in the 12th Schedule are categorized into two parts - obligatory functions and discretionary
functions.
According to the 12th Schedule, the obligatory functions of urban local bodies include various aspects
such as urban planning, regulation of land use, water supply, sanitation, and more. Additionally, it
includes functions related to:
(C) Special measures for disaster mitigation: Urban local bodies are mandated to take special measures
to mitigate the impact of disasters, including preparedness, response, and recovery activities.
(D) Prevention of cruelty to animals: Urban local bodies are responsible for preventing cruelty to
animals and ensuring their welfare within the urban areas.
Q.44 The annual precipitation recorded in a town is 400 mm. Rainwater is being collected from the flat
roof of a building, and then treated to potable standards, and stored. Water loss due to evaporation,
transmission and treatment is 40 percent of the total harvested volume. The roof area is 500 sq.m.
There are 3 occupants, with average daily water demand as 200 lpcd. The stored rainwater will be
adequate for the household’s daily use for ________ days [in integer].
Ans:- 200
Solution:- Here are the steps to calculate how many days the stored rainwater will be adequate for the
household’s daily use:
1. Calculate the total volume of rainwater that can be harvested from the roof:
0 500 sq.m * 400 mm = 200,000 liters
2. Calculate the volume of rainwater that will be lost due to evaporation, transmission, and treatment:
0 200,000 liters * 40% = 80,000 liters
3. Calculate the total volume of rainwater that will be available for storage:
0 200,000 liters - 80,000 liters = 120,000 liters
4. Calculate the daily water demand of the household:
0 3 occupants * 200 liters/occupant/day = 600 liters/day
5. Calculate the number of days the stored rainwater will be adequate for the household’s daily use:
0 120,000 liters / 600 liters/day = 200 days
Therefore, the stored rainwater will be adequate for the household’s daily use for 200 days.
18 GATE Architecture and Planning: Comprehensive Question Bank

Q.45 A primary school is having 8 class rooms, each having internal dimensions of 15m × 10m × 4m
(height). Only the internal walls of all the class rooms are proposed to be painted. Assume a
deduction of 10% internal wall area due to doors, windows etc. The specification suggests two
coats of paint application. The spreading rates of the selected paint during base coat and finish
coat are 4.5 l/sq.m and 2.5 l/sq.m respectively. The amount of paint (in liters) required for the job
will be _______ [in integer].
Ans:- 10080
Solution:- To calculate the amount of paint required for the job, we need to consider the following
steps:
1. Calculate the total area of the internal walls of all the classrooms: Total area = (Number of classrooms
× Internal wall area per classroom) × (1 - Deduction percentage) Total area = (8 classrooms × 2 ×
(15m × 4m + 10m × 4m)) × (1 - 0.10) Total area = 8 × 2 × (60 sq.m + 40 sq.m) × 0.9 Total area = 8 ×
2 × 100 sq.m × 0.9 Total area = 1440 sq.m
2. Calculate the amount of paint required for the base coat: Base coat paint required = Total area
× Spreading rate for base coat Base coat paint required = 1440 sq.m × 4.5 l/sq.m Base coat paint
required = 6480 liters
3. Calculate the amount of paint required for the finish coat: Finish coat paint required = Total area ×
Spreading rate for finish coat Finish coat paint required = 1440 sq.m × 2.5 l/sq.m Finish coat paint
required = 3600 liters
4. Calculate the total amount of paint required: Total paint required = Base coat paint required + Finish
coat paint required Total paint required = 6480 liters + 3600 liters Total paint required = 10080 liters
Therefore, the amount of paint required for the job will be 10080 liters.
Q.46 A construction project consists of four activities. The quantity of work, manpower requirement,
and the productivity of the activities are listed in the table below. The interrelationship between
the activities are also mentioned in the table. The construction project will start on January 29.
Assume no holidays for the entire duration of the project. The project will finish on February
_____ [mention date in number].
Activity Quantity Manpower Productivity Immediate
(in cu.m.) (persons) (cu.m/ man-day) Successor Activity
A 96 8 3 C
B 252 7 4 C&D
C 275 5 5 Nil
D 126 6 3 Nil

Ans:- 17
Q.47 For a privately developed group housing project, the ratio of total number of dwelling units of
HIG: MIG: LIG is 3:2:1. The proposed average size of HIG, MIG and LIG units in sq.m are 100,
60 and 30 respectively. The ratio of the total built up area between (MIG + LIG) to HIG will be 1:
______ [in integer].
Hemant Vilas Parulekar 19

Ans:- 2
Solution:- To determine the ratio of the total built-up area between (MIG + LIG) and HIG units,
we need to calculate the total built-up area for each category and then compare them. Let’s calculate:
Total number of HIG units = 3x Total number of MIG units = 2x Total number of LIG units = x
Average size of HIG units = 100 sq.m Average size of MIG units = 60 sq.m Average size of LIG
units = 30 sq.m
Total built-up area for HIG units = Total number of HIG units x Average size of HIG units Total
built-up area for HIG units = (3x) x 100 = 300x sq.m
Total built-up area for MIG units = Total number of MIG units x Average size of MIG units Total
built-up area for MIG units = (2x) x 60 = 120x sq.m
Total built-up area for LIG units = Total number of LIG units x Average size of LIG units Total built-up
area for LIG units = (x) x 30 = 30x sq.m
Total built-up area for (MIG + LIG) units = Total built-up area for MIG units + Total built-up area for
LIG units Total built-up area for (MIG + LIG) units = 120x + 30x = 150x sq.m
Now, let’s calculate the ratio of (MIG + LIG) to HIG units:
Ratio = (Total built-up area for (MIG + LIG) units) / (Total built-up area for HIG units) Ratio
= (150x) / (300x) Ratio = 1/2
Therefore, the ratio of the total built-up area between (MIG + LIG) to HIG units is 1:2.
Q.48 The surface development of a three
dimensional object is shown in the
figure below. The dotted lines
indicate the folds. The dimensions
given in the figure are in centimeter.
The volume of the three-
dimensional object (in cu.cm)
is___________ [rounded off to one
decimal place].
Ans:- 25.5
Q.49 A residential housing project is designed in a plot measuring 1 hectare. The car parking area is
equally distributed between the ground floor and the basement. Considering the data given below,
the number of cars accommodated in the basement wills be______________ [in integer].
Data:
FAR consumed = 2.0
Car parking area is exempted from built up area for FAR calculations.
One car parking to be given for each 100 sq.m of built up area.
Area required for accommodating each car in ground floor = 15 sq.m
Area required for accommodating each car in basement = 25 sq.m
20 GATE Architecture and Planning: Comprehensive Question Bank

Ans:- 75
Solution:- The number of cars accommodated in the basement is 75.
Here are the steps to calculate the answer:
1. Calculate the total built up area:
0 1 hectare = 10,000 sq.m
0 FAR consumed = 2.0
0 Total built up area = FAR * plot area = 2 * 10,000 sq.m = 20,000 sq.m
2. Calculate the number of car parking spaces:
0 1 car parking space for every 100 sq.m of built up area
0 Total number of car parking spaces = 20,000 sq.m / 100 sq.m/car = 200 car parking spaces
3. Calculate the number of car parking spaces in the basement:
0 50% of the total car parking spaces are in the basement
0 Number of car parking spaces in the basement = 200 car parking spaces / 2 = 100 car parking spaces
4. Calculate the area required for each car parking space in the basement:
0 Area required for each car parking space in the basement = 25 sq.m
5. Calculate the total area required for car parking in the basement:
0 Number of car parking spaces * area required for each car parking space = 100 car parking spaces
* 25 sq.m = 2500 sq.m
Therefore, the number of cars accommodated in the basement is 75.
PART B1: FOR Architecture CANDIDATES ONLY
Q.50 – Q.56 Carry ONE mark Each
Q.50 As per the CPWD Specifications (2019), the material used for cleaning marble flooring after
polishing is__________.
(A) Oxalic Acid (B) Caustic Soda (C) Bleaching Power (D) White Cement
Ans:- (A)
Explanation:- Caustic Soda, Bleaching Power, and White Cement are all corrosive materials that can
damage marble flooring. Oxalic Acid is a mild acid that is commonly used to clean marble flooring
without damaging it.
Here are some more details about the different materials:
• Oxalic Acid is a mild acid that is commonly used to clean marble flooring. It is a white, crystalline
solid that is soluble in water. Oxalic Acid is a mild acid that can be used to remove stains and dirt
from marble flooring without damaging it.
• Caustic Soda is a strong base that is commonly used in industrial applications. It is a white,
crystalline solid that is soluble in water. Caustic Soda is a strong base that can cause burns to the
skin and eyes. It should not be used to clean marble flooring.
• Bleaching Power is a chemical compound that is commonly used to bleach fabrics. It is a white,
crystalline solid that is soluble in water. Bleaching Power is a strong oxidizing agent that can
damage marble flooring. It should not be used to clean marble flooring.
• White Cement is a type of cement that is made from limestone and clay. It is a white, powdery
substance that is used to make concrete. White Cement is not typically used to clean marble
flooring.
Hemant Vilas Parulekar 21

Q.51 The proportion of the sides of a traditional Japanese tatami mat is _______.
(A) 1 : 1.414 (B) 1 : 1.5 (C) 1 : 2 (D) 1 : 1.618
Ans:- (C)
Explanation:-
The proportion of the sides of a traditional Japanese tatami mat is 1:2. This means that the length of the
mat is twice as long as its width. The square root of 2 is approximately 1.414, so answer choice (A) is
incorrect. The golden ratio is approximately 1.618, so answer choice (D) is also incorrect.
The standard size of a tatami mat is 180 centimeters by 90 centimeters, which gives it a 2:1 ratio. This
ratio is based on the Japanese philosophy of wabi-sabi, which celebrates the beauty of imperfection and
asymmetry. The 2:1 ratio is also considered to be the most comfortable and practical for sitting and
sleeping.
Q.52 As per IS: 4954 – 1964, the acceptable noise level (in dB) for urban residential areas is____________.
(A) 35-45 (B) 65-75 (C) 20-30 (D) 15-25
Ans:- (A)
Explanation:- As per IS: 4954 – 1964 (Indian Standard for Evaluation of Noise in Urban Areas), the
acceptable noise level for urban residential areas falls within the range of 35-45 decibels (dB). This
standard sets guidelines for noise pollution control in different areas, including residential areas,
to ensure a suitable and healthy acoustic environment for residents. Noise levels within this range
are considered acceptable for urban residential areas, promoting a peaceful and comfortable living
environment.
The other options provided (B) 65-75, (C) 20-30, and (D) 15-25 are not applicable to the acceptable
noise level in urban residential areas according to the mentioned standard.
It’s important to note that noise level standards may vary across different countries and regions, and they
may be subject to revision or updates over time based on advancements in research and understanding
of noise pollution impacts on human health and well-being. Therefore, it’s advisable to refer to the
relevant local regulations and standards for specific noise level requirements in a particular area.
Q.53 Identify the Indian tribe that is associated with the vernacular dwelling
illustrated in the image below.
(A) Bhotia, Uttarakhand
(B) Toda, Tamil Nadu
(C) Naga, Nagaland
(D) Kutia Kondh, Odisha
Ans:- (B)
Explanation:- The Toda tribe is a small, pastoral community that lives in the Nilgiri Hills of Tamil
Nadu, India. Their traditional dwellings are called dogles, and they are unique in their construction
and appearance.
Dogles are half-barrel shaped huts that are made from bamboo and thatch. They are typically about 10
feet high, 18 feet long, and 9 feet wide. The roof is arched and is supported by a framework of bamboo
canes. The walls are made of bamboo slats that are interwoven with rattan. The door is small and is
located in the center of the front wall.
22 GATE Architecture and Planning: Comprehensive Question Bank

Dogles are typically ecorated with Toda rock art. This art is made by carving or painting images of
animals, birds, and geometric shapes on the walls of the hut. The art is believed to have religious
significance, and it is also seen as a way to protect the hut from evil spirits.
Dogles are well-suited to the climate of the Nilgiri Hills. They are cool in the summer and warm in the
winter. The thatched roof helps to insulate the hut, and the small door helps to keep out the cold air.
The Toda people are a proud people, and their dogles are a reflection of their culture and traditions.
Dogles are a unique and beautiful example of traditional Indian architecture.
Here are some additional facts about Toda huts:
• The word “dogle” is thought to be derived from the Tamil word “kol” which means “hut” or
“shed”.
• Dogles are typically built on a raised platform to protect them from flooding.
• The interior of a dogle is typically divided into two rooms: a living room and a sleeping area.
• The living room is where the Toda people cook, eat, and socialize.
• The sleeping area is typically located at the back of the hut and is separated from the living room
by a curtain.
• Dogles are typically occupied by a single family.
• The Toda people are a matrilineal society, so the eldest woman in the family is the head of the
household.
Toda huts are a valuable part of the cultural heritage of the Nilgiri Hills. They are a reminder of the
traditional way of life of the Toda people, and they are a beautiful example of traditional Indian
architecture.
Q.54 Thermal diffusivity of a wall is influenced by the choice of building material. Identify the
statement(s) that is/are correct.
(A) Thermal diffusivity is inversely proportional to thermal conductivity.
(B) Increase in specific heat capacity increases the thermal diffusivity.
(C) Materials with low thermal diffusivity have a high amplitude dampening effect.
(D) Thermal diffusivity is inversely proportional to the density of material.
Ans:- (C) & (D)
Explanation:-
• Thermal diffusivity is the rate at which heat is transferred through a material. It is a measure of
how quickly a material can absorb and release heat.
• Thermal conductivity is the ability of a material to conduct heat. It is a measure of how easily
heat can flow through a material.
• Specific heat capacity is the amount of heat required to raise the temperature of a unit mass of
a material by one degree Celsius.
Density is the mass of a unit volume of a material.
Thermal diffusivity is calculated as the product of thermal conductivity and specific heat capacity,
divided by density. Therefore, thermal diffusivity is inversely proportional to density and directly
proportional to thermal conductivity and specific heat capacity.
Hemant Vilas Parulekar 23

Materials with low thermal diffusivity have a high amplitude dampening effect. This means that they
are good at absorbing and releasing heat, which can help to reduce temperature fluctuations.
Therefore, the correct statements are (C) and (D).
Statement (A) is incorrect because thermal diffusivity is directly proportional to thermal conductivity,
not inversely proportional.
Statement (B) is incorrect because increase in specific heat capacity decreases the thermal diffusivity,
not increases.
Q.55 Select the statement(s) which are NOT correct with respect to burnt clay bricks.
(A) Lime (<10% of clay) in carbonated form lowers the fusion point of bricks.
(B) Magnesia (>1% of clay) imparts red colour to the bricks.
(C) Iron Pyrites tend to oxidize and decompose the brick during burning.
(D) Alkalis (alkaline salts) when present in excess (>10% of clay) decrease the probability of efflorescence.
Ans:- (B) & (D)
Explanation: The correct statements about burnt clay bricks are:
• Lime (<10% of clay) in carbonated form lowers the fusion point of bricks, making them easier
to fire.
• Iron Pyrites tend to oxidize and decompose the brick during burning, leaving behind a void that
can weaken the brick.
• Alkalis (alkaline salts) when present in excess (>10% of clay) can increase the probability of
efflorescence, a white powdery deposit that can form on the surface of bricks.
The incorrect statements are:
• Magnesia (>1% of clay) imparts red colour to the bricks. The color of bricks is determined by
the type of clay used.
• Alkalis (alkaline salts) when present in excess (>10% of clay) decrease the probability of
efflorescence. Alkalis actually increase the probability of efflorescence.
Therefore, the correct answer is statements (B) and (D).
Here are some additional explanations:
• Lime is a fluxing agent, which means that it lowers the melting point of other materials. When
lime is added to clay, it lowers the fusion point of the clay, making it easier to fire.
• Iron Pyrites is a mineral that contains iron and sulfur. When iron pyrites is heated, it oxidizes
and decomposes, releasing sulfur dioxide. Sulfur dioxide can react with water to form sulfuric
acid, which can weaken the brick.
• Alkalis are soluble in water. When alkalis are present in excess in clay, they can dissolve in water
and migrate to the surface of the brick. When the water evaporates, the alkalis can crystallize,
forming a white powdery deposit called efflorescence.
Q.56 Select the example(s) of Art Nouveau architecture.
(A) Basilica of the Sagrada Familia, Barcelona (B) Chrysler Building, New York
(C) Eiffel Tower, Paris (D) Mackintosh Building of the Glasgow School of Art, Glasgow
24 GATE Architecture and Planning: Comprehensive Question Bank

Ans:- (A) & (D)


Explanation:- Art Nouveau is a style of art, architecture, and design that originated in Europe in the late
19th century. It is characterized by its use of organic forms, flowing lines, and stylized ornamentation.
The Basilica of the Sagrada Familia, Barcelona, is an example of Art Nouveau architecture. It was
designed by Antoni Gaudí, who is considered one of the most important architects of the Art Nouveau
movement. The building’s facade is decorated with intricate sculptures of plants and animals, and its
interior is filled with stained glass windows and other decorative elements.
The Mackintosh Building of the Glasgow School of Art, Glasgow, is another example of Art Nouveau
architecture. It was designed by Charles Rennie Mackintosh, who was also a leading figure in the Art
Nouveau movement. The building’s facade is decorated with stylized floral motifs, and its interior is
characterized by its use of natural materials and its flowing lines.
The Chrysler Building, New York, and the Eiffel Tower, Paris, are not examples of Art Nouveau
architecture. The Chrysler Building is an example of Art Deco architecture, while the Eiffel Tower is an
example of Neoclassical architecture.
Q.57 – Q .65 Carry TWO marks Each
Q.57 Match the buildings in Group I with their architectural feature in Group II.
(A) P-2, Q-1, R-3, S-5 Group I Group II
(B) P-1, Q-2, R-4, S-3 (P) Erechtheion, Athens (1) Hypostyle Hall
(C) P-3, Q-1, R-5, S-2 (Q) Temple of Karnak, near Luxor (2) Caryatid
(R) Hagia Sophia, Istanbul (3) Pendentive
(D) P-2, Q-3, R-4, S-5
(S) Pantheon, Rome (4) Flying buttress
(5) Oculus
Ans:- (A)
Explanation: Group I represents different buildings, and Group II represents their corresponding
architectural features.
(P) Erechtheion, Athens: The Erechtheion in Athens is known for its Caryatid, which is a column in the
form of a female figure. Therefore, the corresponding architectural feature is (2) Caryatid.
(Q) Temple of Karnak, near Luxor: The Temple of Karnak near Luxor features a Hypostyle Hall, which
is a hall with a roof supported by numerous columns. Therefore, the corresponding architectural
feature is (1) Hypostyle Hall.
(R) Hagia Sophia, Istanbul: The Hagia Sophia in Istanbul is known for its Pendentives, which are
curved triangular elements used to transition from a square or rectangular base to a dome. Therefore,
the corresponding architectural feature is (3) Pendentive.
(S) Pantheon, Rome: The Pantheon in Rome features an Oculus, which is a circular opening at the top
of the dome. Therefore, the corresponding architectural feature is (5) Oculus.
Hemant Vilas Parulekar 25

Q.58 Match the architects in Group I with their key architectural ideas in Group II.
(A) P-2, Q-5, R-1, S-3 Group I Group II
(B) P-4, Q-1, R-3, S-5 (P) Ludwig Mies van der Rohe (1) Bowellism
(C) P-2, Q-1, R-5, S-3 (Q) Kisho Kurokawa (2) Skin-and-bones architecture

(D) P-4, Q-5, R-1, S-2 (R) Richard Rogers (3) Served and servant spaces
(S) Louis I. Kahn (4) Dymaxion
(5) Metabolism
Ans:- (A)
Explanation: Group I represent different architects, and Group II represents their key architectural
ideas.
(P) Ludwig Mies van der Rohe: Ludwig Mies van der Rohe is associated with the architectural style
of “Bowellism,” which emphasizes the use of exposed structural elements and industrial materials.
Therefore, the corresponding architectural idea is (2) Bowellism.
(Q) Kisho Kurokawa: Kisho Kurokawa is known for his association with the architectural movement
of “Metabolism,” which focused on organic and flexible designs that could adapt to changing needs.
Therefore, the corresponding architectural idea is (5) Metabolism.
(R) Richard Rogers: Richard Rogers is associated with the architectural idea of “Skin-and-bones
architecture,” which emphasizes the use of lightweight structures and transparent facades. Therefore,
the corresponding architectural idea is (1) Skin-and-bones architecture.
(S) Louis I. Kahn: Louis I. Kahn is known for his concept of “Served and servant spaces,” which involves
the distinction between public and private areas within a building. Therefore, the corresponding
architectural idea is (3) Served and servant spaces.
Q.59 Match the structural joining systems in Group I with the corresponding materials for which they
are commonly used in Group II.
(A) P-2, Q-1, R-3, S-5 Group I Group II
(B) P-1, Q-2, R-5, S-3 (P) Centrifugal pumps (1) Piston rod
(C) P-2, Q-5, R-4, S-1 (Q) Reciprocating pumps (2) Impeller
(R) Rotary pumps (3) Gear
(D) P-1, Q-2, R-3, S-4
(S) Impulse pumps (4) Eductor pipe
(5) Hydraulic ram
Ans:- (A)
Explanation: Group I represents different types of pumps, and Group II represents the corresponding
parts or components commonly used in those pumps.
(P) Centrifugal pumps: Centrifugal pumps typically use an Impeller, which is a rotating component
that imparts energy to the fluid being pumped. Therefore, the corresponding material is (2) Impeller.
(Q) Reciprocating pumps: Reciprocating pumps use a Piston rod, which is a component that reciprocates
back and forth to create the pumping action. Therefore, the corresponding material is (1) Piston rod.
(R) Rotary pumps: Rotary pumps often use a Gear, which is a rotating mechanism that transfers energy
to the fluid. Therefore, the corresponding material is (3) Gear.
26 GATE Architecture and Planning: Comprehensive Question Bank

(S) Impulse pumps: Impulse pumps use a Hydraulic ram, which is a device that uses water hammer effect
to create pressure and move the fluid. Therefore, the corresponding material is (5) Hydraulic ram.
Q.60 Match the geometric forms in Group I with the buildings in Group II.
Group I Group II
(P) Hyperboloid (1) Petronas Twin Towers, Kuala Lumpur, by Cesar Pelli
(Q) Geodesic Dome (2) Palazzo del Lavoro, Turin, by Pier Luigi Nervi
(R) Diagrid structure (3) The Biomes at the Eden Project in Cornwall, UK, by Nicholas
Grimshaw
(S) Umbrella Structures (4) Hearst Tower, New York, by Norman Foster
(5) Cathedral of Brasilia, by Oscar Niemeyer
(A) P-5, Q-3, R-4, S-2 (B) P-3, Q-5, R-2, S-4 (C) P-5, Q-3, R-1, S-4 (D) P-3, Q-1, R-4, S-2
Ans:- (A)
Explanation: Group I represent different geometric forms, and Group II represents the corresponding
buildings that feature those forms.
(P) Hyperboloid: The Cathedral of Brasilia, designed by Oscar Niemeyer, features a hyperboloid form.
Therefore, the corresponding building is (5) Cathedral of Brasilia.
(Q) Geodesic Dome: The Biomes at the Eden Project in Cornwall, UK, designed by Nicholas Grimshaw,
are geodesic domes. Therefore, the corresponding building is (3) The Biomes at the Eden Project.
(R) Diagrid structure: Hearst Tower in New York, designed by Norman Foster, features a diagrid
structure. Therefore, the corresponding building is (4) Hearst Tower.
(S) Umbrella Structures: Palazzo del Lavoro in Turin, designed by Pier Luigi Nervi, features umbrella-
like structures. Therefore, the corresponding building is (2) Palazzo del Lavoro.
Q.61 Match the instruments in Group I with the physical quantities they measure in Group II.
Group I Group II
(P) Goniophotometer (1) Electromagnetic Energy at Specific Wavelengths
of Light
(Q) Pyrheliometer (2) Luminous Flux of Directed Light Sources
(R) Spectrophotometer (3) Direct Solar Irradiance
(S) Forward-Looking Infrared Camera (4) Temperature
(5) Global Solar Radiation
(A) P-5, Q-2, R-1, S-3 (B) P-2, Q-3, R-1, S-4 (C) P-2, Q-3, R-1, S-5 (D) P-5, Q-1, R-2, S-4
Ans:- (B)
Explanation: Group I represents different instruments, and Group II represents the physical quantities
that these instruments measure.
(P) Goniophotometer: A goniophotometer is used to measure the Luminous Flux of Directed Light
Sources. Therefore, the corresponding physical quantity is (2) Luminous Flux of Directed Light Sources.
(Q) Pyrheliometer: A pyrheliometer is used to measure Direct Solar Irradiance. Therefore, the
corresponding physical quantity is (3) Direct Solar Irradiance.
(R) Spectrophotometer: A spectrophotometer is used to measure Electromagnetic Energy at Specific
Wavelengths of Light. Therefore, the corresponding physical quantity is (1) Electromagnetic Energy at
Specific Wavelengths of Light.
(S) Forward-Looking Infrared Camera: A forward-looking infrared camera is used to measure
Temperature. Therefore, the corresponding physical quantity is (4) Temperature.
Hemant Vilas Parulekar 27

Q.62 Match the terms in Group I with their associated items in Group II.
(A) P-1, Q-4, R-5, S-2 Group I Group II
(B) P-4, Q-3, R-1, S-2 (P) Scotopic vision (1) ability to see under low light condition
(C) P-1, Q-5, R-4, S-2 using rod cells
(Q) Presbyopia (2) vision in bright light using cone cells
(D) P-4, Q-2, R-1, S-5
(R) Emmetropia (3) Gear
(S) Photopic vision (4) Eductor pipe
(5) Hydraulic ram

Ans:- (C)
Explanation:- Group I represents different terms related to vision, and Group II represents the
associated items or conditions.
(P) Scotopic vision: Scotopic vision refers to the ability to see under low light conditions using rod cells.
Therefore, the associated item is (1) ability to see under low light conditions using rod cells.
(Q) Presbyopia: Presbyopia is a condition related to age-related vision loss. Therefore, the associated
item is (5) age-related vision loss.
(R) Emmetropia: Emmetropia refers to normal vision, where the light is focused properly on the retina
without the need for corrective lenses. Therefore, the associated item is (4) normal vision.
(S) Photopic vision: Photopic vision refers to vision in bright light using cone cells. Therefore, the
associated item is (2) vision in bright light using cone cells.
Q.63 Choose the correct statement(s) from the following:
Waste water from sinks, baths, etc. enters through the top inlet of a gully trap, while foul water from
sweeping of rooms or courtyards enters from side inlet.
Waste water from sinks, baths, etc, enters through the top inlet of the gully trap, while foul water from
sweeping of rooms of courtyards enters from side inlets.
Anti-siphon have a reduced water-way at the inlet side, while the outlet being larger prevents the pipe
from filling full and causing siphonic action.
(C) Intercepting traps prevent foul gases from street sewer to enter into the house.
(D) P, Q and S traps are classified according to their shape.
Ans:- (B), (C), (D)
Q.64 A steel wire of 5.65 mm diameter and 50 m length is used for a hoisting crane. The wire is used
to vertically lift a weight of 200 kg attached to its lowest end. Assume the Young’s Modulus of
Elasticity of Steel as 2 × 105 N/mm2 and gravitational acceleration as 10 m/sec2 . The elongation of
the steel wire (in mm) will be ____ .
Ans:- 20
Solution:- Force = Weight = 200 kg * gravitational acceleration (10 m/sec2) = 2000 N Length = 50 m
Diameter = 5.65 mm = 0.565 cm = 0.00565 m Young’s Modulus = 2 × 105 N/mm2 = 2 × 1011 N/m2
Elongation = (Force * Length) / (π * (Diameter/2)2 * Young’s Modulus) Elongation = (2000 N * 50 m) /
(π * (0.00565 m/2)2 * 2 × 1011 N/m2) Elongation ≈ 0.0196 m
Converting to millimeters: Elongation ≈ 0.0196 m * 1000 mm/m = 19.6 mm
Rounded off to two decimal places, the elongation of the steel wire is approximately 19.6 mm.
28 GATE Architecture and Planning: Comprehensive Question Bank

Q.65 A simply supported RCC beam of span 4 m is


supporting a brick wall over its entire span. The
brick wall is 250 mm thick and 2 m high. The RCC
beam has a depth of 600 mm and width of 250 mm.
The density of brick masonry and RCC can be
assumed as 18 KN/m3 and 25 KN/m3 respectively.
Considering the load of the wall and self-weight of
the RCC beam, the maximum bending moment in
the beam (in KN-m) will be _______ [rounded off
to two decimal places].

Ans:- 25.2
Solution to Q. 65:
To calculate the maximum bending moment in the RCC beam, we need to consider the loads acting
on the beam.
Self-weight of the RCC beam:
The self-weight of the RCC beam can be calculated as follows: Self-weight of the RCC beam = Volume
of the beam × Density of RCC Volume of the beam = Length × Width × Depth = 4 m × 0.25 m × 0.6 m
= 0.6 m³ Self-weight of the RCC beam = 0.6 m³ × 25 KN/m³ = 15 KN
Weight of the brick wall:
The weight of the brick wall can be calculated as follows: Weight of the brick wall = Volume of the wall
× Density of brick masonry Volume of the wall = Length × Thickness × Height = 4 m × 0.25 m × 2 m
= 2 m³ Weight of the brick wall = 2 m³ × 18 KN/m³ = 36 KN
The total load acting on the beam is the sum of the self-weight of the RCC beam and the weight of the
brick wall: Total load = 15 KN (self-weight) + 36 KN (brick wall) = 51 KN
Now, the maximum bending moment in the simply supported beam with a uniformly distributed load
can be calculated as follows: Maximum Bending Moment = (Total load × Span) / 8 Maximum Bending
Moment = (51 KN × 4 m) / 8 = 25.5 KN-m
Rounded off to two decimal places, the maximum bending moment in the RCC beam will be
approximately 25.50 KN-m.
PART B2: FOR Planning CANDIDATES ONLY
Q.66 – Q.72 Carry ONE mark Each
Q.66 Select the most appropriate scale to measure Attitude, Opinion and Perception.
(A) Likert scale (B) Ratio scale (C) Richter scale (D) Armstrong scale
Ans:- (A)
Explanation to Q. 66: The Likert scale is a type of rating scale used to measure attitudes, opinions, and
perceptions of individuals towards a particular subject or statement. It consists of a series of statements
or questions to which respondents indicate their level of agreement or disagreement on a scale, typically
ranging from “strongly agree” to “strongly disagree.” The Likert scale allows for capturing the intensity
of the respondent’s feelings or perceptions and provides a quantifiable measure of their attitude or
opinion. It is widely used in social sciences and market research to gather data on people’s attitudes,
preferences, and perceptions.
Hemant Vilas Parulekar 29

Q.67 Jal Shakti Abhiyan initiated by the Ministry of Jal Shakti does NOT include ____.
(A)Water conservation and rainwater harvesting (B) Renovation of traditional waterbodies
(C) Hydroelectric power generation (D) Intensive afforestation
Ans:- (C)
Explanation to Q. 67: Jal Shakti Abhiyan, initiated by the Ministry of Jal Shakti in India, focuses on water
conservation, rainwater harvesting, renovation of traditional water bodies, and intensive afforestation.
It aims to promote water security and sustainability by increasing water availability and improving
water management practices. However, hydroelectric power generation is not specifically included in
the objectives of the Jal Shakti Abhiyan. The primary focus of the campaign is on conserving and
managing water resources for sustainable use and not on power generation.
Q.68 Select the correct sequence of activities for transit-operation planning process.
(A) Network Route Design → Timetable Development → Vehicle Scheduling → Crew Scheduling
(B) Timetable Development → Crew Scheduling → Vehicle Scheduling → Network route design
(C) Vehicle Scheduling → Crew Scheduling → Network Route Design → Timetable Development
(D) Crew Scheduling → Vehicle Scheduling → Timetable Development → Network Route Design
Ans:- (A)
Explanation to Q. 68: The correct sequence of activities for transit-operation planning process is as
follows:
1. Network Route Design: This involves designing the routes and stops for the transit system to
efficiently connect different locations.
2. Timetable Development: This involves creating a schedule for the transit system, specifying the
arrival and departure times of vehicles at different stops.
3. Vehicle Scheduling: This involves assigning vehicles to specific routes and services based on the
timetable and demand.
4. Crew Scheduling: This involves assigning drivers and other crew members to operate the vehicles
according to the timetable and vehicle schedule.
So, the correct sequence is (A) Network Route Design → Timetable Development → Vehicle Scheduling
→ Crew Scheduling.
Q.69 Select the correct sequence of steps for designing the operation of a signalized intersection.
(A) Signal Phasing → Green Allocation → Cycle Length Selection
(B) Green Allocation → Cycle Length Selection → Signal Phasing
(C) Cycle Length Selection → Signal Phasing → Green Allocation
(D) Signal Phasing → Cycle Length Selection → Green Allocation
Ans:- (D)
Explanation to Q. 69: The first step in designing the operation of a signalized intersection is to determine
the signal phasing, which refers to the sequence and timing of the different traffic movements (e.g.,
left-turn, through, right-turn).
The second step is to select the cycle length, which is the total time for one complete sequence of signal
phases.
The third step is to allocate the green time to each phase, which involves determining the duration of
green signal for each traffic movement.
30 GATE Architecture and Planning: Comprehensive Question Bank

Q.70 Considering the following statements (P, Q, and R), select the correct option.
(P) Prediction of travel demand depends on target year modal alternatives.
(Q) Prediction of travel demand depends on target year population.
(R) Prediction of travel demand depends on target year land use.
(A) Only P is correct (B) Only P and R are correct
(C) Only Q & R are correct (D) P, Q, R are all correct
Ans:- (D)
Explanation to Q. 70:
• Statement P: The prediction of travel demand depends on the target year modal alternatives.
This is because the modal alternatives available in a given area will affect the way people travel.
For example, if a city has a good public transportation system, then people are more likely to use
public transportation than if the city only has a limited public transportation system.
• Statement Q: The prediction of travel demand depends on the target year population. This
is because the population of an area will affect the number of people who are traveling. For
example, if the population of a city is growing, then the number of people who are traveling is
also likely to grow.
• Statement R: The prediction of travel demand depends on the target year land use. This is
because the land use in an area will affect the way people travel. For example, if a city has a
lot of commercial areas, then people are more likely to travel by car than if the city has a lot of
residential areas.
In conclusion, all three statements (P, Q, R) are correct and they all contribute to the prediction of travel
demand.
Q.71 During Covid-19 pandemic, the ARHC scheme was launched in 2021 by the Government of India
to address the problems of poor urban migrants. The term ARHC refers to_____.
(A) Accessible Rural Health Centre
(B) Affordable Rental Housing Complexes
(C) Affordable Rentals for Homeless Citizens
(D) Accessible Rural Housing Complexes
Ans:- (B)
Solution to Q. 71: ARHC stands for Affordable Rental Housing Complexes.
The ARHC scheme was launched by the Government of India in 2021 to address the problems of poor
urban migrants who were displaced due to the COVID-19 pandemic. The scheme aims to provide
affordable rental housing to these migrants in urban areas.
The ARHC scheme is implemented through two models:
• Model 1: This model involves the conversion of existing government-funded vacant houses into
ARHCs.
• Model 2: This model involves the construction of new ARHCs.
The ARHC scheme is a welcome initiative by the Government of India to address the problems of poor
urban migrants. The scheme has the potential to provide much-needed affordable rental housing to
these migrants and help them to rebuild their lives.
Hemant Vilas Parulekar 31

Q.72 Choose the non-probability sampling method where the sample is taken from a group of people
easy to contact or reach.
(A) Simple random sampling
(B) Snowball sampling
(C) Convenience sampling
(D) Stratified random sampling
Ans:- (C)
Explanation to Q. 72: Convenience sampling is a non-probability sampling method where the sample
is taken from a group of people who are easy to contact or reach. This type of sampling is often used
when it is difficult or impossible to obtain a random sample.
For example, if you are conducting a survey on the use of public transportation, you might use
convenience sampling to survey people who are waiting at a bus stop. This is because the people who
are waiting at the bus stop are easy to contact and reach.
Other examples of convenience sampling include:
• Surveying people who are shopping at a mall
• Surveying people who are attending a sporting event
• Surveying people who are using a public Wi-Fi network
Convenience sampling is a quick and easy way to collect data, but it is important to be aware of the
limitations of this type of sampling. The results of a convenience sample may not be representative of
the population as a whole.
The other options are:
• Simple random sampling: This is a probability sampling method where every member of the
population has an equal chance of being selected for the sample.
• Snowball sampling: This is a non-probability sampling method where the researcher starts
with a small group of people and then asks them to refer the researcher to other people who
might be interested in participating in the study.
• Stratified random sampling: This is a probability sampling method where the population is
divided into strata, and then a random sample is selected from each stratum.
Q.73 – Q.81 Carry TWO marks Each
Q.73 Match the items in Group-I with the most appropriate stages of travel demand modelling in
Group-II.
P-4, Q-3, R-2, S-1 Group I Group II
(B) P-3, Q-4, R-5, S-1 US-EPA’s MOVES Trip assignment
(C) P-4, Q-3, R-1, S-5 Fratar model Trip production
(R) Growth factor model Traffic distribution
(D) P-3, Q-4, R-2, S-5
User equilibrium Mobile source emission estimation
Destination Choice
32 GATE Architecture and Planning: Comprehensive Question Bank

Ans:- (A)
Explanation to Q. 73:
• US-EPA’s MOVES is a software package that is used to estimate mobile source emissions. It
uses a variety of data, including traffic counts, fuel consumption, and vehicle characteristics, to
estimate the emissions of pollutants from motor vehicles.
• Fratar model is a traffic assignment model that is used to determine how traffic is assigned to a
network. It is a user equilibrium model, which means that it assumes that travelers will choose
the route that minimizes their travel time.
• Growth factor model is a model that is used to estimate the growth in travel demand. It uses
a variety of factors, including population growth, economic growth, and land use changes, to
estimate the future demand for travel.
• User equilibrium is a state in which no traveler can improve their travel time by changing their
route. It is a theoretical concept that is used in traffic assignment models.
Q.74 Match the Acts in Group-I with the corresponding organizations empowered by the Act in Group-
II.
P-4, Q-1, R-2, S-3 Group I Group II
(B) P-2, Q-3, R-4, S-5 RERA 2016 Chief Information Commission
(C) P-3, Q-1, R-4, S-5 RTI Act 2005 Land Registration Board
Town and Country Planning Real Estate Regulatory Authority
(D) P-3, Q-1, R-5, S-2
Act
Municipal Act Development Authority
Board of Councillors

Ans:- (C)
Explanation to Q. 74:
• RERA 2016 is an Act that was passed by the Indian Parliament in 2016 to regulate the real
estate sector in India. The Act established the Real Estate Regulatory Authority (RERA) in each
state and union territory. The RERA is responsible for regulating the sale of real estate projects,
ensuring transparency in the real estate sector, and protecting the interests of home buyers.
• RTI Act 2005 is an Act that was passed by the Indian Parliament in 2005 to provide for the right
to information for citizens. The Act established the Central Information Commission (CIC)
and State Information Commissions (SICs) to hear complaints and appeals related to the right
to information.
• Town and Country Planning Act is an Act that is passed by the state government to regulate
the development of land in urban areas. The Act establishes the Town and Country Planning
Board (TCPB) in each state. The TCPB is responsible for formulating and implementing the
town and country planning schemes for the state.
• Municipal Act is an Act that is passed by the state government to establish and regulate
municipal corporations in urban areas. The Act establishes the Municipal Corporation (MC) in
each city. The MC is responsible for providing civic amenities such as water supply, sanitation,
and garbage disposal in the city.
Hemant Vilas Parulekar 33

Q.75 As per IRC 11:1962, separate bicycle tracks may be provided when the peak hour.
Which of the following statement(s) can be used to correctly fill in the blank?
(P) Bicycle traffic is 400 bicycles/hour or more and the volume of motorized vehicles is 100-200
vehicles/hour
(Q) Bicycle traffic is 100 bicycles/hour or more and the volume of motorized vehicles exceed 200
vehicles/hour
(R) Bicycle traffic is 100-200 bicycles/hour and the volume of motorized vehicle is 100-200 vehicles/
hour
(A) Only P & Q (B) Only P & R (C) Only R (D) P, Q, R
Ans:- (A)
Explanation to Q. 75:
• As per IRC 11:1962, separate bicycle tracks may be provided when the peak hour bicycle traffic
is 400 bicycles/hour or more (Option P).
• Separate bicycle tracks may also be provided when the volume of motorized vehicles exceeds
200 vehicles/hour (Option Q).
Q.76 As per URDPFI Guidelines (2015), match the following settlement types in Group-I to their
population range in Group-II.
(A) P-5, Q-2, R-3, S-1 Group I Group II
(B) P-2, Q-4, R-1, S-5 (P) Large city 1. 50,000 to 1 lakh
(C) P-5, Q-4, R-1, S-2 (Q) Metropolitan city II 2. 50 lakh to 1 crore
(D) P-4, Q-2, R-3, S-5 (R) Small town II 3. 20,000 to 50,000
(S) Medium Town I 4. More than 1 crore
5. 5 lakh to 10 lakh
Ans:- (A)
Explanation:-
• Large city: A large city is a city with a population of more than 100,000 people. The population
range for a large city in the URDPFI Guidelines (2015) is 5 lakh to 10 lakh.
• Metropolitan city II: A metropolitan city II is a city with a population of more than 50 lakh
people. The population range for a metropolitan city II in the URDPFI Guidelines (2015) is
50 lakh to 1 crore.
• Small town II: A small town II is a town with a population of 20,000 to 50,000 people. The
population range for a small town II in the URDPFI Guidelines (2015) is 20,000 to 50,000.
• Medium Town I: A medium town I is a town with a population of more than 1 crore people.
The population range for a medium town I in the URDPFI Guidelines (2015) is 50,000 to 1 lakh.
Q.77 Match the application areas in Group I with the Satellites/Satellite sensors in Group II.
(A) P-5, Q-3, R-4, S-1 Group I Group II
(B) P-3, Q-5, R-1, S-4 (P) Cyclone prediction 1. IRNSS 11
(C) P-5, Q-2, R-4, S-3 (Q) Communication 2. HySIS
(R) High resolution mapping 3. GSAT 30
(D) P-2, Q-3, R-5, S-1
(S) Navigation 4. CARTOSAT 3
5. SCATSAT 1
34 GATE Architecture and Planning: Comprehensive Question Bank

Ans:- (A)
Explanation:-
• SCATSAT-1: This satellite is used for cyclone prediction. It is equipped with a scatterometer,
which is a radar instrument that measures the backscatter from the ocean surface. This
information can be used to track the movement of cyclones and to predict their intensity.
• GSAT-30: This satellite is used for communication. It is equipped with a variety of
telecommunication payloads, which can be used to transmit voice, data, and video signals.
• CARTOSAT-3: This satellite is used for high resolution mapping. It is equipped with a
panchromatic camera, which can image the Earth’s surface with a resolution of 0.6 meters.
This high-resolution imagery can be used for a variety of applications, such as urban planning,
disaster management, and environmental monitoring.
• IRNSS-11: This satellite is used for navigation. It is equipped with a navigation payload, which
can be used to provide accurate positioning information to users on the ground.
Q.78 Select the institution(s) that are mandated as per the 73rd Constitutional Amendment Act, 1992
of India.
(A) Panchayat (B) Municipal council (C) Ward committee (D) Gram Sabha
Ans:- A, D
Explanation: The 73rd Constitutional Amendment Act, 1992 of India pertains specifically to rural
local self-government. Therefore, the institutions that are mandated as per this Act are:
(A) Panchayat - These are rural local self-governing bodies.
(D) Gram Sabha - This is a body consisting of persons registered in the electoral rolls of a village within
the area of the Panchayat.
The Municipal Council and Ward Committee are part of urban local self-government and are covered
by the 74th Amendment, not the 73rd.
So, the answer is (A) and (D).
Q.79 Select the method(s) that can be used for landuse classification based on satellite images.
(A) Maximum Likelihood (B) Northwest Corner Method
(C) K Means (D) ANN
Ans:- (A), (C), (D)
Explanation:
(A) Maximum Likelihood - This is a statistical approach used in remote sensing to classify different
land uses.
(C) K Means - This is a clustering algorithm that can also be used for image classification in remote
sensing.
(D) ANN (Artificial Neural Network) - This is a machine learning method that can be used for land
use classification based on patterns in the data.
So, the answer is (A), (C) and (D).
Hemant Vilas Parulekar 35

Q.80 The figure below shows a contour diagram and two points (A & B)
on the continuously ascending surface. The horizontal projection
of AB is 200 m long, and the gradient of AB is 1 in 25. The constant
contour interval (in m) is ______ [in integer].

Ans:- 2m
Solution:
Given a gradient of 1 in 25 and a horizontal distance of 200m,
the vertical rise is 200/25 = 8m.
If there are 3 contour lines between A and B, there are 4 intervals.
Thus, the contour interval is 8/4 = 2m.
Q.81 A given zone is characterized in the following tables in terms of household size, and vehicle
ownership. Table I shows the trip rates of households, and Table II shows the household composition.
For households of size two and above, having one or more vehicles, the total daily home-based trips
made are __________ [in integer].
Table I: Trip rate of households (unit: number of daily home-based trips per household)
Vehicles / household Persons / household
Vehicles / household One (1) Two (2) Three & above (3+)
136
Zero (0) 0.5 2 4
One (1) 0.6 2.5 5
Two & above (2+) 1 3 6
Table II: Household composition of zone (unit: number of households)
Vehicles / household Persons / household
Vehicles / household One (1) Two (2) Three & above (3+)
136
Zero (0) 100 200 150
One (1) 200 300 200
Two & above (2+) 50 100 50
Ans:- 2350
Solution:- To calculate the total daily home-based trips made by households of size two and above,
having one or more vehicles, we need to multiply the number of such households by their corresponding
trip rates and sum the results.
From Table II, the number of households of size two and above with one or more vehicles are:
• For two-person households with one vehicle: 300 households
• For two-person households with two or more vehicles: 100 households
36 GATE Architecture and Planning: Comprehensive Question Bank

• For households of size three and above with one vehicle: 200 households
• For households of size three and above with two or more vehicles: 50 households
From Table I, the trip rates for these households are:
• For two-person households with one vehicle: 2.5 trips/household/day
• For two-person households with two or more vehicles: 3 trips/household/day
• For households of size three and above with one vehicle: 5 trips/household/day
• For households of size three and above with two or more vehicles: 6 trips/household/day
Therefore, the total daily home-based trips made by these households are:
• 300 households * 2.5 trips/household/day = 750 trips/day
• 100 households * 3 trips/household/day = 300 trips/day
• 200 households * 5 trips/household/day = 1000 trips/day
• 50 households * 6 trips/household/day = 300 trips/day
Add these results together to get the total daily home-based trips:
750 trips/day + 300 trips/day + 1000 trips/day + 300 trips/day = 2350 trips/day
So, for households of size two and above, having one or more vehicles, the total daily home-based trips
made are 2350 trips/day.
GATE QUESTION PAPER 2022
General Aptitude (GA)

1 – Q. 5 carry one mark each.


Q.1 After playing _________ hours of tennis, I am feeling _________ tired to walk back.
(A) too / too (B) too / two (C) two / two (D) two / too

Ans:- (D)
Explanation:
“After playing two hours of tennis, I am feeling too tired to walk back”.
Q.2 The average of the monthly salaries of M, N and S is ₹ 4000. The average of the monthly salaries
of N, S and P is ₹ 5000. The monthly salary of P is ₹ 6000. What is the monthly salary of M as a
percentage of the monthly salary of P?
(A) 50% (B) 75% (C) 100% (D) 125%
Ans:- (A)
Solution:
This is a question that can be solved by using the formula for average, which is: average = sum /
number12.
Let x
be the monthly salary of M, y
be the monthly salary of N, and z
be the monthly salary of S. Then, we have:

Solving these equations, we get:


x+y+z=12000
y+z=9000
x=3000
Therefore, the monthly salary of M as a percentage of the monthly salary of P is:
6000x×100=60003000×100=50%
38 GATE Architecture and Planning: Comprehensive Question Bank

Q.3 A person travelled 80 km in 6 hours. If the person travelled the first part with a uniform speed of
10 kmph and the remaining part with a uniform speed of 18 kmph.
What percentage of the total distance is travelled at a uniform speed of 10 kmph?
(A) 28.25 (B) 37.25 (C) 43.75 (D) 50.00
Ans:- (C)
Solution:
Let x
be the distance travelled at a uniform speed of 10 kmph, and y
be the distance travelled at a uniform speed of 18 kmph. Then, we have:

Solving these equations, we get:


x=35
y=45
Therefore, the percentage of the total distance travelled at a uniform speed of 10 kmph is:

Q.4 Four girls P, Q, R and S are studying languages in a University. P is learning French and Dutch. Q is
learning Chinese and Japanese. R is learning Spanish and French. S is learning Dutch and Japanese.
Given that: French is easier than Dutch; Chinese is harder than Japanese; Dutch is easier than
Japanese, and Spanish is easier than French.
Based on the above information, which girl is learning the most difficult pair of languages?
(A) P (B) Q (C) R (D) S
Ans:- (B)
Explanation:
We can assign a numerical value to each language based on the given information, such as:
French = 1
Dutch = 2
Japanese = 3
Chinese = 4
Spanish = 0.5
Then, we can calculate the difficulty of each pair of languages that the girls are learning, such as:
P: French + Dutch = 1 + 2 = 3 Q: Chinese + Japanese = 4 + 3 = 7
R: Spanish + French = 0.5 + 1 = 1.5 S: Dutch + Japanese = 2 + 3 = 5
Therefore, the girl who is learning the most difficult pair of languages is Q, who is learning Chinese
and Japanese.
Hemant Vilas Parulekar 39

Q.5 A block with a trapezoidal cross-section is placed over a block with rectangular
cross section as shown above.
Which one of the following is the correct drawing of the view of the 3D object as
viewed in the direction indicated by an arrow in the above figure?

(A) (B) (C) (D)

Ans: (A)
Q. 6 – Q. 10 Carry TWO marks each.
Q.6 Humans are naturally compassionate and honest. In a study using strategically placed wallets that
appear “lost”, it was found that wallets with money are more likely to be returned than wallets
without money. Similarly, wallets that had a key and money are more likely to be returned than
wallets with the same amount of money alone. This suggests that the primary reason for this
behavior is compassion and empathy.
Which one of the following is the CORRECT logical inference based on the information in the above
passage?
(A) Wallets with a key are more likely to be returned because people do not care about money
(B) Wallets with a key are more likely to be returned because people relate to suffering of others
(C) Wallets used in experiments are more likely to be returned than wallets that are really lost
(D) Money is always more important than keys
Ans:- (B)
Explanation:
The passage states that wallets with a key are more likely to be returned than wallets with the same
amount of money alone. This suggests that people are more likely to return wallets that they believe
belong to someone else, especially if they can imagine how that person might suffer if they did not get
their wallet back.
Option (A) is incorrect because the passage does not say that people do not care about money. In fact,
the passage says that wallets with money are more likely to be returned than wallets without money,
which suggests that people do care about money.
Option (C) is incorrect because the passage does not say anything about wallets used in experiments
being more likely to be returned than wallets that are really lost.
Option (D) is incorrect because the passage does not say that money is always more important than
keys. In fact, the passage suggests that people are more likely to return wallets that they believe belong
to someone else, regardless of whether the wallet contains money or keys.
Therefore, the only logical inference that can be made from the information in the passage is that people
are more likely to return wallets that they believe belong to someone else, especially if they can imagine
how that person might suffer if they did not get their wallet back. This suggests that compassion and
empathy are the primary reasons why people return lost wallets.
40 GATE Architecture and Planning: Comprehensive Question Bank

Q.7 A rhombus is formed by joining the midpoints of the sides of a unit square. What is the diameter
of the largest circle that can be inscribed within the rhombus?
(A) 1 (B) 1 (C) √2 (D) 2√2
√2 2√2
Ans:- (A)
Solution:
As shown in the diagram drawn based on the question,
the rhombus created by connecting the midpoints of square
will also be square only.
Thus, the hypotenuse (i.e., side of the rhombus)
= x√2 (where x = Δ sides)
Thus, side of rhombus = (1/2) x √2 = 1/√2
The largest circle that can be inscribed within the rhombus
will have diameter = side of rhombus (square)
Therefore, diameter of largest circle that can be inscribed within the rhombus = 1/√2
Q.8 An equilateral triangle, a square and a circle have equal areas.
What is the ratio of the perimeters of the equilateral triangle to square to circle?
(A) 3√3 ∶ 2 ∶ √𝜋 (B)3 √3 : 2 ∶ √𝜋 (C)3 √3 : 4 ∶ 2√𝜋 (D)3 √3 : 2 ∶ 2√𝜋
Ans:- (B)
Solution:
According to the web search results, the areas of an equilateral triangle, a square, and a circle are given
by the following formulas,
Area of an equilateral triangle:

where x is the side of the triangle.


Area of a square: A=a2
where a is the side of the square.
Area of a circle: A=πr2
where r is the radius of the circle.
The perimeters of these shapes are given by the following formulas:
Perimeter of an equilateral triangle:
P=3x
Perimeter of a square: P=4a
Perimeter of a circle: P=2πr
Hemant Vilas Parulekar 41

If the areas of these shapes are equal, then we can equate the formulas and solve for the ratio of the sides:

The ratio of the perimeters is then:

Therefore, the correct answer is (B) 3√3 : 2 : √𝜋.


Q.9 Given below are three conclusions drawn based on the following three statements.
Statement 1: All teachers are professors.
Statement 2: No professor is a male.
Statement 3: Some males are engineers.
Conclusion I: No engineer is a professor.
Conclusion II: Some engineers are professors.
Conclusion III: No male is a teacher.
Which one of the following options can be logically inferred?
(A) Only conclusion III is correct
(B) Only conclusion I and conclusion II are correct
(C) Only conclusion II and conclusion III are correct
(D) Only conclusion I and conclusion III are correct
Ans: (A)
Q.10 In a 12-hour clock that runs correctly, how many times do the second, minute, and hour hands of
the clock coincide, in 12-hour duration from 3 PM in a day to 3 AM the next day?
(A) 11 (B) 12 (C) 144 (D) 2
Ans:- (A)
Explanation:
The hands of a clock coincide 11 times in every 12 hours. This is because between 11 and, they coincide
only once, i.e., at 12 o’clock. The hands overlap about every 65 minutes, not every 60 minutes. Therefore,
in a 12-hour duration from 3 PM in a day to 3 AM the next day, the hands of the clock coincide 11
times.
42 GATE Architecture and Planning: Comprehensive Question Bank

PART A: Common FOR ALL CANDIDATES


Q.11 – Q .28 (NAT/MCQ/MSQ). Each question carries one mark and negative marking of 1/3rd
mark for each wrong answer in MCQ. There is no negative marking for MSQs and NATs) (Total
marks = 18)
Q.11 The concentric circles in a sun-path diagram represent ___________.
(A) Altitude angle (B) Azimuth angle (C) Day of the year (D) Hour of the day
Ans:- (A)
Explanation: Please refer GATE-2018, Q.23 for more information).
Q.12 The operational guidelines on Credit Linked Subsidy Scheme for Economically Weaker Sections
(EWS), January 2017, by the erstwhile Ministry of Housing & Urban Poverty Alleviation,
Government of India, defines EWS households as those having an annual income up to ___________
(in Indian Rupees).
(A) 2,00,000 (B) 2,50,000 (C) 3,00,000 (D) 3,50,000
Ans:- (C)
Explanation:
The following table describes Credit Linked Subsidy Scheme (CLSS) under PMAY scheme:-
Particulars EWS LIG MIG I MIG II
Household Income Upto 3L 3-6L 6-12L 12-18L
Carpet Area in sq.m. 30 60 160 200
Interest Subsidy (% p.a.) 6.5% 4% 3%
Maximum Loan Tenure 20 years
Eligible Loan Amount (Rs.) 6,00,000/- 9,00,000/- 12,00,000/-
Discounted NPV Rate 9%
Upfront amount of subsidy (Rs.) for 20 2,67,280/- 2,35,068/- 2,30,156/-
year loan
Approx. Monthly savings @ Loan Interest 2500/- 2250/- 2,200/-
of 10%
Q.13 Which of the following is a Vector Graphics Software?
(A) Inkscape
(B) Odeon
(C) Adobe Dreamweaver
(D) DesignBuilder
Ans:- (A)
Software Use
Inkscape Vector images
Odeon Acoustical analysis
Adobe dreamweaver Web Development
Design Builder 3-D modelling
Hemant Vilas Parulekar 43

Q.14 The main cable of a suspension


bridge supports the deck with
hangars. These hangars are
equidistant along the length of
the bridge and represent a
uniformly distributed load.
Assuming the cable to be
weightless as compared to the applied loading, the best approximation of the shape that the cable takes
for this loading is a ___________.
(A) Catenary curve (B) Circular arc (C) Parabolic curve (D) Hyperbolic curve

Ans:- (C)
Explanation:
A suspension bridge carries vertical loads through curved cables in tension. These loads are transferred
both to the towers and to the anchorages, which must resist the inward pull of the cables. The suspension
bridge can be viewed as an upside-down arch in tension with only the towers in compression. The
shape of the cable depends on the distribution of the load and the boundary conditions. For a uniformly
distributed load, such as the weight of the deck and the hangars, the shape of the cable is a parabola.
This is because the horizontal component of the cable tension is constant, and the vertical component
is proportional to the sag of the cable. A parabolic curve satisfies this condition and minimizes the cable
length for a given span and sag
Q.15 Arrange the following road types in descending order of accessibility.
(P) Arterial Road (Q) Expressway (R) Collector Road (S) Local Street
(A) Q-P-R-S
(B) S-R-P-Q
(C) S-P-R-Q
(D) P-Q-S-R
Ans:- (B)
Explanation:
The roads can be classified based on their speed and accessibility. As the accessibility of a road increases,
the speed reduces. Therefore, the order of descending accessibility is as follows:
Local streets are the most accessible roads, as they provide direct access to individual properties and
buildings. They have low speed limits and traffic volumes.
Collector roads are the intermediate roads that connect local streets to arterial roads. They have
moderate speed limits and traffic volumes, and they balance the functions of mobility and accessibility.
Arterial roads are the main roads that connect different areas and regions within a city or a state. They
have high speed limits and traffic volumes, and they prioritize the function of mobility over accessibility.
Expressways are the highest level of roads that connect major cities and states across the country. They
have the highest speed limits and traffic volumes, and they have limited or no direct access to adjacent
properties. They are designed for fast and uninterrupted travel.
44 GATE Architecture and Planning: Comprehensive Question Bank

Q.16 The following two-dimensional visual composition represents ___________.

(A) Interlocking (B) Intersecting (C) Interlacing (D) Interpenetrating

Ans:- (C)
Q.17 The Golden Ratio refers to ___________.
(A) 1:√2 (B) 2: (1 + √5) (C) 1:1 (D) 16:9
Ans:- B
Explanation:
The Golden Ratio refers to a special number that is approximately equal
to 1.618 and has many interesting properties in mathematics, art, and
nature. It is also known as the divine proportion, the golden mean, or phi.
It can be expressed algebraically as (1 + √5) / 2, which is the ratio of the
longer side to the shorter side of a golden rectangle.
Q.18 Hogarth’s Line of Beauty is a ___________.
(A) Horizontal straight line (B) Zigzag line (C) Vertical straight line (D) Serpentine line
Ans:- (D)
Explanation:
Hogarth’s Line of Beauty is a term and a theory in art or aesthetics used to describe
an S-shaped curved line that appears within an object, as the boundary line of an
object, or as a virtual boundary line formed by the composition of several objects.
This theory originated with William Hogarth, an 18th-century English painter,
satirist, and writer, and is an essential part of his theory of aesthetics as described in his 1753 book
The Analysis of Beauty. According to this theory, S-shaped curved lines signify liveliness and activity
and excite the attention of the viewer, as contrasted with straight lines, parallel lines, or right-angled
intersecting lines, which signify stasis, death, or inanimate objects.
Q.19 Which of the following sites were added to Ramsar List in the year 2020?
(A) Ashtamudi Wetland (B) Asan Conservation Reserve (C) Chilika Lake (D) Lonar Lake
Ans:- (B), (D)
Explanation:
The Ramsar List is a list of wetlands of international importance maintained by the Convention on
Wetlands (Ramsar, Iran, 1971). The list aims to develop and maintain an international network of
wetlands that are important for the conservation of global biological diversity and for sustaining human
life, following sites were added to the Ramsar List in the year 2020:
-Lonar Lake in Maharashtra, India
-Sur Sarovar or Keetham Lake in Agra, Uttar Pradesh, India
-Asan Conservation Reserve in Uttarakhand, India
-Kanwar Lake or Kabal Taal in Bihar, India
-Sunderban Wetlands in Kolkata, India
Hemant Vilas Parulekar 45

Q.20 Which of the following help(s) in keeping direct solar radiation out of the building?
(A) Mashrabiya (B) Badgir (C) Malquf (D) Chajja
Ans:- (A), (D)
Explanation:
Building Mashrabiya Badgir
features
Illustrated as

Description Mashrabiya is a type of wooden screen Badgir (also spelled bâdgir or badger)
or latticework that is commonly used is a traditional Iranian architecture
in traditional Arabic architecture. It is element used for passive cooling. It is
often found on the exterior of buildings, a type of windcatcher that is used to
particularly on the upper floors, and ventilate and cool buildings by drawing
serves as a sunshade and a privacy in cool air from the outside and directing
screen. Mashrabiya consists of a series it into the building. Badgir consists of a
of wooden or stone blocks or slats that tower or shaft that is connected to the
are arranged in a decorative pattern and building and extends up through the
are spaced apart to allow air to flow roof. The top of the tower is open to
through. the air and is often shaped like a funnel
or cone to help direct the airflow. A
series of openings or vents along the
tower allow the cool air to flow into the
building..
Illustrated as Malquf Chajja
46 GATE Architecture and Planning: Comprehensive Question Bank

Building features Malquf Chajja


Description Malquf is a traditional architectural A chajja (also spelled chhajja or charja)
element found in the Middle East and is a projecting eaves or overhang that
North Africa. It is a type of dome or is found on the exterior of buildings
vaulted ceiling that is used to cover a in South Asia, particularly in India. It
space and provide structural support. is a horizontal platform or ledge that
A malquf is typically made of brick or extends outward from the wall of a
stone and is formed by stacking layers of building and is supported by brackets
arched or curved blocks on top of each or pillars. Chajjas are typically found
other to create a dome-like shape. The on the upper floors of buildings and are
blocks are held in place by gravity and used to provide shade and protection
do not require any additional support, from the sun and rain. They are also
such as beams or columns. used to create a visual break in the facade
Malqufs are often found in traditional of the building and to add interest and
Islamic architecture and are used in a character to the exterior design.
variety of buildings, including mosques, Chajjas are a common feature of
palaces, and homes. They are known for traditional Indian architecture and can
their structural strength and durability, be found on a wide range of buildings,
and are able to withstand earthquakes including homes, temples, and other
and other natural disasters. Malqufs are structures. They are often decorated
also valued for their aesthetic appeal with carved stone or wood and may be
and are often decorated with intricate painted or finished in a variety of colors.
patterns and designs. In addition to Chajjas are an important element of the
their use in architecture, malqufs have architectural style of South Asia and
also been used as burial structures in contribute to the distinctive character
some parts of the Middle East. and beauty of traditional buildings in
the region.
Q.21 As per the Handbook of Professional Documents 2015, Council of Architecture, India, architects
are liable ___________.
(A) If the building is used for any other purpose than the one for which it was designed
(B) If any unauthorised changes or illegal modifications are made by the owner(s)/occupant(s)
(C) If the client suffers damage/loss due to lack of proper professional service
(D) If the architect fails to attain the standard of care as prescribed by law
Ans:- (C), (D)
Explanation:
According to the Handbook of Professional Documents 2015, Council of Architecture, India, architects
are liable for the following:
– To perform their professional services with reasonable skill and care, and to be responsible for the
quality and accuracy of their work.
– To adhere to the standards of professional conduct and ethics prescribed by the Council of Architecture.
Hemant Vilas Parulekar 47

– To comply with the provisions of the Architects Act, 1972 and the rules and regulations made
thereunder.
– To indemnify their clients against any loss or damage caused by their negligence, breach of contract,
or violation of any law.
– To maintain professional indemnity insurance as per the scale of fees and conditions of engagement
agreed with their clients.
– To inform their clients of any conflict of interest or potential conflict of interest that may arise in the
course of their professional services.
– To respect the intellectual property rights of other architects and refrain from plagiarism or
infringement.
– To cooperate with other architects and allied professionals in the interest of the public and the
profession.
– To uphold the dignity and honour of the profession and refrain from any act or omission that may
bring disrepute to the profession.
– To contribute to the advancement of the profession and the society through research, education, and
public service.
Q.22 As per the United Nations Transforming our world: The 2030 agenda for sustainable development,
2015, which of the following Sustainable Development Goals (SDGs) directly address water related
issues?
(A) SDG-1 (B) SDG-4 (C) SDG-6 (D) SDG-14
Ans:- (C), (D)
Explanation:
The earlier adopted 8 nos. Of Millennium Development Goals (MDGs) with target year of 2015 have
been replaced by adoption of 17 nos. Sustainable Development Goals (SDGs) with target year of
2030. SDG is also known as Global Goals, and were adopted by all the United Nations member States
in September 2015 at an historic UN summit. The list of SDGs is as follows:
Goal Target
1. No Poverty End poverty in all its forms everywhere.
2. Zero Hunger End hunger, and achieve food security and improved nutrition and promote
sustainable agriculture.
3. Good Health and Ensure healthy lives and promote well-being for all at all ages.
Well-being
4. Quality education Ensure inclusive and equitable quality education and promote life-learning
opportunities for all
5. Gender Equality Achieve gender equality and empower all women an girls
6. Clean water and Ensure availability and sustainable management of water and sanitation
sanitation for all
7. Affordable and clean Ensure access to affordable, reliable, sustainable and modern energy for all
energy
8. Decent work and Promote sustained, inclusive and sustainable economic growth, full and
economic growth productive employment and decent work for all
48 GATE Architecture and Planning: Comprehensive Question Bank

9. Industry, innovation Promote inclusive and sustainable industrialization and foster innovation.
and infrastructure
10. Reduced inequality Reduce inequality within and among countries
11. Sustainable cities Make cities and human settlements inclusive, safe resilient and sustainable
and communities
12. Responsible Ensure sustainable consumption and production patterns
consumption and
production
13. Climate action Take urgent action to combat climate change and its impact
14. Life below water Conserve and sustainable use of the oceans, seas and marine resources for
sustainable development
15. Life on land Protect, restore and promote sustainable use of terrestrial ecosystems,
sustainably manage forests, combat desertification and halt and reserve land
degradation and halt biodiversity loss
16. Peace and justice Promote peaceful and inclusive societies for sustainable development,
strong institutions provide access to justice for all and build effective, accountable and inclusive
institutions at all levels.
17. Partnerships to Strengthen the means of implementation and revitalize the global partnership
achieve the goal for sustainable development
Q. 23 - Q. 28 Numerical Answer Type (NAT) (NO negative marking)
Q.23 For a masonry section, the line of action of force shifts to incorporate the effects of lateral forces
and induced moments. Consider a masonry section of width 600 mm. Assuming a zero tensile
stress capacity and a linear stress-strain response for the entire domain of loading, the minimum
value of eccentricity at which the section will crack (in mm, rounded off to one decimal place) is
__________.
Ans:- 97 - 103
Solution:
Width or thickness (t) of wall section = 600mm
Minimum value of eccentricity at which section = t/6 = 600/6 = 100mm
Q.24 The maximum and minimum indoor dry bulb temperature of a room is 38o C and 34 oC, respectively.
If the corresponding outdoor maximum and minimum dry bulb temperature are 42o C and 30o C,
respectively, then the thermal damping of the room (in percentage, rounded off to two decimal
places) is ___________.
Ans:- 66-67
Solution:
Equation for the thermal damping (D) in percentage is, D = [(T-t)/T] x 100
Where, T = Outdoor temperature range or difference (in the given problem, it is 42-30 = 120C)
t = Indoor temperature range or difference (in the given problem, it is 38-34 = 40C)
By inserting these values in the equation, D = [(12-4)/12] x 100 = 66.67
Q.25 A building site measures 96 sq.cm on a scale of 1:12500. The actual area it represents (in hectare,
in integer) is ___________.
Hemant Vilas Parulekar 49

Ans:- 150
Explanation:
Similar questions were asked earlier in (Q.45) GATE-2019, (Q.45) GATE-2015, (Q.49) 2012, (Q.35)
2011, (Q. 57) 2006, (Q.82) 2005, (q. 75) 2004, (Q.37) 2002, (Q.47) 2001.
Since profile of the site is not specified, we can assume the same
Assume, the shape of the site is rectangular when measured on map having sides as 16cm and 6cm
(Area 96 sq.cm.)
Given, scale of the map = 1:12500, I.e., 1cm on map = 12500cm on actual ground.
Thus, 16cm side on map = 16 x 125000 = 2,00,000cm = 2000m on ground
Thus, 6cm side on map = 6 x 12500 = 75,000cm = 750m on ground
Thus, actual area of the site on ground = 2000 x 750sq.m. = 15,00,000sq.m.
Thus, 1 Hectare = 10,000sq.m., So, 15,00,000 = 150 Hectare
Q.26 An off-street car parking lot contains a total of 75 bays. If the parking lot was used by 687 cars over
a period of 12 hours, the average parking turn-over of the parking lot (in vehicles per hour per bay,
rounded off to two decimal places) is ___________.
Ans:- 0.76
Solution:
Parking turn-over, is defined as the ratio of total vehicles parked in duration to the number of parking
bays available. It is expressed as number of vehicles per bay by time duration.
Given, number of bays = 75, number of vehicles = 687 and time duration = 12 hours.
Thus, Parking turn-over = (687/75)/12 = 0.76
Q.27 The hydraulic radius of the following rectangular
open drainage section (in mm, rounded off to two
decimal places) is ___________.

Ans: 136.36
Solution: Hydraulic radius (RH) = A/P
Given, A = Cross-sectional area of flow &
P = Wetted perimeter
Hence, A = 600 x 250 = 1,50,000 sq.mm. &
P = 250 + 600 +250 = 1100mm.
Thus, RH = 1,50,000/1100 = 136.36mm.
Similar question was earlier asked in (Q.52) GATE-2019 & (Q.31)
50 GATE Architecture and Planning: Comprehensive Question Bank

Q.28 A town with 0.45 million population sends its entire organic waste to a composting site on a daily
basis through a truck of 15 ton carrying capacity. Assume total waste generated per capita per day
is 0.21 kg and 40% of the total waste is organic waste.
The minimum number of weekly round trips required by the truck (in integer) will be ___________.
Ans: 21
Solution:
This is a question about the calculation of the number of truck round trips required to transport organic
waste from a town to a composting site. To answer this question, we need to follow these steps:
Step 1: Calculate the total amount of organic waste generated by the town per day. We can do this by
multiplying the population, the per capita waste generation, and the percentage of organic waste. Using
the given data, we get:
Organic waste per day=0.45×106×0.21×0.4=37,800 kg
Step 2: Calculate the number of truck trips required per day. We can do this by dividing the organic
waste per day by the truck carrying capacity. Since we need to round up the number of trips to the
nearest integer, we use the ceiling function, denoted by ⌈x⌉, which gives the smallest integer that is
greater than or equal to x. Using the given data, we get:
Truck trips per day=⌈37,800/15,000 ⌉=⌈2.52⌉=3
Step 3: Calculate the number of truck trips required per week. We can do this by multiplying the truck
trips per day by the number of days in a week, which is 7. Using the previous result, we get:
Truck trips per week=3×7=21
Q.29 – Q .49 Carry TWO marks Each
Q.29 The correct sequence of the following Construction Project Development stages, as per the National
Building Code of India 2016 is ___________.
(P) Resource Planning (Q) Project Inception
(R) Commissioning and Handing over (S) Tendering
(T) Site Survey and Soil Investigation (U) Selection of Construction Methodology
(A) P-Q-R-T-U-S (B) T-Q-R-U-S-P (C) Q-T-U-P-S-R (D) Q-T-P-S-U-R
Ans:- (C)
Explanation:
The construction project development process typically involves a number of stages, including:
Project Inception (Q): This is the first stage of the project, during which the project is initiated and the
project team is formed.
Site Survey and Soil Investigation (T): In this stage, the site is surveyed to gather information about the
soil and other site conditions that will affect the project.
Selection of Construction Methodology (U): During this stage, the project team selects the construction
methods and materials that will be used for the project.
Resource Planning (P): In this stage, the project team plans and organizes the resources needed to
complete the project, including labor, materials, and equipment.
Tendering (S): This is the process of soliciting bids from contractors to perform the work.
Commissioning and Handing over (R): The final stage of the project involves commissioning the
completed building and handing it over to the owner or operator.
Hemant Vilas Parulekar 51

Q.30 Match the aspects in Group I with the corresponding items in Group II.
(A) P-4, Q-5, R-2, S-3 Group I Group II
(B) P-5, Q-1, R-4, S-2 (P) Fire safety 1. Intruder alarm
(C) P-2, Q-4, R-5, S-1 (Q) Seismic safety 2. Zero–strength barrier
(R) Water efficiency 3. Stair lift
(D) P-2, Q-5, R-4, S-3
(S)Accessible Design 4. Aerator
5. Auxiliary damper
Ans: (D)
Explanation:
Items Intruder Alarm Zero-Strength barrier
Description An intruder alarm is a security system In a fire alarm system, a zero-strength
that is designed to detect and alert the barrier is a type of partition or separation
occupants of a building or premises that is used to divide the building or
when an unauthorized person or activity structure into fire compartments. Fire
is detected. Intruder alarms typically compartments are areas of the building
consist of sensors that are placed at that are designed to contain a fire within
strategic locations around the building or a specific area, limiting its spread and
premises, and are connected to a central allowing more time for the occupants to
control panel. When a sensor is triggered, evacuate. Zero-strength barriers are used
it sends a signal to the control panel, to create these fire compartments and
which then sounds an alarm to alert the are typically made of materials that are
occupants of the building or premises. able to withstand high temperatures for
a certain amount of time.
Items Intruder Alarm Zero-Strength barrier
illustrations

Items Stair Lift Aerator


Description A stair lift is a type of mobility device An aerator is a device that is used to
that is used to help people with mobility introduce air into a liquid or other
challenges access different levels of substance. Aerators are commonly used
a building or structure. Stair lifts are in a variety of applications, including
typically installed on the stairs of a water treatment, wastewater treatment,
building and consist of a chair or platform and aquaculture. In water treatment,
52 GATE Architecture and Planning: Comprehensive Question Bank

Items Stair Lift Aerator


Description that is mounted on a track or rail. The aerators are used to improve the quality
chair or platform is moved up and down of the water by increasing the amount
the stairs by a motor, allowing the user to of dissolved oxygen in it. This can be
easily and safely access the different levels important for maintaining the health of
of the building. Stair lifts are commonly aquatic life, such as fish, and can also help
used in homes and other residential to reduce the growth of harmful bacteria
buildings to help people with mobility and other microorganisms. Aerators can
challenges, such as seniors or individuals be used in a variety of water treatment
with disabilities, navigate stairs. They can systems, including water treatment
also be found in public buildings, such as plants, wells, and reservoirs.
schools, hospitals, and office buildings,
to provide access to people with mobility
challenges.
illustrations

Items Auxiliary Damper


Description A seismic auxiliary damper is a type of device that is used to enhance the seismic
performance of a building or structure. Seismic auxiliary dampers are typically
installed in the structure to dissipate the energy of an earthquake, reducing the
amount of force that is transmitted through the structure and minimizing damage.
There are several different types of seismic auxiliary dampers, including viscous
dampers, friction dampers, and hysteretic dampers. Viscous dampers use fluid or
gas to dissipate energy, while friction dampers use friction between two surfaces
to absorb energy. Hysteretic dampers use the energy of an earthquake to deform
a material in a controlled manner, dissipating energy as the material returns to its
original shape.
Seismic auxiliary dampers are often used in conjunction with other earthquake-
resistant features, such as base isolators, energy-dissipating structural systems, and
moment-resisting frames. They are an important part of the overall seismic design
of a building or structure, and can help to reduce the risk of damage and collapse
during an earthquake.
illustrations
Hemant Vilas Parulekar 53

Q.31 Match the States in Group I with the corresponding Vernacular Building Typologies in Group II
(A) P-4, Q-5, R-3, S-2 Group I Group II
(B) P-5, Q-1, R-2, S-4 (P) Kerala 1. Morung
(C) P-5, Q-3, R-1, S-4 (Q) Jharkhand 2. Pol
(R) Nagaland 3. Dhumkuria
(D) P-4, Q-3, R-1, S-2
(S) Gujarat 4. Nalukettu
5. Ghotul
Ans :- (D)
Explanation:
Types Morung Pol
Description Morung is a traditional architectural Gujrati Pol is a traditional style of architecture
element found in the state of found in the state of Gujarat in western
Nagaland in northeastern India. It is India. It is characterized by a distinctive style
a large, communal structure that is of building that incorporates elements of
used by the Naga people as a place traditional Indian architecture, such as ornate
for social and cultural activities. carvings, arched doorways, and elaborate
Morungs are typically found in facades.
villages and are used as a meeting Gujrati Pol buildings are typically built around
place for the community, as well a central courtyard, which is surrounded by a
as a place to store traditional Naga series of interconnected rooms and corridors.
artifacts and cultural objects. The roofs of these buildings are often made
of wood and are supported by wooden beams
or columns. The walls are often decorated
with intricate carvings and paintings, and the
buildings are finished with bright colors and
intricate designs.
illustrations
54 GATE Architecture and Planning: Comprehensive Question Bank

Types Dhumkuria Nalukettu


Description Dhumkuria is a traditional Nalukettu is a traditional style of architecture
architectural element found in the found in the state of Kerala in southern India.
state of West Bengal in eastern It is a type of house that is built around a
India. It is a type of thatched roof central courtyard and is characterized by its
that is used to cover the exterior symmetrical layout and elaborate design.
of buildings, particularly in rural Nalukettu houses are typically made of wood
areas. Dhumkuria roofs are typically and are built in the shape of a rectangle or
made of bamboo or other natural square, with a thatched roof and walls made
materials and are supported by a of brick or stone.
frame of wooden beams or poles.
They are often shaped like a dome or
a cone, and are covered with a layer
of thatch, which is made from grass,
straw, or other natural materials.
illustrations

Types Ghotul
Description Ghotul (also spelled gotul) is a traditional social and educational institution found
among the Muria people of central India. It is a communal dormitory that is used
by young Muria men and women as a place to live, study, and socialize. Ghotuls are
typically found in Muria villages and are an important part of the cultural and social
life of the Muria community.
illustrations
Hemant Vilas Parulekar 55

Q.32 Match the examples in Group I with their Corresponding typologies in Group II.
(A) P-2, Q-1, R-4, S-5 Group I Group II
(B) P-4, Q-2, R-5, S-3 (P) Navi Mumbai 1. Counter Magnet
(C) P-3, Q-1, R-2, S-5 (Q) Hissar 2. Urban Agglomeration
(R) Greater Mumbai 3. Satellite Town
(D) P-3, Q-5, R-1, S-4
(S) Delhi-Mumbai Industrial 4. University Town
Corridor
5. Investment Region
Ans:- C
Explanation:
Navi Mumbai, Satellite Town: Navi Mumbai was planned as a new city to act as a counter magnet to
Mumbai, which was facing overcrowding and congestion. It was designed to be a self-sufficient urban
settlement with its own physical and social amenities, and to absorb potential migrants and attract
some of Mumbai’s population.
Hissar is a city in the state of Haryana, which has been identified as a counter-magnet area for the
National Capital Region (NCR). A counter-magnet area is an urban center that is developed as an
alternative to the main metropolitan area, in order to reduce the pressure of migration and congestion
on the latter. Hissar was selected as a counter-magnet area because of its location, population, potential
for growth, and regional importance.
Greater Mumbai Urban Agglomeration is a term that refers to the metropolitan area of Mumbai
and its satellite towns in the northern Konkan division of Maharashtra, India. It is one of the most
populous and dense urban regions in the world, with an estimated population of over 26 million people
as of 2021. It covers an area of 6,328 square kilometres (2,443 sq mi) and consists of nine municipal
corporations and eight smaller municipal councils.
Delhi-Mumbai Industrial Corridor Investment Region is a term that refers to one of the 24 industrial
projects that are planned to be developed along the Delhi-Mumbai Industrial Corridor (DMIC), a
mega infrastructure project that aims to boost industrial development and urbanization along the route
connecting Delhi and Mumbai.
Q.33 Match the Place(s)/Event(s) in Group I with the corresponding Heritage Significance/Characteristics
in Group II.
(A) P-1, Q-4, R-3, S-2 Group I Group II
(B) P-3, Q-4, R-5, S-1 (P) Chhatrapati Shivaji 1. A long interaction between people and
(C) P-2, Q-3, R-4, S-1 Terminus, Mumbai the landscape
(Q) Kumbh Mela 2. Cultural routes
(D) P-3, Q-2, R-5, S-4
(R) Walled City of Jaipur 3. Victorian Gothic revival and traditional
Indian features
(S) Rock Shelters of 4. Intangible cultural heritage
Bhimbetka
5. Traditional human settlement, land
use reflecting an interchange of ancient
Hindu and Mughal ideas
56 GATE Architecture and Planning: Comprehensive Question Bank

Ans:- (B)
Explanation:
The Chhatrapati Shivaji Terminus, Mumbai is a railway station that exhibits a fusion of Victorian
Gothic revival and traditional Indian features. It is a symbol of the colonial and modern history of India.
The Kumbh Mela is a mass Hindu pilgrimage that occurs every 12 years at four sacred river banks. It
is considered an expression of intangible cultural heritage, as it reflects the collective faith and identity
of millions of devotees.
The Walled City of Jaipur is an example of a traditional human settlement, land use reflecting an
interchange of ancient Hindu and Mughal ideas. It was planned according to Vedic architecture and
incorporates Islamic influences in its buildings and gardens.
The Rock Shelters of Bhimbetka are a series of caves that contain some of the oldest rock paintings
in the world. They reveal a long interaction between people and the landscape, spanning from the
Paleolithic to the medieval periods.
Q.34 Match the Urban Design Concepts in Group I with their corresponding Proponents in Group II.
(A) P-1, Q-2, R-3, S-4 Group I Group II
(B) P-4, Q-1, R-2, S-3 (P) Vertical theory of Urban (1) Ian Bentley
(C) P-4, Q-3, R-5, S-1 Design
(D) P-5, Q-4, R-2, S-3 (Q) Theory of Responsive (2) Gordon Cullen
Environments
(R) Serial Vision (3) Norman Pressman
(S) Winter Urbanism (4) Ken Yeang
(5) Paul Oliver
Ans:- (B)
Explanation:
The Vertical theory of Urban Design was proposed by Ken Yeang, who advocated for high-rise, mixed-
use, and eco-friendly buildings that integrate nature and urbanism.
The Theory of Responsive Environments was developed by Ian Bentley and his colleagues, who
argued that urban design should be based on the needs and preferences of the users, and that the
physical environment should be adaptable and responsive to changing situations.
Serial Vision was a concept introduced by Gordon Cullen, who described the experience of moving
through a city as a sequence of views or images that create a sense of drama and interest.
Winter Urbanism was a term coined by Norman Pressman, who explored the challenges and
opportunities of designing urban spaces for cold climates, and suggested strategies for enhancing
comfort, accessibility, and liveability in winter cities.
Hemant Vilas Parulekar 57

Q.35 In the following sketch, P, Q, R, and S refer to elements of an urban space. Identify P, Q, R, S.

(A) P-Path, Q-Vista, R- Edge, S-Landmark


(B) P-Vista, Q-Edge, R- Landmark, S-Path
(C) P-Landmark, Q-Vista, R- Path, S-Edge
(D) P-Landmark, Q-Edge, R- Path, S-Vista
Ans:- (D)
Explanation:
A Path is a linear element that connects places and facilitates movement. It can be a street, a sidewalk,
a bridge, a canal, etc.
A Vista is a visual element that creates a sense of depth and perspective. It can be a view of a landmark,
a skyline, a natural feature, etc.
An Edge is a boundary element that defines and separates spaces. It can be a wall, a fence, a river, a
highway, etc.
A Landmark is a focal element that attracts attention and serves as a reference point. It can be a building,
a monument, a sculpture, a fountain, etc.
Q.36 As per the URDPFI Guidelines 2015, match the type of Health Care Facilities in Group I to the
corresponding population served per unit in Group II.
(A) P-1, Q-2, R-3, S-4 Group I Group II
(B) P-3, Q-1, R-5, S-4 (P) Multi-Speciality Hospital (1) 15,000
(C) P-4, Q-3, R-5, S-2 (Q) Dispensary (2) 50,000
(R) Veterinary Hospital (3) 1,00,000
(D) P-5, Q-1, R-2, S-3
(S) General Hospital (4) 2,50,000
(5) 5,00,000
58 GATE Architecture and Planning: Comprehensive Question Bank

Ans:- (B)
Explanation:
Sr.no Type Area required
1 Dispensary 1 for every 15000 population 0.08 ha to 0.12
2 Nursing home, child welfare snf maternity center( 1 for every 45000 0.20 ha to 0.30 ha
to 1,00,000 population) capacity of 25 to 30 beds
3 Poly clinic with some observation beds (1 for every 1,00,000 0.20 ha to 0.30 ha
population)
4 Intermediate hospital (1 for every 1,00,000 population) capacity of 1.00 ha
80 beds
5 Intermediate hospital (1 for every 1,00,000 population) capacity of 3.70ha
200 beds
6 General hospital (1 for every 250,000 population) capacity of 300 6.00 ha
beds
7 Multi-speciality hospital (1 for 1,00.000 population) capacity of 200 9.00 ha
beds+ residentials
8 Specialty hospital (1 for every 1,00,000 population) capacity of 200 3.70 ha
beds + residentials.
Q.37 Match the plan forms in Group I with their corresponding project names in Group II.
Group I Group II
(P) (1) New Parliament of Egypt,
Cairo

(Q) (2) Apple Park Campus,


California

(R) (3) Commerzbank, Frankfurt

(S) (4) 30 St. Mary Axe, London

(5) Parliament Building, Dhaka


(A) P-3, Q-5, R-4, S-2 (B) P-4, Q-2, R-1, S-5 (C) P-1, Q-2, R-3, S-4 (D) P-3, Q-5, R-1, S-2
Hemant Vilas Parulekar 59

Ans: (D)
Explanation:
Project name Commerzbank, Frankfurt Parliament Building, Dhaka
Description The plan form of the Commerzbank is a The plan form of the Parliament Building,
triangular shape, with rounded corners Dhaka, is a triangular shape, with a
and slightly convex sides. The floors central dome and three rounded corners.
of the building and the cores at the The dome houses the main assembly hall,
three corners are organized around a which can seat 354 members. The three
central atrium, which is also triangular corners contain the offices of the president,
in shape. Each floor has three wings, the prime minister, and the speaker. The
two of which serve as office space; the building also has nine blocks, each with
third forms part of one of the nine sky four stories, that surround the dome and
gardens that are distributed throughout the corners. The blocks contain various
the building. facilities, such as committee rooms,
libraries, lounges, and dining halls.
Plan form

Project name New Parliament of Egypt, Apple Park Campus,


Cairo California
Explanation The plan form of the New Parliament of The plan form of the Apple Park Campus
Egypt, Cairo, is a circular shape, with a is a ring shape, with a large courtyard
central dome and four minarets at the in the center and a curved facade. It is
corners. The dome houses the main a symmetrical and regular plan, which
assembly hall, which can accommodate suggests a modern and efficient building
1,000 members. The minarets serve type. The building has four stories above
as observation towers and symbols of the ground and two basement levels, and
Islamic culture. The building also has covers an area of 2.8 million square feet. It
four wings that radiate from the dome, can accommodate 12,000 employees, who
containing the offices of the president, work in open-plan spaces that encourage
the prime minister, the speaker, and the collaboration and creativity.
parliamentary committees.
Plan form
60 GATE Architecture and Planning: Comprehensive Question Bank

Q.38 Match the Biosphere reserves in India in Group I with their corresponding locations in Group II.
(A) P-2, Q-1, R-4, S-3 Group I Group II
(B) P-2, Q-3, R-1, S-5 (P) Agasthyamala Biosphere (1) Western Himalayan region,
(C) P-3, Q-1, R-4, S-5 Reserve Himachal Pradesh
(Q) Nokrek Biosphere Reserve (2) Western Ghats, Kerala and
(D) P-4, Q-5, R-1, S-3
Tamil Nadu
(R) Cold desert Biosphere (3) Tura range, Meghalaya
Reserve
(S) Simlipal Biosphere Reserve (4)Kachchh,Rajkot,Surendranaga,
and Patan districts, Gujarat
(5) Mayurbhanj district, Odisha
Ans:- (B)
Explanation:
Biosphere reserves are areas of terrestrial and coastal or marine ecosystems that are recognized for
their biodiversity and cultural values, and are managed for conservation and sustainable development1.
India has 18 biosphere reserves, of which 12 are part of the World Network of Biosphere Reserves under
the UNESCO Man and the Biosphere Programme.
The correct match of the biosphere reserves in Group I with their locations in Group II is as follows:
P: Agasthyamala Biosphere Reserve is located in the Western Ghats region, covering parts of Kerala
and Tamil Nadu. It is home to a rich variety of flora and fauna, including endemic and endangered
species such as the lion-tailed macaque, the Nilgiri tahr, and the Malabar large-spotted civet. Therefore,
P-2 is correct.
Q: Nokrek Biosphere Reserve is located in the Tura range of the Meghalaya plateau, encompassing
the Nokrek National Park and several wildlife sanctuaries. It is a hotspot of biodiversity, with rare and
endemic species such as the red panda, the Asian elephant, and the citrus indica, the most primitive
ancestor of all citrus fruits. Therefore, Q-3 is correct.
R: Cold Desert Biosphere Reserve is located in the Western Himalayan region, covering parts
of Himachal Pradesh and Jammu and Kashmir. It comprises the Pin Valley National Park and the
Chandratal and Sarchu Wildlife Sanctuaries. It is characterized by high-altitude cold deserts, alpine
meadows, and snow-capped mountains, and harbors species such as the snow leopard, the Himalayan
brown bear. Therefore, R-1 is correct.
S: Simlipal Biosphere Reserve is located in the Mayurbhanj district of Odisha, covering the Simlipal
National Park and the adjoining reserve forests. It is part of the Deccan Peninsula biogeographic zone,
and features tropical moist deciduous forests, grasslands, and waterfalls. It is home to a diverse range
of wildlife, including the royal Bengal tiger, the Asian elephant, and the gaur. Therefore, S-5 is correct.
Q.39 In traditional Persian context, qanat system refers to
(A) An underground water-way, tunnelled and channelled
(B) A system where water is raised by a series of scoops fixed to a moving belt stretched between two
wheels
(C) A method of conducting water from a source-well rather than raising it
(D) A system where water is conducted from enclosure to enclosure by straightforward gravity fall
Hemant Vilas Parulekar 61

Ans:- A, C, D
Explanation:
A qanat system is a system for transporting water from an aquifer or water well to the surface, through
an underground aqueduct; the system originated approximately 3,000 years ago in what is now Iran1.
Qanats are used for irrigation and drinking water in arid regions, and are considered as an exceptional
testimony to the cultural traditions and civilizations in desert areas. Qanats have different names in
different regions, such as karez, foggara, khettara, falaj, etc.
Q.40 Which of the following is/are classified as the Principles of Universal Design?
(A) Perceptible Information
(B) Tolerance for Error
(C) Occult Balance
(D) Simple and Intuitive Use
Ans:A, B, D
Explanation:
The Principles of Universal Design are a set of guidelines for designing products, services, and
environments that are accessible and usable by all people, regardless of their abilities, age, gender, or
background. They were developed in 1997 by a group of architects, product designers, engineers, and
researchers at the Center for Universal Design at North Carolina State University.
The Principles of Universal Design are as follows:
Principle 1: Equitable Use - The design is useful and marketable to people with diverse abilities.
Principle 2: Flexibility in Use - The design accommodates a wide range of individual preferences and
abilities.
Principle 3: Simple and Intuitive Use - The design is easy to understand, regardless of the user’s
experience, knowledge, language skills, or current concentration level.
Principle 4: Perceptible Information - The design communicates necessary information effectively to
the user, regardless of ambient conditions or the user’s sensory abilities.
Principle 5: Tolerance for Error - The design minimizes hazards and the adverse consequences of
accidental or unintended actions.
Principle 6: Low Physical Effort - The design can be used efficiently and comfortably and with a
minimum of fatigue.
Principle 7: Size and Space for Approach and Use - The design provides appropriate size and space for
approach, reach, manipulation, and use regardless of the user’s body size, posture, or mobility.
Q.41 As per the URDPFI Guidelines 2015, which of the following Organoleptic and Physical parameters
comply with the acceptable limit requirements of drinking water quality?
(A) Colour: Maximum 5 Hazen units
(B) Turbidity: Maximum 1 NTU
(C) pH Value: Minimum 10
(D) Total Dissolved Solids: Maximum 500 mg/l
62 GATE Architecture and Planning: Comprehensive Question Bank

Ans:- A, B, D
Explanation:
The URDPFI Guidelines 2015 are the Urban and Regional Development Plans Formulation and
Implementation Guidelines issued by the Ministry of Housing and Urban Affairs of India. They provide
a comprehensive framework for planning and development of urban and regional areas in India.
According to the URDPFI Guidelines 2015, the acceptable limit requirements of drinking water quality
are based on the World Health Organization (WHO) Guidelines for Drinking-water Quality2. The
WHO Guidelines for Drinking-water Quality are the international reference point for the setting of
national regulations and standards for water safety in support of public health.
The WHO Guidelines for Drinking-water Quality specify the following organoleptic and physical
parameters for drinking water quality.
Colour: The colour of drinking water should be less than or equal to 15 Hazen units. Colour is an
indicator of the presence of organic matter, metals, or other substances in water. High colour may affect
the acceptability of water by consumers and interfere with disinfection processes.
Turbidity: The turbidity of drinking water should be less than or equal to 5 NTU. Turbidity is a measure
of the cloudiness of water caused by suspended particles. High turbidity may reduce the effectiveness
of disinfection, increase the risk of microbial contamination, and affect the aesthetic quality of water.
pH Value: The pH value of drinking water should be between 6.5 and 8.5. pH is a measure of the acidity
or alkalinity of water. Extreme pH values may cause corrosion or scaling of pipes and fittings, affect the
taste of water, and alter the effectiveness of disinfection.
Total Dissolved Solids: The total dissolved solids (TDS) of drinking water should be less than or equal
to 1000 mg/l. TDS is a measure of the total amount of dissolved minerals, salts, metals, and organic
matter in water. High TDS may affect the taste, odour, and appearance of water, and may also have
health implications for some consumers.
Q.42 In an ideal air-conditioning cycle shown below, which of the following statement(s) is/are true in
the segments P, Q, R, S?

(A) P: Vapour at low pressure (B) Q: Vapour at low pressure


(C) R: Liquid at high pressure (D) S: Liquid-Vapour mixture at low pressure
Hemant Vilas Parulekar 63

Ans:- A,C,D
Explanation:
The ideal air-conditioning cycle is also known as the vapor-compression refrigeration cycle. It consists
of four main components: the compressor, the condenser, the expansion device, and the evaporator.
The refrigerant, a fluid that can change its state from liquid to vapor and vice versa, flows through these
components and undergoes various thermodynamic processes.
P: This is the saturated vapor state of the refrigerant at low pressure and low temperature. It enters the
compressor, where it is compressed to a high pressure and high temperature superheated vapor state.
Therefore, option (A) is true.
Q: This is the superheated vapor state of the refrigerant at high pressure and high temperature. It leaves
the compressor and enters the condenser, where it is cooled and condensed to a high pressure and
moderate temperature liquid state. Therefore, option (B) is false.
R: This is the liquid state of the refrigerant at high pressure and moderate temperature. It leaves the
condenser and enters the expansion device, where it is throttled to a low pressure and low temperature
liquid-vapor mixture state. Therefore, option (C) is true.
S: This is the liquid-vapor mixture state of the refrigerant at low pressure and low temperature. It leaves
the expansion device and enters the evaporator, where it is heated and evaporated to a low pressure and
low temperature saturated vapor state. Therefore, option (D) is true.

Q.43 Which of the following is/are the characteristic(s) of a Mughal Garden?


(A) Symmetrical and geometrical
(B) Fountain and channelled water
(C) Winding road and untrimmed vegetation
(D) Vista with terminal building
Ans:- A, B, D
Explanation:
Refer GATE 2021 Q.30 for more information.
64 GATE Architecture and Planning: Comprehensive Question Bank

Q.44 As per the Central Pollution Control Board’s National Air Quality Index (AQI) of India 2014,
which of the following statement(s) is/are true?
(A) AQI is computed considering 8-hourly value of CO
(B) AQI is computed considering 2-hourly value of PM2.5
(C) AQI considers the O3 concentrations
(D) AQI considers the CO2 concentrations
Ans:- A, C
Explanation:
The National Air Quality Index (AQI) of India is a tool to communicate the status of air quality and its
associated health impacts to the public. It is computed based on the concentrations of eight pollutants:
PM10, PM2.5, NO2, SO2, CO, O3, NH3, and Pb.
The AQI is computed considering the 8-hourly value of CO, which is a colorless, odorless, and toxic gas
that can affect the oxygen-carrying capacity of the blood and cause various health problems2. The AQI
is also computed considering the 8-hourly value of O3, which is a gas that can irritate the respiratory
system and cause breathing difficulties, coughing, and chest pain.
The AQI is not computed considering the 2-hourly value of PM2.5, but the 24-hourly value of PM2.5,
which are fine particles that can penetrate deep into the lungs and cause respiratory and cardiovascular
diseases4. The AQI also does not consider the CO2 concentrations, as CO2 is not one of the eight
pollutants that are used to calculate the AQI.
Q.45 The decadal population data of a city are given in the following Table. The domestic water
consumption of the city is estimated to be 175 litres per capita per day in the year 2041. Considering
2011 population as the base year and using arithmetic growth method of population forecasting,
the daily domestic water demand of the city in the year 2041 (in million litres per day, rounded off
to two decimal places) will be ___________.
Year 1981 1991 2001 2011
Population 1,80,750 1,95,850 2,15,300 2,45,450
Ans:- 53.00 to 56.00
Solution:
The daily domestic water demand of the city in the year 2041 will be 5.01 million litres per day. Here is
how to find the answer:
The arithmetic growth method is a method of population forecasting that assumes that the population
grows at a constant rate over time. It estimates the future population by adding a fixed number of
individuals to the current population in each period.
To use the arithmetic growth method, we need to calculate the average rate of increase of population
(x̄) and the number of decades (n) between the base year and the future year. Then, we can apply the
formula:
Pn=P0+nx
Hemant Vilas Parulekar 65

here P_n is the future population, P_0 is the base population, n is the number of decades, and x� is the
average rate of increase of population.
Given that the base year is 2011 and the future year is 2041, we can calculate the number of decades as
follows:

Given that the population data of the city are given in the table, we can calculate the average rate of
increase of population as follows:

x =21533.33
Therefore, the future population in 2041 is:
Pn=P0+nx
P2041=P2011+3×21533.33
P2041=245450+64599.99
P2041=309949.99
To round off the future population to the nearest integer, we need to look at the decimal part and see if it
is 0.5 or more. If it is, we round up the integer part by one. If it is less than 0.5, we leave the integer part
as it is. In this case, the decimal part is 0.99, which is more than 0.5, so we round up the integer part by
one. Therefore, the future population rounded off to the nearest integer is:
P2041=309950
Given that the domestic water consumption of the city is estimated to be 175 litres per capita per day in
the year 2041, we can calculate the daily domestic water demand of the city as follows:
Water demand=Population× Consumption
Water demand=309950×175
Water demand=54241250
To convert the water demand from litres per day to million litres per day, we need to divide it by
1000000. Therefore, the water demand in million litres per day is:
66 GATE Architecture and Planning: Comprehensive Question Bank

Water demand=54.24125
To round off the water demand to two decimal places, we need to look at the third decimal place and
see if it is 5 or more. If it is, we round up the second decimal place by one. If it is less than 5, we leave the
second decimal place as it is. In this case, the third decimal place is 1, which is less than 5, so we leave
the second decimal place as it is. Therefore, the water demand rounded off to two decimal places is:
Water demand=54.24
Q.46 The activity duration, early start, early finish, late start,
and late finish (in weeks) for activities P, Q, R, and S are
shown in the following figure. The interfering float of
activity R (in weeks, in integer) is ___________.

Ans:- 6
Explanation:
Term Interfering float Free float
Definition Interfering float is the amount of time Free float is the amount of time an
an activity can be delayed or extended activity can be delayed or extended
without: without delaying the early start of its
Delaying the early start of any of its immediate successor activity.It represents
successors. Being affected by delays in its the flexibility you have to adjust a task’s
predecessors. It represents the flexibility timing without negatively impacting the
you have to adjust an activity’s timing start dates of tasks that directly depend
within a specific window, independent on it.
of both its preceding and succeeding
tasks
Formula Interfering Float (IF) = Minimum(Late Free Float (FF) = Early Start of the
Finish of Successors) - Early Finish (EF) Successor (ES(succ)) - Early Finish (EF)

Term Independent float Total float


Definition Independent float is the amount of time Total float is the maximum amount of time
an activity can be delayed or extended an activity can be delayed or extended
without delaying the early start of any of from its early start date without delaying
its successors and without being affected the project finish date.It represents the
by the delays of its predecessors. flexibility you have within a project
It represents the flexibility you have to schedule to adjust task timings without
adjust a task’s timing within a specific impacting the overall timeline.
window, independent of both its
predecessors and successors.
Hemant Vilas Parulekar 67

Formula Independent Float (IF) = Minimum(Late There are two common formulas to
Finish of Successors) - Early Finish (EF) calculate total float:
Total Float (TF) = Late Finish (LF) - Early
Finish (EF)
Total Float (TF) = Late Start (LS) - Early
Start (ES)
From the figure,
Activity Duration Predecessors Early Early Late Late Total Free
Activity intF
start finish start finish float float
P 2 -
P 10 12 10 12 0 0 0
Q 9 P Q 12 21 12 21 0 0 0
R 3 P R 12 15 24 27 12 6 6
S 2 Q,R S 21 23 27 29 6 - -
Q.47 A 230 mm thick brick wall of 10 m length and 3 m height is built using a Flemish bond. The size of
the bricks used is 230 mm x 112.5 mm x 70 mm. Assuming a mortar thickness of 5 mm, the number
of bricks required (in integer) is ___________.
Ans:- 3400-3500
Solution:
Total length of brick wall to be built=10m
As per the size of brick used, 1 unit of 3 bricks (header and stretcher as shown in the figure below) 5
mm mortar will cover length of 347.5 mm= 0.347 m
Thus, number of units ( 3 bricks each) required in single layer for covering 10 mm length=
10/0.347=28.81=29
Thus, actual number of bricks required in single layer=29×3=87…….(1)
The height of each brick is 70 mm
Thus, a single layer including mortar thickness of 5mm covers height of 75mm 0.075 m
Thus the number of courses of bricks required to built wall of height 3m=40……(2)
Thus from one and 2 the number of bricks required to build the wall of given dimension=87×40=3480
68 GATE Architecture and Planning: Comprehensive Question Bank

Q.48 The reflected ceiling plan and section of a reinforced cement concrete roof are shown in the following
Figure. All the beams are 300 mm wide, 600 mm deep (including 150 mm slab) equidistantly placed
center to center. Assuming 1% of concrete volume is occupied by reinforcement bars, the volume of
concrete (in cubic meters, rounded off to two decimal places) is ___________.

Ans:- 24.50-25.30
Solution:
The volume of roof without considering the intermediate sunks =10×10×600mm=60m3
From, the figure, the dimensions of square sunks between beams can be calculated as 2.94m×2.94m=8.64m2
Net depth of square sunk=depth of beam-dept of slab=600-150=450mm=0.45m
The volume of 9 no. of sunks=9×8.64×0.450=34.992 m3
Thus, volume of formwork available for pouring RCC (slab+beam) =60-35=25m3
It is given that; 1% volume is occupied by reinforcement bars
Thus, volume of concrete=25-(0.01×25) =24.75m3
Q.49 The following graph represents the income
distribution among the population of a country.
The Gini Coefficient of the country (rounded off
to three decimal places) is ___________.
Hemant Vilas Parulekar 69

Ans:- 0.240 to 0.27


Solution:
The Gini Coefficient is a measure of income inequality that ranges from 0 (perfect equality) to 1 (perfect
inequality). It is calculated as the ratio of the area between the line of perfect equality and the Lorenz
curve (the blue curve in the graph) to the total area under the line of perfect equality.
Gini coefficient= A/A+B
Where A= area above Lorenz curve
B= area below Lorenz curve
Here area above Lorenz curve A
= A(ΔPTS)-Area below lorez curve
=1/2×100×100=5000
Area below Lorenz curve B= A(ΔPVQ)+ A(QVUR)+ A(ΔRUTS)
=270+1260+2175=3705
Thus, Area above lorenz curve A= 5000-3705=1295
Thus,Gini coefficient= 1295/ (1295+3705)= 0.259
PART B1: FOR Architecture CANDIDATES ONLY
Q.50 – Q.56 Carry ONE mark Each
Q.50 Which of the following processes is used for surface treatment of metals?
(A) Soldering (B) Thermoplating (C) Extrusion (D) Riveting
Ans:- (B)
Explanation:
Surface treatment is a process of applying a coating or modifying the surface of a material to improve
its appearance, performance, or durability.
Process Soldering Thermoplating
Description Soldering is a process of joining Thermoplating is a type of surface
two metal pieces by melting a filler treatment that involves heating a metal and
metal (solder) between them. It is dipping it into a molten metal bath to form
not a surface treatment, but a joining a protective layer on the surface. It is also
technique. known as hot-dip galvanizing or hot-dip
coating.
Image
70 GATE Architecture and Planning: Comprehensive Question Bank

Process Extrusion Riveting


Description Extrusion is a process of forcing a Riveting is a process of fastening two metal
metal through a die to create a desired pieces by inserting and deforming a metal
shape or profile. It is not a surface pin (rivet) through holes in them. It is not a
treatment, but a forming technique. surface treatment, but a fastening technique.
Image

Q.51 Among the following monuments of ancient Greece, the only Octastyle Peripteral temple with
eight towering Doric columns lining both east and west facades is ___________.
(A) Temple of Athena (B) Temple of Apollo (C) The Parthenon (D) Temple of Horus
Ans:- (C)
Explanation:
An octastyle peripteral temple is a temple that has eight columns on the front and back facades and a
single row of columns around the main chamber (cella). A Doric column is a type of column that has a
simple capital (top part) and no base (bottom part).
The Parthenon is a famous octastyle peripteral temple that was dedicated to the goddess Athena. It was
built in the 5th century BC on the Acropolis of Athens. It is considered one of the finest examples of
classical Greek architecture.
The Temple of Athena is not a specific temple, but a general name for any temple that was dedicated
to the goddess Athena. There were many temples of Athena in ancient Greece, but not all of them were
octastyle peripteral or had Doric columns.
The Temple of Apollo is also not a specific temple, but a general name for any temple that was dedicated
to the god Apollo. There were many temples of Apollo in ancient Greece, but not all of them were
octastyle peripteral or had Doric columns.
The Temple of Horus is not a Greek temple, but an Egyptian temple that was dedicated to the god
Horus. It was built in the 2nd century BC in Edfu, Egypt. It has a different architectural style than the
Greek temples, with pylons, hypostyle halls, and hieroglyphs.
Q.52 An Ultrasonic Pulse Velocity (UPV) test was done on a hardened concrete element using a direct
transmission method as per IS 516 (Part 5/Section 1): 2018. The distance between the transducer
and receiver was 600 mm. The time taken for the induced wave to travel this distance is measured
as 0.18 milliseconds. Based on the following Table, the concrete quality grading is ___________.
Velocity (km/s) – cross probing Concrete quality grading
Above 4.4 Excellent
3.75 – 4.4 Good
3.0 – 3.75 Doubtful
Less than 3.0 Poor
(A) Excellent (B) Good (C) Doubtful (D) Poor
Hemant Vilas Parulekar 71

Ans:- (C)
Explanation:
To find the concrete quality grading, we need to calculate the ultrasonic pulse velocity (UPV) of the
concrete element. The UPV is the ratio of the distance between the transducer and receiver to the time
taken for the induced wave to travel this distance.
Given that the distance between the transducer and receiver is 600 mm and the time taken for the
induced wave to travel this distance is 0.18 milliseconds, we can calculate the UPV as follows:
UPV = Distance / Time UPV = 600 mm / 0.18 ms UPV = 3.33 m/s
To convert the UPV from m/s to km/s, we need to multiply it by 1000. Therefore, the UPV in
km/s is:
UPV = 3.33 m/s * 1000 UPV = 3.33 km/s the range in table shows doubtfull.
Q.53 Which of the following is/are example(s) of Tomb Architecture of Ancient Egypt?
(A) Step Pyramid of Zoser, Sakkara (B) Great Temple of Abu-Simbel
(C) Temple of Khons, Karnak (D) Mastabas of Gizeh
Ans:- A,D
Explanation:
The Step Pyramid of Zoser is the oldest known pyramid in Egypt, built for the pharaoh Djoser in the
27th century BCE. It consists of six mastabas (rectangular structures) of decreasing size stacked on top
of each other. It is located in the necropolis of Sakkara, near Memphis.
The Mastabas of Gizeh are the tombs of the nobles and officials who served the pharaohs of the Fourth
Dynasty (c. 2575–c. 2465 BCE). They are rectangular structures with sloping sides and flat roofs, built
of mud bricks or stone. They are located near the famous pyramids of Gizeh, such as the Great Pyramid
of Khufu.
he Great Temple of Abu-Simbel is not a tomb, but a temple dedicated to the pharaoh Ramesses II and
the gods Amun, Ra-Horakhty, and Ptah. It was built in the 13th century BCE and features four colossal
statues of Ramesses II at the entrance. It is located near the border with Sudan.
The Temple of Khons is not a tomb, but a temple dedicated to the god Khons, the son of Amun and
Mut. It was built in the 12th century BCE by the pharaoh Ramesses III and later expanded by other
rulers. It is located within the complex of Karnak, near Luxor.
Q.54 If Aluminium: Anodisation : : Glazing : X, which of the following choices represent X?
(A) Hard coating (B) External cement plastering
(C) Tempering (D) Free-standing vertical greening
Ans:- A, C
Explanation:
Anodisation is a process of forming a protective oxide layer on the surface of aluminium by electrolysis.
It increases the corrosion resistance, hardness, and durability of aluminium1.
Glazing is a process of applying a thin layer of glass or ceramic on the surface of pottery or metal by
heating. It improves the appearance, water resistance, and strength of the material2.
72 GATE Architecture and Planning: Comprehensive Question Bank

Tempering is a process of heating and cooling glass or metal to a specific temperature and rate. It
reduces the brittleness and increases the toughness of the material.
External cement plastering is a process of applying a mixture of cement, sand, and water on the
external walls of a building. It provides a smooth and protective surface for the walls. It is not analogous
to anodisation or glazing.
Free-standing vertical greening is a system of growing plants on a vertical structure that is independent
of a building.
Q.55 A blackbody radiant heating panel of 5 m2 surface area at 35 oC surface temperature is located 1 m
away from a 1 m2 surface at 20 oC. The Stefan-Boltzmann constant is 5.6703×10−8 W m−2 K−4. The
rate of radiant heat emission by the radiant heating panel (in W, rounded off to two decimal places)
is ___________.
Ans:- 2550 to 2560
Solution:
To find the rate of radiant heat emission by the radiant heating panel, we can use the Stefan-Boltzmann
law, which states that the power emitted per unit area of a blackbody is proportional to the fourth power
of its absolute temperature. Therefore, we can write:
P=σAT4
Where P is the power emitted, σ is the Stefan-Boltzmann constant, A is the surface area, and T is the
absolute temperature. We can convert the given temperatures from degrees Celsius to kelvins by adding
273.15. Thus, we have:
273.15+35=308.15k
Putting values in the equation:
P=(5.6703×10−8)(5)(308.15)4=2556.37W
Therefore, the rate of radiant heat emission by the radiant heating panel (in W, rounded off to two
decimal places) is 2560W.
Q.56 A hypothetical truss comprising of
weightless members is shown in the
following Figure. Assuming tension to be
positive and compression to be negative,
the value of force in member TU (in kN,
rounded off to one decimal place) is
___________.

Ans:- 0
Explanation:
The joint T has two collinear members (VT and TS) and third member TU is non-colinear member.
AS, there is no external force acting at joint T, thus, the non-colinear member TU is zero-force member.
Hemant Vilas Parulekar 73

Q.57 – Q .65 Carry TWO marks Each


Q.57 Match the illustrations of Arch Types in Group I with their corresponding names in Group II.
Group I Group II
(P) (1) Venetian Arch

(Q) (2) Ogee Arch

(R) (3) Moorish Multifoil Arch

(S) (4) Corbelled Arch

(5) Shouldered Arch


(A) P-2, Q-3, R-1, S-4 (B) P-3, Q-1, R-2, S-5 (C) P-3, Q-2, R-5, S-4 (D) P-5, Q-4, R-3, S-1
Ans:- (B)
Explanation:
Arch type Venetian Arch Ogee Arch
Description A Venetian arch is an arch that has An ogee arch is an arch that has a pointed
a pointed apex and two symmetrical apex and two symmetrical curves that
curves that form a flattened S-shape. It form a sharp S-shape. It is often used in
is commonly found in Venetian Gothic Gothic architecture, especially in windows
architecture, especially in palaces and and doorways.
churches.
Image
74 GATE Architecture and Planning: Comprehensive Question Bank

Arch type Moorish Multifoil Arch Corbelled Arch


Description A Moorish multifoil arch is an arch that A corbelled arch is an arch-like structure
has multiple lobes or foils on its inner that is formed by overlapping blocks
curve. It is commonly found in Islamic of stone or brick that gradually project
architecture, especially in Spain and towards the center until they meet at the
North Africa. top.
Image

Arch type Shouldered Arch


Description A shouldered arch is an arch that has a flat section at the base of each side, creating
a rectangular shape.
Image

Q.58 Match the architectural projects in Group I with their corresponding architects in Group II.
(A) P-4, Q-5, R-2, S-3 Group I Group II
(B) P-4, Q-1, R-5, S-2 (P) Indian Institute of Management (1) Revathi Kamath
(C) P-2, Q-4, R-5, S-1 Bangalore
(Q) Osho International Meditation (2) Brinda Somaya
(D) P-3, Q-5, R-1, S-2
Resort, Pune
(R) Nalanda International School, (3) Roger Anger
Vadodara
(S) Matrimandir, Auroville (4) B. V. Doshi
(5) Hafeez Contractor
Ans:- (A)
Explanation:
The Indian Institute of Management Bangalore is a prestigious business school in India, designed by
B. V. Doshi, a renowned architect and urban planner who won the Pritzker Prize in 2018. The campus
features a blend of modern and traditional elements, such as courtyards, terraces, and red brick walls.
The Osho International Meditation Resort in Pune, India, was designed by renowned Indian architect
Hafeez Contractor. He is known for his distinctive modernist style, often incorporating elements of
traditional Indian architecture into his designs.
Hemant Vilas Parulekar 75

The architect behind the beautiful Nalanda International School in Vadodara is the renowned Brinda
Somaya, a name synonymous with sustainable and culturally sensitive design in India.
She is also the founder and principal architect of Somaya & Kalappa Consultants, a Mumbai-based
architectural firm that specializes in conservation, restoration, and sustainable design. She has won
several awards and honors for her work, such as the UNESCO Asia-Pacific Heritage award, the Leading
European Architects Forum award, and the Pritzker Architecture Prize. She is also a founder trustee of
the HECAR Foundation, which promotes the works of women architects in South Asia.
The Matrimandir is a spherical temple and the focal point of Auroville, an experimental township in
Tamil Nadu. The temple was designed by Roger Anger, a French architect and disciple of Sri Aurobindo
and The Mother, the founders of Auroville. The temple has a golden metallic exterior and a white
marble interior, symbolizing the sun and the soul.
Q.59 Match the structural joining systems in Group I with the corresponding materials for which they
are commonly used in Group II.
(A) P-4, Q-1, R-2, S-5 Group I Group II
(B) P-3, Q-5, R-1, S-2 (P) Welding (1) Glass
(C) P-2, Q-3, R-5, S-1 (Q) Spider Connector (2) Plastic
(R) Mortise and Tenon (3) Brick
(D) P-4, Q-1, R-5, S-3
(S) Mortar (4) Steel
(5) Timber

Ans:- (D)
Explanation:
Welding is a process of joining two pieces of metal by melting them together using high heat. It is
commonly used for steel structures, such as bridges, buildings, and pipelines.
Spider connector is a type of glass fitting that consists of a metal hub and several arms that attach to the
glass panels. It is commonly used for glass facades, canopies, and balustrades.
Mortise and tenon is a type of wood joint that consists of a hole (mortise) in one piece of wood and
a protruding tongue (tenon) in another piece of wood that fits into the hole. It is commonly used for
timber frames, furniture, and doors.
Mortar is a mixture of sand, water, and cement or lime that is used to bond bricks or stones together. It
is commonly used for brick walls, chimneys, and fireplaces.
Q.60 Match the Instruments in Group I with the corresponding climate parameters in Group II.
(A) P-3, Q-5, R-1, S-2 Group I Group II
(B) P-3, Q-4, R-5, S-2 (P) Pyranometer (1) Humidity
(C) P-5, Q-3, R-2, S-4 (Q) Disdrometer (2) Wind
(R)Hygrometer (3) Solar Radiation
(D) P-1, Q-2, R-3, S-5
(S)Anemometer (4) Pressure
(5) Precipitation
76 GATE Architecture and Planning: Comprehensive Question Bank

Ans:- (A)
Explanation:
A pyranometer is an instrument that measures solar radiation on a planar surface. It has a thermopile
sensor that detects the difference in temperature between a black surface that absorbs radiation and a
reference surface that does not. It also has a glass dome that limits the spectral response and protects
the sensor from environmental factors.
A disdrometer is an instrument that measures the size and velocity of raindrops. It uses a laser beam
or a piezoelectric sensor to count and classify the raindrops that fall through a sampling area. It can
provide information about the type and intensity of precipitation.
A hygrometer is an instrument that measures humidity, which is the amount of water vapor in the
air. There are different types of hygrometers, such as hair hygrometers, psychrometers, and electronic
hygrometers, that use different methods to measure the change in moisture content or relative humidity.
An anemometer is an instrument that measures wind speed and wind pressure. It usually has three
cups or propellers that spin around a vertical axis when the wind blows. The faster the cups spin, the
higher the wind speed. Anemometers are important tools for meteorologists and physicists who study
the weather and the air movement.
Q.61 In traditional Indian temple architecture, which of the following statement(s) is/are true?
(A) Jagamohana refers to a dancing hall
(B) Gopuram refers to an entrance tower
(C) Char-chala refers to a roof composed of four triangular segments
(D) Vimana refers to the structure over the Garbhagriha
Ans:- B, C, D
Explanation:
(A) Jagamohana: A jagamohana is a porch or hall that is attached to the main shrine of a Hindu temple.
It is usually the first structure that a visitor encounters after entering the temple compound. It serves
as a place for devotees to gather, worship, and witness rituals. The term jagamohana literally means
“delighting the world”.
(B) Gopuram: A gopuram is a monumental tower that marks the entrance of a Hindu temple, especially
in South India. It is usually decorated with sculptures, paintings, and carvings depicting various gods,
goddesses, and mythological scenes. The term gopuram means “cow’s
horn”, as it resembles the shape of a cow’s horn.
(C).Char-chala: A char-chala is a type of roof that is composed of four triangular segments that meet
at a point. It is a characteristic feature of Bengali temple architecture, especially in West Bengal and
Bangladesh. The term char-chala means “four slopes”.
(D) Vimana: A vimana is a structure that covers the garbhagriha or the sanctum sanctorum of a Hindu
temple. It is usually shaped like a pyramid, dome, or spire, and it represents the mythical abode of the
deity. The term vimana means “flying palace” or “vehicle”.
Q.62 Which of the following factors impact Daylight Autonomy of a built space?
(A) Orientation of building (B) Glare caused by daylight
(C) Latitude and longitude of building location (D) Fenestration size
Hemant Vilas Parulekar 77

Ans:- A, C, D
Explanation:
The factors that impact Daylight Autonomy of a built space are:
(A) Orientation of building: The orientation of the building affects the amount and direction of sunlight
that enters the space. Different orientations have different advantages and disadvantages depending on
the climate and the season. For example, south-facing windows can provide more daylight and passive
heating in winter, but may cause overheating and glare in summer.
(B) Glare caused by daylight: Glare is the sensation of discomfort or reduced visibility caused by
excessive brightness or contrast in the visual field. Glare can reduce the quality and usability of daylight,
and may require shading devices or artificial lighting to mitigate its effects. Glare can be caused by
direct sunlight, reflected sunlight, or bright sky.
(C)Latitude and longitude of building location: The latitude and longitude of the building location
affect the angle and duration of sunlight that reaches the space. Higher latitudes have lower sun angles
and shorter daylight hours in winter, and higher sun angles and longer daylight hours in summer.
Longitude affects the time zone and the solar time of the location, which may differ from the standard
time.
D) Fenestration size: Fenestration is the arrangement, design, and size of windows and other openings
in a building. Fenestration size affects the amount and distribution of daylight that enters the space.
Larger fenestration sizes can provide more daylight and views, but may also cause more heat loss or
gain, glare, and noise. Fenestration size should be balanced with other factors, such as orientation,
shading, insulation, and ventilation.
Q.63 For the beam shown in the following
Figure, assuming a sagging moment
(generating tensile stresses at the bottom
fibre) as positive and a hogging moment
(generating tensile stresses at the top fibre)
as negative, the bending moment
(in kN.m, rounded off to one decimal place)
at section X-X is ___________.
Ans: - 20
Explanation:
Rw + Ry = (4 x 20 + 120) = 200 kN
Taking moment at w =0
(20 x 4 x 2) – (Ry x 4) + (120 x 5) = 0
160 – 4Ry + 600 = 0
Ry = 190kN
Thus, Rw = 10 kN
Thus, bending moment at X-X = (Rw x 2) – (20 x 2 x 1) = 20 -40 = -20 kN.m
78 GATE Architecture and Planning: Comprehensive Question Bank

Q.64 The acoustical absorption of a wall panel in each octave band is tabulated below.
The Noise Reduction Coefficient of the wall panel (rounded off to two decimal places) is
___________.
63 125 250 500 1000 2000 4000 8000 16000
Hz Hz Hz Hz Hz Hz Hz Hz Hz
0.1 0.2 0.5 0.5 0.7 0.8 0.8 0.9 0.9
Ans:- 0.60 to 0.65
Explanation:
Noise Reduction Coefficient = arithmetic average of sound absorption coefficient for
Frequencies given, Thus, here Noise Reduction Coefficient = (0.1 + 0.2 + 0.5 + 0.5 + 0.7 + 0.8 + 0.8 +
0.9 + 0.9)/9 = 5.4/9 = 0.60
Q.65 A room is maintained at a wet bulb temperature of 25 oC, globe temperature of 30 oC, and air
velocity of 0.5 m/s. The decrease in Tropical Summer Index when the air velocity is increased to 3
m/s (in oC, rounded off to two decimal places) is ___________.
Ans: - 2.20 to to -1.50 or 1.50 to 2.20
Explanation:
Please Refer GATE-2018, Q. 11, for information related to definition and formula of ‘Tropical
Summer Index (TSI)’
TSI = 1/3 tw + 3/4 tg - 2V 1/2
Where, TSI = Tropical Summer Index, tw = Wet bulb Temperature (0C), tg = Globe temperature (0C), V
1/2
= Square root of air velocity (m/s)
Thus, in case 1st, TSI1 = (1/3 x 25) + (3/4 x 30) - 2(0.5)1/2
= 8.34 +22.5 - 1.41 = 29.43
In case 2nd, the air velocity increased to 3m/s
Thus, TSI2 = (8.34 = 22.5) - 2(3)1/2 = 30.84-3.46 = 27.38
Thus, decrease in Tropical Summer Index = 29.43 - 27.38 = 2.05

PART B2: FOR Planning CANDIDATES ONLY


Q.66 – Q.72 Carry ONE mark Each
Q.66 Which of the following is the National Electronic Toll Collection System implemented by the
National Payment Corporation of India?
(A) e-Pass
(B) E-ZPass
(C) HashTag
(D) FASTag
Hemant Vilas Parulekar 79

Ans:- (D)
Explanation:
The National Electronic Toll Collection System implemented by the National Payment Corporation
of India is (D) FASTag. FASTag is a device that employs Radio Frequency Identification (RFID)
technology for making toll payments directly while the vehicle is in motion. FASTag is linked to a
prepaid or savings/current account and the toll fare is deducted from the linked account. FASTag offers
the convenience of cashless payment and reduces traffic congestion at toll plazas.
Q.67 The shaded area in the following demand-supply
graph is known as ___________.
(A) Consumer Surplus ‘
(B) Consumer Deficit
(C) Producer Surplus
(D) Producer Deficit

Ans:- (A)
Explanation:
The shaded area shows, that ‘price’ of the commodity decreases if the ‘supply’ increases beyond the
consumer demand (I.e., consumer surplus).
Q.68 Identify the following traffic interchange.
Directional
Trumpet
Clover-Leaf
(D) Diamond

Ans:- (C)
Explanation: Please refer (Q. 18) GATE-2017 for more information
Q.69 Which of the following is/are Value Capture Method(s)?
(A) Building construction fees
(B) Fees for changing agricultural to non-agricultural land use
(C) User charge
(D) Premium on additional FSI/FAR
80 GATE Architecture and Planning: Comprehensive Question Bank

Ans:- B, D
Explanation:
Value capture is a type of public financing that recovers some or all of the value that public infrastructure
generates for private landowners. It can be used to offset the costs of providing and maintaining public
services and amenities, such as roads, parks, schools, and transit.
There are different methods of value capture that can be applied depending on the context and the
objectives of the public sector. Some of the common methods are:
Land value tax: A tax levied on the unimproved value of land, which reflects the benefits of public
infrastructure and services. This tax encourages efficient land use and discourages land speculation.
Betterment levy: A one-time charge imposed on landowners who benefit from a specific public project,
such as a new road or a rezoning. This charge captures a portion of the increase in land value resulting
from the project.
Development charges: A fee charged to developers who build new projects that require additional
public infrastructure and services, such as water, sewer, and roads. This fee covers the costs of providing
and expanding the infrastructure and services4
Tax increment financing: A mechanism that uses the future increase in property tax revenue from
a designated area to finance public improvements within that area, such as urban renewal or transit
projects. This mechanism stimulates private investment and development in the area5
Joint development: A partnership between the public and private sectors to develop land owned or
acquired by the public sector, such as land near transit stations. The public sector provides the land and
the private sector provides the capital and expertise to build mixed-use projects that generate revenue
and ridership for the public sector.
Transfer of development rights: A scheme that allows landowners to sell or transfer their development
rights to other landowners who want to increase their development potential, such as building height or
density. The public sector regulates the transfer of development rights to preserve certain areas, such as
heritage or environmental sites, and to promote development in other areas, such as transit corridors.
Q.70 Which among the following is/are model(s) of Public-Private Partnership (PPP) used for
infrastructure projects?
(A) BOLD (B) BOLT (C) BOOT (D) BPOT
Ans:- B, C
Explanation:
Public-Private Partnership (PPP) is a form of collaboration between a government agency and a private-
sector company that can be used to finance, build, and operate projects, such as public transportation
networks, parks, and convention centres. There are different models of PPP that vary in the degree
of involvement and risk sharing of the public and private partners. Some of the commonly adopted
models of PPP are:
BOT (Build-Operate-Transfer): The private partner builds the project, operates it for a specified
period of time, and then transfers it to the public partner.
BOO (Build-Own-Operate): The private partner builds the project, owns it, and operates it indefinitely,
without transferring it to the public partner.
BOOT (Build-Own-Operate-Transfer): The private partner builds the project, owns it, operates it for
a specified period of time, and then transfers it to the public partner.
Hemant Vilas Parulekar 81

BLT (Build-Lease-Transfer): The private partner builds the project, leases it to the public partner for
a specified period of time, and then transfers it to the public partner.
BOLT (Build-Own-Lease-Transfer): The private partner builds the project, owns it, leases it to the
public partner for a specified period of time, and then transfers it to the public partner.
DBFO (Design-Build-Finance-Operate): The private partner designs, builds, finances, and operates
the project for a specified period of time, and then transfers it to the public partner.
LDO (Lease-Develop-Operate): The public partner leases the project to the private partner, who
develops and operates it for a specified period of time, and then transfers it to the public partner.
OMT (Operate-Maintain-Transfer): The public partner builds the project and transfers it to the
private partner, who operates and maintains it for a specified period of time, and then transfers it back
to the public partner.
Q.71 The measured spot speeds (in km/h) of 10 vehicles from a traffic stream are 45, 35, 25, 51, 45, 38,
61, 42, 47, and 49. The Time Mean Speed of the traffic stream (in km/h, rounded off to one decimal
place) is ___________.
Ans:- 43 to 44
Solution:
To find the time mean speed of the traffic stream, you need to sum all of the spot speeds and divide by
the number of spot speeds.
The sum of the spot speeds is 45 + 35 + 25 + 51 + 45 + 38 + 61 + 42 + 47 + 49 = 438. Dividing this sum
by the number of spot speeds, which is 10, gives a time mean speed of 438/10 = 43.8 km/h. Therefore,
the time mean speed of the traffic stream is 43.8 km/h.
Q.72 In a township, the price of each house was 25,00,000 (in Indian Rupees) last month.
The number of houses sold in a month (Q in thousands) is sensitive to the price of the house
(P in Indian Rupees) and establishes a relationship as Q = 6685 – 0.00158P. If the price of each
house increases by 20% in the current month, then the decrease in sale of the houses (in percentage,
rounded off to two decimal places) compared to last month will be ___________.
Ans:- 27.00 to 30.00
Explanation:
The question based on ‘Price Elasticity of Demand (PED)’was earlier asked in (Q.24)GATE-2019
Assume price of each house sold last month as (P1) 25,00,000
Thus, as per given relation between no. of houses sold = Q1 = 6685-0.00158P1
& price of each house, the number of houses sold last = 6685-0.00158 x 2,50,000
month (Q1)
= 6685-3950
Q1 = 2735
Assume the increased price (by 20%) in the current P2 = P1 + (20% of P1)
month as (P2) 30,00,000
Thus, the number of houses sold (Q2) will be Q2 = 6685 – 0.00158P2
= 6685 – (0.00158 x 30,00,000)
Q2 = 1945
82 GATE Architecture and Planning: Comprehensive Question Bank

Thus, net decrease in sale of houses will be Q1 – Q2 = 2735 – 1945 = 790


Thus, % decrease in sale of houses compared to last (790/2735) x 100 = 28.89
month will be
Q.73 Match the models in Group I with their corresponding applications in Group II
(A) P-2, Q-1, R-5, S-4 Group I Group II
(B) P-1, Q-5, R-2, S-3 (P) Logit model 1.Trip assignment
(C) P-2, Q-3, R-5, S-4 (Q) Greenshield model 2.Modal split
(R) Gravity model 3.Traffic flow
(D) P-5, Q-3, R-4, S-2
(S) Multiple regression model 4.Trip generation
5.Trip distribution
Ans:- (C)
Explanation:
(P) Logit model: A statistical model that estimates the probability of an event occurring based on a
given dataset of independent variables. It can be used for classification and predictive analytics, and it
produces a probability between 0 and 1.
(Q) Greenshield model: A traffic flow model that assumes a linear relationship between speed and
density of vehicles on a road. It can be used to estimate the free flow speed, the jam density, the maximum
flow, and the optimum speed and density of traffic.
(R) Gravity model: A model that predicts the volume of flows of goods, services, or people between two
or more locations based on the population size and distance of the locations. It can be used to analyse
trade, migration, transportation, and other types of movement between places.
(S) Multiple regression model: A statistical model that estimates the relationship between two or more
independent variables and one dependent variable. It can be used to test hypotheses, explain variation,
and make predictions based on the data.
Trip assignment: The process of determining the routes that travellers will take to reach their destinations
based on the travel demand and the network characteristics.
Modal split: The proportion of travellers using different modes of transportation, such as car, bus, bike,
or walk.
Traffic flow: The movement of vehicles on a road or a network of roads. It can be characterized by
variables such as speed, density, and flow.
Trip generation: The process of estimating the number of trips that will be originated or attracted by
different zones or land uses.
Trip distribution: The process of estimating the number of trips that will be made between different
zones or lands uses.
Hemant Vilas Parulekar 83

Q.74 Match the proponents in Group I with the corresponding theories in Group II.
Group I Group II
P-2, Q-4, R-3, S-5 James Q Wilson and Creative Class
(B) P-4, Q-2, R-5, S-1 George K. Kelling
(C) P-5, Q-4, R-2, S-1 Sherry Arnstein Right to City
(R) Henry Lefebvre Drive-in Culture
(D) P-3, Q-5, R-2, S-4
Richard Florida Ladder of Citizen Participation
Broken Window
Ans:- (C)
Explanation:
Theory Description
Creative Class The term was coined by American urban studies theorist Richard Florida, who
argued that the creative class is a key driving force for economic development of
post-industrial societies. Florida identified two main groups within the creative
class: the super-creative core, which includes occupations that fully engage in
the creative process, such as computer programmers, scientists, writers, and
musicians; and the creative professionals, which include occupations that draw on
complex bodies of knowledge to solve specific problems, such as lawyers, doctors,
teachers, and engineers.
Florida also proposed that the creative class is attracted to places that have the
“three T’s” of economic growth: talent, technology, and tolerance. Talent refers
to the availability and diversity of skilled workers; technology refers to the level
of innovation and infrastructure; and tolerance refers to the openness and
acceptance of different cultures, lifestyles, and identities. Florida suggested that
cities and regions that have high levels of the three T’s can foster a creative and
entrepreneurial environment that stimulates economic and social progress.
Florida’s theory of the creative class has been influential and controversial in the
fields of urban planning, economic development, and cultural studies. Some
critics have challenged his empirical evidence, his causal claims, his normative
assumptions, and his policy implications. Some supporters have applied and
extended his ideas to different contexts, such as countries, industries, and
organizations
Right to City Right to the city is a concept and slogan that emphasizes the need for inclusivity,
accessibility, and democracy in urban spaces. The idea was first articulated by
French philosopher Henri Lefebvre in his 1968 book Le Droit à la Ville, in which
he argued that urban space should not be solely controlled by market forces, such
as commodification and capitalism but should be shaped and governed by the
citizens who inhabit it. The concept of the right to the city has been taken up by
a variety of social movements and urban activists around the world, who use it as
a rallying cry for greater social justice and democracy in the urban environment.
The right to the city can encompass a variety of demands, including demands for
government subsidized housing, access to public space, participation in urban
governance, and laws against displacement and gentrification, all of which aim to
address spatial inequalities in urban areas.
84 GATE Architecture and Planning: Comprehensive Question Bank

Ladder of The Ladder of Citizen Participation is a model proposed by Sherry Arnstein in


Citizen 1969, which describes the different levels of involvement and power that citizens
Participation can have in decision-making processes that affect their lives and communities.
The model uses the metaphor of a ladder with eight rungs, each representing a
degree of participation from non-participation to citizen power. The rungs are:
Manipulation: The lowest level of non-participation, where citizens are used as
puppets by power-holders to legitimize their decisions and actions.
Therapy: Another form of non-participation, where citizens are treated as passive
and ignorant patients who need to be cured by power-holders through education
or counselling.
Informing: A basic level of participation, where citizens are given access to
information by power-holders, but have no channel for feedback or influence.
Broken Window James Q. Wilson and George Kelling first introduced the Broken Window Theory
in 1982. It was based on their belief that signs of disorder, such as broken windows,
act as a signal that no one cares about the environment. This can lead to further
criminal activities like vandalism and theft.
By implementing policies and procedures that create a clean and orderly
environment, you can send the message that your business values order and
respect. This can help deter criminal activity and create a sense of safety.
It is important to remember that the Broken Window Theory is more than just
about physical disorder. It can also be applied to non-physical aspects of your
business, such as customer service and workplace policies.
Q.75 Match the Artists/Scientists in Group I with their corresponding contributions in Group II.
P-4, Q-3, R-1,S-2 Group I Group II
P-4, Q-1, R-5, S-2 Robert Park and Louis Wirth Poverty Map
P-5, Q-3, R-1, S-4 (Q)Jacob August Riis Cholera Map
(R)Charles Booth Tenement Shelter Photography
(D) P-4, Q-3, R-5, S-1
John Snow Urban Ethnography
Underground Sewerage Systems
Ans :- (A)
Explanation:
To answer this question, we need to know the contributions of the artists/scientists in Group I to the
fields of sociology, public health, and social reform. Here is a brief summary of their achievements:
(P) Robert Park and Louis Wirth were American sociologists who were part of the Chicago School of
Sociology. They developed a theoretical and empirical approach to the study of urban life, focusing on
issues such as immigration, assimilation, social ecology, and collective behaviour. They are considered
pioneers of urban sociology and urban ethnography.
(Q) Jacob August Riis was a Danish-American journalist and social reformer who used photography
and journalism to expose the harsh conditions of the poor and the immigrants in New York City’s
tenements. He published his famous book How the Other Half Lives in 1890, which shocked the public
and inspired social change. He is regarded as one of the first muckrakers and social documentary
photographers.
Hemant Vilas Parulekar 85

(R) Charles Booth was a British shipowner and social researcher who conducted a massive survey
of the life and labour of the people in London from 1886 to 1903. He collected and analysed data
on poverty, industry, religion, and other aspects of urban society. He produced a series of maps that
showed the distribution of poverty and wealth in different areas of the city. He also advocated for social
reforms such as old-age pensions and improved sanitation. He is considered one of the founders of
social administration and social policy.
(S) John Snow was a British physician and epidemiologist who is regarded as the father of modern
epidemiology and early germ theory. He is best known for his investigation of the cholera outbreak in
London in 1854, where he traced the source of the infection to a contaminated water pump on Broad
Street. He also conducted a natural experiment that compared the mortality rates of different water
suppliers in the city. He also made significant contributions to the development of anaesthesia and
medical hygiene.
Q.76 In the conceptual diagram of the city given below, P, Q, R, and S refer to urban patterns. Among the
choices given below, the correct association is

(A) P-Satellite town, Q-Urban fringe, R-TOD, S-Central Business District


(B) P-Central Business District, Q-Satellite town, R-TOD, S-Urban fringe
(C) P-Urban fringe, Q-TOD, R-Satellite town, S-Central Business District
(D) P-Satellite town, Q-Central Business District, R-TOD, S-Urban fringe
Ans:- (A)
Explanation:
Please refer GATE-2018, (Q. 36) (TOD) for more information.
A satellite town is a smaller town that is located near a larger city and is dependent on it for economic
and social activities.
An urban fringe is the outer edge of a city, where urban and rural land uses mix and often conflict.
A TOD (transit-oriented development) is a type of urban development that maximizes the amount of
residential, business, and leisure space within walking distance of public transport3.
A central business district (CBD) is the commercial and business center of a city, where land values are
high and building densities are greatest.
86 GATE Architecture and Planning: Comprehensive Question Bank

Based on these definitions, we can compare the choices with the diagram and see which one matches
the best. The diagram shows four zones (P, Q, R, and S) that are connected by a circular rail line. The
zones have different sizes, shapes, and colors, which indicate their different characteristics.
Option (A) is incorrect, because P is not a satellite town, but a large and dense zone that resembles a
CBD. Q is not an urban fringe, but a smaller and less dense zone that resembles a satellite town. R is not
a TOD, but a zone that has no rail access and is isolated from the other zones. S is not a CBD, but a zone
that has rail access and is close to the other zones, which resembles a TOD.
Q.77 Which among the following is/are the component(s) of the assimilative carrying capacity of urban
environment?
(A) Air (B) Water (C) Economy (D) Soil
Ans:- A, B, D.
Explanation:
Assimilative carrying capacity refers to the ability of the environment or a portion of the environment
to carry waste material without adverse effects on the environment or on users of its resources1. It
is a concept that involves both natural and human factors, such as resources, ecosystems, pollution,
population, and development.
Based on this definition, the components of the assimilative carrying capacity of urban environment
are (A) Air, (B) Water, and (D) Soil. These are the natural resources that can absorb and degrade the
waste generated by urban activities, such as transportation, industry, and consumption. However, the
assimilative capacity of these resources is limited by the quality and quantity of the resources, as well as
the type and amount of the waste.
(C) Economy is not a component of the assimilative carrying capacity of urban environment, because
it is not a natural resource that can carry waste material. Rather, it is a human factor that affects the
demand and supply of resources, and the level and pattern of development in urban areas.

Q.78 In the transportation network given below, P, Q, R, S, T,


and U are the nodes and values mentioned on the links
denote time in minutes. Which of the following options
represent the minimum spanning tree? (MSQ)
(A) PQ, QR, QT, TS, SU (B) PR, QR, RT, TU, SU
(C) PQ, QR, RT, TS, SU (D) PQ, QR, RS, ST, TU
Ans:- A, C
Explanation:
A minimum spanning tree (MST) is a subset of the edges of a connected, edge-weighted undirected
graph that connects all the vertices together, without any cycles and with the minimum possible total
edge weight graph /transportation network can have multiple spanning trees. To find the MST of a
given graph, we can use algorithms such as Kruskal’s or Prim’s2.
Using Kruskal’s algorithm, we can follow these steps:
Sort all the edges by their weights in ascending order.
Hemant Vilas Parulekar 87

Pick the smallest edge that does not form a cycle with the edges already in the MST. Repeat until there
are n - 1 edges in the MST, where n is the number of vertices.
Using Prim’s algorithm, we can follow these steps:
Start with any vertex and mark it as visited.
Pick the smallest edge that connects a visited vertex to an unvisited vertex. Add it to the MST and mark
the unvisited vertex as visited. Repeat until all the vertices are visited.
Using either algorithm, we can find that the MST of the given graph is option The weight of a spanning
tree is sum of weights given to each edge (‘node’ in case of transportation network) of the spanning tree.
A minimum spanning tree has (V-1) edges (nodes), where ‘V’ is the number of vertices I the given
graph/network. In the given problem, the minimum spanning tree has (6-1)b’5’ nodes, From the
definition, the weight of spanning tree mentioned in options A, B, C, & D can be calculated as shown
in the adjacent table.
It can be observed that, option A & C shows minimum spanning tree with total weight of 12 minutes.
Option Nodes Total Weight (time)
A PQ QR QT TS SU 12
2 2 3 3 2
B PR QR RT TU SU 14
2 2 3 4 2
C PQ QR RT TS SU 12
2 2 3 3 2
D PQ QR RS ST TU 17
2 2 6 3 4
Q.79 A vehicle count survey (in Passenger Car Unit) is conducted on a mid-block section of a road at
regular intervals of 15 minutes from 8:00 AM to 10:00 AM. Based on the data given in Table below,
the Peak Hour Factor (rounded off to two decimal places) for the given survey duration is ______
_____.
Time Interval Passenger Car Unit
8:00 AM – 8:15 AM 212
8:15 AM – 8:30 AM 248
8:30 AM – 8:45 AM 272
8:45 AM – 9:00 AM 315
9:00 AM – 9:15 AM 337
9:15 AM – 9:30 AM 405
9:30 AM – 9:45 AM 320
9:45 AM – 10:00 AM 267
Ans:- 0.83 to 0.87
Explanation:
Peak hour volume is a term used in traffic engineering to measure the volume of traffic that uses a road
segment, an intersection, or a lane during the hour of the day that observes the highest traffic volumes.
It is usually expressed in terms of passenger car units (pcu), which is a unit that converts different types
of vehicles into equivalent numbers of passenger cars based on their size and impact on traffic flow. It
can be identified as between 8.45 AM to 9.45 AM
88 GATE Architecture and Planning: Comprehensive Question Bank

Thus, Peak Traffic Volume (in PCU) during this hour = 315 + 337 + 405 + 320 =1377
Peak-Hour Factor (PHF): It is the average volume during the peak 60 minutes (hour) divided by four
times the average volume during the peak 15 minutes’ period.
Actual (Design) flow rate: It is calculated by dividing the peak hour volume by Peak Hour Factor (PHF)
or by multiplying the peak 15-minute volume by four.
Thus, Actual design flow rate = 1377/0.85
= 1620 PCU/h
Or, 405 x 4 = 1620 PCU/h
Time Interval Passenger Car Unit
8:00 AM – 8:15 AM 212
8:15 AM – 8:30 AM 248
8:30 AM – 8:45 AM 272
8:45 AM – 9:00 AM 315
9:00 AM – 9:15 AM 337
9:15 AM – 9:30 AM 405
9:30 AM – 9:45 AM 320
9:45 AM – 10:00 AM 267
Q.80 A land owner has shown interest in a Town Planning Scheme. Based on the details of the scheme
given in the following Table, the estimated Net Benefit to the land owner after land development
(in Indian Rupees, in integer) is ___________. in the following Table, the estimated Net Benefit to
the land owner after land development (in Indian Rupees, in integer) is ___________.
Original plot size 500 Sq. m
Original land value 1200 Indian Rupees per Sq.m
Plot deduction for development 40%
Developed land value 2800 Indian Rupees per Sq.m
Total betterment cost to be paid by the land owner 50% of the increased total land value
Ans:- 118000 to 122000
Solution:
Given, Original plot size (underdeveloped land) = 500 sq.m.
Given, Original land value (Undeveloped land) = 1200 INR per sq.m.
Thus, original value of total undeveloped plot = 1200 x 500 = Rs. 6,00,000
Given, Plot deduction for development = 40% of original plot size
Thus, net deduction of plot = 0.4 x 500 = 200 sq.m.
Thus, after deduction, net area of developed plot = 300 sq.m.
Given, Developed land value per sq.m. = 2800 INR per sq.m.
Thus, total land value = 2800 x 300 = Rs.8,40,000
Thus, net increase in land value after development = 8,40,000 - 6,00,000 = Rs. 2,40,000
Given Total betterment cost to be paid by the land owner = 0.5 x 2,40,000 = Rs. 1,20,000
Thus, Net benefit to land owner = 2,40,000 - 1,20,000 = Rs. 1,20,000.
Hemant Vilas Parulekar 89

Q.81 The year-wise cash flows (in Indian Rupees) of a construction project are given in the following
Table. If the annual discount rate for the project is assumed to be 12%, the Net Present Value (in
Indian Rupees, rounded off to two decimal places) for the project will be ___________.
Year Annual Cash Outflow Annual Cash Inflow
0 5,00,000 0
1 0 0
2 0 0
3 50,000 1,80,000
4 50,000 2,20,000
5 50,000 2,90,000
6 0 3,30,000
Ans: 3800 to 4200
Solution:
Net Present Value = Present Value of Cash Inflow - Present value of Cash Outflow…………..(A)
Discounting Factor (Dn) = 1 / (1+r)n
Where, r = Annual Discount rate
n = number of years ahead
The discounting factor for year 0 will always be ‘1’
Thus, by using this the discounting factor for year ‘1’ can be calculated as follows:
Dn = 1/(1+r)n = 1/(1+0.12)1 = 0.8929
Likewise, the discounting factor factor for year 2 to 6 can be derived as shown adjacent
Year Discounting Factor
0 1
1 0.8929
2 0.7971
3 0.7117
4 0.6355
5 0.5674
6 0.5066
As, Present value of annual cash outflow = Annual cash outflow x Discounting factor & present
value of annual cash inflow = annual cash inflow x discounting factor
Thus, the present value of annual cash outflow and inflow can be calculated as follows:
Year Annual Cash Present Value Annual Cash Present Value (Inflow)
Outflow (Outlow) Inflow
0 5,00,000 5,00,000 0 0
1 0 0 0 0
2 0 0 0 0
90 GATE Architecture and Planning: Comprehensive Question Bank

Year Annual Cash Present Value Annual Cash Present Value (Inflow)
Outflow (Outlow) Inflow
3 50,000 35,585 1,80,000 1,28,106
4 50,000 31,775 2,20,000 1,39,810
5 50,000 28,370 2,90,000 1,64,546
6 0 0 3,30,000 1,67,178
Total 5,95,730 Total 5,99,640
From the formula mentioned as (A), Net Present Value = 5,99,640 - 5,95,730 =3910
GATE QUESTION PAPER 2021
General Aptitude (GA)

Q. 1 – Q. 5 carry one mark each.


Q.1 (i) Arun and Aparna are here.
(ii) Arun and Aparna is here.
(iii) Arun’s families is here.
(iv) Arun’s family is here.
Which of the above sentences are grammatically CORRECT?
(A) (i) and (ii) (B) (i) and (iv) (C) (ii) and (iv) (D) (iii) and (iv)
Ans:- (B)
Q.2

The mirror image of the above text about the x-axis is


(A) (B)

(C) (D)

Ans:- (B)
Q.3 Two identical cube shaped dice each with faces numbered 1 to 6 are rolled simultaneously. The
probability that an even number is rolled out on each dice is:
(A) 1/36 (B) 1/12 (C) 1/8 (D) 1/4
Ans:- (D)
Solution:
Given,
Two cubes numbered 1 to 6
So, total outcomes = {1,2,3,4,5,6} = 6
Possibility of even number = Favorable outcomes = {2,4,6} = 3
The probability that an even number is rolled out for Cube 1 and Cube 2,
P(A) = P(B) = 3/6 = 1/2
The probability that an even number is rolled out on each dice is
P(A∩B) = P(A) × P(B) = 1/2 × 1/2 = 1/4
92 GATE Architecture and Planning: Comprehensive Question Bank

Q.4 Φ and ʘ are two operators on numbers p and q such that p ʘ q = p - q, and p Φ q = p × q Then,
(9 ʘ (6 Φ 7)) ʘ (7 Φ (6 ʘ 5))=
(A) 40 (B) -26 (C) -33 (D) -40
Ans:- (D)
Solution:
(9 ʘ (6 Φ 7)) ʘ (7 Φ (6 ʘ 5)) shall be 9 - (6 × 7) - [7 × (6 - 5)]
= 9 - 42 - (7 × 1)
= 9 - 42 - 7 = - 40 Ans.
Q.5 Four persons P, Q, R and S are to be seated in a row. R should not be seated at the second position
from the left end of the row. The number of distinct seating arrangements possible is:
(A) 6 (B) 9 (C) 18 (D) 24
Ans:- (B)
Solution:
Arrangements Possible:

The total number of ways = 3 × 3! = 3 × 3 × 2 × 1 = 18


Q. 6 – Q. 10 Multiple Choice Question (MCQ), carry TWO marks each (for each wrong answer: –2/3).
Q.6 On a planar field, you travelled 3 units East from a point O. Next you travelled 4 units
South to arrive at point P. Then you travelled from P in the North-East direction such
that you arrive at a point that is 6 units East of point O. Next, you travelled in the
North-West direction, so that you arrive at point Q that is 8 units North of point P.
The distance of point Q to point O, in the same units, should be _________
(A) 3 (B) 4 (C) 5 (D) 6

Ans:- (C)
Solution:
OQ = √(3)² + (4)² = 5
Hemant Vilas Parulekar 93

Q.7 The author said, “Musicians rehearse before their concerts. Actors rehearse their roles before the
opening of a new play. On the other hand, I find it strange that many public speakers think they
can just walk on to the stage and start speaking. In my opinion, it is no less important for public
speakers to rehearse their talks.”Based on the above passage, which one of the following is TRUE?
(A) The author is of the opinion that rehearsing is important for musicians, actors and public speakers.
(B) The author is of the opinion that rehearsing is less important for public speakers than for musicians
and actors.
(C) The author is of the opinion that rehearsing is more important only for musicians than public
speakers.
(D) The author is of the opinion that rehearsal is more important for actors than musicians.
Ans:- (A)
Q.8 1. Some football players play cricket.
2. All cricket players play hockey.
Among the options given below, the statement that logically follows from the two statements 1 and
2 above, is:
(A) No football player plays hockey. (B) Some football players play hockey.
(C) All football players play hockey. (D) All hockey players play football.
Ans:- (B)

Q.9

In the figure shown, PQRS is a square. The shaded portion is formed by the inter section
of sectors of circles with radius equal to the side of the square and centers at S and Q.The
probability that any point picked randomly within the square falls in the shaded area is
(A) 4 - π/2 (B) 1/2 (C) π/2 - 1 (D) π/4
94 GATE Architecture and Planning: Comprehensive Question Bank

Ans :- (C)
Solution:
Total area of the square = r × r = r²
Shaded area = (π.r²/4 - r²/2) + (π.r²/4 - r²/2)
where, r²/4 is area of quarter of circle and r²/2 is area of the triangles ∆PQR, ∆PSR
Therefore, Probability required = Shaded area ÷ Square area
= [(π.r²/4 - r²/2) + (π.r²/4 - r²/2)] / r²
= 2 r² (π/4 - 1/2)/ r²
= π/2 - 1 Answer
Q.10 In an equilateral triangle PQR, side PQ is divided into four equal parts, side QR is divided into six
equal parts and side PR is divided into eight equal parts. The length of each subdivided part in cm
is an integer. The minimum area of the triangle PQR possible, in cm², is
(A) 18 (B) 24 (C) 48√3 (D) 144√3

Ans:- (D)
Solution:
Let’s assume that the side of ∆PQR = d
Given,
(a/4, a/6, a/8) is an integer
Minimum area of ∆PQR when “d” is the LCM
[Least common multiple of (4,6,8)]
d = LCM (4,6,8) = 24
Area of ∆PQR = √3/4 × 24² = 144√3 Answer
Architecture and Planning (AR)
Q.1 – Q.25 Multiple Choice Question (MCQ), carry TWO marks each (for each wrong answer: 1/3).
Q.1 As per National Building Code of India, 2016, the function of an Automatic Rescue Device is to
(A) bring a stuck lift to the nearest landing level
(B) control fire in electrical system at plenum level.
(C) control the escape route lighting system.
(D) trigger fire sprinkler system.
Ans:- (A)
Hemant Vilas Parulekar 95

Q.2 Which among the following acronyms represents a thermal comfort index?
(A) PMV (B) NDVI (C) DEM (D) PCA
Ans:- (A)
Explanation:
(A) PMV: Predicted Mean Vote (PMV) created by Paul Ole Fanger. PMV is an index that aims to predict
the mean value of votes of a group of occupants on a seven-point thermal sensation scale.
(B) NDVI: Normalized Difference Vegetation Index
(C) DEM: Digital elevation model
(D) PCA: Principal Component Analysis PCA is defined as an orthogonal linear transformation that
transforms the data to a new coordinate system such that the greatest variance by some scalar projection
of the data comes to lie on the first coordinate (called the first principal component), the second greatest
variance on the second coordinate, and so on.
Q.3 Indian satellite sensor that can be used for very high-resolution mapping of urban areas is
(A) LANDSAT (B) CARTOSAT (C) RESOURCESAT (D) MODIS.
Answer:- AS per final answer key all answers have been marked correct.
Explanation:
(A) LANDSAT: This joint NASA/USGS program provides the longest continuous space based record
of Earth’s land in existence.
(B) CARTOSAT: Are satellite series launched by ISRO. The data from the satellite is used for detailed
mapping and other cartographic applications at cad-astral level, urban and rural infrastructure
development and management, as well as applications in Land Information System (LIS) and
Geographical Information System (GIS).
(C) RESOURCESAT: Digital elevation model
(D) MODIS: MODIS is an extensive program using sensors on two satellites that each provide complete
daily coverage of the earth, initiative of NASA.
Q.4 What is the smallest entity of raster data used in GIS?
(A) Line
(B) Pixel
(C) Point
(D) Polygon.
Answer:- (B)
Explanation:
Raster data is a type of spatial data that is made up of pixels or cells, which are usually regularly spaced
and square. Each pixel has an associated value and location coordinates. Raster data is useful for storing
data that varies continuously, such as elevation, temperature, or imagery.
A pixel is the smallest entity of raster data, as it represents the smallest unit of information that can be
stored and displayed in a raster image. The size of a pixel determines the resolution and the level of detail
of the raster data. A smaller pixel size means a higher resolution and more detail, but also a larger file
size and more processing time.
96 GATE Architecture and Planning: Comprehensive Question Bank

A line is a type of vector data that connects two or more vertices with paths. A line can represent linear
features, such as roads, rivers, or boundaries. A line is not a raster entity, as it is not made up of pixels
or cells.
A point is a type of vector data that has a single coordinate pair. A point can represent discrete features,
such as cities, trees, or wells. A point is not a raster entity, as it is not made up of pixels or cells.
A polygon is a type of vector data that has a closed shape with three or more vertices. A polygon can
represent areal features, such as buildings, lakes, or countries. A polygon is not a raster entity, as it is not
made up of pixels or cells.
Q.5 The correct sequence of stages during firing/burning of bricks is
(A) Dehydration – Oxidation – Vitrification – Cooling.
(B) Vitrification – Dehydration – Oxidation – Cooling.
(C) Oxidation – Dehydration – Vitrification – Cooling.
(D) Cooling – Oxidation – Vitrification – Dehydration.
Ans:- (A)
Explanation:
Dehydration is the stage where the bricks are heated to remove any remaining moisture from the clay.
This stage occurs at temperatures below 200°C and lasts for several hours.
Oxidation is the stage where the bricks are exposed to air and oxygen, which causes the organic matter
and iron compounds in the clay to burn and change color. This stage occurs at temperatures between
200°C and 800°C and lasts for several hours.
Vitrification is the stage where the clay particles melt and fuse together, forming a strong and durable
ceramic material. This stage occurs at temperatures above 800°C and lasts for a few hours.
Cooling is the stage where the bricks are gradually cooled down to room temperature. This stage occurs
at temperatures below 800°C and lasts for several hours.
Q.6 Industry Foundation Classes (IFC) in BIM is
(A) a module used to improve energy savings.
(B) an algorithm related to the precision of the BIM model.
(C) a program based on Bezier Splines.
(D) an object oriented data model to facilitate interoperability
Ans:- (D)
Explanation:
- Industry Foundation Classes (IFC) in BIM is an open international standard for Building Information
Model (BIM) data that are exchanged and shared among software applications used by the various
participants in the construction or facility management industry sector.
- IFC is an object-oriented data model that codifies the identity, characteristics, relationships, and
processes of objects and concepts in the built environment, such as buildings, structures, systems,
materials, and people.
Hemant Vilas Parulekar 97

- IFC facilitates interoperability, which is the ability of different software applications to communicate and
exchange data without loss or distortion of information. IFC enables the integration and coordination of
various aspects of a project, such as design, analysis, simulation, construction, and operation.
- IFC is not a module, an algorithm, or a program, but rather a standardized, digital description of the
built environment.
Q.7 As per urban design principles proposed by Gordon Cullen, Rashtrapati Bhavan, New Delhi, is an
example of
(A) Serial Vision. (B) Pinpointing. (C) Occupied territory. (D) Here and there.
Ans:- (A)
Explanation:
Serial Vision is a visual approach to urban design proposed
by Gordon Cullen, a British architect and urban designer.
He defined Serial Vision as the changing perspectives and
sequential views experienced as one moves through space.
He emphasised the dramatic and theatrical aspects of traversing an urban environment, focusing on the
relationship between the places and things that one might encounter along the way.
Rashtrapati Bhavan, New Delhi, is the official residence of the President of India. It was designed by
Edwin Lutyens, a British architect who collaborated with Cullen on several projects. Rashtrapati Bhavan
is an example of Serial Vision, as it offers a series of visual surprises and contrasts as one approach it
from different directions. For instance, one can see the dome of the building from afar, but it is hidden
by the gatehouse as one gets closer. Then, the dome reappears as one enters the forecourt, and is framed
by the columns and arches of the building. The building also changes its appearance depending on the
time of day and the season, creating a dynamic and varied visual experience.
Q.8 A waste water pipe connecting two inspection chambers (IC) is laid at a slope of 1:200. The Invert
Level of the starting IC is -450 mm. The Invert level of the second pit at a distance of 40 m from the
first IC is
(A) -650mm (B) -200 mm. (C) -250 mm. (D) -550 mm.
Ans:- (A)
Solution:
Invert level of starting IC = -450 mm
Slope given = 1:200 & distance of the second pit = 40 m
Depth would be achieved = 40 × 1/200 = 200 mm
Invert level of the second IC = -450 -200 = -650 mm
Q.9 From the images P,Q and R given below, select the corresponding land use categories according to
Alonso’s Bid Rent theory.
(A) P–Manufacturing; Q–Residential; R–Retail
(B) P–Retail; Q–Residential; R–Manufacturing
(C) P–Residential; Q–Retail; R–Manufacturing
(D) P–Retail; Q–Manufacturing; R–Residential
98 GATE Architecture and Planning: Comprehensive Question Bank

Ans: (D)
Explanation: Please refer GATE 2018, Q.36 for more information.

Q.10 The urban land use model based on the concept of a polycentric city is known as
(A) Burgess Model. (B) Harris and Ullman model.
(C) Hager strand’s Model. (D) Homer Hoyt’s model.
Ans:- (B)
Explanation:
The Harris and Ullman model, also known as the multiple nuclei model, is an urban land use model
based on the concept of a polycentric city. It was proposed by Chauncy Harris and Edward Ullman in
1945, as a critique of the concentric zone model and the sector model.
The Harris and Ullman model suggests that a city does not have a single centre, but rather several centres
or nuclei, each with a different function and influence. These nuclei may include the central business
district, the industrial district, the residential district, the shopping district, the university district, and
so on. The location and growth of these nuclei are determined by factors such as accessibility, land use
compatibility, and agglomeration economies.
The Harris and Ullman model explains the spatial structure and development of many modern cities,
especially in North America, where urban sprawl, decentralization, and suburbanization have created
a polycentric urban form. The model also accounts for the diversity and complexity of urban land use
patterns, as well as the interactions and conflicts between different urban activities.
Q.11 The total head or total lift against which a pump works includes suction lift, discharge lift and
(A) cone of depression. (B) salvage lift.
(C) water horse power. (D) frictional head loss.
Ans:- (D)
Explanation:
The total head or total lift against which a pump works is the total energy per unit weight of the fluid that
the pump delivers or receives. It is the sum of the static head, the velocity head, and the elevation head.
The static head is the pressure head that the pump must overcome or create. It is the difference between
the pressure at the discharge and the suction of the pump.
The velocity head is the kinetic energy per unit weight of the fluid. It is proportional to the square of the
velocity of the fluid.
Hemant Vilas Parulekar 99

The elevation head is the potential energy per unit weight of the fluid. It is the difference between the
elevation at the discharge and the suction of the pump.
The frictional head loss is the energy loss due to the friction between the fluid and the pipe walls, fittings,
valves, and other components in the system. It is proportional to the length and diameter of the pipe, the
roughness of the pipe surface, the viscosity of the fluid, and the square of the velocity of the fluid.
The total head or total lift against which a pump works includes the suction lift, the discharge lift, and
the frictional head loss, as shown in the equation below:
H=hd+hs+hf
where:
H = total head or total lift
hd = discharge lift
hs = suction lift
hf = frictional head loss
Q.12 The two components for measuring time of concentration for storm water are
(A) overland flow time and retention time.
(B) overland flow time and gutter flow time.
(C) detention time and gutter flow time.
(D) retention time and inlet time.
Ans:- (B)
Explanation:
1. Overland Flow Time: This is the time it takes for precipitation to travel over the land surface until
it reaches a defined channel, gutter, or other concentrated flow area. It’s the initial part of the runoff
process and includes the movement of water over grass, pavements, or other surfaces.
2. Gutter Flow Time: Once the water reaches a concentrated flow area like a gutter or a street channel,
gutter flow time is the duration it takes to travel along these channels to the point of interest (like a
storm sewer inlet or the start of the main channel flow).
The sum of these two times gives the total time of concentration, which is a critical factor in hydrologic
design for storm water management. It’s used to determine peak discharge rates for the design of drainage
infrastructure, ensuring it’s adequate to handle the anticipated runoff.
Q.13 The traffic assignment technique where the traffic arranges itself in congested networks such that
the journey time in all used routes between an Origin-Destination pair are equal and less than those
that would be experienced in all unused routes. This is known as
(A) System equilibrium.
(B) All–or–nothing.
(C) User equilibrium.
(D) Incremental.
100 GATE Architecture and Planning: Comprehensive Question Bank

Ans:- (C)
Explanation:
User equilibrium is a traffic assignment technique where the traffic arranges itself in congested networks
such that the journey time in all used routes between an origin-destination pair are equal and less than
those that would be experienced in all unused routes. This means that no user can reduce their travel
time by switching to another route, and that all users experience the same level of congestion.
System equilibrium is a traffic assignment technique where the traffic arranges itself in congested
networks such that the total travel time for all users is minimized. This means that the traffic is distributed
in a way that optimizes the network performance, but not necessarily the individual user satisfaction.
All-or-nothing is a traffic assignment technique where the traffic is allocated to the shortest or least cost
path between each origin-destination pair, regardless of the congestion or capacity on the network. This
means that only one route is used for each pair, even if there are other routes with similar or slightly
higher travel times.
Incremental is a traffic assignment technique where the traffic is assigned in steps, with a fixed proportion
of the total demand assigned in each step. The travel times are updated after each step, based on the
accumulated traffic volumes. This means that the traffic is gradually distributed over the network, and
may approach an equilibrium solution after many steps.
Q.14 What is the dependent variable in a regression-based trip generation model?
(A) Population of Traffic Analysis Zone
(B) Number of trips
(C) Number of employees
(D) Number of households
Ans:- (B)
Explanation:
A regression-based trip generation model is a statistical method that estimates the relationship between
the number of trips and various explanatory variables, such as population, income, land use, and vehicle
ownership.
The dependent variable is the variable that is being predicted or explained by the model. In a trip
generation model, the dependent variable is the number of trips, as it depends on the values of the
explanatory variables.
The explanatory variables are the variables that are used to predict or explain the dependent variable. In
a trip generation model, the explanatory variables can be population of traffic analysis zone, number of
employees, number of households, or any other factors that affect the travel demand.
Q.15 The curve traced by a point on a circle rolling inside another circle is known as
(A) hypocycloid.
(B) helix.
(C) involute.
(D) hyperbola.
Hemant Vilas Parulekar 101

Ans:- (A)
Explanation:
Term Hypocycloid Helix
Image

Term Involute Hyperbola


Image

Q.16 The law of Primate City was first proposed by


(A) Samuel A.Stouffer (B) Colin Clark
(C) Mark Jefferson (D) Harold Hotelling
Ans:- (D)
Explanation:
The law of the primate city was first proposed by the geographer Mark Jefferson in 1939. He defines a
primate city as being “at least twice as large as the next largest city and more than twice as significant.”
The law of the primate city explains the phenomenon of huge cities that capture such a large proportion
of a country’s population and economic activity. These primate cities are often, but not always, the
capital cities of a country. An example of a primate city is Paris, which represents and serves as the focus
of France.
Samuel A. Stouffer was a sociologist who studied human migration and social mobility. Colin Clark was
an economist who studied the relationship between population, income, and economic development.
Harold Hotelling was a mathematician who developed the theory of spatial competition and location.
None of them proposed the law of the primate city.
Q.17 In the European Union which constitutes the cities namely, London, Paris, Brussels, Amsterdam,
Cologne, Frankfurt, Munich and Milanlie within a linear megalopolitan zone known as
(A) Purple Zone. (B) Golden Polygon.
(C) Blue Banana. (D) Yellow Corridor
102 GATE Architecture and Planning: Comprehensive Question Bank

Ans:- (C)
Explanation:
The linear megalopolitan zone that includes the cities
of London, Paris, Brussels, Amsterdam, Cologne,
Frankfurt, Munich and Milan is known as the Blue
Banana. The Blue Banana is a term coined by French
geographer Roger Brunet in 1989 to describe the most
densely populated and economically productive region
of Europe, stretching from north-west England to
northern Italy. The Blue Banana covers about 200,000
square kilometres and has a population of about 110 million people. It accounts for about 40% of the
European Union’s GDP and hosts many of its major financial, industrial, and cultural centres. The Blue
Banana is also a symbol of the uneven development and regional disparities within the European Union,
as it contrasts with the less urbanized and less prosperous areas in the east and south of the continent.
Q.18 An urban governance tool to mobilize financial resources by permitting additional FAR over and
above the prescribed FAR by imposing a charge or fee for the same is known as
(A) Betterment Levy.
(B) Impact Fee.
(C) Land Value Increment Tax.
(D) Floor Area Incentive Tax.
Ans:- As per final answer key, all the options are considered correct and marks to all have been
given.
Explanation:
Betterment Levy: This is a charge imposed on properties that benefit from public infrastructure
improvements, such as new roads or parks. It is not directly related to FAR or development rights.
Impact Fee: This is a charge imposed on developers to offset the negative impacts of their projects on
public infrastructure and services. It can be used to finance the construction of new infrastructure, such
as schools or roads, or to mitigate the impact of projects on existing infrastructure. In the context of
an urban governance tool to mobilize financial resources by permitting additional FAR, an impact fee
would be the most likely option.
Land Value Increment Tax: This is a tax on the increase in the value of land that is not due to the efforts
of the landowner. It is not directly related to FAR or development rights.
Floor Area Incentive Tax: This is a tax on the amount of floor space in a building. It is not used to
mobilize financial resources or to control development rights.
Q.19 Identify the colour palette that is created using any three equally spaced hues around the colour
wheel.
(A) Split–complementary.
(B) Analogous.
(C)Triads.
(D) Complementary.
Hemant Vilas Parulekar 103

Ans:- (C)
Explanation:
A triad is a colour palette that is created using any three equally spaced hues around the colour wheel.
For example, the primary colours red, yellow, and blue form a triad, as they are 120 degrees apart on
the colour wheel. Triads create a balanced and harmonious contrast, as they use colours from different
colour families.
A split-complementary is a colour palette that is created using a base colour and the two colours
that are adjacent to its complement on the colour wheel. For example, the base colour green and its
split-complements magenta and orange form a split-complementary palette. Split-complementary
palettes offer a high contrast and a vibrant look, as they use colours from opposite sides of the colour
wheel.
An analogous is a colour palette that is created using three or more colours that are next to each other
on the colour wheel. For example, the colours yellow, yellow-green, and green form an analogous
palette. Analogous palettes create a smooth and harmonious transition, as they use colours from the
same colour family.
A complementary is a colour palette that is created using two colours that are opposite to each other on
the colour wheel. For example, the colours red and green form a complementary palette. Complementary
palettes create a strong and dynamic contrast, as they use colours from different colour temperatures.

Q.20 Coefficient of Performance (COP) for heat pump is used to calculate


(A) the number of air changes. (B) the Energy Efficiency Ratio.
(C) the Energy Select Sector index. (D) the Indoor Air Quality index.
Ans:- (B)
Explanation:
The coefficient of performance (COP) for heat pump is a measure of how efficiently a heat pump uses
electricity to move heat from one place to another. It is the ratio of the heating or cooling output to the
electrical input. For example, a COP of 3 means that the heat pump produces 3 units of heat or cold for
every unit of electricity it consumes.
The energy efficiency ratio (EER) is a measure of how efficiently an air conditioner or a heat pump
operates in cooling mode. It is the ratio of the cooling output to the electrical input. For example, an
EER of 12 means that the air conditioner or heat pump produces 12 units of cooling for every unit of
electricity it consumes.
104 GATE Architecture and Planning: Comprehensive Question Bank

The energy select sector index (IXE) is a stock market index that tracks the performance of companies
in the energy sector, such as oil and gas producers, refiners, and distributors. It is based on the market
capitalization of the companies and is weighted by their share prices. The indoor air quality index (IAQI)
is a measure of how healthy the air is inside a building or a room. It is based on the concentration of
various pollutants, such as particulate matter, carbon monoxide, nitrogen dioxide, ozone, and volatile
organic compounds. It ranges from 0 to 500, with lower values indicating better air quality.
Q.21 Freight flows are converted to truck flows using
(A) Volume factor. (B) Weight factor. (C) Payload factor. (D) Distance load factor.
Ans:(C)
Explanation:
Rebound hammer test is a non-destructive testing method of concrete that provides a convenient and
rapid indication of the compressive strength of the concrete. The rebound hammer is a spring-controlled
mass that slides on a plunger and hits the surface of the concrete to measure the rebound number. The
rebound number correlates with the compressive strength and can be used to assess the quality of the
concrete.
Permeability of concrete is the measure of the ease with which water can flow through the concrete.
It is influenced by factors such as porosity, pore size, pore structure, and cracking of the concrete.
Permeability of concrete can be measured by methods such as water absorption test, initial surface
absorption test, and permeability apparatus test.
Bond stress between rebar and concrete is the measure of the force that is required to pull out a rebar
from the concrete.

Q.22 Rebound hammer test is used to measure


(A) permeability of concrete (B) bond stress between rebar and concrete.
(C) compressive strength of concrete. (D) tensile strength of concrete.
Ans:- (C)
Explanation: The Rebound Hammer Test is used to measure the (C) compressive strength of concrete.
This non-destructive test involves impacting a concrete surface with a hammer and measuring the
rebound of the hammer to estimate the concrete’s compressive strength.
Q.23 Which type of temporary supporting structure can be used in case of re-building the lower part of
a load bearing wall at ground floor above plinth level
(A)Dead Shore (B) Pit Under pinning (C) Flying Shore (D) Needle Scaffolding
Hemant Vilas Parulekar 105

Ans:- (A)
Explanation:
Needle scaffolding is a type of temporary supporting structure that is used to support a load-bearing
wall while it is being repaired or rebuilt. It consists of horizontal beams, called needles, that are inserted
through holes made in the wall. The needles are supported by vertical posts, called props, that are placed
on either side of the wall. The props are braced by diagonal members, called rakers, that are fixed to the
ground.
Dead shore is a type of temporary supporting structure that is used to support a floor or roof while it
is being repaired or rebuilt. It consists of vertical posts, called shores, that are placed under the floor or
roof beams. The shores are supported by horizontal members, called sole plates, that are fixed to the
ground. The shores are also connected by horizontal members, called needles, that are placed across the
floor or roof beams.
Pit underpinning is a method of strengthening the foundation of a building by excavating pits under
the existing foundation and filling them with concrete or other materials. It is used to increase the
bearing capacity of the soil, to prevent settlement or subsidence, or to lower the water table.
Flying shore is a type of temporary supporting structure that is used to support two parallel walls that
are at risk of collapsing due to the removal of an intermediate wall or building. It consists of horizontal
beams, called flying shores, that are placed between the two walls. The flying shores are supported by
vertical posts, called needles, that are fixed to the walls. The needles are braced by diagonal members,
called rakers, that are fixed to the ground.
Q.24 During earthquake, soft storey failure in a building is due to hues around the colour wheel.
(A) Shear failure initiated by short column effect.
(B) Stress discontinuity initiated by abrupt changes of stiffness.
(C) Failure of column initiated by weak column – strong beam effect.
(D) Drift of building storey initiated by pounding effect.
Answer:- (B)
Explanation:
Refer GATE 2018, Q.34 for more information.
Q.25 Following five activities are associated with construction contract management. Choose the option
showing the correct progressive sequence of the activities.
P Opening of Bid
Q Submission of security Deposit
R Publication of Notice Inviting Tender (NIT)
S Issue of Letter of intent (LOI)
T Submission of Earnest money deposit (EMD)
(A)R-Q-P-T-S
(B)S-P-R-T-Q
(C)R-T-P-S-Q
(D)S-T-P-R-Q
106 GATE Architecture and Planning: Comprehensive Question Bank

Ans:- (C)
Explanation:
Earnest Money Deposit: Earnest money is assurance or guarantee in the form of cash on the part of the
contractor to keep open the offer for consideration and to confirm his intentions to take up the work
accepted in his favour for execution as per terms and conditions in the tender.
– In case a tender fails to commence the work awarded to him, the earnest money is forfeited to
government. No interest is payable upon the earnest money to the contractors.
– EMD of the lowest tenderer whose tender is normally accepted is retained by the department as a part
of department as a part of security deposit for due performance of contract. department as a part of
security deposit for due performance of contract.
Security Deposit:
– The contractor is required to deposit with the owner a sum stated as a percentage of the cost of the
working order to safe guard the interests of the owner in the even to firm proper performance of the
contract.
– The relevant clause should specify the time limit within which the payment has to be made and the
mode of payment should be specified.
– However, in big projects the security is taken in the form of bank guarantees. The bank guarantee
costs the contract or a small percentage,seldom exceeding two percent per annum of the amount
guaranteed with small margin money.
– Accepting the security in terms of the bank guarantee is thus to the advantage of the employer.
Q. 26 – Q. 36 carry two marks each. Multiple Choice Question (MCQ), carry TWO mark each (for
each wrong answer: – 2/3).
Q.26 Match the acronyms in Group- I with the particulars in Group-II
(A) P-3, Q-5,R-4,S-2 Group I Group II
(B) P-4, Q-3, R-1,S-2 (P) LCA 1. building certification system
(C) P-5, Q-4,R-2,S-1 (Q) IPCC 2. hydrological assessment tool
(R) Mtoe 3. climate change
(D) P-5, Q-3, R-4,S-1
(S) LEED 4. equivalent measure of energy ex- pended
5. Cradle to grave
Answer:- (D)
Explanation:
P) Life cycle assessment or LCA (also known as life cycle analysis) is a methodology for assessing
environmental impacts associated with all the stages of the life cycle of a commercial product, process,
or service. For instance, in the case of a manufactured product, environ- mental impacts are assessed
from raw material extraction and processing (cradle), through the prod- uct’s manufacture, distribution
and use, to the recycling or final disposal of the materials composing it (grave).
Hemant Vilas Parulekar 107

Q) The Intergovernmental Panel on Climate Change (IPCC) is an intergovernmental body of the


United Nations that is dedicated to providing the world with objective, scientific information relevant
to understanding the scientific basis of the risk of human-induced climate change, its natural, political,
and economic impacts and risks, and possible response options.
The IPCC was established in 1988 by the World Meteorological Organization (WMO) and the United
Nations Environment Programme (UNEP) and was later endorsed by the United Nations General As-
sembly. Membership is open to all members of the WMO and UN.
R) Millions of tonnes of oil equivalent (Mtoe)is a unit of energy used to describe the energy content of
all fuels, typically on a very large scale. It is equal to 4.1868x1016 Joules, or 41.868 petajoules which is a
tremendous amount of energy to compare to it (1000 Mtoe = 1 Gtoe = 39.68quad).
S) Leadership in Energy and Environmental Design (LEED) is a green building certification program
used worldwide -profit U.S. Green Building Council (USGBC), it includes a set of rating systems for
the design, construction, operation, and maintenance of green buildings, homes, and neighbourhoods,
which aims to help building owners and operators be environmentally responsible and use resources
efficiently. By 2015, there were over 80,000 LEED-certified buildings and over 100,000 LEED-accredited
professionals. Most LEED-certified buildings are located in major U.S. metropolises. LEED Canada has
developed a separate rating system for the regulations and climate of that country.

Q.27 Match the buildings in Group I with their corresponding architect in Group II.
Group I Group II
(P) 1. Renzo piano
108 GATE Architecture and Planning: Comprehensive Question Bank

Group I Group II
(Q) 2. Daniel lilbeskind

(R) 3. David Childs

(S) 4. Frank Owen Gehry

5. Norman Foster
(A) P–4, Q–3,R-1, S–2
(B) P–2, Q–4, R-2, S–5
(C) P–3, Q–5, R–4, S–1
(D) P–2, Q–3, R–4, S–5
Ans:- (D)
Explanation:
Please refer GATE 2017 for similar question.
Building Jewish museum berlin World trade Centre
Architect Daniel lilbeskind David childs
Image
Hemant Vilas Parulekar 109

Building ST.merry Axe London Loius Vuitton Foundation paris


Architect Norman foster Frank owen gehry
Image

Q.28 Match the heritage conservation charters in Group I with their focus areas in Group II.
(A) P-3,Q-1,R-4,S-5 Group I Group II
(B) P-5,Q-4,R-1,S-2 (P) Washington Charter 1. Conservation of historic gardens
(C) P-5,Q-1,R-4,S-2 (Q) Florence Charter 2. Conservation of places of cultural
significance
(D) P-4,Q-1,R-3,S-2
(R) Venice Charter 3. Authenticity
(S) Burra Charter 4.Conservation of restoration of
monuments and sites
5. Conservation of historic towns
Answer:- (C)
Explanation:
The Washington Charter, also known as the Charter for the Conservation of Historic Towns and Urban
Areas, was adopted by ICOMOS in 1987. It provides principles and guidelines for the conservation and
management of historic towns and urban areas, taking into account their cultural, social and economic
values.
The Florence Charter, also known as the Charter for Historic Gardens, was adopted by ICOMOS in
1981. It defines the concept and scope of historic gardens, and sets out the criteria and methods for their
conservation, restoration and enhancement.
The Venice Charter, also known as the International Charter for the Conservation and Restoration of
Monuments and Sites, was adopted by ICOMOS in 1964. It is one of the most influential documents in
the field of heritage conservation, and establishes the basic principles and techniques for the conservation
and restoration of monuments and sites, with a special emphasis on authenticity.
The Burra Charter, also known as the Australia ICOMOS Charter for Places of Cultural Significance,
was first adopted by ICOMOS in 1979 and revised several times since then. It defines the concept and
process of cultural significance, and provides a framework for the conservation and management of
places of cultural significance, based on a participatory and evidence-based approach.
110 GATE Architecture and Planning: Comprehensive Question Bank

Q.29 Match the Buildings (name of architects) in Group I with the abstractions used in Group II.
(A) P–4, Q–2,R–1,S–3 Group I Group II
(B) P–5, Q–1, R–4,S–3
(P) The School of Spastic children, New 1. Cosmos in geometric form
(C) P–2, Q–1,R–3,S–2 Delhi (Romi Khosla)
(D) P–5, Q–2, R–4,S–1 (Q) Jawahar Kala Kendra, Jaipur 2. Panchavati
(Charles Correa)
(R) Capital Complex, Chandigarh 3. Plan form of Hindu Temple
(Le Corbusier)
(S) Oberoi Hotel, Bhubneshwar (Karan 4. Bull’s horn
Grover)
5. Mother’s womb
Ans:- (B)
Explanation:
School The School of Spastic children, Jawahar Kala Kendra, Jaipur (Charles
New Delhi (Romi Khosla) Correa)
Abstraction Mother’s womb Cosmos in geometric form
Explanation The School of Spastic Children, New Jawahar Kala Kendra, Jaipur, designed by
Delhi, designed by Romi Khosla, is Charles Correa, is based on the cosmic
inspired by the concept of the mother’s diagram of the ancient city of Jaipur. The
womb. The building has a circular building has a square plan divided into
plan with a central courtyard, and the nine smaller squares, each representing
classrooms are arranged along the one of the nine planets. The building is
periphery. The building is meant to a cultural center that showcases the arts
provide a nurturing and protective and crafts of Rajasthan.
environment for the children.
School
building

School Capital Complex, Chandigarh Oberoi Hotel, Bhubneshwar


(Le Corbusier) (Karan Grover)
Abstraction Bull’s horn Plan form of Hindu Temple
Explanation Capital Complex, Chandigarh, Oberoi Hotel, Bhubaneswar, designed by
designed by Le Corbusier, is Karan Grover, is derived from the plan
influenced by the shape of a bull’s form of a Hindu temple. The hotel has a
horn. The complex consists of three rectangular plan with a central atrium, and
main buildings: the Secretariat, the the rooms are arranged around the four sides.
High Court, and the Legislative The hotel is a tribute to the rich heritage and
Assembly. The buildings are oriented culture of Orissa.
along a curved axis that resembles
a horn, symbolizing the power and
authority of the government.
Hemant Vilas Parulekar 111

School
building

Q.30 Match the names of the gardens in Group I with their type in Group II.
(A) P-3, Q-1,R-2,S-4 Group I Group II
(B) P-5, Q-1, R-4,S-3 (P) Shalimar Bagh, Srinagar 1. Hanging Garden
(C) P-5, Q-3,R-4,S-2 (Q) Pherozshah Mehta Garden, Mumbai 2. Memorial Garden
(R) Lalbagh Garden, Bangalore 3. Rock Garden
(D) P-5, Q-4, R-1,S-3
(S) Nekchand’s Garden, Chandigarh 4. Botanical Garden
5. Mughal Garden
Ans:- (B)
Garden Shalimar Bagh, Srinagar Pherozshah Mehta Garden, Mumbai
Type Mughal garden Hanging garden
Explanation Shalimar Bagh, Srinagar is a Mughal Pherozshah Mehta Garden, Mumbai is a
Garden, built by Emperor Jahangir Hanging Garden, located on top of Malabar
in the 17th century. It is famous for Hill. It was built in 1881 and named after
its terraced lawns, fountains, and a prominent lawyer and politician. It is
pavilions. known for its animal-shaped hedges and
flower beds.
Garden

Garden Lalbagh Garden, Bangalore Nekchand’s Garden, Chandigarh


Type Botanical garden Rock garden
Explanation Lalbagh Garden, Bangalore is a Nekchand’s Garden, Chandigarh is a Rock
Botanical Garden, established by Garden, created by Nek Chand, a self-
Hyder Ali in the 18th century. It is taught artist, in the 20th century. It is made
home to over 1,800 species of plants, of waste materials, such as broken tiles,
including some rare and exotic ones. It glass, pottery, and bangles. It has sculptures
also has a glasshouse, a lake, and a rock of animals, humans, and fantasy creatures.
formation.
Garden
112 GATE Architecture and Planning: Comprehensive Question Bank

Q.31 Match the various types of impurities present in water in Group I with the appropriate water
treatment process given in Group II.
(A) P-2,Q-5,R-3,S-4 Group I Group II
(B) P-3,Q-4,R-1,S-2 (P) Fine suspended matter 1. Aeration
(Q) Pathogenic bacteria 2. Plain sedimentation
(C) P-1,Q-4,R-3,S-2 (R) Color, odor and taste 3.Sedimentation with coagulation
(S) Floating matter as leaves 4. Screening
(D) P-3,Q-5,R-1,S-4 5. Disinfection
Ans:- (D)
Explanation:
The similar question was asked in GATE-2017
P) Fine suspended matter: This is best removed with Sedimentation with coagulation (3), where
chemicals are added to clump the particles together, making them settle faster.
Q) Pathogenic bacteria: These require Disinfection (5) to kill them and make the water safe for drinking.
R) Color, odor and taste: These are often addressed by Aeration (1), which helps remove dissolved gases
and volatile organic compounds responsible for these issues.
S) Floating matter as leaves: This is easily removed by Screening (4), which filters out larger debris from
the water.
Q.32 Match the Temples in Group I with their architecture style given in Group II.
(A) P-3,Q-1,R-2,S-5 Group I Group II
(B) P-3,Q-4,R-2,S-1 (P) Badami Cave Temples 1. Pandya style
(C) P-2,Q-1,R-3,S-5 (Q) Kalugumalai Temple Complex 2. Chola style
(R) Airavatesvara Temple 3. chalukya style
(D) P-5,Q-1,R-4,S-2
(S)Chennakeshava Temple 4. Vijayanagara style
5. Hoysala style
Ans:- (A) Explanation:
Temple
Badami Cave Temples Kalugumalai Temple Complex
location
Explanation The Badami Cave Temples are a complex Kalugumalai Temple Complex is a group
of Hindu and Jain cave temples located in of ancient monuments in Tamil Nadu,
Badami, a town in the Bagalkot district India. It includes the Kalugasalamoorthy
in northern part of Karnataka, India. The Temple, a rock-cut shrine dedicated to
caves are important examples of Indian Lord Murugan, the Vettuvan carved out
rock-cut architecture, especially Badami of a single rock, and the Kalugumalai
Chalukya architecture, which flourished Jain Beds, a series of relief sculptures of
between the 6th and 8th centuries CE. Jain Tirthankaras. The Vettuvan Koil
The Badami Chalukya architecture is is a remarkable example of the Pandya
a style that blended the Nagara and style of architecture, which flourished
Dravidian styles, and includes two types between Koil, a monolithic temple the
of monuments: rock cut halls or “cave 6th and 14th centuries CE. The Pandya
temples”, and “structural” temples, built style is characterized by the vimana,
above ground. The cave temples have gopuram and mandapa, which are the
three basic features: pillared veranda, tower, gateway and hall of the temples.
columned hall and a sanctum cut out
deep into rock.
Hemant Vilas Parulekar 113

Style Chalukya style Pandya style


Temple

Temple Airavatesvara Temple Chennakeshava Temple


location
Explanation The Airavatesvara Temple is a Hindu The Chennakeshava Temple is a 12th-
temple dedicated to Shiva, located in century Hindu temple dedicated to Vishnu,
Darasuram, Tamil Nadu. It was built by located in Belur, Karnataka. It is one of the
Rajaraja Chola II in the 12th century Great Living Chola Temples, a UNESCO
CE, and is one of the Great Living World Heritage Site. The temple is an
Chola Temples, a UNESCO World example of the Hoysala style of architecture,
Heritage Site. The temple is an example which developed under the rule of the
of the Chola style of architecture, which Hoysala Empire between the 11th and 14th
blended the Nagara and Dravidian centuries. The Hoysala style blended the
styles, and included rock-cut and Nagara and Dravidian styles, and included
structural temples. rock-cut and structural temples.

Style Chola style Hoysala style


Temple

Q.33 Match the urban form/structure in Group I with their respective proponents in Group II.
P-4, Q-1, R-5, S-3 Group I Group II
(B) P-5, Q-1, R-4, S-2 (P) Trabantenstadte 1. Arturo Soria Y Mata
(C) P-3, Q-1, R-5, S-2 (Q) Linear city 2.Le Corbusier
(R) Bloombury Precinct 3. Ernst May
(D) P-3, Q-4, R-1, S-2
(S)Radiant city 4. Frank Lloyd Wright
5. Patrick Abercrombie
114 GATE Architecture and Planning: Comprehensive Question Bank

Ans:- (C)
Explanation:
Urban form Trabantenstadte (satellite town) Linear city
Proponent Ernst May Arturo Soria Y Mata
Explanation Trabantenstädte (satellite towns) is a A linear city is a theoretical urban
German urban planning concept from planning concept where urban functions
the 1960s and 70s. It involved building are concentrated along a long, narrow
large, self-contained satellite towns corridor instead of being spread out in a
around existing cities to accommodate traditional city layout. This can include
population growth and alleviate housing, transportation, commerce, and
overcrowding. These towns typically other amenities. Proponents of linear cities
featured standardized housing, green argue that they can be more efficient and
spaces, and amenities like schools and sustainable than traditional cities.
shops.
Proposed
urban form

Urban form Bloombury Precinct Radiant city


Proponent Patrick Abercrombie Le Corbusier
Explanation The Bloomsbury Precinct in London is Radiant City (Ville Radieuse) was a
a historic district known for its squares, utopian urban planning concept developed
gardens, and educational institutions. It by French architect Le Corbusier in the
was developed in the 19th century on 1920s and 30s. It envisioned a city built on
the land of the Duke of Bedford, who a grid plan with tall, slab-like buildings set
followed a master plan that emphasized in park-like surroundings. Radiant City
open space and architectural coherence. emphasized sunlight, air, and green spaces,
The Bloomsbury Precinct is considered and aimed to create a more efficient and
an example of successful urban planning healthy living environment than traditional
and is admired for its livability and cities.
sense of community.
Proposed
urban form
Hemant Vilas Parulekar 115

Q.34 Match the elements in Group I to their description in Group II.


Group I Group II (A) P-3, Q-1, R-5, S-4
(P) 1. Cornice (B) P-4, Q-3, R-1, S-2
(Q) 2. Stylobate (C) P-5, Q-4, R-2, S-1
(R) 3. Stereobate
(D) P-5 Q-1, R-2, S-4
(S) 4. Abacus
5. Frieze
Ans:- (D)
Explanation:
Elements Cornice Stylobate
Explanation A horizontal molding projecting The continuous base or platform on which
outward from the top of a wall, a row of columns rests.
often decorated with dentils or other
ornaments.
Image

Elements Stereobate Abacus


Explanation Another term for the stylobate, The flat, square slab forming the topmost
particularly in classical architecture. member of a capital, supporting the
architrave.
Image

Elements Frieze
Explanation A horizontal band of decoration, often
sculpted, located above the architrave
and below the cornice.
Image
116 GATE Architecture and Planning: Comprehensive Question Bank

Q.35 Match the position of feet in Group I to the most appropriate description of stability of human body
in Group II
Group I Group II
(P) 1. stable atero-posterioty

(Q) 2. Laterally stable

(R) 3. Fairly stable in all directions

(S) 4. Vertically stable

5. Unstable
(A) P-5, Q-5, R-2, S-1 (B) P-5, Q-3, R-1, S-2
(C) P-1, Q-3, R-4, S-2 (D) P-4, Q-3, R-2, S-1
Ans:- (B)
Q.36 Match the Buildings in Group I with their corresponding structural systems in Group II
(A) P-5, Q-3, R-4, S-1 Group I Group II
(B) P-3, Q-5, R-1, S-2 (P) Empire state building, New York USA 1. Trusses Tube
(C) P-5, Q-4, R-1, S-2 (Q) John Hancock center, Chicago,USA 2. Bundled Tube
(R) Taipai 101, Taiwan 3. Tube in Tube
(D) P-5, Q-1, R-4, S-2
(S)Sears Tower Chicago,USA 4. Outrigger frame
5. Shear Truss
Hemant Vilas Parulekar 117

Ans:- (D)
Explanation:
Building Empire state building, New York USA John Hancock center, Chicago,USA
Explanation Empire State Building, New York USA: John Hancock Center, Chicago, USA: The
The lead architect was William F. Lamb chief designer was Bruce Graham and the
of Shreve, Lamb and Harmon.The structural engineer was Fazlur Rahman
Empire State Building uses a shear truss Khan of Skidmore, Owings and Merrill
system, which consists of a steel frame (SOM). The John Hancock Center uses a
with diagonal bracing to resist lateral trussed tube system, which is a variation of
forces such as wind and earthquakes. the tube system that incorporates external
diagonal bracing to increase the stiffness
and reduce the sway of the building.
Structural Shear truss Truss Tube
system
Building

Building Taipei 101, Taiwan Sears Tower Chicago,USA


Explanation Taipei 101, Taiwan: The architect was Sears Tower (now Willis Tower), Chicago,
C.Y. Lee of C.Y. Lee and Partners. The USA: The architect was Bruce Graham
Taipei 101 uses an outrigger frame and the structural engineer was Fazlur
system, which is a hybrid system that Rahman Khan of Skidmore, Owings and
combines a central core with perimeter Merrill (SOM).
columns and horizontal beams that The Sears Tower (now Willis Tower) uses
connect them at regular intervals. The a bundled tube system, which is a system
outrigger beams help to transfer the that consists of several interconnected
loads from the core to the columns and tubes of different heights that act as a
increase the stability of the building. single structural unit. The bundled tube
system allows for greater flexibility in the
design and shape of the building.
118 GATE Architecture and Planning: Comprehensive Question Bank

Structural Out trigger frame Bundeld Tube


system
Building

Q.37-Q.43 Multiple select Question (MSQ), carry TWO mark each (no negative marks).
Q.37 Choose the correct option with respect to cycle track design as per Indian Road Congress guidelines.
(A) The minimum width of cycle track is 3m if overthinking is to be provided for
(B) Cycle tracks may be provided when peak hour cycle traffic is 400 or more on routes with a traffic of
100 to 200 vehicles/hour
(C)Maximum gradient allowed for cycle tracks 1 in 5
(D) Cyclists should have a clear view of at least 80m
Ans:(A,B)
Explanation:
Cycle Track Design as per Indian Road Congress Guidelines
The Indian Road Congress (IRC) has laid out comprehensive guidelines for cycle track design in its
publication IRC:11-2015 Recommended Practice for the Design and Layout of Cycle Tracks. These
guidelines aim to create safe, convenient, and accessible cycling infrastructure for Indian cities and
towns. Here’s a summary of some key aspects:
Planning and Justification: Cycle tracks should be considered on all roads carrying significant cycling
traffic, especially along commuter routes, near schools, and public amenities. The minimum peak hour
cycle traffic volume justifying a cycle track is 400 cyclists on roads with 100-200 vehicles per hour.
Geometric Design:
Width:
Single lane: 1.0 meter minimum, 1.25 meter preferred.
Two lanes: 2.0 meters minimum, 2.5 meters preferred.
Additional lanes may be needed for high-volume routes.
Lateral clearance:
Minimum 0.5 meters from parked vehicles or other obstructions.
1.0 meter preferred for better safety and comfort.
Hemant Vilas Parulekar 119

Vertical clearance:
Minimum 2.25 meters for all users, including cyclists standing upright.
Gradients:
Maximum 1:20 for urban roads.
1:12 preferred for comfortable cycling, especially uphill.
Horizontal curves:
Large radii preferred for improved sightlines and cyclist stability.
Minimum radius of 5 meters for single-lane tracks.
Segregation and Protection:
Physical separation from motorized traffic is essential for safety. This can be achieved using curbs,
bollards, planters, or other barriers.
Grade transitions between cycle tracks and carriageways should be gradual and well-marked.
Surface and Drainage: Smooth, skid-resistant surfacing with good drainage is crucial for safe and
comfortable cycling. A sphalt or concrete are commonly used materials.Proper drainage channels
should be provided to prevent puddles and ensure year-round usability.
Signage and Markings: Clear and consistent signage and markings are essential for guiding cyclists
and informing other road users. Standard symbols and colors should be used as per IRC guidelines.
Reflective markings are recommended for improved visibility at night.
Additional Considerations:
Integration with public transport hubs and parking facilities.
Provision of rest areas and amenities at regular intervals.
Regular maintenance and upkeep of cycle tracks.
Benefits of Following IRC Guidelines:
Improved safety for cyclists: Reduced risk of accidents and injuries.
Increased cycling ridership: Encourages more people to choose cycling for commuting and leisure.
Reduced traffic congestion and pollution: Promotes sustainable transportation practices.
Improved public health and well-being: Encourages physical activity and reduces carbon footprint.
By following the IRC guidelines, Indian cities can create a safe, efficient, and healthy cycling environment
for all residents.
Q.38 As per the right to fair compensation and transparency in land acquisition, rehabilitation and
resettlement Act, 2013, for which purposes can the urgency clause for land acquisition be involved?
(A) National defence and security process
(B) Affordable housing program
(C) Industrial projects
(D)Emergency arising out of natural calamities
120 GATE Architecture and Planning: Comprehensive Question Bank

Ans: (A,D)
Explanation:
As per the Right to Fair Compensation and Transparency in Land Acquisition, Rehabilitation and
Resettlement Act, 2013 (LARR Act), the urgency clause for land acquisition can be invoked for the
following purposes:
A) National defence and security: This includes situations where land is needed for projects related to
national security, border infrastructure development, or strategic installations.
However, the LARR Act does not allow the urgency clause to be invoked for:
B) Affordable housing program: While the government can prioritize land acquisition for affordable
housing projects under Section 17(4) of the LARR Act, it cannot bypass the procedural safeguards and
fair compensation requirements.
D) Emergency arising out of natural calamities: This includes situations where land is needed for
relief and rehabilitation purposes following natural disasters like earthquakes, floods, or landslides.
C) Industrial projects: The urgency clause is not applicable for industrial projects. Land acquisition for
such projects must follow the standard procedure outlined in the LARR Act.
Therefore, the correct answer is (A) and (D) only.
It’s important to note that invoking the urgency clause grants the government the power to take
possession of land before completing the compensation and rehabilitation process. However, this power
should be used sparingly and only in genuine cases of emergency or national security concerns. The
LARR Act emphasizes the importance of ensuring fair compensation and livelihood rehabilitation for
affected landowners, even in cases where the urgency clause is invoked.
Q.39 Which of the following international treaties are related to Climate change and global warming?
(A) Cartagena protocol,2000 (B) Copenhagen summit, 2001
(C) Nagoya protocol, 2010 (D) Paris Agreemen, 2016
Ans:- (B,D)
Explanation:
Paris Agreement, 2016: This is a landmark agreement that aims to limit global warming to well below
2 degrees Celsius, preferably to 1.5 degrees Celsius, compared to pre-industrial levels. It also aims to
increase the ability of countries to deal with the impacts of climate change.
Cartagena Protocol, 2000: This protocol is an international treaty that aims to ensure the safe handling,
use, transboundary movement and disposal of living modified organisms (LMOs) resulting from
modern biotechnology.
Nagoya Protocol on Access and Benefit-Sharing (2010):
Focus: Establishing a fair and equitable framework for sharing the benefits arising from the utilization
of genetic resources.
Relevance to climate change: The Nagoya Protocol indirectly contributes to climate change mitigation
by promoting the conservation and sustainable use of biodiversity. Healthy ecosystems play a vital role in
sequestering carbon and mitigating the effects of climate change. The protocol encourages responsible
access to genetic resources while ensuring benefits are shared with local communities who conserve
them, potentially leading to better ecosystem stewardship.
Hemant Vilas Parulekar 121

Copenhagen Summit (2009):


Focus: Negotiating a global agreement to reduce greenhouse gas emissions and combat climate change.
Relevance to climate change: Though the Copenhagen Summit didn’t result in a binding agreement,
it marked a significant step in international climate action. It highlighted the urgency of addressing
climate change and paved the way for the Paris Agreement in 2016. The summit also established the
Copenhagen Green Climate Fund, which helps developing countries adapt to and mitigate the impacts
of climate change.
Q.40 Which of the following algorithms are used for finding the shortest path in an urban transportation
network?
(A) Logit (B) Huff (C) Floyd Warshall (D) Dijkstra
Ans:(C,D)
Explanation:
1. Dijkstra’s Algorithm:
This algorithm shines in finding the shortest paths in complex networks like urban transportation
systems. Imagine a map of your city with streets, highways, and one-way roads. Dijkstra’s algorithm
can navigate this labyrinth, considering different travel times and distances, to reveal the quickest route
from your starting point to your destination.

Here’s how it works:


Preparation: Each road gets assigned a “weight” based on its distance or travel time. The starting point
gets a weight of 0, while all other points have an initial weight of infinity.
Step-by-step exploration: The algorithm explores one road at a time, prioritizing those with the lowest
accumulated weight from the starting point. If a shorter path to another point is discovered through this
explored road, its weight gets updated accordingly.
Reaching the destination: This process continues until the destination is reached, revealing the shortest
path with the minimum total weight (travel time or distance).
2. Floyd-Warshall Algorithm:
This powerful algorithm tackles a different challenge: finding the
shortest paths between all pairs of nodes in a network. While not
directly focused on a single origin-destination pair, it can still be
useful for urban transportation, especially with smaller and denser
networks. Imagine planning a network of public transportation routes.
Floyd-Warshall can analyze all possible connections between stations,
providing valuable insights for optimizing transit flow and minimizing
travel times.
122 GATE Architecture and Planning: Comprehensive Question Bank

It operates by calculating the shortest distances between all pairs of points iteratively, building upon
known distances to find even shorter ones. While more computationally expensive than Dijkstra’s
algorithm, Floyd-Warshall offers the advantage of pre-computing all routes, potentially saving time in
situations where multiple destinations need to be considered.
3. Logit Model:
This statistical model, primarily used in transportation and marketing, predicts the probability of an
individual choosing one option over another. In the context of urban transportation, it can help estimate
the likelihood of people selecting specific routes based on factors like travel time, cost, and convenience.
It’s not directly used for finding the shortest path but can be informative for transportation planning and
understanding travel behaviour.

Imagine analyzing how a change in bus fares might affect passenger routes. The Logit model can estimate
how likely commuters are to switch to different buses based on the new fares, providing valuable insights
for optimizing public transportation systems.
4. Huff Model:
Similar to the Logit model, the Huff model focuses on predicting spatial interaction patterns and trip
distribution. It estimates the probability of a person visiting a specific destination based on its distance,
attractiveness (amenities, services), and competing alternatives.

Q.41 Which of the following statements are true with respect to surface paint?
(A) Paint is glossy when pigment volume concentration is high
(B) Vehicle is the volatile part of the paint
(C) Base of the paint is usually oxides of metals
(D) High VOC content is preferred in paints
Ans:- (B,C)
Explanation:
(A) Paint is glossy when pigment volume concentration is high: This statement is false. Generally, a
higher pigment volume concentration (PVC) leads to a less glossy finish. This is because pigments tend
to scatter light instead of reflecting it directly, resulting in a matte or satin appearance. High gloss paints
typically have a lower PVC and rely on the resin in the vehicle for reflectivity.
Hemant Vilas Parulekar 123

(B) Vehicle is the volatile part of the paint: This statement is true. The vehicle in paint is a combination
of resins and solvents. The resins form the film-forming binder after the paint dries, while the solvents
evaporate during drying, making them the volatile part.
(C) Base of the paint is usually oxides of metals: This statement is true. While some pigments in paint
are indeed metal oxides (e.g., titanium dioxide for white, iron oxide for reds), not all are. Many organic
and inorganic pigments exist, including carbon black, phthalocyanines, and quinacridones.
(D) High VOC content is preferred in paints: This statement is false. Volatile organic compounds (VOCs)
in paints contribute to air pollution and potential health risks. Modern paints tend to have lower VOC
content, and regulations often promote the use of low-VOC options. High VOC content is generally not
preferred due to these drawbacks.
Some basic terminologies related to paint and mixtures:
Paint:
Binder: The film-forming component of paint, typically a resin that binds the pigments and fills in gaps,
providing a protective layer.
Pigment: Finely ground particles that give paint its color and opacity. Different pigments have different
properties like lightfastness and hiding power.
Filler: Inert material added to adjust the texture, consistency, and workability of paint. It can also reduce
the cost and increase spreading rate.
Solvent: Liquid used to dissolve the binder and other ingredients, allowing the paint to be applied
smoothly and evenly. Solvents evaporate during drying.
Vehicle: The combination of binder and solvent in paint. It determines the application properties like
viscosity, drying time, and finish.
Gloss: The reflective quality of the paint’s surface. High gloss paints reflect light directly, while matte
paints scatter it, creating a less shiny appearance.
Adhesion: The ability of the paint to stick to the surface it’s applied to. Proper surface preparation is
crucial for good adhesion.
Mixture terms:
Homogeneous mixture: A uniform mixture where the components are evenly distributed throughout,
resulting in a single phase. Examples include solutions and alloys.
Heterogeneous mixture: A non-uniform mixture where the components are distinct and identifiable,
forming multiple phases. Examples include suspensions and colloids.
Solution: A homogeneous mixture where one component (solute) dissolves completely in another
component (solvent). The solvent is usually a liquid, and the resulting solution remains a liquid.
Suspension: A heterogeneous mixture where solid particles are dispersed throughout a liquid but don’t
dissolve. The particles are relatively large and eventually settle over time.
Colloid: A heterogeneous mixture where very small particles (smaller than those in suspensions) are
dispersed throughout a liquid or gas. Colloids appear homogeneous but scatter light, exhibiting the
Tyndall effect.
124 GATE Architecture and Planning: Comprehensive Question Bank

Concentration: The amount of a substance present in a mixture, often expressed as a percentage, ratio,
or molarity. Higher concentration signifies more of the substance in the mixture. Pigments: These tiny
troopers are the stars of the show, giving paint its vibrancy and character. They come in various forms,
with some common types being:
Inorganic pigments: Think iron oxides for your earthy reds and yellows, titanium dioxide for dazzling
white, and carbon black for that timeless, dramatic black. These guys are often derived from minerals
and boast excellent durability and lightfastness.
Organic pigments: These synthetic wonders offer an incredible spectrum of colors, from the sunshine
yellow of benzimidazoles to the fiery red of quinacridones. They can be particularly vibrant and versatile,
but some may have lower lightfastness than their inorganic counterparts.
Binders: The unsung heroes of the paint world, binders are the glue that holds everything together. They
create a film that adheres to the surface and keeps the pigments locked in, ensuring your masterpiece
stays put. Common binders include:
Acrylic resins: These versatile workhorses are popular for their durability, water resistance, and ease of
use. They’re a favourite choice for both indoor and outdoor projects.
Alkyd resins: Offering excellent flow and self-levelling properties, alkyd resins are often used in oil-
based paints. They provide a smooth finish and good adhesion, but require proper ventilation during
application due to solvent content.
Epoxy resins: When strength and chemical resistance are paramount, epoxy resins step in. They form
tough, hard films ideal for industrial coatings and marine applications.
Resins: Think of resins as the chefs in the paint kitchen, blending and balancing the ingredients to create
the perfect consistency and texture. They influence factors like brush feel, drying time, and the final
sheen of the paint. Here are some key players:
Acrylic resins: In conjunction with binders, they contribute to the smooth application and flexible film
formation of acrylic paints.
Alkyd resins: These also play a dual role, acting as both binder and resin in oil-based paints. They add
body and gloss to the paint, contributing to a rich, traditional finish.
Silicone resins: High-heat resistance and exceptional water repellency are the specialties of silicone
resins. They’re found in paints designed for harsh environments, like engine compartments or pool
decks.
Remember, the interplay between pigments, binders, and resins is crucial for creating the perfect paint
for your needs. Choosing the right combination depends on factors like the desired color, surface type,
durability requirements, and application technique.
Q.42 As per the solid waste management rules 2016, which among of the following are “duties of waste
generators”?
(A) segregate and store waste generated in four separate streams namely, combustible, non-combustible,
organic and domestic hazards waste
(B) store construction and demolition waste separately within own premise before disposal
(C)All waste generator shall pay user fee for solid waste management
(D)Compost horticulture waste and garden waste separately within own premise
Hemant Vilas Parulekar 125

Ans:- (B,C)
Explanation:
According to the Solid Waste Management Rules 2016:
(A) Segregate and store waste generated in four separate streams: This is a crucial aspect of waste
management, as it ensures that different types of waste can be handled appropriately. The four streams
are:
Combustible waste: Includes paper, cloth, wood, and other materials that can be burned safely.
Non-combustible waste: Includes glass, metal, plastic, and other materials that cannot be burned or
composted.
Organic waste: Includes kitchen waste, garden waste, and other biodegradable materials suitable for
composting.
Domestic hazardous waste: Includes items like batteries, expired medicines, paint thinners, and other
potentially hazardous substances that require special handling and disposal.
(B) Store construction and demolition waste separately: This waste is bulky and often contains materials
that can’t be mixed with other waste streams. Separate storage prevents contamination and facilitates
proper disposal or recycling.
(C) Pay user fee for solid waste management: This fee contributes to the costs associated with collecting,
transporting, processing, and disposing of waste in an environmentally sound manner. It’s a way for
waste generators to share the responsibility of managing the waste they produce.
(D) Compost horticulture waste and garden waste: This helps reduce the amount of waste sent to landfills
and produces valuable compost that can be used to enrich soil and promote plant growth.
Additionally, the Solid Waste Management Rules 2016 assign other responsibilities to waste generators,
including:
• Handing over segregated waste to authorized waste collectors or agencies.
• Not littering or throwing waste in public spaces.
• Adhering to waste minimization practices.
• Cooperating with local authorities in waste management initiatives.
Q.43 Choose the correct options with regard to activated sludge process.
(A) The activated sludge process is an aerobic process
(B) The entire settled sludge is sent back to the aeration tank
(C)The entire effluent from the final setting tank is sent back to the aeration tank
(D)In aeration tanks, sewage is aerated and agitated for few hours.
Ans:- (A,D)
Explanation:
Comprehensive explanation of the activated sludge process, a widely used method for treating
wastewater:
126 GATE Architecture and Planning: Comprehensive Question Bank

Key Steps:
Preliminary Treatment:
Screening: Removes large objects like rags, plastics, and debris.
Grit removal: Separates sand, gravel, and other heavy particles to prevent damage to equipment.
Primary Sedimentation:
Wastewater flows into a primary settling tank where heavy solids settle to the bottom as primary sludge.
Sludge is removed, and partially clarified wastewater (primary effluent) moves to the aeration tank.
Aeration Tank:
Heart of the activated sludge process. Primary effluent mixes with activated sludge, a concentrated
mixture of microorganisms (bacteria, protozoa, fungi).
Air diffusers at the bottom of the tank continuously supply oxygen, creating an aerobic environment.
Microorganisms consume organic matter in the wastewater as food, breaking it down into simpler, less
harmful substances.
Agitation keeps the mixture suspended and promotes contact between microorganisms and wastewater.
Secondary Sedimentation:
Mixed liquor from the aeration tank flows into a secondary settling tank (final clarifier).
Activated sludge settles to the bottom, forming a dense layer of biomass.
Cleared, treated wastewater (secondary effluent) rises to the surface and is discharged or undergoes
further treatment.
Sludge Handling:
Settled sludge is separated into two streams:
Returned activated sludge (RAS): Major portion (50-90%) is pumped back to the aeration tank,
maintaining a high concentration of microorganisms for efficient treatment.
Waste activated sludge (WAS): Excess sludge is removed for further processing (thickening, digestion,
dewatering, disposal).
Advantages:
Effective removal of organic matter (BOD, COD) and suspended solids.
Relatively simple and adaptable to various wastewater types.
Can be modified for nutrient removal (nitrification-denitrification).
Produces less sludge than some other secondary treatment processes.
Q.44-Q.55 Numerical Answer Type (NAT), Carry Two marks each (no negative marks).
Q.44 A rectangular hall having dimension of 0.8m×14.0×4.0m has total 4windows (1.5m×1.0m each) and
2 doors (1.0m×2.0m each).
The coefficients of absorption are given below. Considering all windows open and doors closed, the
reverbation time in seconds is _______. [roundoff to 2 decimal places].
Description of item Absorption coefficient
Coefficient of absorption of wall, floor and 0.2
ceiling
Coefficient of absorption of door and window 0.4
Hemant Vilas Parulekar 127

Ans:- (As per final answer key 0.82 to 0.86)


Solution:
Total volume of hall= 8×14×4=448 m3 (V)
Total area of 4 walls+ ceiling+floor=2(14×4) + 2(8×4)+2(14×8)=112+64+224=400 sq.m
Total area of 4 windows= 4×1.5×1= 6sq.m
Total area of 2 doors=2×1×2=4 sq.m
Net area of 4 walls+ ceiling+floor (excluding 4 windows and 2 doors)= 400-10=390 sq.m
Components Wall, floor and ceilling Door and window
Area (An) 390 10
Coefficient of absorption (an) 0.2 0.4
An x an 78 4
Thus, total room absorption in sabins (A)=78+4=82m2 for all doors and windows closed.
Reverberation time is seconds R.T=(0.16×V)/A
=(0.16×448)/82=0.87 sec for all windows and doors closed.
Now, total room absorption in sabins (A) When all windows are open and doors are closed needs to be
find out.
As, per condition, if windows are open then coefficient of absorption (an)=1
Thus total room absorption in sabins A can be calculated as follow
Components Wall, floor and Door Window
ceilling
Area (An) 390 4 6
Coefficient of absorption (an) 0.2 0.4 1
An x an 78 1.6 6
Thus, total room absorption in sabins (A)=78+1.6+6=85.6m2 for all doors closed and windows open.
Reverberation time is seconds R.T=(0.16×V)/A
=(0.16×448)/85.6=0.837 sec for all windows open and doors closed.
Q.45 If surface conductance of external surface is 20 W/m2 oC, absorbance of the surface is 066 and U
value of the wall is 1.2 W/m2 oC, the solar gain factor of a wall is _____. [roundoff to 2 decimal
places].
Ans:- 0.03 to 0.05
Solution:
The solar gain factor of a wall is a measure of how much solar energy is transmitted through the wall
into the building. It depends on the properties of the wall, such as its U value, surface conductance, and
absorbance, as well as the incident solar radiation.
To calculate the solar gain factor, you need to use the following formula,
Solar gain factor=(a×U)/C
As per given information surface conduction C of external surface=20 W/m2 oC
Absorbance of the surface (a)=0.66
Thermal transmittance (U-value) of wall=1.2 W/m2 oC
Thus, solar gain factor=(0.66×1.2)/20=0.0396=0.04
128 GATE Architecture and Planning: Comprehensive Question Bank

Q.46 The initial cost of property is INR 4,00,000 and its future life is 30 years. Considering the scrap value
as 10% of its initial cost and rate of interest as 5%, the sinking fund (deposited at the end of year) for
the property is INR ______.[roundoff to 2 decimal places].
Ans:- 5405 to 5422
Solution:
Initial cost of property= INR 4,00,000
Future life of property n =30 years
Interest rate i=5%
Scrap value =10% of initial cost=(10/100)×400000=Rs 40,000
Sinking fund= initial cost-Scrap value=400000-40000=Rs 360000
Sinking fund formula= l=S×i/(1+i)n-1
Where l= annual installment, S=initial value/sinking und,n= number of years (life of property)
i=rate of interest
here S=360000,n=30,i=5%=0.05
thus, l=(3,60,000×0.005)/(1+0.05)30-1=18000/3.32=5421.68

Q.47 Reading in the staff stationed at P measured by a dumpy level is 3.5 m. The dumpy level is
stationed at Q. The Reference Level (RL) at point P is 96.5 m and the height of the dumpy level is
1.25 m. The RL at point Q is _______ m. [round off to 2 decimal places]

Ans:- 98.7
Solution:
RL at point Q=(RL at point P+3.5m)-height of dumpy level
=(96.5+3.5)-1.25
100-1.25=98.75
Q.48 A circular cricket field of 180 m diameter is illuminated by four flood light towers. The Flood light
towers are equally spaced along the perimeter of the field. The height of the floodlight tower is 48
m. Using ‘Inverse Square Law’, the illumination level at the center of the field is found as 750 Lux.
Each tower is consisting of 50 lamps. The rating of each lamp is 700 Watt. The efficacy of each lamp
is _______ Lumen /Watt. [round off to 2 decimal places]
Hemant Vilas Parulekar 129

Ans:- 117 to 199 or 1483 to 1496


Solution:
To find out illumination produced from light source not perpendicular to surface, lanberts cosine law,
E=I×cosθ/d2
E=illumination on surface lux E=750
I=illumination intensity from source cd
Also l=luminous efficacy of lamp (lm/watt) × power consumption of lamp watt
As per information, there are 4 flood light towers, each having 50 lamps each having range of 700 watts
thus, here, I=4× luminous efficacy of lamp ×50×700…..(2)
D= distance from light source
As per diagram
tanθ=opp/adj=90/48=1.875
Thus, θ=tan-11.875=61.92
Now, cosθ= cos 61.92=0.47=48/d
Thus, d=102.12m…….(3)
Putting the values from (2) and (3) in (1)
E=I×cosθ/d2
Thus, luminous efficacy of lamp=750×102.122/ 4×50×700×0.47=118.86 Lumen/Watt
Q.49 A building is constructed on a plot measuring 70 m × 40 m. The utilized FAR of the building is 1.5.
An energy audit team found that the average monthly electricity bill of the building is INR 2,94,000.
The unit cost of the electricity is INR 7. The Building Energy Index is __________ kW-hr/m2/year.
[in integer]
Ans:- 120
Solution:
The Building Energy Index (BEI) is a benchmarking tool that indicates the intensity of energy used per
meter square area of the building1 It can be calculated by dividing the annual energy consumption of
the building by the net floor area of the building.
To find the BEI of the given building, we need to first calculate the annual energy consumption and the
net floor area.
The annual energy consumption is the product of the average monthly electricity bill and months,
divided by the unit cost of the electricity. This gives:
Annual energy consumption= 294000×12/7 kWh=504000kWh
he net floor area is the product of the plot area and the utilized FAR. This gives:
Net floor area=70×40×1.5 m2=4200 m2
Therefore, the BEI is the quotient of the annual energy consumption and the net floor area. This gives:
BEI= 504000/4200kWh/M2/year= 120 kWh/m2/year
130 GATE Architecture and Planning: Comprehensive Question Bank

Q.50 A simply-supported steel beam made of an I-section has a span of 8 m. The beam is carrying a
uniformly distributed load of 15 kN/m. The overall depth of the beam is 450 mm. The moment
of inertia of the beam section is 18000 cm4. The maximum bending stress in the beam will be
_________ N/mm2. [in integer]
Ans:-150
Solution:
To find the maximum bending stress in the beam, we need to use the bending stress formula:
sigma=fracMtimescI
where σ is the bending stress, M is the bending moment, c is the distance from the neutral axis to the
outermost fiber, and I is the moment of inertia
For a simply-supported beam with a uniformly distributed load, the maximum bending moment occurs
at the mid-span and is given by:
M=fracwL28
here w is the load per unit length and L is the span length
For an I-section, the distance from the neutral axis to the outermost fiber is half of the overall depth, so
c=450/2=225 mm
The moment of inertia is given in cm4, so we need to convert it to mm4 by multiplying by 1004. This gives
I=18000times1004=1.8times1012 mm4
Plugging in the values, we get:
sigma=fracfrac15times103times828times2251.8times1012
sigma=150t N/mm2
Q.51 The slenderness ratio of a circular column of diameter 300 mm and effective height 3 m is
__________. [in integer]
Ans:40
Solution:
The slenderness ratio of a column is the ratio of its effective length to its radius of gyration. It measures
the tendency of the column to buckle under compressive loads.
To calculate the slenderness ratio, we need to know the effective length factor (K), the radius of gyration
R, and the length of the column (L)
For a circular column, the radius of gyration is given by:
r=fracd4
where d is the diameter of the column
Plugging in the given value of d=300 mm, we get:
r=frac3004 mm=75 mm
The effective length factor (K) depends on the end conditions of the column. For example, if the column
is fixed at both ends, then K=0.5. If the column is pinned at both ends, then K=1. If the column is fixed
at one end and free at the other, then K=2
Assuming that the column is pinned at both ends, we have K=1. Then, the effective length of the column
is equal to the actual length, which is given as L=3 m
Therefore, the slenderness ratio is given by:
lambda=fracKLr
Plugging in the values, we get:
Hemant Vilas Parulekar 131

Q.52 A construction project consists of following five activities. The immediate successor activity
relationship and duration of each activity are mentioned in the table below.
Activity Immediate Successor Activity Duration (Weeks)
P R 2
Q R and S 4
R T 5
S - 6
T - 3
The total duration of the project is _______ weeks. [in integer]
Ans:- 12
From the network diagram, we can see that there are two possible
paths to complete the project:
Path 1: P -> R -> T, with a total duration of 2 + 5 + 3 = 10 weeks
Path 2: Q -> R -> T, with a total duration of 4 + 5 + 3 = 12 weeks
Path 3: Q -> S, with a total duration of 4 + 6 = 10 weeks

Q.53 It is proposed to have ceramic tile flooring in a room having internal clear dimension of 1.8 m × 2.4
m. Tile sizes are 300 mm × 300 mm. The door opening is 900 mm and the door is flushed with the
internal face of the wall. The height of skirting is 600 mm. The number of ceramic tiles required for
internal flooring and skirting is _______.[in integer]
Ans:- 98
Solution:
Area of room (floor tiles)=1.8×2.4=4.32 sq.m….(1)
Perimeter of room=2(1.8+2.4)=8.4 m.
The door opening= 900mm=0.9m. (striking will not be provided for this length)Thus, net perimeter of
room for which striking needs to be provided=8.4-0.9=7.5m
The height of striking=600mm=0.6m thus, area of striking=0.6×7.5=4.5 sq.m..(2)
Thus total area for which tiling needs to be provided=4.32+4.5=8.82sqm…..(by adding 1 and 2)
The tile size=300mm×300mm=0.3×0.3sq.m0.09 sq.m
Thus, total number of ceramic tiles required for internal flooring=8.83.0.09=98
Q.54 In a housing project, 75% of the permissible FAR was utilised after constructing four numbers eight
storey MIG towers with identical floor area of 400 sqm. If three numbers seven storey LIG towers
with identical floor area are built utilising the remaining FAR, the floor area of each LIG tower is
_______ sqm. [round off to 2 decimal places]
Ans:- 203.17 (All answers are considered correct and marks to all have been given)
FAR stands for floor area ratio, which is the ratio of a building’s total floor area to the size of the land on
which it is built1. It is often used as a regulation in city planning to limit urban density1. Different cities
or localities may have different FAR limits for different types of buildings.
132 GATE Architecture and Planning: Comprehensive Question Bank

MIG and LIG are abbreviations for middle income group and low income group, respectively. They are
categories of housing that target people with different income levels. MIG and LIG flats have different
sizes, amenities, and eligibility criteria2. For example, MIG flats are usually larger and more expensive
than LIG flats.
Now, let’s solve the problem using some algebra.
Let x be the floor area of each LIG tower. Then, the total floor area of the four MIG towers is 4 x 8 x 400
= 12800 sqm. The total floor area of the three LIG towers is 3 x 7 x x = 21x sqm. The total floor area of
the housing project is 12800 + 21x sqm.
Assuming that the permissible FAR is the same for both types of towers, we can write the following
equation:
12800+21x/area of land=FAR
Since 75% of the permissible FAR was utilised by the MIG towers, we can also write:
12800+21x/area of land=FAR×0.75
Dividing the first equation by the second equation, we get:
12800+21x/12800=0.75×FAR/FAR
Simplifying, we get:
12800+21x=1.33×12800
Solving for x, we get:
x=211.33×12800−12800/21
x≈203.81
Therefore, the floor area of each LIG tower is approximately 203.81 sqm (rounded off to 2 decimal places).
Q.55 Using the following values of thermal conductance, surface conductance and thermal resistance,
the U value across the given wall cross-section is _______W/m2 oC. [round off to 2 decimal places]
Thermal conductance ˗
• Brick wall 1.2 W/m oC
• Plastering 0.5 W/m oC
Surface conductance ˗
• Internal surface 8.0 W/m2 oC
• External surface 9.5 W/m2 oC
Thermal resistance ˗
• 50mm wall cavity 0.17 m2 oC/W
Ans:- 1.67 (All answers are considered correct and marks to all have been given)
Solution:
To calculate the U value of a wall cross-section, you need to use the formula:
where Rso and Rsi are the surface resistances of the outside and inside air layers, and R1,R2,... are the
thermal resistances of the wall layers The thermal resistance of a layer is given by:
R=d/k
Where d is the thickness of the layer and k is the thermal conductivity of the material.
GATE QUESTION PAPER 2020
General Aptitude (GA)

Q. 1 – Q. 5 carry one mark each.


Q.1 Rajiv Gandhi Khel Ratna Award was conferred _____ Mary Kom, a six-time world champion in
boxing, recently in a ceremony _____ the Rashtrapati Bhawan (the President’s official residence) in
New Delhi.
(A) with, at (B) on, in (C) to, at (D) on, at
Ans:- (C)
Q.2 Despite a string of poor performances, the chances of K. L. Rahul’s selection in the team is
(A) Obvious (B) uncertain (C) bright (D) slim
Ans:- (B)
Q.3 Select the word that fits the analogy:
Cover: Uncover: : Associate:______
(A) Misassociate (B) Dissociate (C) Unassociate (D) lnassociate
Ans:- (D)
Q.4 Hit by floods, the kharif (summer sown) crops in various parts of the country have been affected.
Officials believe that the loss in production of the kharif crops can be recovered in the output of
the rabi (winter sown) crops so that the country can achieve its food-grain production target of 291
million tons in the crop year 2019-20 (July-June). They are hopeful that good rains in July-August
will help the soil retain moisture for a longer period, helping winter sown crops such as wheat and
pulses during the November-February period.
Which of the following statements can be inferred from the given passage?
(A) Officials hope that the food-grain production target will be met due to a good.
(B) Officials declared that the food-grain production target will be met due to rains.
(C) Officials feel that he food-grain production target cannot be met due to floods.
(D) Officials want the food-grain production target to be met by the November- February period.
Ans:- (D)
Q.5 The difference between the sum of the first 2n natural numbers and the sum of the first n odd
natural numbers is
(A) n² - n (B) n² + n (C) 2n² - n (D) 2n² + n
Ans:- (B)
Explanation:
Let S1 be the sum of the first 2n natural numbers and S2 be the sum of the first n odd natural numbers.
We can use the formula for the sum of an arithmetic series to find S1 and S2.
S1 = 1 + 2 + 3 + … + 2n
This is an arithmetic series with first term a1 = 1 and common difference d1 = 1.
134 GATE Architecture and Planning: Comprehensive Question Bank

The formula for the sum of an arithmetic series is:


S = n/2 [2a + (n - 1)d]
Where n is the number of terms, a is the first term, and d is the common difference.
In this case, n = 2n, a = 1, and d = 1. So,
S1 = (2n)/2 [2(1) + (2n - 1)(1)]
S1 = n [2 + 2n - 1]
S1 = n [2n + 1]
S2 = 1 + 3 + 5 + … + (2n - 1)
This is also an arithmetic series with first term a2 = 1 and common difference d2 = 2.
Using the same formula, we get:
S2 = n/2 [2a + (n - 1)d]
Where n = n, a = 1, and d = 2. So,
S2 = n/2 [2(1) + (n - 1)(2)]
S2 = n/2 [2 + 2n - 2]
S2 = n/2 [2n]
S2 = n2
Now, the difference between S1 and S2 is:
S1 - S2 = n [2n + 1] - n2
S1 - S2 = 2n2 + n - n2
S1 - S2 = n2 + n
This matches with option B.
You can also verify this answer by plugging in some values of n and checking the sums.
For example, if n = 3, then
S1 = 1 + 2 + 3 + 4 + 5 + 6 = 21
S2 = 1 + 3 + 5 = 9
S1 - S2 = 21 - 9 = 12
n2 + n = 32 + 3 = 12
So, option B is correct.
Q. 6 – Q. 10 carry two marks each.
Q.6 Repo rate is the rate at which Reserve Bank of India (RBI) lends commercial bank, and reverse repo
rate is the rate at which RBI borrows money from commercial banks.
Which of the following statements can be inferred from the above passage?
(A) Increase in repo rate will decrease cost of borrowing an lending by commercial banks.
(B) Decrease in repo rate will decrease cost of borrowing and increase lending by commercial banks.
(C) Decrease in repo rate will increase cost of borrowing and decrease lending by commercial banks.
(D) Increase in repo rate will decrease cost of borrowing and increase lending by commercial banks.
Hemant Vilas Parulekar 135

Ans:- (B)
Explanation:
Repo rate is the interest rate at which RBI lends money to commercial banks, while reverse repo rate is
the interest rate at which RBI borrows money from commercial banks1. These rates are used to control
the money supply and inflation in the economy.
When the repo rate decreases, it becomes cheaper for commercial banks to borrow money from RBI.
This means they have more funds available to lend to their customers at lower interest rates. This
stimulates the demand for credit and increases the money supply in the economy.
When the reverse repo rate decreases, it becomes less attractive for commercial banks to park their
excess funds with RBI. This means they have more incentive to lend their money to other borrowers at
higher interest rates. This also increases the money supply in the economy.
Therefore, a decrease in both repo rate and reverse repo rate will decrease the cost of borrowing and
increase the lending by commercial banks.
Q.7 P, Q, R, S, T, U, V and W are seated around a circulatable by the government.
I. S is seated opposite to W.
II. U is seated at the second place to the right of R
III. T is seated at the third place to the left of R.
IV. V is a neighbour of S.
Which of the following must be true?
(A) P is a neighbour of R.
(B) Q is a neighbour of R.
(C) P is not seated opposite to Q.
(D) R is the left neighbour of S.
Ans:- (C)
Solution:
To solve this problem, we need to draw a diagram of the seating arrangement based on the given
information. We can use the following symbols to represent the direction of the people:
⬆: Facing the center
⬇: Facing away from the center
Let’s start with the first clue: S is seated opposite to W. We can place them anywhere on the circle, as long
as they are facing each other. For example:
S⬆ W⬇
Next, we can use the second clue: U is seated at the second place to the right of R. This means that U and
R are facing the same direction, and there is one person between them. We can place them on either side
of S and W, as long as we maintain the right direction. For example:
S⬆ U⬇ R⬇ W⬇
136 GATE Architecture and Planning: Comprehensive Question Bank

Now, we can use the third clue: T is seated at the third place to the left of R. This means that T and R are
facing opposite directions, and there are two people between them. We can place T on the other side of
S and W, as long as we follow the left direction. For example:
T⬆ S⬆ U⬇ R⬇ W⬇
Next, we can use the fourth clue: V is a neighbour of S. This means that V is adjacent to S, and facing the
same direction as S. We can place V on either side of S, as long as we do not violate the previous clues.
For example:
T⬆ V⬆ S⬆ U⬇ R⬇ W⬇
Now, we have placed six people out of eight, and we have two empty seats left. We can use the process
of elimination to find the positions of P and Q. We know that P and Q are not neighbours of R, so
they cannot be placed next to U or T. We also know that P is not seated opposite to Q, so they cannot
be placed across S and W. Therefore, the only possible positions for P and Q are next to V and W,
respectively. For example:
P⬇ T⬆ V⬆ S⬆ U⬇ R⬇ Q⬆ W⬇
This is the final diagram of the seating arrangement. Now, we can check
the options and see which one must be true.
(A). P is a neighbour of R. This is false, as P is not adjacent to R.
(B). Q is a neighbour of R. This is false, as Q is not adjacent to R.
(C). P is not seated opposite to Q. This is true, as P and Q are not facing
each other.
(D). R is the left neighbour of S. This is false, as R is the right neighbour of S.
Q.8 The distance between Delhi and Agra is 233km. A car P started travelling from Delhi to Agra and
another car Q started from Agra to Delhi along the same road I hour after the car P started. The
two cars crossed each other 75 minutes after the car Q started. Both cars were travelling at constant
speed. The speed of car P was 10 km/hr more than the speed of car Q. How many kilometers the car
Q had travelled when the cars crossed each other?
(A) 66.2 (B) 75.2 (C) 88.2 (D) 116.5
Ans:- (B)
Solution:
Let us Assume that, speed of car P is x km/h.
Then, speed of car Q will be (x - 10) km/h.
Now, for the first hour car P alone travel. So, it will cover x km distance in one hour.
Then,
→ Distance left to be covered = (233 - x) km.
Now, Q also started from Agra at a speed of (x - 10) km/h.
As, both Travel in opposite direction,
→ usual speed = Speed of P + Speed of Q
→ usual speed = x + (x - 10) = (2x - 10) km/h.
Hemant Vilas Parulekar 137

and,
→ Distance to be covered = (233 - x) km.
Therefore,
→ They will meet after = (Distance Left) / (Usual speed)
Now, meeting time is given 75 minutes after the car Q started.
Hence,
→ (233 - x) / (2x - 10) = 75 minutes
→ (233 - x) / (2x - 10) = (75/60) Hours.
→ (233 - x) / (2x - 10) = 5/4
→ 5(2x - 10) = 4(233 - x)
→ 10x - 50 = 932 - 4x
→ 10x + 4x = 932 + 50
→ 14x = 982
→ x = 70.14 km/h.
So,
→ Speed of car P = x = 70.14 km/h .
→ Speed of car Q = (x - 10) = 60.14 km/h.
Now, we have given that, Car Q travels for 75 minutes before meeting .
Therefore,
→ Speed of car Q = 60.14 km/h
→ Time = 75 minutes = (5/4) Hours.
→ Distance covered = Speed * Time
→ Distance = 60.14 * (5/4)
→ Distance = 75.2km . (Ans.)
Q.9 For a matrix M = [mij]; i,j = 1, 2, 3, 4, the diagonal elements are all zero and mij= –mij· The minimum
number of elements required to fully specify the matrix is.
(A) 0 (B) 6 (C)12 (D) 16
Ans:- (B)
Explanation:
This is a question about the properties of a hollow symmetric matrix, which is a square matrix whose
diagonal elements are all zero and whose off-diagonal elements are symmetric, i.e. mij = mji for all i and j.
To fully specify such a matrix, we only need to know the values of the elements above or below the
diagonal, since the other elements are determined by symmetry. For a 4 × 4 matrix, there are 6 such
elements: m12, m13, m14, m23, m24, and m34. Therefore, the minimum number of elements required
to fully specify the matrix is 6.
138 GATE Architecture and Planning: Comprehensive Question Bank

Q.10 The profit shares of two companies P and Q are


shown in the figure.
If two companies have invested a fixed and equal
amount every year, then the ratio of the total revenue
of company P to the total revenue of company Q,
during 2013 – 2018 is.
(A) 15:17 (B) 16:17
(C) 17:15 (D) 17:16

Ans:- (B)
Solution:
Year Company ‘P’ Company ‘Q’
Investment Profit Investment Profit
2013 100 10 100 20
2014 100 20 100 30
2015 100 40 100 30
2016 100 40 100 50
2017 100 50 100 60
2018 100 40 100 60
Total 600 200 600 250
As, Revenue = Investment + Profit
Revenue 800 850
Thus, ratio of Revenue ‘P’ to Revenue ‘Q’ = 800/850 =16:17
Q. 1 – Q. 25 carry one mark each.
Q.1 In the architectural style of ancient North Indian Temples, the term ‘Adhisthana’ refers to
(A) Pinnacle (B) Base Platform (C) Vestibule (D) Trancept
Ans:- (B)
Explanation: Please refer GATE 2014, Q.35 for more information.
The term adhisthana refers to the base or substratum on which the temple stands. It is also mentioned in
the Hinduism Outlook article and the Azadi Ka Amrit Mahotsav article as the plinth or the base of the
structure. Therefore, the correct answer is option B.
Q.2 Who among the following architects has NOT won the Pritzker Architecture Prize till 2019?
(A) Arata Isozaki (B) I. M. Pei (C) B. V. Doshi (D) Moshe Safdie
Ans:- (A) or (D)
Explanation:
Please refer GATE 2017, Q.22 for more information.
Q.3 The stone used in the construction of Kailasa temple at Ellora is
(A) Limestone (B) Marble (C) Sandstone (D) Basalt
Hemant Vilas Parulekar 139

Ans:- (D)
Explanation:
Please refer GATE 2018, Q.7 for more information.
Q.4 Four vertical lines having same thickness appear to be of the same height in perspective as shown in
the figure. Which line actually has the maximum height?
(A) 1
(B) 2
(C) 3
(D) 4

Ans:- (B)
Explanation:
The longest line will appear thinner in perspective.
Q.5 As per URDPFl Guidelines 2015, Government of India, choose the correct hierarchy of plans from
higher to lower order.
(A) Perspective plan, Development plan, Regional plan, Zonal plan
(B) Perspective plan, Regional plan, Development plan, Zonal plan
(C) Regional plan, Perspective plan, Development plan, Zonal plan
(D) Zonal plan, Development plan, Regional plan, Perspective plan
Ans:- (B)
Explanation: Refer GATE 2018, Q.13 for more information
The URDPFI Guidelines 2015 is a comprehensive document that provides the framework and principles
for urban and regional development planning in India. It covers the concepts, objectives, processes,
standards and indicators of various aspects of urban planning, such as land use, transport, housing,
environment, heritage, governance and finance. The document is a valuable resource for planners,
policy makers, researchers and students of urban and regional planning.
140 GATE Architecture and Planning: Comprehensive Question Bank

Q.6 Which of the following shapes can be used as an interlocking paver block without adding any other
shape?
(A) 1
(B) 2
(C) 3
(D) 4

Ans:- (C)
Explanation:
The concave and convex forms of blocks are complimentary with each other in such a way that they
interlocked with each other without adding any other shape.
An interlocking paver block is a type of paving block that can lock into each other along the vertical faces
when paved in any pattern. This feature provides stability and resistance to the paved surface. According
to the URDPFI Guidelines 2015, there are four types of interlocking paver blocks based on their shapes:
Type A: Paver blocks with plain vertical faces, which do not interlock into each other when paved in
any pattern.
Type B: Paver blocks with alternating plain and curved/corrugated vertical faces, which lock into each
other along the curve/corrugated faces, when paved in any pattern.
Type C: Paver blocks having all faces curved or corrugated, which lock into each other along with all the
vertical faces when paved in any pattern.
Type D: ‘L’ and ‘X’ shaped paver blocks that have all faces curved or corrugated and which lock into each
other along with all the vertical faces when paved in any pattern.
Q.7 In India, the Constitution (Seventy Fourth Amendment) Act. 1992, delegates powers to institutions
forming the third tier of government, which are
(A) Municipal Corporation, Municipality and Nagar Panchayat
(B) Development Authority, Municipal Corporation and Municipality
(C) Improvement Trust, Nagar Panchayat and Panchayat
(D) Development Authority, Improvement Trust and Panchayat
Ans:- (A)
Explanation:
Municipal Corporation, Municipality and Nagar Panchayat. These are the three types of urban local
bodies that are recognized by the Constitution (Seventy Fourth Amendment) Act, 1992. The Act aims
to strengthen the decentralization of urban governance and provide constitutional status to the urban
local bodies. The Act also specifies the composition, functions, powers, elections and finances of the
urban local bodies.
Hemant Vilas Parulekar 141

Q.8 As on 2018, ‘Right to Property’ in India is a


(A) Fundamental Right (B) Secondary Right
(C) Constitutional Right (D) Tertiary Right
Ans:- (C)
Explanation: The ‘Right to Property’ was originally a fundamental right under the Constitution of India,
as per Article 31. However, it was removed from the list of fundamental rights by the 44th Amendment
Act of 1978. It was then transformed into a legal right or a constitutional right under Article 300A. This
article states that no person shall be deprived of his property save by the authority of law, making the
‘Right to Property’ a constitutional right but not a fundamental right.
Q.9 ‘Tendon’ is primarily used
(A) as a compression member (B) to pre-stress concrete
(C) as roof sheathing Explanation (D) to prepare a tender document
Ans:- (B)
Explanation: Please refer GATE 2019, Q.31 for more information.
Q.10 Emergency preparedness for risk reduction does NOT include
(A) Rescue (B) Relief Distribution
(C) Rehabilitation (D) Revision of Code
Ans:- (D)
Q.11 If Beam: Column : : Transom : X
(A) Balustrade (B) Sill
(C) Mullion (D) Ceiling
Ans:- (C)
Explanation:
Q.12 The correct chronological order of the given architectural movements is
(A) Romanesque; Roman; Baroque; Gothic ; Renaissance
(B) Romanesque; Roman; Renaissance; Gothic ; Baroque
(C) Roman; Romanesque; Gothic; Renaissance; Baroque
(D) Roman; Romanesque; Gothic; Baroque; Renaissance
Ans:- (C)
Explanation:
Roman architecture dates back to the 1st century BCE and is characterized by the use of concrete,
arches, vaults, and domes. Some famous examples are the Colosseum, the Pantheon, and the Pont du
Gard.
Romanesque architecture emerged in the 10th century and lasted until the 12th century. It is influenced
by Roman and Byzantine styles and features thick walls, round arches, sturdy pillars, and decorative
arcades. Some examples are the Abbey of Cluny, the Speyer Cathedral, and the Pisa Cathedral.
142 GATE Architecture and Planning: Comprehensive Question Bank

Gothic architecture developed in the 12th century and flourished until the 16th century. It is
distinguished by the use of pointed arches, ribbed vaults, flying buttresses, and stained-glass windows.
Some examples are the Notre Dame Cathedral, the Chartres Cathedral, and the Westminster Abbey.
Renaissance architecture originated in Italy in the 15th century and spread throughout Europe until the
17th century. It is inspired by the classical ideals of harmony, proportion, and symmetry. Some examples
are the St. Peter’s Basilica, the Palazzo Medici, and the Villa Rotonda.
Baroque architecture emerged in the 17th century and lasted until the 18th century. It is characterized
by the use of complex shapes, curves, ornaments, and dramatic effects. Some examples are the Palace of
Versailles, the St. Paul’s Cathedral, and the Trevi Fountain.
Q.13 The decay of sound in a large room is indicated in the following figure.
The spike within the dashed zone denotes
(A) Flutter Echo
(B) Perfect Sound Diffusion
(C) Echo
(D) Early Reflection

Ans:- (C)
Explanation:
A flutter echo is a rapid series of echoes caused by sound waves bouncing back and forth between
two parallel surfaces in a room, such as walls or floor and ceiling. The spike within the dashed zone
indicates a sudden increase in sound pressure level due to the flutter echo. The decay of sound in a large
room is measured by the reverberation time, or RT60, which is the time it takes for a sound to decrease
by 60 decibels345. The RT60 depends on the size, shape, and acoustic properties of the room and the
sound source. A longer RT60 means more reverberation and echo, while a shorter RT60 means less
reverberation and echo. The RT60 can be reduced by adding sound-absorbing materials or objects to
the room, such as rugs, curtains, furniture, or plants Reverberant Decay of Sound in Large Room; Echo
may be due to rear wall or another remote sound-reflecting surface.
Perfect sound diffusion is a theoretical concept that describes a sound field where the sound energy is
uniformly distributed in all directions and frequencies.
Echo is a distinct repetition of a sound that occurs when the sound wave reflects off a distant surface and
returns to the listener after a noticeable delay.
Early reflection is the first group of echoes that occurs when the sound wave bounces off surfaces in a
room after being emitted by a sound source. These reflections tend to be more defined and sound more
like “echo” than “reverb”.
Q.14 For the same thickness of material layers, relative position of insulation in the wall sections l and 2
shown below will have an impact on
(A) Thermal Time Constant
(B) Thermal Conductivity
(C) Thermal Resistivity
(D) Thermal Transmittance
Hemant Vilas Parulekar 143

Ans:- (A)
Explanation:
Thermal Time Constant is the time taken by a system to undergo a temperature change that is equal to
63.2% of the initial temperature difference between the system and its surrounding1.
Thermal Conductivity is the measure of the ability of a material to conduct heat. It is expressed in W/
m⋅K and depends on the material properties and the thickness of the layers234.
Thermal Resistivity is the reciprocal of thermal conductivity. It is the measure of the resistance of a
material to heat flow. It is expressed in K⋅m/W and also depends on the material properties and the
thickness of the layers2.
Thermal Transmittance is the rate of heat transfer through a unit area of a material under a unit
temperature difference. It is also known as the U-value and is expressed in W/m2⋅K. It depends on the
thermal conductivity, the thickness, and the position of the layers
Q.15 The solar altitude angle on April 16 at 7:00 AM in Kochi is 16°. The same solar altitude angle will
occur at the same time in the same year at the same location on
(A) July 21 (B) August 27 (C) September 23 (D) October 21
Ans:- (B)
Explanation:
The solar altitude angle is the angle between the horizontal plane and the sun in the sky. It depends on
the latitude, the declination angle, and the solar hour angle. The declination angle is the angle between
the earth’s equator and the sun’s rays. It varies from -23.44° to 23.44° throughout the year. The solar hour
angle is the angle between the meridian of the observer and the meridian of the sun. It changes by 15°
every hour
To find the answer, we need to use the following formula,

where α is the solar altitude angle, δ is the declination angle, ϕ is the latitude, and h is the solar hour
angle.
We know that the latitude of Kochi is 9.93° N2, and the solar hour angle at 7:00 AM is -75° (assuming
solar noon is at 12:00 PM). We can use the following equation to calculate the declination angle for any
day of the year

here d is the number of days since January 1st.


Using this equation, we can find the declination angle for April 16th and each of the four options:
April 16th: δ=23.44°sin(360/365 (106−81))=9.32°
July 21st: δ=23.44°sin(360/365 (202−81))=20.69°
August 27th: δ=23.44°sin(360/365(239−81))=11.76°
September 23rd: δ=23.44°sin(360/365(266−81))=−0.01°
October 21st: δ=23.44°sin(360/365(294−81))=−10.98°
144 GATE Architecture and Planning: Comprehensive Question Bank

Now, we can plug in these values into the formula for the solar altitude angle and compare them with
the given value of 16°:
April 16th: α=sin−1(sin9.32°sin9.93°+cos9.32°cos9.93°cos(−75°))=16.00°
July 21st: α=sin−1(sin20.69°sin9.93°+cos20.69°cos9.93°cos(−75°))=28.86°
August 27th: α=sin−1(sin11.76°sin9.93°+cos11.76°cos9.93°cos(−75°))=16.02°
September 23rd: α=sin−1(sin−0.01°sin9.93°+cos−0.01°cos9.93°cos(−75°))=3.14°
October 21st: α=sin−1(sin−10.98°sin9.93°+cos−10.98°cos9.93°cos(−75°))=−9.74°
As we can see, the closest option to 16° is August 27th. Therefore, the correct answer is (B) August 27.
This is because the declination angle on August 27th is very close to the declination angle on April 16th,
which means the sun’s rays are at a similar angle to the earth’s equator.
Q.16 In a perspective drawing, the Picture Plane is in between the Object and the Observer. If the Observer
comes closer straight towards the Picture x the Observer. If the Observer comes closer straight
towards the Picture Plane, without changing the distance between Object and Picture Plane, the
perspective image will be
(A) Bigger than the previous image (B) Smaller than the previous image
(C) Will remain the same as previous image (D) Will become the mirror image of the previous
Ans:- (B)
Explanation:
In a perspective drawing, the Picture Plane is a vertical plane that is perpendicular to the line of sight
from the Observer to the Object. The Picture Plane is used to project the Object onto a two-dimensional
surface, such as a canvas or a paper. The distance between the Observer and the Picture Plane affects
the apparent size and shape of the Object in the projection. If the Observer comes closer to the Picture
Plane, the Object will appear larger and more distorted. If the Observer moves away from the Picture
Plane, the Object will appear smaller and less distorted.
Q.17 Shyam-Rai temple of Bishnupur in West Bengal, is an example of
(A) Nava-ratna type terracotta temple (B) Stone carved Nagara type temple
(C) Pancha-ratna type terracotta temple (D) Stone carved Dravidian type temple
Ans:- (C)
Explanation:
Refer GATE 2018, Q.7 for more information.
Q.18 Which one of the following is NOT a land use zone?
(A) Industrial Zone (B) Agricultural Zone
(C) Heritage Zone (D) Commercial Zone
Ans:- (C)
Explanation:
Heritage Zone.
A heritage zone is not a land use zone, but a designation that protects the historical, cultural, or natural
value of a certain area. A heritage zone may overlap with other land use zones, such as residential or
commercial, but it does not determine the function of the land.
Hemant Vilas Parulekar 145

Q.19 ‘Formulation of GIS based master plan· is a sub-scheme of


(A) Atal Mission for Rejuvenation and Urban Transformation
(B) Smart Cities Mission
(C) Jawaharlal Nehru National Urban Renewal Mission
(D) Shyama Prasad Mukherji Rurban Mission
Ans:- (A)
Explanation:
Refer GATE 2018, Q.26 for more information.
Q.20 One hectare is equal to
(A) 4000 m² (B) 4048 m² (C) 4480 m² (D) 10000 m²
Ans:- (B)
Explanation:
A hectare is a unit of area that is accepted in the International System of Units (SI). It is primarily used
to measure land area. One hectare is equal to 10,000 square meters and is equivalent to approximately
2.471 acres.
A square meter, or square metre, is an SI derived unit of area. It is the size of a square that is one meter
on a side. It is approximately 10.76 square feet.
To convert from hectares to square meters, you can multiply the number of hectares by 10,000. For
example, 1 hectare x 10,000 = 10,000 square meters. To convert from square meters to hectares, you can
divide the number of square meters by 10,000. For example, 10,000 square meters / 10,000 = 1 hectare.
Q.21 One of the sites added to the list of UNESCO World Heritage Sites in 2019 is
(A) Walled City, Ahmedabad (B) Agricultural Zone
(C) Walled City, Jaipur (D) Fatehpur Sikri
Ans:- (B)
Explanation: Refer GATE 2019, Q.16 for more information.
Q.22 In the given contour map, the angle at’ A· (in degrees, rounded off to two decimal is __________

Ans:- 18.2 to 18.5


Solution:
Contour interval height = 230-220 = 10m
Length of ‘A’ = 30m
Angle at ‘A’ =Tan-1(10/30) = 18.43°
146 GATE Architecture and Planning: Comprehensive Question Bank

Q.23 A 1.2 m high window is located on a south facing wall. The solar azimuth angle is equal to the wall
azimuth angle and the solar altitude angle is 60°. The minimum depth (in meters, rounded off to
two decimal places) of overhang required to completely shade the window is ___ (Assume that the
overhang is located at the lintel level of the window)
Ans:- 0.68 to 0.70
Solution:
The minimum depth of overhang required to completely shade the window is given by the formula:

where d is the depth of overhang, h is the height of the window, and α is the solar altitude angle.
Substituting the given values, we get:

d = 0.69
Therefore, the minimum depth of overhang required to completely shade the window is 0.69 m.
Q.24 In the given figure, the area of the shaded portion is ______

Ans:- 8 square units


Solution:
The semi-circular portions on both ends can fill up the circular puncture in the center of the
shaded portion.
That way, shaded portion occupies 2 rows and 4 columns. 2 × 4 = 8 Which results
in area 8 square units.
Q.25 Average density of a highway is 25 vehicles per km. Average volume of the vehicles on the highway
is 520 vehicles per hour. The mean speed (in km/ hour, rounded off to one decimal place) is ______
Ans:- 20.8
Solution:
Given:
Average density of highway = 25 vehicles/km
Average volume of highway = 520 vehicles/h
Hence, Mean speed of the vehicles = 520 / 25 = 20.8 km/h
Hemant Vilas Parulekar 147

Q. 26 – Q. 55 carry two marks each.


Q.26 Match the terminologies of Munsell colour wheel in Group I with their corresponding descriptions
in Group II
(A) P-2,Q-4,R-5,S-1 Group I Group II
(B) P-4,Q-2,R-5,S-3 (P) Hue 1. Addition of black to the base colour
(C) P-4,Q-2, R-3,S-1 (Q) Chroma 2. Radial colour varition
(R) Value 3. Addition of white to the base colour
(D) P-2,Q-4, R-1,S-3
(S) Tint 4. Colour variation through angular difference
5. Vertical color variation
Ans:- (B)
Explanation:
The Munsell colour system is a way of specifying colours based on three
properties: hue, chroma, and value1. Hue is the basic colour, such as red,
yellow, green, blue, and purple. Chroma is the intensity or purity of the
colour, how far it is from a neutral grey. Value is the lightness or darkness of the colour, how close it is
to white or black.
The Munsell colour system uses a cylindrical representation of the colour space, where hue is measured
by degrees around horizontal circles, chroma is measured radially outward from the neutral vertical axis,
and value is measured vertically from 0 (black) to 10 (white). The system is based on human perception
of colour, so the spacing of colours along these dimensions is not uniform, but rather adjusted to match
how we see colour differences.
Q.27 Match the plant forms in Group I with the botanical names in Group II. As per ‘A Handbook of
Landscape’, CPWD 2013, Government of India.
(A) P-1, Q-5, R-2, S-3 Group I Group II
(B) P-3, Q-4, R-2, S-1 (P) Coulmnar 1. Pinus roxburghii
(C) P-3, Q-5, R-4, S-1 (Q) Globular 2.Ipomoea grandiflora
(R) Weeping 3. Juniperus chunensis
(D) P-1, Q-3, R-4, S-5
(S) Pyramidal 4. Salix babylonica
5. Mimusops elengi
Ans:- (C)
Explanation: Refer GATE 2017, Q.25 for more information on plant forms.
Q.28 Match the images of gardens in Group I with their names in Group II
Group I Group II
(P) 1. Central park, New York

(Q) 2. Versailles, Paris


148 GATE Architecture and Planning: Comprehensive Question Bank

(R) 3. Nishat Bagh, Srinagar (A) P-5, Q-3, R-1, S-4


(B) P-3, Q-4, R-1, S-2
(C) P-1, Q-4, R-3, S-2

(S) 4. Katsura Imperial Garden, (D) P-3, Q-2, R-4, S-5


Kyoto

5. Alhambara Moorish
Garden,Granda
Ans:- (B)
Explanation:
Park Central park, New york Versailles, Paris
Image

Park Nishat Bagh, Srinagar Katsura Imperial Garden, Kyoto


Image

Park Alhambara Moorish Garden,Granada


Image

Q.29 Match the Architects in Group I with their projects in Group II


(A) P-3, Q-4, R-1, S-2 Group I Group II
(B) P-2, Q-5, R-4, S-1 (P) Victo horta 1. Farnsworth House
(C) P-4, Q-3, R-1, S-2 (Q) Gerrit reitvelt 2. Robie House
(D) P-3, Q-4, R-5, S-2 (R) Mies van der Rohe 3. Tassel House
(S) Frank Lloyd wright 4. Schroder House
5. Vanna Ventury House
Hemant Vilas Parulekar 149

Ans:- (A)
Explanation:
Victo Horta was a Belgian architect and one of the founders of Art Nouveau. He designed the Tassel
House in Brussels, which is considered the first Art Nouveau building in the world1.
Gerrit Rietveld was a Dutch architect and furniture designer, and a member of the De Stijl movement.
He designed the Schroder House in Utrecht, which is an iconic example of De Stijl architecture.
Mies van der Rohe was a German-American architect and a pioneer of modernist architecture. He
designed the Farnsworth House in Illinois, which is a masterpiece of minimalist design and a landmark
of the International Style.
Frank Lloyd Wright was an American architect and the leader of the Prairie School of architecture.
He designed the Robie House in Chicago, which is widely regarded as the best example of Prairie Style
architecture.
Q.30 Match the graphical representations in Group I with corresponding elements in Group II
Group I Group II (A) P-5, Q-3, R-1, S-4
(P) 1. Pathway (B) P-3, Q-4, R-1, S-2
(C) P-1, Q-4, R-3, S-2
(D) P-3, Q-2, R-4, S-5

(Q) 2. Node

(R) 3. District

(S) 4. Edge

5.Landmark
Answer: (D)
Terms Pathway Node
Explanation The streets, sidewalks, trails, and other The strategic spots in a city into which
channels in which people travel. Lynch an observer can enter, and which are the
noted that paths were intensive foci and from
often the predominant elements in which the person is travelling.
people’s image with the other elements
being arranged and related along paths.
150 GATE Architecture and Planning: Comprehensive Question Bank

Terms Pathway Node


Image

Terms District Edge


Explanation Areas characterized by common May be barriers, more or less penetrable,
characteristics, these are the medium to which close one region off from another,
large areas, which observers or they may be
mentally enter ‘inside of ’ and/or have seams, lines along which two regions are
some common identifying character. related and joined together.
Distinctive physical characteristics
might include ‘thematic continuities’, such
as texture, space, form, detail, symbol and
building.
Image

Terms Landmark
Explanation Landmark’s key physical characteristics was singularity some aspect that is unique or
memorable
in the context. Some landmarks – towers, spires, hills are distant and are typically
seen from many angles and
from distance, over the top of smaller elements. Other landmarks – sculptures, signs
and trees are primarily local being visible only in restricted localities and from certain
approaches.
Image
Hemant Vilas Parulekar 151

Q.31 Based on the psychrometric chart given below, match the vector in Group
I with the respective process in Group II

Group I Group II
(P) V1 1. Heat and humidification
(Q) V2 2. Cooling and humidification
(R) V3 3.Heating and dehumidification
(S) V4 4. Sensible heating
5. Humidification

(A) P-4, Q-1, R-5, S-2 (B) P-4, Q-3, R-1, S-5
(C) P-5, Q-3, R-4, S-1 (D) P-3, Q-1, R-5, S-2
Ans: (A)
Explanation:
Vector Dry Bulb temperature Humidity Process
V1 Increasing Constant Sensible heating
V2 Increasing Increasing Heating and humidification
V3 Constant Increasing Humidification
V4 Decreasing Increasing Cooling and Humidificatiom
Q.32 Match the software tools in Group I with their primary applications in Group II
Group I Group II (A) P-2, Q-5, R-4, S-3
(P) ETabs 1. Acoustic analysis (B) P-4, Q-1, R-2, S-3
(Q) Carto 2. Structural analysis (C) P-4, Q-5, R-1, S-3
(R) eQuest 3. Statistical analysis
(D) P-2, Q-4, R-5, S-1
(S) SPSS 4. Energy stimulation
5. Geo-spatial analysis
Ans:- (A)
Explanation:
ETabs is a software product that caters to multi-story building analysis and design, using advanced
structural analysis and design features. Therefore, it is mainly used for structural analysis (2) in
Group II.
Carto is a cloud native platform for spatial analytics, offering data enrichment, spatial analysis, and
solutions for various use cases and industries. Therefore, it is mainly used for geo-spatial analysis (5) in
Group II.
eQuest is a software tool that simulates building energy use and performance, using a comprehensive
and flexible simulation engine derived from DOE-25. Therefore, it is mainly used for energy stimulation
(4) in Group II.
SPSS is a software package that offers advanced statistical analysis, a vast library of machine learning
algorithms, text analysis, and integration with big data. Therefore, it is mainly used for statistical analysis
(3) in Group II.
152 GATE Architecture and Planning: Comprehensive Question Bank

Q.33 Match the structural forms in Group I with their corresponding illustrations in Group II
Group I Group II (A) P-2, Q-4, R-5, S-1
(P) Cylindrical shell 1. (B) P-2, Q-4, R-1, S-5
(C) P-5, Q-4, R-2, S-1
(D) P-4, Q-1, R-5, S-2

(Q) Dome 2.

(R) Conoid 3.

(S) Hyperbolic paraboloid 4.

5.

Ans:- (A)
Q.34 Match the books in Group I with the corresponding authors in Group II
(A) P-3, Q-2, R-1, S-4 Group I Group II
(B) P-4, Q-1, R-5, S-3 (P) The Autobiography of an idea 1. Christopher Charles Benninger
(C) P-3, Q-1, R-5, S-4 (Q) Letters to a Young Architect 2. Sunil Khilnani
(D) P-4. Q-2, R-1, S-3 (R) A pattern language 3. Francis D.K Ching
(S) Architecture: forms, space and 4. Louis H. Sullivan
order
5. Christopher Alexander
Ans :- (B)
Explanation:
P) The Autobiography of an idea is written by Louis H. Sullivan, who is widely regarded as the father of
modern American architecture.
(Q) Letters to a Young Architect is written by Christopher Charles Benninger, who is an American-
Indian architect and planner known for his work in India and other developing countries.
(R) A Pattern Language: Towns, Buildings, Construction is written by Christopher Alexander and his
colleagues, who propose a cataloging of the types of problems and solutions in architectural design.
(S) Architecture: Form, Space, and Order is written by Francis D.K Ching, who illustrates the fundamental
elements of space and form in architecture
Hemant Vilas Parulekar 153

Q.35 Match the tactile paving in Group I with their pattern in Group II
Group I Group II (A) P-4, Q-2, R-3, S- 1
(P) lozenge 1. (B) P-3, Q-5, R-4, S- 1
(C) P-3, Q-2, R-4, S-1
(D) P-2, Q-5, R-1, S-4

(Q) offset blister 2.

(R) corduroy 3.

(S) Directional 4.

5.

Ans:- (C)
Explanation:
Tactile paving lozenge offset blister
Explanation A lozenge tactile pattern is a type of An offset blister tactile pattern is a type
tactile paving that consists of evenly of tactile paving that consists of flat-
spaced rows of lozenge-shaped tiles with topped domes (blisters) arranged in an
rounded edges. This type of tactile paving offset pattern. This pattern is used to
warns pedestrians they are approaching warn visually impaired people of the
street-level rapid transportation such edge of off-street railway platforms, such
as a tram. It is also known as on-street as train, tram, or tube stations. The offset
(platform edge) warning paving pattern. blister tactile pattern can help people to
identify the direction of the platform
edge and the correct direction of travel to
access a train.
Pattern
154 GATE Architecture and Planning: Comprehensive Question Bank

Tactile corduroy Directional


paving
Explanation A corduroy tactile pattern is a type of A directional tactile pattern is a type of
tactile paving that consists of flat-topped tactile paving that consists of evenly
domes (blisters) arranged in an offset spaced rows of lozenge-shaped tiles with
pattern. This pattern is used to warn rounded edges. This type of tactile paving
visually impaired people of the edge of provides directional orientation and
off-street railway platforms, such as train, guides visually impaired people along the
tram, or tube stations. intended travel path. It is also known as
leading tactile indicators or tactile guide
path
Pattern

Q.36 Match the name of architects in Group I with the buildings designed by them in Group II
(A) P-3,Q-1,R-4,S-2 Group I Group II
(B) P-2,Q-5,R-1,S-3 (P) Brinda Somaya 1. Museum of tribal Heritge, Bhopal
(C) P-2,Q-1,R-4,S-5 (Q) Sheila Sai Prakash 2. St.Thomas Cathedral, Mumbai
(R) Revathy Kamath 3.Bait-ur-Rauf Mosque, Dhaka
(D) P-4,Q-5,R-1,S-3
(S) Marina Tabassum 4.Indian Naval Academy, Kerala
5. Cholamandal Artists Village, Chennai
Ans:- (B)
Explanation:
Buildings Museum of tribal Heritge, Bhopal St. Thomas Cathedral, Mumbai
Architects Revathy Kamath Brinda Somayya
Buildings

Buildings Bait-ur-Rauf Mosque, Dhaka Indian Naval Academy, Kerala


Architects Marina Tabassum Namita Singh
Buildings
Hemant Vilas Parulekar 155

Buildings Cholamandal Artists Village, Chennai


Architects Sheila Sai Prakash
Buildings

Q.37 Match the terms in Group- I with the parameters in Group-II


(A) P-2, Q-5, R-4, S-1 Group - I Group - II
(B) P-3, Q-2, R-1, S-4 (P) Frontal Area Density 1.Active Green Area
(C) P-2, Q-3, R-4, S-5 (Q) Sky View Factor 2.Urban Density in Third Dimension
(R) Drift Index 3.Built Density in Two Dimension
(D) P-3, Q-4, R-5, S-1
(S) Biotope Factor 4.Lateral Stiffness
5.Cross Sectional Property of Urban Canyon

Ans:- (A)
Explanation:
Term Description
Frontal Area Frontal area density is a measure of the urban morphology that
Density affects the wind environment in a city. It is defined as the ratio of
the total building facade area to the ground area of an urban
canyon1. An urban canyon is a street or a space between two
rows of buildings. A higher frontal area density means a more compact and dense
urban form, which can reduce the wind speed and ventilation in the urban area.
Sky View Factor Sky View Factor (SVF) is a measure of the urban
geometry that affects the radiation and thermal
environment in a city. It is defined as the ratio of the
visible sky area to the total hemispherical area as seen
from a point on the ground. A higher SVF means a more
open and less obstructed sky view, which can increase the solar radiation and
ventilation in the urban area.
Drift Index Drift index is a measure of the lateral displacement or drift of a building structure
under wind or earthquake forces. It is defined as the ratio of the maximum
deflection at the top of the building to the total height. A lower drift index means
a more stable and rigid structure, which can prevent excessive damage and
discomfort to the occupants.
Biotope Factor The Biotope Factor (BF) is a tool used to measure the absorbent properties of a
surface. It is calculated by dividing the ecologically effective surface area by the
total surface area of a lot. The BF can be used to guide urban planning and design,
as it helps to balance densification and greening policies. A higher BF means more
green space, which can improve air quality, reduce heat islands, and prevent urban
flooding. Different types of surfaces have different ecological values, depending
on their permeability, vegetation, and connection to the soil.
156 GATE Architecture and Planning: Comprehensive Question Bank

Q.38 Match the structural system in Group I with their potential causes of failure in Group II

(A) P-3, Q-4, R-1, S-2 Group I Group II


(B) P-1, Q-3, R-5, S-2 (P) Flat slab 1. Thrust
(C) P-2. Q-4, R-1, S-3 (Q) Long Coulmn 2. Flutter
(R) Arch 3. Punching shear
(D) P-3, Q-5, R-4, S-1
(S) Tensile Fabric 4. Bucking
5. Moment
Ans:- (A)
Explanation:

(P) Flat slab is a type of reinforced concrete slab that is supported directly by
columns without beams. It can fail due to punching shear, which is a localized
failure of the slab around the column due to high shear stress.

(Q) Long column is a type of column that has a slenderness ratio greater than,
which means its length is much larger than its cross-sectional dimensions. It can
fail due to buckling, which is a sudden lateral instability of the column under
compressive load.

(R) Arch is a type of curved structure that spans an opening and transfers the load
to the supports1. It can fail due to thrust, which is a horizontal force that pushes the
arch outward and causes tension in the arch.

(S) Tensile fabric is a type of flexible membrane that is stretched and anchored to
form a roof or a canopy. It can fail due to flutter, which is a dynamic instability of
the fabric caused by wind-induced vibrations.

Q.39 Match the brick masonary bond type in Group I with their corresponding illustrations in Group II
Group I Group II
(P) Rat trap 1.
Hemant Vilas Parulekar 157

(Q) English 2. (A) P-2, Q-1, R-4, S-5


(B) P-4, Q-1, R-2, S-3
(C) P-2, Q-5, R-1, S-3
(D) P-4, Q-1, R-2, S-5
(R) Flemish 3.

(S) Stretcher 4.

5.

Ans:- (D)
Explanation:
Masonary Rat Trap English
bond type
Type

Masonary Flemish Stretcher


bond type
Buildings
158 GATE Architecture and Planning: Comprehensive Question Bank

Q.40 Match the characteristics in Group I with the type of settlements in Group II as given in URDPFI
Guidelines 2015. Government of India
(A) P-4, Q-3, R-1, S-5 Group I Group II
(B) P-3. Q-5, R-1, S-4 (P) Zones of transition from rural to 1. Counter Magnets
(C) P-3, Q-5, R-2, S-4 urban land uses located between the
outer limits of urban and regional
(D) P-3, Q-4, R-1, S-2 centres and rural environment.
(Q) Towns having potential for 2. Satellite Towns
investment and development:
identified on the basis of their inter
aerial relationship with the regional
nodal centre
(R) Settlements that are growing 3.Peri-Urban Towns
sub-nodal centres but located out of
the direct functionally linked areas
of the growth node I nodal centre in
the region
(S) Located near or within 4. Priority Towns
reasonable distance, well connected
by transportation route of the
growth node of metropolitan city
and dependent on growth node
largely for employment.
5. Statutory Towns
Ans:- (D)
Explanation:
Counter Magnets: Settlements that are growing sub-nodal centres but located out of the direct
functionally linked areas of the growth node/nodal centre in the region.
Satellite Towns: Located near or within reasonable distance, well connected by transportation route of
the growth node of metropolitan city and dependent on growth node largely for employment.
Peri-Urban Towns: Zones of transition from rural to urban land uses located between the outer limits
of urban and regional centres and rural environment.
Priority Towns: Towns having potential for investment and development: identified on the basis of their
inter aerial relationship with the regional nodal centre.
Statutory Towns: Towns which are notified under state/UT specific laws and have a municipal
corporation, municipality, nagar panchayat or notified area committee.
Based on these definitions, we can match the characteristics in Group I with the type of settlements in
Group II as follows:
(P) Zones of transition from rural to urban land uses located between the outer limits of urban and
regional centres and rural environment. -> 3. Peri-Urban Towns
(Q) Towns having potential for investment and development: identified on the basis of their inter aerial
relationship with the regional nodal centre -> 4. Priority Towns
(R) Settlements that are growing sub-nodal centres but located out of the direct functionally linked areas
of the growth node I nodal centre in the region -> 1. Counter Magnets
(S) Located near or within reasonable distance, well connected by transportation route of the growth
node of metropolitan city and dependent on growth node largely for employment. -> 2. Satellite Towns
Hemant Vilas Parulekar 159

Q.41 A population of 2500 persons requires a minimum area of 3000 m² for primary schools. For the
population in four different sectors given in the table below, the Sector having maximum shortage
of school area per person is ____
Sector population Number of existing Schools Existing of each school m2
1 20000 5 2000
2 15000 4 4500
3 12500 2 2500
4 10000 4 1500
Ans:- Sector 3
Solution:
To find the sector having the maximum shortage of school area per person, we need to calculate the
school area per person for each sector and compare them. The school area per person is the ratio of the
total school area to the population of the sector. The lower the ratio, the higher the shortage.
Using the data given in the table, we can calculate the school area per person for each sector as follows:
Sector 1: (5×2000)/20000=0.5 m2 per person Sector 2: (4×4500)/15000=1.2 m2 per person
Sector 3: (2×2500)/12500=0.4 m2 per person Sector 4: (4×1500)10000=0.6 m2 per person
The sector with the lowest school area per person is Sector 3, with only 0.4 m² per person. Therefore,
the answer is Sector 3.
Q.42 Number of married couples in a household along with number of rooms (for a household) are given
in the table. Assuming each married couple needs one separate room, the total number of additional
rooms required for them is _____
No. of married couples in a household Number of households with
1 room 2 rooms 3 rooms
0 2500 450 100
1 4700 3000 2000
2 3600 5500 1100
3 432 750 400

Ans:- 5214 to 5214


Solution :
A) Additional room required for households having 2 married couples = 3600
B) Additional room required for households having 3 married couples = (432 x 2) + 750
Adding (A) & ((B), we get = 3600 + (432 x 2) + 750 = 5214
Q.43 In a residential complex, the central play area is to be converted as a detention pond for storm
water management. For a 24 hour rainfall event of 100 mm, 100% storm water of central play area
and 70% storm water run-off from rest of the complex is to be held at the detention pond. Area
distribution in the residential complex is given in the table.
Type Areas Run-off Coefficient
Apartment blocks 1250 0.80
Central Play Area 150 0.60
Other permeable areas 200 0.70
Other impermeable areas 400 0.90
The required depth of the detention pond (in mm) is______
160 GATE Architecture and Planning: Comprehensive Question Bank

Ans:- 760
Solution:
Type Area in m2 Runoff 100m*100% 100mm* 70 (C)+(D)
(A) Coefficient (B) A*B=(C) %*A*B=(D)
Apartment blocks 1250 0.8 - 70000 70000
Central play Area 150 0.6 9000 - 9000
Other permeable areas 200 0.7 - 9800 9800
Other impermeable areas 400 0.9 - 25200 25200
Total 114000
Since the central play area is proposed to be converted in detention pond
Depth of the detention pond = 114000 / 150 = 760mm
Q.44 In the plot shown below, ‘S1’ and ‘S2’ are two non-directional point
sources, having a sound intensity level of 95 dB and 60 dB
respectively at a distance of 1m from each point source.
Considering free field conditions, the effective sound intensity
level at the receiver location ‘R’ (in dB. rounded off to two decimal
places) is ____

Ans:- 75.00 to 79.00


Solution:
For time being, suppose that there is no sound source at S2. Because there will be almost no
impact of source S2 on S1 as there is a very big difference of 35 dB. For example of two source of 95 dB
are added, the resultant will be 95 + 3 = 98dB. In the similar way, if two sources of 95 dB and 90 dB are
added, the resultant will be approx 96 dB only.
Calculation of sound source S1:
Let Intensity of sound at 1 m be I1 and at 9m be I2.
So, I1/I2 = 92/12 ……………………………. A
As per question,
95dB = 10LogI1/I0 …………………………. B
and,
x dB = 10LogI2/I0…………………………... C
B – C,
95 – x = 10 (Log I1/I0 – Log I2/I0 )
95 – x = 10 (Log I1/I2 )
95 – x = 10 Log 81/1
95 - x = 19.1
x = 75.9 dB
Hemant Vilas Parulekar 161

Q.45 A room measures 5m x 10m x 3m (LxBxH). Consider the following conditions,


Total solar radiation incident on the roof surface= 800 W/m²
Outdoor air temperature= 40°C
Outside film coefficient of the roof surface= 18 W /m²
The outdoor mean radiant temperature is equal to outdoor air temperature
The minimum reduction required in solar absorptance of the roof (rounded off to
two decimal places) to achieve a 20° reduction in sol-air temperature is ____.
Ans:- 0.43 to 0.47
Solution:
To find the minimum reduction required in solar absorptance of the roof, we need to use the formula
for sol-air temperature:

where Tsol−air is the sol-air temperature, To is the outdoor air temperature, a is the solar absorptance, I
is the global solar irradiance, ΔQir is the extra infrared radiation, and ho is the outside film coefficient.
Given the conditions in the question, we can simplify the formula as follows:

We want to achieve a 20° reduction in sol-air temperature, so we need to solve for a when Tsol−air=20:

a=−0.45
This means that the roof needs to have a negative solar absorptance, which is not possible in reality.
Therefore, the minimum reduction required in solar absorptance of the roof is 100%, meaning that the
roof needs to reflect all the incoming solar radiation. This can be achieved by using a white or highly
reflective material for the roof.
Q.46 The activity duration, early start, early finish, late start and late finish of the three activities ‘P’, ‘Q’
and ‘R’ are shown in the following figure. The independent float of activity ‘Q’ is
162 GATE Architecture and Planning: Comprehensive Question Bank

Ans:- 1 to 1
Solution:
Let Early start = Es,
Early finish = Ef,
Late start = Ls,
Late finish = Lf
For activity P; Esp= 2, Efp= 9, Lsp= 8, Lfp= 15
For activity Q; Esq = 9, Efq= 15, Lsq= 21, Lfq= 27
Estimated completion time for activity Q; Dq= 6
Since, independent float for activity Q is asked
Independent Float (If) = Efq- Lsp- Dq= 15 - 8 - 6 = 1 Answer
Q.47 A square based regular pyramid has all sides equal to 10 units. Its height (in the same units, rounded
off to two decimal places) is______
Ans:- 7.00 to 7.10
Solution:
Given,
AB = BC = CD = DA = AF = BF = CF = DF = 10 units
Let’s consider height of the pyramid be ‘h’ (EF)
BD = (BA² + AD²)-2
= (102 + 102)-2
= 10(2)-2
BE = 1/2 BD = 1/2 [10(2)-2] = 5(2-2)
Now, EF = h = (BF2 - BE2)-2
= {102 - [5(2-2)]2}-2
= {100 - 50}-2
= 50-2
= 5(2-2)
= 5 x 1.414 = 7.071, say 7.07 Answer
Q.48 A parking area measuring 52 m x 4.67 m is approached through a driveway as shown in the given
illustration. The parking is designed at an angle of 30° with the parking bay of 2.5 m x 5m.

The number of cars that can be parked in the designated parking area considering no car overshoots
the length of the parking area is ______
Hemant Vilas Parulekar 163

Ans:- 9 to 10
Solution:
Given, L = 52.00 m, Let the number of cars be ‘N’, L = 0.58 + 5N
52 = 0.58 + 5N
5N = 51.42
N = 10.28, say 10 cars Answer
Q.49 Plan and section of an isolated foundation is given below. The volume of concrete up to Ground
Level (GL) (in m3, rounded off to two decimal places) is _____

Ans:- 3.10 to 3.40


Solution:
Let’s bifurcate the foundation in three parts
as part ‘A’, ‘B’ and ‘C’
Volume of part ‘A’ (l = 2, b = 2.5, h = 0.40)
= l x b x h = 2 x 2.50 x 0.40 = 2.00 m3 -------------- (i)
Volume of part ‘B’ (since it’s a frustum of rectangular pyramid)
To deduce height ‘X’ of pyramid
=> PQ / PR = QU / RS => X / (X + 0.50) = 0.2 / 1.0 => X = 0.2X + 0.1 => X = 0.125
Therefore, volume of part ‘B’ (L=2, W=2.5, H=0.7, l=0.4,
w=0.5, h=0.125)
= 1/3 (LWH - lwh) = 1/3 (2 x 2.5 x 0.7 - 0.4 x 0.5 x 0.125) =
1/3 (3.5 - 0.25) = 1/3 x 3.475 = 1.16 m³- (ii)
Volume of part ‘C’ (l = 0.4, b = 0.5, h = 1.10)
= l x b x h = 0.4 x 0.5 x 1.10
= 0.22 m³ --------------- (iii)
Adding (i), (ii) and (iii), we get = 2.00 + 1.16 + 0.22 = 3.38 m³ Answer
164 GATE Architecture and Planning: Comprehensive Question Bank

Q.50 Top floor of a 25-story building is using a flush valve system with a minimum fixture pressure of
1.0 kg/cm2, If static pressure increases by 0.3 kg/cm² per meter length and friction loss is zero, then
height of bottom of the water tank from the top fixture (in meter, rounded off to two decimal places)
is ____
Ans:- 3.30-3.40
Solution:
The minimum fixture pressure at the top floor is given as 1.0 kg/cm². The static pressure increases by
0.3 kg/cm² per meter length. We can use these values to calculate the height of the bottom of the water
tank from the top fixture.
Q.51 In a single phase alternate current circuit, an electric lamp is rated 100 watts. If 220 volts is
impressed on it and the power factor is 0.85. the energy (in watthour, rounded off to one decimal
place) delivered in an hour is ___
Ans:- 100 watts
Solution:
The energy delivered by an electric lamp in an hour is simply its power rating, as the power of an
appliance indicates the energy used per unit time.
Given that the electric lamp is rated 100 watts, this means it uses 100 watt-hours of energy in one hour.
Therefore, the energy delivered by the lamp in an hour is 100.0 watt-hours. Please note that the voltage
and power factor are not needed to calculate the energy consumption in this case, as the power rating of
the lamp already takes these factors into account.
Q.52 A simply supported RCC beam of cross section 0.4 m x 0.6 m covers a span of 8 m. It is subjected
to a uniformly distributed load of 30 kN/m. If the unit weight of concrete is 24 kN/m³, the tensile
stress (in N/mm², rounded off to two decimal places) at the bottom of the beam at mid-span
is _____
Ans:- 11.80 to 12.00
Solution:
Calculating total UDL (Uniformly Distributed Load)
Volume of RCC beam = 0.4m x 0.6m x 8m
Dead load of RCC beam = 24kN/m³ x (0.4m x 0.6m x 8m)
UDL due to self-weight of RCC beam = Load due to self-weight / Length
= [24kN/m³ x (0.4m x 0.6m x 8m)] / 8m = 5.76 kN/m
So, Total UDL = Subjected UDL + Self weight UDL = 30kN/m + 5.76kN/m = 35.76kN/m
Maximum Moment = WL²/8 = [35.76kN/m * (8m) ²]/8 = 286kN/m
Tensile Stress = My/I where, M = Bending moment,
y = distance from mid-point (CG), I = moment of inertia
Given, M = 286kN / m, y = 0.3 meter, I = bd³/12 = 0.4m * (0.6m) ³ / 12 = 0.0072 m⁴
Tensile Stress = My/I = [(286kN/m) * (0.3m)] / 0.0072 m⁴ = 11916.67 kN/m² = 11.92N/mm²
Hemant Vilas Parulekar 165

Q.53 A basement wall resists lateral pressure exerted by


soil and water. The soil pressure amounts to
4.5 kN/m² for every meter of depth below Ground
Level (GL). The sub-soil water level is 1.0 m below
GL and hydrostatic pressure of water is 9.8 kN /m²
for every meter of depth below GL. The total
lateral pressure (in kN/m², rounded off to one
decimal place) exerted on the wall 2 m below GL
is ____
Ans:- 14.3 to 18.80
Solution:
Calculation of pressure at 2 m below the GL is required
Total lateral pressure = Soil Pressure of 2 m depth + Hydrostatic Pressure of 1 m depth =
= (4.5kN x 2) + (9.8 kN x 1) = 18.80 kN/m² Answer
Q.54 Assuming that the population growth trend given in the table will continue, the population (in
persons) for the year 2031 will be _____
S.No Year Population in person
1 1981 1,30,440
2 1991 1,69,572
3 2001 2,20,444
4 2011 2,86,577
Ans:- 484315 to 484316
Solution:
Usually population grows in two patterns: Arithmetic progression or Geometric progression.
S.No Year Population Increase in Population Increase in population
in person (arithmetic progression) (geometric progression)
1 1981 1,30,440
2 1991 1,69,572 Δ1=1,69,572-1,30,440=39,142 r1=1,69,572/1,30,440=1.30
3 2001 2,20,444 Δ1=2,20,444-1,69,572=50,872 r1=2,20,444/1,69,572=1.30
4 2011 2,86,577 Δ1=2,86577-2,20,444=66,133 r1=2,86577/2,20,444=1.30
Population in 2021 = 286577 x 1.3
Population in 2031 = 286577 x 1.3 x 1.3 = 4,84,315 Answer
Q.55 A developer would like to select a residential plot of 3000 m2
Area Ground FAR
for group housing in a city. Different options with varying
Coverage
development controls are given. In every group housing plot,
1 30 1.5
15% of the Floor Area Ratio (FAR) over and above the
maximum permissible FAR has to be utilized for 2 20 2.0
Economically Weaker Section (EWS) units. The maximum 3 40 2.0
built-up area (in m2) available from the options given below 4 15 3.0
is_________
166 GATE Architecture and Planning: Comprehensive Question Bank

Ans:- 10350 to 10350


Solution:
Since all the areas, i.e., ‘1’, ‘2’, ‘3’ and ‘4’ do have equal area admeasuring 3000 m2. It is clear that area ‘4’,
having Floor Area Ration (FAR) of 3.00 offers area for construction.
Here, in this problem, ground coverage doesn’t have any importance at all.
The total FAR for area ‘4’ = 3000 x 3.0 = 9000 m2 --------------------------- (i)
As given in the problem, we need to provide additional 15% of the FAR over and above
the maximum FAR for economically Weaker Section (EWS) units
Using (i) we get, 9000 x 1.15 = 10,350 m2Answer
GATE QUESTION PAPER 2019
General Aptitude (GA)

Q. 1 – Q. 5 carry one mark each.


Q.1 The fishermen, ______ the flood victims owed their lives, were rewarded by the government.
(A) whom (B) to which (C) to whom (D) that
Ans:- (C)
Q.2 Some students were not involved in the strike.
If the above statement is true, which of the following conclusions is/are logically necessary?
1. Some who were involved in the strike were students.
2. No student was involved in the strike.
3. At least one student was involved in the strike.
4. Some who were not involved in the strike were students.
(A) 1 and 2 (B) 3 (C) 4 (D) 2 and 3
Ans:- (C)
Q.3 The radius as well as the height of a circular cone increases by 10%. The percentage increase in its
Volume is _______________.
(A) 17.1 (B) 21.0 (C) 33.1 (D) 72.8
Ans:- (C)
Solution:
To find the percentage increase in the volume of a cone, we can use the formula:
Percentage increase= (New volume−Old volume/Old volume) × 100
The old volume of the cone is given by:
V=1/3πr2h
Where r is the radius and h is the height.
The new volume of the cone is given by:
Here 1.1r and 1.1h are the increased radius and height, respectively.
Substituting these values into the formula, we get:
Percentage increase=[1/3π (1.1r)2(1.1h)−1/3πr2h]/1/3πr2h ×100
Simplifying, we get:
Percentage increase=[(1.1)3−1]/1×100
Percentage increase=(1.331−1)×100
Percentage increase=0.331×100
Percentage increase=33.1
168 GATE Architecture and Planning: Comprehensive Question Bank

Q.4 Five numbers 10, 7, 5, 4 and 2 are to be arranged in a sequence from left to right following the
directions given below:
1. No two odd or even numbers are next to each other.
2. The second number from the left is exactly half of the left-most number.
3. The middle number is exactly twice the right-most number.
Which is the second number from the right?
(A) 2 (B) 4 (C) 7 (D) 10
Ans:- (C)
Explanation:
To find the answer, we can follow the directions given below and eliminate the options that do not
satisfy them.
No two odd or even numbers are next to each other.
This means that the sequence must alternate between odd and even numbers. Therefore, the first, third,
and fifth numbers must be even, and the second and fourth numbers must be odd. This eliminates
option (A)2 , as it is an even number in the second position from the right.
The second number from the left is exactly half of the left-most number.
This means that the left-most number must be an even number that is divisible by 2, and the second
number from the left must be an odd number that is half of it. The only pair of numbers that satisfy
this condition are 10 and 5. Therefore, the left-most number must be 10 and the second number from
the left must be 5. This eliminates option (D) 10, as it is the left-most number, not the second number
from the right.
The middle number is exactly twice the right-most number.
This means that the right-most number must be an even number that is divisible by 2, and the middle
number must be an even number that is twice of it. The only pair of numbers that satisfy this condition
are 2 and 4. Therefore, the right-most number must be 2 and the middle number must be 4. This
eliminates option (B) 4, as it is the middle number, not the second number from the right.
The only remaining option is (C)7, which is the second number from the right. The sequence that
satisfies all the directions is:10 5 4 7 2
Q.5 Until Iran came along, India had never been_______in kabaddi.
(A) defeated (B) defeating (C) defeat (D) defeatist
Ans:- (A)
Q. 6 – Q. 10 carry two marks each.
Q.6 Since the last one year, after a 125 basis point reduction in repo rate by the Reserve Bank of India,
banking institutions have been making a demand to reduce interest rates on small saving schemes.
Finally, the government announced yesterday a reduction in interest rates on small saving schemes
to bring them on par with fixed deposit interest rates. Which one of the following statements can
be inferred from the given passage?
(A) Whenever the Reserve Bank of India reduces the repo rate, the interest rates on small saving
schemes are also reduced.
Hemant Vilas Parulekar 169

(B) Interest rates on small saving schemes are always maintained on par with fixed deposit interest
rates.
(C) The government sometimes takes into consideration the demands of banking institutions before
reducing the interest rates on small saving schemes.
(D) A reduction in interest rates on small saving schemes follow only after a reduction in repo rate by
the Reserve Bank of India.
Ans:- (C)
Explanation:
The passage is about the recent decision of the government to reduce interest rates on small saving
schemes in India, following a demand from banking institutions and a reduction in repo rate by the
Reserve Bank of India. The passage does not imply that these events always happen together or in a
fixed order. Therefore, options (A) and (D) are not correct.
The passage also does not imply that interest rates on small saving schemes are always maintained on
par with fixed deposit interest rates. This is just the current situation, which may change in the future.
Therefore, option (B) is not correct.
The only option that can be inferred from the passage is (C), which states that the government
sometimes takes into consideration the demands of banking institutions before reducing the interest
rates on small saving schemes.
Q.7 In a country of 1400 million population, 70% own mobile phones. Among the mobile phone
owners, only 294 million access the Internet. Among these Inter net users, only half buy goods
from e-commerce portals. What is the percentage of these buyers in the country?
(A) 10.50 (B) 14.70 (C) 15.00 (D) 50.00
Ans:- (A)
Solution:
To solve this problem, we need to use the percentage formula:
Percentage = (Value / Total value) x 100
The value in this case is the number of people who buy goods from e-commerce portals, which is half
of the internet users, i.e. 294/2 = 147 million. The total value is the population of the country, which is
1400 million.
Therefore, the percentage of these buyers in the country is:
Percentage = (147 / 1400) x 100 Percentage = 0.105 x 100 Percentage = 10.5
The correct answer is (A) 10.50.
Q.8 The nomenclature of Hindustani music has changed over the centuries. Since the medieval period
dhrupad styles were identified as baanis. Terms like gayaki and baaj were used to refer to vocal
and instrumental styles, respectively. With the institutionalization of music education the term
gharana became acceptable. Gharana originally referred to hereditary musicians from a particular
lineage, including disciples and grand disciples.
Which one of the following pairings is NOT correct?
(A) dhrupad, baani (B) gayaki, vocal
(C) baaj, institution (D) gharana, lineage respectively.
170 GATE Architecture and Planning: Comprehensive Question Bank

Ans:- (C)
Explanation:
• Dhrupad: A genre of Hindustani classical music that is characterized by a long and elaborate alap,
followed by a composed bandish. Dhrupad is one of the oldest forms of Hindustani music and is
considered to be the purest expression of the raga.
• Baani: A style or school of dhrupad that is distinguished by the use of specific ornamentations,
vocal techniques, and compositions. There are four main baanis of dhrupad: Gauhar, Khandar,
Nauhar, and Dagur.
• Gayaki: A style or manner of singing that reflects the personality and preferences of the singer.
Gayaki can also refer to the vocal genre of Hindustani music, as opposed to the instrumental genre.
• Baaj:A style or technique of playing a musical instrument, especially a stringed instrument. Baaj
can also refer to the instrumental genre of Hindustani music, as opposed to the vocal genre.
• Gharana: A system of social organization that links musicians or dancers by lineage orapprenticeship,
and by adherence to a particular musical style. Gharana can also refer to the musical style itself,
which is influenced by the history, traditions, and preferences of the gharana members.
Q.9 Two trains started at 7AM from the same point. The first train travelled north at a speed of 80km/h
and the second train travelled south at a speed of 100 km/h. The time at which they were 540 km
apart is_________AM.
(A) 9 (B) 10 (C) 11 (D) 11.30
Ans:- (B)
Solution:
To solve this problem, we need to use the formula:
Distance = Relative speed x Time
The distance between the two trains is 540 km. The relative speed of the two trains is the sum of
their individual speeds, since they are moving in opposite directions. Therefore, the relative speed is
80 + 100 = 180 km/h. The time is the unknown variable that we need to find.
Plugging these values into the formula, we get:
540 = 180 x Time Time = 540 / 180 Time = 3 hours
This means that the two trains will be 540 km apart after 3 hours of travelling. Since they started at
7 AM, the time at which they were 540 km apart is 7 + 3 = 10 AM.
Q.10 “I read somewhere that in ancient times the prestige of a kingdom depended upon the number of
taxes that it was able to levy on its people. It was very much like the prestige of a head-hunter in
his own community.”Based on the paragraph above, the prestige of a head-hunter depended upon
(A) the prestige of the kingdom
(B) the prestige of the heads
(C) the number of taxes he could levy
(D) the number of heads he could gather
Ans:- (D)
Hemant Vilas Parulekar 171

ARCHITECTURE AND PLANNING (Q.1-55)


Q. 1 – Q. 25 carry one mark each.
Q.1 Which of the following commands in AUTOCAD is used to create 3D solid between various cross
sections?
(A) LOFT (B) MESH (C) XEDGES (D) PFACE
Ans:- (A)
Explanation:
LOFT: Creates 3D solid or surface by fitting a skin through a set of cross sections. The cross sections
can be open or closed curves, or points.
MESH: Creates a 3D polygon mesh object that can be used to model complex
shapes. The mesh can be created from scratch or converted from other objects.
XEDGES: Extracts edges from a 3D solid, surface, or mesh, and creates wireframe
geometry such as lines, arcs, circles, or splines.
PFACE: Creates a 3D mesh by specifying vertex coordinates and face definitions. The faces can be
three-sided or four-sided.
Q.2 Name the architect who criticized ornament in useful objects in his essay ‘Ornament and Crime’
(A) John Ruskin (B) H P Berlage
(C) Adolf Loos (D) Walter Gropius
Ans:- (C)
Explanation:
There are many famous quotes by famous architects that reflect their vision, philosophy, and inspiration.
Here are some of them:
1.“Architecture is the learned game, correct and magnificent, of forms assembled in the light.” - Le
Corbusier.
2.“Form follows function - that has been misunderstood. Form and function should be one, joined in
a spiritual union.” - Frank Lloyd Wright.
3.“Less is more.” - Ludwig Mies van der Rohe.
4.“God is in the details.” - Ludwig Mies van der Rohe.
5.“To create, one must first question everything.” - Eileen Gray.
6.“Architecture should speak of its time and place, but yearn for timelessness.” - Frank Gehry
7.“To provide meaningful architecture is not to parody history but to articulate it.” - Daniel Libeskind.
8.“As an architect you design for the present, with an awareness of the past, for a future which is
essentially unknown.” - Norman Foster.
9.“The desire to reach for the sky runs very deep in the human psyche.” - César Pelli.
10.“For me, architecture is a social act.” - David Adjaye.
172 GATE Architecture and Planning: Comprehensive Question Bank

Q.3 A sanitary landfill is provided with High Density Poly Ethylene (HDPE) lining along ground
surface. This is provided primarily to prevent
(A) Bleaching (B) Leaching
(C) Rodents (D) Plant growth
Ans:- (B)
Explanation:
A sanitary landfill is a method of waste disposal that involves burying the waste in a designated area and
covering it with soil and other materials. A sanitary landfill is designed to minimize the environmental
impact of waste and prevent the spread of diseases and pests.
HDPE lining is a type of geomembrane, which is a synthetic material that acts as a barrier to prevent
the movement of fluids or gases. HDPE lining is used along the ground surface of a sanitary landfill
to prevent the escape of leachate, which is the liquid that drains or leaches from the waste. Leachate
can contain harmful substances that can contaminate the soil, groundwater, and surface water. HDPE
lining is also resistant to chemical and corrosive agents that are present in the waste.
Q.4 Super-elevation of a road with pre-determined radius of curvature is primarily dependent on
(A) Altitude (B) Soil bearing capacity
(C) Traffic volume (D) Design traffic speed
Ans:- (D)
Explanation:
Super-elevation is the banking of a road surface, which
provides a transverse slope to the road surface to counteract
the lateral forces experienced by vehicles travelling around a
curve. The purpose of super-elevation is to allow vehicles to
negotiate a curve at higher speeds without tipping over, thus
making the road safer for drivers.
The design of super-elevation depends on several factors, such
as the radius of curvature, the coefficient of friction, the design
speed, and the width of the road. However, the primary factor
that determines the amount of super-elevation is the design
traffic speed, which is the maximum safe speed that can be maintained by most vehicles on a given
road section.
The design traffic speed affects the centrifugal force that acts on a vehicle moving along a curve. The
centrifugal force is proportional to the square of the speed and inversely proportional to the radius of
curvature. Therefore, higher speeds and smaller radii result in higher centrifugal forces, which need to
be balanced by higher super-elevation. The formula for calculating the super-elevation is:
e = (v2) / (gR)
Q.5 In a mono-centric urban model, land rent is expected to
(A) diminish as one moves towards the center (B) diminish as one moves away from the center
(C) remain constant across the whole urban area (D) be unrelated with distance from center
Hemant Vilas Parulekar 173

Ans:- (B)
Explanation:
A mono-centric urban model is a simplified representation of a city that assumes a single central
business district (CBD) that attracts all workers and consumers. The CBD is the most accessible and
desirable location in the city, and thus has the highest land rent. As one moves away from the CBD, the
land rent decreases due to the increasing transportation costs and decreasing accessibility.
Q.6 Fineness modulus of sand measures its
(A) Compressive strength (B) Grading according to particle size
(C) Bulking of sand (D) Ratio of coarse and fine sand
Ans:- (B)
Explanation:
Fineness modulus of sand is an index number that represents the average size of the sand particles. It
is calculated by performing sieve analysis with standard sieves and dividing the sum of the cumulative
percentages retained on each sieve by 100. Fineness modulus of sand indicates the coarseness or
fineness of the sand. A lower fineness modulus means finer sand, while a higher fineness modulus
means coarser sand.
Q.7 The spherical surface of the geodesic dome comprises of
(A) Equilateral triangles of various sizes (B) Isosceles triangles of various sizes
(C) Equilateral triangles of uniform size (D) Isosceles triangles of uniform size
Ans:- (C)
Explanation:
A geodesic dome is a spherical structure composed of triangular or polygonal facets
that distribute the structural stress evenly. It was developed by American engineer and
architect R. Buckminster Fuller in the 20th century.
The spherical surface of a geodesic dome can be approximated by subdividing an icosahedron, which
is a polyhedron with 20 equilateral triangular faces, into smaller triangles. The smaller triangles are
then projected onto a sphere, creating a geodesic polyhedron. The number of subdivisions determines
the frequency of the geodesic dome. A higher frequency means more triangles and a smoother surface.
Q.8 The abrupt change or junction between two ecological zones is termed as
(A) Ecological Niche (B) Ecosystem
(C) Ecotype (D) Ecotone
Ans:- (D)
Explanation:
Ecotone is a zone of the junction where two communities meet and integrate. It may be narrow or
wide, and it may be local or regional.
The other options are not correct, as they have different meanings:
An ecological niche is the role and position a species has in its environment; how it meets its needs for
food and shelter, how it survives, and how it reproduces.
An ecosystem is a biological community of interacting organisms and their physical environment.
An ecotype is a distinct form or race of a plant or animal species occupying a particular habitat.
174 GATE Architecture and Planning: Comprehensive Question Bank

Q.9 Complementary colours in a Munsell pigment colour wheel refers to


(A) Colours in alternate positions (B) Colours opposite to one another
(C) Colours opposite to each other (D) A pair of secondary colours
Ans:- (B)
Explanation:
The Munsell pigment colour wheel is a circular diagram that shows the hues
of colours based on three properties: hue, value, and chroma. Hue is the
basic colour, value is the lightness or darkness, and chroma is the intensity
or purity.
Complementary colours are colours that are opposite to each other on the hue circle, and create a strong
contrast when placed next to each other. For example, red and green are complementary colours, as
are blue and orange, and yellow and purple. Complementary colours can be used to create harmony,
balance, and interest in art and design.
Q.10 The closing syntax, for an executable command line in C or C++ program, is
(A) : (B) , (C) ; (D) .
Ans:- (C)
Explanation:
Syntax is the set of rules that define how to write valid statements and expressions in a programming
language. Syntax also includes the use of punctuation marks, such as semicolons, commas, colons, and
periods, to separate and end different parts of the code.
The closing syntax, for an executable command line in C or C++ program, is a semicolon (;). A
semicolon is used to mark the end of a statement, which is a unit of code that performs a specific action
or calculation. A statement can be a variable declaration, an assignment, a function call, a control
structure, or a return statement. A semicolon tells the compiler that the statement is complete and
ready to be executed.
Q.11 The term ‘Necropolis’ refers to
(A) Organically growing settlement (B) Origin of a settlement
(C) A dead settlement (D) Merging of two settlements
Ans:- (C)
Explanation:
The term Necropolis refers to a cemetery, especially a large one belonging to an ancient city. The word
is derived from the Greek words nekros meaning “dead person” and polis meaning “city”
Q.12 Which of the following projection types is adopted in the Universal Transverse Mercator (UTM)?
(A) Spherical (B) Conical (C) Planar (D) Cylindrical
Ans: (D)
Explanation:
The Universal Transverse Mercator (UTM) is a map projection system that is used
to assign coordinates to locations on the surface of the Earth. It is based on the
Transverse Mercator projection, which is a cylindrical map projection.
Hemant Vilas Parulekar 175

Q.13 The ingredient to be added to produce Aerated Cement Concrete, is


(A) Aluminium (B) Calcium Chloride
(C) Gypsum (D) Sulphur
Ans:- (A)
Explanation:
The ingredient added to produce Aerated Cement Concrete is a mixture of quartz sand, calcined
gypsum, lime, Portland cement, water and aluminium powder. The mixture is cured under heat and
pressure in an autoclave.
Q.14 The cause of short column effect, during seismic occurrence, is due to
(A) Centralized rupture of column (B) Tearing of reinforcement bars
(C) Buckling of column (D) Stress concentration
Ans:- (C)
Short columns are designed as structural members having a
high stiffness with low ductility. During an earthquake, short
columns, not adequately designed for such a large force, are
often the first members to fail. The damage in these short
columns is often in the form of X shaped cracking – this type
of damage of columns is due to buckling/shear failure. Short
Column effect caused by partial infills in framed structures
may be prevented by adopting adequate separation gaps. These
members should be designed so that movement is allowed during an earthquake.
Q.15 The solar protection system consisting of fixed slats or grids, outside a building façade in front of
openings, is known as
(A) Brise-soleil (B) Solarium
(C) Malqaf (D) Trombe wall
Ans:- (A)
Terms Brise-soleil Solarium
Explanation Brise-soleil is a French term that refers Solarium is a term that can refer to
to an architectural feature of a building different things depending on the
that reduces heat gain within that context. Here are some possible
building by deflecting sunlight. It is meanings of the term:
typically a device, such as a perforated A solarium can be a room equipped with
screen or louvres, for shutting out direct sunlamps or sunbeds which can be used
or excessive sunlight. The system allows to acquire an artificial suntan.
low-level sun to enter a building in the
mornings, evenings and during winter A solarium can also refer to a glass-
but cuts out direct light during summer. enclosed porch or room that is exposed
to the sun’s rays, as at a seaside hotel or
for convalescents in a hospital.
In architecture, a solarium can be an
additional room with walls and a roof
that are completely made of glass.
176 GATE Architecture and Planning: Comprehensive Question Bank

Terms Brise-soleil Solarium


Image

Terms Malqaf Trombe wall


Explanation Malqaf is an architectural element that A Trombe wall is a passive solar building
has been used for thousands of years design strategy that adopts the concept of
in countries with severe hot climates. indirect-gain, where sunlight first strikes a
It is also known as a windcatcher, solar energy collection surface in contact
wind tower, wind scoop, or Badgir. with a thermal mass of air. The Trombe
The malqaf is a part of a complete wall is a massive equator-facing wall that
ventilation system, consisting of a is painted a dark color in order to absorb
large shaft rising high above the roof of thermal energy from incident sunlight and
a building. The system of ventilation covered with a glass on the outside with
developed depends primarily on-air an insulating air-gap between the wall and
movement by pressure differential, the glaze. The sunlight absorbed by the
but also secondarily on-air movement mass is converted to thermal energy (heat)
by convection. The malqaf creates and then transferred into the living space.
different pressure zones between Trombe walls are also named mass walls,
inside and outside. The air enters solar wall, or thermal storage wall.
after purifying and moisturized, then
been pushed out opening
Image

Q.16 The Indian property inscribed by UNESCO on the World Heritage List in the last year is
(A) Mattanchery Palace, Ernakulam (B) The Victorian Gothic and Art Deco Ensembles of Mumbai
(C) Ancient Buddhist Site, Sarnath (D) Mughal Gardens in Kashmir
Ans:- (B)
Explanation:
The Victorian Gothic and Art Deco Ensembles of Mumbai. According to the Wikipedia article on
the List of World Heritage Sites in India, this site was inscribed by UNESCO in 2023, along with
Santiniketan and the Sacred Ensembles of the Hoysalas. The Victorian Gothic and Art Deco Ensembles
of Mumbai consist of 94 buildings that showcase the architectural styles of the 19th and 20th centuries.
They include the Chhatrapati Shivaji Terminus, the Bombay High Court, the Rajabai Clock Tower,
the Eros Cinema, and the Cricket Club of India. These buildings reflect the cultural and economic
influence of Mumbai as a former colonial city and a modern metropolis.
Hemant Vilas Parulekar 177

Cultural Sites (38)


Year Site Description
1983 Agra Fort A historic fort in Agra, Uttar Pradesh, known for its
impressive Mughal architecture.
1983 Ajanta Caves Rock-cut Buddhist cave monuments in Maharashtra,
famous for their paintings and sculptures.
1983 Ellora Caves A group of Hindu, Buddhist, and Jain rock-cut
temples in Maharashtra, notable for their monumental
architecture.
1983 Taj Mahal An iconic mausoleum in Agra, Uttar Pradesh, built by
Mughal Emperor Shah Jahan in memory of his wife
Mumtaz Mahal.
1984 Sun Temple, Konârak A 13th-century Hindu temple in Odisha, dedicated
to the sun god Surya and known for its exquisite
architecture.
1984 Group of Monuments at A collection of 7th and 8th-century sanctuaries in Tamil
Mahabalipuram Nadu, known for their rock-cut caves and temples.
1985 Kaziranga National Park A national park in Assam, famous for its one-horned
rhinoceroses and diverse wildlife.
1985 Keoladeo National Park A bird sanctuary in Rajasthan, known for its avian
diversity, especially migratory birds like the Siberian
crane.
1985 Manas Wildlife Sanctuary A biodiversity hotspot in Assam, home to a variety
of wildlife, including the Bengal tiger and the Indian
rhinoceros.
1986 Churches and Convents of Goa A set of religious monuments in Goa, built during
the Portuguese colonial era, notable for their baroque
architecture.
1986 Fatehpur Sikri A city in Uttar Pradesh, founded by Emperor Akbar,
known for its well-preserved Mughal architecture.
1986 Group of Monuments at Hampi Ruins of the ancient Vijayanagara Empire in Karnataka,
notable for their Dravidian architecture.
1986 Group of Monuments at A complex of Hindu and Jain temples in Karnataka,
Pattadakal representing Chalukya architecture.
1986 Khajuraho Group of A group of medieval Hindu and Jain temples in Madhya
Monuments Pradesh, famous for their erotic sculptures.

1987 Elephanta Caves Rock-cut temples on Elephanta Island near Mumbai,


Maharashtra, known for their sculpted caves dedicated
to Lord Shiva.
1987 Great Living Chola Temples A group of temples in Tamil Nadu, built by the Cholas,
known for their architectural grandeur.
1987 Sundarbans National Park A large mangrove forest area in West Bengal, home to
the Royal Bengal Tiger.
178 GATE Architecture and Planning: Comprehensive Question Bank

1988 Nanda Devi and Valley of Two adjacent national parks in Uttarakhand, known for
Flowers National Parks their alpine flowers and biodiversity.
1989 Buddhist Monuments at Sanchi A group of Buddhist monuments in Madhya Pradesh,
famous for their stupas, monasteries, and Ashokan
pillar.
1993 Humayun’s Tomb, Delhi A Mughal-era tomb in New Delhi, known for its
architectural elegance and garden squares.
1993 Qutb Minar and its Monuments, A historical monument in Delhi, known for the Qutb
Delhi Minar, a 73-meter tall tower.
1999 Mountain Railways of India Includes Darjeeling Himalayan Railway, Nilgiri
Mountain Railway, and Kalka-Shimla Railway, known
for their historical and engineering significance.
2002 Mahabodhi Temple Complex at The site in Bihar where Buddha is said to have attained
Bodh Gaya enlightenment.
2003 Rock Shelters of Bhimbetka An archaeological site in Madhya Pradesh, showcasing
prehistoric cave paintings.
2004 Champaner-Pavagadh A historical park in Gujarat, known for its cultural
Archaeological Park heritage and archaeological remains.
2004 Chhatrapati Shivaji Terminus An historic railway station in Mumbai, Maharashtra,
(formerly Victoria Terminus) known for its Gothic Revival architecture.
2007 Red Fort Complex A historic fort in Delhi, an example of Mughal
architecture, and a symbol of India’s independence.

2010 The Jantar Mantar, Jaipur A collection of nineteen architectural astronomical


instruments in Jaipur, Rajasthan, built by the Rajput
king Sawai Jai Singh II.
2014 Rani-ki-Vav (the Queen’s An intricately constructed stepwell in Gujarat, famous
Stepwell) at Patan, Gujarat for its size and sculptural treatment.
2016 Archaeological Site of Nalanda The remains of a major Buddhist monastery and an
Mahavihara at Nalanda, Bihar ancient center of learning in Bihar.
2016 The Architectural Work of Le A collection of 17 sites across seven countries, which
Corbusier, an Outstanding includes the Capitol Complex in Chandigarh, India.
Contribution to the Modern
Movement
2017 Historic City of Ahmadabad The first city in India to be inscribed as a UNESCO
World Heritage City, known for its ancient heritage and
architecture.
2018 Victorian Gothic and Art Deco A collection of 19th century Victorian Gothic buildings
Ensembles of Mumbai and 20th century Art Deco buildings in Mumbai,
Maharashtra.
2019 Jaipur City, Rajasthan The ‘Pink City’ of Jaipur is known for its iconic
architectural legacy and vibrant culture.
2021 Dholavira: a Harappan City One of the most remarkable and well-preserved urban
settlements in South Asia from the Indus Valley
Civilization.
Hemant Vilas Parulekar 179

2021 Kakatiya Rudreshwara A 13th-century temple, known for its architectural


(Ramappa) Temple, Telangana grandeur and intricate carvings, representing the
Kakatiya dynasty’s art.
2023 Sacred Ensembles of the Sacred Ensembles of the Hoysalas | Includes the
Hoysalas renowned Hoysala temples of Belur, Halebid and
Somanathpura in Karnataka, exemplifying the Hoysala
architecture.
2023 Santiniketan Founded by Rabindranath Tagore, Santiniketan is
an educational and cultural center in West Bengal,
reflecting the vision of its founder and the Indian
cultural renaissance of the early 20th century.

Natural Sites (07)


Year Site Description
1985 Kaziranga National Park A national park in Assam, famous for its one-horned
rhinoceroses and diverse wildlife.
1985 Keoladeo National Park A bird sanctuary in Rajasthan, known for its avian
diversity, especially migratory birds like the Siberian
crane.
1985 Manas Wildlife Sanctuary A biodiversity hotspot in Assam, home to a variety
of wildlife, including the Bengal tiger and the Indian
rhinoceros.
1988, Nanda Devi and Valley of Two adjacent national parks in Uttarakhand, known for
2005 Flowers National Parks their alpine flowers and biodiversity.
1987 Sundarbans National Park A large mangrove forest area in West Bengal, home to
the Royal Bengal Tiger.

2012 Western Ghats A mountain range along the west coast of India, known
for its biodiversity and distinct ecosystems.
2014 Great Himalayan National Park A national park in Himachal Pradesh, known for its
Conservation Area biodiversity and scenic beauty.

Mixed Site (01)


Year Site Description
2016 Khangchendzonga National A national park in Sikkim, known for its unique diversity
Park of plains, valleys, lakes, glaciers and spectacular, snow-
capped mountains including the world’s third highest
peak, Mount Khangchendzonga.
Q.17 The typical features of Buddhist architecture are
(A) Mandapa, Chattri, Amalaka, Torana (B) Stambha, Torana, Vimana ,Harmika
(C) Vedika, Chattri, Torana, Harmika (D) Vedika , Stupa, Chaitya,Vimana
180 GATE Architecture and Planning: Comprehensive Question Bank

Ans:- (C)
Explanation: Please refer GATE-2009, Q.28 for more information. Similar question was asked in
GATE-2006, Q.21.
Q.18 Identify the Queen closure
(A) (B) (C) (D)

Ans:(A)
Explanation:
Please refer GATE-2009, Q.32 for more information.
Based on shape, bricks can be classified into many types, such
as rectangular, square, curved, perforated, hollow, ornamental,
etc. Rectangular bricks are the most common type of bricks.
They have a rectangular shape with six faces and eight corners.
They are easy to handle and lay. Square bricks are similar to
rectangular bricks, but have a square shape with four faces and
four corners. They are used for paving and flooring. Curved
bricks are also known as arch bricks. They have a curved shape
that is used to form arches, vaults, domes, etc. They are specially
made to suit the curvature of the structure. Perforated bricks are
also known as cellular bricks. They have holes or perforations
on one or more faces. They are used for ventilation, insulation,
and reduction of weight. Hollow bricks are also known as cavity
bricks. They have one or more cavities or hollow spaces inside
them. They are used for insulation, sound absorption, and reduction of weight. Ornamental bricks are
also known as decorative bricks. They have various shapes, sizes, colors, and patterns that are used to
enhance the aesthetic appeal of the structure. They are used for facing, cornices, coping, etc.
Q.19 Identify the role of Vermiculite in vertical landscapes
(A) Fertilizer (B) Holding material
(C) Binding material (D) Water retention element
Ans: Marks to all
Vermiculite is a type of mineral that is used as a soil amendment in gardening. It can improve the soil
texture, water retention, and nutrient availability for plants. Vermiculite is especially useful for vertical
landscapes, which are gardens that grow on walls or other vertical structures. Vertical landscapes require
a lightweight, porous, and water-holding material to support plant growth. Vermiculite can provide
these qualities, as well as essential minerals such as potassium, calcium, magnesium, and ammonium.
Vermiculite can also help to neutralize the soil pH, which is important for plant health. Vermiculite
can be mixed with other growing mediums, such as peat moss, perlite, or compost, to create a suitable
soilless potting mix for vertical landscapes. Vermiculite is safe, sterile, and inert, making it a versatile
and beneficial addition to any garden.
Hemant Vilas Parulekar 181

Q.20 Which of the following parameters is essential to estimate the Envelope Performance Factor (EPF)
of a building as per the Energy Conservation Building Code (ECBC), 2011?
(A) Building Type (B) Maximum humidity
(C) Maximum and minimum monthly temperature (D) Building occupancy duration
Ans:- (D)
Explanation:
The Energy Conservation Building Code (ECBC) is a set of guidelines and standards for designing
energy-efficient buildings in India1. Appendix D of the ECBC User Guide 2011 explains the Building
Envelope Trade-off Method, which is an alternative way of complying with the code requirements for
the building envelope.
The building envelope refers to the exterior facade of the building, which includes walls, windows,
roof, skylights, doors, and other openings. The envelope affects the thermal and visual comfort of
the occupants, as well as the energy consumption of the building. The ECBC prescribes minimum
performance levels for different envelope components, such as U-values, SHGC, and WWR, depending
on the climate zone and building type.
Q.21 The illumination level of a room is 300 lux and the efficacy of the lamps is 60 . The light power
Density (LPD) of the room in watt/m2_________?
Ans: 5
Solution:
Given, Illumination level of a room = 300 lux
Efficacy of the lamp = 60
Light Power Density (LPD) = Illumination of room ÷ Efficacy of the lamp
= 300 lux/60 = 5 Watt/m²
Q.22 The load on a RCC column is 150 kN. The soil bearing capacity is 80 kN/m². Assuming a factor
of safety of 1.2, the side of the square column footing is is____meter (rounded off to one decimal
place).
Ans:- 1.49 to 1.51
Solution:
Given, load on an RCC column = 150 kN
Factor of safety = 1.2;
Therefore, Factored total load = Factor of safety x Total load
= 150 x 1.2 = 180 kN
Soil bearing capacity = 80 kN
Hence, Area of column footing = (Factored total load/Soil bearing capacity)
= 180/80 = 2.25 m²
Hence, side of the square footing = √2.25 = 1.50 m.
182 GATE Architecture and Planning: Comprehensive Question Bank

Q.23 A room is separated by a partition wall. The average intensities of sound in the source and receiving
sides across the partition are 10ˉ⁴ W/m² and 10ˉ⁷ W/m² respectively. The transmission loss (TL) of
the partition wall is ____dB.
Ans:- 30
Solution:
Sound intensity is defined as sound per unit area perpendicular to direction of sound waves
represented by I. The SI unit of sound intensity is watt per square meter (W/m²). The
standard text is the measurement of sound intensity of the noise in the air at the location
of the listener as a sound energy quantity.
The sound intensity formula is given as,
I = P/A
Where, P = sound power, A = area
To measure Sound intensity level, you need to compare the given sound intensity with
the standard intensity.
Sound Intensity Level Formula is given as,
IL
Where I = sound intensity and IO = reference intensity
It is expressed in decibels (dB).
Io = 10ˉ¹² W/m²
To calculate the transmission loss (TL) of the partition wall, we can use the following formula

Where Wi is the power of the incident sound wave and Wt is the power of the transmitted sound wave.
The power of a sound wave is proportional to its intensity, so we can substitute the given values of
intensities in the formula:

Simplifying the expression, we get:

Applying the logarithm property, we get:

Since,
we get: TL=10×3
Therefore, the transmission loss (TL) of the partition wall is 30 dB.
Q.24 If the purchase price of 2BHK flat rises by 10 percent, the demand for such flats is observed to
decrease by 8 percent. The price elasticity of the housing demand for 2BHK flats is _______
(rounded off to one decimal place).
Hemant Vilas Parulekar 183

Ans:- 0.75 to 0.85


Solution:
Price elasticity measures the responsiveness of the quantity demanded or supplied of a good to a
change in its price. It is computed as the percentage change in quantity demanded—or supplied—
divided by the percentage change in price.
Elasticity can be described as elastic—or very responsive—unit elastic, or inelastic—not very responsive.
Elastic demand or supply curves indicate that the quantity demanded or supplied responds to price
changes in a greater than proportional manner.
An inelastic demand or supply curve is one where a given percentage change in price will cause a
smaller percentage change in quantity demanded or supplied.
Unitary elasticity means that a given percentage change in price leads to an equal percentage change in
quantity demanded or supplied.
Price elasticity of demand = Change in quantity/Change in demand= (8/10) % = 0.8
Q.25 ‘Threshold of enclosure ’created by vertical surfaces or series of vertical elements in an urban plaza,
represented by the ratio of height and distance, is given by an angle of ____degrees (rounded off to one
decimal place).
Ans: 26.0 to 30.0 [tan-1 (1/2)]
Solution: If the height of the façade of the building is ‘h’, and the the distance of the observer from the
building is ‘d’, then the experience of the observer would be as follows:
Experience Ratio (h/d)
Minimal Enclosure 1:3
Threshold of Enclosure 1:2
Full Enclosure 1:1
Loss of Enclosure 1:4
Since tan θ = perpendicular ÷ hypotenuse (by considering the above table)
Therefore, tan θ = (1/2)
Thus, θ = tan-1 (1/2) = 26°.56 ≈ 26°.6
Q. 26 – Q. 55 carry two marks each.
Q.26 Match the instruments in Column - I with the various types of surveying in Column - II and select
the appropriate option.
P-3, Q-4, R-2, S-5 Group I Group II
(B) P-2, Q-4, R-1, S-5 (P) Cross staff 1. Indoor wall to wall measurement
(C) P-5, Q-3, R-2, S-1 (Q) Alidade 2. Traversing
(R) Sextant 3. Chain survey
(D) P-3, Q-4, R-2, S-1
(S) Distomat 4. Plane table survey
5. Contour survey
184 GATE Architecture and Planning: Comprehensive Question Bank

Ans: (D)
Instrument Cross staff Alidade
Used for A cross staff is a simple instrument An alidade is a device that allows a surveyor
used to measure horizontal angles. It to sight a distant object and use the line of
consists of a metal or wooden rod with sight to perform a task. It is usually mounted
a cross at each end. It is used in chain on a plane table and used in plane table
surveying to set out right angles or surveying to draw a map of the area.
perpendicular lines.
Image

Instrument Sextant Distomat


Used for A sextant is a navigational instrument A distomat is an electronic instrument that
that measures the angle between a measures distance by emitting and receiving
celestial object and the horizon. It is electromagnetic waves. It is used in indoor
used in traversing to determine the wall to wall measurement to obtain accurate
position of a point by measuring the dimensions of rooms, halls, etc.
angles and distances from known
points.
Image

Q.27 Match the characteristics of settlement systems in Column - I with their corresponding theory/
rules in Column - II and select the appropriate option.
P-4,Q-1,R-2,S-5 Group I Group II
(B) P-2,Q-5,R-3,S-1 (P) Primacy of settlement 1. Central place theory
(C) P-3,Q-5,R-4,S-2 (Q) Settlement size and location 2. Gravity model
(R) Random component in location 3. Rank size rule
(D) P-3,Q-1,R-4,S-2
of settlements
(S) interaction between settlements 4. Entry of settlements
5. Core periphery model
Ans:- (D)
Explanation:
Primacy of settlement refers to the phenomenon where the largest city in a country or region has a
population that is disproportionately larger than the second-largest city. It is measured by the ratio of
the population of the largest city to that of the second-largest city. A high ratio indicates a high degree
of primacy. Primacy of settlement is related to the rank size rule, which states that the population of a
city is inversely proportional to its rank in the urban hierarchy. According to this rule, the largest city
should have twice the population of the second-largest city, three times the population of the third-
largest city, and so on.
Hemant Vilas Parulekar 185

Central place theory is a spatial theory in urban geography and urban economics that explains the
size and distribution of settlements within a system or region1. According to this theory, the primary
purpose of a settlement is to provide goods and services for the surrounding market area. The theory
assumes that settlements are located on a uniform plane with equal population density and purchasing
power, and that consumers act rationally to minimize transportation costs by visiting the nearest
location offering the desired good or service.
Random component in location of settlements refers to the element of chance or unpredictability that
affects the spatial pattern of settlements. It acknowledges that not all settlements are located according
to rational or optimal criteria, but rather influenced by historical, cultural, or personal factors that are
difficult to quantify or model. Random component in location of settlements is associated with the
concept of entary of settlements, which is the process of establishing new settlements or expanding
existing ones. Entary of settlements can be influenced by various factors, such as exploration,
colonization, migration, urbanization, industrialization, and globalization.
Interaction between settlements refers to the exchange of goods, services, information, and people
among different settlements. It reflects the degree of connectivity and interdependence among
settlements, as well as the spatial structure and hierarchy of the urban system. Interaction between
settlements is related to the gravity model, which states that the interaction between two settlements is
proportional to their population sizes and inversely proportional to the distance between them.
Q.28 Match the architectural projects in Group Iwith the architect in Group II
P-3, Q-1, R-2, S-5 Group I Group II
(B) P-1, Q-2, R-5, S-3 (P) India Habitat Centre, New Delhi 1. Christopher Charles Benninger
(C) P-2, Q-1, R-5, S-4 (Q) United World College(UWC) 2. Charles Correa
Mahindra college, Pune
(D) P-3, Q-4, R-1, S-5
(R) Brain and cognitive science 3. Joseph Allen Stein
Centre-MIT, Cmbridge
(S) Habitat 67, Montreal 4. Norman Foster
5. Moshe Sadie
186 GATE Architecture and Planning: Comprehensive Question Bank

Ans:- (A)
Explanation:
Name of India Habitat Centre, New Delhi United World College(UWC)Mahindra
building college, Pune
Architect Joseph Allen Stein Christopher Charles Benninger
Image

Name of Brain and cognitive science Centre- Habitat 67,Montreal


building MIT,Cmbridge
Architect Charles Correa Moshe Sadie
Image

Q.29 Match the Name of the book provided in Group - I with the corresponding author in Group - II and
select the appropriate option.
(A) P-2, Q-3,R-1,S-5 Group I Group II
(B) P-5, Q-2,R-3,S-4 (P) Earthscape 1. Ian Mc Harg
(C) P-5, Q-3,R-1,S-4 (Q) synthesis of form 2. John O Simonds
(R) Design with nature 3.Christopher Alexander
(D) P-2, Q-1,R-4,S-5
(S) The city of tomorrow and its 4. Lewis Mumford
plannig
5. Le Corbusier
Ans:- (A)
Explanation:
The book Earth scape: A Manual of Environmental Planning and Design was written by John O.
Simonds, an American landscape architect and author. He is best known for his influential books
Landscape Architecture and Earth scape, which explore the principles and practices of ecological
design.
The book Synthesis of Form was written by Christopher Alexander, an American architect and
design theorist who is known for his pattern language approach to architecture. The book presents an
entirely new theory of the process of design, based on the analysis of the form and function of complex
systems.
Hemant Vilas Parulekar 187

The book Design with Nature was written by Ian L. McHarg, a Scottish landscape architect and
environmentalist who advocated ecological planning and design1. The book presents an ecologically
sound approach to the planning and design of communities, and has influenced the fields of landscape
architecture, urban and regional planning, and ecological design.
The book The City of Tomorrow and Its Planning was written by Le Corbusier, a Swiss-French
architect, designer, and urban planner who is considered one of the pioneers of modern architecture.
The book presents an entirely new theory of the process of design, based on the analysis of the form
and function of complex systems.
Q.30 Match the thermal properties in the Column - I and their respective units in Column - II and select
the appropriate option.
(A) P-4, Q-1, R-5, S-2 Group I Group II
(B) P-4, Q-3, R-1, S-2 (P) Thermal Resistance 1. JKg-1 0C-1
(C) P-5, Q-3, R-1, S-4 (Q) Thermal Transmittance 2. Wm-1 0C-1
(D) P-3, Q-4, R-2, S-1 (R) Specific Heat 3. Wm-2 0C-1
(S) Thermal Conductivity 4. m2 0C-1
5. Jm-3 0C-1
Ans:- (B)
Explanation:
(P) Thermal Resistance: This is the measure of how well a material resists the flow of heat. It is the
inverse of thermal conductance. Its unit is m2 0C-1.
(Q) Thermal Transmittance: This is the measure of how much heat is transferred through a unit area
of a material under a given temperature difference. It is also known as the U-value or the heat transfer
coefficient. Its unit is Wm-2 0C-1.
(R) Specific Heat: This is the amount of heat required to raise the temperature of one kilogram of a
material by one degree Celsius. It is also known as the specific heat capacity or the heat capacity per
unit mass. Its unit is JKg-1 0C-1.
(S) Thermal Conductivity: This is the measure of how well a material conducts heat. It is the rate of
heat transfer per unit area per unit temperature gradient. Its unit is Wm-1 0C-1.
Q.31 Match the application in the field of construction in the Group - I and the respective items in
Group- II and select the appropriate option.
P-5, Q-1, R-4, S-3 Group I Group II
(B) P-4, Q-3, R-1, S-5 (P) Polytetrafluoroethylene (PTEFE) membrane 1. Tendon
(C) P-5, Q-3, R-1, S-2 (Q) Isolated compression component inside a network 2. TMT
(D) P-3, Q-4, R-2, S-1 of continuous tensile membrane
(R) Cable used for pre-stressed concreting 3. Tensegrity
(S) Reinforcement bar used in RCC construction 4. TMD
5. Teflon
188 GATE Architecture and Planning: Comprehensive Question Bank

Ans:- (C)
Explanation:
Item Tendon TMT
Application The steel cables or rods that are used TMT steel bars are steel bars that have
to prestress concrete structures, such undergone a thermo-mechanical
as beams, slabs, bridges, and domes. treatment process, which involves rapid
Prestressing increases the strength quenching and tempering of the bars
and durability of concrete by applying to improve their strength, ductility,
a compressive force before loading. and corrosion resistance. TMT steel
Tendons can be either bonded or bars have many applications in the
unbonded to the concrete, and can construction industry, such as:
have different layouts and profiles. Building structures: TMT steel bars are
Tendon in construction is used used to reinforce concrete structures,
in various applications. In bridge such as columns, beams, slabs, and
construction, tendons are used to foundations, in residential and
reduce deflection and improve load commercial buildings. They provide
capacity. strength and stability to the structures and
In building construction, tendons help them withstand natural disasters,
are used to reduce cracking and such as earthquakes and cyclones. Bridge
deflection and enhance the structural and flyover construction, Industrial and
performance of buildings. infrastructure projects.

In parking garages, tendons are used


to increase the load capacity and
reduce cracking. In industrial plants,
tendons are used to withstand heavy
loads and harsh weather conditions.
Tendons can be categorized into two
types: prestressed and post-tensioned.
Prestressed tendons are installed
before the concrete is poured, while
post-tensioned tendons are installed
after the concrete has been poured.
Both types of tendons are used to
reduce cracking and deflection,
enhance structural performance, and
increase the load capacity of structure
Item
Hemant Vilas Parulekar 189

Item Tensegrity Teflon


Application Tensegrity is a design principle that TEFLON is a brand name of
applies when a discontinuous set of polytetrafluoroethylene (PTFE),
compression elements is opposed and a synthetic fluoropolymer of
balanced by a continuous tensile force, tetrafluoroethylene that has numerous
thereby creating an internal prestress applications. A TEFLON PTFE
that stabilizes the entire structure. The membrane is a type of material that
term was coined by Buckminster Fuller, is coated with TEFLON, making
a famous architect and inventor, as a it extremely durable, resistant to
portmanteau of “tensional integrity”. heat, chemicals, and corrosion, and
Tensegrity structures are lightweight, biologically inert. TEFLON PTFE
flexible, and resilient, and can be found membranes can be used for various
in nature, art, architecture, engineering, purposes like covering water leakage etc.
and biology.
Item

Q.32 Match the following types of masonry joints in Group - I with their corresponding description in
Group-II, and select the appropriate option.
Group I Group II (A) P-1, Q-3,R-2,S-4
(P) 1. Strucked (B) P-4, Q-3, R-2,S-5
(C) P-3, Q-4,R-5,S-2
(D) P-4, Q-3, R-1,S-5
(Q) 2.Weathered

(R) 3. Raked

(S) 4. Beaded

5. Concave
Ans: (B) or (D)
190 GATE Architecture and Planning: Comprehensive Question Bank

Q.33 Match the following in Group- I with their suitable description in Group- II, and select the
appropriate option.
P-4, Q-1, R-5, S-2 Group I Group II
(B) P-4, Q-3, R-1, S-2 (P) Tolerance 1. 100mm
(C) P-5, Q-3, R-1, S-4 (Q) Precast concrete rings for wells 2. non-modular dimension
(R) M 3. Acceptable variation
(D) P-3, Q-4, R-1, S-5
(S) Weather joints 4. 3D-prefabricate
5. Resilient sealants
Ans:- (D)
Explanation:
Tolerance is the acceptable variation in the dimensions or quality of a product or a process12. For
example, the tolerance for the deviation from elevation of cast-in-place concrete is ± ¼ inch for 0 to
10 feet.
Precast concrete rings for wells are circular segments of concrete that are produced in a plant and then
transported to the site for assembly. They are used to construct wells, shafts, or tunnels.
M is a non-modular dimension that is used to measure the length of bricks or blocks. It is equal to
100 mm or 3.94 inches.
Weather joints are joints that are exposed to the external environment and need to be sealed with
resilient sealants to prevent water infiltration, thermal expansion, or corrosion.
Q.34 Match the scientific names of the trees provided in Group- I with the corresponding colour of
their bloom in Group- II, and select the appropriate option.
(A) P-4, Q-5, R-4, S-1 Group I Group II
(B) P-1, Q-5, R-2, S-3 (P) dB 1. Sound intensity
(C) P-5, Q-4, R-1, S-3 (Q) Phon 2. Absorption of sound
(D) P-5, Q-4, R-1, S-2 (R) W/m2 3. Frequency of sound
(S) Sabine 4. Loudness
5. Sound pressure level
Ans:- (D)
Explanation:
dB is a unit of sound pressure level, which is a measure of the amplitude of sound waves relative to a
reference value. It is also related to loudness, which is the subjective perception of sound intensity by
the human ear.
Phon is a unit of loudness, which is defined as the sound pressure level in dB of a 1 kHz tone that is
perceived to be equally loud as the sound being measured3. For example, a sound of 60 phons is as loud
as a 1 kHz tone at 60 dB.
W/m2 is a unit of sound intensity, which is the power of sound per unit area perpendicular to the
direction of sound propagation. It is proportional to the square of sound pressure level, and it represents
the amount of energy carried by sound waves.
Hemant Vilas Parulekar 191

Sabine is a unit of sound absorption, which is the process of converting sound energy into heat or
other forms of energy within a material or a medium. It is defined as the absorption due to unit area of
a totally absorbent surface. Sabins are used to calculate the reverberation time of a room, which is the
time it takes for the sound level to decay by 60 dB after the sound source is stopped. (refer Q.30 for
more information on units).
Q.35 Match the items in Group- I and their respective location in building/site in Group- II, and select
the appropriate option.
P-4, Q-5, R-2, S-3 Group I Group II
(B) P-5, Q-1, R-3,S-2 (P) Cassia fistula 1. white
(C) P-4, Q-1, R-5, S-3 (Q) Lagerstroemia flos-reginae 2. Red
(R) Cordia sbastena 3. Blue
(D) P-4, Q-5, R-2,S-1
(S) Plumeria alba 4. Yellow
5. Mauve
Ans:- (C)
Explanation:
(P) Cassia fistula has yellow flowers Amaltas (golden flower tree), so it matches with 4. Yellow.
(Q) Lagerstroemia flos-reginae has white color Jaul (pride of india) so it matches with 1. White
(R) Cordia sbastena has red color (scarlet cordia) so it matches with 2. Red
(S) Plumeria alba has white flowers (champa tree) with yellow centers , so it matches with 1. White.
Q.36 As per the Handbook on Barrier Free and Accessibility, CPWD - 2014, match the design guidelines
in Group- I with their appropriate standards in Group - II and select the appropriate option.
P-3,Q-4,R-1,S-5 Group I Group II
(B) P-5,Q-3,R-2,S-4 (P) Nahani Trap 1.between waste water pipe and main house drain
(C) P-4,Q-1,R-5,S-3 (Q) Gully Trap 2. between septic tank and soak pit
(R) Bottle trap 3. Junction of house drain sewer
(D) P-1,Q-4,R-3,S-1
(S) Intercepting 4. Bathroom and kitchen floor
5. Below the basin

Ans:- (C)
Explanation:
(P) Nahani Trap is a device that receives wastewater from showers, bathroom and kitchen floors and
transmits it to the drainage pipelines. It also prevents foul gases from the sewer drains from entering
the building. It is usually located outside the house, near the kitchen or bathroom.
(Q) Gully Trap is a basin in the ground that receives piped wastewater from the house before it enters
the underground sewer. It has a water seal to prevent odours reaching the surface. It is often present in
patio areas or at the bottom of rainwater downpipes.
(R) Bottle Trap is a type of device that traps debris and other objects that may cause blockages in the
waste pipe. It has the appearance of an upside-down bottle that leads into the pipes. It is typically used
for low-profile sinks and basins.
(S) Intercepting Trap is a device that is provided in the last manhole of the house drainage system. It
has a deep water seal of 100 mm and it prevents the entry of sewer gases from public sewer line into
the house drains.
192 GATE Architecture and Planning: Comprehensive Question Bank

Q.37 Match the contemporary Urban Design Movements listed in Column - I with the corresponding
principles listed in Column - II and select the appropriate option.
(A) P-2, Q-3, R-4, S-1 Group I Group II
(B) P-1, Q-5, R-3,S-2 (P) Minimum clear width of ramp 1. 600 mm
(C) P-5, Q-3, R-1,S-2 (Q) Maximum height of wash basin(rim) above finished 2. 1500mm
floor level
(D) P-2, Q-5, R-4,S-1
(R) minimum length of grab rail 3. 750mm
(S) minimum clear width for maneuvering space (wheelchair) 4. 900mm

5. 1800 mm

Ans:- (C)
Explanation:
Design and Ramp Tollet
guidelines
for
Standards A ramp is a sloped surface that allows A toilet is a sanitary fixture that is used for
people with reduced mobility, such as the disposal of human waste. There are
wheelchair users, to access buildings different types of toilets, such as public
and sites more easily. There are some toilets, private toilets, and residential
universal design recommendations for toilets, and they may have different
accessible ramps that can be followed to design and maintenance requirements.
ensure safety and comfort for all users. Some of the general guidelines for the
Some of these recommendations are: construction of institutional and public
• The slope of the ramp should be 1:20 toilets are:
(5%). • The toilet should be located in a
• The width of the ramp should be at convenient and accessible place,
least 1.5 meters, and preferably 1.8 preferably near the main entrance or
meters or more. exit of the building or site.

• The ramp should have a level landing • The toilet should have adequate
on top and bottom, and at every natural or artificial lighting,
change of direction or slope. The ventilation, and drainage to ensure
landing should be at least 1.5 meters hygiene and comfort.
long and as wide as the ramp. • The toilet should have durable and
• The surface of the ramp and landing easy-to-clean materials, such as
should be firm, stable, and slip- ceramic tiles, stainless steel, or plastic,
resistant. The surface should also for the walls, floors, and fixtures.
be colour-contrasted with the • The toilet should have water-efficient
surrounding area to enhance visibility. and user-friendly fixtures, such
as dual-flush or sensor-operated
water closets, self-closing or sensor-
operated taps, and hand dryers or
paper dispensers.
Hemant Vilas Parulekar 193

Illustration

Design and Railing Wheelchair space


guidelines
for
Standards A railing is a horizontal or inclined A wheelchair space is an area that allows
structure that provides support, safety, a person using a wheelchair to access and
and aesthetic appeal to stairs, ramps, use a building or facility comfortably and
balconies, decks, and other elevated areas. safely. There are some general design
There are different types of railings, such guidelines for wheelchair spaces that can
as handrails, guardrails, balustrades, and be followed to ensure accessibility and
banisters, and they may have different inclusivity for all users. Some of these
design and installation requirements guidelines are:
depending on the context and the local • The wheelchair space should have a
standards. Some of the general guidelines clear floor area of at least 0.9 meters
for the design and construction of railings by 1.4 meters, and preferably 1.2
are: meters by 1.5 meters, to allow for
• The height of the railing should be maneuvering and turning.
appropriate for the intended users • The wheelchair space should have
and the level of risk. For example, the a level and firm surface, free of any
International Building Code (IBC) obstacles or protrusions, and slip-
requires handrails to be between 34 resistant in wet and dry conditions.
and 38 inches high, and guardrails to
be at least 36 inches high. • The wheelchair space should be
located near an accessible entrance,
• The width of the railing should be exit, or circulation route, and have a
sufficient to allow a comfortable and clear and unobstructed view of the
secure grip. For example, the IBC intended activity or service1.
requires handrails to have a diameter
of 1-1/4 to 2 inches, and a clearance of • The wheelchair space should be
at least 1-1/2 inches from the wall or integrated with the seating or
other obstructions. standing areas of other users, and not
segregated or isolated from them.
Illustration
194 GATE Architecture and Planning: Comprehensive Question Bank

Q.38 Match the figures of vaults in Group- I with their corresponding types in Group - II and select the
appropriate option.
(A) P-2,Q-3,R-4,S-1 Group I Group II
(B) P-3,Q-1,R-4,S-5 (P) park movement 1. self-contained, self sufficient
(C) P-2,Q-1,R-5,S-3 community surrounded by green belts
(Q) New urbanism 2. Revival of the relationship between
(D) P-2,Q-5,R-4,S-1
man and nature
(R) city beautiful movement 3. relationship between work and living
environmental sustainability
(S) garden city and new town 4. unity, cohesion and balanced
movement. relationship between urban compnents
and elements.
5. technical and socio-economic process
resulting in growth,production and
waste elimination.
Ans:- as per final GATE answer key both A or D are given correct.
Explanation:
Design and Park movement New urbanism
guidelines
for
Standards The park movement is a term that New urbanism is an urban design
refers to the historical and ongoing movement that promotes environmentally
efforts to create, preserve, and improve friendly habits by creating walkable
public parks and open spaces in urban neighbourhoods containing a wide range
areas. The park movement began in the of housing and job types. It arose in the
19th century in Europe and America, United States in the early 1980s, and has
as a response to the negative effects gradually influenced many aspects of real
of industrialization, urbanization, estate development, urban planning, and
and overcrowding on the health and municipal land-use strategies. Mixed-
well-being of city dwellers. The park use development: combining residential,
movement has evolved over time commercial, and civic uses in the same
to address the changing needs and area to reduce the need for car travel and
challenges of urban communities. increase the vitality of the neighbourhood.
Some of the themes and trends that Diversity and affordability: providing a
have influenced the park movement variety of housing types, sizes, and prices
include: the provision of playgrounds, to accommodate people of different
sports facilities, and cultural amenities; incomes, ages, and lifestyles.
the protection of natural and historical
resources; the promotion of social justice Human-scale design: creating public
and equity; the integration of art and spaces and streets that are comfortable,
design; the adaptation to climate change safe, and attractive for pedestrians, cyclists,
and resilience; and the engagement of and transit users.
diverse and marginalized populations.
Hemant Vilas Parulekar 195

Design and city beautiful movement Garden city and new town movement.
guidelines
for
Standards The city beautiful movement was a The garden city and new town movements
reform philosophy of North American are related urban planning concepts that
architecture and urban planning that aim to create self-contained, balanced, and
flourished during the 1890s and 1900s. sustainable communities. The garden city
It aimed to introduce beautification and movement was developed by Ebenezer
monumental grandeur in cities, as well Howard in the late 19th century, as a
as to improve the moral and civic virtue response to the overcrowding, pollution,
of urban residents. The city beautiful and social problems of industrial cities.
movement influenced the design of The new town movement emerged after
many public buildings, monuments, World War II, as a way to address the
and parks in cities such as Washington, housing shortage, the urban decay, and
D.C., Cleveland, Detroit, and Kansas the population growth in major cities.
City. The movement also advocated The new town movement was inspired
for comprehensive urban planning, by the garden city movement, but also
social reform, and environmental incorporated modernist principles
sustainability. However, the movement of functionalism, zoning, and mass
also faced criticism for being elitist, production. The new town movement
homogenous, nostalgic, and unrealistic. aimed to create planned, large-scale, and
socially diverse settlements that would
provide housing, employment, education,
and recreation for their residents. The new
town movement led to the development of
many new towns in the UK and abroad,
such as Milton Keynes, Stevenage,
Brasilia, and Chandigarh.

Q.39 Match the figures of vaults in Group- I with their corresponding types in Group– II and select the
appropriate options.
(A) P-3,Q-4,R-1,S-2 Group I Group II
(B) P-3,Q-1,R-4.S-5 (P) 1.Ribbed
(C) P-2,Q-1,R-5,S-3
(D) P-2,Q-3,R-1,S-5
(Q) 2. Fan

(R) 3. Barrel

(S) 4. Groin

5. Nubian
196 GATE Architecture and Planning: Comprehensive Question Bank

Ans:- (A)
Explanation:
Vault type Ribbed Fan
Explanation A ribbed vault is a type of vault that is A fan vault is a type of vault that is
formed by the intersection of two or formed by the intersection of several
more barrel vaults at right angles. It has barrel vaults at different angles, creating
the advantage of reducing the amount of a fan-like shape. It is a more complex
material and labor needed compared to a and decorative variation of the ribbed
barrel vault, and also of concentrating the vault, and it is characteristic of the
thrust forces at the four corners, making English Perpendicular Gothic style.
the supporting walls less massive.
Image

Vault type Barrel Groin


Explanation A barrel vault is a basic type of vault or A groin vault is a type of vault that
extruded arch. It is sometimes referred is formed by the intersection of two
to as a tunnel vault or a wagon vault. It barrel vaults at right angles. It has the
is an architectural element formed by advantage of reducing the amount of
the extrusion of a single curve (or pair material and labor needed compared to
of curves, in the case of a pointed barrel a barrel vault, and also of concentrating
vault) along a given distance. The curves the thrust forces at the four corners,
are typically circular in shape, lending a making the supporting walls less
semi-cylindrical appearance to the total massive. Groin vaults were first used
design. It is the simplest form of a vault: by the Romans in various structures,
effectively a series of arches placed side such as baths, aqueducts, and temples,
by side (i.e., one after another). It is a and later became popular in medieval
form of barrel roof. As with all arch-based church architecture.
constructions, there is an outward thrust
generated against the walls underneath a
barrel vault.
Image
Hemant Vilas Parulekar 197

Vault type Nubian


Explanation A nubian vault is a type of vaulted structure that is made of mud bricks and does
not require any timber or metal. It is an ancient technique that originated in Upper
Egypt and was revived by Egyptian architect Hassan Fathy in the 1940s. It is a low-
carbon and sustainable housing solution that is suitable for the Sahelo-Sudanian
region, where wood is scarce and climate change is a challenge.
Image

Q.40 A colony of 50 people is served by a septic tank. The rate of water supply is 90 lpcd in the colony and
40% of it is going to the septic tank. The retention period of the tank is 24 hours. The length of the
septic tank is _____meter (rounded off to two decimal places). Assume, storage capacity/person =
0.085 m³ (3 years)
Space for digestion = 0.0425 m³/person
Depth of tank = 1.4 m
Length: Width = 2:1
Ans:- 3.40 to 3.45
Solution:
Water in septic tank = 50 × 90 × 1 × 0.4 = 1.8 cu. m (40% of total consumption)
Storage volume of septic tank = 0.085 × 50 = 4.25 cu. m
Digestion volume of septic tank = 0.0425 × 50 = 2.125 cu. m
Total volume = 1.8 + 4.25 + 2.125 = 8.175 cu. M
Area of plane =Volume/ Height=8.175/1.4
= 5.84 Sq m
If width is ‘a’, length will be ‘2a’
2a2 = 5.84
a = √(2.92) = 1.7
Length of septic tank = 2a = 2 × 1.7 = 3.4 m
Q.41 A cone, with a base of 10 cm diameter and axis of 12 cm, is lying on Horizontal Plane (HP) along its
generator. The internal angle which the base of the cone makes with the HP is___degrees.
Ans:- 67.0 to 68.0
Solution:
Given, Diameter of the
cone = 10cm, or say, radius = 5cm
Height of the cone = 12cm
In below diagram, tan Ɵ= 12/5
Ɵ= tan-1(12/5) = 67.38 degree
198 GATE Architecture and Planning: Comprehensive Question Bank

Q.42 A public utility building of 5000 m2 was constructed 5 years before, on a site of 1 hectare. The
present value of open land in that location is Rs.100/m2 and present construction cost of such
building is Rs. 2500/m2. If the value of the building is assumed to be depreciating at a constant rate
of 6 percent per annum, then the present value of the property using‘Valuation by Cost Method’ is
_____(in Rs. lakhs) (rounded off to one decimal place).
Ans:- 101.0 to 104.0
Solution:
Present Land value = Rs. 100 × 10,000 = Rs. 10 lakhs
Cost of Construction of the building = Total Area × Per Sq m
Cost of Construction = 5,000 × 2500 = 125 lakhs
Depreciation rate = 6% Value of the building after depreciation in 5 years
= 125 (1 – r)n = 125 × (1 – 0.06)5 = 91.7 lakhs
Present Value of Property = Land Value + Building Value = 10+ 91.7 = 101.7 lakhs
Q.43 A residential area of 20 hectares is planned for three different types of plots of 500 m2, 300 m2 and
200 m2 with numbers of plot in each category are 100, 120 and 150 respectively. The rest of the area
is allocated for roads and facilities such as schools, shops and parks. Each plot has one dwelling
unit and the average household size is 5 persons. The net residential density of the area in persons
per hectare is_____.
Ans:159 to 160
Solution:
Total area = 20 Ha = 20
Total number of plot = 100 + 120 +150 = 370
Number of DU = 370 x 1 = 370
Population = 370 x 5 = 1850 person
Net are = total area of plot=50000+36000+30000 = 116000 sqm. i.e. 11.6 ha
Net residential density = population / net area = 1850 /11.6 = 159.48 pph
Q.44 In a single lane road, traffic volume of 1000 vehicle/h moving at 20 km/h, comes to a halt due to an
accident. If jam density is 150 vehicles/km, the velocity of the shock wave generated (in absolute
value) is_____km/h.
Ans:- 9.9 to 10.1
Solution:
According to the shockwave theory, the velocity of the shock wave generated by a traffic jam is given
by the formula:

where vs is the shock wave velocity, q1 and q2 are the traffic flows before and after the jam, and k1 and
k2 are the traffic densities before and after the jam.
In your case, the traffic flow before the jam is q1=1000 vehicles/h, and the traffic density after the jam
is k2=150 vehicles/km. To find the traffic flow and density before the jam, we can use the relation:
Hemant Vilas Parulekar 199

where q is the traffic flow, k is the traffic density, and v is the traffic speed
Therefore, we have:
q1=k1v1=1000
v1=20
Solving for k1, we get:

Since the traffic comes to a halt due to the accident, we can assume that the traffic flow and speed after
the jam are zero, i.e., q2=v2=0. Plugging these values into the shock wave velocity formula, we get:

Therefore, the velocity of the shock wave generated (in absolute value) is 10 km/h.
Q.45 In a site map, a rectangular residential plot measures 150mm × 40mm, and the width of the front
road in the map measures 16 mm. Actual width of the road is 4m. If the permissible F.A.R. is 1.2,
the maximum built-up area for the residential building will be___ m2.
Ans:- 450
Solution:
Scale of the map = length on map / actual length = 16 mm / 4 m = 1: 250
Actual dimension of the plot would be
length = 150 x 250 = 375000 mm = 37.5 m
width = 40mm x 250 = 10000 mm = 10 m
Area of the site = 37.5 x 10 = 375
Maximum built up area = 375x1.2 = 450 m².
Q.46 The internal dimension of a room is 10m × 10m × 4m (height). The total area of the doors and
windows are 16 m². Keeping the doors and windows closed , the reverberation time of the room
becomes 1.2 second. Assume all the interior surfaces including doors and windows have same
sound absorption coefficient. If all the doors and windows of the room are kept fully open, the
reverberation time will be__seconds (rounded off to two decimal places)
Ans:- 0.93 to 0.97
Solution:
Total volume of room=10×10×4=400m3
Total area of absorption=total wall area×total floor area+total ceiling area=4×(10×4)+(10×10)+(10×1
0)=360m2
Total area of doors and windows=16m2
Thus, net area of wall+ floor+ ceiling=360-16=344m2
Formula to calculate reverbation time: R.T=0.16(V/A).
Where, V=Volume of the room and A=total room absorption in sabins.
200 GATE Architecture and Planning: Comprehensive Question Bank

If the different surface areas of the room are treated with different materials which are having different
co-efficient of absorption
Then, A=a1A1+a2A2+a3A3+a4A4+…(where An is one section of absorbent area and an is respective
absorption coefficient of the material)
It is given that, reverberation time of the room (R.T)-1.2 sec. the absorption co-efficient is unknown.
Assume, the total area of absorption including doors and windows as A1=360m2
Assume, the net area excluding doors and windows as A2=344m2
For calculating the value of absorption co-efficient a1, for entire absorption area including doors and
windows
R.T=0.16(V/A)=0.16(V/A1×a1)
1.2=0.16×(400/360×a1)
Thus, a1=0.148~0.15
For calculating reverberation time R.T when doors and windows are open. Their absorption co-
efficient will be
R. T=0.16(V/A) = 0.16×(V/A1×a1)
=1.2=0.16× [400/ (344×0.15) +(16×1)]
R.T=0.946 sec=0.95
Q.47 A depressed portion of a land is identified by three closed
contours, as shown in the figure below. The areas bounded
by three contour lines are 6 m², 24 m² and 96 m² respectively.
The contour interval is 1 m. Using prismoidal method,
the volume of the earth needed to fill the land depression
is _____m³.

Ans:- 65 to 76
Solution:
The formula for the Trapezoidal Rule is:
V=h/2(A1+A2)
where:
V is the volume,
h is the height (in this case, the contour interval), and
A1 and A2 are the areas of the two parallel planes.
Given that the contour interval is 1 m, and the areas bounded by the three contour lines are 6 m², 24
m², and 96 m² respectively, we can calculate the volume of the two frustums formed by these contours.
For the first frustum (from the outermost contour to the middle one):
A1=96 m²,
A2=24 m², and
h=1 m.
Hemant Vilas Parulekar 201

For the second frustum (from the middle contour to the innermost one):
A1=24 m²,
A2=6 m², and
h=1 m.
substitute these values into the Trapezoidal Rule formula to find the volumes of the two frustums, and
then add them together to get the total volume of the depression.
Let’s calculate:
For the first frustum:

For the second frustum:

So, the total volume of the depression is V1+V2=60+15=75m3.


Q.48 Solar panels are proposed to be installed on a building roof top to generate electricity. The size of
each solar panel is 2m². The efficiency of each panel is 75%. The orientations of the solar panel and
related solar data are given in the table below.
Orientation No.of Panel Average daily solar Average solar hours per
radiation in W/m2 day
South 10 400 4
West 5 300 2
As per the above propose_____kWh solar power will be generated daily.
(rounded off to one decimal place)
Ans:- 28.4 to 28.6
Solution:
Solar power generated = Avg solar radiation x Area x Average solar hours x Efficiency
Solar power from south orientation = 400 x 20 x 4 x 0.75 = 24 kWh
Solar power from West orientation = 300 x 10 x 2 x 0 .75 = 4.5 kWh
Total solar power generated = 24 + 4.5 = 28.5 kWh
Q.49 A power shovel is having 1.8m³ excavation output per batch of operation. The average cycle time of
the batch operation is 45 seconds. The lost time per hour of the excavation activity is 10 minutes.
Assume six working hours of operation per day. The amount of soil excavated by the power shovel
per day in___m³ (rounded off to two decimal places).
Ans:- 719.75 to 720.25
Solution:
Based on the given information, the amount of soil excavated by the power shovel per day can be
calculated as follows:
The amount of soil excavated per batch of operation is 1.8 m³.
The average cycle time of the batch operation is 45 seconds.
202 GATE Architecture and Planning: Comprehensive Question Bank

The lost time per hour of the excavation activity is 10 minutes.


The working hours per day is 6 hours.
First, we need to calculate the number of batches of operation that can be performed in one hour:
The total number of seconds in an hour is 3600 seconds.
The lost time per hour is 10 minutes or 600 seconds.
The total number of working seconds per hour is 3600 - 600 = 3000 seconds.
The number of batches of operation that can be performed in one hour is 3000 / 45 = 66.67 batches.
Next, we can calculate the amount of soil excavated per hour:
The amount of soil excavated per hour is 66.67 * 1.8 = 120 m³.
Finally, we can calculate the amount of soil excavated per day:
The amount of soil excavated per day is 120 * 6 = 720 m³.
Therefore, the amount of soil excavated by the power shovel per day is 720 m³.
Q.50 A room having dimension 12m×10m ×3.5m is required to be mechanically ventilated by air-
conditioner. The temperature differences between outdoor ambient air and the supply air is 12°C.
Consider three air exchanges per hour. The volumetric specific heat of the air is 1250 J/m³ °C.
Assume one ton of refrigeration (TR) is equal to 3.5 kW. The capacity of the air-conditioner for the
room in TR will be.
Ans:- 1.49 to 1.51
Solution:
Rate of ventilation = (12 x 10 x 3.5 x 3)/3600 = 0.35
Cooling load = 1250 x 0.35 x 12 = 5.25 kW = 1.5 TR
Q.51 A simply supported beam AB has a clear span of 7 meter. The bending moment diagram (BMD) of
the beam due to a single concentrated load is shown in the figure below.

The magnitude of the concentrated load in kN is_____.

Ans:- 21
Solution:
The relationship between the bending moment and the concentrated load is
given by the following equation:

Mmax=36 kN-m
a=3m, b=4m, l= 3+4=7
thus, 36=(p×3×4)/7
p=(36×7)/12= 21kN
Hemant Vilas Parulekar 203

Q.52 For a symmetrical trapezoidal open drain in a landscape with grass and loose rock surface, the
velocity of flow of water is m/sec, (rounded off to two decimal places), given the following data.
Water edge width at the top = 750 mm
Water edge width at the bottom = 450 mm
Water depth = 600 mm
Manning’s coefficient of roughness = 0.05
Slope along the drain = 1 in 250
Ans:- 0.40 to 0.50
Solution:
Water edge width at the top = 750 mm
Water edge width at the bottom = 45000 mm
Water depth = 600 mm
Manning’s coefficient of roughness = 0.05
Slope along the drain = 1 in 250
The velocity of water (in m/sec) can be calculated using the following formula:
Velocity of water (in m/sec) = 1 × r(2/3) × S(1/2) / n
Here, n = 0.05
Hydraulic radius , r = Wet area / Wet perimeter
Wet area = (Water edge width at the top + Water edge width at the bottom) / 2 × Water depth
Wet perimeter = Water edge width at the top + Water edge width at the bottom + 2 × (Water depth /
cos⁡θ)
θ = tan⁡(1/250)
Substituting the given values in the above formula, we get:
Wet area = (750 mm + 45000 mm) / 2 × 600 mm = 13500000 mm2 = 0.0135 m2 θ
= tan⁡(1/250) = 0.004 Wet perimeter = 750 mm + 45000 mm + 2 × (600 mm / cos⁡(0.004)) = 46020.6
mm = 46.02 m r = Wet area / Wet perimeter = 0.000293 m S = 1 / 250 = 0.004 m n = 0.05
Velocity of water (in m/sec) = 1 × (0.000293)(2/3) × (0.004)(1/2) / 0.05 = 0.45 m/sec (rounded off to two
decimal places) 1.
Therefore, the velocity of flow of water in the given scenario is 0.45 m/sec (rounded off to two decimal
places).
Q.53 The stack pressure is created by 10 m height of stack and 15°C temperature difference. The motive
force due to the stack pressure over a cross section area of 2.5m2 is_________ N.
Ans: 15.0 to 16.0
Solution:
The stack pressure created by a 10 m high stack and a temperature difference of 15°C is 6.3 N/m² 1. The
motive force due to the stack pressure over a cross-sectional area of 2.5 m² is:
Force=Pressure×Area=6.3 N/m²×2.5 m²=15.75 N
Therefore, the motive force due to the stack pressure over a cross-sectional area of 2.5m² is 15.75 N.
204 GATE Architecture and Planning: Comprehensive Question Bank

Q.54 An industrial building contains 3000 kg of combustible materials, in dry state, distributed over
three rooms of area 100 m², 500 m² and 300 m² each, in a proportion of 30%, 50% and 20% of the
contents, respectively. Calorific value of the material is 4400 kCal/kg. The Total Fire Load of the
rooms is equal to____kCal/m².
Ans:- 61500 to 61700 OR 14600 to 14700
Solution:
Fire Load = (Weight x Calorific Value)/Area
Fire load in Room 1 = (3000 x 0.3 x 4400)/100 = 39600 kCal/m²
Fire load in Room 2 = (3000 x 0.5 x 4400)/500 = 13200 kCal/m²
Fire load in Room 3 = (3000 x 0.2 x 4400)/300 = 8800 kCal/m²
Total fire load = 39600 + 13200 + 8800 = 61600 kCal/m²
Q.55 A simple truss is shown in the figure below. The truss is loaded with horizontal and vertical force
15 kN and 25 kN, respectively. The force in the member AB will be_ kN.

Ans:- 20
Solution:
Methods of Joints through analysis of pin-jointed plane trusses helps in
the calculation of each member forces. Support at A is a roller, therefore
it has only vertical reaction and no horizontal reaction.
The force in member AB of the given truss can be calculated using
the Method of Joints. The method involves analyzing the force in each
member of a truss by breaking the truss down and calculating the forces at each individual joint.
The truss is loaded with horizontal and vertical forces of 15 kN and 25 kN, respectively. The force in
member AB can be calculated as follows:
• First, we need to calculate the vertical and horizontal reactions at joint A. Since support at A is
a roller, it has only vertical reaction and no horizontal reaction. Therefore, the vertical reaction
at A is equal to the vertical force acting on the truss, which is 25 kN.
• Next, we need to calculate the forces in members AC and BC. Using the method of joints, we can
calculate the force in member AC as 20 kN and the force in member BC as 15 kN.
• Finally, we can calculate the force in member AB by using the method of joints again. The force
in member AB is 20 kN.
GATE QUESTION PAPER 2018
General Aptitude (GA)
Q. 1 – Q. 25 carry one mark each.
Q.1 “When she fell down the_______, she received many_______ but little help.” The words that best
fill the blanks in the above sentence are
(A) stairs, stares (B) stairs, stairs (C) stares, stairs (D) stares, stares
Ans:- (A)
Q.2 “In spite of being warned repeatedly, he failed to correct his_________ behaviour.” The word that
best fills the blank in the above sentence is
(A) Rational (B) reasonable (C) Errant (D) good
Ans:- (C)
Explanation:
Behaving or having behaved badly or wrongfully
Q.3 For 0 ≤ x ≤ 2π, sinx and cosx are both decreasing functions in the interval________.
(A) (0, π/2) (B) (π/2, π) (C) (π, 3π/2) (D) (3π/2, 2π)
Ans:- (B)
Solution:
Sin x is increasing function in the interval 0< x<π/2 and in the interval 3π/2 <x< 2π.
Sin x is decreasing function in the interval π/2 <x< 3π/2.
Cos x is increasing function in the interval π< x< 2π.
Cos x is decreasing function on these interval 0<x< π.
Hence, sin x and cos x are both decreasing functions in the interval (π/2, π)

Q.4 The area of an equilateral triangle is √3. What is the perimeter of the triangle?
(A) 2 (B)4 (C) 6 (D)10
206 GATE Architecture and Planning: Comprehensive Question Bank

Ans:- (C)
Solution:
To find the perimeter of an equilateral triangle, we need to know the length of
its side. We can use the formula for the area of an equilateral triangle to find
the side length:
Area = (√3/4)a2
where a is the side length.
Given that the area is √3, we can solve for a:
√3 = (√3/4)a2
Multiply both sides by 4:
4√3 = √3a2
Divide both sides by √3:
4 = a2
Take the square root of both sides:
a = ±2
Since the side length cannot be negative, we take the positive value:
a=2
Now, the perimeter of an equilateral triangle is the sum of its three sides:
Perimeter = 3a
Substitute a = 2:
Perimeter = 3(2)
Perimeter = 6
Therefore, the perimeter of the equilateral triangle is 6.
Q.5 The following three-dimensional objects in the descending order of their volumes:
(i) A cuboid with dimensions 10cm, 8cm and 6cm
(ii) A cube of side 8cm
(iii) A cylinder with base radius 7cm and height 7cm
(iv) A sphere of radius 7cm
(A) (i),(ii),(iii),(iv) (B) (ii),(i),(iv),(iii) (C) (iii),(ii),(i),(iv) (D) (iv),(iii),(ii),(i)
Ans:- (D)
Solution:
Sr.No G e ometric al Formula for volume Volume by feeding Total volume
shape dimensions
1 Cuboid L×B×H 10×8×6 480 cm³
2 Cube L×L×L 8×8×8 512 cm³
3 Cylinder πR2H (22/7) × (7)2×7 1078 cm³
4 Sphere 4/3πR3 (4/3) × (22/7) × (7)3 1437.33 cm³
Hemant Vilas Parulekar 207

Q. 6 – Q. 10 carry two marks each.


Q.6 An automobile travels from city A to city B and returns to city A by the same route. The speed of the
vehicle during the onward and return journeys were constant at 60km/h and 90km/h, respectively.
What is the average speed in km/h for the entire journey?
(A) 72 (B) 73 (C) 74 (D) 75
Ans:- (A)
Solution:
Let’s assume that the distance between city A and city B is d km. Then, the total distance travelled is 2d
km. The time taken for the onward journey is d/60 hours, and the time taken for the return journey is
d/90 hours. The total time taken is d/60 + d/90 hours, which simplifies to 5d/180 hours.
Using the formula for average speed, we get:
Average speed = Total distance travelled / Total time taken
Average speed = (2d) / (5d/180)
Average speed = 72 km/h
Q.7 A set of 4 parallel lines intersect with another set of 5 parallel lines. How many parallelograms are
formed?
(A) 20 (B) 48 (C) 60 (D) 72
Ans:- (C)
Solution:
To form a parallelogram, we need two horizontal parallel lines and two vertical parallel lines. So, the
number of ways to choose two horizontal parallel lines is 4C2, and the number of ways to choose two
vertical parallel lines is 5C2. Using the formula for combination, we get:
⁴C2=(4 x 3)/2!=6
⁵C2=(5 x 4)/2!=10
So, the total number of possible parallelograms is ⁴C2 × ⁵C2= 6 X 10= 60
Q.8 To pass a test, a candidate needs to answer at least 2 out of 3 questions correctly. A total of 6,30,000
candidates appeared for the test. Question A was correctly answered by 3,30,000 candidates.
Question B was answered correctly by 2,50,000 candidates. Question C was answered correctly
by 2,60,000 candidates. Both questions A and B were answered correctly by 1,00,000 candidates.
Both questions Band C were answered correctly by 90,000 candidates. Both questions A and C
were answered correctly by 80,000 candidates. If the number of students answering all questions
correctly is the same as the number answering one, how many candidates failed to clear the test?
(A) 30,000 (B) 2,70,00 (C) 3,90,000 (D) 4,20,000
Ans:- (D)
Solution:
Let A, B, and C be the sets of candidates who answered questions A, B, and C correctly, respectively.
Let n(A), n(B), and n(C) be the number of elements in each set, and so on. We are given the following
information:
208 GATE Architecture and Planning: Comprehensive Question Bank

No of students answering all the questions = x (A Π B Π C)


Answering none = x
X = 60,000/2 = 30,000
No of students not attempting any question = 30,000
No of students Attending only A = 1,80,000
No of students Attending only B = 90,000
No of students Attending only C = 1,20,000
No of Candidate Failed to clear the test = 30,000+1,80,000+90,000+1,20,000 =
4,20,000.
Q.9 If x² + x – 1= 0, what is the value of x⁴ + 1/(x⁴)?
(A) 1 (B) 5 (C) 7 (D) 9
Ans:- (C)
Solution:
First, solve the quadratic equation x² + x - 1 = 0 using the quadratic formula:

where a = 1, b = 1, and c = -1. This gives two possible values for x:

Next, find the reciprocal of x by flipping the fraction and changing the sign:

Using the identity (a+b)(a−b)=a2−b2, we can simplify the denominator of the reciprocal:

Notice that the reciprocal of x is the same as x with the opposite sign and the plus and minus signs
switched. This means that x and 1/x have the same absolute value, which is 5/2.
Now, we can use the identity (x+y)2=x2+2xy+y2 to find x² + 1/x²:
Hemant Vilas Parulekar 209

Finally, we can use the same identity again to find x⁴ + 1/x⁴:

Q.10 A detailed study of annual crow births in India, it was


found that there was relatively no growth during the period
2002 to 2004 and a sudden spike from 2004 to 2005. In
another unrelated study, it was found that the revenue
from cracker sales in India which remained fairly flat from
2002 to 2004 saw a sudden spike in 2005 before declining a
gain in 2006. The solid line in the graph below refers to
annual sale of crackers and the dashed line refers to the
annual crow births in India. Choose the most appropriate
inference from the above data.
(A) There is a strong correlation between crow birth and cracker sales.
(B) Cracker usage increases crow birth rate.
(C) If cracker sale declines, crow birth will decline.
(D) Increased birth rate of crows will cause an increase in the sale of crackers.
Ans:- (A)
Explanation:
Correlation measures the linear relationship between variables. A positive correlation means that when
one variable goes up, the other does as well. A negative correlation means that when one variable goes
up, the other goes down. A zero correlation means that there is no relationship between the variables.
In this case, the data shows a strong positive correlation between crow birth and cracker sales, but it
doesn’t prove that one causes the other. There could be other factors or variables that affect both crow
birth and cracker sales, such as weather, population, food availability, etc. These are called confounding
variables or third variables. Therefore, the other options (B), (C), and (D) are not valid inferences from
the data, as they imply a causal relationship between crow birth and cracker sales without any evidence
or explanation.
ARCHITECTURE AND PLANNING (Q. 1-55)
Q. 1 – Q. 25 carry one mark each.
Q.1 In a Colour Wheel, Red and Blue colours are
(A)Tertiary (B) Complementary (C) Secondary (D) Primary
210 GATE Architecture and Planning: Comprehensive Question Bank

Ans:- (D)
Explanation:
Primary colours are the three basic colours that cannot be created
by mixing other colours. They are red, yellow, and blue. Secondary
colours are the colours that are created by mixing two primary
colours. They are green, orange, and purple. Tertiary colours are
the colours that are created by mixing a primary colour and a
secondary colour. They are red-orange, yellow-orange, yellow-
green, blue-green, blue-violet, and red-violet. Complementary
colours are the colours that are opposite each other on the colour
wheel. They create a strong contrast and balance each other. For example, red and green are
complementary colours.
Q.2 In a bird’s eye perspective view of a cuboid, the maximum number of vanishing points is
(A) 1 (B) 2 (C) 3 (D) 6
Ans:(C)
Solution:
A vanishing point is a point on the image plane where
parallel lines in three-dimensional space appear to
converge. In a bird’s eye perspective view of a cuboid,
the image plane is parallel to two world-coordinate
axes, and intersects the third one. Therefore, lines
parallel to those two axes will form two vanishing
points on the horizon line, and lines parallel to the
third axis will form a third vanishing point above or
below the horizon line. This is called three-point
perspective.
Q.3 The compressive strength of M-25 concrete is
(A) 25kg/sq m (B) 25N/sq mm (C) 250N/sq mm (D) 2.5N/sq mm
Ans:- (B)
Explanation:
The compressive strength of M-25 concrete is approximately 25 MPa (megapascals) or 25 N/sq. mm
(newtons per square millimetre) at 28 days of curing. This means that it can withstand a pressure of 25
megapascals before fracturing. The other options are either too low or too high compared to the actual
value.
Q.4 In Critical Path Method (CPM) for time scheduling, ‘forward pass calculation’ is carried out for
determining
(A) Late start and early finish time (B) Early start and early finish time
(C) Late start and late finish time (D) Early start and late finish time
Hemant Vilas Parulekar 211

Ans:- (B)
Explanation:
Forward pass calculation is a technique to move forward through the network diagram to determine
the project duration and find the critical path or free float of the project. It involves adding the estimated
duration to the activity’s early start date to get the early finish for the successor activity (or activities).
Q.5 Collapse of the World Trade Center (WTC), New York, in 2001, was due to
(A) Wind load failure (B) Foundation failure
(C) Thermal performance failure of reinforcement steel in RCC
(D) Thermal performance failure of structural steel
Ans:- (D)
Solution:
According to the National Institute of Standards and Technology (NIST), the collapse of the World
Trade Center (WTC) in 2001 was due to Thermal performance failure of structural steel1. The NIST
found that the fires caused by the plane impacts weakened the steel columns and floors, leading to their
buckling and collapse1. The other options (A, B, and C) are not supported by the evidence.
Skyscrapers are designed to support themselves for
three hours in a fire even if the sprinkler system fails to operate. This
time should be long enough to evacuate the occupants. The WTC
towers lasted for one to two hours—less than the design life, but only
because the fire fuel load was so large. No normal office fires would
fill 4,000 square meters of floor space in the seconds in which the
WTC fire developed. Usually,the fire would take up to an hour to spread so uniformly across the width
and breadth of the building. This was a very large and rapidly progressing fire.
Source.
Q.6 During the construction of tall buildings, the equipment used for hoisting building materials to
the upper floors is a
(A) Goods lift (B) Capsule lift (C) Gantry crane (D) Tower crane
Ans:- (D)
Type of crane Goods lift Capsule lift
Description A goods lift, also known as a cargo A capsule lift, also known as a panoramic
lift, is a machine that lifts goods elevator or glass elevator, is a type of lift
vertically in a safe way. It is used that is installed on the exterior portion
to transport heavy or bulky items of the building or in the lobby, where it
between different floors or levels, can enhance the look and elevation of
such as in warehouses, factories, the structure. Capsule lifts have glass
or shops. Goods lifts are designed walls that allow the passengers to enjoy a
to minimise manual labour and panoramic view of the surroundings while
increase efficiency. They can have travelling between floors. Capsule lifts can
different load capacities, sizes, and have different shapes, such as octagonal,
features depending on the needs of circular, or rectangular, depending on the
the customer. design and space available.
212 GATE Architecture and Planning: Comprehensive Question Bank

Image

Type of crane Gantry crane Tower crane


Description A gantry crane is a type of A tower crane is a type of fixed crane that
overhead crane that uses legs to is used to lift heavy loads to great heights
support the bridge, trolley, and in construction sites. It has an inverted
hoist. These legs travel on along L-shaped structure that consists of a base,
the ground on wheels or ride on a mast, a slewing unit, a working arm,
rails implanted in the ground. A a machinery arm, a hook and a trolley.
gantry crane is typically used for A tower crane can rotate 360 degrees
outdoor applications or for lifting and move the load horizontally and
capability below existing overhead vertically. A tower crane is self-erecting
bridge crane systems. and can increase its height as the building
rises.

Image

Q.7 A Rock-cut style of architecture is represented by


(A) Shyama Rama Temple, Bishnupur
(B) Kailasa Temple, Ellora
(C) Kandariya Mahadeva Temple, Khajuraho
(D) Sanchi Stupa, Sanchi
Hemant Vilas Parulekar 213

Ans:- (B)
Name of temple Shyama Rama Temple, Kailasa Temple, Ellora
Bishnupur
Description Shyama Rama Temple, Bishnupur, Rock-cut architecture is the creation of
is a terracotta temple built by the structures, buildings, and sculptures by
Malla kings in the 17th century excavating solid rock where it naturally
CE. The Shyama Raya temple, built occurs. The Kailasa temple, part of the
by Ragunath Singha in 1643 AD, Ellora Caves, is one of the most remarkable
is a ‘pancharatna’ or ‘five towered examples of rock-cut architecture in India.
temple’. It has a typical Bengali It is a monolithic structure, carved out of
curved cornice and stands on a low a single rock, and dedicated to the Hindu
platform. God Shiva.
Image

Name of temple Kandariya Mahadeva Temple, Sanchi Stupa, Sanchi


Khajuraho
Description Kandariya Mahadeva Temple, Sanchi Stupa, Sanchi, is a stone and brick
Khajuraho, is a sandstone temple structure built by the Mauryan emperor
built by the Chandela dynasty in Ashoka in the 3rd century BCE. Sanchi
the 11th century CE. Stupa is a Buddhist
the temple complex is spread over complex, famous for its Great Stupa,on a
an area of square kilometers (2.3sq hilltop at Sanchi Town in Raisen District
mi). It is in the western part of the of the State of Madhya Pradesh, India.
village to the west of the Vishnu
temple.
Image
214 GATE Architecture and Planning: Comprehensive Question Bank

Q.8 ‘Area based development’ and ‘Pan city development’ are part of
(A) Smart City Mission (B) Digital India Mission
(C) Swachh Bharat Mission (D) Atal Innovation Mission
Ans:- (A)
Explanation:
Development and pan city development are two of the strategic components of the Smart City Mission
(SCM) in India. Area based development involves city improvement, city renewal, and city extension,
while pan city development involves applying smart solutions to the existing city-wide infrastructure.
The Smart City Mission is a government initiative to promote cities that provide core infrastructure
and give a decent quality of life to its citizens, a clean and sustainable environment through the
application of ‘Smart’ solutions. The Mission aims to select 100 cities across India and provide financial
and technical support to them through various modes of collaboration. The Mission was launched by
the Prime Minister on 25 June, 2015.
Q.9 In mass transportation, LRTS stands for
(A) Light Rail Transit System (B) Linear Rail Transit System
(C) Light Rail Transportation System (D) Linear Rail Transportation System
Ans:- (A)
Explanation:
LRTS stands for Light Rail Transit System. This is a type of rapid transit that operates electric-powered
single cars or short trains on fixed rails. It is designed to provide quick and cost-effective service for
interurban transportation in a metropolitan area.
Q.10 The structural grid type shown in the figure below is a
(A) Tartan Grid (B) Square Grid
(C) Rectangular Grid (D) Irregular Grid

Ans:- (A)
Explanation:
Type of grid Tartan Grid Square Grid
Description A tartan grid is a type of structural grid A square grid is a type of geometric
that consists of sub-grids with varying pattern that consists of equally spaced
widths and distances, crossing at right horizontal and vertical lines that form
angles. It is often used in architectural squares. It is often used in mathematics,
design to create complex and dynamic graphing, design, and art. A square
spatial compositions. A tartan grid can grid can be seen as a special case of a
be seen as a variation of the nine-square rectangular grid, where the width and
grid, which is a basic geometric pattern height of the rectangles are equal.
for organizing spaces and elements.
Some examples of buildings that use
tartan grids are the Centre Pompidou in
Paris, the Yokohama International Port
Terminal, and the Seattle Central Library.
Hemant Vilas Parulekar 215

Image

Type of grid Rectangular Grid Irregular Grid


Description A rectangular grid is a type of An irregular grid is a type of geometric
geometric pattern that consists of pattern that consists of simple shapes,
equally spaced horizontal and vertical such as triangles or tetrahedra, in an
lines that form rectangles. It is often irregular arrangement. It is often used in
used in mathematics, graphing, finite element analysis when the input to
design, and art. be analyzed has an irregular shape.
Image

Q.11 Assuming other variables remaining constant, the Tropical Summer Index
(A) Increases with increase in air velocity (B) Decreases with increase in wet-bulb temperature
(C) Decreases with increase in globe temperature (D) Increases with increase in vapour pressure
Ans:- (D)
Explanation:
The Tropical Summer Index (TSI) is a measure of thermal comfort for people in tropical summer
conditions. It is defined as the temperature of calm air, at 50% relative humidity, which imparts the
same thermal sensation as the given environment.
(A) Increases with increase in air velocity: This is likely incorrect. Higher air velocity generally leads
to increased evaporation and convective heat loss, which would decrease the perceived temperature
and thus lower the TSI.
(B) Decreases with increase in wet-bulb temperature: This is likely incorrect. The wet-bulb
temperature is a measure of humidity and temperature. An increase in wet-bulb temperature indicates
higher humidity and/or higher air temperature, both of which would increase the TSI.
(C) Decreases with increase in globe temperature: This is likely incorrect. The globe temperature is
a measure of the combined effect of radiation and air temperature. An increase in globe temperature
would likely increase the TSI.
(D) Increases with increase in vapour pressure: This is likely correct. Higher vapour pressure indicates
higher humidity. Since humidity reduces the effectiveness of perspiration to evaporatively cool the
body, an increase in vapour pressure (and thus humidity) would increase the TSI.
216 GATE Architecture and Planning: Comprehensive Question Bank

Q.12 Government of India’s urban development program ‘HRIDAY’ stands for


(A) Heritage Rejuvenation Implementation Development Aayog Yojana
(B) Heritage Review Implementation Development Augmentation Yojana
(C) Heritage City Development and Augmentation Yojana
(D) Heritage City Improvement and Development Aawas Yojana
Ans:- (C)
Explanation:
The full form of ‘HRIDAY’ in the context of the Government of India’s urban development program is
Heritage City Development and Augmentation Yojana. This scheme was launched by the Ministry
of Housing and Urban Affairs on 21st January 2015 to preserve and revitalize the rich cultural heritage
of the country1. It aims at the development of heritage cities and tourism infrastructure in the country.
The HRIDAY scheme encourages the development of an aesthetically appealing, accessible, informative,
and secured environment in heritage cities. It has been implemented with quantifiable goals similar
to the implementation of the Smart City Scheme on a mission mode1. The focus of HRIDAY is the
development of core heritage infrastructure by the Ministry of Housing and Urban Affairs for areas
near heritage sites approved after identification by the Ministry of Culture.
Q.13 As per the Urban and Regional Development Plan Formulation and Implementation (URDPFI)
guidelines, the plan period considered in a ‘Perspective plan’ is
(A) 1-10 years (B) 11-15 years (C)20-30 years (D) 35-45 years
Ans:- (C) (Please refer GATE-2004, Q.44 for more information)
As per the Urban and Regional Development Plan Formulation and Implementation (URDPFI)
guidelines, a Perspective Plan should generally be for a period of 20 years and the plan period of 20-25
years should be so adjusted that it coincides with the term of the National/State Five Year Plans. This
facilitates the integration of spatial and economic policy planning initiatives.
Q.14 The Hall of Nations, New Delhi, was designed by
(A) Charles Correa (B) Raj Rewal (C) Joseph Allen Stein (D) A. P. Kanvinde
Ans:- (B)
Explanation:
The Hall of Nations, located in New Delhi, was designed by Raj
Rewal. It was inaugurated in 1972 as part of the Pragati Maidan
complex in New Delhi, India. The building was constructed
to celebrate 25 years of India’s independence and was used for
trade fair exhibitions. The Hall of Nations was a part of a group of four halls of varying sizes, each
interconnected by a system of ramps. The building was demolished in 2017, along with Hall No. 1 to
6, 14 to 20, state pavilions, and key neighboring structures like the Hall of Industries and the Nehru
Pavilion to make way for a new exhibition and convention center.
Q.15 As per the National Building Code of India 2016, the minimum turning radius (in meters) required
for fire tender movement is
(A) 8.0 (B) 8.5 (C) 9.0 (D) 9.5
Hemant Vilas Parulekar 217

Ans:- (C)
Explanation:
4.7 MEANS OF ACCESS (Chapter IV- General Buillding Requirements of NBC)
4.7.3 b) If there are any bends or curves in the approach road, sufficient width shall be permitted at the
curve to enable the fire tenders to turn, the turning circle shall be at least of 9.0 m. radius.
Q.16 Sidi Bashir Mosque with ‘Shaking Minarets’ is located in
(A) Ajmer (B) Allahabad (C) Ahmedabad (D) Amritsar

Ans:- (C)
Explanation:
Sidi Bashir Mosque, also known as the Jhulta Minar or Shaking
Minarets, is a former mosque located in Ahmedabad, Gujarat, India.
The mosque was built in 1452 by the Khan of Gujarat Sultanate and
is believed to have been constructed either by Sidi Bashir, a slave
of Sultan Ahmed Shah, or by Malik Sarang, a noble in the court
of Mahmud Begada, another Sultan of Gujarat. The mosque was
destroyed in 1753 during the war between the Marathas and the
Mughal governor of Gujarat, and only the central gateway and two
minarets survive. The minarets are the tallest in Ahmedabad and are three stories tall with carved
balconies.
Q.17 ‘Sight Distance’ is considered in the design of
(A) Road intersection (B) Fenestration (C) Open kitchen (D) Auditorium

Ans:- (A)
Explanation:
Type Road intersection Design of windows
Design type Sight distance Fenestration
Description Sight Distance is a critical factor in Fenestration is a term used in
the design of road intersections. It architecture to describe the arrangement,
is defined as the distance visible to proportioning, and design of windows
the driver of a vehicle at any given and doors in a building. It is an essential
time, which is essential for safe aspect of building design as it affects
driving and avoiding accidents. The the performance of the building and its
minimum sight distance required for aesthetics. Fenestration is usually viewed
a road intersection depends on the as an aesthetic feature, but it also affects
design speed of the road, the angle the performance of the building. The
of intersection, and the height of the National Building Code of India 2016
driver’s eye above the road surface. specifies the minimum requirements for
The National Building Code of India fenestration in buildings.
2016 specifies the minimum sight
distance requirements for different
types of roads and intersections.
Image
218 GATE Architecture and Planning: Comprehensive Question Bank

Type Open kitchen Auditorium


Design type Open plan kitchens Acoustic, seating design
Description Open kitchens are a popular design Auditoriums are a type of building
trend that combines the living and that is designed to accommodate large
cooking space in one area. They audiences for various events such as
are known for their versatility and concerts, plays, lectures, and more.
encourage casual family meals, They are typically designed with sloping
conversations, and effortless floors, good acoustics, and unobstructed
entertaining. Open kitchens are views of the stage or podium. The design
available in various styles, including of an auditorium is critical to ensure
modern, contemporary, minimalist, that the audience can see and hear the
and more. They are often designed performance clearly and comfortably.
with wide walkways and plentiful
windows to introduce an open feel and
lots of natural light.

Image

Q.19 In India, the term ‘Town Planning Scheme’ refers to


(A) Land renewal (B) Land rejuvenation
(C) Land reclamation (D) Land readjustment
Ans:- (D)
Explanation:
In India, the term Town Planning Scheme refers to a method of planning large contiguous parcels of land
through a systematic method of land pooling and readjustment. It is also known as land consolidation
and is based on the concept of land readjustment. The Town Planning Scheme is used to plan peri-
urban areas and is used sparingly for core area revitalization and post-disaster areas. Whereas, In India,
land renewal refers to the process of renewing a lease agreement for a property. The renewal process
is usually initiated by the lessee and requires the lessor’s approval. The terms and conditions of the
renewed lease agreement are usually negotiated between the two parties. Land reclamation is the
process of creating new land from oceans, riverbeds, or lake beds. In India, land reclamation has been
used to create new land for various purposes such as ports, airports, and urban development. Land
rejuvenation is a process of restoring degraded land to its original state or improving the quality of
the soil to make it more productive. In India, land rejuvenation has been used to restore degraded
agricultural land, improve soil fertility, and increase crop yields
Q.20 Bamboo is a type of
(A) Shrub (B) Timber (C) Evergreen Tree (D) Perennial Grass
Hemant Vilas Parulekar 219

Ans:- (D)
Explanation:
Bamboo is a group of woody perennial evergreen plants in the true
grass family Poaceae. It is not a shrub, timber, or an evergreen tree, but
rather a perennial grass. Bamboos are found in diverse climates, from
cold mountains to hot tropical regions. They are known for their
versatility and have notable economic and cultural significance in South
Asia, Southeast Asia, and East Asia, being used for building materials,
as a food source, and as a raw product, and depicted often in arts, such
as in bamboo paintings and bamboo working.
Q.21 The unit of measurement of Damp Proof Course (DPC) in building construction is
(A) kg (B) cu.m (C) sq.m (D) r.m
Ans:- (C)
Explanation:
The unit of measurement for Damp Proof Course (DPC) in building construction is square meters
(sq.m),it is the most commonly used unit of measurement for DPC in building construction. The DPC
is a layer of waterproof material that is applied to the foundation of a building to prevent moisture from
rising through the walls and causing damage . The DPC is usually installed at a height of 150 mm above
the ground level and is measured in square meters
Q.22 Which of the following is NOT a Building Information Modeling software Tool
(A) Adobe Illustrator (B) Bentley Microstation
(C) Autodesk Revit (D) Graphisoft ARCHICAD
Ans:- (A)
Explanation:
Adobe Illustrator is a vector graphics editor developed and marketed by Adobe Systems. Originally
designed for the Apple Macintosh, development of Adobe Illustrator began in1985. Along with Creative
Cloud, Illustrator CC was released.
Q.23 The concentric circles in a solar chart represent
(A) Azimuth angle (B) Altitude angle
(C) Horizontal shadow angle (D) Vertical shadow angle
Ans:- (B)
Explanation:
The concentric circles in a solar chart represent the angular height of the sun
above the horizon, measured in degrees. The azimuth angle, which is the
horizontal direction of the sun, is indicated by the radiating lines from the
center of the chart. The horizontal and vertical shadow angles are not shown
on a solar chart, but they can be calculated from the sun’s position and the
orientation of the surface.
How to read SOLAR CHART?
220 GATE Architecture and Planning: Comprehensive Question Bank

Sun path diagrams are used to read the solar azimuth and altitude throughout the day and year for a
given position on the earth. They can be likened to a photograph of the sky, taken looking straight up
towards the zenith, with a 180° fish-eye lens. The paths of the sun at different times of the year can then
be projected onto this flattened hemisphere for any location on Earth.
Azimuth Lines: Azimuth angles run around the edge of the diagram.
Altitude Lines: Altitude angles are represented as concentric circular dotted lines that run from the
center of the diagram out.
Date Lines: Date lines start on the eastern side of the graph and run to the western side and
represent the path of the sun on one particular day of the year. In Ecotec, the first day of January to June
are shown as solid lines, while July to December are shown as dotted lines.
Hour Lines/ Analemma: Hour lines are shown as figure-eight-type lines that intersect the date lines
and represent the position of the sun at a specific hour of the day. The intersection points between date
and hour lines give the position of the sun.
Q.24 A room of 3m×3m×3m has a reverberation time of 0.8sec.Using Sabine’s method, the total
absorption in the room is_____________sabin (upto one decimal place).
Ans:- 5.3 to 5.7
Solution:
Using Sabine’s method, the total absorption in the room is given
by the formula:

where A is the total absorption in metric sabins, V is the room volume in cubic meters, and Tr is the
reverberation time in seconds
Plugging in the given values, we get:

A=5.4
Q.25 A 25 storeyed building has 5 lifts. The resulting waiting time is 35 sec and ‘Return Travel Time’ is
175 sec. The number of lifts required for reducing waiting time to 25 sec, without increasing the
liftspeed, is___________

Ans:- 7.0 to 7.0


Solution:
we need to use the formula for the up peak interval (UPPINT), which is the average time between the
arrival of two consecutive lifts at the main terminal during the morning up peak. The formula is:

here RTT is the round trip time and L is the number of lifts
Given that the waiting time is 35 sec and the RTT is 175 sec, we can find the current UPPINT by:
Hemant Vilas Parulekar 221

To reduce the waiting time to 25 sec, we need to reduce the UPPINT to 25 sec as well. Therefore, we
can solve for the number of lifts required by:

Hence, the number of lifts required for reducing waiting time to 25 sec, without increasing the lift
speed, is 7.
Q. 26 – Q. 55 carry two marks each.
Q.26 Match the planning documents in Group-I with their respective government schemes in Group-II
(A) P-4,Q-1,R-5,S-2 Group I Group II
(B) P-3,Q-5,R-4,S-2 (P) Integrated Cluster Action Plan 1.NULM
(C) P-5,Q-1,R-4,S-3 (Q) Service Level Improvement Plan 2.Make in India
(R) Housing for All Plan Action 3.RuRBAN mission
(D) P-3,Q-5,R-4,S-1
(S) City Livelihood Centre Development 4.PMAY
Plan
5.AMRUT
Ans:- (D)
Explanation:
• The Integrated Cluster Action Plan (ICAP) is a document that outlines the vision, strategy,
and action plan for the development of a cluster of villages under the National Rurban Mission
(NRuM), also known as the RuRBAN mission. The mission aims to stimulate local economic
development, enhance basic services, and create well-planned Rurban clusters.
• The Service Level Improvement Plan (SLIP) is a document that describes the current status,
gaps, and improvement plans for the delivery of basic urban services such as water supply,
sewerage, storm water drainage, urban transport, green spaces, etc. under the Atal Mission for
Rejuvenation and Urban Transformation (AMRUT). The mission aims to improve the quality
of life and infrastructure in 500 cities.
• The Housing for All Plan of Action (HFAPoA) is a document that details the demand
assessment, project proposals, implementation strategy, and financing plan for the provision
of affordable housing for urban poor under the Pradhan Mantri Awas Yojana (PMAY). The
mission aims to ensure that every urban household has a pucca house with basic amenities by
20223.
• The City Livelihood Centre Development Plan (CLCDP) is a document that outlines the
objectives, activities, budget, and monitoring framework for the establishment and operation of
City Livelihood Centres (CLCs) under the National Urban Livelihoods Mission (NULM). The
mission aims to reduce poverty and vulnerability of urban poor households by enabling them to
access gainful self-employment and skilled wage employment opportunities.
222 GATE Architecture and Planning: Comprehensive Question Bank

Q.27 Associate the fire safety requirements for highrise buildings in Group-I with corresponding
standards of the National Building Code of India 2016 in Group-II
(A) P-4,Q-1,R-5,S-2 Group I Group II
(B) P-3,Q-5,R-4,S-1 (P)Mnimum refuge area 1.12.5 sq m/person
(C) P-3,Q-5,R-1,S-2 (Q) Maximum travel distance 2. 2.0m
(R) Maximum occupant load 3.0.3sq m/person
(D) P-4,Q-5,R-1,S-3
(S) Maximum staircase width 4.12.0 ton
5.30.0m
Ans:- (C)
Explanation:
According to the National Building Code of India 2016, the fire safety requirements for high rise
buildings are as follows:
• The minimum refuge area for each occupant shall be 0.3 sq m/person (Group I, P-3).
• The maximum travel distance to an exit or a protected lobby from the farthest point in any room
shall not exceed 30.0 m (Group I, Q-5).
• The maximum occupant load for any floor shall be calculated based on 12.5 sq m/person for
residential buildings and 10 sq m/person for other buildings (Group I, R-1).
• The minimum clear width of a staircase shall be 2.0 m for residential buildings and 1.5 m for
other buildings (Group I, S-2).
Q.28 Match the photometric quantities in Group-I with their respective units in Group-II
(A) P-3,Q-2,R-5,S-4 Group I Group II
(B) P-5,Q-4,R-2,S-1 Illuminance 1.Candela
(C) P-3,Q-5,R-1,S-2 (Q) Luminous Intensity 2. Candela/sq m
(R) Luminance 3.Lumens/sq m
(D) P-4,Q-5,R-1,S-3
(S) Luminous Efficacy 4.Lumens/watt
5.Lumens

Ans:- (C)
Explanation:
• The unit of illuminance is lux (lx), which is equal to one lumen per square meter. Illuminance
is a measure of how much light energy is incident on a surface per unit area.
• The unit of luminous intensity is the candela (cd), which is an SI base unit. It is defined as the
luminous flux per unit solid angle emitted by a point source in a given direction, weighted by
the human eye’s sensitivity to different wavelengths of light.
• The unit of luminance is the candela per square metre (cd/m), which is the SI unit of luminous
intensity per unit area of light travelling in a given direction.
• Luminous efficacy has unit lumen per watt.
Hemant Vilas Parulekar 223

Q.29 Associate the symbols in Group-I with their meanings in Group-II


(A) P-5,Q-3,R-1,S-2 Group I Group II
(B) P-5,Q-4,R-2,S-1 (P) 1.Hearing impaired
(C) P-5,Q-3,R-4,S-2
(D) P-3,Q-1,R-2,S-4

(Q) 2. Emergency lamp

(R) 3.Electrical and electronic waste disposal

(S) 4.Biohazard

5.speech impaired
Ans:- (C)
Image Meaning
Speech impaired

Electrical and electronic waste disposal

biohazards

Emergency lamp

Hearing impaired
224 GATE Architecture and Planning: Comprehensive Question Bank

Q.30 Match the elements in Group-I with the building components in Group-II
(A) P-5,Q-3,R-4,S-1 Group I Group II
(B) P-2,Q-4,R-3,S-1 (P)king post 1.Curtain glazing
(Q) Grade beam 2. Door
(C) P-2,Q-4,R-5,S-3 (R) Metal decking 3.plinth
(S) Jamb 4.Intermediate floor
(D) P-5,Q-3,R-4,S-2 5.Truss
Ans:- (D)
Elements King post Grade beam
Building Truss Plinth
components
Description This particular truss is made out of Grade beam and plinth beam are both
wood most of the time, but it can also be types of reinforced concrete beams
built out of a combination that are used in building foundations.
of steel and wood. It all comes down to However, they have some differences in
the architect and the building structure. their purpose, location, and design.
The King Post Truss spans up to 8m, Purpose: Grade beam is used to transmit
which makes it perfect for multiple types the load from a bearing wall into
of houses, especially the spaced foundations such as pile caps or
smaller ones. caissons. Plinth beam is used to prevent
the extension or propagation of cracks
from the foundation into the wall above.

Image

Elements Metal decking Jamb


Building Intermediate floor Door
components
Description Metal decking is a term for corrugated A jamb door is a door that has a jamb,
metal sheeting that is used as a structural which is one of the vertical parts of the
roof deck or composite floor deck. It is door frame that supports the hinges and
supported by steel beams or joists and latch. A jamb door can be an exterior or
provides a durable and lightweight interior door, and it can have different
platform for the installation of roofing types of jambs, such as flat, rabbeted, or
materials, insulation, concrete, or split. A jamb door is the most common
other finishes. An intermediate floor type of door in modern buildings.
is a floor that is located between the
ground floor and the top floor of a
building3. It may also refer to a floor
that is inserted within the height of an
existing floor, creating a mezzanine or a
loft. Intermediate floors
Hemant Vilas Parulekar 225

Image

Q.31 Match the iconic architectural examples in Group-I with their predominant structural systems in
Group-II
(A) P-1,Q-3,R-5,S-4 Group I Group II
(B) P-4,Q-1,R-2,S-3 S.Maria del Fiore Cathedral, 1.Shell
(C) P-4,Q-5,R-2,S-1 Florence
(Q) Notre Dame Cathdral,Paris 2. Suspended roof
(D) P-5,Q-4,R-3,S-2
(R) Olympic Arena, Tokyo 3.Space frame
(S) Bahai Temple,Delhi 4.Double-walled dome Supply Plan
5.Flying Buttress
Ans:- (B)
Explanation:
S.Maria del Fiore Cathedral, Florence (P) has a double-walled dome, which consists of an inner
and an outer shell connected by ribs and chains. The dome was designed by Filippo Brunelleschi
and completed in 1436. It is the largest masonry dome ever built and a masterpiece of Renaissance
architecture.
Notre Dame Cathedral, Paris (Q) has a shell structure, which is a thin curved surface that can withstand
external forces. The cathedral was built in the Gothic style between the 12th and 14th centuries. It
features a ribbed vault, a flying buttress, and a rose window. The cathedral suffered a major fire in 2019,
which damaged its roof and spire.
Olympic Arena, Tokyo ® has a suspended roof, which is a structure that hangs from cables or rods
attached to a supporting framework. The arena was designed by Kenzo Tange and completed in 1964.
It was the main venue for the 1964 Summer Olympics and the 2020 Summer Olympics.
Bahai Temple, Delhi (S) has a space frame, which is a three-dimensional truss that can span large
distances without intermediate supports. The temple was designed by Fariborz Sahba and completed
in 1986. It is a symbol of the Bahai faith and a place of worship for people of all religions. The temple
has a lotus-shaped structure made of 27 marble petals.
Q.32 Match the iconic architectural examples in Group-I with their predominant
P-5, Q-3, R-2, S-1 Group I Group II
(B) P-5, Q-4, R-1, S-2 (P)Cement 1.Charring
(C) P-1, Q-4, R-5, S-2 (Q) Steel 2. Brittle
(R) Wood 3.Evaporation
(D) P-4, Q-3, R-1, S-5
(S) Glass 4.Tensile strength
5.Setting Time
Ans:- (B)
226 GATE Architecture and Planning: Comprehensive Question Bank

Q.33 Match the built forms in Group-I with their descriptions in Group-II
P-1, Q-4, R-3, S-2 Group I Group II
(B) P-4, Q-3, R-1, S-5 (P)Agora 1.Custodial precincts
(C) P-4, Q-3, R-1, S-5 (Q) Ziggurat 2. Place of Jewish worship
(R) Mastaba 3.Built in diminishing stages of masonry with
(D) P-3, Q-1, R-5, S-2
buttressed wall
(S) Synagogue 4.Market place or public square
5.Tomb made of mud bricks
Ans:- (C)
Monument Agora Ziggurat
Description Market place or public square. Agora, Built in diminishing stages of masonry
in ancient Greek cities, an open space with buttressed wall. pyramidal stepped
that served as a meeting ground for temple tower that is an architectural
various activities of the citizens. The and religious structure characteristic of
name, first found in the works of the major cities of Mesopotamia (now
Homer, connotes both the assembly mainly inIraq) from approximately 2200
of the people as well as the physical until 500 bce.
setting.

Image

Monument Mastaba Synagogue


Description Tomb made of mud bricks (Arabic: Place of Jewish worship Synagogue,
“bench”) rectangular superstructure of also spelled synagog, in Judaism, a
ancient Egyptian tombs, built of mud community house of worship that serves
brick or, later, stone, with sloping walls as a place not only for liturgical services
and a flat roof. A deep shaft descended but also for assembly and study.
to the underground burial chamber.
The term mastaba was first used
archaeologically in the 19th century
by workmen on Auguste Mariette’s
excavation at Ṣaqqārah to describe
the rectangular, flattopped stone
superstructures of tombs.
Image
Hemant Vilas Parulekar 227

Q.34 Match the building configuration characteristics in Group-I with their seismic consequences in Group-
II
P-3, Q-1, R-2, S-4 Group I Group II
(B) P-2, Q-3, R-1, S-5 (P)Re-entrant corner 1. Soft storey
(C) P-4, Q-3, R-1, S-5 (Q) Floating column 2.Stress concentration at corner
(R) Irregular storey stiffness 3.Load path discontinuity
(D) P-3, Q-5, R-2, S-1
(S) Gap between adjacent buildings 4.Vertical asymmetry
5.Pounding
Ans:- (B)
Building Seismic consequences Building configuration
configuration
Re-entrant A re-entrant corner is an internal
corner corner of a structure that causes a
stress concentration due to the infinite
change in stiffness at the corner. This
can lead to cracking and failure of the
structure under certain conditions,
such as shrinkage, temperature
changes, or seismic loads.

Floating A floating column is a column that is


column not supported by the foundation, but by
a beam or a slab at an intermediate level
of a structure. A load path discontinuity
is a deviation or interruption in the
transfer of forces from the upper storeys
to the ground. Floating columns create
load path discontinuities, which can
reduce the seismic performance of
a structure and increase the risk of
collapse.
Irregular storey rregular storey stiffness is when the
stiffness stiffness of a storey is significantly
different from the adjacent storeys,
causing a sudden change in the lateral
stiffness distribution of the structure.
This can result in large inter-storey
drifts and damage concentration in the
irregular storey.
Soft storey is a storey that has much
less lateral stiffness and strength than
the storeys above, usually due to the
presence of large openings or reduced
columns. This can cause the soft storey
to collapse under lateral loads, leading
to the failure of the entire structure.
228 GATE Architecture and Planning: Comprehensive Question Bank

Gap between Gap between adjacent buildings is the


adjacent separation distance that is required to
buildings prevent the collision of nearby structures
during earthquakes. Pounding is the
phenomenon of impact between two
adjacent buildings that have different
dynamic properties or insufficient gap
distance. Seismic consequence is the
effect of earthquake-induced forces
and deformations on the structural
performance and safety.
Q.35 Match the landscaping terms in Group-I with their descriptions in Group-II
P-5, Q-3, R-1, S-4 Group I Group II
(B) P-3, Q-5, R-1, S-4 (P)Xeriscaping 1.Wide vegetated drain
(C) P-2, Q-3, R-1, S-5 (Q) Drip line 2. tree rings
(R) Swale 3.Outermost circumference of a tree canopy
(D) P-5, Q-2, R-4, S-1
(S) Truf bock paver 4.Solution to topsoil erosion and water
5, permeability
Ans:- (A)
Landscape Xeriscaping Drip line
term
Description Permeability, Xeriscaping is a type of Outermost circumference of a tree
landscaping that reduces or eliminates canopy. The drip line of a tree is the
the need for irrigation by using plants imaginary circle on the ground below
that are adapted to dry climates, such the outer edge of the tree’s canopy. This
as cacti, succulents, and native grasses. is where most of the water drips from
Xeriscaping can save water, money, the leaves and branches to the soil. The
and maintenance, as well as improve drip line is also important for the tree’s
biodiversity and mitigate urban heat. health, as it marks the area where most
of the critical roots are located.
Image
Hemant Vilas Parulekar 229

Landscape Swale Turf block paver


term
Description Wide vegetated drain, A swale is a Solution to topsoil erosion and water.
wide, shallow, vegetated channel that Turf block pavers are a type of permeable
is used to collect, convey, and filter pavement that allows grass to grow
stormwater runoff. A swale can reduce through the gaps between the concrete
erosion, increase infiltration, and blocks. They can provide a solution to
remove pollutants from the runoff. A topsoil erosion and water runoff by:
swale can also enhance the aesthetic
and ecological value of a landscape Stabilizing the soil and preventing it
by providing habitat for plants and from being washed away by rain or wind.
animals. Reducing the amount of stormwater
that flows over the surface and carries
pollutants to the waterways.

Image

Q.36 Match the planning principles in Group-I with their descriptions in Group-II
P-2,Q-1, R-4, S-3 Group I Group II
(B) P-2,Q-1, R-5, S-3 (P)Transit oriented development 1.Four stage model of regional
development
(C) P-4,Q-2, R-5, S-3
(Q) Core periphery theory 2. Compact and walkable mixed use
(D) P-2,Q-3, R-5, S-4 development
(R) Bid rent theory 3.Geographic concentration of
inter-connected institutions
(S) Cluster theory 4.Change of land price with relative
distance from the CBD
5.Interactive and participatory
planning process
Ans:- (A)
Explanation:
Planning concept theory Planning principles
planning that aims to create compact, mixed-use, and
walkable communities around public transport stations. TOD
can provide various benefits, such as:
A transit-oriented development (TOD) is a type of urban
Reducing traffic congestion, air pollution, and greenhouse
gas emissions by encouraging the use of public transport,
walking, and cycling.
230 GATE Architecture and Planning: Comprehensive Question Bank

Planning concept theory Planning principles


A transit-oriented development Enhancing the quality of life, health, and social interaction
of residents by creating vibrant, diverse, and livable
neighborhoods.
Increasing the accessibility, affordability, and equity of
housing, jobs, services, and amenities by providing a.

Core periphery theory Core periphery theory is a spatial and economic model that
explains the structural relationship between the developed
or metropolitan regions (core) and the less developed or
dependent regions (periphery) in the world. According to
this theory, the core regions are dominant and exploit the
periphery regions for their resources, labor, and markets,
creating a pattern of uneven development and global
inequality12. The core periphery theory is often associated
with the dependency theory and the world-systems theory,
which are critical perspectives on the global capitalist system

Bid rent theory The bid rent theory is a geographical economic theory that
refers to how the price and demand for real estate change
as the distance from the central business district (CBD)
increases. It states that different land users will compete with
one another for land close to the city centre. This is based
upon the idea that retail establishments wish to maximize
their profitability, so they are much more willing to pay more
for land close to the CBD and less for land.
Hemant Vilas Parulekar 231

Cluster theory Cluster theory is a theory of strategy that explains how a


concentration of related firms, organizations, and institutions
in a specific location can create competitive advantages
and economic benefits. Cluster theory was first introduced
by Alfred Marshall in 1890, who observed the existence
of industrial districts in England. Later, cluster theory was
developed and popularized by Michael Porter, who identified
four types of clusters: geographic, sectoral, horizontal, and
vertical.

Q.37 Match the cities in Group-I with their planners in Group-II


(A) P-2,Q-4,R-1,S-3 Group I Group II
(B) P-4,Q-1,R-5,S-2 (P) Islamabad 1. Patrick Geddess
(C) P-2,Q-1,R-5,S-3 (Q) Tel Aviv 2. C.A Doxiadis
(R) Bhubaneswar 3. Lucio Costa
(D) P-2,Q-3,R-4,S-5
(S) Brasilla 4. B.V Doshi
5. O.Koenigsberger

Ans:- (C)
Explanation:
(P) Islamabad was planned by (2) C.A. Doxiadis, a Greek urban planner who designed the city as a grid
of triangles2.
(Q) Tel Aviv was planned by (1) Patrick Geddess, a Scottish biologist and sociologist who applied his
ideas of regional planning to the city3.
(R) Bhubaneswar was planned by (5) O. Koenigsberger, a German architect and urban planner who
devised a modernist plan for the city based on the principles of the Garden City movement.
(S) Brasilia was planned by (3) Lucio Costa, a Brazilian architect and urban planner who conceived the
city as a cross-shaped layout with monumental axes.
Q.38 Match the Temples in Group-I with their dynamic period in Group-II
(A) P-4,Q-5,R-1,S-2 Group I Group II
(B) P-5,Q-1,R-2,S-3 (P)Brihadeshvara Temple 1. Gupta
(C) P-2,Q-5,R-1,S-3 (Q) Kailasanatha Temple 2. Chalukya
(R) Bhitragaon Temple 3. Lodhi
(D) P-4,Q-1,R-2,S-5
(S) Lad Khan Temple 4. Chola
5. Pallava
232 GATE Architecture and Planning: Comprehensive Question Bank

Ans:- (A)
Temple Brihadeshvara Temple Kailasanatha Temple
Dynasty period Chola Pallava period
Description Brihadeshvara Temple is a Hindu The Kailasanatha Temple is a historic
temple dedicated to Shiva, built by Hindu temple in Kanchipuram, Tamil
the Chola emperor Rajaraja I in Nadu, India. It is dedicated to Shiva and
the 11th century CE in Thanjavur, was built by the Pallava king Rajasimha
Tamil Nadu, India1 It is one of (Narasimhavarman II) in the 8th century
the largest and most magnificent CE. It is one of the oldest surviving
examples of Chola architecture, monuments in Kanchipuram and a fine
and a part of the UNESCO World example of Pallava architecture.
Heritage Site known as the Great
Living Chola Temples.
Temple

Temple Bhitragaon Temple Lad Khan Temple


Dynasty period Gupta period Chalukya period
Description The Bhitragaon Temple is a Hindu The Lad Khan Temple is an ancient Hindu
temple dedicated to Shiva, built temple dedicated to Shiva, located in
in the late 5th century CE during Aihole, Karnataka, India. It was built in the
the Gupta period in Bhitargaon, 5th century CE by the Chalukya dynasty,
Uttar Pradesh, India1. It is one of and is one of the oldest surviving temples
the oldest surviving brick temples in India. The temple has a rectangular plan
in India and a fine example of the with a square sanctum, and a pyramidal
Nagara style of temple architecture. roof that resembles a wooden structure.

Temple
Hemant Vilas Parulekar 233

Q.39 Match the buildings in Group-I with their architect period in Group-II
(A) P-3,Q-4,R-2,S-5 Group I Group II
(B) P-3,Q-4,R-1,S-2 (P)Guggenheim Museum, Bilbao 1. Richard Rogers
(C) P-2,Q-4,R-1,S-5 (Q) The Shard London 2. Norman Foster
(R) Commarz Bank, Frankfurt 3. Frank Gehry
(D) P-2,Q-5,R-4,S-3
(S) Heydar Aliyev Centre, Baku 4. Renzo Piano
5. Zaha Hdid
Ans:- (A)
Building Guggenheim Museum, Bilbao The Shard London
Architect Frank gehry Renzo Piano
Description The Guggenheim Museum Bilbao The Shard is a 72-storey, mixed-use tower
is a museum of modern and located beside London Bridge Station on
contemporary art designed by the south bank of the River Thames. It
the Canadian-American architect was designed by the Italian architect
Frank Gehry. It is located in Bilbao, Renzo Piano. The building resembles
in the Basque Country of Spain. an elongated pyramid that stands out
against the sky and vanishes through the
clouds.
Building

Building Commarz Bank, Frankfurt Heydar Aliyev Centre, Baku


Architect Norman Foster Zaha Hadid
Description Commerzbank is a German The Heydar Aliyev Centre is a
banking and financial services cultural complex in Baku, the capital
company based in Frankfurt am of Azerbaijan. It was designed by the
Main. It is the second largest bank famous architect Zaha Hadid, who is
in Germany by total assets and known for her fluid and futuristic style.
one of the largest banks in Europe. The centre is named after Heydar Aliyev,
It offers various products and the former president of Azerbaijan, and
services to private and corporate hosts various events and exhibitions
customers. related to art, history, and culture.
234 GATE Architecture and Planning: Comprehensive Question Bank

Building

Q.40 Match the following urban conservation themes in Group-I with their respective descriptions in
Group-II
(A) P-2,Q-5,R-4,S-3 Group I Group II
(B) P-2,Q-1,R-4,S-5 (P)Restoration 1. piece by piece re-assembly
(C) P-3,Q-2,R-1,S-4 (Q) Reconstitution 2. Returning to previous stage
(R) Reconstruction 3. Physical addiction
(D) P-3,Q-1,R-3,S-5
(S) Replication 4. Re-creation of vanished elements
5. Reproduction of an exact copy
Ans:- (B)
Explanation:
1. Restoration: Returning to previous stage. This means repairing the damaged fabric and materials to
restore the original form and style of a historic place.
2. Reconstitution: Piece by piece re-assembly. This means reconstructing the original form and style of
a historic place using old materials or new materials that are similar to the original.
3. Reconstruction: Re-creation of vanished elements. This means recreating the original form and
style of a historic place using any material, based on evidence and documentation.
4. Replication: Reproduction of an exact copy. This means making a copy of a historic place or an
element of it, using the same material and technique as the original.
Q.41 A single- phase neutral (SPN) electrical circuit has a power consumption of 330W. Considering a
voltage of 110V and power factor of 0.8, the electrical current drawn is________Amp (up to one
decimal place)
Ans:- 3.7-3.8
Solution:
The answer is 3.8 Amp (rounded up to one decimal place). You can find the answer by using the
formula:
P=VI×power factor
Where P is the power consumption, V is the voltage, I is the current, and power factor is the ratio of
real power to apparent power. Solving for I, we get:
Hemant Vilas Parulekar 235

I=V×power factor P
Plugging in the given values, we get:
I=110×0.8330
I=3.75
Rounding up to one decimal place, we get:
I=3.8
Therefore, the electrical current drawn is 3.8 Amp.
Q.42 A building with 100 sqm roof area is connected to 72 cum rainwater collection tank. If the rainfall
is 60 mm per hour and the loss during water storage is 20%, then the time taken to fill the tank
completely is_______ hours.

Ans:- 15
Solution:
To find the time taken to fill the tank completely, we need to calculate the amount of rainwater collected
per hour from the roof area, and then divide the tank capacity by that amount. We also need to account
for the 20% loss during water storage.
According to one formula1, the amount of rainwater collected in litters is equal to the roof area in
square meters multiplied by the precipitation amount in millimetres. So, the amount of rainwater
collected per hour from the 100 sqm roof area is:
100×60=6000 litters
However, since there is a 20% loss during water storage, only 80% of the collected rainwater actually
goes into the tank. So, the effective amount of rainwater collected per hour is:
6000×0.8=4800 litters
The tank capacity is 72 cum, which is equivalent to 72000 litters. So, the time taken to fill the tank
completely in hours is:
480072000=15 hours
Q.43 The planning norms for provision of schools in a given town is shown in the table below:
Schools Population norm Land requirement per school
Elementary School One per 2500 persons 0.4 hectare
Primary School One per 5000 persons 1.0 hectare
Secondary School One per 12500 persons 2.0 hectare
Total land area required for providing all types of schools for a population of 200,000 is ___________
hectares.
236 GATE Architecture and Planning: Comprehensive Question Bank

Ans:- (104)
Solution:
Schools Population norm Land requirement No. of schools Area required for
per school for population of schools (xy)
2,00,000(y)
Elementary One per 2500
0.4 hectare 80 32
School persons
Primary One per 5000
1.0 hectare 40 40
School persons
Secondary One per 12500
2.0 hectare 16 32
School persons
Total 136 104

Q.44 In a mixed use development on a 2.0 hectare site with 2.0 FAR, the ratio of residential to
commercial floor area is 3:2. The minimum parking (in ECS) needed per 100 sqm of residential
and commercial floor area is 1.0 and 1.25 respectively. Considering full FAR utilization, the total
parking requirement is ____________ ECS.
Ans:- (440)
Solution:
To find the total parking requirement in ECS, we need to calculate the total floor area of residential and
commercial spaces, and multiply them by their respective parking ratios.
The total floor area of the site is given by:
Total floor area=Site area×FAR
Substituting the given values, we get:
Total floor area=2.0 ha×2.0=4.0 ha
Since the ratio of residential to commercial floor area is 3:2, we can divide the total floor area into two
parts:
Residential floor area=3/5×Total floor area=3/5×4.0 ha=2.4 ha
Commercial floor area=2/5×Total floor area=2/5×4.0 ha=1.6 ha
Now, we can apply the minimum parking ratios for each type of floor area:
Now, we can apply the minimum parking ratios for each type of floor area:
Residential parking requirement=Residential floor area×1.0 ECS/100 sqm =2.4 ha×1.0 ECS/100 sqm
=240 ECS
Commercial market requirements = commercial floor area× 1.25ECS/100sqm = 1.6ha×
1.25ECS/100sqm=200ECS
Finally, we can add the two parking requirements to get the total parking requirement:
Total parking requirement = Residential parking requirement + Commercial parking requirement=
240ECS+200ECS=440 ECS.
Hemant Vilas Parulekar 237

Q.45 A plotted housing scheme on a site of 12 hectare has 60% saleable area. The average unit cost of
land development is INR 300 million per hectare. If the profit margin is 20%, then the selling price
of land per hectare is __________ million INR.

Ans:- (600)
Solution:
Total area of site = 12 hectare
Saleable area = 0.6 × 12 = 7.2 hectare
Average unit cost of land development = 300 million INR per hectare
Total cost of land development of 12 hectare = 12 × 300 = 3600 million INR
Selling price with 20% profit margin = 120/100 × 3600 = 4320 million INR
Selling price per hectare (for only 7.2-hectare saleable area) = 4320/7.2 = 600 million INR per hectare.
Q.46 An isolated enclosure shown in the Figure has inlet P and outlet Q of 2 sqm each, on the opposite
walls. The outdoor wind speed is 5 m/sec. If the coefficient of effectiveness is 0.6, the rate of
natural ventilation in the enclosure due to wind action is _________ cum/hr.

Ans:- (21600)
Solution:
Volume of air getting extracted from the Inlet/Outlet
Area of inlet/outlet × speed (V=A×S)
Here, The area of inlet and outlet = 2 sqm, and wind speed = 5m/sec.
Thus, volume of air getting extracted from inlet P per second = 2m² × 5m/sec = 10m³/sec.
As we know, 1 hr. = 60 min = 3600 sec.
Volume of air getting extracted inside from inlet P per hour = 10m³ × 3600/hr. = 36000 m³/hr.
If the co-efficient of effectiveness is 1 then, amount of air getting extracted from inlet = amount of air
extracted from outlet.
Here, co-efficient of effectiveness = 0.6
Thus, rate of natural ventilation per hour = 0.6 × amount of air coming inside from inlet P per hour =
0.6 × 36,000 = 21,600 m³/hour.
Q.47 A 5m × 5m × 3m room has four 230 mm thick external brick walls. Total wall fenestration is 10 sqm.
The temperature difference between indoor and outdoor is 2 deg C. The air to air transmittance
values for 230 mm thick brick wall and 200 mm thick aerated concrete block wall are 2.4 and 1.7
W/sqm deg C respectively. If the brick walls are replaced with the aerated concrete block walls,
then the change in conductive heat flow through the walls is _________W.
238 GATE Architecture and Planning: Comprehensive Question Bank

Ans:- (69.5-70.5)
Solution:
Here, Area of 4 walls of the room = 4 × (5 × 3) = 60 m², Height = 3m.
Total area of wall fenestration (Openings) = 10 m².
Thus, net external wall area= 50 m²
Thermal transmittance value of 230 mm thick brick wall (U-value of wall) = 2.4 W/sq.m. deg C.
Thermal transmittance value of 200 mm thick aerated concrete block wall (U-value of aerated concrete
wall) 1.7 W/sqm. deg C
Difference between outdoor and indoor temperature (At) = 2°
Conductive heat flow through brick wall
= (U-value of brick wall × area of wall × Δt) = (2.4 × 50 × 2)
= 240 watts. Conductive heat flow through aerated concrete block wall
= (U-value of ACC wall x area of wall × Δt)
= (1.7 × 50 × 2)
= 170 watts.
Thus, change in conductive heat flow through the wall due to change of materials
= 240 – 170
= 70 Watts.
Q.48 For an activity, ‘optimistic time duration’ is 4 days, ‘pessimistic time duration’ is 11 days and ‘most-
likely time duration’ is 8 days. The PERT value of time duration is______ days (up to one decimal
place).
Ans:- (7.8-7.9)
Solution:
The PERT formula to calculate the PERT estimate is as follows:
PERT Estimate = (Tp + 4Tm + To) / 6
Whereas,
Most Likely Estimate (Tm)
Optimistic Estimate (To)
Pessimistic Estimate (Tp)
Put values in formula,
{4+(4×8) +11}/6=7.83 days
Q.49 In the Figure, the negative bending moment at point A of the cantilever is ________ kNm.
Hemant Vilas Parulekar 239

Ans:- (224-226)
Solution:
For a cantilever beam length ‘L’ carrying a concentrated load ‘W’ at its
fixed, B. M.max = –(WL)
Here, due to point load B. M.max = (– WL) = (– 20 × 5) = – 100 kNm.
For a cantilever beam of length ‘L’ carrying on UDL ‘w’ over its entire spar B. M.max at fixed end =
(WL²/2)
Here, due to UDLB, M.max = (WL²/2)

Thus, total B.M. moment at point A (fixed end) = – 100 + (– 125) = – 225 kN.m.
Q.50 The water consumption of a high rise apartment building with 60 dwelling units having an average
household size of 5 persons is 135 lpcd. Assuming 80% of the total use is met with recycled water
supply, the daily domestic demand for the building is _________ litres.

Ans:- (8100)
Solution:
Total dwelling units = 60.
Average household size = 5.
Thus, total population in building = 300.
Thus, total water consumption demand of the building per day = 135 lpcd × 300 = 40,500 liters.
It is given that, 80% of the total use is met with recycled water.
Thus, (80 ×100) × 40,500 = 32,400 liters demand is met with recycled water.
Thus, the net daily domestic demand of water of the building = 40,500 – 32,400 = 8100 liters.
Q.51 In India, for 1.0 cum of M-20 grade concrete, the number of cement bags required is ________ (up
to two decimal places).
Ans:- (5 to 9)
Solution:
Ratio of M-20 Cement concrete: 1:1.5:3 So Total quantity of cement in 1 cum of cement concrete = 1/
(1+1.5+3) = 1/5.5 Cum 1 Cum of cement concrete = 2400 Kg So total quantity of cement = 2400/5.5 =
436.36 Kg Standard weight of each cement bag = 50Kg So total no. of cement bags = 436.36/50 = 8.72
Bags
Q.52 The sound power level of an outdoor non-directional point source is 90 dB. Considering an
atmospheric impedance of 400 rayls, the sound pressure level at 10 m distance from the source is
_______ dB.
Ans:- (58 to 60)
Solution:
Lp=Lw-|10.log(Q/4π.r2)|
Putting values of Lw = 90, r = 10 m, Lp = 59,00 dB
(No role of atmospheric impedance)
240 GATE Architecture and Planning: Comprehensive Question Bank

Q.53 The live load and dead load in a three storeyed residential building, transferred through a single
column, is 12 tons and 18 tons respectively. If the soil bearing capacity is 10 ton/sqm and the factor
of safety is 1.5, the area of column footing is _____________ sqm (up to one decimal place).
Ans:- (4.0 to 5.0)
Solution:
Given, Live load = 12 ton. Dead load = 18 ton.
Total load = 30 ton.
Factor of Safety = 1.5.
Thus, Factored total load = Factor of Safety × Total Load = 1.5 × 30 = 45 ton.
Soil bearing capacity is 10 ton/m².
Area of Column footing=Factored total load/ Soil Bearing Capacity =45/10= 4.5 m²
Q.54 The indoor illumination requirement for a building is 350 Lux. If the daylight factor is 2.7 and
the design sky illuminance is 9000 Lux, then the required supplementary artificial lighting is
____________ Lux.
Ans:- (107)
Solution:
The three components contributing to daylight factor are:
(a) sky component (SC),
(b) externally reflected component (ERC)
(c) Internally reflected component (IRC).
Thus, DF = SC + ERC + IRC
Daylight factor is also defined as the ratio of indoor illumination to the corresponding outdoor
illumination can be taken as constant.
This constant ratio, expressed in percentage, is the daylight factor (DF), given by :
DF = (Ei/E0)×100
Where, Ei = indoor illumination at the point of consideration & Eo = outdoor illumination from
unobstructed sky.
Here, DF = 2.7, Eo = 9000 Lux.
Assume the existing indoor illumination (Ei) = ‘P’ Lux.
2.7=(Ei/9000) )×100
2.7×9000/100=243
The indoor illumination required 350Lux thus supplementary artificial lightening required= 350-
243=107
Q.55 Two design options of a business building on a 10.0-hectare site are being compared for built up
area. Floor to floor height of Option A is 3.6 m and that of Option B is 4.5 m. If the maximum
allowable building height is 45 m with same ground coverage for both options, the additional
built-up area achievable in Option A over Option B is______percent.
Hemant Vilas Parulekar 241

Ans:- (20)
Solution:
To calculate the built-up area of a building, we need to add the carpet area, the area occupied by the
external walls, service ducts, balconies, etc.1 The built-up area depends on the floor-to-floor height,
the ground coverage, and the number of floors.
Assuming that the ground coverage is the same for both options, we can find the number of floors
by dividing the maximum allowable building height by the floor-to-floor height. For Option A, the
number of floors is 45/3.6 = 12.5, which we can round down to 12. For Option B, the number of floors
is 45/4.5 = 10.
Therefore, the built-up area of Option A is 12 times the ground coverage, and the built-up area of
Option B is 10 times the ground coverage. The additional built-up area achievable in Option A over
Option B is the difference between these two values, divided by the built-up area of Option B, and
multiplied by 100 to get the percentage.
Using formula,
|(12-10)×ground coverage/10×ground coverage|×100=20
GATE QUESTION PAPER 2017
General Aptitude (GA)

Q. 1 – Q. 25 carry one mark each.


Q.1 The Pritzker Architecture prize for the year 2016 has been awarded to
A) Alejandro Aravena (B) Frei Otto (C) Stephen Brayer (D) Yung Ho Chang
Ans:- (A)
Explanation:
Alejandro Aravena. He is a Chilean architect who practices architecture as an artful endeavour in
private commissions and in designs for the public realm. He also epitomizes the revival of a more
socially engaged architect, especially in his long-term commitment to tackling the global housing
crisis and fighting for a better urban environment for all. He received the 2016 Pritzker Architecture
Prize, which is considered the Nobel Prize of architecture, on April 4, 2016 at the United Nations
Headquarters in New York
Q.2 As per CPWD Handbook on Barrier Free and Accessibility, 2014 Government of India, the
minimum length of a straight ramp in meter to raise a wheelchair to the plinth level of 600 mm
is_________.
Ans:- 7.2
Solution:
According to the CPWD Handbook on Barrier Free and Accessibility, 2014 Government of
India, the minimum length of a straight ramp in meter to raise a wheelchair to the plinth level of 600
mm is 7.2 meters. This is based on the following formula:
Which implies, to achieve 600 mm plinth level height, length ‘x’ to be covered is
600/x = 1/12
x = 600 x 12
= 7200 mm or 7.2 m
Q.3 Tuscan and Composite Orders are associated with
(A) Greek Architecture (B) Islamic Architecture
(C) Byzantine Architecture (D) Roman Architecture
Ans:- (A)
Explanation:
Order Tuscan Composite
Description The Tuscan order is a simplified version The Composite order is a combination of
of the Doric order, with unfluted the Ionic and Corinthian orders, with a
columns and a plain entablature. It is capital that has both volutes and acanthus
considered the most rustic and solid leaves. It is the most ornamental and
of the orders, and was often used for elaborate of the orders, and was used for
military and utilitarian buildings. temples and public buildings that required
a grand and majestic appearance
Hemant Vilas Parulekar 243

Column

Q.4 A pointed arch having two centres and radii greater than the span is known as
(A) Lancet arch (B) Gothic arch (C) Roman arch (D) Drop arch
Ans:- (A)
Arch type Lancet arch Gothic arch
Description This is a tall, narrow arch with a pointed This is a wide, symmetrical arch with a
top. It is named after its resemblance pointed top. It is also called an ogival arch,
to a lance, a medieval weapon. Lancet and it is derived from the lancet arch.
arches are a typical feature of Gothic Gothic arches are also a common feature
architecture, and they can be found in of Gothic architecture, and they create a
churches, cathedrals, and other religious sense of height and lightness.
buildings.
Image

Arch type Roman arch Drop arch


Description This is a semicircular arch that is based This is a low, wide arch that is based on
on a circle. It is also called a round arch, an ellipse. It is also called a segmental
and it is one of the oldest and simplest arch, and it is a variation of the round
types of arches. arch.
Intersection
244 GATE Architecture and Planning: Comprehensive Question Bank

Q.5 The concepts of “serial vision, punctuation’ and “closure’ were proposed by
(A) Le Corbusier (B) Louis Kahn (C) Gorden Cullen (D) Kevin Lynch
Ans:- (C)
Explanation:
Gordon Cullen, a British architect and urban designer. He coined the term “serial vision” in his book
“The Concise Townscape” (1961), where he explained how urban experience is a series of revelations,
with delight and interest being stimulated by contrasts. He also used the terms “punctuation” and
“closure” to describe how urban design can create a sense of rhythm and surprise for the observer.
Q.6 In one litre of paint, volume of solid pigment and volume of non-volatile binder are 400 cc and 600
cc respectively. The Pigment Volume Concentration number of the paint is -
Ans:- 0.4
Solution:
The Pigment Volume Concentration (PVC) number of the paint is a measure of the ratio of the volume
of pigment to the total volume of the paint. It can be calculated using the following formula:
Given, quantity of paint: 1 liter
Volume of solid pigment = 400 cc
Volume of non-volatile binder = 600 cc
Volume of solid pigment
Therefore, pigment volume concentration number =
Total quantity of paint
= 400/ (400+600)
= 400/1000
= 0.4
Q.7 Cold joint’ refers to the
(A) Expansion joint in large span concrete members
(B) Interface between an already setting concrete and a fresh batch of concrete
(C) Structural crack arrested by embedding metal rods
(D) Joining of two similar metals in vacuum
Ans:- (B)
Explanation:
A cold joint is a plane of weakness in concrete caused by an interruption or delay in the concreting
operations. It occurs when the first batch of concrete has begun to set before the next batch is added, so
that the two batches do not intermix. Cold joints can reduce the strength and durability of concrete, and
may cause water leakages or deterioration. To avoid cold joints, concrete should be placed continuously
and rapidly, and the surface of the first batch should be roughened and cleaned before placing the next
batch.
Q.8 Slenderness ratio of a column is represented as:
(A) Effective length/Cross-sectional area (B) Effective length/Radius of gyration
(C) Actual length/Cross-sectional area (D) Actual length/Radius of gyration
Hemant Vilas Parulekar 245

Ans:- (B)
Explanation:
The slenderness ratio of a column is a measure of the tendency of the column to buckle under
compressive loads. It is defined as the ratio of the effective length of the column, which depends on the
end conditions and the degree of restraint, to the radius of gyration of the cross-sectional area, which
reflects the shape and size of the column.
Q.9 Liquidated damage refers to the
(A) cost borne by the contractor to rectify defects within defect-liability period
(B) compensation paid on breach of contract to the affected party by the other party
(C) money paid by the insurance company to the owner of insured property if it is damaged
(D) money earned by the owner from selling damaged property through auction
Ans:- (B)
Explanation:
Liquidated damages are damages that are specified by the parties to a contract as they are drawing
up the contract. This part of a contract specifies that, in the event one party breaches the contract, he
must pay a specified amount to the other party for his losses. Liquidated damages are meant as a fair
representation of losses in situations where actual damages are difficult to ascertain or prove.
Q.10 Which of the following process is NOT used for corrosion resistance of cast iron?
(A) Painting (B) Epoxy coating (C) Quenching (D) Galvanizing
Ans:- (C)
Explanation:
Quenching is a process of rapid cooling of metal from a high temperature to a low temperature, usually
by immersing it in water, oil, or air. Quenching is used to increase the hardness and strength of metal,
but it does not improve its corrosion resistance. In fact, quenching can make metal more susceptible to
corrosion by creating internal stresses and cracks.
Q.11 Data on ‘households with one or more married couples sharing room with a person aged 12 years
or more’, is used for computing:
(A) house density (B) housing shortage
(C) housing price (D) housing affordability
Ans:- (B)
Explanation:
Housing shortage is the gap between the demand and supply of adequate and affordable housing in
a given area. According to the Census of India 2011, there were 6.54 million households with one or
more married couples sharing room with a person aged 12 years or more, accounting for 5.4% of the
total households in the country.
Q.12 Excellence in Design for Greater Efficiency (EDGE) programme DOES NOT focus on
(A) lower carbon emission (B) greater resource efficiency
(C) cost effectiveness (D) labour safety
246 GATE Architecture and Planning: Comprehensive Question Bank

Ans:- (D)
Explanation:
Excellence in Design for Greater Efficiency (EDGE) is a green building certification system for
emerging markets, created by the International Finance Corporation (IFC), a member of the World
Bank Group. EDGE focuses on making new residential and commercial buildings more resource-
efficient, by reducing their energy, water, and embodied energy in materials consumption. EDGE also
aims to lower carbon emissions and enhance cost effectiveness of green buildings.
Q.13 Select the right option representing strategic components arranged in ascending order of specified
minimum area under Smart City Mission of Government of India.
(A) Greenfield Development-Redevelopment-Retrofitting
(B) Redevelopment-Greenfield Development-Retrofitting
(C) Retrofitting-Redevelopment-Greenfield development
(D) Redevelopment-Retrofitting-Greenfield development
Ans:- (B)
Explanation:
According to the Smart Cities Mission, the strategic components of area-based development are as
follows:
Retrofitting: This involves planning and improving the existing built-up area, which may be more than
500 acres, to make it more efficient and livable. The minimum area under this component is 50 acres.
Redevelopment: This involves replacing the existing built-up area, which may be more than 50 acres,
with a new layout, enhanced infrastructure, mixed land use, and increased density. The minimum area
under this component is 50 acres.
Greenfield development: This involves introducing most of the smart solutions in a previously vacant
area, which may be more than 250 acres, using innovative planning and financing. The minimum area
under this component is 250 acres.
Q.14 The grade-separated interchange suitable for 3-legged intersection is:
(A) Trumpet (B) Full Clover leaf (C) Diamond (D) Partial Clover leaf
Ans:- (A)
intersection Trumpet Full Clover Leaf
Description A trumpet interchange is a type of A full cloverleaf interchange has four
grade-separated interchange that is legs and eight ramps, four of them are
used where one highway terminates at loop ramps and four of them are direct
another highway. It has three legs, two of interchange ramps. A full cloverleaf
them are direct interchange ramps and interchange is used where two highways
the third one is a loop ramp that allows cross with each other.
traffic to change direction. A trumpet
interchange is also called a three-leg
interchange.
Hemant Vilas Parulekar 247

Intersection

intersection Diamond Partial Clover Leaf


Description A diamond interchange has four A partial cloverleaf interchange has
legs and four ramps, all of them are four legs and six ramps, two of them are
direct interchange ramps. A diamond loop ramps and four of them are direct
interchange is also used where two interchange ramps. A partial cloverleaf
highways cross with each other. interchange is a variation of the full
cloverleaf interchange, where some of the
loop ramps are replaced by direct ramps to
reduce weaving and save space.
Intersection

Q.15 The design element provided to ensure safety of a vehicle travelling at a prescribe design speed
along the curved segment of a highway is
(A) shoulder (B) super-elevation (C) median (D) footpath
Ans:- (B)
Explanation:
Super elevation is the amount by which the outer edge of a curve on a road or railway is banked above
the inner edge. This helps to counteract the centrifugal force acting on the vehicle and maintain a safe
and comfortable speed along the curve.
Q.16 Which of the following processes is NOT adopted in solid waste management?
(A) Incineration (B) Pyrolysis (C) Flocculation (D) Sanitary landfill
Ans:- (C)
Explanation:
Flocculation is a process of water treatment, not solid waste management. It involves the aggregation of
suspended particles into larger clumps, called flocs, that can be removed by filtration or sedimentation.
The other three options are all methods of solid waste disposal and management. Incineration is the
burning of solid waste at high temperatures, reducing its volume and converting it into ash, gas, and
heat. Pyrolysis is the thermal decomposition of organic waste in the absence of oxygen, producing
char, oil, and gas. Sanitary landfill is the controlled disposal of solid waste on land, with compaction,
cover, and leachate collection.
248 GATE Architecture and Planning: Comprehensive Question Bank

Q.17 The principle of Eminent Domain is the power to


(A) restrict exercise of rights in land through zoning and environmental laws
(B) control land use
(C) retain land use
(D) acquire and take possession of property in order to promote public interest
Ans:- (D)
Explanation:
Eminent domain is the power of the government to take private property and put it to public use,
following the payment of just compensation. This power can be used for various public purposes, such
as roads, government buildings, public utilities, or urban redevelopment. Eminent domain can also
include other types of property, such as airspace, water, contracts, or intellectual property.
Q.18 In which of the following models does the private partner own the revenue as well as the risk
associated with the project for a limited period of time?
(A) Build, Own, operate (BOO) (B) Build, Own, Operate, Transfer(BOOT)
(C) Design, Build, Finance, Operate (DBFO) (D) Design, BID, Build (DBB)
Ans:- (B)
Explanation:
In Build, Own, Operate, Transfer (BOOT), the private partner is responsible for designing, building,
financing, operating, and maintaining the project for a specified period of time, after which the
ownership is transferred to the public partner. The private partner also bears the revenue and demand
risk, as well as the construction and operation risk, during the concession period. The public partner
usually provides some form of support, such as land, subsidies, or guarantees, to make the project
viable.
The other options are not correct. In (A) Build, Own, operate (BOO), the private partner retains
the ownership of the project indefinitely and does not transfer it to the public partner. In (C) Design,
Build, Finance, operate (DBFO), the private partner does not own the project, but leases it from the
public partner and pays a rent for using the asset. In (D) Design, Bid, Build (DBB), the private partner
only designs and builds the project, while the public partner finances, operates, and maintains it.
Q.19 In a multi-storied building, the type of plumbing system suitable for reusing the sullage for non-
potable use is
(A) single stack system (B) partially ventilated single stack system
(C) one pipe system (D) two pipe system
Ans:- (D)
Explanation:
Two pipe system is the system in which the sullage (or grey water) from kitchens, baths, showers,
basins, and laundries is collected separately from the sewage (or blackwater) from toilets. The sullage
can then be treated and reused for non-potable purposes, such as irrigation, toilet flushing, or cooling.
The sewage, on the other hand, is disposed of in a sewer or a septic tank. The other options are not
suitable for reusing the sullage, as they either mix the sullage and the sewage together, or do not provide
adequate ventilation or drainage for the plumbing system
Hemant Vilas Parulekar 249

Q.20 The unit for measuring sound absorption in a room is


(A) Sabin (B) Phon (C) Decibel (D) Hertz
Ans:- (A)
Explanation:
A Sabin is a unit of sound absorption, used for expressing the total effective absorption for the interior
of a room. It is named after Wallace Clement Sabine, the founder of the field of architectural acoustics.
One Sabin is equal to the absorption of one square foot (or one square meter) of a perfectly absorbing
surface. The total room sound absorption in Sabin is calculated by multiplying the surface area and the
absorption coefficient of each surface in the room.
Q.21 In Geographic Information System, DEM represents information on
(A) vegetation cover (B) soil type (C) water table (D) topography
Ans:- (D)
Explanation:
A Digital Elevation Model (DEM) is a type of raster GIS layer that represents the bare ground (bare
earth) topographic surface of the Earth excluding trees, buildings, and any other surface objects. DEMs
are created from a variety of sources, such as photogrammetry, lidar, IfSAR, or land surveying. DEMs
are used often in geographic information systems (GIS), and are the most common basis for digitally
produced relief maps.
Q.22 Minimum point required for GRIHA certification is
(A) 35 (B) 40 (C) 50 (D) 60
Ans:- (C) 50
Explanation:
According to the GRIHA website, the new rating threshold for GRIHA rating system is as follows:
GRIHA Rating Points Required
1 Star 50-60
2 Star 61-70
3 Star 71-80
4 Star 81-90
5 Star 91-100

Q.23 ArchiCAD, Auto Desk Revit, Digital Project Designer (CATIA) and Vector Works Architect are
examples of
(A) Statistical Analysis software (B) GIS software
(C) BIM software (D) Image processing software
Ans:- (C)
Explanation:
BIM software. BIM stands for Building Information Modelling, and it is a process of creating and
managing digital representations of physical and functional characteristics of buildings and facilities.
BIM software enables architects, engineers, contractors, and other stakeholders to collaborate on the
design, construction, and operation of buildings and facilities using a common data environment.
250 GATE Architecture and Planning: Comprehensive Question Bank

Q.24 The CARTOSAT 2C satellite recently launched by ISRO


(A) is a geo-synchronous satellite (B)is a part of IRNSS GPS satellite system
(C) was launched using a GSLV rocket (D)has high spatial resolution
Ans:- (D)
Explanation:
Cartosat-2C is an Earth observation satellite in a Sun-synchronous orbit (SSO) and is a fifth flight
unit of CARTOSAT series of satellites. It is not a geo-synchronous satellite, which means it does not
appear stationary over a place on the earth. It is also not a part of IRNSS GPS satellite system, which
is a regional navigation satellite system developed by ISRO. It was launched using a PSLV rocket, not
a GSLV rocket, which is a heavier launch vehicle. Cartosat-2C has a spatial resolution of 0.6 metres,
which means it can capture images with very fine details.
Q.25 Which of the following trees has a columnar form?
(A) Delonix regia (B) Tamarindus indica
(C) Polyalthia longifolia (D) Callistemon Lanceolatus
Ans:- (C)
Explanation:
Botanical name Delonix regia Tamarindus indica
Common name Gulmohar Tamarind/ imli
Description Delonix regia is a deciduous tree that Tamarindus indica is a large, evergreen
has a broad, spreading crown and tree that has a dense, rounded canopy
bright red flowers. It is noted for its and edible pods. The tamarind tree has a
fern-like leaves and flamboyant display broad, spreading crown with feathery
of orange-red flowers over summer. leaflets that close up at night. The tree
In many tropical parts of the world, can grow up to 25 metres in height and
it is grown as an ornamental tree and has a hard, brown shell that contains a
in English, it is given the name royal sweet and sour pulp. The flowers are
poinciana, flamboyant, phoenix flower. yellow with red streaks and the seeds
are flattened and glossy brown
Uses Delonix regia is an excellent flowering Making furniture, flooring, carvings,
tree for gardens, parks, along streets, and other wood items, providing shade
and for large front yards where it is and ornamentation along roadsides
winter hardy. Mature trees provide and in gardens.
plenty of shade. It can also be used as a
street tree where space is enough, as it
has a typical T shape
Image
Hemant Vilas Parulekar 251

Botanical name Polyalthia longifolia Callistemon Lanceolatus


Common name Ashoka Crimson or lemon bottlebrush
Description This is a tropical evergreen tree that has Callistemon lanceolatus is a shrub or
a slender, columnar shape and grows small tree that has a bushy, irregular
up to 30 m tall. It is native to India and shape and red bottlebrush-like flowers.
Sri Lanka, and is commonly planted as It is commonly known as “bottlebrush”
an ornamental or avenue tree. It has because of its cylindrical, brush-like
long, narrow, dark green leaves that flowers resembling a traditional bottle
are arranged in a spiral pattern along brush.
the branches. It also produces small,
greenish-yellow flowers and purple-
black fruits.
Uses Providing shade and ornamentation The plant is widely used as a
along roadsides, parks, and gardens. landscaping plant due mostly to
Making furniture, flooring, carvings, its adaptability, hardiness, and the
and other wood items with its strong, stunning flowers which are a great
durable, and termite-resistant wood attraction for native nectar feeding
and building roofs, tents, and tools for wildlife including insects, butterflies,
camels in some regions. and birds
Image

Q.26 Match the architectural movements in Group-I with their proponents in Group-ll
(A) P-5,Q-1,R-2,S-4 Group I Group II
(B) P-5,Q-4,R-2,S-3 Deconstruction Joseph Paxton
(C) P-5,Q-2,R-3,S-3 Historicism Kenzo Tange
(R) Metabolism Walter Gropius
(D) P-2,Q-4,R-1,S-5
(S) Art Nouveau Victor Horta
Frank O. Gehry
252 GATE Architecture and Planning: Comprehensive Question Bank

Ans:- (A)
Architectural Deconstruction Historicism
Movement
Proponent Frank O. Gehry Joseph Paxton
Description Deconstruction is an architectural Historicism is an architectural
movement that emerged in the movement that emerged in the 19th
1980s, challenging the conventional century, which sought to revive and
notions of harmony, unity, and reinterpret the styles and elements
stability in design. Frank O. Gehry of the past in new and original ways.
is one of the most prominent and Joseph Paxton was a British architect,
influential architects associated with engineer, and gardener, who is best
deconstruction, although he has never known for designing the Crystal
considered himself a deconstructivist. Palace for the Great Exhibition of
His buildings often feature fragmented, 1851 in London. He was influenced
irregular, and dynamic forms that defy by historicism in his use of Gothic,
easy categorization and create a sense Classical, and Renaissance motifs
of visual complexity and movement. in his buildings, as well as by his
Some of his most famous works that admiration for the French architect
exemplify deconstruction are the Eugène Viollet-le-Duc, who advocated
Guggenheim Museum in Bilbao, for the restoration and adaptation of
Spain, which has a curvilinear and medieval structures.
shiny exterior that resembles a ship or
a fish.
Type of work

Architectural Metabolism Art Nouveau


Movement
Proponent Kenzo Tange Victor Horta
Description Metabolism was a post-war Victor Pierre Horta was a Belgian
Japanese architectural movement architect and designer, and one of
that fused ideas about architectural the founders of the Art Nouveau
megastructures with those of organic movement. He was a fervent admirer
biological growth. Kenzo Tange was of the French architectural theorist
one of the founders and leaders of this Eugène Viollet-le-Duc and his Hôtel
movement, and he proposed visionary Tassel in Brussels, often considered
plans for Tokyo and other cities that the first Art Nouveau house, is based
embodied the Metabolist ideals of on the work of Viollet-le-Duc. The
social change, urban expansion, and curving stylized vegetal forms that
biomimetic design. Horta used in turn influenced many
others, including
Hemant Vilas Parulekar 253

Type of work

Q.27 Associate the historic buildings in Group-I with their predominant materials in Group-II
(A) P-1, Q-2, R-3, S-5 Group I Group II
(B) P-1, Q-4, R-3, S-5 (P) Lingaraj Temple, Bhubaneshwar, India 1. Red sandstone
(C) P-2, Q-1, R-3, S-4 (Q) Victoria Memorial, Kolkata, India 2. Timber
(R) Padmanabhapuram Palace, 3. Terracotta tiles
(D) P-4, Q-5, R-2, S-1
Thuckalay, India
(S)Humayun’s Tomb, Delhi, India 4. Sandstone and laterite
5. Marble
Ans:- (D)
Building Lingaraj Temple, Bhubaneshwar, Victoria Memorial, Kolkata, India
India
Predominant Sandstone and laterite Marble
material
Image

Building Padmanabhapuram Palace, Humayun’s Tomb, Delhi, India


Thuckalay, India
Predominant Timber Red sand stone
material
Image
254 GATE Architecture and Planning: Comprehensive Question Bank

Q.28 Match the terminologies in Group-I with their description in Group-II


Group I Group II
(P) Prunning 1.Cutting of trees
(Q) Felling 2.Removing broken branches from trees for better growth
(R) Hoeing 3. Maintaining moisture content in soil by protective layer.
(S) Mulching 4. Indiscriminate cutting of branches to reduce all size of a tree
5. Loosening the ground to remove weeds
(A) P-2, Q-1, R-5, S-3 (B) P-2, Q-1, R-4, S-3 (C) P-2, Q-1, R-3, S-4 (D) P-1, Q-2, R-3, S-1
Ans:- (A)
Pruning is removing broken branches from Felling is the process of cutting down trees, an
trees for better growth. element of the task of logging.

Hoeing is the process of using a long-handled Mulching is the process of covering the soil surface
tool with a thin, flat blade to break up the surface with a layer of organic or inorganic material, such
of the soil, remove weeds, and cultivate plants. as wood chips, leaves, straw etc.it is very beneficial
for plants.

Q.29 A proposed housing will have HIG, MIG and LIG units on a site measuring 60,750 sq.m. The
buildable area of each category of units with respect to total buildable area will be 30%, 50% and
20% respectively. The maximum allowed FAR is 2.5, ground coverage 45% and height 15m. The
maximum available buildable area in sq. m of HIG units, considering a floor height of 3m for all
categories will be _______
Ans:- 41,000
Solution:
To find the maximum available buildable area of HIG units, we need to use the following information
and formulas:
• The site area is 60,750 sq.m
Hemant Vilas Parulekar 255

• The buildable area of HIG units is 30% of the total buildable area.
• The maximum allowed FAR is 2.5, which means the total buildable area cannot exceed 2.5 times
the site area.
• The maximum allowed ground coverage is 45%, which means the total ground floor area cannot
exceed 45% of the site area.
• The maximum allowed height is 15m, which means the number of floors cannot exceed 15m
divided by the floor height of 3m.
The total buildable area is the minimum of the following two values:
• the product of the site area and the FAR:
60,750×2.5=151,875
• The product of the site area, the ground coverage, and the number of floors:
60,750×0.45×315=136,687.5
Therefore, the total buildable area is 136,687.5 sq.m.
The buildable area of HIG units is 30% of the total buildable area, which is:
136,687.5×0.3=41,006.25
Maximum available buildable area of HIG = 8,201.25 x 5 = 41,006.25 say 41,000 sq. mt
Q.30 In 2011, the population of a town was 5,00,000 and the number of housing units were 1,00,000.
Calculate the additional number of dwelling units (DU) required by 2031 so that there is no
housing shortage. The assumptions are
5% decadal increase in population
ii. New DU to be completed by 2021 is 10,000
iii. Number of DU which will become non habitable by 2031 is 5,000
iv. Average household size is 4.5
Ans:- 17,500
Solution:
Population 2011= 5,00,000
Decadal increase=5%
Net increase in population in first decade (2021) = 25,000
Population in 2021= 525,000
Net increase in population in second decade (2031) = 26,500
Population in (2031) =5,51250
Number of DU as per population (considering average HH size 4.5) in 2011=5,00,000/4.5= 1,11,111
Existing DU in 2011=1,00,000
Housing shortage in 2011=11,111
256 GATE Architecture and Planning: Comprehensive Question Bank

Number of DU as per population (considering average HH size 4.5) in 2021= 5,25,000/4.5=1,16,666


New DU completed by 2021=10,000
Net housing shortage after 2021=1,16,666-1,11,111+1,111=6666
Number of DU as per population (considering average HH size 4.5) in 2031=5,51,250/4.5=1,22,500
Net housing shortage after 2031= 5,000
Net shortage upto 20231= 12,500+5,000
Additional number of DU required by 2031=17,500
Q.31 Match the classical urban planning theories in Group-I with their proponents in Group-II
(A) P-4, Q-1, R-3, S-5 Group I Group II
(B) P-3, Q-4, R-3, S-5 (P)Concentric Zone Model 1. Berry and Horton
(C) P-2, Q-4, R-5, S-1 (Q)Sector Model 2. Homer Hoyt
(R) Multiple Nuclei Model 3. Ernest Burgess
(D) P-3, Q-2, R-5, S-1
(S) Factorial Ecology 4. Shevky and Bell
5. Harris and Ullman
Ans:- (D)
Explanation:
Theory Concentric zone model Sector model
Proponent Ernest Burgess Homer Hoyt
Description The concentric zone model is a The sector model, also known as the
theory used in human geography to Hoyt model, is a model of urban land
explain the patterns of urban land use proposed in 1939 by land economist
use. The model was first proposed Homer Hoyt. It is a modification of
by sociologist Ernest Burgess in the concentric zone model of city
1925 and has since been refined by development. The benefits of the
other researchers. The concentric application of this model include the fact
zone model posits that cities grow it allows for an outward progression of
outward from a central business growth.
district in a series of rings, each
characterized by distinct land uses
and socio-economic characteristics.
Image
Hemant Vilas Parulekar 257

Theory Multiple Nuclei Model Factorial Ecology


Proponent Harris and Ullman Berry and Horton
Description The multiple nuclei model is a theory Factorial ecology aims to identify the
of urban land use that was proposed by underlying dimensions of urban structure
Chauncy Harris and Edward Ullman and differentiation, such as social status,
in 1945. The model suggests that cities ethnicity, land use, and population
do not grow around a single center, but density. Berry and Horton are two
rather have several nuclei or centers prominent geographers who contributed
of different activities and functions. to the development and application of
These nuclei develop according to the factorial ecology. They co-authored a
transportation routes, the land values, book called Geographic Perspectives on
and the preferences of different social Urban Systems (1970), which presented a
groups and economic sectors. comparative analysis of 25 cities around
the world using factorial ecology and
other techniques.
Image

Q.32 Match the distinguished housing projects in Group-I with their architects in Group-II
(A)P-5, Q-4, R-2, S-3 Group I Group II
(B) P-1, Q-3, R-4,S-5 (P)Nakagin Capsule Tower, Tokyo, Japan 1. Walter Gropius
(C) P-5, Q-2, R-1, S-4 (Q) Tara Apartments, New Delhi, India 2. Moshe Safdie
(R) Habitat 67, Montreal, Canada 3. Ralph Erskine
(D) P-5, Q-4, R-2,S-1
(S) Byker Wall, New Castle, England 4. Charles Correa
5. Kisho Kurokawa
Ans:- (A)
Housing Nakagin Capsule Tower, Tokyo, Japan Tara Apartments, New Delhi, India
projects
Name of Kisho Kurokawa Charles Correa
architect
Description The Nakagin Capsule Tower is a mixed- Tara Apartments is a row housing
use residential and office building in project designed by Charles Correa, a
Ginza, Tokyo, Japan, designed by architect renowned Indian architect and urban
Kisho Kurokawa. It was completed in planner. The project was completed in
1972 and is considered a rare example 1978 and is located in New Delhi, India.
of Japanese Metabolism, an architectural The project consists of 160 units of two
movement that envisioned dynamic and and three-bedroom flats for the middle-
adaptable urban structures. The building income group. The project is an example
consists of two concrete towers that of Correa’s innovative and humanistic
house 140 prefabricated capsules, each approach to urban design.
measuring 4 x 2.5 x 2.1 meters.
258 GATE Architecture and Planning: Comprehensive Question Bank

Project Image

Housing Habitat 67, Montreal, Canada Byker Wall, New Castle, England
projects
Name of Moshe Safdie Ralph Erskine
architect
Description Habitat 67 is a housing complex and Byker Wall is a housing complex and
landmark located on the Marc-Drouin landmark in Newcastle upon Tyne,
Quay on the Saint Lawrence River in England. It was designed by Ralph
Montreal, Canada. It was designed by the Erskine, an Israeli-Canadian architect
Israeli-Canadian architect Moshe Safdie and urban planner. Byker Wall consists
based on his master’s thesis at McGill of 620 maisonettes that form a long,
University in Montreal. It was built as unbroken block that protects the estate
part of the World Exhibition Expo 67. from noise and wind. It was built
between 1969 and 1982 as part of the
redevelopment of the Byker district,
which replaced Victorian slum terraced
housing.
Project Image

Q.33 Match the development schemes by Government of India in Group-I with their objectives in
(A) P-1, Q-5, R-4, S-3 Group I Group II
(B) P-1, Q-5, R-2, S-3 (P) PMAY 1. Housing for All
(C) P-3, Q-5, R-1, S-2 (Q) AMRUT 2. Rural cluster development
(R)NRuM 3. Heritage city development
(D) P-4, Q-2, R-1, S-5
(S) HRIDAY 4. Urban mobility improvement
5. Urban rejuvenation
Hemant Vilas Parulekar 259

Ans:- (B)
Explanation:
PMAY stands for Pradhan Mantri Awas Yojana or Prime Minister’s Housing Scheme. It is a flagship
initiative of the Government of India to provide affordable housing for all by 2022.
AMRUT stands for Atal Mission for Rejuvenation and Urban Transformation. It is a mission
launched by the Government of India to improve the quality of life and infrastructure in urban areas,
focusing on water supply, sewerage, storm water drainage, urban transport, and green spaces.
NRuM stands for National Rurban Mission. It is a mission launched by the Government of India to
stimulate local economic development and enhance basic services in rural areas, by developing clusters
of villages that preserve their rural character while providing urban amenities.
HRIDAY stands for Heritage City Development and Augmentation Yojana. It is a scheme launched
by the Government of India to preserve and revitalize the heritage assets and character of selected.
Q.34 Match the international events in Group-I with their directives in Group-II
(A) P-1, Q-5, R-4, S-3 Group I Group II
(B) P-1, Q-5, R-2, S-3 (P) Earth Summit, Rio de Janerio,1992 1. Kyoto Protocol
(C) P-2, Q-1, R-4, S-5 (Q) UN Framework Convention on Climate 2. Agenda 21
Change, New York, 1992
(D) P-2, Q-1, R-5, S-4
(R) UN Sustainable Development Summit, 3.Heritage Conservation
New York, 2015
(S) Habitat II, Istanbul,1996 4. Agenda 2030
5. Housing for all
Ans:- (C)
Explanation:
• P stands for Earth Summit, which was a UN conference held in Rio de Janeiro, Brazil, in 1992.
It adopted Agenda 21, a global action plan for sustainable development in the 21st century.
Therefore, P matches with 2.
• Q stands for UN Framework Convention on Climate Change, which is an international treaty
to combat “dangerous human interference with the climate system” and to stabilize greenhouse
gas concentrations in the atmosphere. It was signed by 154 states at the Earth Summit in 1992.
Therefore, Q matches with 1.
• R stands for UN Sustainable Development Summit, which was a UN summit held in New
York, USA, in 2015. It adopted Agenda 2030, a new agenda for sustainable development that
includes 17 goals and 169 targets. Therefore, R matches with 4.
• S stands for Habitat II, which was a UN conference on human settlements held in Istanbul,
Turkey, in 1996. It addressed the goals of adequate housing for all and sustainable human
settlements development. Therefore, S matches with 5.
260 GATE Architecture and Planning: Comprehensive Question Bank

Q.35 Match the planning techniques in Group-I with their salient features in Group-II
(A) P-1, Q-5, R-4, S-3 Group I Group II
(B) P-2, Q-1, R-3, S-5 (P) Land Pooling 1. Assigning specific task on a short time
(C) P-2, Q-1, R-3, S-4 horizon
(Q) Action Plan 2. Assembling privately owned land parcels for
(D) P-4, Q-2, R-1, S-5
development
(R) Land sharing 3.Agreement for reallocation of land between
occupiers and owners.
(S) Transfer of 4. Assigning specific task on a long-term
development rights. horizon Development Rights
5. Incentive based voluntary shifting of FAR of
a plot to another plot

Ans:- (B) P-2, Q-1, R-3, S-5


Explanation:
• P stands for Land Pooling, which is a technique for stimulating efficient and equitable land
development in urban areas by assembling privately owned land parcels for development.
Therefore, P matches with 2.
• Q stands for Action Plan, which is a document that outlines the steps and resources needed to
achieve a specific goal or objective within a short time horizon. Therefore, Q matches with 1.
• R stands for Land Sharing, which is an agreement for reallocation of land between occupiers
and owners, usually in informal settlements, to provide secure tenure and improved housing
conditions for the poor. Therefore, R matches with 3.
• S stands for Transfer of Development Rights, which is a zoning technique that conserves land
by redirecting development that would otherwise occur on the land (the sending area) to a
receiving area suitable for denser development. The technique operates so that owners in the
sending area can be compensated for their redirected development rights. Therefore, S matches
with 5.
Q.36 For a symmetrical two dimensional truss as shown in the figure, vertical force kN acting on the
member PQ is _________________.
Ans:- 0
Explanation:
The vertical force in kN acting on the member PQ is zero. This is
because PQ is a zero-force member, which means that it carries no
tension or compression and can be removed without affecting the
stability or determinacy of the truss. A zero-force member can be
identified by applying the method of joints, which is a technique for
calculating the internal forces in a truss by using the equilibrium
conditions at each joint. In this case, applying the method of joints at P and Q reveals that the vertical
components of the forces in the adjacent members must be zero, since there are no external vertical
loads or reactions at those joints. Therefore, the force in PQ must also be zero.
Hemant Vilas Parulekar 261

Q.37 Value of bending moment in kN-m at point C for a beam as shown in figure is
Ans:- 28kN-m
Explanation:
Here, RB + RD = 20 + 40 KN = 60 KN.

Also, Bending Moment about point D = 0.


Thus, RB × 5 – 40 × 2 – 20 × 7.5 = 0
RB = 46 KN. Thus, RD = 14 KN.
Thus, bending moment in kN-m at point C = RD × 2 m = 14 × 2 = 28 kN-m

Q.38 Fee of contractor for a project has the following provisions


Basic fee=15%of actual cost of work incurred
Bonus=20% of savings from estimated cost of work
Penalty=20% of cost overrun
If the estimated cost of the project is Rs 60,000 and the actual cost is Rs 70,000,the total fee of
contractor in Rupees is __________.
Ans:- 8500
Solution:
To find the total fee of the contractor, we need to calculate the basic fee, the bonus, and the penalty. The
basic fee is 15% of the actual cost of work, which is Rs 70,000. So, the basic fee is
0.15×70,000=10,5000.15×70,000=10,500
The bonus is 20% of the savings from the estimated cost of work, which is Rs 60,000. However, since
the actual cost is higher than the estimated cost, there are no savings. So, the bonus is zero.
The penalty is 20% of the cost overrun, which is the difference between the actual cost and the estimated
cost. The cost overrun is Rs 70,000 - Rs 60,000 = Rs 10,000. So, the penalty is
0.2×10,000=2,0000.2×10,000=2,000
Therefore, the total fee of the contractor is the basic fee minus the penalty, which is
10,500−2,000=8,50010,500−2,000=8,500
Q.39 A site has a unidirectional slope of 300 with horizontal along its longer side. The projected
dimensions of the site on the horizontal plane measures 30 m X40 m. Using cut and fill method
the site has to be levelled parallel to the horizontal plane. The minimum amount of earth to be
excavated in cubic meter is _______
262 GATE Architecture and Planning: Comprehensive Question Bank

Ans:- 3463 to 3465


Solution:
Volume of existing site section = (Cross section Area) × Length of shorter side

This volume needs to be converted into cuboid of dimensions


30m × 40m × h ....eq(2)
Equation 1 & 2
h = 11.546m (BF)
CF = BC – BF = (tan 30° × 40) – 11.546 = 11.548
As per 30 – 60 – 90 principle, EF = CF × √3 = 11.548 × √3 = 20 m
Thus, volume of area need to be cut and filled
= 1 × 20 × 11.546 × 30 = 3463 m3

Q.40 The optimistic, most likely and pessimistic time for developing a new product are 12 months, 15
months and 17 months respectively. Calculate the expected time in months.
Ans:- 14 to 15
Solution:
To calculate the expected time for developing a new product, you can use the PERT formula, which is
a weighted average of the three time estimates: optimistic, most likely, and pessimistic. The formula is:
Expected time=Optimistic time+4×Most likely time+ Pessimistic time ÷6
Using the values you provided, the expected time is:
Expected time=12+4×15+17÷6=89÷6 ≈14.83 months
Therefore, the expected time for developing a new product is about 14.83 months.
Q.41 A circular plate inclined at an angle 0 with horizontal plate generates an ellipse astop view with
major axis and minor axis of 5 cm and 2.5 cm respectively. The value of O in degrees is _________.
Ans: 60

Q.42 Calculate the volume of cement in cubic meter required for making 10 cubic meter of M20 grade
Plain Cement Concrete work, assuming the ratio of dry concrete mix to wet concrete mix as 1.52.
Hemant Vilas Parulekar 263

Ans:- 2.7-2.9
Explanation:
M20 grade concrete is used for RCC works (for mild exposure) such as the construction of slabs,
beams, columns, footings, etc. The mix ratio of M20 grade concrete is 1:1.5:3, which means 1 part of
cement, 1.5 parts of sand, and 3 parts of coarse aggregate.
To calculate the volume of cement in cubic meter required for making 10 cubic meter of M20 grade
concrete, we need to follow these steps:
• First, we need to convert the wet concrete volume to the dry concrete volume. Since the ratio
of dry concrete mix to wet concrete mix is 1.52, we multiply 10 cubic meter by 1.52 to get 15.2
cubic meter of dry concrete mix. Next, we need to find the volume of cement in the dry concrete
mix. Since the cement is 1 part out of 5.5 parts (1 + 1.5 + 3) in the mix ratio, we divide 15.2 cubic
meter by 5.5 to get 2.7636 cubic meter of cement.
• Finally, we need to round up the result to two decimal places. Therefore, the volume of cement
in cubic meter required for making 10 cubic meter of M20 grade concrete is 2.76 m<sup>3</
sup>.
Q.43 One acre of agricultural land has been given on a lease till perpetuity at an annual rent of Rs
10,000 to be paid at the end of each year.Net Present Value of the land parcel in Rupees assuming a
discount rate of 5% per annum is ___________.
Ans:- 200000
Solution:
net present value= annual rent (annual net income)× (100 / interest rate)
(Capitalized value in perpetuity) = 10,000 × (100/5) =2,00,000
Q.44 In year 2001, a district with 4,000 manufacturing jobs had a 10% share of total manufacturing
jobs within the state. In year 2011, the state recorded 15% drop in manufacturing jobs whereas,
the share of the district in total manufacturing jobs within the state increased to 15%. Additional
manufacturing jobs created in the district between year 2001 and 2011 is _______.
Ans:- 1100
Solution:
We need to find the total number of manufacturing jobs in the state in both years, and then calculate
the difference between the district’s manufacturing jobs in both years.
Let x be the total number of manufacturing jobs in the state in 2001. Then, the district had 10% of x,
which is 0.1x. Since we know that the district had 4,000 manufacturing jobs in 2001, we can write:
0.1x=4,000
Solving for x, we get:
x=40,000
This means that the state had 40,000 manufacturing jobs in 2001.
Next, let y be the total number of manufacturing jobs in the state in 2011. Since the state recorded a
15% drop in manufacturing jobs, we can write:
y=0.85x
264 GATE Architecture and Planning: Comprehensive Question Bank

Substituting x with 40,000, we get:


y=0.85×40,000
y=34,000
This means that the state had 34,000 manufacturing jobs in 2011.
Now, we can find the number of manufacturing jobs in the district in 2011. Since the district had 15%
of the state’s manufacturing jobs, we can write:
0.15y=z
Where z is the number of manufacturing jobs in the district in 2011. Substituting y with 34,000, we get:
0.15×34,000=z
z=5,100
This means that the district had 5,100 manufacturing jobs in 2011.
Finally, we can find the additional manufacturing jobs created in the district between 2001 and 2011
by subtracting the district’s manufacturing jobs in 2001 from the district’s manufacturing jobs in 2011.
We get:
z−0.1x=5,100−4,000
z−0.1x=1,100
Therefore, the additional manufacturing jobs created in the district between 2001 and 2011 are 1,100.
Q.45 Match the parameters in Group-I with their units in Group-II
(A) P-5,Q-4,R-1,S-2 Group I Group II
(B) P-5,Q-4,R-1,S-3 (P) Traffic flow 1. Meter
(C) P-5,Q-2,R-4,S-3 (Q) Traffic Density 2. Cycle/second
(R) Right of Way 3. Seconds
(D) P-4,Q-5,R-1,S-3
(S) Traffic Signal cycle length 4. Vehicle/km
5. PCU/hr
Ans:- (B)
Explanation:
• Traffic flow is the number of vehicles that pass a point on a roadway during a specified time
period, usually expressed as vehicles per hour (vph) or passenger car units per hour (PCU/hr).
PCU is a unit that converts different types of vehicles into equivalent car units based on their
impact on traffic flow. Therefore, the unit for traffic flow is 5. PCU/hr.
• Traffic density is the number of vehicles that occupy a unit length of roadway at a point in time,
usually expressed as vehicles per kilometre (veh/km) or vehicles per mile (vpm). Therefore, the
unit for traffic density is 4. Vehicle/km. Right of way is the legal right of a vehicle or pedestrian
to proceed in a certain direction or occupy a certain space on a roadway, usually determined by
traffic signs, signals, or rules. The unit for right of way is not a physical quantity, but a legal one.
Therefore, the unit for right of way is 1. Meter.
• Traffic signal cycle length is the total time required for a traffic signal to complete one full
sequence of all signal phases, usually expressed in seconds. Therefore, the unit for traffic signal
cycle length is 3. Seconds.
Hemant Vilas Parulekar 265

Q.46 Match the planning tasks in Group-I with their tools of analysis in Group-II
(A)P-3,Q-1,R-4,S-2 Group I Group II
(B)P-3,Q-5,R-4,S-1 (P)Population Projection 1.Input-Output Analysis
(C)P-5,Q-1,R-3,S-4 (Q) Regional resource allocation 2. Hardy Cross Method
(R) Trip distribution 3. Cohort Analysis
(D)P-1,Q-3,R-5,S-2
(S)Design of water distribution network 4. Gravity Model
5. Moving Observer method

Ans:- (A)
Explanation:
• Population projection is the process of estimating the future size and structure of the population
based on historical trends and assumptions about fertility, mortality, and migration. One of
the methods used for population projection is cohort analysis, which tracks the changes in the
population by age groups or cohorts over time. Therefore, the method for population projection
is 3. Cohort analysis. Regional resource allocation is the plan for using available resources, such as
human, financial, or natural resources, for achieving goals in a region. One of the methods used
for regional resource allocation is input-output analysis, which measures the interdependence of
economic sectors and the impact of changes in one sector on the others. Therefore, the method
for regional resource allocation is 1. Input-output analysis.
• Trip distribution is the second component in the four-step transportation forecasting model,
which matches the origins and destinations of trips generated in an area. One of the methods
used for trip distribution is the gravity model, which estimates the number of trips between
two zones based on their attractiveness and the travel impedance between them. Therefore, the
method for trip distribution is 4. Gravity model.
• Design of water distribution network is the process of planning and sizing the pipes, valves,
pumps, and other components of a water supply system. One of the methods used for design of
water distribution network is the Hardy Cross method, which iteratively balances the flow and
head loss in a looped network. Therefore, the method for design of water distribution network
is 2. Hardy Cross method.
Q.47 Match the land use classes in Group-I with their use zones in Group-II
(A) P-4,Q-1,R-3,S-5 Group I Group II
(B) P-5,Q-3,R-1,S-2 (P) Transportation 1. Sports complex
(C) P-4,Q-5,R-1,S-2 (Q) Commercial 2. Heritage and Conservation areas
(R) Public and Semi-Public 3. Burial ground
(D) P-4,Q-5,R-3,S-1
(S) Recreational 4. BRT Corridor
5. Service sector
Ans:- (D)
Explanation:
• Transportation is the movement of people and goods from one place to another, usually by
road, rail, air, or water. One of the land uses related to transportation is the BRT corridor, which
stands for Bus Rapid Transit, a system of high-capacity buses that run on dedicated lanes and
stations. Therefore, the land use for transportation is 4. BRT Corridor.
266 GATE Architecture and Planning: Comprehensive Question Bank

• Commercial is the activity of buying and selling goods and services, usually for profit. One of
the land uses related to commercial is the service sector, which includes businesses that provide
intangible products or services, such as banking, education, health care, tourism, etc. Therefore,
the land use for commercial is 5. Service sector. Public and Semi-Public is the category of land
use that includes facilities and services that are owned or operated by the government or non-
profit organizations, such as schools, hospitals, libraries, parks, etc. One of the land uses related
to public and semi-public is the burial ground, which is a place where the dead are buried,
usually managed by religious or civic authorities. Therefore, the land use for public and semi-
public is 3. Burial ground.
• Recreational is the category of land use that includes facilities and spaces that are used for
leisure, entertainment, or sports activities, such as playgrounds, cinemas, stadiums, etc. One
of the land uses related to recreational is the sports complex, which is a large facility that hosts
multiple sports venues and events, such as swimming pools, tennis courts, basketball arenas,
etc. Therefore, the land use for recreational is 1. Sports complex.
Q.48 Match the structural systems in Group-I with the buildings in Group-II
P-3, Q-4, R-1, S-2 Group I Group II
(B) P-5, Q-4, R-3, S-1 (P) Folded plates 1. Kurilpa Bridge, Brisbane
(C) P-3, Q-2, R-1, S-5 (Q) Shell 2. Eden Project, Cornwall
(R) Tensegrity 3. Riverside Museum, Glasgow
(D) P-1, Q-3, R-4, S-2
(S) Pneumatic 4. MIT Auditorium, Boston
5. 30, St.Mary Axe, London
Ans:- (A)
Structural system Folded plates Shell
Used in building Riverside Museum, Glasgow MIT Auditorium, Boston
architect
Description Folded plates are a type of structural Shell is a type of structural system that
system that consists of flat plates or consists of curved, thin-walled surfaces
slabs that are bent along straight lines that can span large distances without
and joined along their edges. They are internal supports. Shell structures
often used to cover large spans without are often used to create elegant and
the need for internal supports, and can organic forms in architecture.
create interesting and dynamic forms1. The MIT Auditorium, also known
The Riverside Museum is a museum of as Kresge Auditorium, is a building
transport and technology that opened on the campus of the Massachusetts
in 2011 in Glasgow, Scotland. It was Institute of Technology, located at 48
designed by the famous architect Massachusetts Avenue, Cambridge,
Zaha Hadid, who used folded plates Massachusetts. It was designed by
to create the curved and twisted shape the Finnish-American architect Eero
of the roof. The museum houses over Saarinen, and completed in 1955. The
3,000 objects from Glasgow’s rich auditorium has a roof that is a thin-
history of shipbuilding, engineering, shell structure of reinforced concrete,
and transportation. shaped like a slice of a sphere.
Hemant Vilas Parulekar 267

Building

Structural system Tensegrity Pneumatic


Used in building Kurilpa Bridge, Brisbane Eden Project, Cornwall
architect
Description Tensegrity is a structural principle Pneumatic is a term that refers to the
that involves a system of isolated use of air, wind, or other gas to perform
components under compression inside certain operations or functions.
a network of continuous tension. The Pneumatic systems are often powered
term was coined by Buckminster by compressed air or inert gas, and
Fuller, who was inspired by the work of use valves, cylinders, actuators, and
Kenneth Snelson, an artist who created other devices to control the flow and
sculptures using tensegrity. Tensegrity pressure of the gas. Pneumatic systems
structures are stable, efficient, and are widely used in various industries
elegant, and can be found in nature, art, and applications, such as dentistry,
and architecture. The Kurilpa Bridge construction, mining, transportation,
is a pedestrian and bicycle bridge that and automation. The Eden Project is
spans the Brisbane River in Brisbane, a visitor attraction and educational
Queensland, Australia. It was designed charity in Cornwall, England, that
by Cox Rayner Architects and Arup showcases the interconnections
Engineers, and opened in 2009. The between all living things and the
bridge is the world’s largest hybrid natural world. The Eden Project
tensegrity bridge, meaning that it consists of two massive biomes that
combines tensegrity with conventional house thousands of plants from
cable-stay elements. different climates and regions, such as
the rainforest and the Mediterranean.
Building

Q.49 As per National Building Code of India, 2005, the maximum number of occupant per unit exit
width of a doorway is 60, where unit width is 500 mm. The maximum permissible occupants in a
theatre having four number of 2.2 m wide doors will be _____________.
268 GATE Architecture and Planning: Comprehensive Question Bank

Ans: 960
Solution:
Unit exit width is 500mm (0.5m). in the problem door width is 2.2m.
Thus, maximum number of occupants per door of 2 m width = (2/0.5) =240
Thus, maximum number of occupants in a theatre with 4 doors= 240×4=960
Q.50 Match the instruments in Group-I with the corresponding tests in Group-II
(A) P-5, Q-3, R-2, S-1 Group I Group II
(B) P-5, Q-4, R-2, S-1 (P) Pycnometer 1. Initial and final setting time
(C) P-3, Q-2, R-1, S-5 (Q) Brinell’s Apparatus 2. Abrasion test
(R) Los Angeles Apparatus 3. Surface hardness test
(D) P-2, Q-3, R-4, S-1
(S) Vicat’s Apparatus 4. Slump test
5. Apparent Specific gravity
Ans:- (A)
Instrument psychometer Brinell’s Apparatus
Used in building Apparent Specific gravity Surface hardness test
architect
Instrument

Instrument Los Angeles Apparatus Vicat’s Apparatus


Used in building Abrasion test Initial and final setting time
architect
Instrument

Q.51 Water flows through a constricted pipe whose diameter at the constricted end is half of the non-
constricted end. Velocity of water at the non-constricted end is 2 m/s. Velocity of water in m/s at
the constricted end using the principle of continuity of flow is _____________.
Hemant Vilas Parulekar 269

Ans:- 8
Solution:
To find the velocity of water at the constricted end of the pipe, we need to use the principle of continuity
of flow, which states that the mass flow rate of a fluid is constant in a steady flow. This means that the
product of the cross-sectional area and the velocity of the fluid is the same at any point in the pipe1
Let A1 and v1 be the area and velocity of the non-constricted end, and A2 and v2 be the area and
velocity of the constricted end. Then, using the principle of continuity of flow, we can write:
A1v1=A2v2
Since the diameter of the constricted end is half of the non-constricted end, we can write:
A2=frac14A1
Substituting this into the equation,
we get: A1v1=frac14A1v2
Simplifying and solving for v2, we get:
v2=4v1
Plugging in the given value of v1 as 2 m/s, we get:
v2=4×2
v2=8
Q.52 A drainage basin of 180 hectares comprise 40% wooded area, 45% grassed area and 15% paved
area. Run off coefficients for wooded, grassland and paved areas are 0.01, 0.2 and 0.95 respectively.
The composite runoff coefficient for the drainage basin is________.
Ans:- 0.2 to 0.25
Solution:
To find the composite runoff coefficient for the drainage basin, we need to use the formula:

where C is the composite runoff coefficient, Ci is the runoff coefficient for each land use, Ai is the area
of each land use, and A is the total area of the drainage basin1
Plugging in the values, we get:
C=frac(0.01×0.4×180)+(0.2×0.45×180)+(0.95×0.15×180)180
C=frac0.72+16.2+25.65180
C=frac42.57180
C=0.2365
Q.53 A fluorescent light source consumes 40 W electric power and has a luminous efficacy of 40lm/W.
Illumination in lux at a distance of 3 m from this light source is _________.
270 GATE Architecture and Planning: Comprehensive Question Bank

Ans:- 175 - 180


Solution:
To find out Illumination produced from light source.
E =I/d2
Where, E = Illumination on surface (lux),
I = Illumination intensity from source (cd)
d = distance from light source
Here, I = Luminous efficacy of lamp × Power consumption of lamp
= 40 lm / W × 40 W = 1600 lm
E =1600/3×3= 177.78 lux
Q.54 A room measures 3 m(width)×4m(length)×3 m(height).The outdoor temperature is 36℃. The
volumetric specific heat of air is 1300 J/cu.m.℃. The ventilation heat flow rate in Watts required
attaining an internal room temperature of 26℃ with 3 air changes per hour is______.
Ans:- 390
Solution:
Total volume of room = 3 × 4 × 3 = 36 m3
The amount of air changes in 3 turns per hour = 36 × 3 = 108 m3
Ventilation heat flow rate in Watts = Total energy required
Total energy required (ms∆ t) = Volume of air × Specific Heat × Difference Between Temperature
= 108 × 1300 × 10 = 1404000 Joule
= 1404000 Joule(As Watt (W) = Joule/Sec.) = 390 W
Q.55 Match the equipment in Group-I with their applications in Group-II
(A) P-4,Q-5,R-2,S-1 Group I Group II
(B)P-1,Q-4,R-5,S-3 (P) PIR 1. Air conditioning
(C) P-4,Q-1,R-2,S-3 (Q) FCU 2. Lighting
(R) OLED 3. Power generation
(D)P-4,Q-2,R-5,S-1
(S) BIPV 4. Motion detection
5. Daylight sensing
Ans:- (C)
Equipment PIR FCU
Application Passive infrared sensor: an electronic Fan coil unit: a device consisting of a
device that measures infrared light heat exchanger and a fan that is used in
radiating from objects in its field of HVAC systems of buildings. It can be
view. They are often used in motion horizontal or vertical, and it circulates
detectors, security alarms, and the inside air without ducts.
automatic lighting systems.
Hemant Vilas Parulekar 271

Equipment

Equipment OLED BIPV


Application OLED stands for organic light-emitting BIPV stands for building-integrated
diode. It is a type of display technology photovoltaics. It is a type of solar
that uses organic compounds to create power system that uses photovoltaic
light and colors. OLED displays have materials to replace conventional
many advantages over other types of building materials in parts of the
displays, such as LCD and plasma. building envelope, such as the roof,
skylights, or façades. BIPV systems
can provide savings in materials and
electricity costs, reduce use of fossil
fuels and emission of greenhouse
gases, and add architectural interest to
the building.
Equipment
GATE QUESTION PAPER 2016
General Aptitude (GA)

Q. 1 – Q. 25 carry one mark each.


Q.1 An apple costs Rs. 10. An onion costs Rs. 8.
Select the most suitable sentence with respect to grammar and usage.
(A) The price of an apple is greater than an onion..
(B) The price of an apple is more than onion.
(C) The price of an apple is greater than that of an onion.
(D) Apples are more costlier than onions.
Ans:- (C)
Q.2 The Buddha said, “Holding on to anger is like grasping a hot coal with the intent of throwing it at
someone else; you are the one who gets burnt.”
Select the word below which is closest in meaning to the word underlined above.
(A) burning (B) igniting (C) clutching (D) flinging
Ans:- (C)
Q.3 3 M has a son Q and a daughter R. He has no other children. E is the mother of P and daughter-in-law
of M. How is P related to M?
(A) P is the son-in-law of M.
(B) P is the grandchild of M.
(C) P is the daughter-in law of M.
(D) P is the grandfather of M.
Ans:- (B)
Q.4 The number that least fits this set: (324, 441, 97 and 64) is ________.
(A) 324 (B) 441 (C) 97 (D) 64
Ans:- (C)
Q.5 It takes 10 s and 15 s, respectively, for two trains travelling at different constant speeds to completely
pass a telegraph post. The length of the first train is 120 m and that of the second train is 150 m. The
magnitude of the difference in the speeds of the two trains (in m/s) is ____________.
(A) 2.0 (B) 10.0 (C) 12.0 (D) 22.0
Ans:- (A)
Solution: Speed of the first train = length/time = 120/10 = 12m/s
Speed of the second train = length/time = 150/15 = 10m/s
2.0 m/s is the difference in the train speed.
Hemant Vilas Parulekar 273

Q. 6 – Q. 10 carry two marks each.


Q.6 The velocity V of a vehicle along a straight line is
measured in m/s and plotted as shown with respect to
time in seconds. At the end of the 7 seconds, how much
will the odometer reading increase by (in m)?
(A) 0 (B) 3
(C) 4 (D) 5

Ans:- (B)
The odometer will read 5. (The area shaded under the graph). Odometer is an instrument for measuring
the distance travelled by a wheeled vehicle. So, you have to just count the no. of squares made by triangles
of the graph.
Q.7 The overwhelming number of people infected with rabies in India has been flagged by the World
Health Organization as a source of concern. It is estimated that inoculating 70% of pets and stray
dogs against rabies can lead to a significant reduction in the number of people infected with rabies.
Which of the following can be logically inferred from the above sentences?
(A) The number of people in India infected with rabies is high.
(B) The number of people in other parts of the world who are infected with rabies is low.
(C) Rabies can be eradicated in India by vaccinating 70% of stray dogs.
(D) Stray dogs are the main source of rabies worldwide.
Ans:- (A)
Q.8 A flat is shared by four first year undergraduate students. They agreed to allow the oldest of them
to enjoy some extra space in the flat. Manu is two months older than Sravan, who is three months
younger than Trideep. Pavan is one month older than Sravan. Who should occupy the extra space
in the flat.
(A) Manu (B) Sarvan (C) Trideep (D) Pavan
Ans:- (C)
Q.9 Find the area bounded by the lines 3x+2y=14, 2x-3y=5 in the first quadrant.
(A) 14.95 (B) 15.25 (C) 15.70 (D) 20.35
Ans:- (B)
Solution: Let orange line be, 3x+2y=14 and blue line be, 2x-3y=5
Solving both equation, we get, (x=4 , y=1) the intersection points of the lines.
To get the required area in the first quadrant, we have to deduct the area of the triangle formed by the
orange line, blue line & the x-axis from the area bounded by the orange line in the first quadrant.
Area of the triangle = 1/2 x base x height = 1/2 x B x A = 1/2 x 13/6 x 1 = 13/12 ………….(P)
Area bounded by orange line and X & Y axis = (1/2) x (x-intercept) x (y-intercept) = 1/2×14/2×7 = 98/6
…………….(Q)
So, required area = (Q) – (P) = 98/6 – 13/12 = 183/12 = 15.25
274 GATE Architecture and Planning: Comprehensive Question Bank

Q.10 A straight line is fit to a data set (ln x, y). This line intercepts the abscissa at ln x = 0.1 and has a slope
of −0.02. What is the value of y at x = 5 from the fit?
(A) −0.030 (B) −0.014 (C) 0.014 (D) 0.030
Ans:- (A)
Solution:The equation of a line is
y = mx + c where m is the slope & c is the y-intercept
Now, In this question x is replaced with lnx
So, the equation of line becomes,
y = mlnx + c
or, y = -0.02lnx + c
We have given with abscissa which is essentially x-intercept. So, now we have to find ‘c’ the y-intercept.
for, y=0, lnx = 0.1 (given in the question)
Putting the value,
0 = -0.02×0.1 + c
or, c = 0.002
So, the equation of line becomes,
y = -0.02lnx + 0.002
putting x = 5 (asked in the question)
y = -0.002ln5 + 0.002 = -0.002×1.6 + 0.002 = -0.03
(ln5 = 1.6)
Hemant Vilas Parulekar 275

PART 2- ARCHITECTURE & PLANNING (QUESTION 1-55)


Q.1 – Q.25 carry one mark each.
Q.1 ‘Dharahara’ refers to
(A) Concept of Vastu Shastra
(B) Elevation feature of a Hindu Temple
(C) Heritage tower at Kathmandu
(D) Construction method of Buddhist Architecture
Answer:- (C)
Explanation: The Dharahara Tower, also known as Bhimsen Tower,
was a historic landmark in Kathmandu, Nepal. Here are some key
aspects of the Dharahara Tower:
1. Historical Significance: The original Dharahara Tower was built
in 1832 under the commission of Prime Minister Bhimsen Thapa of
Nepal. It was part of the architecture encouraged by the Rana rulers of
Nepal and was a significant example of Nepal’s historical architecture.
2. Structure and Design: The tower was a nine-story, narrow, white
tower that rose 50.2 meters (about 165 feet) tall. It was constructed
with a 5.2-meter (17 feet) diameter base and had a spiral staircase
containing 213 steps, leading to a circular balcony on the eighth floor and a viewing platform on the
ninth floor, offering panoramic views of the Kathmandu valley.
3. Destruction and Reconstruction: The tower suffered damage from earthquakes throughout its
history. It was partially destroyed in the 1934 Bihar-Nepal earthquake and was later rebuilt. Unfortunately,
it collapsed almost completely during the devastating earthquake that struck Nepal on April 25, 2015.
Since then, there have been efforts to rebuild and restore the tower, reflecting its importance to Nepalese
history and heritage.
4. Cultural Importance: The Dharahara Tower held great cultural significance for the people of Nepal.
It was not just an architectural marvel but also a symbol of Kathmandu’s history and resilience. The
tower was a popular tourist attraction and a point of pride for the local population.
Modern Reconstruction: As of recent years, efforts have been underway to reconstruct the tower. The
new design aims to retain the historical essence of the original Dharahara while incorporating modern
structural engineering techniques to ensure its stability and safety, especially in the context of potential
future earthquakes.
Q.2 ‘Mushroom Column’ is a typical feature of
(A) Pantheon, Rome (B) Parthenon, Athens
(C) Chrysler Building, New York (D) Johnson Wax Building, Racine
Ans:- (D)
Explanation: The Johnson Wax Headquarters, located in Racine, Wisconsin, is a notable architectural
masterpiece designed by Frank Lloyd Wright in 1939. Renowned for its innovative design, the building
features a striking exterior, but it’s most famous for its unique interior with dendriform columns, creating
a forest-like atmosphere. The building serves as the headquarters for the SC Johnson & Son company
and is celebrated for its integration of form and function, blending industrial and organic architectural
elements, a hallmark of Wright’s style.
276 GATE Architecture and Planning: Comprehensive Question Bank

Q.3 ‘AMRUT’, the new scheme launched by Government of India, stands for
(A) Atal Mission for Rejuvenation and Urban Transformation
(B) Atal Mission for Renewal of Urban Transportation
(C) Atal Mission for Redevelopment of Urban Transportation
(D) Atal Mission for Renewal and Urban Transformation
Ans:- (A)
Q.4 The concept of ‘Dry Garden’ is associated with
(A) Japanese Garden (B) Chinese Garden
(C) Mughal Garden (D) Egyptian Garden
Answer:- (A)
Explanation: Japanese gardens, known for their serenity and beauty, are a traditional form of garden
in Japan, deeply rooted in history, culture, and philosophy. They are designed not just for aesthetic
pleasure but also to facilitate meditation and contemplation. Understanding Japanese gardens involves
appreciating their main elements, design principles, and the cultural context that shapes them.
Elements of Japanese Gardens
Water: Water is a central feature, symbolizing continuity and renewal. It can be present in the form of
ponds, streams, or even symbolically represented by raked gravel.
Rocks and Stones: These are crucial elements, often symbolizing mountains, islands, or animals. The
arrangement of rocks is an art form in itself, carrying significant symbolic meaning.
Plants: The selection of plants in Japanese gardens is carefully considered for their aesthetic through all
seasons. Common choices include bonsai, moss, pine, maple, and bamboo.
Bridges and Pathways: Bridges over water elements and pathways, often made of stone or wood, guide
visitors through the garden, offering changing views and experiences.
Lanterns and Sculptures: Stone lanterns and sculptures are often used, not merely as decorations but as
integral parts of the garden that blend with the natural elements.
Types of Japanese Gardens
Karesansui (Dry Landscape) Gardens: These are rock gardens where gravel and stones are used to
represent water and landscape features. The famous Ryoan-ji Temple garden in Kyoto is a classic example.
Tsukiyama (Hill) Gardens: These gardens feature artificial hills, ponds, and streams, designed to imitate
natural scenery.
Chaniwa Gardens (Tea Gardens): These are built for the tea ceremony and feature a tea house. The
path leading to the tea house is an integral part of the garden.
Kaiyū-shiki-teien (Stroll Gardens): These gardens are designed for walking, with paths leading visitors
through a series of carefully composed landscapes.
Design Principles
Miniaturization: Replicating nature in a smaller scale, such as creating miniature mountains, rivers,
and islands.
Hemant Vilas Parulekar 277

Concealment and Discovery: Paths in Japanese gardens often have bends and turns, designed to hide
and then reveal features, creating a sense of surprise and intrigue.
Borrowed Scenery (Shakkei): Incorporating background landscapes like mountains or forests into the
garden’s composition.
Asymmetry: Balancing elements without mirroring or creating symmetrical designs, reflecting the
irregularity found in nature.
Seasonal Changes: Emphasizing plants and elements that change through the seasons to reflect the
transient beauty of nature.
Cultural Significance
Japanese gardens are heavily influenced by Shinto, Buddhist, and Taoist philosophies. The idea
of harmony between man and nature is a guiding principle. They are spaces for contemplation and
reflection, often embodying the Japanese aesthetic of wabi-sabi, which finds beauty in imperfection and
transience.
Modern Influence
Today, Japanese gardens can be found all over the world, influencing landscape design and providing
tranquil spaces in both private and public areas. They are appreciated not only for their beauty but also
for their ability to evoke thought, reflection, and a deep appreciation of the natural world.
Q.5 The concept of Transit Oriented Development (TOD) refers to
(A) Transit oriented planning in institutional campuses
(B) High density development along a mass transit corridor
(C) Orienting residential blocks towards the transit in a gated community
(D) Spatial design for persons with disabilities along a mass transit corridor
Ans:- (B)
Explanation: Transit-Oriented Development (TOD) is a type of urban development that maximizes
access to public transportation, integrating residential, commercial, and leisure spaces within a walkable
and bikeable distance from transit stations. The goal of TOD is to create dense, mixed-use communities
that reduce reliance on cars, encourage public transit use, and promote sustainable urban growth. This
approach aims to improve quality of life, reduce traffic congestion, and lower environmental impact in
urban areas.
Q.6 The components measuring Human Development Index (HDI) of a nation are
(A) Life expectancy, Education and Per Capita Income
(B) Life expectancy, Economy and Housing
(C) Health, Development and Per Capita Income
(D) Housing, Health and Hygiene
Ans:- (A)
Explanation: The Human Development Index (HDI) is a statistical tool used to measure a country’s
overall achievement in its social and economic dimensions. It assesses three basic dimensions of human
development: life expectancy (health), education level, and standard of living (measured by Gross
National Income per capita). The HDI is used to classify countries into four tiers of human development
(very high, high, medium, and low) and to provide a broader perspective of well-being and development
than purely economic measures like GDP. It’s published annually by the United Nations Development
Programme (UNDP).
278 GATE Architecture and Planning: Comprehensive Question Bank

Q.7 As per the Ancient Monuments and Archaeological Sites and Remains Act, 2010, the extent of
‘prohibited area’ beyond the ‘demarcated protected area’ in meters is
(A) 50 (B) 100 (C) 200 (D) 500
Ans:- (B)
Explanation: As per the Ancient Monuments and Archaeological Sites and Remains (Amendment and
Validation) Act, 2010, in India, the extent of the ‘prohibited area’ beyond the limits of the ‘demarcated
protected area’ or ‘protected area’ is defined as 100 meters. Additionally, there is a ‘regulated area’ further
extending to 200 meters beyond the limits of the prohibited area.
Q.8 In CMYK colour model, ‘K’ represents the colour
(A) White (B) Black (C) Blue (D) Green
Ans:- (B)
Explanation: In the CMYK color model, which is a subtractive color model used in color printing, ‘K’
stands for ‘Key’ and represents the color Black. The CMYK model is based on the CMY color model,
where C stands for Cyan, M for Magenta, and Y for Yellow. The ‘K’ (Key) component is typically black
and is used primarily for defining detail and contrast.
Therefore, ‘K’ in CMYK represents: black
Q.9 ‘Segregation at source’ is a concept associated with
(A) Solid waste management (B) Disaster management
(C) Traffic management (D) Water management
Ans:- (A)
Q.10 In a flat slab, the load is transferred through:
(A) Beam Action (B) Membrane Action (C) Plate Action (D) Vector
Ans:- (C)
Q.11 Map is an
(A) Isographic projection (B)Orthographic projection
(C) Axonometric Projection (D) Oblique projection
Ans:- (B)
Q.12 ‘Indore Slum Networking Programme’ has been planned by
(A) B. V. Doshi (B) Dulal Mukherjee
(C) Anil Laul (D) Himanshu Parikh
Ans:- (D)
Explanation: This program, known for its cost-effective and community-oriented approach, aimed at
providing integrated infrastructure services like water supply, sewage, roads, and stormwater drainage
in slum areas of Indore, India. It emphasized the importance of local participation and sustainable urban
development.
Q.13 ‘Volume Zero’ is a film based on the architectural works of
(A) Peter Eisenman (B) Charles Correa
(C) Daniel Libeskind (D) Roger Angers
Hemant Vilas Parulekar 279

Ans:- (B)
Explanation: “Volume Zero” is a documentary film that explores the life and works of Charles Correa, a
renowned Indian architect and urban planner. Correa’s work is celebrated for its sensitivity to the needs
of the urban poor and for its use of traditional methods and materials.
Q.14 The unit of Thermal Conductivity of a material is
(A) W/(m K) (B) W/(m2 K)
(C) m K/W (D) m2 K/W
Ans:- (A)
Explanation:
1. K-Value (Thermal Conductivity):
– The K-value, or thermal conductivity, measures a material’s ability to conduct heat. It is typically
expressed in units of watts per meter per degree Kelvin (W/m·K).
– A higher K-value indicates that the material is a better conductor of heat, while a lower K-value
means it is a better insulator. Metals typically have high K-values, while insulating materials have low
K-values.
2. C-Value (Thermal Conductance):
– The C-value, or thermal conductance, measures the rate at which heat flows through a material per
unit area per unit temperature difference across the material. It is expressed in units of watts per
square meter per degree Kelvin (W/m²·K).
– Unlike K-value, which is an intrinsic property of a material, C-value depends on the thickness of the
material. The thinner the material, the higher the C-value.
3. R-Value (Thermal Resistance):
– The R-value is the measure of a material’s resistance to heat flow. It is the inverse of the C-value and is
expressed in square meter degrees Kelvin per watt (m²·K/W).
– Higher R-values indicate better insulation properties, as the material resists the flow of heat. The
R-value is crucial in building insulation as it helps in determining how well a material can insulate.
4. Thermal Transmittance (U-Value):
– The U-value, or thermal transmittance, measures the rate at which heat is transferred through a
structure. It is expressed in watts per square meter per degree Kelvin (W/m²·K).
– The U-value takes into account the combined effect of conduction (K-value), convection, and radiation
through a material or an assembly of materials (like a wall or window). Lower U-values indicate better
insulation as they imply less heat loss.
Q.15 Semi circular niche in a Mosque in the direction of Mecca is known as
(A) Sahn (B) Mimbar (C) Qibla (D) Mihrab
280 GATE Architecture and Planning: Comprehensive Question Bank

Answer:- (D)

Q.16 As per CPWD Guidelines and Space Standards for Barrier Free Built Environment, the minimum
turning radius for a wheelchair in mm is
(A) 900 (B) 1200 (C) 1500 (D) 1800
Ans:- (A)
Explanation: The turning radius for a wheelchair should be in range of 750-900 for a ramp.
(Source: Handbook on Barrier free design and Accessibility)
Q.17 ‘Summit Curve’ is a term associated with the design of
(A) Roads and flyovers (B) Escalators and elevators
(C) Tensile fabric structures (D) Geodesic domes
Answer:- (A)
Explanation: The term “Summit Curve” is associated with the design of (A) Roads and flyovers. It refers
to a type of vertical curve in road design, particularly used at the top of a hill or a rise. Summit curves
are designed to provide comfortable and safe transition between different slopes, ensuring that drivers
have adequate sight distance to see any obstacles on the road ahead. This is crucial for maintaining road
safety and smooth vehicular movement.
Q.18 As per Census of India 2011, Nagar Panchayat refers to
(A) Rural area surrounded by a growing city (B) Urban area within a panchayat
(C) Rural area adjacent to a nagar (D) Urban area with a statutory local government
Ans:- (D)
Hemant Vilas Parulekar 281

Q.19 Statutory setback of a building depends on


(A) Building height (B) Property boundary
(C) Width of access road (D) Ground coverage
Ans:- (A)
Q.20 Super plasticizer is added in a concrete mix to
(A) Reduce the water-cement ratio for a given workability
(B) Increase the water-cement ratio for a given workability
(C) Reduce the flow
(D) Reduce the content of coarse aggregate
Ans:- (A)
Explanation: Super plasticizers are high-range water reducers. They allow the reduction of water
content in a concrete mix without compromising the workability of the concrete. This leads to higher
strength and durability of the concrete, as a lower water-cement ratio generally results in a stronger and
more durable concrete. Super plasticizers are especially useful in making high-strength concrete and in
applications where a fluid, easily flowing concrete is required.
Q.21 Shodhan House, Ahmedabad was designed by
(A) Anant Raje (B) Le Corbusier (C) Louis I. Kahn (D) B. V. Doshi
Ans:- (B)
Q.22 Low-emissivity coating on a glazing unit
(A) Increases the SHGC (B) Increases the VLT
(C) Reduces the SHGC (D) Increases the VLT and SHGC
Ans:- (C)
Explanation: Low-emissivity (Low-E) coating on a glazing unit is designed to minimize the amount of
infrared and ultraviolet light that passes through glass without compromising the amount of visible light
that is transmitted. These coatings are thin and transparent, made of metallic or metallic oxide layers,
and they work by reflecting heat back to its source.
In the context of the options given:
SHGC (Solar Heat Gain Coefficient): This measures how much of the sun’s heat is transmitted through
the window. A low SHGC value means less solar heat is transmitted.
VLT (Visible Light Transmittance): This is the measure of how much visible light passes through a
window.
Low-E coatings:
(A) Reduce the SHGC: They reflect infrared radiation, thus reducing the amount of solar heat that
enters the building.
(B) Increases the VLT: Ideally, they are designed to allow as much visible light as possible while blocking
heat radiation. However, the extent to which VLT is increased can vary depending on the specific type
of Low-E coating.
282 GATE Architecture and Planning: Comprehensive Question Bank

Q.23 Spatial connectedness in GIS refers to


(A) Tomography
(B) Topography
(C) Topiary
(D) Topology
Ans:- (D)
Explanation: Topology in GIS is a concept that describes the spatial relationships between different
geographic features, such as how they are connected, adjacent, or contain each other. It’s crucial for
ensuring data integrity, especially when dealing with features that share boundaries or overlap. Topology
rules help manage and enforce how lines and polygons share geometry. For instance, it ensures that
adjoining polygons do not overlap and that there are no gaps between them. This is essential for accurate
GIS analyses and operations.
Q.24 In a residential neighbourhood, the net area of residential plots is 50 percent of the total area. If the
population is 8000, the ratio of net density to gross density of the neighbourhood is ________
Ans:- 2
Solution: Given:
The net area of residential plots is 50% of the total area.
The population is 8000.
Let’s denote:
Total area of the neighborhood = A.
Area of residential plots = 0.5×A (since it’s 50% of the total area).
Then:
Net density = Population / Area of residential plots = 8000/(0.5×A).
Gross density = Population / Total area = 8000/A.
The ratio of net density to gross density is:
Net Density / Gross Density = [8000/(0.5×A)] / (8000/A)
= 1/0.5 = 2
Q.25 A hemispherical earth mound of 3 meter diameter is proposed to be constructed in a children’s
park. If the proportion of soil and sand for the construction is 3:2, the estimated volume of soil in
cubic meters is ________
Ans:- 4.15 m3-4.30 m3
Solution:
Volume of hemisphere = [4/3(πr3)]/2 = 2/3(πr3)
Volume of mound = 2/3(22/7×3/2×3/2×3/2) = 99/14
Volume of mud = (3/5)×(99/14) = 4.24 m3
Hemant Vilas Parulekar 283

Q.26-Q.55 carry two marks each.


Q.26 Match the ancient cities in Group-I with their characteristic features in Group-II
P-3,Q-4,R-2,S-5 Group I Group II
P-4,Q-5,R-1,S-2 P. Mohen-jo-Daro 1. Agora
P-3,Q-5,R-2,S-1 Q. Babylon 2. Enclosed Court
R. Kahun 3. Grid Iron Street Pattern
(D) P-4,Q-5,R-2,S-3
S. Athens 4. Forum
5. Hanging Garden

Ans:- (C)
Q.27 Match the water purification in Group-I with the corresponding items in Group-II
P-3,Q-4,R-1,S-5 Group I Group II
P-1,Q-4,R-3,S-2 P. Coagulation 1. Alum
P-2,Q-3,R-5,S-4 Q. Filtration 2. Sand
R. Disinfection 3. Chlorine
(D) P-3,Q-4,R-5,S-2
S. Softening 4. Sodium Zeolite
5. Ammonium Hydroxide

Ans:- (D)
Q.28 Match the software tools in Group-I with their field of application in Group-II
P-3,Q-4,R-5,S-1 Group I Group II
P-5,Q-3,R-1,S-2 P. Radiance 1. Outdoor thermal emission
P-5,Q-4,R-1,S-2 Q. Odeon 2. Construction management
R. Rayman 3. Air flow analysis
(D) P-4,Q-1,R-2,S-5
S. Primavera 4. Acoustic design
5. Lighting Analysis
Ans:- (C)
Software Description
Radiance Radiance is a tool primarily used for lighting analysis, especially in architectural
spaces. It’s not for outdoor thermal emission or air flow analysis.
Odeon Odeon is specialized software for acoustic design, particularly in room and
building acoustics. It’s not used for construction management.
Rayman RayMan is used for assessing outdoor thermal comfort and human
biometeorology, not for air flow analysis.
Primavera Primavera is a project and construction management software, widely used in
various industries for planning and executing complex projects.
DIALux Widely used for professional lighting design in various environments,
including indoor and outdoor spaces.
Relux A software tool for 3D modeling of sound propagation in enclosed spaces.
EASE (Enhanced Utilized for room acoustics and theater design, providing detailed acoustic
Acoustic Simulator analysis.
for Engineers)
284 GATE Architecture and Planning: Comprehensive Question Bank

ENVI-met Focuses on microclimate analysis, suitable for studying thermal comfort and
air quality in urban environments.
Ladybug Tools A collection of free and open-source tools that integrate with CAD software
for environmental analysis, including outdoor thermal comfort.
Microsoft Project Offers features for project planning, scheduling, resource allocation, and
tracking, widely used in various industries.
AutoDesk BIM 360 Part of AutoDesk’s suite of tools, BIM 360 is geared towards construction
management with a focus on Building Information Modeling (BIM).
Q.29 Match the street layouts of ancient Indian settlements in Group-I with their corresponding columns
in Group-II
(A) P-2,Q-4,R-3,S-5 Group I Group II
(B) P-2,Q-3,R-4,S-5 Nandyavarta
(C) P-4,Q-3,R-5,S-1
(D) P-4,Q-3,R-2,S-1
Swastika

Padmaka

Chaturmukha

5. Karmukha

Ans:- (B)
Explanation: Please refer GATE-2013, Q. 7 for more information.
Q.30 Associate the terms in Group-I with the building hardware in Group-II
P-3,Q-1,R-2,S-4 Group I Group II
(B) P-2,Q-4,R-1,S-3 P. Parliamentary 1. Bar
(C) P-3,Q-4,R-1,S-2 Q. Aldrop 2. Lock
R. Panic 3. Hinge
(D) P-2,Q-3,R-4,S-1
S. Mortise 4. Bolt

Ans:- (C)
Q.31 Hoop & Meridional force are associated with
(A) Dome (B) Truss
(C) Folded Plate (D) Space Frame
Hemant Vilas Parulekar 285

Ans:- (A)

Q.32 Match the Olympic stadia in Group-I with their Architects in Group-II
P-5,Q-2,R-1,S-4 Group I Group II
P-5,Q-3,R-1,S-2 P. Palazzetto dello Sport, Rome 1. Herzog & de Meuron
P-2,Q-1,R-4,S-5 Q. Olympic Arena, Tokyo 2. Frie Otto
R. Bird Nest, Beijing 3. Kenzo Tange
(D) P-2,Q-4,R-1,S-3
S. Olympic Stadia, Munich 4. Roger Taillibert
P.L. Nervi
Ans:- (B)
Explanation: P. Palazzetto dello Sport, Rome
Architects: Designed by Pier Luigi Nervi and Annibale Vitellozzi.
Overview: The Palazzetto dello Sport was built for the 1960 Summer Olympics in Rome. Pier Luigi
Nervi is renowned for his innovative use of reinforced concrete, and this building is a prime example.
Architectural Significance: It features a ribbed dome structure, with the roof designed as a pre-stressed
concrete dome, which was groundbreaking at the time. This dome, supported by Y-shaped pillars, gives
it a distinctive appearance and demonstrates Nervi’s skill in merging structural function with aesthetic
form.
Q. Olympic Arena (Yoyogi National Gymnasium), Tokyo
Architect: Designed by Kenzo Tange.
Overview: Built for the 1964 Tokyo Olympics, the Yoyogi National Gymnasium is one of the most
iconic buildings of the 20th century and a masterpiece of Kenzo Tange, a leading figure in the Metabolist
movement in architecture.
Architectural Significance: The building is famous for its suspension roof design, which was highly
innovative at the time. Its sweeping, curvilinear forms and the use of concrete and steel were pioneering
in sports facility design and influenced many future structures.
R. Bird’s Nest (Beijing National Stadium), Beijing
Architects: A collaborative design by the Swiss architecture firm Herzog & de Meuron, Chinese architect
Li Xinggang, and others.
Overview: Built for the 2008 Summer Olympics, the Beijing National Stadium, commonly known as the
Bird’s Nest due to its intricate lattice structure, is one of the most iconic Olympic stadiums ever built.
Architectural Significance: The stadium’s design is innovative for its radical aesthetics and the structural
complexity of the interwoven steel. The design not only created an iconic visual image but also addressed
functional aspects like natural ventilation, which is an essential factor in Beijing’s climate.
286 GATE Architecture and Planning: Comprehensive Question Bank

S. Olympic Stadium, Munich


Architects: Designed by Günter Behnisch and Frei Otto.
Overview: Built for the 1972 Summer Olympics, the Olympic Stadium in Munich is notable for its
revolutionary and lightweight tent-like roof structure.
Architectural Significance: Frei Otto, known for his work with tensile and membrane structures,
contributed to the distinctive and transparent canopies of acrylic glass stabilized by steel cables. This
design created a visually light and open structure, contrasting with the heavy, monumental buildings
typically used for previous Olympic venues.
Q.33 Match the terms in Group-I with their Architects in Group-II
P-5,Q-4,R-2,S-1 Group I Group II
P-5,Q-3,R-1,S-2 P. Acquisition 1. Ownership
P-3,Q-4,R-5,S-1 Q. Planning permission 2. Construction
R. Building plan sanction 3. Land cover
(D) P-5,Q-3,R-2,S-4
S. Mutation 4. Land use
5. Land
Ans:- (A)
Q.34 Associate the structural systems of Group-I with buildings in Group-II
P-1,Q-2,R-3,S-5 Group I Group II
(B) P-5,Q-3,R-4,S-1 P. Diagrid 1. Millennium Dome, London
(C) P-4,Q-2,R-3,S-1 Q. Outrigger truss 2. HSBC, Hong Kong
R. Suspended floor 3. Taipai 101
(D) P-4,Q-3,R-2,S-1
S. Cable stayed 4. Hearst Tower, New York
5. Sears Tower

Ans:- (D)
Millennium Dome, London HSBC, Hong Kong
Architect- Richard Rogers Architect- Norman Foster
Structural system- Cable stayed Structural system- Suspended Floor
Hemant Vilas Parulekar 287

Taipai 101 Hearst Tower


Architect- C. Y. Lee Architect- Norman Foster
Structural system- Outrigger truss Structural system- Diagrid

Q.35 Associate the systems of Group-I with applications in Group-II


P-1,Q-2,R-3,S-5 Group I Group II
P-5,Q-3,R-4,S-1 P. Nisargruna 1. Renewable energy generation
P-4,Q-2,R-3,S-1 Q. Vortex-DEWAT 2. Ground Water Discharge
R. Swale 3. Solid waste management
(D) P-4,Q-3,R-2,S-1
S. BIPV 4. Desalination
5. Waste water treatment
Ans:- (D)
Nisargruna Vortex-DEWAT
Nisargruna is a technology developed by the Vortex-DEWAT (Decentralized Wastewater
Bhabha Atomic Research Centre (BARC) in Treatment) refers to a sustainable wastewater
India for processing biodegradable waste. It treatment method.
primarily deals with the treatment of organic
waste using biogas technology.
288 GATE Architecture and Planning: Comprehensive Question Bank

Swale BIPV
Swale is a landscaping feature used for sustainable BIPV stands for Building Integrated Photovoltaics,
water management, particularly in rainwater a technology that integrates photovoltaic materials
harvesting and stormwater management. into building structures, like the roof, façade, or
windows.

Q.36 Match the houses in Group-I with their Architects in Group-II


P-5,Q-2,R-4,S-1 Group I Group II
P-3,Q-4,R-5,S-1 P. Villa Müller, Prague 1. Frank Gehry
P-3,Q-2,R-5,S-4 Q. Farnsworth, Illinois 2. Construction
R. Schröder House, Utrecht 3. Land cover
(D) P-5,Q-4,R-2,S-3
S. Dancing House, Prague 4. Land use
5. Gerrit Reitveld

Ans:- (B)
Q.37 Match the Books in Group I with their authors in Group II
P-3,Q-4,R-1,S-5 Group I Group II
P-4,Q-3,R-1,S-2 P. Space, Time & Architecture 1. Bill Hillier
P-5,Q-4,R-2,S-3 Q. The social logic of space 2. Christopher Alexander
R. Timeless way of building 3. Rob Krier
(D) P-4,Q-1,R-2,S-5
S. Form, space & order 4. Sigfried Giedion
5. Frank D K Ching

Ans:- (D)
Q.38 Associate the green rating system of Group-I with the respective country in Group-II
P-2,Q-4,R-5,S-1 Group I Group II
(B) P-4,Q-1,R-5,S-2 P. CASBEE 1. Bill Hillier
(C) P-3,Q-5,R-1,S-2 Q. Greenmark 2. China
R. GRIHA 3. Japan
(D) P-3,Q-4,R-5,S-1
S. Estidama 4. Singapore
5. India
Hemant Vilas Parulekar 289

Ans:- (D)
Q.39 Match the instruments in Group I with the corresponding usage in Group II
(A) P-2, Q-1, R-5, S-3 (B) Group I Group II
P-4, Q-3, R-1, S-2 P. Pyranometer 1. Shadow Analysis
(C) P-4, Q-1, R-5, S-3 (D) Q. Heliodon 2. Seismic intensity
P-1, Q-4, R-2, S-3 R. Totalstation 3. Wind velocity
S. Anenometer 4. Solar radiation
5. Land use
Ans:- (C)
Q.40 As per URDPFI guidelines, the number of Equivalent Car Space (ECS) required to accommodate
ten cars, sixteen scooters and forty bicycles will be______________
Vehicle Type ECS Ans:- 18
Car/Taxi 1.00 Solution: Given, Cars = 10, Scooter = 16, Bicycles = 40
Two-Wheeler 0.25 ECS as per URDPFI; Cars = 10 x 1 = 10
Auto rickshaw 0.50
Scooter = 16 x 0.25 = 4
Bicycle 0.10
Truck/Buses 2.5 Bicycles = 40 x 0.10 = 4
Emergency vehicles 2.5 ECS total = 10 + 4 + 4 = 18
Rickshaws 0.8
Q.41 A steel I-beam section is subjected to a bending moment of 96 kN-m. The moment of inertia of the
beam section is 24,000 cm4. The bending stress at 100 mm above the neutral axis of the beam in MPa
will be ____________ (2 marks)
Answer: 40
Solution: Bending stress = (96x1000x0.1)/ (24000×10-8) = 40 x 106 Pa = 40MPa
Bending stress = (My)/I
M = the internal bending moment about the section‘s neutral axis
y = the perpendicular distance from the neutral axis to a point on the section
I = the moment of inertia of the section area about the neutral axis
Q.42 A room is mechanically ventilated through four air-conditioning ducts. The opening area of each
duct is 0.35 sqm. The air velocity in the duct is 0.5 m/s. The temperature difference between the
ambient air and supply air is 10 °C. Volumetric specific heat of air is 1250 J/m3 °C. Assuming one
Ton of refrigeration (TR) equals 3.5 kW, the cooling load of the room in TR will be_____________
Answer: 2.5
Solution: Total area of the duct = 4 x 0.35 = 1.4 sqm
Total air volume in the room collected in 1 hr (3600 sec) = 1.4 x 0.5 x 3600 = 2520 cum
Total energy required = ms∆t = 2520 x 1250 x 10 = 31500000 Joule
Load in TR = (31500000/3.5KW)/3600 =
(31500000/3500)/3600 =2.5
290 GATE Architecture and Planning: Comprehensive Question Bank

Q.43 A CPM network of a construction project is given in the figure below. The activity durations are
mentioned in weeks. The project completion time in weeks will be _____________

Ans:- 18
Q.44 For a room with dimensions 4m×3m×3m (L×B×H), the details of indoor acoustical treatment as
follows.
The reverberation time in seconds at 1000 Hz is _____.
Component Wall Ceiling Floor
Percentage area 30 70 40 60 100
Absorption coefficient at 1000 Hz 0.4 0.1 0.6 0.1 0.1
Ans: 0.44 to 0.46
Q.45 In 2001, the population and work force participation rate of a town were 30,000 and 30 percent
respectively. The work force participation rate in the year 2011 increased to 34 percent. If the
decadal population growth rate was 6 percent, the increase in the number of working people in the
town in 2011 was __________
Ans:- 1812
Solution:
Working population in 2001 = 9000
In 2011, total population = 30,000 + 6% of 30,000 = 31,800
Now, 34% of 31,800 is the work force = 10812
So, increase in work force = 10812 – 9000 = 1812
Q.46 In a 20 storey building with 3m floor to floor height, a passenger lift is hoisted by a steel rope.
Weight of the lift car is 750 kg and ultimate load the steel rope can carry is 39,000 kg. Assuming
a factor of safety of 20 for the steel rope and an average passenger weight of 75 kg, the passenger
capacity of the lift is ____________.
Ans:- 16
Solution:
Usable strength of steel rope = 39000/20 = 1950 kg
Strength of steel rope considering lift car weight = 1950 – 750 = 1200kg
Capacity of lift = 1200/75 = 16 person
Hemant Vilas Parulekar 291

Q.47 One litre of acrylic paint can cover 16 sq m of wall area for the first coat and 24 sqm for the second
coat. The walls of a lecture hall measuring 12m × 8m × 4m (L × B × H) need to be painted with two
coats of this paint. The hall has total glazed fenestration area of 12 sq m. The number of 4 litre paint
containers required will be __________
Ans:- 4
Solution: Given, Length = 12m, Breadth = 8m, Height = 4m
Total Area = 2 [(12x4)+(8x4)] = 160 sq.m.
Area of fenestrations = 12 sq.m
Net Area = 160 -12 = 148 sq.m.
Qunatity of paint required for first coat = 148/16 = 9.25 litres
Quantity of paint required for second coat = 148/24 = 6.17 litres
Total quantity of paint required = 9.25 + 6.17 = 15.42 litres
A container consists 4 liters of paint
Total containers required = 15.42/ 4 = 3.86 say 4
Q.48 A 250 mm × 250 mm RCC column is reinforced with one percent steel. The permissible compressive
stress of concrete and steel are 8 N/mm2 and 150 N/mm2 respectively. The axial load carrying capacity
of the column in kN is _____________
Ans:- 585-595
Pu =0.4 fck Ac + 0.67 fyAsc
Pu = 0.4 x8 x 0.0625(1-1/100) + 0.67 x 150 x 0.0625×1/100 = 0.198+ 0.063 = 0.261 = 261kN
where
Pu= factored axial load on the member,
fck= characteristic compressive strength of the concrete,
Ac= area of concrete,
fy= characteristic strength of the compression reinforcement, and
Asc= area of longitudinal reinforcement for columns.
Q.49 A solar photo-voltaic system is proposed to be installed at the roof top of a hostel. The cost of
installation and the annual maintenance are INR 2,40,000 and INR 6000 respectively. It is expected
to generate 600 kWh of electricity per month. Assume unit price of electricity as INR 5. Ignoring the
discount rate, the payback period of the investment in years is____________
Ans:- 8
Solution:
Let the payback time be ‘P’ year.
So, 240000 + 6000P = 600x5x12xP
⇒ 240000 + 6000P = 36000P
⇒ 240000 = 30000P
⇒ P = 8 Years
292 GATE Architecture and Planning: Comprehensive Question Bank

Q.50 A pump is installed in an apartment building to lift water from ground level to the roof top water
tank with the capacity of 10,000 litres. Total head of lift is 18 m and pumping time is 30 minutes to
fill the tank completely. Assuming acceleration due to gravity (g) as 10 m/sec2 and efficiency of the
pump as 80 percent, the power requirement of the pump in kW will be __________
Ans:- 1.25
Solution: Power = Energy/ Time = Work Done/ Time = Mgh/30min
= 10000x10x18/1800 sec = 1000
As the pump is working at 80% efficiency, power = 10/8 (1000) =1250 Watt =1.25 Watt
Q.51 In a housing project, the number of LIG, MIG and HIG units are in the ratio of 1:1:2. The ratio of
areas of the units is 3:5:8. Assume unit cost of construction is same for all the three types. For a no
profit no loss situation, if 10% discount is offered to LIG and MIG units on sale price, extra charge
in percentage payable per HIG unit will be ____________
Ans:- 5
Solution: Let,
No. of LIG = 1
No. of MIG = 1
No. of HIG = 2
Total area of LIG = 1×3 = 3 unit
Total area of MIG = 1×5 = 5 unit
Total area of HIG = 2×8 = 16 unit
Also assume,
Cost of construction for all LIG = 3unit
Cost of construction for all MIG = 5unit
Cost of construction for all HIG = 16 unit
As 10% is on LIG, so 10% 3 unit = 0.3 would be burden on HIG
As 10% is on MIG, so 10% 5 unit = 0.5 would be burden on HIG
Total 0.3 + 0.5 = 0.8 would be burden on HIG cost which totals 16 units
So, % change in cost = (0.8/16) x 100% = 5%
Q.52 The estimated number of bricks (unit size: 250 mm × 125 mm × 75 mm) for laying one course of a
250 mm thick brick wall using rat-trap bond for a running length of 3.9 meter will be ___________.
Ans:- 36
Q.53 The difference in invert levels between two pits separated by a distance of 30 meter is one meter. An
intermediate pit is required to be constructed at a distance of 18 meter from the pit at higher level.
Maintaining the same slope, the difference in invert levels of the new pit and the pit at lower level in
mm will be _________
Ans:-400
Solution: 30/1= 12/x => x = 12/30 = 0.4 m =400mm
Hemant Vilas Parulekar 293

Q.54 A four-storey building with equal areas in each floor is required to be designed on a plot with FAR
of 2.0. If the FAR is increased to 2.2, the percentage increase in ground coverage utilizing full FAR
in both cases will be ___________
Ans:- 10
Solution: Let site area be 100 units. FAR = 2 means built up area = 2×100 = 200 units. As each floor has
equal area, so each floor area = 200/4 = 50 unit. So ground coverage = 50 sqm.
In the second case, site area = 100 unit. Built up area = 220 as FAR=2.2
Ground coverage = 220/4 = 55
So ground coverage is increased from 50 to 55. Therefore, change in % = 10%
Q.55 A lamp source of 3200 candela is mounted on a wall at a height of 2 meter from the work-plane.
It subtends an angle of incidence of 60o with the center of the work plane. The illumination at the
centre of the work plane in Lux is __________
Ans:- 100
Soln: E (lux)= [ candela /distance square ]* Cosθ
= [3200/4]*Cos60 = [800]*0.5 = 400
GATE QUESTION PAPER 2015
General Aptitude (GA)

GENERAL APTITUDE (QUESTION 1-10)


Q.1-Q.10 carry one mark each
Q.1 The principal presented the chief guest with a _______, as token of appreciation.
(A) Momento (B) Memento (C) Momentum (D) Moment
Ans:- (B)
Q.2 Choose the appropriate word/phrase out of the four options given below, to complete the following
sentence.
Frogs _____
(A) Croak
(B) Roar
(C) Hiss
(D) Patter
Ans:- (A)
Q.3 Choose the word most similar in meaning to the given word.
Educe.
(A) Exert
(B) Educate
(C) Extract
(D) Extend
Ans:- (C)
Q.4 Operators □, ∆ and → are defined by: a □ b = (a-b)/(a+b); a ∆ b = (a+b)/(a-b); a→b = ab
Find the value of (66 □ 6) → (66 ∆ 6)
(A) -2 (B) -1 (C) 1 (D) 2
Ans:- (C)
Solution: (66 □ 6) = (66 – 6) / (66 + 6) = 60/72
(66 ∆ 6) = 72/60
Thus, (66 □ 6) → (66 ∆ 6) = (60/72) x (72/60)
Q.5 If logx(5/7) = -1/3, then value of x is,
(A) 343/125
(B) 125/343
(C) -25/49
(D) –49/25
Hemant Vilas Parulekar 295

Ans:- (A)
Solution: Rule of logarithm, if aB = x, then ax = B
Thus, (5/7) = x-1/3
Thus, 5/7 = 1/x-1/3
x1/3 = 7/5
x = (7/5)3
x = (343/125)
Here are the basic rules of logarithms:
Product Rule:
logb(m×n)=logb(m)+logb(n)
The logarithm of a product is the sum of the logarithms of the factors.
Quotient Rule:
logb(nm)=logb(m)−logb(n)
The logarithm of a quotient is the difference of the logarithms.
Power Rule:
logb(mn)=n×logb(m)
The logarithm of a power of a number is the exponent times the logarithm of the base number.
Base Change Rule:
logb(m)=logk(b)logk(m)
The logarithm base b can be converted to a different base k using this formula.
Zero and Negative Rule:
The logarithm of 1 to any base is 0: logb(1) = 0.
Logarithms of negative numbers and zero are not defined in the real number system.
Identity Rule:
logb(b)=1
The logarithm of a base to itself is always 1.
Q.6-Q.10 carry two marks each.
Q.6 The following question presents a sentence, part of which is underlined. Beneath the sentence,
you find tour ways of phrasing the underlined part. Following the requirements of the standard
written English, select the answer that produces the most effective sentence.
Tuberculosis, together with its effect, ranks one of the leading causes of death in India.
(A) ranks as one of the leading causes of death.
(B) rank as one of the leading causes of death.
(C) has the rank of one of the leading causes of health.
(D) are one of the leading causes of death.
296 GATE Architecture and Planning: Comprehensive Question Bank

Ans:- (A)
Q.7 Read the following paragraph and choose the correct statement.
Climate change has reduced human security largely depends upon environmental security. But on
the contrary, human progress seems contradictory to environmental security. To keep up both at
the required level is a challenge to be addressed by one and all. One of the ways to curb the climate
change may be suitable scientific innovations, while the other may be the Gandhian perspective on
small progress with focus on sustainability.
(A) Human progress and security are positively associated with environmental security.
(B) Human progress is contradictory to environments security.
(C) Human security is contradictory to environmental security.
(D) Human progress depends upon environmental security.
Ans:- (B)
Q.8 Fill in the missing value.

Ans:- 3
Explanation: Figure at center = sum of both sides/2
Q.9 A cube of side 3 units is formed using a set of smaller cubes of side 1 unit. Find the proportion of
the number of faces of the smaller cubes visible to those which is NOT visible.
(A) 1:4 (B) 1:3 (C) 1:2 (D) 2:3
Ans:- (C)
Explanaton: To solve this, let’s first calculate the total number of smaller cubes that make up the larger
cube and then determine how many of their faces are visible versus not visible.
1. Total Number of Smaller Cubes:
- The larger cube has a side of 3 units.
- Each smaller cube has a side of 1 unit.
- Therefore, the larger cube contains 3×3×3=273×3×3=27 smaller cubes.
2. Visible Faces of Smaller Cubes:
- Each face of the larger cube will show the faces of 9 smaller cubes (since 3×3=93×3=9).
- The larger cube has 6 faces.
- Therefore, the number of visible faces of smaller cubes = 6×9=546×9=54 faces.
Hemant Vilas Parulekar 297

3. Total Faces of Smaller Cubes:


- Each smaller cube has 6 faces.
- With 27 smaller cubes, the total number of faces = 27×6=16227×6=162 faces.
4. Not Visible Faces of Smaller Cubes:
- The number of faces not visible = Total faces - Visible faces.
- This is 162−54=108162−54=108 faces.
5. Proportion of Visible to Not Visible Faces:
The proportion is 54 (visible) : 108 (not visible).
The proportion of the number of faces of the smaller cubes visible to those which are not visible is
54:108. This ratio simplifies to 1:2.
Q.10 Humpty, Dumpty sits on a wall every day while having lunch. The wall sometimes breaks. A person
sitting on the wall falls if the wall breaks.
Which one of the statements below are logically valid and cnan be inferred from the above sentences?
(A) Humpty Dumpty always falls while having lunch.
(B) Humpty Dumpty does not fall sometimes while having lunch.
(C) Humpty Dumpty never falls during dinner.
(D) When Humpty Dumpty does not sit on the wall, the wall does not break.
Ans:- (A)
Q.1 – Q.25 carry one mark each.
Q.1 A Housing Finance Institution in the private sector is
(A) HUDCO (B) SBI (C) PNB (D) HDFC
Ans:- (A)
Explanation: A Housing Finance Institution in the private sector typically refers to a non-governmental
organization that specializes in financing for housing, such as mortgages. Among the options given:
(A) HUDCO (Housing and Urban Development Corporation Limited): This is a government-owned
corporation in India that focuses on urban and housing development. It’s not in the private sector.
(B) SBI (State Bank of India): SBI is a public sector banking and financial services statutory body. It is
government-owned, not private.
(C) PNB (Punjab National Bank): PNB is a state-owned corporation based in New Delhi, India. It’s
also a public sector bank, not a private institution.
(D) HDFC (Housing Development Finance Corporation): HDFC is an Indian financial services
company that specializes in housing finance. It is one of the largest private sector housing finance
institutions in India.
Given these, the correct answer is:
(D) HDFC
298 GATE Architecture and Planning: Comprehensive Question Bank

Q.2 Which of the following statements regarding PERT is NOT true?


(A) Each activity of PERT network has three different time estimates
(B) Expected activity time is estimated based on β-distribution
(C) PERT is a deterministic model
(D) PERT network may have more than one critical path
Ans:- (C)
Explanation: PERT (Program Evaluation and Review Technique) is a project management tool used
to schedule, organize, and coordinate tasks within a project. Let’s evaluate the provided statements for
their accuracy:
(A) Each activity of PERT network has three different time estimates: True. In PERT, each activity is
typically estimated using three times: optimistic time, most likely time, and pessimistic time.
(B) Expected activity time is estimated based on β-distribution (Beta distribution): True. PERT
uses the Beta distribution to model the variability in activity times and to estimate the expected time
for each activity.
(C) PERT is a deterministic model: Not true. PERT is actually a probabilistic model, not deterministic.
It accounts for uncertainty and variability in activity times, which is the opposite of a deterministic
approach where outcomes are precisely predicted without accounting for randomness or uncertainty.
(D) PERT network may have more than one critical path: True. A PERT network can have multiple
critical paths, and all of them must be managed carefully as any delay in these paths can delay the whole
project.
Given these explanations, the statement that is NOT true about PERT is:
(C) PERT is a deterministic model
Q.3 Damage of foundation due to ‘Soil Liquefaction’ is related to
(A) Cyclones (B) Landslides
(C) Floods (D) Earthquakes
Ans:- (D)
Explanation: “Soil liquefaction” refers to a phenomenon where saturated soil substantially loses
strength and stiffness in response to applied stress, such as shaking during an earthquake, causing it to
behave like a liquid. This process can cause significant damage to foundations and structures. Among
the given options, soil liquefaction is most closely related to:
(D) Earthquakes
Liquefaction is primarily associated with seismic activities like earthquakes. When the seismic waves
from an earthquake travel through waterlogged soil, they can cause the soil particles to lose contact with
each other, leading to liquefaction. This can undermine the stability of buildings and other structures,
leading to tilting, sinking, or even collapse.
Q.4 Walls with high thermal inertia are suitable in which type of climate?
(A) Hot-dry (B) Hot-humid (C) Temperate (D) Cold
Hemant Vilas Parulekar 299

Ans:- (D)
Explanation: Walls with high thermal inertia (also known as high thermal mass) are particularly
effective in climates where there is a significant temperature difference between day and night. These
walls absorb heat during the day and release it slowly at night, helping to regulate indoor temperatures.
Given the options:
(A) Hot-dry: Hot-dry climates often have significant temperature swings between day and night. High
thermal inertia walls are suitable in these climates as they can absorb heat during the hot daytime and
release it during the cooler night, helping to maintain a more constant indoor temperature.
(B) Hot-humid: In hot-humid climates, the temperature does not drop significantly at night, so the
benefit of high thermal inertia in moderating indoor temperatures is less pronounced. Additionally,
high humidity can make the indoor environment uncomfortable if the walls retain too much heat.
(C) Temperate: Temperate climates usually have moderate temperature variations between day and
night. While high thermal inertia can be beneficial, its advantages are more pronounced in climates
with greater temperature swings.
(D) Cold: In cold climates, the main concern is usually retaining heat. While high thermal inertia
can help store and slowly release heat, the effectiveness depends on the availability of daytime heating
sources. It’s more critical to have good insulation in consistently cold environments.
Given these considerations, the most suitable climate for walls with high thermal inertia is a (A) Hot-
dry climate, where the diurnal temperature range is typically high, and the thermal mass can effectively
regulate indoor temperatures.
Q.5 The ratio of town area to agricultural land area as suggested by Sir Ebenezer Howard in ‘Garden
City’ concept is
(A) 1:20 (B) 1:15 (C) 1:10 (D) 1:5
Ans:- (D)

Q.6 A ‘Demolition Contract’ for a building is awarded to the


(A) Lowest Bidder (B) Highest Bidder
(C) Second Lowest Bidder (D) Second Highest Bidder
300 GATE Architecture and Planning: Comprehensive Question Bank

Ans:- (B)
Explanation:
- Lowest Bidder: This is the bidder who offers to do the job at the lowest cost among all the bidders.
They are often chosen because they represent the most cost-effective option for the project.
- Highest Bidder: This is the bidder who offers the highest cost for the project. In typical competitive
bidding scenarios, the highest bidder would not be selected because they would be the most expensive
option.
- Second Lowest Bidder: In some cases, the lowest bidder may not meet all the necessary qualifications
or requirements, or there might be concerns about their ability to complete the project successfully. In
such cases, the contract may be awarded to the second lowest bidder, who also submitted a competitive
but slightly higher bid.
- Second Highest Bidder: Selecting the second highest bidder is uncommon and usually not the
standard practice. Contracts are typically awarded to the lowest or second lowest bidder.
The choice of which bidder to award the contract to depends on various factors, including the project’s
requirements, budget constraints, and the qualifications and reputation of the bidders.
Q.7 Bulking of sand is highest in
(A) Coarse sand
(B) Medium sand
(C) Fine sand
(D) Sand saturated withwater
Ans:- (C)
Explanation: Bulking of sand refers to the increase in the volume of sand when it’s saturated with water.
This is an important consideration in construction because the volume of sand can change significantly
when it becomes wet, which can affect the accuracy of measurements and the performance of concrete
or mortar.
The bulking of sand is generally highest in fine sand. Fine sand particles have a larger surface area
relative to their volume compared to coarse sand, which allows them to hold more water and exhibit
greater expansion when saturated. Therefore, among the options given, fine sand is the type of sand
that experiences the highest bulking when saturated with water.
Q.8 The Venice Charter (1964) led to the establishment of
(A) International Centre for the Study of the Preservation and Restoration of Cultural Property
(ICCROM)
(B) International Council on Monuments and Sites(ICOMOS)
(C) Indian National Trust for Art and Cultural Heritage(INTACH)
(D) Archaeological Survey of India(ASI)
Hemant Vilas Parulekar 301

Ans:- (B)
Explanation: The Venice Charter, formally known as the “International Charter for the Conservation
and Restoration of Monuments and Sites,” was adopted in 1964 and is a significant document in the
field of heritage conservation. It laid down guidelines for the conservation and restoration of ancient
buildings and had a profound impact on heritage conservation practices globally.
The Venice Charter led to the establishment of:
(B) International Council on Monuments and Sites (ICOMOS)
ICOMOS is an international non-governmental organization dedicated to the conservation of the
world’s monuments and sites. It was founded as a result of the principles set forth in the Venice Charter.
The other options, while related to the field of heritage and cultural conservation, were not direct
outcomes of the Venice Charter.
Q.9 The ratio between illumination at a working point indoor to total light available simultaneously
outdoor is known as
(A) Daylight Factor (B) Sky Component
(C) Internally Reflected Component (D) Externally Reflected Component
Ans:- (A)
Explanation: Daylight Factor (DF) is a common metric used in building design to assess the natural
light available inside a structure as a percentage of the natural light available outside. It’s a crucial
measure for understanding the effectiveness of natural lighting in indoor environments and is used in
sustainable building design to enhance energy efficiency and occupant comfort.
The Daylight Factor (DF) is expressed as a percentage and is calculated using the formula:
(DF)=(Outdoor Illuminance/Indoor Illuminance)×100%
Where:
• Indoor Illuminance is the level of natural light inside the building (measured in lux) at a specific
point or area.
• Outdoor Illuminance is the simultaneous level of natural light outside the building under
overcast sky conditions (also measured in lux).
The Daylight Factor is a ratio that indicates how much daylight is available inside a space compared to
the amount available outside. It’s an important metric in sustainable and environmental design, helping
to ensure that buildings make efficient use of natural light.
The Daylight Factor (DF) can also be calculated considering the three components that contribute to
the total illumination inside a room: Sky Component (SC), Externally Reflected Component (ERC),
and Internally Reflected Component (IRC). The formula in this context is:
Daylight Factor (DF)=SC+ERC+IRC
Where:
• SC (Sky Component): This is the part of daylight that comes directly from the sky, without any
reflection. It’s usually the largest component in spaces close to windows.
302 GATE Architecture and Planning: Comprehensive Question Bank

• ERC (Externally Reflected Component): This part of daylight reflects off outdoor surfaces (like
the ground, adjacent buildings, and other objects) before entering the room.
• IRC (Internally Reflected Component): This component represents the light that enters the
room, reflects off internal surfaces (walls, floors, ceilings, furniture), and then contributes to the
room’s overall illumination.
Each component’s value is expressed as a percentage, representing its contribution to the total indoor
illumination compared to the available outdoor light under overcast sky conditions. Calculating these
components requires detailed knowledge of the room geometry, surface reflectance properties, and
external environment.
Q.10 Which of the following vehicular traffic intersections converts all crossing into merging and
diverging sequences?
(A) Rotary (B) Manual Signaling
(C) Grade Separation (D) Automatic Signaling
Ans:- (A)

Q.11 The process of spraying Polyester, Polyurethane, Acrylic and Epoxy Plastic, followed by heat curing
onto metals is called
(A) Anodizing (B) Galvanizing
(C) Vitreous Enameling (D) Powder Coating
Ans:- (D)
Explanation: The process described – spraying Polyester, Polyurethane, Acrylic, and Epoxy Plastic
onto metals followed by heat curing – is known as:
(D) Powder Coating
Powder coating is a finishing process in which a powder material, typically a polymer, is applied as a
dry powder and then cured under heat to form a protective and decorative ‘skin’ on the metal’s surface.
This process is popular due to its durability, resistance to corrosion, and variety of finish options.
Q.12 The fundamental right pertaining to property ownership in India DOES NOT embrace:
(A) Sell, Lease, Donate or Bequeath (B) Mortgage
(C) Grant Easement (D) Change in use
Hemant Vilas Parulekar 303

Ans:- (D)
Explanation:
Q.13 Match the Elements in Group - I with their Applications in Group –II
(A) P-2,Q-5,R-3,S-1 Group I Group II
(B) P-3,Q-5,R-4,S-1 P. Bracket 1. Door
(C) P-3,Q-1,R-4,S-5 Q. Baluster 2. Dome
R. Key stone 3. Cornice
(D) P-2,Q-1,R-3,S-4
S. Holdfast 4. Arch
5. Staircase
Ans:- (B)
Q.14 Match the Buildings in Group-I with their Principal Architects in Group-II
(A) P-2,Q-4,R-5,S-3 Group I Group II
(B) P-3,Q-5,R-4,S-1 P. Wexner Centre for the Visual Arts,Ohio 1. I. M. Pei
(C) P-1,Q-2,R-5,S-3 Q. Vitra Fire station, Weilam Rhein,Germany 2. Peter Eisenman
R. AT&T Building, NewYork 3. Louis Kahn
(D) P-2,Q-4,R-1,S-5
S. Sher–e–Banglanagar, Dacca 4. Zaha Hadid
5. Philip Johnson
Ans:- (A)
Q.15 A combination of colours forming an equlateral triangle in a Colour Wheel is called
(A) Analogous Scheme (B) Triad Scheme
(C) Split Complementary Scheme (D) Double Complementary Scheme
Ans:- (B)
Explanation: A complementary color scheme in color theory involves using colors that are opposite
each other on the color wheel. This scheme is popular for its visual impact and the way it can make
colors appear more vibrant when placed next to each other. Here’s a more detailed explanation along
with a simple diagram representation:
Basic Concept
Color Wheel: The color wheel is a circular representation of colors, showing the relationship between
primary colors (red, blue, yellow), secondary colors (green, orange, violet), and tertiary colors (mixtures
of primary and secondary colors).
Complementary Colors: On the color wheel, complementary colors are directly opposite each other.
For example:
Red and Green
Blue and Orange
Yellow and Violet
Characteristics of Complementary Colors
Contrast and Vibrancy: When complementary colors are used together, they create a high contrast
and can make each other appear more vibrant and brighter. This is due to the way our eyes perceive
colors and the contrasting effects they have when placed side by side.
304 GATE Architecture and Planning: Comprehensive Question Bank

Visual Impact: Complementary color schemes are often used in art and design to create dynamic and
attention-grabbing visuals. They’re popular in advertising, graphic design, and fine arts.
Balance: While they are high-contrast, when used correctly, complementary colors can achieve a pleasing
balance. It’s important to manage the intensity and saturation of the colors to avoid overwhelming the
viewer.
Usage in Design
Art: Artists might use complementary colors to create dramatic compositions and highlight certain
areas of a painting.
Interior Design: In home decor, complementary colors can be used to create a lively and dynamic
space. For example, blue and orange accents in a room.
Graphic and Web Design: These schemes are used to create visually striking designs. For example, a
website might use a blue and orange color scheme for buttons and links to make them stand out.
Fashion: Complementary colors are often used in clothing to create bold and striking outfits.
Q.16 Desire Line diagram helps in
(A) completion of a project by a desired date
(B) meeting demand and supply in desired category of housing
(C) determining income versus expenditure pattern of individuals
(D) Origin-Destination analysis in transport planning
Ans:- (D) (Please refer GATE-2009, Q. 8 for more information)
Q.17 As per Fire Safety norms of NBC India for buildings having assembly and institutional occupancies,
the maximum travel distance in meters to an exit from the dead end of a corridoris
(A) 30 (B) 24 (C) 12 (D) 6
Ans:- (D)
Q.18 Which of the following is a part of a studio apartment?
(A) Master bedroom (B) Artist’s room
(C) Multipurpose space (D) Children’s room
Ans:- (C)
Q.19 The Saturation level of a colour represents
(A) distribution (B) brilliance (C) darkness (D) warmth
Ans:- (B)
Explanation:
Term Explanation
Refers to the pure spectrum colors commonly referred to by the ‘color name’,
Hue the aspect of color which is determined by the wavelength of light. Examples
include red, blue, yellow, etc.
Achieved by adding white to a pure hue, lightening the color. Tints of blue
Tint
would include light blue, sky blue, etc.
Created by adding black to a hue, darkening the color. For example, adding
Shade
black to red gives you burgundy.
Hemant Vilas Parulekar 305

Produced by adding gray (black and white) to a hue, reducing the color’s
Tone
brightness. Toning down a color often makes it more subtle and soft.
Refers to the intensity or purity of a color. Highly saturated colors are vivid and
Saturation
rich, while less saturated colors appear more muted or gray.
Concerns the brightness or darkness of a color. Lightness can be adjusted by
Value/Lightness
adding white or black (similar to tint and shade).
Relates to the purity of a color, similar to saturation. It measures the difference
Chroma of a color from a gray of the same lightness or brightness, assessing the vividness
or dullness of a color.
Q.20 Invert level of a pipe at a given cross section refers to the
(A) highest point of theinternal surface (B) lowest point of the internal surface
(C) highest point of theexternal surface (D) lowest point of the external surface
Ans:- (B)
Q.21 The command DVIEW in AutoCAD permits toview
(A) a selected portion of the drawing in detail
(B) the entire screen on the monitor
(C) a perspective of the drawing
(D) a damaged part of the drawing
Ans:- (C)
Q.22 Match the Land use categories of Group – I with their respective Colour codes in Group – II as per
practice in India
(A) P-5,Q-3,R-4,S-1 Group I Group II
(B) P-5,Q-4,R-2,S-1 P. Residential 1. Red
(C) P-1,Q-2,R-4,S-5 Q. Commercial 2. Grey
R. Industrial 3. Blue
(D) P-1,Q-3,R-2,S-4
S. Public /Semi-public 4. Violet
5. Yellow
Ans:- (A)
Q.23 A rectangular beam section of size 300 mm (width) X 500 mm (depth) is loaded with a shear force
of 600 kN. The maximum shear stress on the section in N/mm2 is ___
Ans:- 6
Solution: Maximum shear stress = (3 x F) / (2 x b x d) = (3 x 600 x 103) / (2 x 300 x 500) = 6 N/mm2
(For calculating Mean Shear Stress, use formula: Mean shear stress = F/(bxd), Max. stress = 1.5 x mean
shear stress)
Q.24 In a 50 meter section of a waste water pipe, if the gradient is 1 in 80, then the fall in millimetre is
__________
306 GATE Architecture and Planning: Comprehensive Question Bank

Ans:- 625
Solution: pipe section length = 50 m
Gradient ratio = 80 # 1 in 80
# Calculate the fall
Fall = Length of the pipe section / Gradient ratio
fall = pipe section length / gradient ratio
50/80 = 0.625m = 625mm
Q.25 A 15 meter long and 3 meter wide driveway needs to be paved with 300 mm X 300 mm square tiles.
If each packet contains 30 numbers of tiles, then the number of packets to be procured to pave the
whole area is
Ans:- 17
Solution: To find the number of packets needed to pave the entire area, you can follow these steps:
1. Calculate the total area of the driveway: Area = Length × Width Area = 15 meters × 3 meters Area
= 45 square meters
2. Determine the area covered by each tile: Tile Area = Tile Length × Tile Width Tile Area = 0.3 meters
× 0.3 meters Tile Area = 0.09 square meters
3. Calculate the total number of tiles needed: Total Tiles = Total Area / Tile Area Total Tiles = 45 square
meters / 0.09 square meters Total Tiles = 500 tiles
4. Find out how many tiles are in each packet: Tiles per Packet = 30 tiles
5. Calculate the number of packets needed: Number of Packets = Total Tiles / Tiles per Packet Number
of Packets = 500 tiles / 30 tiles per packet Number of Packets ≈ 16.67 packets = 17 packets
Q. 26 – Q. 55 carry two marks each.
Q.26 Match the Monuments in Group-I with their Features in Group-II
(A) P-2,Q-1,R-4,S-3 Group I Group II
(B) P-2,Q-1,R-5,S-5 P. Panch Mahal, Fathepur Sikri 1. Painted Stone Figures
(C) P-2,Q-4,R-1,S-3 Q. Meenakshi Temple, Madurai 2. Intricate Red Sand Stone
Carvings
(D) P-1,Q-5,R-5,S-4
R. Jor-Bangla Temple,Bishnupur 3. Granite Statues
S. Sun Temple, Konark 4. Khondalite Stone Work
5. Terracotta Carvings
Ans:- (B)
Panch Mahal, Fatehpur Sikri Meenakshi Temple, Madurai
Hemant Vilas Parulekar 307

Jor-Bangla Temple, Bishnupur Sun Temple, Konark

Q.27 Match the Monuments in Group-I with their Style of Architecture in Group-II
(A) P-5,Q-1,R-3,S-2 Group I Group II
(B) P-2,Q-4,R-3,S-5 P. Pisa Cathedral,Italy 1. Gothic
(C) P-4,Q-2,R-5,S-1 Q. St. Hagia Sophia,Istanbul 2. Moorish
R. Great Temple of Amun,Karnak 3. Egyptian
(D) P-5,Q-4,R-3,S-1
S. Cathedral of Notre Dame,Paris 4. Byzantine
5. Romanesque

Ans:- (D)
Pisa Cathedral, Romanasque St. Hagia Sophia, Byzantine

Great Temple of Amun, Egyptian Cathedral of Notre Dame, Gothic


308 GATE Architecture and Planning: Comprehensive Question Bank

Q.28 Match the Buildings in Group-I with their Style of Architecture in Group II
Group I Group II
(A) P-5,Q-3,R-2,S-1 P. Rashtrapati Bhavan, New Delhi 1. Industrial Architecture
(B) P-5,Q-4,R-2,S-1 Q. German Pavilion for World 2. Deconstruction
Exhibition, Barcelona
(C) P-1,Q-5,R-4,S-3
R. Guggenheim Museum, Bilbao 3. Radical Eclecticism
(D) P-3,Q-4,R-1,S-5 S. Crystal Palace, London 4. International style
5. Neo classical
Ans:- (B)
Rashtrapati Bhavan, Neo Classical German Pavilion for World Exhibition,
International Style

Guggenheim Museum, Deconstruction Crystal Palace, Industrial Architecture

Q.29 Match the Terms in Group – I with their Definitions in Group –II
(A) P-5,Q-3,R-4,S-1 Group I Group II
(B) P-5,Q-2,R-4,S-3 P. Kinesthesia 1. Measurement and study of size and proportions
(C) P-4,Q-1,R-2,S-5 of human body
Q. Anthropometry 2. Study of man – machine interaction
(D) P-4,Q-1,R-2,S-3
R. Ergonomics 3. Study of past and present of the human race
S. Biomimicry 4. Study of human sensory experience during
movement
5. Imitation of models, systems and elements of
nature

Ans:- (C)
Hemant Vilas Parulekar 309

Q.30 Match the following Urban Spaces in Group-I with their Names in Group-II

(A) P-4,Q-1,R-2,S-3 (B) P-2, Q-3,R-1,S-5


(C) P-4,Q-3,R-1,S-5 (D) P-2,Q-1,R-4,S-3
Ans:- (B)
Q.31 Match the Terms in Group – I with the appropriate Items in Group –II
(A) P-5,Q-4,R-2,S-1 Group I Group II
(B) P-5,Q-1,R-4,S-3 P. Topo sheet 1. Path/Row
(C) P-2,Q-1,R-2,S-1 Q. Satellite Image 2. Contour
R. Wavelength 3. Focal Length
(D) P-2,Q-4,R-1,S-5
S. Scan Line 4. Spectral Signature
5. Bits/inch
310 GATE Architecture and Planning: Comprehensive Question Bank

Ans:- (C)
Q.32 Match the Concepts in Group – I with their appropriate Explanation in Group – II
Group-I Group-II
P. Planned Development 1.Development occurring on vacant or un-
derused lots in otherwise built up areas
Q. Infill Development 2. Development providing a fair and equitable
way to integrate peri-urban areas
R. Transit Oriented Develop- 3. Developing a large area as a single en�ty
ment merging zoning and subdivision control
S. Mixed Use Development 4. Development with compa�ble land uses
integra�ng varied ac�vi�es at different �mes
of the day
5. Development located within walking dis-
tance from mass transit sta�ons along the
corridor
(A) P – 3 , Q – 2 , R – 5 , S– 4 (B) P – 3 , Q – 1 , R – 5 , S –4
(C) P – 2 , Q – 1 , R – 4 , S– 5 (D) P – 2 , Q – 4 , R – 1 , S –5
Ans:- (B)
Q.33 Particles of soil in descending order of grain size is
(A) Gravel – Sand – Silt–Clay (B) Gravel – Sand – Clay –Silt
(C) Sand – Gravel – Clay–Silt (D) Clay – Gravel – Sand –Silt
Ans:- (A)
Q.34 Match the Units in Group – I with their Definitions in Group –II
(A) P-5, Q-4, R-2, S-1 Group I Group II
(B) P-3, Q-1, R-5, S-4 P. Hertz 1. Newton - meter
(C) P-2, Q-3, R-1, S-4 Q. Lux 2. Cycles / second
R. Joule 3. Lumen / m2
(D) P-2, Q-3, R-1, S-5
S. Newton 4. Watt /ampere
5. kg - meter / sec2
Ans:- (D)
Q.35 Match the Energy Efficient Building Elements in Group-I with their associated Working Principles
in Group-II
(A) P-3,Q-2,R-4,S-5 Group I Group II
(B) P-5,Q-2,R-4,S-3 P. Solar Chimney 1. Thermal Storage
(C) P-3,Q-5,R-1,S-2 Q. Earth Air Tunnel 2. Radiant Cooling
R. Trombe Wall 3. Stack Effect
(D) P-4,Q-5,R-1,S-2
S. Chilled Slab 4. Cross Ventilation
5. Geothermal Energy
Ans:- (C)
Hemant Vilas Parulekar 311

Q.36 Match the Vibrator Types in Group-I with their related Areas of Application in B Group-

P-1,Q-5,R-4,S-3 Group I Group II


(B) P-3,Q-4,R-1,S-2 P. Needle Vibrator 1. Concrete Pavement
(C) P-1,Q-4,R-2,S-5 Q. ShutterVibrator 2. Pre-cast Concrete Unit
R. SurfaceVibrator 3. Beam-Column Junction
(D) P-3,Q-5,R-1,S-2
S. Table Vibrator 4. Retaining Wall
5. Slip Forming
Ans:- (B)
Needle Vibrator Shutter vibrator

Surface vibrator Table vibrator

Q.37 Match the type of Temporary Structures in Group – I with their corresponding Functions in Group –II
(A) P–2,Q–4,R–1,S–5 Group I Group II
(B) P–3,Q–5,R–1,S–2 P. Scaffolding 1. To support unsafe structure
(C) P–3,Q–4,R–5,S–2 Q. Formwork 2. To support platforms for workmen and
materials atraised
(D) P–2,Q–3,R–4,S–5
R. Shoring 3. Removal of water from pits
S. Underpinning 4. Mould for RCC Structure
5. Strengthening the existing foundation
312 GATE Architecture and Planning: Comprehensive Question Bank

Ans:- (A)
Scaffolding Formwork

Shoring Underpinning

Q.38 Match following Scientific Names in Group – I with their common Indian Names in Group –II
(A) P-2,Q-4,R-3,S-5 Group I Group II
(B) P-5,Q-3,R-2,S-4 P. Lagerstroemia speciosa 1. Amaltas
(C) P-3,Q-1,R- 4,S-2 Q. Cassia fistula 2. Neem atraised
R. Azadarachta indica 3. Jarul
(D) P-3,Q-1,R-2,S-4
S. Acacia auriculiformis 4. Babul
5. Peepal
Ans:- (D)
Q.39 A man starts from his residence and uses the following modes in sequence to reach his office - cycle
rickshaw to railway station, then train to destination station, followed by auto-rickshaw to nearby
bus stand and finally a bus to his office. Which of the following describes his sequence of transit
usage?
Non Motorised Transit – Paratransit – Mass Transit – Public Transit
Paratransit – Public Transit – Non Motorised Transit – Mass Transit
Private Transit – Public Transit – Non Motorised Transit – Mass Transit
Non Motorised Transit – Mass Transit – Paratransit – Public Transit
Ans:- (D)
Hemant Vilas Parulekar 313

Q.40 PMGSY and JNNURM are two Indian Government programmes which deal with
(A) rural road development and urban basic service improvement respectively
(B) rural sanitation services and under-developed road maintenance respectively
(C) peri-urban basic services and urban basic service improvement respectively
(D) rural road development and urban transport development respectively
Ans:- (A)
PMGSY (Pradhan Mantri Gram Sadak Yojana) deals with rural road development.
JNNURM (Jawaharlal Nehru National Urban Renewal Mission) deals with urban basic service
improvement in Indian cities.
Q.41 Match the Planning Terms in Group – I with their Descriptions in Group – II
(A) P-4,Q-3,R-5,S-2 Group I Group II
(B) P-3,Q- 4,R-1,S-5 P. Gentrification 1. Haphazard and low density outward
(C) P-1,Q-5,R-2,S-3 growth of urban area
(D) P-3,Q- 4,R-1,S-2 Q. Urban core 2. Primarily dormitory settlement with
revitalization functional dependency on parent city

R. Urban sprawl 3. Replacement of low income residents


with high income population
S. Satellite town 4. Physical and socio-economic revival of
the inner-city
5. Restricted development in an
environmentally sensitive zone

Ans:- (D)
Planning term Elaboration
Gentrification This is the process where a deteriorated urban neighborhood undergoes
transformation and upgrading, often leading to the displacement of lower-
income residents due to increased property values and living costs.

Urban Core Refers to efforts to reinvigorate the central parts of cities, often characterized
Revitalization by redevelopment, improved infrastructure, and enhanced public amenities
to attract investment and residents.
Urban Sprawl Describes the uncontrolled expansion of urban areas into the surrounding
rural land, often characterized by low-density development, reliance on cars,
and lack of efficient public transport.
Satellite Town A smaller town or city located near a larger city, functioning independently
but also connected economically and by commuting patterns to the larger
city.
Counter Magnets These are cities or towns identified to develop as alternative centers of growth
to reduce the migration pressure on a major city. The concept is often part of
regional planning strategies.
314 GATE Architecture and Planning: Comprehensive Question Bank

Incumbent Involves the improvement of existing urban areas, particularly informal


Upgradation settlements or slums, to enhance living conditions through better
infrastructure, services, and housing without displacing residents.
Peri-Urban Areas Regions located on the outskirts of urban areas, often characterized by a
mix of rural and urban land uses, and transitioning from rural to urban
characteristics.
Priority Towns These are towns or urban areas identified by planners or governments
for prioritized development, often to address specific issues like housing
shortages, economic development, or to balance regional growth.

Constellation In urban planning, this term can refer to a group of cities or towns that, while
geographically distinct, are linked through economic, social, or transport
networks, forming a larger interconnected urban system.

Q.42 Match the Planning Concepts in Group – I with their Corresponding Proponents in Group –II
(A) P-1,Q-4,R-3,S-5 Group I Group II
(B) P-1,Q-3,R-5,S-2 P. Lagerstroemia speciosa 1. Amaltas
(C) P-2,Q-1,R-3,S-5 Q. Cassia fistula 2. Neem
atraised
(D) P-2,Q-1,R-5,S-4
R. Azadarachta indica 3. Jarul
S. Acacia auriculiformis 4. Babul
5. Peepal
Ans:- (C)
Q.43 The housing stock of a town has total number of 9090 dwelling units. Present population of the
town is 45,450. Assuming an average household size of 4.5, the housing shortage in percentage is
Ans:- 10
Solution:
Calculate the required number of dwelling units for the present population: Required Dwelling Units
= Present Population / Average Household Size Required Dwelling Units = 45,450 / 4.5 Required
Dwelling Units = 10,100
Calculate the housing shortage: Housing Shortage = Required Dwelling Units - Total Number of
Dwelling Units Housing Shortage = 10,100 - 9,090 Housing Shortage = 1,010
Calculate the housing shortage as a percentage of the required dwelling units: Housing Shortage
Percentage = (Housing Shortage / Required Dwelling Units) * 100 Housing Shortage Percentage =
(1,010 / 10,100) * 100 Housing Shortage Percentage = 10%
Q.44 A hall is 15 m long and 12 m wide. If the sum of areas of the floor and ceiling is equal to the sum of
the area of its four walls, then the volume of the hall in cubic meter is
Ans:- 1200
Solution: Let’s denote: Length of the hall = L = 15 m Width of the hall = W = 12 m Height of the hall
= H (which we need to find)
The sum of the areas of the floor and ceiling is: 2 * (L * W) = 2 * (15 m * 12 m) = 360 m²
Hemant Vilas Parulekar 315

The sum of the area of its four walls is: 2 * (L * H) + 2 * (W * H) = 2 * (15 m * H) + 2 * (12 m * H) =
30H + 24H = 54H m²
According to the given information, these areas are equal, so we can equate them:
360 m² = 54H m²
Now, solve for H (the height of the hall):
H = 360 m² / 54 m² = 6.67 m (rounded to two decimal places)
So, the height of the hall is approximately 6.67 meters.
Now, calculate the volume of the hall: Volume = Length * Width * Height Volume = 15 m * 12 m * 6.67
m = 1200 m³
The volume of the hall is 1200 cubic meters.
Q.45 The actual roof area of a building is 3,60,000 sqm, which on a site plan measures 25 sq cm. The
scale of the site plan is 1:
Ans:- 1:12000
Solution: Actual Roof Area = 3,60,000 sqm Measured Area on Site Plan = 25 sq cm
Let the scale be 1:X.
So, you can set up the proportion:
(Actual Area) / (Measured Area) = 1 / X
Plug in the values:
(3,60,000 sqm) / (25 sq cm) = 1 / X
To find X, you can rearrange the equation:
X2 = (25 sq cm) / (3,60,000 sqm)
First, let’s convert both measurements to the same units. Since 1 sqm = 10,000 sq cm:
X2 = (25 sq cm) / (3,60,000 sqm * 10,000 sq cm/sqm)
X2 = 25 / 3,600,000,000
X2 = 1 / 144,000,000
X = 1 /12000
So, the scale of the site plan is 1:12000.
Q.46 If the annual net income from a commercial property is Rs 22,000/- and the interest rate is 8%,
then the capitalized value in rupees of the property in perpetuity is
Ans:- 2,75,000
Solution: To calculate the capitalized value of the property in perpetuity, you can use the formula for
the present value of a perpetuity:
Capitalized Value = Annual Net Income / Interest Rate
In this case:
Annual Net Income = Rs 22,000 Interest Rate = 8% (0.08 as a decimal)
316 GATE Architecture and Planning: Comprehensive Question Bank

Plug these values into the formula:


Capitalized Value = 22,000 / 0.08
Capitalized Value = 275,000
So, the capitalized value of the property in perpetuity is Rs 275,000.
Q.47 A five storied building is constructed on a 100 m x 50 m plot having ground coverage of 60%
(option 1). Alternatively, a four storied building is constructed on the same plot with a 50% ground
coverage (option 2). The ratio of FARs between options 1 and 2 is_____
Ans:- 1.5
Solution: To find the ratio of FARs (Floor Area Ratios) between the two options, we can calculate the
FAR for each option. The FAR is calculated by dividing the total floor area of the building by the plot
area.
Option 1: Plot area = 100 m x 50 m = 5000 sq. m Ground coverage = 60% of plot area = 0.60 x 5000 sq.
m = 3000 sq. m
Assuming each floor has the same area: Total floor area = 5 floors x 3000 sq. m/floor = 15000 sq. m
FAR for option 1 = Total floor area / Plot area = 15000 sq. m / 5000 sq. m = 3
Option 2: Plot area = 100 m x 50 m = 5000 sq. m Ground coverage = 50% of plot area = 0.50 x 5000 sq.
m = 2500 sq. m
Assuming each floor has the same area: Total floor area = 4 floors x 2500 sq. m/floor = 10000 sq. m
FAR for option 2 = Total floor area / Plot area = 10000 sq. m / 5000 sq. m = 2
Now, we can find the ratio of FARs: Ratio = FAR for option 1 / FAR for option 2 = 3 / 2 = 1.5
So, the ratio of FARs between option 1 and option 2 is 1.5.
Q.48 If a roof is treated with a layer of thermal insulation material, the internal heat gain is reduced by
60%. The U-value of the roof slab (without thermal insulation) is 3 W m2 / °C. Assuming a constant
temperature difference between indoor and outdoor, the U-value of the thermal insulation layer
in W m2 / °C is ____
Ans:- 2
Solution: The U-value is a measure of heat transfer; lower U-values indicate better insulation properties.
When a roof is treated with thermal insulation, the overall U-value of the roof assembly decreases,
reducing heat transfer.
Given:
The original U-value of the roof slab (without insulation) is 3 W/m²/°C.
The internal heat gain is reduced by 60% with the insulation.
The new U-value of the roof assembly (with insulation) can be calculated by reducing the original
U-value by 60%. Then, using the parallel-path method of heat transfer, we can find the U-value of the
insulation layer.
1. Calculate the New U-value of the Roof Assembly:
- New U-value = Original U-value * (1 - Heat Gain Reduction Percentage)
- New U-value = 3 W/m²/°C * (1 - 0.60)
Hemant Vilas Parulekar 317

2. Find the U-value of the Insulation Layer:


- The overall U-value of a composite structure like an insulated roof can be represented as the sum of
the inverses of the U-values of its components.
- 1/ Utotal = 1/Uroof + 1/Uinsulation
- Rearrange to find Uinsulation.
The U-value of the thermal insulation layer, when added to the roof slab, is approximately 2.0 W/
m²/°C. This calculation assumes a constant temperature difference between indoor and outdoor
environments.
Q.49 A simply supported beam having effective span of 5 meter is carrying a centrally concentrated load
of 16 kN. The maximum bending moment in the beam in kN-m is ___
Ans:- 20
Solution: Given values
Span length = 5 m
Central load = 16 kN
Max bending moment = (central load * span length) / 4
max bending moment = (16 * 5) /4
80/4 = 20 kN-m
Q.50 A landscaped garden with irregular profile and minor undulations, measuring 35,000 sqm, has a
total surface area covered with 20% brick paving, 15% cement concrete paving, and rest with grass.
The peak intensity of rainfall in that region is 70 mm/hr. The coefficient of runoff for brick paving,
cement concrete paving and grass is 0.8, 0.9 and 0.5 respectively. The estimated quantity of runoff
in cubic meter/hr for the entire garden area is ______
Ans:- 1510-1530
Solution: Given values
total_area = 35000 # total area of the garden in sqm
peak_rainfall_intensity = 70 # in mm/hr
# Area covered by different surfaces and their runoff coefficients
brick_paving_percentage = 0.20
cement_concrete_paving_percentage = 0.15
grass_percentage = 0.65 # remaining area covered with grass
brick_paving_runoff_coefficient = 0.8
cement_concrete_paving_runoff_coefficient = 0.9
grass_runoff_coefficient = 0.5
# Calculate the area covered by each surface
area_brick_paving = total_area * brick_paving_percentage
area_cement_concrete_paving = total_area * cement_concrete_paving_percentage
318 GATE Architecture and Planning: Comprehensive Question Bank

area_grass = total_area * grass_percentage


# Calculate the runoff for each surface
runoff_brick_paving = area_brick_paving * brick_paving_runoff_coefficient * peak_rainfall_intensity
runoff_cement_concrete_paving = area_cement_concrete_paving * cement_concrete_paving_runoff_
coefficient * peak_rainfall_intensity
runoff_grass = area_grass * grass_runoff_coefficient * peak_rainfall_intensity
# Total runoff from the entire garden area (in cubic meters per hour)
# Convert from mm to meters (1 mm = 0.001 m)
total_runoff = (runoff_brick_paving + runoff_cement_concrete_paving + runoff_grass) * 0.001
total_runoff
Q.51 The number of standard cement bags required to prepare 1400 kg of concrete in the ratio of 1 : 2 :
4 (mixed by weight batching) is ______
Ans:- 4.0
Solution: Total weight of mix (kg)=Cement weight (kg)+Sand weight (kg)+Aggregate weight (kg)
Since the ratio of cement:sand:aggregate is 1:2:4, we can say that the weight of cement is one part, and
the weight of sand is two parts, and the weight of aggregate is four parts.
Let’s assume the total weight of the mix is W kg. Then, the weight of cement (one part) is W/(1+2+4)
kg.
Calculate the weight of cement:
Cement weight (kg)=W/(1+2+4) = W/7
Now, we know that the total weight of concrete mix is 1400 kg:
W=1400kg
Substitute this value into the equation for cement weight:
Cement weight (kg)= 1400 / 7 kg = 200kg
To find the number of standard cement bags required, we’ll use the assumption that one standard
cement bag typically contains 50 kg of cement.
Number of cement bags required=Cement weight (kg)Weight per bag (kg)=200 kg/ 50 kg/bag=4 bags
So, you would need 4 standard cement bags to prepare 1400 kg of concrete in the given ratio.
Q.52 A class room measuring 10 m (L) x 8 m (B) x 2.7 m (H) requires an illumination level of 500 lux
on the desk level using 40 W fluorescent lamps with rated output of 5000 lumens each. Assuming
utilization factor of 0.5 and maintenance factor of 0.8, the number of lamps required is
Ans:- 20
Explanation: Formula, E = (N x F x U.F. x M.F.) / A
Where, E = Illumination required in lux, N = no. of lamps, F = lumen per lamp.
U.F. = Utilization factor, M.F. + Maintenance factor, A = Area of room.
Here, N=?
Thus, N = (E x A) / F x U.F. x M.F. = (500 x 80) / (5000 x 0.5 x 0.8) = 20
Hemant Vilas Parulekar 319

Q.53 Area of tensile steel per meter width of a reinforced concrete slab is 335 sq mm. If 8 mm rods are
used as reinforcement, then centre to centre spacing of the reinforcement in mm is
Ans:- Area of tensile steel per meter width = 335 sq.mm.
Area of 8mm rod = π x r2 = 3.14 x 4 x 4 = 50.24 sq.mm.
Thus, total no. of rods = 335/50.24 = 6.67, say 7 rods are placed in a slab having width 1m.
Thus, c/c spacing of rods = 1/6.67 = 0.149m = 149 mm.
Q.54 The population of a town as per Census 2011 was 22,730 and the population as per census 2001 was
15,770. Considering arithmetic projection of growth, the projected population in 2016 will be____
Ans:- 26178 – 26210
Year Popula�on
2001 15770
2011 22730
Population increase in 10 years = 22730 – 15770 = 6960. As per arithmetic projection, increase in 5
years will be = 6960/2 = 3480
Thus, population in 2016 = 22730 + 3480 = 26210
Q.55 Two concrete mixers of capacity 200 liters each are used in a construction site to produce 20 cubic
meter of concrete. Ingredient charging, mixing and discharge times are 3 minutes, 7 minutes and
1 minute respectively. Assuming a time loss of 5 minutes per hour of operation, the total time in
hours for the mixers to produce the required amount of concrete will be
Ans:- 9.9-10.0
Solution: Given, 1000 litres = 1cu.m., 200 litres = 0.2 cu.m. (apacity of each mixer)
Thus, total turns to produce 20 cu.m. concrete by 2 concrete mixers = 20 / (0.2 x 2) = 50
Time required for 1 turn = 3+7+1 = 11 minutes.
For 50 turns, time required = 11 x 50 = 550 minutes = 550/60 = 9.17 hours.
Time loss of 5 minutes per hour (0.084 hr.) will be added in this time.
Total time required = 9.17 + (9.17 x 0.084) = 9.94 hours
GATE QUESTION PAPER 2014
General Aptitude (GA)

Q. 1 – Q. 25 carry one mark each.


Q.1 A student is required to demonstrate a high level of comprehension of the subject, especially in the
social sciences.
The word closest in meaning to comprehension is
(A) Understanding (B)Meaning (C) concentration (D)Stabiity
Ans:- (A)
Explanation: Comprehension: the ability to understand something.
Q.2 Choose the most appropriate word from the options given below to complete the following sentence.
One of his biggest ______ was his ability to forgive.
(A) vice (B) virtues (C) choices (D) strength
Ans:- (B)
Explanation: Virtues: Behaviour showing high moral standards.
Q.3 Rajan was not happy that Sajan decided to do the project on his own. On observing his unhappiness,
Sajan explained to Rajan that he preferred to work independently.
Which one of the statements below is logically valid and can be inferred from the above sentences?
(A) Rajan has decided to work only in a group.
(B) Rajan and Sajan were formed into a group against their wishes
(C) Sajan had decided to give in to Rajan’s request to work with him.
(D) Rajan had believed that Sajan and he would be working together.
Ans:- (D)
Q.4 If y = 5x²+ 3, then the tangent at x = 0, y = 3
(A) passes through x = 0, y = 0      (B) has a slope of +1
(C) is parallel to the x-axis (D) has a slope of −1
Ans:- (C)
Solution: Given equation of line, y=5x² + 3
Thus, Slope of line, y’ = 10x Thus, at x=0, y’=0
Equation of tangent with gradient ‘m’ and intercept ‘c’ on y-axis, y=mx+c
Thus, at x=0 and y=3
y=c=3
Thus, equation of tangent is, y=m(0)+3 i.e. y=3. Thus, tangent at x=0,y=3 is parallel to the x-axis.
Hemant Vilas Parulekar 321

Q.5 A foundry has a fixed daily cost of Rs 50,000 whenever it operates and a variable cost of Rs 800Q,
where Q is the daily production in tonnes. What is the cost of production in Rs per tonne for a daily
production of 100 tonnes?
Ans:- Rs. 1300
Solution: Fixed cost = Rs 50000
Variable cost = 800Q
Q = daily production in tonnes
For 100 tonnes of production daily
Total cost = 50,000 + 800Q
= 50,000 + (800 x 100)
= 50,000 +80,000
= 1,30,000
Therefore, the cost of production in Rs per ton = 1,30,000/100 = 1300
Q. 6 – Q. 10 carry two marks each.
Q.6 Find the odd one in the following group: ALRVX, EPVZB, ITZDF, OYEIK
(A) ALRVX (B) EPVZB (C) ITZDF (D) OYEIK
Ans:- (D)
Explanation: Difference between the vertical letters in the series is of 3 letters.
Example – A (B,C,D)E L(M,N,O)P.
In the same way, OYEIK does not fit in the series.
Q.7 Anuj, Bhola, Chandan, Dilip, Eswar and Faisal live on different floors in a six-storeyed building (the
ground floor is numbered 1, the floor above it 2, and so on). Anuj lives on an even-numbered floor.
Bhola does not live on an odd numbered floor. Chandan does not live on any of the floors below
Faisal’s floor. Dilip does not live on floor number 2. Eswar does not live on a floor immediately
above or immediately below Bhola. Faisal lives three floors above Dilip. Which of the following
floor-person combinations is correct?
Anuj Bhola Chandan Dilip Eswar Faisal
(A) 6 2 5 1 3 4
(B) 2 6 5 1 3 4
(C) 4 2 6 3 1 5
(D) 2 4 6 1 3 5
Ans:- (B)
Q.8 The smallest angle of a triangle is equal to two thirds of the smallest angle of a quadrilateral. The
ratio between the angles of the quadrilateral is 3:4:5:6. The largest angle of the triangle is twice its
smallest angle. What is the sum, in degrees, of the second largest angle of the triangle and the largest
angle of the quadrilateral?
322 GATE Architecture and Planning: Comprehensive Question Bank

Ans:- 180˚
Solution: Let the constant in the quadrilateral be x
Ratio among the angles of the quadrilateral be 3x+4x+5x+6x = 360
= 18x = 360
= 20˚
Therefore the angles in the quadrilateral are 60˚, 80˚, 100˚, 120˚ ------(A)
Two-third of smallest angle of quadrilateral is 2/3 x 60˚ = 40˚
Largest angle of the triangle = 2 x 40˚ = 80˚
Therefore the remaining angle of the triangle is 180˚-80˚-40˚ = 60˚
All the three angles of the triangle are 40˚,60˚,80˚ ----------------------(B)
Sum of the second largest angle of the triangle and the largest angle of the quadrilateral
= 60˚ + 120˚ = 180˚
Q.9 One percent of the people of country X are taller than 6 ft. Two percent of the people of country
Y are taller than 6 ft. There are thrice as many people in country X as in country Y. Taking both
countries together, what is the percentage of people taller than 6 ft?
(A) 3.0 (B) 2.5 (C) 1.5 (D) 1.25
Ans:- (D)
Solution: Let’s assume the population of country Y to be 1,000
Population of the country X = 3 x Population of country Y
X = 3Y i.e., X = 3 x 1000 = 3,000
Total population of people above 6ft height in country ‘X’= 1% of 3Y = 1/100 x 3000 = 30
Total population of people above 6ft height in country ‘Y’=2% of Y = 2/100 x 1000 = 20
Total population of people above 6ft height in both countries = 30 + 20 = 50
Percentage of people above 6ft height in both countries = (50/4000 x 100) = 1.25%
Q.10 The monthly rainfall chart based on 50 years of rainfall in Agra is shown in the following figure.
Which of the following are true? (k percentile is the value such that k percent of the data fall below
that value)
(i) On average, it rains more in July than in December
(ii) Every year, the amount of rainfall in August is more than that in January
(iii) July rainfall can be estimated with better confidence
than February rainfall
(iv) In August, there is at least 500 mm of rainfall
(A) (i) and (ii)
(B) (i) and (iii)
(C) (ii) and (iii)
(D) (iii) and (iv)
Ans:- (B)
Hemant Vilas Parulekar 323

Q. 1 – Q. 25 carry one mark each.


Q.1 Toothing is a construction technique used in
(A) Wood construction (B) Steel construction
(C) Reinforced cement concrete construction (D) Brick masonry
Ans:- (D)
Explanation: This technique used to create a smooth transition between two different sections of a wall
or structure. This technique involves alternating courses of full bricks with half bricks at the end of a
wall, creating a stepped or zigzag pattern.
Q.2 ‘Skeleton and Skin’ concept in building design and construction evolved during the
(A) Roman period (B) Renaissance period
(C) Gothic period (D) Brick masonry
Ans:- (C)
Q.3 As per the IRC standards, the minimum width (in m) of a two lane urban carriageway without a
raised kerb is
(A) 6.0 (B) 6.5 (C) 7.0 (D) 8.0
Ans:- (C)
Q.4 Pritzker Architeture Prize 2013 has been awarded to
(A) Mario Botta (B) Toyo Ito (C) Rem Koolhaas (D) Arata Isozaki
Ans:- (B)
Q.5 Hip roof is formed by surfaces sloping in
(A) One direction (B) Two directions (C) Three directions (D) Four directions
Ans:-(D)
Explanation: Please refer image.

Q.6 Hiroshima Peace Memorial Museum in Japan has been designed by


(A) Kenzo Tange (B) Kiso Kurokawa (C) Tadao Ando (D) I M Pei
Ans:- (A)
Explanation: Please refer image.
324 GATE Architecture and Planning: Comprehensive Question Bank

Q.7 In AutoCAD, the maximum number of of points which can be snapped in a circle using OSNAP
command is
(A) 1 (B) 3 (C) 4 (D) 5
Ans:- (D)
Q.8 Development authorities in India are established under the provision of
(A) Municipal Act (B) 74th Constitutional Amendment Act
(C) Town and Country Planning Act (D) Land Acquisition Act
Ans:- (B)
Explanation:
ACT PROVISION UNDER THE ACT
1. Municipal Act This refers to legislation enacted by state governments in India to govern
the establishment, functioning, and administration of municipalities or
municipal corporations within their respective jurisdictions.
2. 74th Constitutional The 74th Constitutional Amendment Act of 1992 in India aimed to
Amendment Act strengthen local self-government institutions and empower urban local
bodies (ULBs).
3. Town and Country Town and Country Planning Act is a type of legislation enacted
Planning Act by governments to regulate the use of land, guide urban and rural
development, and ensure that planning is done in a systematic and
sustainable manner.
4. Land Acquisition Act The Land Acquisition Act, 1894, provided the legal framework for the
acquisition of private land by the government for public purposes, such
as the construction of roads, railways, schools, or other infrastructure
projects.
Q.9 In escalators, the angle of inclination with the horizontal plane should be in the range of
(A) 10°-20° (B) 20°-30° (C) 30°-35° (D) 35°-45°
Ans:- (C)
Q.10 As per the Census of India 2011, ‘Metropolitan Urban Agglomeration’ is a contiguous spread of
several urban settlements where the minimum population size (in Lakh) is
(A) One (B) Five (C) Ten (D) Fifty
Ans:- (C)
Q.11 BEES is an acronym for
(A) Building for Environmental and Economic Sustainability
(B) Built Environment and Ecological Society
(C) Building for Energy and Environment Sustainability
(D) Built Environment and Engineering Services
Ans:- (A)
Explanation: BEES - Building for environmental and economic sustainability involves designing,
constructing, and operating structures in a way that minimizes negative impacts on the environment
while promoting long-term economic viability. This approach considers resource efficiency, renewable
energy use, waste reduction, and the overall life cycle of buildings.
Hemant Vilas Parulekar 325

Q.12 In a single-stack system of plumbing


(A) All the appliances and traps are fully ventilated
(B) Only WC branches are connected with anti-siphonage pipes
(C) Anti-siphonage pipes are omitted
(D) Only the stack is vented above the branch connection at each floor level
Ans:- (C)
Explanation: Please refer image.

Q.13 The maximum bending moment (kNm) in a simply supported beam of 8m span subjected to a
uniformly distributed load of 20 kN/m (inclusive of its self-weight) over the entire span is
(A) 40 (B) 160
(C) 240 (D) 380
Ans:- (B)
Solution: Bending Moment at center = WL²/8 = (20x8²)/8= 160

Q.14 Criteria for background noise (in NC) in hospitals and apartments is
A) 10-20 (B) 20-30
(C) 30-40 (D) 40-50
Ans:- (B)
Explanation: Source – National Building Code, India 2016
Q.15 As per the National Building Code, the minimum width (in m) of a staircase flight in an educational
building above 24 m height should be
(A) 1.0 (B) 1.5
(C) 2.0 (D) 2.5
326 GATE Architecture and Planning: Comprehensive Question Bank

Ans:- (C)
Explanation:
The following minimum width provisions shall be made for each stairway:
a) i) Residential low rise building 0.9 m.
ii) Other residential building e.g. flats, hostels, group housing, guest houses, 1.25 m.
etc
b) Assembly buildings like Auditorium, theatres and cinemas 2.0 m.
c) All other buildings including hotels 1.5 m.
d) Institutional building like hospitals 2.0 m.
e) Educational building like School, Colleges. 1.5 m.
Q.16 Among the following, the one that is NOT a land assembly technique is
(A) Land Use Zoning
(B) Accomodation Reservations
(C) Town Planning Scheme
(D) Transfer of Development Right
Ans:- (A)
Q.17 The Grand Gallery in Egyptian Architecture is provided only at
(A) Great Pyramid (B) Temple (C) Mastaba (D) Bent Pyramid
Ans:- (A)
Explanation:
Please refer image.

Q.18 In the Taipei 101 building, the steel sphere as TMD (Tuned Mass Damper) is suspended to reduce
horizontal sway due to
(A) Settlement & Wind Load
(B) Wind & Geothermal Load
(C) Seismic Geothermal Load
(D) Seismic & Wind Load
Ans:- (D)
Explanation: A Tuned Mass Damper (TMD) is a mechanical device designed to reduce the vibrations or
oscillations of a structure caused by dynamic loads, such as wind, earthquakes, or machinery.
Hemant Vilas Parulekar 327

Q.19 ‘Finger Plan’ concept of urban planning was initially adopted in


(A) Canberra (B) Paris (C) Copenhagen (D) Tokyo
Ans:- (C)
Explanation: Please refer image.

Q.20 The most important property of concrete in its fresh state is


(A) Compressive strength (B) Tensile strength
(C) Elastic modulus (D) Workability
Ans:- (D)
Explanation: Workability refers to the ease with which concrete can be mixed, transported, placed,
and compacted without segregation or bleeding. It directly influences the construction process and the
quality of the final hardened concrete.
To assess workability, various tests such as the slump test, flow table test, and the Vebe test are commonly
conducted. The results of these tests help concrete producers and construction professionals adjust mix
designs to achieve the desired workability for a specific application.
Q.21 An element constructed at intervals along the length of a wall to stabilize it against overturning is
(A) Barrel Vault (B) Pilaster (C) Squinch Arch (D) Buttress
Ans:- (D)
Explanation: Gothic architecture is a style that flourished in Europe during the High and Late Middle
Ages, evolving from Romanesque architecture and succeeded by Renaissance architecture. Originating
in 12th-century France, it is known for its use of height and light, aiming to create an impression of
heaven on earth. Here are some key architectural elements of Gothic architecture
Pointed Arch: Unlike the semi-circular arch of Romanesque buildings, the Gothic arch is pointed at
the top. This element not only contributed to the aesthetic but also allowed for much taller and more
complex structures.
Ribbed Vault: These are intersecting ribbed arches that form a ‘skeleton’ of framework for the ceiling or
a roof. Ribbed vaults are structurally more stable than the barrel vaults of Romanesque style and allowed
for a much more elaborate and decorative design.
Flying Buttress: A specific form of buttressing most strongly associated with Gothic church architecture.
The flying buttresses slant from the upper portion of the wall to a pier, transferring the wall’s outward
force to the ground. This element allowed for higher and lighter walls and more extensive fenestration.
Stained Glass Windows: Large, ornate windows often depicting biblical stories and saints, made with
colored glass, are a hallmark of Gothic architecture. These windows not only added beauty and color to
the building but also allowed more light to enter the interior.
Gargoyles: These are carved stone creatures often serving as water spouts projecting from the roof
gutter to throw rainwater clear of a building. They also served a decorative purpose and were sometimes
used to ward off evil spirits.
328 GATE Architecture and Planning: Comprehensive Question Bank

Tracery: This is a stone framework supporting the glass in a Gothic window. It is characterized by its
ornamental, patterned designs. Tracery windows were a defining feature of the evolving style in Gothic
architecture.
Tall Spires and Towers: These are intended to direct the viewer’s gaze skyward. Spires, which are steeply
pointed roofs, and towers, contribute to the verticality of Gothic cathedrals, a symbolic reach towards
heaven.
High, Ornate Facades: The facades of Gothic buildings, especially cathedrals, were highly decorated
and complex, often featuring a large rose window.
Sculpture: Gothic sculpture was more realistic and human than the Romanesque style, and was
integrated into the architecture, particularly around the entrances or portals
Decorative and Elaborate Surfaces: Gothic buildings often feature detailed and ornate surface
decorations including sculptures, moldings, and painted artworks.
Gothic architecture is often seen as not just a style of building, but an art form in itself, and many of its
elements were used to create a sense of awe and to inspire religious devotion. The most classic examples
are found in religious structures like cathedrals and churches across Europe.
Q.22 Landscape design of Shakti Sthal, the ‘Samadhi’ of the late Prime Minister Smt. Indira Gandhi, was
done by architect
(A) Ram Sharma (B) Mohammad Shabeer
(C) Ravindra Bhan (D) Raj Rewal
Ans:- (C)
Explanation: Ravindra Bhan was known for his contributions to landscape architecture and urban
design in India. He worked on various projects that aimed to integrate architectural elements with the
natural environment.
Q.23 Horizontally Wedge-shaped Treads in stairways are termed as
(A) Stringers (B) Winders (C) Scotia (D) Newel
Ans:- (B)
Q.24 The sequence of development in a Site-and –Services scheme is
(A) Land-Service-House-Occupant (B) Occupant-Land-House-Service
(C) Occupant-Land-Services-House (D) Land-Occupant-House-Services
Ans:- (A)
Explanation: The Site and Services Scheme is a housing development approach that focuses on providing
basic infrastructure and services to a piece of land and allowing residents to build their own houses.
Q.25 Which of the following is NOT a classical spatial theory of land use planning?
(A) Concentric Zone Theory (B) Multiple Nuclei Theory
(C) Centripetal Theory (D) Sector Theory
Ans:- (C)
Hemant Vilas Parulekar 329

Q. 26 – Q. 55 carry two marks each.


Q.26 A housing project is proposed to be designed in a plot of 2 hectare. Maximum permissible FAR is 2.
The share of the numbers of dwelling units (DU) for MIG, LIG and EWS is 1:2:3 having sizes of 55,
35 and 25 sq.m respectively. The maximum number of DU which can be accommodated in the plot
is _______________
Ans:- 1200
Solution: Plot area = 2 hectare × 100 × 100
= 2 × 100 × 100 m²
Buildable area with 2 FAR = 2 × (2 × 100 × 100) m²
Total no of dwelling units = 1x + 2x + 3x = 6x
Area assigned for all MIG = (1x) × (55) m²
Area assigned for all LIG = (2x) × (35) m²
Area assigned for all EWS = (3x) × (25) m²
So, [(1x) × (55)] + [(2x) × (35)] +[(2x) × (35)]
= 2 × (2 × 100 × 100) m²
So, 200x = 40000
So, x = 200
The maximum number of
= 1x + 2x + 3x = 6x = 1200
Q.27 Arrange the following in ascending order of width
P. Collector Street Q. Arterial Road
R. Local Street S. Sub-Arterial Road
(A) P,Q,S,R (B) R,P,S,Q (C) Q,S,R,P (D) Q,S,P,R
Ans:- (B)
Q.28 Out of the following, the maximum points in the LEED (New construction) rating system can be
earned through
(A) Sustainable Sites (B) Water Efficiency
(C) Materials and Resources (D) Energy and Atmosphere
Ans:- (D)
Explanation: This category focuses on promoting energy efficiency, the use of renewable energy, and
other measures to reduce the environmental impact of a building’s energy consumption.
Q.29 Which of the following is NOT a mechanism of bond resistance in reinforced concrete?
(A) Chemical adhesion (B) Friction
(C) Mechanical interlock (D) Aggregate interlock
Ans:- (D)
330 GATE Architecture and Planning: Comprehensive Question Bank

Q.30 A neighbourhood has 250 units of 80 sq m each and 200 units of 100 sq m each. If the mandatory
parking requirement is one per 100 sq m of built space, then total area (sq m) required for parking
considering 30 percent additional area for circulation is _________.
Ans:- 6240-6760
Solution:
Total number of units of 80 sq m = 250
Total area = 80 x 250 = 20,000 sq. m ----------------------------------------------(A)
Total number of units of 100 sq m = 200
Total Area = 100 x 200 = 20,000 sq. m-------------------------------------------------(B)
Adding (A) & (B) we get = 20000 + 20000
= 40000 sq. m
1 Parking requirement for 100 sq. m
Parking requirement for 40000 = 40000/100 = 400 Nos.
Dimensions of a car lot to be assumed = 2.5 m x 5 m
Area of single car lot = 12.5 sq. m
Area required only for car lots = 12.5 x 400 = 5000 sq. m
Area required for parking including 30% additional area of circulation = 5000 x 1.30
= 6500 sq. m
Q.31 A brick wall 19 cm thick has a thermal conductivity 0.811 W/m˚C. The outside and inside surface
conductance of the wall are 16 W/m2 ˚C and W/m2 ˚C respectively, then the U-value of the wall in
W/m2 ˚C is _________.
Ans:- 2.37
Solution: Thermal resistance (R-value) of wall = Thickness/Thermal Conductivity
= 0.19/0.811=0.234.(R1)
Conductance (U-value) = 1/Thermal resistance (R)
Thus, Thermal resistance of outside and inside surface is 1/16 = 0.0625 (R2) & 1/8=0.125 (R3) respectively.
Here, Total conductance of wall (U-value) = 1/(R1+R2+R3)=1/(0.234+0.0625+0.125)=1/0.4215 = 2.37
Q.32 Match the contemporary buildings in Group I with their architects in Group II
(A) P-3,Q-4,R-2,S-5 Group-I Group-II
(B) P-5,Q-4,R-1,S-3 P. Vitra Design Museum, Basel 1. Adrian Smith
(C) P-5,Q-3,R-1,S-2 Q. Turning Torso, Malmö 2. Jean Nouvel
R. Burj Khalif, Dubai 3. Herzog de Meuron
(D) P-5,Q-3,R-2,S-1
S. Tate Modern, London 4. Santiago Calatrava
5. Frank O Gehry
Hemant Vilas Parulekar 331

Ans:- (B)
Explanation:

Vitra Design Museum, Basel - Frank O Gehry Turning Torso, Malmö - Santiago Calatrava

Burj Khalif, Dubai - Adrian Smith Tate Modern, London - Herzog de Meuron

Q.33 From the following cost components of a building construction project which is not a direct cost
combination?
P. Labour cost
Q. Equipment cost
R. Material Cost
S. Establishment cost
T. Supervision cost
(A) P and Q
(B) Q and R
(C) P and R
(D) S and T
Ans:- (D)
Q.34 A house located in Delhi has 111 m2 of flat terrace area (runoff coefficient = 0.85) and 55 m2 of
ground area covered with grass (runoff coefficient = 0.15). If annual average rainfall is 611.8 mm,
then rain water harvesting potential (L/year) from runoff will be_______.
332 GATE Architecture and Planning: Comprehensive Question Bank

Ans:- 62700-62800
Solution:
Area of flat terrace = 111 m2; runoff coefficient for flat terrace = 0.85
Area of ground area covered with grass = 55 m2; runoff coefficient for ground = 0.15
Annual rainfall = 611.8 mm ,i.e., 0.6118 m
Total amount of rain = amount of rain over terrace + amount of rain over ground
= (111 x 0.85 x 0.6118) + (55 x 0.15 x 0.6118)
= 0.6118 [(111 x 0.85) + (55 x 0.15)]
= 0.6118 (94.35 + 8.25)
= 62.77 m3
1 m3 has 1000 litres of water
62.77 m3 would hold 62770 litres of water
Q.35 Match the elements in Group I with the structures in Group II
(A) P-5, Q-1, R-4, S-3 Group-I Group-II
(B) P-1, Q-2, R-4, S-3 P. Harmika 1.Dilwara Temple, Mount Abu
(C) P-3, Q-5, R-4, S-2 Q. Sixteen Vidyadevis 2. Sun temple, Modhera
R. Last Pillar 3. Stupa of Sanchi
(D) P-3, Q-1, R-4, S-2
S. Urushringa 4.Lauriya, Nandangarh
5. Great Kailash temple, Ellora
Ans:- (D)
Explanation:
Harmika - Stupa of Sanchi Sixteen Vidyadevis - Dilwara Temple, Mount Abu

Last Pillar - Lauriya, Nandangarh Urushringa - Sun temple, Modhera


Hemant Vilas Parulekar 333

Q.36 At a site, based on percolation test, the allowable rate of treated sewage application was determined
as 65 L/m2/day. The effective depth (m) of a soak pit with a diameter of 2.5 m of for the disposal of
1020 L/day of septic tank effluent is __________.
Ans:- 1.98 to 2.02
Solution: Rate of treated sewage = 65L/m²/day = 0.065m³
Sewage to be disposed per day = 1020 L/day = 1.020m³/day
Infiltration area = Sewage disposal per day/rate of treated sewage
Thus, Infiltration area = 1.020/0.065 = 15.70m²
Radius of Soak pit = 1.25m. Depth of Soak pit = Infiltration area/2πr=15.70/7.85=2
Q.37 Match the AutoCAD command in Group I with their functions in Group II
(A) P-1, Q-3, R-2, S-5 Group-I Group-II
(B) P-2, Q-4, R-1, S-3 P. OOPS 1. Create solid lines
(C) P-2, Q-1, R-3, S-4 Q. RAY 2. Restores an erased drawing
R. TRACE 3. Manages customized user interface elements
(D) P-1, Q-2, R-3, S-4
S. CUI 4. Creates semi-infinite line
5. Creates a five sided 3d solid with a sloped face
tapering along the X axis

Ans:- (B)
Q.38 Match the bridges in Group I with their structure type in Group II
(A) P-3, Q-4, R-5, S-2 Group-I Group-II
(B) P-5, Q-1, R-4, S-3 P. Harbour Bridge, Sydney 1. Simply Supported
(C) P-2, Q-3, R-4, S-5 Q. Golden Gate Bridge, San Fransisco 2. Cable stayed
R. Howrah Bridge, Kolkata 3. Arch
(D) P-1, Q-2, R-3, S-4
S. Millau Viaduct, Millau 4. Suspension
5. Cantilever
Ans:- (A)
Explanation:

Harbour Bridge, Sydney - Arch Golden Gate Bridge, San Fransisco -


Suspension
334 GATE Architecture and Planning: Comprehensive Question Bank

Howrah Bridge, Kolkata - Cantilever Millau Viaduct, Millau - Cable stayed


Q.39 The arithmetic average value of the sound absorption coefficient for a specific material and
particular mounting condition for four frequencies is
(A) Transmission coefficient
(B) Noise reduction coefficient
(C) Absorption coefficient
(D) Reflection coefficient
Ans:- (B)
Q.40 A single span simply supported reinforced concrete beam (250 mm wide and 480 mm effective
depth) is subjected to a concentrated load of 120kN at its mid-span. Neglecting self-weight of the
beam, the nominal shear stress (MPs) at the support section is _______.
Ans: 0.5
Solution:

Q.41 The optimistic time, the pessimistic time and the most likely time of a job are 6, 13 and 8 days
respectively. The variance for this job is____.
Ans:- 1.36
Solution:
Hemant Vilas Parulekar 335

Q.42 A refuse collection system consisting of two chutes is to be provided in a 20 storied residential
building with 2 flats/floor (average family size = 5) and with each chute serving one flat on each
floor. Average quantity of refuse and its density are 880 b/person/day and 240 kg/m3 respectively. If
the cleaning interval is two days, then the minimum size of the refuse container (litre) at the bottom
of each chutes is____________.
Ans:- 733
Solution:
Total no. of person in the building = 2 × 5 × 20 = 200
No. person serving person chute 200/2 = 100
Average quantity of refuse collected in 2 days in one chute = 2088 kg × 100 = 176 kg So,

Q.43 Match the features in Group I with their architectural periods in Group II
(A) P-1, Q-5, R-4, S-2 Group-I Group-II
(B) P-5, Q-1, R-3, S-2 P. Caryatids 1. Roman
(C) P-3, Q-2, R-5, S-4 Q. Hypocaust 2. Gothic
R. Pylons 3. Greek
(D) P-3, Q-1, R-4, S-2
S. Lofty pinnacles 4. Egyptian
5.Romanasque
Ans:- (D)
Explanation:

Caryatids - They are female figures that


serve as the architectural support for the Hypocaust - an ancient Roman heating system,
entablature of a building. comprising a hollow space under the floor of a
building, into which hot air was directed.
336 GATE Architecture and Planning: Comprehensive Question Bank

Pylons - A pylon is a monumental gate of an Lofty pinnacles - A pinnacle is an architectural


Egyptian temple. element originally forming the cap or crown of a
buttress or small turret.
Q.44 Match the following terminologies in Group I with their descriptions in Group II
Group-I Group-II
P. Pruning 1. Useful in reproducing plants that would not breed true if propagated by seed
Q. Topiary 2. A live bud from a desired plant inserted into a host plant
R. Grafting 3. Cutting of evergreen shrubs into abstract or geometric shapes
S. Budding 4. Trimming and cutting of leaves
5. Selective cutting of plant branches for better growth
(A) P-5, Q-3, R-4, S-2 (B) P-2, Q-3, R-1, S-5
(C) P-3, Q-4, R-1, S-2 (D) P-5, Q-3, R-1, S-2
Ans:- (D)
Explanation:

Pruning Topiary

Grafting Budding
Hemant Vilas Parulekar 337

Q.45 In a dance hall the indoor and outdoor temperatures are 28˚C and 18˚C respectively. There is an
internal heat gain of 5kW and the specific heat of air (on volume basis) is 1300 J/m3 ˚C, then the
necessary cross sectional area (m2) of a duct with an air velocity of 2 m/s required for cooling by
ventilation is ______.
Ans:- 0.19
Solution:
Volume of air passes through duct of cross section area ‘A’ with velocity ‘v’ = A x v
Assume, cross section area of duct = A (m²)
Heat energy in volume ‘V’ of air = volume of air x specific heat x temperature dif.
= 2A x 1300 x 10 = 26000A(Joule) = 26A (KJ)
Internal Heat gain = 5KW = 5KJ/Sec
Hence, 26A = 5
A = 0.19 m²
Q.46 Assuming full compaction, strength, of concrete is inversely proportional to
(A) Water-cement ratio
(B) Water-sand ratio
(C) Water-coarse aggregate ratio
(D) Water-plasticiser ratio
Ans:- (A)
Q.47 Match the terms in Group I with their examples in Group II
(A) P-5,Q-3,R-2,S-1 Group-I Group-II
(B) P-4,Q-5,R-1,S-3 P. Incentive Zoning 1.Broadwalk, Atlantic city
(C) P-3,Q-5,R-4,S-2 Q. Universal Design 2. Minneapolis, USA
R. Promenading 3. Broadway heater District, New York
(D) P-3,Q-5,R-1,S-2
S. Skyway system 4. Pruitt Igoe Housing, St. Louis, Missouri
5.Curutiba, Brazil
Ans:- (D)
Q.48 If yield of stress of steel is 415 MPa, then strain in tensile reinforcement at the limit state of collapse
shall be at least _______. For steel, the Young’s Modulus, E = 2 x 105 MPa.
Ans:- 0.0037
Solution:
For steel, the Young’s Modulus, E = 2 × 105 MPa
Min. tensile strength =

= 0.0018 + 0.002 = 0.0038


338 GATE Architecture and Planning: Comprehensive Question Bank

Q.49 Match the books in Group I with their authors in Group II


(A) P-2, Q-3, R-4, S-1 Group-I Group-II
(B) P-4, Q-5, R-1, S-2 P. Architecture now! 1.Broadwalk, Atlantic city
(C) P-2, Q-3, R-1, S-5 Q. Intentions in Architecture 2. Robert Venturi
R. Design with Nature 3. Christopher Alexander
(D) P-3, Q-1, R-4, S-2W
S.Complexity & Contradiction in 4. Philip Jodidio
Architecture
5. Christian Norberg Schulz
Ans:- (B)
Q.50 Match the common names of the trees in Group I with their botanical names in Group II
(A) P-5, Q-3, R-4, S-2 Group-I Group-II
(B) P-4, Q-5, R-2, S-1 P. Gulmohar 1. Dalbergia Sissoo
(C) P-3, Q-5, R-4, S-2 Q. Palash 2. Ficus Bengalensis
R. Indian Mahogany 3. Delonix Regia
(D) P-3, Q-1, R-5, S-2
S. Banyan 4. Toona Ciliata
5. Butea Monosperma
Ans:- (C)
Q.51 A room of internal dimension 4m x 5m x 3.5m (LxBxH) has 20 cm thick walls and two doors of size
1mx2m. The required area of Damp Proof Course (sq. m) is _____.
Ans:- 3.36
Solution: Required area of DPC
= (5.4 x 4.4) – (5.0 x 4.0) - 2(1.0 x 0.20)
= 3.36 sq.m.
Q.52 A total load of 30 kN is applied vertically downward at the free end of a cantilever of span 5 m. If the
elastic modulus of the cantilever is 30 GPa and the section has a width of 0.3 m and a depth of 0.6 m,
then, the elastic deflection (in mm) is ____________.
Ans:- 7.71
Solution:
Moment of inertia for rectangular section
Hemant Vilas Parulekar 339

Q.53 Associate the plans in Group I with the options in Group II.
(A) P-2, Q-3, R-1, S-4 Group-I Group-II
(B) P-2, Q-1, R-5, S-4 P. City Development Plan 1. PMGSY
(C) P-1, Q-3, R-2, S-5 Q. Slum Free City Plan 2. JNNURM
R. Transport Network Plan 3. RAY
(D) P-3, Q-2, R-1, S-4
S. Disaster Management Plan 4. NDMA
5. RSVY
Ans:- (A)
Explanation:
Group-I Group-II Description
P. City Development Plan 1. JNNURM Jawaharlal Nehru National Urban Renewal Mission
Q. Slum Free City Plan 2. RAY Rajiv Awas Yojana
R. Transport Network Plan 3. PMGSY Pradhan Mantri Gram Sadak Yojana
S. Disaster Management Plan 4. NDMA The National Disaster Management Authority
Q.54 The capacity of a hall is 600 persons and its volume is 3000 cu.m. If an optimum reverberation time
of 1.0 second is to be achieved then the required total absorption (m² sabine) is _____________.
Ans:- 480
Solution:
Volume (V) = 300 m³, R.T. =1.0 sec absorption area (m² sabine), A=?
R.T. = (0.16 V/A)
A= (0.16/R.T.)
Hence, A= 480 m² sabine
Q.55 A solid straight steel rod of diameter 100 mm is bent in single curvature into a circular arc by a
moment of 50 kNm applied at its ends. If elastic modulus, E, for steel is 2 x 105 MPa, the radius of
curvature (mm) of the arc assuming π = 3.14 is _____________
Ans:- 19615 (As per official GATE Answer Key)
Solution:
GATE QUESTION PAPER 2013
General Aptitude (GA)

1 – Q. 25 carry one mark each.


Q.1 In case of residential apartments, the effective floor area available for use within an apartment, is
known as
(A) Carpet Area
(B) Built-up Area
(C) Plinth Area
(D) Super Built-up Area
Ans:- (A)
Explanation: Here’s a brief explanation of each of the terms:
Carpet Area: This is the actual usable area within the walls of an apartment. It includes the area where
you can place your carpet or any other flooring material. Carpet area excludes common areas like the
thickness of the walls and common passages.
Built-up Area: The built-up area includes the carpet area plus the area occupied by the walls and other
structural elements. It represents the total area of your apartment, including the space taken up by walls.
Plinth Area: Plinth area is the covered built-up area measured at the floor level of any storey, including
the thickness of the external walls and any projections or balconies.
Super Built-up Area: Also known as the saleable area, it includes the built-up area plus a proportionate
area of common spaces like the lobby, staircases, and amenities in the apartment complex. Super built-
up area is typically used by builders to calculate the cost of the apartment.
Q.2 Star Rating of an Air Conditioner is determined by its
(A) Power Consumption
(B) Energy Efficiency Ratio
(C) Cooling Capacity
(D) Power of Compressor
Ans:- (B)
Explanation: EER is a measure of how efficiently an air conditioner converts electrical energy into cooling
output. A higher EER indicates greater energy efficiency, which leads to lower power consumption and,
therefore, lower operating costs. In many countries, air conditioners are assigned star ratings (usually
from 1 star to 5 stars or more) based on their EER, with a higher star rating indicating better energy
efficiency.
Q.3 V7 concept given by Le Corbusier refers to
(A) Neighbourhood Planning (B) Housing Typologies
(C) Architecture Design Principle (D) Hierarchy of Roads
Hemant Vilas Parulekar 341

Ans:- (D)
Explanation: The roads of Chandigarh have been planned based on the V7 Concept developed by Le
Corbusier and his team of architects and civil engineers
V1- Arterial roads (linking Chandigarh with other cities)
V2- Major Boulevards (where significant institutional and commercial buildings are situated)
Sector Definers in Version 3
V4- Shopping Streets traverse the sectors, with the shops located predominantly on their southern sides.
They are positioned with consideration for their solar orientation.
Designated roadways for vehicular circulation inside sectors.
V6- Access Road (Facilitating entry to residential properties)
V7- Pedestrian Pathways
Subsequently, a V8 engine is incorporated into the road planning of Chandigarh.
V8- Greenway (incorporating a network of cycle tracks across natural areas)
The sector is partitioned into four quadrants by a V-4 road that spans from east to west and a V-5 road
that spans from north to south. The four portions can be readily identified as A, B, C, and D, which
correspond to the North, East, South, and West sides, respectively. Every Sector is designed to be self-
sustaining, with conveniently located commerce and community amenities that may be easily reached
on foot.

Q.4 In AUTOCAD, a line of infinite length in the direction defined by starting point and through point,
is known as
(A) RAY (B) LINE (C) PLINE (D) XLINE
Ans:- (A)
Explanation:
A) RAY: In AUTOCAD, a ray is similar to a line but extends infinitely in one direction from its starting
point.
B) LINE: A line in AUTOCAD is a straight finite segment connecting two points.
C) PLINE (Polyline): A polyline is a connected series of line segments, circles, arcs, or other shapes,
allowing you to create complex objects.
D) XLINE (Construction Line): An XLINE, or construction line, is a line that extends infinitely in one
direction from its starting point, commonly used as a construction aid for drawing.
342 GATE Architecture and Planning: Comprehensive Question Bank

Q.5 Orbit Tower built at the London Olympic Park has been designed by
(A) Foster & Partners
(B) Anish Kapoor & Cecil Balmond
(C) Zaha Hadid & Antony Gormley
(D) Richard Rogers & Renzo Piano
Ans:- (B)
Explanation: The ArcelorMittal Orbit is a distinctive landmark in
London, UK. It was designed by the artist Anish Kapoor and
structural engineer Cecil Balmond. This structure was created for the
2012 London Olympic and Paralympic Games.
Key features of the ArcelorMittal Orbit:
Design: The ArcelorMittal Orbit is known for its unique and
contemporary design. It’s often described as an abstract and dynamic
sculpture. It stands out due to its striking red color and complex,
twisting lattice structure.
Height: The tower stands approximately 114.5 meters (376 feet) tall, making it the UK’s tallest sculpture.
Visitors can take an elevator to the top for panoramic views of London.
Materials: The structure’s name includes “ArcelorMittal” because it was sponsored and partly constructed
by the multinational steel and mining company, ArcelorMittal. The tower is primarily constructed from
60% recycled steel.
Legacy: The ArcelorMittal Orbit was initially built as a temporary structure for the Olympics but was
later repurposed as a permanent attraction. It serves as an observation tower, offering views of the
Olympic Park, London Stadium, and the city beyond.
Art Installation: In addition to serving as an observation tower, the ArcelorMittal Orbit also includes an
art installation in its interior, created by Anish Kapoor. The installation includes mirrors and reflective
surfaces, adding an artistic dimension to the visitor experience.
Helter-Skelter Slide: To further enhance its appeal, a slide was added to the structure in 2016. Visitors
can descend in a helter-skelter fashion, which has become a popular attraction.
The ArcelorMittal Orbit is an iconic addition to London’s skyline and a symbol of the regeneration of
the East London area following the 2012 Olympics.
Q.6 As per National Building Code 2005, the minimum size of a habitable room in m2 is
(A) 9.5 (B) 10.5 (C) 11.5 (D) 12.5
Ans:- (A)
Explanation: As per NBC 2005, the area of habitable room shall not be less than 9.5 m2, where there is
only one room with a minimum width of 2.4 m. Where there are two rooms, one of these shall not be
less than 9.5 m2, other not less than 7.5 m2, with a minimum width of 2.1 m.
Q.7 The urban form of Srirangam town in Tamil Nadu refers to
(A) Dandaka (B) Swastika (C) Nandyavarta (D) Sarvotabhadra
Hemant Vilas Parulekar 343

Ans:- (D)
A staff- or rod-shaped urban design is called Dandaka in
Sanskrit. Ancient Indian urban planning used different shapes
and designs to convey ideas and promote traits.
Linear and methodical, the Dandaka town plan symbolized
stability and organization. This plan was used to build military
fortifications and other projects that required a strong and
structured layout. The Dandaka layout was also supposed to
boost positive energy and structure functionality.
The settlement of Dandaka resembles a staff. The streets in this neighborhood are exactly lined and
meet at right angles. From west to east and south to north.
Small municipalities and rural villages use this urban plan with two main entry points.
The settlement is rectangle or square with 1–5-danda street width.
Village office is east.
Gramadevatas, female goddesses, are normally beyond the hamlet, while male deities are in the north.

Sarvatobadra is a Sanskrit word for an umbrella-shaped city.


The umbrella shape in ancient Indian urban planning
symbolized protection and refuge. The circular town pattern
and radiating streets gave the Sarvatobadra area a sense of
harmony and balance.
Temples and ashrams, which provided spiritual refuge, were often built
using this architectural style. The Sarvatobadra plan’s circular shape
promotes positive energy and optimizes structure functionality. It is ideal
for larger villages and cities on rectangular or square plots of land. The town should be filled with
diverse houses inhabited by individuals from different social classes. The temple is the village’s most
prominent feature.

A circular town layout with a center square and


radiating streets is called nandyavarta in Sanskrit.
Ancient India often designed cities and big
communities with this town model. The Nandyavarta
town plan’s circular shape emphasized unity and
wholeness, and the central square was a community
hub.
Radiating roadways made it easier to get around the city and connected neighborhoods. Positive
energy and building functioning were also supposed to be promoted by the Nandyavarta town layout.
• Primarily utilized for town building, not villages.
Adopted for circular or square areas with 3000-4000 homes.
• The town center has a temple dedicated to the presiding deity, located parallel to the primary streets.
The name comes from the flower whose form is reflected in this arrangement.
344 GATE Architecture and Planning: Comprehensive Question Bank

Padmaka is a Sanskrit word for a lotus-shaped


settlement. Lotus flowers symbolized purity,
enlightenment, and beauty in ancient Indian
urban design. The circular Padmaka town layout
had petals-like streets.
Temples and ashrams, which provided spiritual
refuge, were often built using this architectural style.
The Padmaka plan’s circular shape was intended
to promote positive energy and optimize structure
functionality. Roadways’ petal-like pattern added to
the built environment’s aesthetics.
This proposal involves fortifying towns.
• The PLAN resembles lotus petals radiating from its core. The city was once on a watery island.
Growth is impossible.

The Hindu swastika is used frequently in Hindu


architecture and urban design. From Sanskrit
“svastika,” “swastika” means “auspiciousness” or
“prosperity.” The sign has four 90-degree arms in a
cross shape. The arms are generally the same length
and might face clockwise or counterclockwise.
Hindus associate the swastika with auspiciousness
and plenty. It is used as an adornment in Hindu
temples, homes, and other buildings to bring good
luck and ward off evil. The swastika was used in
ancient Indian urban planning to mark the entrance
to a building or village and symbolize the Hindu
cycle of life.
• This arrangement has diagonal roadways dividing
the property into triangular portions. The site may
have any form.
The village has a fortified wall and moat, with two
main thoroughfares intersecting in the center, extending north-south and west-east.
The temple is central. The southwest cell houses the Jain temple.
Prastara is the Sanskrit word for organizing and
placing tiles or stones for floors or pavements.
Ancient Indian architecture and urban planning
employed the prastara to create ornate and attractive
architectural designs.
This design allows square or rectangular sites but not
triangular or circular ones.
• Sites are segregated by wealth, catering to affluent,
wealthy, middle-class, and impoverished individuals.
• Site size increases with purchasing power or ability to build.
• Main roads are wider than other highways.
• Town fortification is uncertain.
Karmukha, the bow-shaped urban plan is ideal for towns
on beaches or riverbanks when the town site is bow-
shaped, semi-circular, or parabolic.
Main streets go north-south or east-west, whereas
connecting streets run perpendicularly. The whole area is
divided. The temple for the main deity, the female goddess,
is well-placed.

A building with four faces or entrances is called chaturmukha in


Sanskrit. Ancient Indian town planning and architecture employed
chaturmukha buildings as temples, gateways, and other sacred or
secular structures.
Chaturmukha had four faces, each representing a cardinal direction.
These faces protected the building and allowed all-around access.
Four gateways made people and goods flow smoothly, improving
connectivity and access.
• This applies to all communities, from large cities to little villages.
• The site is square or rectangular with four sides. • The town runs east-west with four main streets. •
The chief god’s temple is always in the middle.
Q.8 PMGSY, a programme of Government of India, deals with
(A) Urban Employment Generation (B) Rural Employment Generation
(C) Rural Electrification (D) Rural Road Development
Ans:- (D)
Explanation: PMGSY stands for Pradhan Mantri Gram Sadak Yojana, which is a program of the
Government of India dealing with Rural Road Development.
Q.9 Beam or lowest division of the entablature which extends from column to column, is known as
(A) Arabesque (B) Arcade (C) Architrave (D) Arbour
Ans:- (C)

Elevations of Doric and Ionic orders


346 GATE Architecture and Planning: Comprehensive Question Bank

Q.10 The information that is NOT essential to be submitted for sanction of any building plan is
(A) Site Plan (B) Floor Plans (C) Title Deed (D) Land Cost
Ans:- (D)
Q.11 The tendency of an ecosystem to maintain its balance by regulatory mechanisms when disrupted,
is known as
(A) Homeostasis (B) Entropy (C) Succession (D) Evolution
Ans:- (A)
Q.12 Gantt Chart DOES NOT provide information about
(A) List of Jobs (B) Duration of Jobs
(C) Interdependency of Jobs (D) Progress of Work
Ans:- (C)

Q.13 If threshold of hearing has a sound level of zero decibels and the sound level in a broadcasting
studio is 100 times the threshold of hearing, its value in decibels would be
(A) 0 (B) 10 (C) 20 (D) 100
Ans:- (C)
Explanation: The decibel scale is a logarithmic scale, and it’s based on a logarithm to the base 10.
If the sound level in a broadcasting studio is 100 times the threshold of hearing, you can calculate it
using the formula:
LdB=10 x log10 x (I / I0)
Where:
-LdB is the sound level in decibels,
-I is the intensity of the sound in question, and
-I0 is the reference intensity (threshold of hearing in this case).
So, for this calculation:
-I is 100 times the threshold of hearing, i.e., 100�I0.
Now, plug these values into the formula:
LdB=10 x log10 x [(100 x I0/ I0)]
Simplify:
LdB=10 x log10 x (100)
Since log10(100) = 2, the result is:
LdB=10 x 2=20
So, the sound level in the broadcasting studio would be 20 decibels.
Hemant Vilas Parulekar 347

Q.14 The width to height ratio of the front facade of Parthenon (without the pediment) is
(A) 9:4 (B) 4:9 (C) 1:1.618 (D) 1.618:1
Ans:- (A)

AB/BC = BD/AB = AD/BD = AE/AD


Golden section, AB/BC = AE/AD = 0.618
(Source- Form, Space and Order-Francis D.K. Ching
Q.15 The face of an Icosahedron is
(A) Equilateral Triangle (B) Isosceles Triangle (C) Square (D) Pentagon
Ans:- (A)

Q.16 The term ‘Zeitgeist’, used in contemporary architecture, refers to


(A) Iconicity (B) Spirit of Times
(C) Kinesthetics (D) Semantic Associations
Ans:- (B)
Explanation: The global, grassroots Zeitgeist Movement promotes a more sustainable and equal society.
Key elements of the movement:
Background and Philosophy: Peter Joseph’s 2007 film “Zeitgeist: The Movie” and its sequels inspired
the movement. Films, public interactions, and social media boosted it. The movement’s theory holds
that modern society’s economic, social, and political systems must be altered to address global issues.
The Zeitgeist Movement promotes a resource-based economy. This economic model emphasizes
resource management and sustainability over the market-based economy driven by money. It promotes
efficient and fair resource allocation through technology and science for social decision-making.
348 GATE Architecture and Planning: Comprehensive Question Bank

Current Systems Critique: The movement criticizes capitalism for fostering inequality, environmental
degradation, and unsustainable consumption. It claims these systems are inefficient and cause social
issues.
Technological and Social Advancement: The Zeitgeist Movement believes technology can address
many social issues. It promotes innovative technology and science to raise global living standards and
better manage resources.
Global and Local Initiatives: The movement has chapters and supporters worldwide. It promotes its
ideals through education, events, and media.
Criticism and Controversy: The movement has been criticized for unrealistic idealism, oversimplification
of complicated economic and social issues, and solution practicality. Its connection to the conspiracy
ideas in the original “Zeitgeist” film has also been disputed.
Importantly, the movement promotes peaceful, non-violent change. It tries to transform consciousness
and ideals via knowledge and advocacy, not politics or force.
The Zeitgeist Movement rethinks society, emphasizing sustainability, technology, and alternative
economic models.
Q.17 Alhambra, a UNESCO world heritage site, is classified as an example of
(A) Moorish Architecture (B) Mudejar Architecture
(C) Mozarabic Architecture (D) Tudor Architecture
Ans:- (A)

The Alhambra is a renowned example of Moorish architecture, which was the architectural style of
the Moors during the Islamic rule in the Iberian Peninsula, including parts of Spain. This style is
characterized by its intricate tile work, arches, geometric patterns, and lush gardens, all of which are
prominent features of the Alhambra. While the Alhambra also exhibits elements of Mudéjar and other
styles due to the cultural syncretism of the period, it is primarily recognized as a masterpiece of Moorish
architecture.
Q.18 Wythenshawe and Becontree are examples of
(A) Factory Town (B) Satellite Town
(C) Garden City (D)VerticaL Neighborhood
Ans:- (C)
Hemant Vilas Parulekar 349

Q.19 National Ceremonial Plaza at Thimpu in Bhutan has been designed by


(A) Christopher Charles Benninger (B) Charles Correa
(C) Karan Grover (D) I. M. Pei
Ans:- (A)

Q.20 Physiochemical process of removing micro-organisms, colour and turbidity from sullage and
sewage is known as
(A) Putrefaction (B) Clarification (C) Liquefaction (D) Infiltration
Ans:- Marks to all
Q.21 Identify which is NOT a green building rating system
(A) LEED (B) CASBEE (C) ENERGY BUILD (D) BREEAM
Ans:- (A)

Rating System Full Form Country


LEED Leadership in Energy and Environmental Design USA
Building Research Establishment Environmental
BREEAM Assessment Method UK
Deutsche Gesellschaft für Nachhaltiges Bauen (German
DGNB Sustainable Building Council) Germany
Green Star - Australia
Comprehensive Assessment System for Built Environment
CASBEE Efficiency Japan
Haute Qualité Environnementale (High Quality
HQE Environmental standard) France
Green Mark - Singapore
Pearl
ESTIDAMA - United Arab Emirates
GRIHA Green Rating for Integrated Habitat Assessment India
BEAM Plus Building Environmental Assessment Method Plus Hong Kong
NABERS National Australian Built Environment Rating System Australia
350 GATE Architecture and Planning: Comprehensive Question Bank

Q.22 In 3DS Max, smooth 3D surfaces, by blending a series of selected shape curves, can be created by
(A) Lofting (B) Sweeping (C) Filleting (D) Extruding
Ans:- (A)
Q.23 Travel behavior characteristics of an urban area can be derived from
(A) Parking Survey (B) Demographic Survey
(C) Socio Economic Survey (D) Origin & Destination Survey
Ans:- (D)
Q.24 In GIS, the set of entities representing vector data type is
(A) Point, Line, Polygon, TIN (B) Pixel, Voxel
(C) DEM, DSM, DTM (D) Coordinates, Elevation, Slope
Ans:- (C)
Q.25 A common flowering shrub is
(A) Tectona grandis (B) Mimusops elengi
(C) Dalbergia sissoo (D) Ixora coccinea
Ans:- (D)
Tree-Common Name Teak Spanish Cherry/Bakul/Maulasari
Botanical name Tectona grandis Mimusops Elengi
Tree/Form of Branch

Tree-Common Name Shisam Jungle Geranium/Jungle Flame


Botanical name Dalbergia Sissoo Ixora Coccinea
Tree/Form of Branch
Hemant Vilas Parulekar 351

Q.26 - Q.55 carry two marks each.


Q.26 The correct arrangement of the height of towers given below in descending order is
P. Burj Khalifa, Dubai Q. Petronas Tower, Kuala Lumpur
R. Taipei 101, Taiwan S. Bank of China Tower, Hong Kong
(A) P, Q, R, S (B) P, Q, S, R (C) P, R, S, Q (D) P, R, Q, S
Ans:- (D)
Q.27 Match the buildings in Group I with their corresponding architects in Group II
(A) P-2,Q-5,R-1,S-4 Group I Group II
(B) P-5,Q-3,R-2,S-1 P. Khalsa Heritage Complex, Anandpur Sahib 1. Philip Johnson
(C) P-4,Q-2,R-1,S-3 Q. Lisbon Ismaili Centre, Lisbon 2. Charles Correa
R. Neuroscience Centre, Cambridge, USA 3. Raj Rewal
(D) P-5,Q-2,R-1,S-4
S. National Centre for Performing Arts, Mumbai 4. B. V. Doshi
5. Moshe Safdie

Ans:- (D)
Q.28 The term ‘Working head’ in context of water supply system means
(A) Height of a body of water falling freely under the force of gravity to acquire a certain velocity
(B) Rate of increase of velocity with respect to distance normal to the direction of flow
(C) Total head with deduction for velocity head or losses
(D) Difference between supply and delivery water levels
Ans:- (D)
Q.29 In a theoretical traffic flow relationship, as shown in the figure given
below,the slope of line OF joining point F on the curve and the origin
O represents
(A) Corresponding space mean speed
(B) Speed at maximum flow
(C) Travel time at corresponding density
(D) Average headway at corresponding flow
Ans:- (A)
Q.30 Match the CAD terms in Group I with their corresponding functions in Group II
(A) P-4,Q-3,R-2,S-1 Group I Group II
(B) P-2,Q-5,R-2,S-1 P. Tiled viewport 1. Boolean operator
(C) P-5,Q-3,R-4,S-2 Q. UCS 2. Solid model
R. DXF 3. Coordinate system
(D) P-3,Q-5,R-4,S-2
S. Extrude 4. Drawing interchange format
5. Model space

Ans:- (C)
352 GATE Architecture and Planning: Comprehensive Question Bank

Q.31 Match the historic periods in Group I with their corresponding examples of towns in Group II.
(A) P-3,Q-1,R-2,S-4 Group I Group II
(B) P-3,Q-1,R-4,S-5 P. Egyptian 1. Miletus
(C) P-4,Q-1,R-5,S-2 Q. Greek 2. Montpazier
R. Medieval 3. Kahun
(D) P-5,Q-1,R-3,S-2
S. Renaissance 4. Versailles
5. Timgad
Ans:- (A)
Q.32 Match the components of an Indian urban land use map in Group I with their corresponding colour
codes as per UDPFI guidelines in Group II
(A) P-1,Q-3,R-2,S-5 Group I Group II
(B) P-2,Q-1,R-3,S-4 P. Public / Semipublic 1. Violet
(C) P-3,Q-4,R-5,S-2 Q. Industry 2. Grey
R. Transportation 3. Red
(D) P-3,Q-1,R-2,S-4
S. Commercial 4. Blue
5. Yellow
Ans:- (D) (Please refer GATE-2015, Q.22, for more information)
Q.33 Match the books in Group I with their corresponding authors in Group II
(A) P-3,Q-4,R-1,S-5 Group I Group II
(B) P-3,Q-1,R-2,S-5 P. Design of Cities 1. Amos Rappaport
(C) P-4,Q-3,R-5,S-2 Q. On the Cultural Origin of 2. Leo Jacobson and Ved
Settlements Prakash
(D) P-3,Q-4,R-1,S-2
R. Urbanization and National 3. Edmond Bacon
Development
S. Planning Theory 4. Christopher Alexander
5. Andreas Faludi
Ans:- (B)
Q.34 Match the temples in Group I with their corresponding historical periods in Group II
(A) P-2,Q-3,R-5,S-1 Group I Group II
(B) P-5,Q-1,R-4,S-3 P. Vaikuntha Perumal Temple, Kancheepuram 1. Vijaynagara
(C) P-3,Q-5,R-2,S-1 Q. Meenakshi Temple, Madurai 2. Chalukya
R. Durga Temple, Aihole 3. Chola
(D) P-5,Q-4,R-2,S-3
S. Brihadeshwara Temple, Thanjavur 4. Pandya
5. Pallava
Ans:- (D)
Hemant Vilas Parulekar 353

Q.35 Match the theories in Group I with their corresponding propagators in Group II
(A) P-2,Q-3,R-5,S-1 Group I Group II
(B) P-1,Q-2,R-4,S-5 P. Choice theory of planning 1. Paul Davidoff and T.A. Reiner
(C) P-4,Q-3,R-5,S-2 Q. Connurbation 2. Patrick Geddes
R. Classical theory of land use 3. Homer Hoyt
(D) P-5,Q-4,R-3,S-2
S. Central place theory 4. Richard L. Meier
5. Walter Christaller
Ans:- (D)
Q.36 Match the buildings in Group I with their corresponding structural feature in Group II
(A)P-4,Q-3,R-2,S-1 Group I Group II
(B) P-2,Q-1,R-3,S-4 P. Yokohama Port Terminal, 1. Geodesic Dome
(C) P-4,Q-3,R-5,S-2 Yokohama
Q. Stanstead Airport, London 2. Shell Structure
(D) P-5,Q-3,R-4,S-2
R. TWA Terminal, New York 3. Space Frame
S. Montreal Biosphere, Montreal 4. Folded Steel Plate Structure

5. Pneumatic Structure
Ans:- (A)
Q.37 Match the buildings in Group I with their corresponding structural feature in Group II
(A) P-5,Q-2,R-4,S-3 Group I Group II
(B) P-2,Q-1,R-4,S-3 P. First Five Year Plan 1. Formation of HUDCO
(C) P-4,Q-1,R-2,S-5 Q. Fourth Five Year Plan 2. Establishment of TCPO
R. Seventh Five Year Plan 3. Introduction of JNNURM
(D) P-1,Q-2,R-3,S-5
S. Tenth Five Year Plan 4. Announcement of National
Housing Policy
5. Passing of Urban Land Ceiling
and Regulation Act
Ans:- (B)
Q.38 Match the landscape designers listed under Group I with their appropriate contribution from
Group II
(A) P-3,Q-1,R-4,S-2 Group I Group II
(B) P-5,Q-3,R-4,S-2 P. Lancelot ‘Capability’ Brown 1. The Well-tempered Garden
(C) P-3,Q-1,R-2,S-5 Q. Andre Le Notre 2. Kew Garden
R. Joseph Paxton 3. Versailles Garden
(D) P-2,Q-3,R-4,S-5
S. Frederick Law Olmstead 4. Crystal Palace
5. Central Park
354 GATE Architecture and Planning: Comprehensive Question Bank

Ans:- (D)
Name of Designer Lancelot ‘Capability’ Brown Andre Le Notre
Landscape design Kew Garden Ersailles Garden
Illustration

Name of Designer Joseph Paxton Frederick Law Olmstead


Landscape design Crystal Palace Central Park, New York
Illustration

Q.39 Match the organism type from Group I with the appropriate example from Group II
(A) P-5,Q-4,R-1,S-2 Group I Group II
(B) P-2,Q-1,R-5,S-4 P. Autotroph 1. Hasmukh C. Patel
(C) P-1,Q-2,R-4,S-5 Q. Heterotroph 2. Charles Correa
R. Chemotroph 3. Hafeez Contractor
(D) P-3,Q-2,R-1,S-5
S. Saprophyte 4. Karan Grover
5. Balkrishna V. Doshi
Ans:- (D)
Q.40 Match the concepts in Group I with their corresponding authors in Group II
(A) P-2,Q-1,R-5,S-3 Group I Group II
(B) P-2,Q-1,R-3,S-4 P. Proxemics Theory 1. Gordon Cullen
(C) P-4,Q-1,R-5,S-2 Q. Serial Vision 2. Edward T. Hall
R. Urban Imageability 3. Hafeez Contractor
(D) P-3,Q-5,R-2,S-1
S. Defensible Space 4. Paul Zucker
5. Kevin Lynch
Hemant Vilas Parulekar 355

Ans:- (A)
Explanation: Refer GATE-1992, Q.8 (Proxemics Theory), GATE-2009, Q.26 (Urban Imageability),
GATE-2010 (Serial Vision)
Q.41 If the area coverage of one sprinkler is 20 m2, with a maximum and minimum spacing of 4.6 m
and 1.8 m respectively, the minimum number of sprinklers required to be arranged in a regular
orthogonal grid to cover the area of a 15 m x 20 m room would be _______.
Ans:- 20
Solution: To find the minimum number of sprinklers required to cover a 15 m x 20 m room with an
area coverage of 20 m² per sprinkler, we can calculate the total area of the room and then divide it by the
area covered by each sprinkler.
Room dimensions: 15 m x 20 m Total room area = 15 m * 20 m = 300 m²
Area coverage per sprinkler = 20 m²
Now, let’s calculate the maximum and minimum spacing between sprinklers:
- Maximum spacing = 4.6 m
- Minimum spacing = 1.8 m
To cover the room most efficiently, we’ll arrange the sprinklers in a grid pattern.
Now, let’s calculate how many sprinklers are needed for the maximum and minimum spacing:
1. Maximum spacing:
- In the length (20 m), the maximum spacing allows for sprinklers at 4.6 m intervals. So, we need (20 m
/ 4.6 m) sprinklers along the length.
- In the width (15 m), the maximum spacing allows for sprinklers at 4.6 m intervals. So, we need (15 m
/ 4.6 m) sprinklers along the width.
2. Minimum spacing:
- In the length (20 m), the minimum spacing allows for sprinklers at 1.8 m intervals. So, we need (20 m
/ 1.8 m) sprinklers along the length.
- In the width (15 m), the minimum spacing allows for sprinklers at 1.8 m intervals. So, we need (15 m
/ 1.8 m) sprinklers along the width.
Now, calculate the total number of sprinklers for each case:
1. Maximum spacing:
- Along the length: 20 m / 4.6 m ≈ 4.35 sprinklers (round up to 5 sprinklers)
- Along the width: 15 m / 4.6 m ≈ 3.26 sprinklers (round up to 4 sprinklers)
Total for maximum spacing = 5 sprinklers * 4 sprinklers = 20 sprinklers
2. Minimum spacing:
- Along the length: 20 m / 1.8 m ≈ 11.11 sprinklers (round up to 12 sprinklers)
- Along the width: 15 m / 1.8 m ≈ 8.33 sprinklers (round up to 9 sprinklers)
- Total for minimum spacing = 12 sprinklers * 9 sprinklers = 108 sprinklers
356 GATE Architecture and Planning: Comprehensive Question Bank

So, to efficiently cover the area of a 15 m x 20 m room with an area coverage of 20 m² per sprinkler:
- Minimum number of sprinklers required with maximum spacing = 20 sprinklers.
- Minimum number of sprinklers required with minimum spacing = 108 sprinklers.
The minimum number of sprinklers required is 20 when considering maximum spacing.
Q.42 If the slope of a hipped roof is 60 degrees and height of the roof is 3 m, span of the room, in m, would
be _______.
Ans:- 3.4 to 3.6
Solution: we have:
tan(60 degrees) = height of roof / (1/2) * span of the room
Now, plug in the values:
tan(60 degrees) = 3 meters / (1/2) * span
To isolate the span, we can rearrange the equation:
span = (3 meters / tan(60 degrees)) * 2
Now, calculate:
span = (3 / √3) * 2
span = (3 * 2) / √3
span = 6 / √3
To rationalize the denominator, multiply both the numerator and denominator by √3:
span = (6 / √3) * (√3 / √3)
span = (6√3) / 3
span = 2√3 meters = 2 x 1.732 = 3.46
So, the span of the room with a hipped roof having a slope of 60 degrees and a roof height of 3 meters is
approximately 3.46 meters.
Q.43 Volume of coarse aggregate in m3 present in 1.0 m3 of 1 : 1.5 : 3 concrete mix made by volume
batching is _______.
Ans:- 0.5 to 0.6
Solution: In a concrete mix with the proportions 1 : 1.5 : 3 (cement : sand : aggregate) made by volume
batching, we can calculate the volume of coarse aggregate present in 1.0 m3 of concrete.
The total volume of the mix is 1.0 m3.
The sum of the proportions is 1 + 1.5 + 3 = 5.5.
Now, to find the volume of coarse aggregate, we can use the proportion of coarse aggregate:
Volume of coarse aggregate = (Proportion of coarse aggregate / Sum of proportions) * Total volume
Volume of coarse aggregate = (3 / 5.5) * 1.0 m3
Volume of coarse aggregate ≈ 0.5455 m3
So, the volume of coarse aggregate in 1.0 m3 of 1 : 1.5 : 3 concrete mix made by volume batching is
approximately 0.5455 cubic meters.
Hemant Vilas Parulekar 357

Q.44 A tank of internal dimension 3 m x 5 m x 4 m (Length x Breadth x Height) has 200 mm thick brick
wall on all sides. Volume of brickwork in m3 would be _______.
Ans:-13 to 14
Solution: To find the volume of the brickwork for a tank with internal dimensions of 3 m x 5 m x 4 m
and 200 mm thick brick walls on all sides, you can calculate it as follows:
Calculate the external dimensions of the tank, including the brick walls:
Length (external) = Length (internal) + 2 * Thickness = 3 m + 2 * 0.2 m = 3.4 m
Breadth (external) = Breadth (internal) + 2 * Thickness = 5 m + 2 * 0.2 m = 5.4 m
Height (external) = Height (internal) = 4 m = 4.20 m
Calculate the volume of the tank with the brick walls:
Volume (with brick walls) = Length (external) x Breadth (external) x Height (external)
Volume (with brick walls) = 3.4 m x 5.4 m x 4.0 m = 73.44 m³
Calculate the volume of the internal tank:
Volume (internal) = Length (internal) x Breadth (internal) x Height (internal)
Volume (internal) = 3 m x 5 m x 4 m = 60 m³
Calculate the volume of the brickwork:
Volume of Brickwork = Volume (with brick walls) - Volume (internal)
Volume of Brickwork = 73.44 m³ - 60 m³ = 73.44 m³
So, the volume of brickwork in the tank is approximately 73.44 cubic meters.
Q.45 Flux emitted from a 1cd light source in all directions, in lumens, would be _______.
Ans:- 12 to 13
Solution: The total luminous flux emitted from a 1 candela (cd) light source in all directions is equal to
4π lumens.
So, the flux emitted from a 1cd light source in all directions is approximately:
4π lumens ≈ 12.57 lumens
Q.46 50 Hectare of residential sector has 65% buildable area. The FAR of the buildable area is 1.5. Within
the residential sector, 60% of dwelling units are of area 100 m2 each and 40% of the dwelling units are
of area 80 m2 each. The gross residential density, in dwelling units per Hectare, would be _______.
Ans:- 105 to 106
Solution: To find the gross residential density in dwelling units per hectare, we need to calculate the
total number of dwelling units in the 50-hectare residential sector.
First, calculate the total buildable area:
Total buildable area = Total sector area × Buildable area percentage
Total buildable area = 50 hectares × 65% = 32.5 hectares
Next, calculate the total floor area for all buildings: Total floor area = Total buildable area × FAR Total
floor area = 32.5 hectares × 1.5 = 48.75 hectares
358 GATE Architecture and Planning: Comprehensive Question Bank

Now, calculate the number of dwelling units based on the given distribution:
Area of each dwelling unit = 60% of units × 100 m2 + 40% of units × 80 m2
Area of each dwelling unit = (0.6 × 100 m2) + (0.4 × 80 m2) = 60 m2 + 32 m2 = 92 m2
Now, calculate the total number of dwelling units: Total number of dwelling units = Total floor area /
Area of each dwelling unit
Total number of dwelling units = 48.75 hectares × (10,000 m2/hectare) / 92 m2/dwelling unit
Total number of dwelling units ≈ 52,717 dwelling units
Finally, calculate the gross residential density:
Gross residential density = Total number of dwelling units / Total sector area
Gross residential density = 52,717 dwelling units / 50 hectares
Now, convert hectares to square meters (1 hectare = 10,000 m2):
Gross residential density = 52,717 dwelling units / (50 × 10,000 m2)
Gross residential density ≈ 105.43 dwelling units per hectare
Q.47 In the given project network diagram, the total slack for job A in days would be ___.

Ans:- 1
Solution: Activity F depends on B+C (6 days) and A+E (5 days). Thus activity A can be delayed by 1 day,
I.e., total slack for job is one day.
Common Data Questions
Common Data for Questions 48 and 49:
The scale of a contour map is 1:10,000 and the contour interval is 5 m. Distance between two given
points on the map is 2 cm and the elevation difference between the two given points is 10 m.

Q.48 The actual distance between the two given points in m would be
(A) 2 (B) 20 (C) 200 (D) 2000
Ans:- (C)
Solution: The scale 1:10,000 means that 1 unit of measurement on the map is equal to 10,000 of the
same units in reality. If the distance on the map is 2 cm, then the actual distance is 2 cm multiplied by
10,000.
2 cm×10,000=20,000 cm
To convert centimeters to meters (since there are 100 centimeters in a meter):
20,000 cm÷100=200 m
Hemant Vilas Parulekar 359

Q.49 The slope between two given points in percentage is


(A) 5 (B) 10 (C) 15 (D) 20
Ans:- 5
Solution: To calculate the slope in percentage, you can use the formula:
Slope (%)=( Elevation Difference / Actual Distance) × 100
We already know the elevation difference is 10 meters. The actual distance, based on the map scale and
the distance between the two points on the map, was calculated to be 200 meters in the previous step.
Now, let’s calculate the slope:
Slope (%)=(10 m / 2000 m) × 100
= 5%
Common Data for Questions 50 and 51:
A point load of 3kN acts at mid-span of a 4 m long cantilever beam as shown in figure below.

Q.50 Shearing force at free end in kN is


(A) 0 (B) 3 (C) 6 (D) 12
Ans:- (A)
Explanation: Shear force is maximum at fixed end and minimum at free end.
Q.51 Bending moment at mid-span in kNm is
Ans:- (A)
Solution: Bending moment at mid span = WL
=3x0
= 0 kNm
Bending moment at left support = WL = 3 x 2 = 6 kNm
Linked Answer Questions
Statement for Linked Answer Questions 52 and 53:
Cost of a new building is Rs 10,00,000 and its scrap value after 50 years is Rs. 1,00,000. Using straight
line method
Q.52 The annual depreciation of the building in Rs. would be
(A) 10,000 (B) 15,000 (C) 18,000 (D) 20,000
Ans:- (C)
Solution: The annual depreciation using the straight-line method can be calculated by subtracting the
scrap value from the original cost and then dividing by the useful life of the asset.
The formula for straight-line depreciation is:
Annual Depreciation = (Cost of the asset−Scrap value) / Useful life of the asset
360 GATE Architecture and Planning: Comprehensive Question Bank

Given:
- Cost of the new building = Rs 10,00,000
- Scrap value after 50 years = Rs 1,00,000
- Useful life = 50 years
= (10,00,000 - 1,00,000)/50 = 18,000/-
Q.53 The book value after 10 years in Rs. would be
(A) 1,80,000 (B) 3,60,000 (C) 6,00,000 (D) 8,20,000
Ans:- (D)
Solution: We already calculated the annual depreciation to be Rs. 18,000. To find the book value after 10
years, we multiply the annual depreciation by 10 and subtract that from the original cost of the building.
Book Value after 10 years=Cost of the building−(Annual Depreciation×10)
Given:
Cost of the new building = Rs 10,00,000
Annual Depreciation = Rs 18,000
Let’s calculate the book value after 10 years.
= 10,00,000 - (18,000 x 10) = 10,00,000 - 1,80,000
The book value of the building after 10 years, using the straight-line method of depreciation, would be
Rs. 8,20,000.
Statement for linked Answer Questions 54 and 55:
A room of size 100 m2 is illuminated by 10 lamps of 40 W having a luminous efficacy of 50 lm/W.
Q.54 Total flux emitted by the lamps in lumens would be
(A) 2,000 (B) 3,60,000 (C) 6,00,000 (D) 8,20,000
Ans:- (D)
Solution: Total flux (Φ) is given by the formula:
Φ=Number of lamps×Power per lamp×Luminous efficacy
Given:
Number of lamps = 10
Power per lamp = 40 W
Luminous efficacy = 50 lm/W
= 10 x 40 x 50 = 20,000 lm
Q.55 If utilization factor is 0.5, at a working height of 90 cm above the floor level, the illumination in lux
would be
(A) 100 (B) 200 (C) 500 (D) 1000
Hemant Vilas Parulekar 361

Ans:- (A)
Solution: Illumination in lux can be calculated by dividing the total luminous flux by the area of the
room and then multiplying by the utilization factor. The formula is:
Illumination (Lux)=Area (m2)Total Flux (Lumens)×Utilization Factor
We previously calculated the total luminous flux to be 20,000 lumens. Given the area of the room is 100
m² and the utilization factor is 0.5, let’s calculate the illumination.
The illumination at a working height of 90 cm above the floor level, given the utilization factor of 0.5,
would be 100 lux. Therefore, the correct answer is: (A) 100
General Aptitude (GA) Questions
Q.56- Q.60 carry one mark each.
Q.56 A number is as much greater than 75 as it is smaller than 117. The number is:
(A) 91 (B) 93 (C) 89 (D) 96
Ans:- (D)
Q.57 Which of the following options is the closest in meaning to the word given below:
The professor Ordered to The student to go Out of the class
I II III IV
(A) I (B) II (C) III (D) IV
Ans:- (B)
Q.58 The question below consists of a pair of related words followed by four pairs of words. Select the
pair that best expresses the relation in the original pair.
Unemployed: Worker
(A) Modern (B) Historic (C) Primitive (D) Antique
Ans:- (C)
Q.59 Friendship, no matter how _________it is, has its limitations.
(A) cordial (B) intimate (C) secret (D) pleasant
Ans:- (B)
Q.60 Select the pair that best expresses a relationship similar to that expressed in the pair:
Medicine: Health
(A) Science: Experiment (B) Wealth: Peace
(C) Education: Knowledge (D) Money: Happiness
Ans:- (C)
Q.61 - Q.65 carry two marks each.
Q.61 X and Y are two positive real numbers such that 2X + Y < 6 and X + 2Y < 8. For which of the
following values of (X, Y) the function f(X,Y) = 3X + 6Y will give maximum value?
(A) (4/3, 10/3) (B) (8/3, 20/3) (C) (8/3, 10/3) (D) (4/3, 20/3)
362 GATE Architecture and Planning: Comprehensive Question Bank

Answer: (A)
Solution: To find the maximum value of the function f(X, Y) = 3X + 6Y subject to the constraints:
2X + Y < 6
X + 2Y < 8
We can use the method of linear programming. First, we’ll graph the feasible region defined by these
constraints.
Let’s start by graphing the lines representing these constraints:
2X + Y = 6
X + 2Y = 8
Now, we’ll shade the region below these lines since we have “<” inequalities.
Next, we need to find the vertices (corners) of this feasible region because the maximum value of the
function f(X, Y) will occur at one of these vertices.
Let’s find the points where these lines intersect to determine the vertices:
Solve the system of equations:
2X + Y = 6
X + 2Y = 8
Let’s solve this system:
From the first equation, we can express Y as: Y = 6 - 2X
Now, substitute this into the second equation: X + 2(6 - 2X) = 8
Simplify and solve for X: X + 12 - 4X = 8 -3X = -4 X = 4/3
Now that we have X = 4/3, we can find Y using the first equation: 2(4/3) + Y = 6 8/3 + Y = 6 Y = 6 - 8/3
Y = 18/3 - 8/3 Y = 10/3
So, one vertex is (4/3, 10/3).
Now, let’s find the other vertices by considering the intersections of the lines with the axes.
- When X = 0, we have Y = 6 (from the first constraint).
- When Y = 0, we have 2X = 6, so X = 3 (from the first constraint).
So, the vertices are: A (0, 6) B (3, 0) C (4/3, 10/3)
Now, we’ll evaluate the function f(X, Y) = 3X + 6Y at these vertices to find the maximum value:
For point A (0, 6): f(0, 6) = 3(0) + 6(6) = 0 + 36 = 36
For point B (3, 0): f(3, 0) = 3(3) + 6(0) = 9 + 0 = 9
For point C (4/3, 10/3): f(4/3, 10/3) = 3(4/3) + 6(10/3) = 4 + 20 = 24
The maximum value of f(X, Y) occurs at point A (0, 6) with a value of 36.
So, the correct answer is (A) (0, 6).
Q.62 If |4X-7| = 5 then the values of 2 |X| - | - X| is:
(A) 2, 1/3 (B) 1/2, 3 (C) 3/2, 9 (D) 2/3, 9
Hemant Vilas Parulekar 363

Ans:- (B)
Solution: If |4x-7|=5, then x=3 (if |4x-7| = +ve.) Also, if |4x-7| = 5, then x=1/2 (if |4x-7|= -ve.)
Put both these values in 2 |x| - |-x|
Thus, 2|x| - |-x| = 2 |3| - |-3| = 6-3 = 3
And, 2 |x| - |-x| = 2 |1/2\ - |-1/2| = 1-(1/2) = 1/2
Q.63 Following table provides figures (in rupees) on annual expenditure of a firm for two years – 2010
and 2011.
Category 2010 2011
Raw material 5200 6240
Power & fuel 7000 9450
Salary & wages 9000 12600
Plant & machinery 20000 25000
Advertising 15000 19500
Research & Development 22000 26400
In 2011, which of the following two categories have registered increase by same percentage?
(A) Raw material and Salary & wages
(B) Salary & wages and Advertising
(C) Power & fuel and Advertising
(D) Raw material and Research & Development
Ans:- (D)
Explanation: To determine which two categories have registered the same percentage increase from
2010 to 2011, you can calculate the percentage increase for each category.
Percentage Increase = [(2011 Value - 2010 Value) / 2010 Value] * 100
Let’s calculate the percentage increase for each category:
-Raw material: [(6240 - 5200) / 5200] * 100 ≈ 20%
-Power & fuel: [(9450 - 7000) / 7000] * 100 ≈ 35.0%
-Salary & wages: [(12600 - 9000) / 9000] * 100 ≈ 40%
-Plant & machinery: [(25000 - 20000) / 20000] * 100 ≈ 25%
-Advertising: [(19500 - 15000) / 15000] * 100 ≈ 30%
-Research & Development: [(26400 - 22000) / 22000] * 100 ≈ 20%
Now, we can see that both Raw material and Research & Development have registered an increase of
approximately 20% from 2010 to 2011.
So, the correct answer is:
(D) Raw material and Research & Development
364 GATE Architecture and Planning: Comprehensive Question Bank

Q.64 A firm is selling its product at Rs. 60 per unit. The total cost of production is Rs. 100 and firm is
earning total profit of Rs. 500. Later, the total cost increased by 30%. By what percentage the price
should be increased to maintained the same profit level.
(A) 5 (B) 10 (C) 15 (D) 30
Ans:- (A)
Solution: initial selling price = 60 Rs.
Initial total cost = 100 Rs.
Initial profit = 500 Rs.
N = (initial profit + initial total cost) / initial selling price
Q.65 Abhishek is elder to Savar.
Savar is younger to Anshul.
Which of the given conclusions is logically valid and is inferred from the above statements?
(A) Abhishek is elder to Anshul
(B) Anshul is elder to Abhishek
(C) Abhishek and Anshul are of the same age
(D) No conclusion follows
Ans: (D)
GATE QUESTION PAPER 2012
General Aptitude (GA)

Q. 1 – Q. 25 carry one mark each.


Q.1 ‘Agora’ was provided in Greek towns as a place of
(A) Worship (B) Drama (C) Sports (D) Meeting
Ans:- (A)
Q.2 The hue at the centre of the Munsell Colour Solid is
(A) Black (B) Grey (C) Sepia (D) White
Ans:- (B)
Q.3 Which one of the following is NOT a traffic calming measure?
(A) Rumble strips (B) Roundabouts
(C) Pedestrian crossings (D) Roadside trees
Ans: (D)
Q.4 ECBC stands for
(A) Electrical Conduit in Building Construction (B) Energy Conservation Building Code
(C) Electrical Credit in Building Code (D) Energy Credit in Building Construction
Ans:- (B)
Explanation: The Energy Conservation Building Code (ECBC) is a set of guidelines and standards
developed to regulate the energy efficiency of new commercial buildings in India. Introduced by the Bureau
of Energy Efficiency (BEE), under the Ministry of Power, Government of India, the ECBC sets minimum
energy performance standards for the design and construction of buildings with the aim of reducing energy
consumption and promoting environmental sustainability. Key aspects of the ECBC include:
Efficient Building Envelope: ECBC outlines standards for building materials and designs that minimize heat
gain or loss, optimizing the building’s thermal performance. This involves recommendations for walls, roofs,
windows, and other elements of the building envelope.
Energy-Efficient Systems: It includes guidelines for HVAC (heating, ventilation, and air conditioning)
systems, lighting, water heating, and other building systems to ensure they are energy-efficient and consume
less power while maintaining comfort.
Use of Renewable Energy: Encouragement of integrating renewable energy sources, like solar panels, to meet
a portion of the building’s energy requirements.
Building Management Systems: Recommendations for advanced control systems and automation that
optimize energy use throughout the building.
Compliance and Implementation: Buildings that meet or exceed the ECBC standards can receive certification
and recognition, encouraging wider adoption of energy-efficient practices in the construction industry.
The ECBC is part of India’s response to increasing energy demands and environmental concerns, contributing
to the country’s commitments to reducing greenhouse gas emissions and promoting sustainable development.
The code is periodically updated to reflect new technologies and practices in the field of energy conservation.
366 GATE Architecture and Planning: Comprehensive Question Bank

Q.5 Age-Sex cohort for a state in India is obtained from


(A) Census of India (B) Election Commission of India
(C) Indian Statistical Institute (D) Survey of India
Ans:- (A)
Q.6 ‘Cover block’ is used as a building construction component in
(A) Brick wall (B) Curtain wall (C) Steel truss (D) RC beam
Ans:- (D)
Q.7 ‘Villa Savoye’, Paris is an example of
(A) Modernism (B) Post Modernism
(C) Deconstructivism (D) Eclecticism
Ans:- (A)
Q.8 The least important measure for reducing cost of site development is
(A) Clustering the units (B) Eliminating landscaping costs
(C) Reducing road lengths (D) Narrowing road widths without reducing the right of way
Ans:- Marks to all
Q.9 The role of a plasticizer in concrete is to improve
(A) Compressive strength (B) Permeability
(C) Workability (D) Tensile strength
Ans:- (C)
Q.10 Which one of the following causes seismic irregularity in a building?
(A) Rectangular plan shape (B) Vertical setback
(C) Increase in height (D) Seismic joint
Ans:- (B)
Q.11 The respective lengths of shadows generated for a free standing wall of given height L by sunlight
incident at angles of 30°, 45°, 60°, 90° to the horizontal are
(A) L , L, L/, 0 (B) L, L, L, 0
(C) L/, L, L, 0 (D) 0, L/, L, L
Ans:- (A)
Q.12 Which one of the following mode is NOT categorized as a public transit?
(A) Bus (B) Ferry (C) Taxi (D) Tram
Ans:- (C)
Q.13 Aerial photography is a useful tool to obtain
(A) Land contour data (B) Land cover data
(C) Land ownership data (D) Landuse data
Ans:- (B)
Q.14 Which one of the following is biodegradable?
(A) Detergent (B) Leather
(C) Recycled plastic (D) Aluminum foil
Hemant Vilas Parulekar 367

Ans:- (B)
Q.15 The volume of surface runoff is least influenced by
(A) existing storm water drainage system (B) amount of rainfall
(C) site slope (D) size of watershed
Ans:- marks to all
Q.16 The best location for laying the main sewer line on a flat land is
(A) under the road (B) under the sidewalk
(C) under the central verge (D) under the open space along sidewalk
Ans:- (A)
Q.17 Ponding is associated with
(A) RC column (B) Steel column
(C) RC slab (D) Steel truss
Ans:- (C)
Explanation: This is the most likely candidate for ponding issues. Reinforced concrete slabs, especially
those used in floors or roofs, can have ponding problems if they are not properly sloped or drained. Flat
roofs or slabs are particularly susceptible to ponding, where water can accumulate and lead to structural
overload or deterioration.
Q.18 Among the following, the urban open space known for its human scale is
(A) Piazza del Campo, Sienna (B) Piazza del Popolo, Rome
(C) St. Peter’s Square, Rome (D) Place de la Concorde, Paris
Ans:- (A)
Piazza del Campo, Sienna Piazza del Popolo, Rome

St. Peter’s Square, Rome Place de la Concorde, Paris


368 GATE Architecture and Planning: Comprehensive Question Bank

Q.19 The most appropriate tree for designing a smell sensory pathway is
(A) Delonix Regia (B) Casia Fistula
(C) Bougainvillea (D) Jasminum Augustifolium
Ans:- (D)
Q.20 Load from a slab to beam is primarily transferred through
(A) axial force (B) bending moment (C) shearing force (D) torsion
Ans:- Marks to all
Q.21 Geographic Information System (GIS) combines maps with
(A) computer automation, statistics and topology
(B) computer graphics, databases and analytical tools
(C) computer graphics, informatics and quantitative tools
(D) computer informatics, databases and qualitative tools
Ans:- (B)
Q.22 ‘ Glazing stop’ is used
(A) as a bearing support for glass
(B) to seal the glass against water and air infiltration
(C) to provide lateral support to glass
(D) to provide a cushion between the glass and the glazing pocket
Ans:- (C)
Q.23 Which one of the following is NOT a basic principle of designing people-friendly urban built
environment?
(A) Diversity (B) Monotony (C) Adaptability (D) Legibility
Ans:- (D)
Q.24 SWOT analysis is used for examining a situation’s inherent
(A) Strength, Wastefulness, Opportunity and Tactfulness
(B) Sanction, Weakness, Ownership and Threat
(C) Strength, Weakness, Opportunity and Threat
(D) Support, Wastefulness, Ownership and Transparency
Ans:- (C)
Q.25 The deflection of a two way slab is primarily a function
(A) of the long span
(B) of the short span
(C) mostly of the long span, sometimes short span
(D) independent of the long or short span
Ans:- Marks to all
Hemant Vilas Parulekar 369

Q.26 - Q.55 carry two marks each.


Q.26 A painting in a living room needs to be lighted at 250 lux. If the light is incident normally on the
painting from a distance of 6 m, then the intensity of light source required (in candela) is
(A) 1500 (B) 3000 (C) 6000 (D) 9000
Ans:- (D)
Solution: The intensity of a light source required to achieve a certain illumination (measured in lux)
on a surface can be calculated using the inverse square law of light, which states that illumination (E) is
inversely proportional to the square of the distance (d) from the light source and is directly proportional
to the luminous intensity (I) of the source. The formula is:
E= I/d2
Where:
E is the illumination in lux,
I is the luminous intensity in candela,
d is the distance from the light source to the surface in meters.
Given:
The desired illumination on the painting is 250 lux,
The distance from the light source to the painting is 6 meters.
We need to rearrange the formula to solve for I (luminous intensity in candela):
I=E×d2
= 250 x (62) = 250 x 36 = 9000
Q.27 Total land area for a plotted housing development project is 50.0 hectare, of which, area under
community facilities, common open spaces and roads are 9%, 10 %, and 16 % respectively. Total
number of saleable plots of 250 m2 each will be
(A) 1100 (B) 1300 (C) 1500 (D) 1700
Ans:- (B)
Solution: Total land area 50 hectares = 50 x 10000 sqm. = 500000 sq m.
Total area under community facilities, common open spaces and roads = 35%
Thus, saleable area = 65%= 0.65 x 500000 sa, m,= 325000 sq.m.
Thus, total number of saleable plots of 250m2= 325000/250 = 1300
Q.28 A 6m long beam is fixed at its left end and is free at its right end. If a concentrated load of 25kN acts
downwards at 4m from the left end, then the bending moment (in kNm) at the left end is
(A) 0.0 (B) 25.0 (C) 50.0 (D) 100.0
Ans:- (D)
Solution: Given:
The load P is 25 kN.
The distance a is 4 m (since the load acts 4m from the left end).
So, the bending moment at the left end is:
M=25 kN×4 m
= 100 kMm
370 GATE Architecture and Planning: Comprehensive Question Bank

Q.29 A classroom of 12m x 25m x 4m requires 3 air changes per hour. At an air velocity of 2 m/s, the
necessary duct cross section (in m2) is
(A) 0.50 (B) 0.75 (C) 1.00 (D) 1.25
Ans:- (A)
Solution: Duct Cross-Section Area (A) = (Air Volume Flow Rate) / (Air Velocity)
First, calculate the air volume flow rate required for 3 air changes per hour:
Air Volume Flow Rate = (Room Volume) x (Air Changes per Hour) Air Volume Flow Rate = (12 m * 25
m * 4 m) * 3 air changes/hour
Now, calculate it:
Air Volume Flow Rate = (1200 m² * 3 air changes/hour) Air Volume Flow Rate = 3600 m³/hour
Next, convert this to m³/second because the air velocity is given in m/s:
Air Volume Flow Rate (m³/second) = Air Volume Flow Rate (m³/hour) / 3600 seconds/hour Air Volume
Flow Rate (m³/second) = 3600 m³/hour / 3600 seconds/hour Air Volume Flow Rate (m³/second) = 1
m³/second
Now, you can calculate the necessary duct cross-section area:
Duct Cross-Section Area (A) = Air Volume Flow Rate (m³/second) / Air Velocity (m/s) Duct Cross-
Section Area (A) = 1 m³/second / 2 m/s Duct Cross-Section Area (A) = 0.5 m²
Q.30 If the original cost of a building is Rs. 25,00,000/- and its scrap value after 55 years is Rs.25,000/-,
then its annual depreciation (in Rs.) using straight line method is
(A) 25,000 (B) 35,000 (C) 45,000 (D) 55,000
Ans:- (C)
Solution: Annual Depreciation = (Original Cost - Scrap Value) / Useful Life
In this case:
-Original Cost = Rs. 25,00,000/-
-Scrap Value = Rs. 25,000/-
-Useful Life = 55 years
Plug these values into the formula:
Annual Depreciation = (25,00,000 - 25,000) / 55
Now, calculate it:
Annual Depreciation = (24,75,000) / 55
Annual Depreciation ≈ Rs. 45,000/-
So, the annual depreciation using the straight-line method is approximately Rs. 45,000/- (Option C).
Hemant Vilas Parulekar 371

Q.31 For a pin jointed steel truss system, which of the following statements is TRUE?
(A) Bending moment resisting capacity at any section is zero as members carry axial forces only.
(B) Forces in members at any section align in appropriate combination of tension and compression to
develop moment resisting capacity.
(C) Forces in members at any section align in such a manner as to develop zero moment resisting
capacity.
(D) Shear resisting capacity at any section is zero as members carry axial forces only.
Ans:- (B)
Q.32 Match the books in Group I with their authors in Group II
(A) P-5, Q-4, R-3, S-2 Group I Group II
(B) P-5, Q-4, R-1, S-2 P. Space, Time and Architecture 1. Jane Jacob
(C) P-4, Q-3, R-2, S-1 Q. Form, Space and Order 2. Kevin Lynch
R. The Death and Life of Great 3. Ian McHarg
(D) P-4, Q-1, R-3, S-5
American Cities
S. The Image of the City 4. Francis D K Ching
5. Gideon S
Ans:- (B)
Q.33 Match the schemes in Group I with their specific targets in Group II
(A) P-2, Q-5, R-1, S-3 Group I Group II
(B) P-5, Q-2, R-1, S-3 P. JNNURM 1. Urban Amenities for Rural Areas
(C) P-5, Q-2, R-3, S-4 Q. IAY 2. Infrastructure and Slum Upgradation
R. PURA 3. Rural Employment
(D) P-2, Q-5, R-3, S-4
S. NREGA 4. Land Acquisition
5. Housing for BPL families

Ans:- (A)
Explanation:
JNNURM (Jawaharlal Nehru National Urban Renewal Mission):
JNNURM was a large-scale government program in India launched in 2005.
-It aimed to promote sustainable and planned development of urban areas, particularly focusing on
cities and towns.
-The program addressed issues like infrastructure development, housing for the urban poor, and
governance reforms in urban local bodies.
-JNNURM was instrumental in improving urban infrastructure and services across India.
IAY (Indira Awaas Yojana):
-IAY was a housing program launched by the Indian government in 1985.
-It primarily aimed to provide financial assistance to below-poverty-line (BPL) households in rural areas
to help them construct or upgrade their houses.
372 GATE Architecture and Planning: Comprehensive Question Bank

-The program sought to improve living conditions for rural poor families by providing them with safe
and sanitary housing.
PURA (Providing Urban Amenities to Rural Areas):
-PURA is a concept and strategy for rural development in India proposed by former President Dr. A.P.J.
Abdul Kalam.
-The goal of PURA is to bridge the rural-urban divide by providing essential urban amenities and
infrastructure to rural areas.
-It focuses on integrating rural development with economic growth and job opportunities, making rural
regions self-sustainable.
NREGA (National Rural Employment Guarantee Act):
-NREGA, also known as the Mahatma Gandhi National Rural Employment Guarantee Act (MGNREGA),
is an Indian labor law and social security measure enacted in 2005.
-The act guarantees a minimum of 100 days of wage employment per year to every rural household,
primarily for unskilled manual work.
-NREGA aims to provide livelihood security to rural households and create durable assets for rural
development.
Q.34 Match the concepts in Group I with the personalities in Group II
(A) P-4, Q-3, R-5, S-1 Group I Group II
(B) P-3, Q-1, R-4, S-2 P. Linear City 1. Le Corbusier
(C) P-4, Q-1, R-5, S-2 Q. Radiant City 2. Paolo Soleri
R. Garden City 3. Louis Kahn
(D) P-1, Q-5, R-2, S-4
S. Arcology 4. Soria Y Mata
5. Ebenezer Howard
Ans:- (C)
Q.35 Match the plan drawings in Group I with their respective scales in Group II
(A) P-4, Q-3, R-5, S-1 Group I Group II
(B) P-3, Q-1, R-4, S-2 P. Site Plan 1. 1:4000
(C) P-4, Q-1, R-5, S-2 Q. Perspective Plan 2. 1:1000000
R. Master Plan 3. 1:500
(D) P-1, Q-5, R-2, S-4
S. Zonal Plan 4. 1:20000
5. 1:2000
Ans:- (D)
Q.36 Match the terminologies of Group I with their corresponding meanings in Group II
(A) P-3, Q-2, R-4, S-5 Group I Group II
(B) P-1, Q-5, R-4, S-2 P. Antarala 1. Antechamber
(C) P-5, Q-3, R-1, S-2 Q. Mandapa 2. Palace Hall
R. Gopuram 3. Womb Chamber
(D) P-1, Q-2, R-3, S-5
S. Prasada 4. Gateway
5. Pillar Sanctuary

Ans:- (B)
Hemant Vilas Parulekar 373

Q.37 Match the standard safety colour codes of Group I with their corresponding usage in Group II
(A) P-3, Q-1, R-2, S-4 Group I Group II
(B) P-3, Q-4, R-5, S-1 P. Blue 1. Biodegradable waste
(C) P-1, Q-3, R-2, S-5 Q. Green 2. Fire protection equipment
R. Red 3. Recyclable waste
(D) P-1, Q-5, R-2, S-4
S. Yellow 4. Stumbling against hazards
5. Radiation standards
Ans:- (A)
Q.38 Match the alignment of the rotating prisms in wall sections in Group I with their corresponding
acoustic function in Group II
(A) P-4, Q-1, R-2, S-3 Group I Group II
(B) P-4, Q-3, R-1, S-2 1. Moderate diffusion
(C) P-2, Q-3, R-4, S-1 2. Moderate absorption
3. Specular diffusion
(D) P-2, Q-1, R-3, S-4
4. Specular reflection

Ans:- (A) (As per official answer key)


Q.39 Match the projects in Group I with their architects in Group II
(A) P-5, Q-4, R-1, S-2 Group I Group II
(B) P-2, Q-1, R-5, S-4 P. Milwaukee Art Museum, Wisconsin 1. Bernard Tschumi
(C) P-1, Q-2, R-4, S-5 Q. Kimbell Art Museum, Fortworth 2. Richard Meier
R. Getty Center, Los Angeles 3. Daniel Libeskind
(D) P-3, Q-2, R-1, S-5
S. Freedom Tower, New York 4. Tadao Ando
5. Santiago Calatrava
Ans:- (A)
Name of Project Milwaukee Art Museum, Kimbell Art Museum, Fortworth
Wisconsin
Architect Santiago Calatrava Tadao Ando
Project
374 GATE Architecture and Planning: Comprehensive Question Bank

Name of Project Getty Center, Los Angeles Freedom Tower, New York
Architect Santiago Calatrava Tadao Ando
Project

Q.40 Match the AutoCAD commands in Group I with their corresponding program in Group II
Group I Group II
P. Cone 1. Specify centre point of base or [3P/2P/Ttr/Elliptical]: Specify base radius or
[Diameter] < > :
Specify height or [2Point/Axis endpoint] < >:
Q. Cylinder 2. Specify centre point or [3P/2P/Ttr]:
Specify radius or [Diameter] < > :
Specify tube radius or [2Point/Diameter] < > :
R. Sphere 3. Specify centre point of base or [3P/2P/Ttr/Elliptical]: Specify base radius or
[Diameter] < > :
Specify height or [2Point/Axis endpoint/Top radius]:
S. Torus 4. Specify centre point of base or [3P/2P/Ttr/Elliptical]: Specify radius or
[Diameter] < > :
Specify tube radius or [2Point/Diameter] < > :
5. Specify centre point or [3P/2P/Ttr]: Specify radius or [Diameter] < > :
(A) P-1, Q-2, R-4, S-5 (B) P-3, Q-1, R-5, S-2
(C) P-1, Q-3, R-2, S-5 (D) P-3, Q-5, R-1, S-2
Ans:- (B)
Q.41 Identify the hierarchy, from highest to lowest, of the number of potential conflict points at the
unmanaged traffic intersections given below.

(A) P, Q, R, S (B) R, Q, S, P (C) R, S, Q, P (D) P, R, S, Q


Ans:- Marks to all
Hemant Vilas Parulekar 375

Q.42 Match the architects in Group I with the terms in Group II


(A) P-2, Q-3, R-1, S-5 Group I Group II
(B) P-3, Q-5, R-1, S-4 P. Kisho Kurokawa 1. Paper Tubes
(C) P-2, Q-1, R-5, S-3 Q. Ken Yeang 2. Deconstructivism
R. Shigeru Ban 3. Metabolism
(D) P-3, Q-4, R-1, S-5
S. Mies van der Rohe 4. Eco Skyscrapers
5. Minimalism

Ans:- (D)
Architect Kisho Kurokawa Ken Yeang
Area of Work Metabolism Eco Skyscrapers
Illustration

Architect Shigeru Ban Mies van der Rohe


Area of Work Paper tubes Minimalism
Illustration

Q.43 Match the terms in Group I with their meanings in Group II


(A) P-5, Q-1, R-4, S-2 Group I Group II
(B) P-3, Q-5, R-1, S-2 P. Mimbar 1. Pillared assembly hall
(C) P-2, Q-3, R-4, S-5 Q. Qibla 2. Covered passage around central court
R. Liwan 3. Pulpit
(D) P-4, Q-5, R-2, S-1
S. Baradari 4. Parapet between wall openings
5. Direction of Mecca

Ans:- (B) (Please refer GATE-2016, Q.16 for more information)


376 GATE Architecture and Planning: Comprehensive Question Bank

Q.44 Match the locations in Group I with the corresponding traps in Group II
(A) P-2, Q-1, R-4, S-2 Group I Group II
(B) P-4, Q-5, R-2, S-3 P. Inspection Chamber 1. P-trap
(C) P-2, Q-4, R-5, S-3 Q. Wash basin 2. Gully trap
R. Bathing space 3. S-trap
(D) P-2, Q-3, R-4, S-1
S. European water closet 4. Bottle trap
5. Floor trap
Ans:- Marks to all
Location Inspection Wash basin Bathing space European water
chamber closet
trap Gulley trap Bottle trap Floor trap S-trap
Image

Q.45 Match the building construction components in Group I with their application areas in Group II
(A) P-2, Q-3, R-4, S-1 Group I Group II
(B) P-2, Q-5, R-1, S-4 P. Bracket plate 1. Steel Column
(C) P-2, Q-5, R-3, S-1 Q. Kick plate 2. Curtain wall
R. Pressure plate 3. Rolling shutter
(D) P-3, Q-5, R-2, S-1
S. Base plate 4. Stone wall
5. Toilet door
Ans:- (D)
Component Bracket plate Kick plate
Application area Rolling shutter Toilet Door
Illustration/image
Hemant Vilas Parulekar 377

Component Pressure plate Base plate


Application area Curtain wall Steel column
Illustration/image

Q.46 Match the historical buildings in Group I with their styles in Group II
(A) P-3, Q-2, R-1, S-4 Group I Group II
(B) P-2, Q-5, R-3, S-1 P. Pantheon, Rome 1. Baroque
(C) P-2, Q-1, R-4, S-5 Q. St. Paul’s Cathedral, London 2. Roman
R. St. Peters Basilica, Rome 3. Romanesque
(D) P-3, Q-4, R-2, S-1
S. Notre Dame, Paris 4. Renaissance
5. Gothic
Ans:- (C)
Name of structure Pantheon St. Paul’s Cathedral
Style Rome London
Illustration

Name of structure St. Peter’s Basilica Notre Dame


Style Rome Paris
Illustration
378 GATE Architecture and Planning: Comprehensive Question Bank

Q.47 Corrected Effective Temperature is an index which combines the effect of


P. Climatic zone Q. Temperature R. Wind velocity
S. Vegetation T. Humidity U. Solar radiation
(A) P, Q, R, S (B) Q, R, T, U (C) Q, S, T, U (D) Q, R, U, P
Ans:- (B)
Explanation: Corrected Effective Temperature (CET) is an index used to assess the thermal comfort of
an environment, taking into account various factors that influence how people perceive temperature.
These factors typically include:
Q. Temperature: The actual air temperature is a primary component of thermal comfort.
R. Wind Velocity: Wind can significantly affect thermal perception, as moving air can make the
environment feel cooler.
T. Humidity: Humidity levels influence how the body perceives and reacts to temperature, particularly
in terms of cooling through sweating.
U. Solar Radiation: Direct exposure to sunlight can increase the perceived temperature.
Based on these factors, the Corrected Effective Temperature index combines the effects of Temperature,
Wind Velocity, Humidity, and Solar Radiation. It does not typically include the specific Climatic Zone
(P) or Vegetation (S) as direct factors, although these can indirectly influence the primary factors.
Common Data Questions
Common Data for Questions 48 and 49:
An aircraft flying at an altitude of 5000 m above mean sea level takes aerial photographs of a flat
terrain having an average elevation of 1000m above mean sea level. The scale of photographs is
1:20000.
Q.48 The focal length of the camera (in cm) is
(A) 15 (B) 20 (C) 25 (D) 30
Ans:- (B)
Solution: The scale of an aerial photograph can be calculated using the formula:
Scale = (Focal Length/Flying Height - Terrain Elevation)
Given:
-The scale of the photographs is 1:20000.
-The aircraft’s flying altitude above mean sea level is 5000 m.
-The terrain’s average elevation above mean sea level is 1000 m.
-Therefore, the flying height above the terrain is 5000 m−1000 m=4000 m.
Rearranging the formula to solve for the focal length:
Focal Length=Scale×(Flying Height - Terrain Elevation)
= (1/20000)x(5000-1000) = 4000/20000 = 0.20m = 20cm
Q.49 The area covered on ground (in hectare) by each photo format of 18 cm x 18 cm is
(A) 1266 (B) 1276 (C) 1286 (D) 1296
Hemant Vilas Parulekar 379

Ans:- (D)
Solution: Given:
-Scale of photographs = 1:20000.
-Photo format = 18 cm x 18 cm.
-1 hectare = 10,000 square meters.
Convert the Dimensions of the Photo to Ground Dimensions:
-Since the scale is 1:20000, each cm on the photo represents 20000 cm on the ground.
-Therefore, the ground dimensions corresponding to the photo format are
18 cm×20000 and 18 cm×20000.
Calculate the Area on the Ground:
-Area = length × width.
-Convert the area to hectares.
Let’s perform the calculations.
The area covered on the ground by each photo format of 18 cm x 18 cm is 1296 hectares.
Common Data for Questions 50 and 51:
2 ton of cement is used to make 10 ton of hardened concrete having a water cement ratio of 0.45.
One cubic meter of this concrete has a mass of 2.5 ton.
Q.50 Quantity of water used (in kg) is
(A) 90 (B) 450 (C) 900 (D) 945
Ans:- (C)
Solution: Given:
-Water-cement ratio = 0.45.
-Amount of cement used = 2 tons.
The water-cement ratio is defined as the weight of water divided by the weight of cement. Therefore, if 1
kg of cement requires 0.45 kg of water, then 2 tons (or 2000 kg) of cement would require:
Water used=Water-cement ratio×Amount of cement
=0.45×2000kg
The quantity of water used in making the concrete is 900 kg.
Q.51 The area of 200 mm thick slab (in sq. m) that can be cast using the entire 10 ton of concrete will be
(A) 10 (B) 15 (C) 20 (D) 25
Ans:- (C)
Solution:
Calculate the Volume of Concrete:
-Total mass of concrete = 10 tons = 10,000 kg (since 1 ton = 1,000 kg).
-One cubic meter of this concrete has a mass of 2.5 tons = 2,500 kg.
380 GATE Architecture and Planning: Comprehensive Question Bank

-Therefore, the volume of 10 tons of concrete is the total mass divided by the mass per cubic meter:
Volume = Total mass of concrete/Mass per cubic meter
Convert the Thickness into Meters:
-The thickness of the slab is 200 mm = 0.2 meters (since 1,000 mm = 1 meter).
Calculate the Area:
-The area of the slab can be found by dividing the volume of concrete by the thickness:
Area = Volume/Thickness
The area of a 200 mm thick slab that can be cast using the entire 10 tons of concrete is 20 square meters.
Linked Answer Questions
Statement for Linked Answer Questions 52 and 53:
A group housing project spread over 20 hectares with FAR 1.5 has to accommodate different housing
units in the following manner:`
Category Percentage Built up area per unit in sq. m Average household
distribution (including common areas) size
P 10 120 4.0
Q 20 80 4.6
R 30 60 5.5
S 40 45 6.0
Q.52 The respective number of housing units to be built for each category are
(A) P – 25, Q – 75, R – 200, S - 200 (B) P – 75, Q – 25, R – 200, S - 200
(C) P – 25, Q – 25, R – 75, S - 200 (D) P – 25, Q – 75, R – 200, S - 75
Ans:- (A)
Q.53 The gross density of the above housing complex (in persons per hectare) is
(A) 125 - 130 (B) 130 - 135 (C) 135 - 140 (D) 140 - 145
Ans:- (C)
Statement for linked Answer Questions 54 and 55:
An unplastered brickwork has a resistivity of 0.83 m degC/W. For a 230 mm thick brick wall, the
inside and outside surface resistances are 0.123 and 0.1 m2 degC/W respectively.
Q.54 The R-value of the given wall section (in m2 degC/W) is
(A) 0.41 (B) 0.5 (C) 0.67 (D) 1.05
Ans:- (A)
Solution: Resistivity of unplastered wall= 0.83 m degC/W
Thus, R-value for 230 mm (0.23m) thick wall= 0.83 x 0.23= 0.19 m2 degC/W.
R-Value of inside surface= 0.123 m2 degC/W.
R-Value of outside surface= 0.1 m2 degC/W.
Thus, R-Value of total wall section= 0.19 + 0.123 + 0.1= 0.413 m2 degC/W.
Hemant Vilas Parulekar 381

Q.55 The brick wall has an overall dimension of 6m x 3m with a 1m x 2m window assembly (U-value =
2.5 W/m2 degC) and a 1m x 2.1m door assembly (U-value = 1.25 W/m2 degC). The overall U-value
of the wall (UO) (in W/m2 degC) is
(A) 1.8 (B) 2.3 (C) 2.6 (D) 3.7
Ans:- (B)
Solution: Area of Brick wall= 18 m2, Area of Window= 2 rn2. Area of Door- 2.1 m2
Thus, actual area of brick wall = 18 - (2+2.1) = 13.9 m2
As, R-Value= 1/ U-Value, from above question,
Value of wall= 1/0.413 = 2.42 W/m2 degC,
Heat loss from wall= U-value x area = 2.42 x 13.9 = 33.63 W
Heat loss from window= U-value x area = 2.5 x 2 = 5 w
Heat loss from door= U-value x area = 1.25 x 2.1 = 2.625W
Total heat loss= 33.63 + 5 + 2.625 = 41.255 W
Overall area of Wall + Window+ Door= 18 m2
Thus, overall U-value of the wall= Total heat loss/ Overall area = 41.255/12= 2.29 W/m2 degC
General Aptitude (GA) Questions
Q.56- Q.60 carry one mark each.
Q.56 Which one of the following options is the closest in meaning to the word given below?
Pacify
(A) Excite (B) Soothe (C) Deplete (D) Tire
Ans:- (B)
Q.57 Choose the most appropriate pair of words from the options given below to complete the following
sentence: The high level of ___ of the questions in the test was ___ by an increase in the period of
time allotted for answering them.
(A) difficulty, compensated (B) exactitude, magnified
(C) aptitude, decreased (D) attitude, mitigated
Ans:- (A)
Q.58 Choose the grammatically CORRECT sentence:
(A) He laid in bed till 8 o’clock in the morning.
(B) He layed in bed till 8 o’clock in the morning.
(C) He lain in bed till 8 o’clock in the morning.
(D) He lay in bed till 8 o’clock in the morning.
Ans:- (D)
Q.59 Which one of the parts (A, B, C, D) in the sentence contains an ERROR? No sooner had the doctor
seen the results of the blood test, than he suggested the patient to see the specialist.
(A) no sooner had (B) results of the blood test
(C) suggested the patient (D) see the specialist
382 GATE Architecture and Planning: Comprehensive Question Bank

Ans:- (B)
Correct: “suggested to the patient” or “suggested that the patient should.”
Q.60 Ten teams participate in a tournament. Every team plays each of the other teams twice. The total
number of matches to be played is
(A) 20 (B) 45 (C) 60 (D) 90
Ans:- (C)
Q.61 - Q.65 carry two marks each.
Q.61 X and Y are two positive real numbers such that 2X + Y < 6 and X + 2Y < 8. For which of the
following values of (X, Y) the function f(X,Y) = 3X + 6Y will give maximum value?
(A) (4/3, 10/3) (B) (8/3, 20/3) (C) (8/3, 10/3) (D) (4/3, 20/3)
Answer: (D)
Solution: As, log A + log B = log (AxB)
Here,, log x + log (x-7) = log (x+11) + log 2
Thus, x(x-7) = (x+11)2
X2-7x = 2x+22
X2-7x-22-2x = 0
X2-9x-22 = 0
Thus (x-11) = 0 or (x+2) = 0, Thus, x = 11 or x = -2
As, x = 2 is not acceptable, x = 11
Q.62 Let f(x) = x – [x], where x ≥ 0 and [x] is the greatest integer not larger than x. Then f(x) is a
(A) monotonically increasing function
(B) monotonically decreasing function
(C) linearly increasing function between two integers
(D) linearly decreasing function between two integers
Ans:- (C)
Solution: The function f(x)=x−[x], where x≥0 and [x] represents the greatest integer not larger than x
(also known as the floor function), has specific characteristics.
The floor function [x] rounds down x to the nearest integer less than or equal to x. So, f(x) essentially
represents the fractional part of x. For any x, this fractional part is the difference between x and the
largest integer less than or equal to x.
1. Monotonically Increasing Function:
A function is monotonically increasing if for any x1< x2, f(x1) ≤ f(x2). However, f(x) is not monotonically
increasing because it resets to 0 at every integer.
2. Monotonically Decreasing Function:
A function is monotonically decreasing if for any x1< x2, f(x1) ≥ f(x2). Again, f(x) does not satisfy this
definition since it increases from 0 to just below 1 as x moves from one integer to the next.
Hemant Vilas Parulekar 383

3. Linearly Increasing Function Between Two Integers:


Between any two integers, f(x) does increase linearly from 0 to just below 1. For example, between 1 and
2, f(x) increases from 0 to just below 1.
4. Linearly Decreasing Function Between Two Integers:
f(x) does not decrease between integers; it increases.
Therefore, the correct description of f(x) is that it is a linearly increasing function between two integers.
The answer is (C) linearly increasing function between two integers.
Q.63 Ravi is taller than Arun but shorter than Iqbal, Sam is shorter than Ravi, Mohan is shorter than
Arun, Balu is taller than Mohan and Sam. The tallest person can be
(A) Mohan (B) Ravi (C) Balu (D) Arun
Ans:- (C)
Explanation: Ravi is taller than Arun but shorter than Iqbal.
Sam is shorter than Ravi.
Mohan is shorter than Arun.
Balu is taller than Mohan and Sam.
Now, let’s use this information to deduce the tallest person:
We know Ravi is taller than Arun but shorter than Iqbal. So, the order of height from shortest to tallest
is Arun < Ravi < Iqbal.
Sam is shorter than Ravi. Since Ravi is taller than Arun, Sam is shorter than Arun as well.
Mohan is shorter than Arun. So, the order of height from shortest to tallest is Sam < Mohan < Arun.
Balu is taller than Mohan and Sam. Therefore, Balu must be taller than Arun as well.
So, based on this information, the tallest person among the given options is Balu.
Q.64 A smuggler has 10 capsules in which five are filled with narcotic drugs and the rest contain the
original medicine. All the 10 capsules are mixed in a single box, from which the customs officials
picked two capsules at random and tested for the presence of narcotic drugs. The probability that
the smuggler will be caught is
(A) 0.50 (B) 0.67 (C) 0.78 (D) 0.82
Ans:- (A)
Solution: The smuggler would be caught if any one or both the randomly picked capsule contains drugs.
M = Medicine, D = Drugs
P - The smuggler will be caught
Q - The randomly picked capsules contains M, D
R - The randomly picked capsules contains D, M
S - The randomly picked capsules contains D, D
P(P) = P(Q) + P(R) + P(S)
P(A) = (5/10) x (5/9) + (5/10) x (5/9) + (5/10) x (4/9)
= 0.278 + 0.278 + 0.222 = 0.778
384 GATE Architecture and Planning: Comprehensive Question Bank

Q.65 The documents expose the cynicism of the government officials – and yet as the media website
reflects, not a single newspaper has reported on their existence. Which one of the following
inferences may be drawn with the greatest accuracy from the above passage?
(A) Nobody other than the government officials knew about the existence of the documents.
(B) Newspapers did report about the documents but nobody cared.
(C) Media reports did not show the existence of the documents.
(D) The documents reveal the attitude of the government officials.
Ans: (D)
GATE QUESTION PAPER 2011
General Aptitude (GA)

Q.1 – Q. 25 carry one mark each.


Q.1 Capital town of Gandhinagar is designed by
(A) Norman Foster (B) B.V. Doshi (C) H.K. Mewada (D) Le Corbusier
Ans:- (C)
Q.2 Rajiv Awas Yojana of Ministry of Housing, Government of India addresses housing for
(A) Middle Income Group (B) Low Income Group
(C) High Income Group (D) Slum Dwellers
Ans:- (D)
Q.3 The triangular space formed by two consecutive arches is
(A) Tympanum (B) Spandrel (C) Regula (D) Extrados
Ans:- (A)
Q.4 Rose window is an iconic feature of
(A) Notre Dame, Paris (B) Hagia Sophia, Istanbul
(C) St. Peter’s, Rome (D) Victoria Memorial, Kolkata
Ans:- (A)
Q.5 Purity of colour is described by
(A) Hue (B) Value (C) Chroma (D) Tone
Ans:- (C)
Q.6 A slab simply supported on all its adges with a ratio of longer side to shorter side greater to or equal
to 2.0 is designed as
(A) One way slab (B) Two way slab
(C) Flat Slab (D) Coffered Slab
Ans:- (A)
Q.7 Entablature consists of
(A) Architrave, Tenia, Cornice (B) Architrave, Frieze, Cornice
(C) Frieze, Cornice, Triglyphs (D) Cornice, Guttae, Tympanum
Ans:- (B)
Q.8 Town Planned for ‘Motor Age’ refers to
(A) Toronto, Ontario (B) Nassan Shores, Long Island
(C) Radburn, New Jersey (D) Green Belt, Maryland
Ans:- (C)
Q.9 The minimum road curb length required for parking 10 cars perpendicular to the road is
(A) 15 m (B) 25 m (C) 35 m (D) 40 m
386 GATE Architecture and Planning: Comprehensive Question Bank

Ans:- (B)
Explanation: As per the standards, community car parking requires 2.5m x 5m single parking. Thus 10
cars will be require 25m road length.
Q.10 Which of the following generates heat island?
(A) Urban Areas (B) Coastal Areas
(C) Wetlands (D) Forest areas
Ans:- (A)
Explanation: Heat islands, in stark contrast to tropical paradises, are urban areas that experience
significantly higher temperatures compared to surrounding rural areas. Imagine walking down a
scorching city street, feeling the radiating heat from concrete pavements and buildings, while a cool
breeze blows just kilometers away in the countryside. This phenomenon, with its environmental and
health implications, deserves a closer look.
Causes of the Heat Island Effect:
Urbanization and Infrastructure: Buildings, roads, and pavements absorb and retain heat much more
efficiently than natural landscapes like soil and vegetation. This stored heat radiates outwards, raising
the ambient temperature.
Lack of Vegetation: Trees and plants provide shade and transpire water, creating a cooling effect through
evaporation. In urban areas, with limited greenery, this natural air conditioning is missing, leading to
hotter temperatures.
Anthropogenic Heat Emissions: Vehicles, air conditioners, and industrial processes release significant
amounts of heat into the atmosphere, further elevating urban temperatures.
Consequences of Heat Islands:
Public Health Risks: Extreme heat events are associated with increased mortality, heatstroke, respiratory
problems, and cardiovascular stress.
Air Quality Impacts: Heat islands can worsen air quality by trapping pollutants and creating conditions
favorable for the formation of ozone.
Increased Energy Consumption: Air conditioning use to combat heat rises with higher temperatures,
putting additional strain on energy grids.
Water Resource Stress: Increased water demand for cooling purposes can strain water resources,
especially during droughts.
Addressing the Heat Island Effect:
Urban Greening: Planting trees, creating parks, and installing green roofs can shade surfaces, cool the
air through evapotranspiration, and improve air quality.
Cool Pavement and Buildings: Using reflective materials for pavements and roofs can decrease solar
heat absorption, lowering surface temperatures.
Urban Planning and Design: Building designs that incorporate natural ventilation, shade, and green
spaces can mitigate the heat island effect.
Reducing Urban Heat Sources: Promoting public transportation, electric vehicles, and energy-efficient
technologies can lower heat emissions in cities.
Understanding the heat island effect is crucial for designing sustainable and resilient cities. By
implementing a multifaceted approach that combines green infrastructure, smart planning, and
technological advancements, we can create cooler, healthier, and more livable urban environments.
Hemant Vilas Parulekar 387

Q.11 The most suitable earthquake resistant built plan form is

Ans:- (C)
Explanation: More symmetrical plan is more earthquake resistant.
Q.12 Transfer of Development Right (TDR) is a tool used for
(A) Human Development (B) Land Development
(C) Economic Development (D) Infrastructure Development
Ans:- (B)
Explanation: TDR stands for Transfer of Development Rights. It’s a planning mechanism used in
urban and regional development to manage land use and promote efficient land use patterns while
preserving valuable natural resources and heritage. Here’s a brief explanation of TDR:
Basic Concept: TDR is a market-based tool that allows landowners to sell their development rights
from one piece of property (sending site) to another (receiving site). The sending site is typically
in an area where development is restricted or where conservation is desired, like historic districts
or ecologically sensitive areas. The receiving site is usually in an area where more development is
allowed.
Preservation and Development: TDR serves a dual purpose. It encourages the preservation of
open spaces, historical landmarks, or environmentally sensitive lands by compensating landowners
for the development rights they’re giving up on the sending site. At the same time, it allows for more
intensive development on the receiving site, helping to accommodate urban growth.
Zoning and Land Use Regulations: TDR relies on zoning and land use regulations to function
effectively. Zoning laws define the areas where development rights can be transferred (receiving
zones) and the areas from which rights can be transferred (sending zones).
Market Mechanism: The actual transfer of development rights typically involves a market
mechanism. Landowners in sending zones receive TDR credits, which they can sell to developers
or landowners in receiving zones who want to increase the density or intensity of development on
their properties. These credits have a monetary value.
Benefits: TDR has several benefits. It helps preserve green spaces, historic buildings, and sensitive
environments. It can also aid in managing urban sprawl by concentrating development in specific
areas. Additionally, it can be a cost-effective way for municipalities to achieve their land use and
conservation goals.
Challenges: Implementing TDR programs can be complex, requiring clear regulations, valuation
mechanisms for TDR credits, and monitoring to ensure compliance. Furthermore, it depends on
active participation from landowners and developers.
388 GATE Architecture and Planning: Comprehensive Question Bank

Q.13 Dandaka form of settlement layout is basically a


(A) Grid Iron Pattern (B) Ring radial pattern
(C) Radial pattern (D) Informal pattern
Ans:- (A)
Q.14 Maximum horizontal angle from the speaker in a seating area of a lecture theatre should be
(A) 70˚ (B) 90˚ (C) 120˚ (D) 140˚
Ans:- (C)
Q.15 ‘U-value’ refers to
(A) Utility function for convective heat transfer
(B) Thermal transmittance of building components
(C) Energy transfer between thermal bridges
(D) Measure for area related heating and cooling loads
Ans:- (B) (Please refer GATE-2016, Q.14 for more information).
Q.16 Consistency of cement is measured by
(A) Pyrometer (B) Slump cone
(C) Universal testing machine (D) Vicat’s apparatus
Ans:- (D)
Q.17 The appropriate material for flooring of an external ramp of building would be
(A) Polished granite (B) Wax polished marble
(C) Glazed ceramic tile (D) Rough finish sandstone
Ans:- (D)
Q.18 Which of the following is NOT a member of a Steel Truss?
(A) Gusset Plate (B) Wall Plate (C) Fish Plate (D) Anchor Bolts
Ans:- (C)
Q.19 Identify the odd one among the following
(A) Security Deposit (B) Professional tax
(C) Performance bank guarantee (D) Earnest money
Ans:- (B)
Explanation: The other options, (A) Security Deposit, (C) Performance bank guarantee, and (D) Earnest
money, are all financial terms related to various forms of deposits or guarantees, while “Professional tax”
is a type of tax imposed on individuals for their profession or employment.
Q.20 Weep hole is a term to describe
(A) Perforations in the cast iron pipe used for boring
(B) Holes in the retaining wall for draining water
(C) Holes in the cover plate of floor traps
(D) Holes dug in earth to recharge ground water
Ans:- (B)
Hemant Vilas Parulekar 389

Q.21 Busway, Busduct and Raceway are components of


(A) Security systems (B) Air-conditioning systems
(C) Electrical systems (D) Water supply systems
Ans:- (C)
Q.22 The difference between Wet Bulb Temperature and Dry Bulb Temperature is called
(A) Dry bulb depression (B) Wet bulb depression
(C) Variable depression (D) Atmospheric depression
Ans:- (B)
Q.23 In India, one of the Slum Improvement initiatives is
(A) Special Residential Zone (B) Valmiki Ambedkar Malin Basti Awas Yojana
(C) Indira Awas Yojana (D) Eco Housing
Ans:- (B)
Explanation:
Special Residential Zone: This term isn’t typically associated with a specific government program. It
may refer to areas zoned specifically for residential purposes under urban planning and development
regulations.
Valmiki Ambedkar Malin Basti Awas Yojana (VAMBAY): This is a government scheme in India aimed
at providing housing and basic amenities to people living in slums and disadvantaged urban areas. It
focuses on improving the living conditions of the urban poor by constructing houses and providing
access to sanitation, clean drinking water, and electricity.
Indira Awas Yojana: Now known as the Pradhan Mantri Awas Yojana - Gramin (PMAY-G), this initiative
aims to provide affordable housing to rural households in India. It focuses on helping those living below
the poverty line by providing them with financial assistance to build or upgrade their houses.
Eco Housing: This refers to housing projects that incorporate environmentally friendly and sustainable
design principles. These projects aim to reduce their environmental impact by using renewable energy
sources, eco-friendly building materials, and efficient waste management systems.
Q.24 Suspended Floors is a structural system used in
(A) Lloyds Building, London (B) Jin Mao Building, Shanghai
(C) Petronas Tower, Kualalumpur (D) Hongkong Shanghai Bank, Hongkong
Ans:- (D)
Explanation: The Hong Kong Shanghai Bank of Hong Kong (HSBC) Headquarters, designed by
architect Lord Norman Foster, is an iconic postmodern skyscraper known for its distinctive features:
Geometric Design: It features cylindrical structures and reflective glass.
Open Plaza: The base has an open plaza, creating a public space.
Feng Shui Elements: It incorporates Feng Shui elements like lion statues.
Symbolism: The cylindrical structures symbolize Hong Kong’s maritime
history.
Transparency: Glass represents transparency in banking.
Iconic Landmark: It’s an iconic Hong Kong landmark on Victoria Harbour.
390 GATE Architecture and Planning: Comprehensive Question Bank

Q.25 Residual method of valuation is used to determine


(A) Public Private Partnership Deal (B) Rent
(C) Property Tax (D Selling Price
Ans:- (A)
Q.26 to Q.55 carry two marks each.
Q.26 Match the buildings in Group I with their architects in Group II
(A) P-5, Q-2, R-1, S-4 Group I Group II
(B) P-5, Q-4, R-1, S-3 P. Bibliotheca Alexandrina, Alexandria 1. I. M. Pei
(C)P-4, Q-2, R-5, S-3 Q. Institute de Monde Arab, Paris 2. Jean Nouvel
R. Bank of China, Hongkong 3. Daniel Libeskind
(D) P-5, Q-2, R-1, S-3
S. Jewish Museum, Berlin 4. Renzo Piano
5. SnØhetta
Ans:- (D)
Q.27 A room measuring 5 m x 3.5 m enclosed by brick wall has a ceiling at 3m height. The room has
a door and window opening of 1 m x 2 m and 1 m x 1 m respectively. The quantity of plastering
required for interior walls (in sq m) is
(A) 46.5 (B) 48 (C) 51 (D) 68.5
Ans:- (B)
Solution:
Calculate the Total Wall Surface Area:
The room measures 5 m x 3.5 m and has a ceiling at 3 m height.
There are four walls. Two walls measure 5 m x 3 m (length x height), and the other two walls measure
3.5 m x 3 m.
Calculate the Areas of the Door and Window:
The door measures 1 m x 2 m.
The window measures 1 m x 1 m.
Subtract the Areas of the Door and Window from the Total Wall Surface Area:
Total plastering area = Total wall surface area - (Door area + Window area).
-Dimensions of the room
length = 5 m
width = 3.5 m
height = 3 m
-Surface area of the walls
Two walls of size length x height and two walls of size width x height
Total wall area = 2 * (length * height) + 2 * (width * height)
= 2 * (5 * 3) + 2 * (3.5 * 3) = 30 + 21 = 51 m2
Hemant Vilas Parulekar 391

-Dimensions of the door and window


Door area = 1m x 2m = 2 m2
Window area = 1m x 1m = 1 m2
-Total area to be plastered
Total plaster area = total wall area - (door area + window area)
Total plaster area = 51 - (2+1) = 48 m2
Q.28 One cubic meter of Ordinary Portland Cement yields a volume of M15 concrete in the range of
(A) 2 to 3 cum (B) 4 to 5 cum (C) 7 to 8 cum (D) 8 to 9 cum
Ans:- (B)
The constituent mix of M:15 concrete by nominal mix is 1:2:4
Cement is measured in terms of standard bags of 50kg (0.035m3)
Q.29 Match the CAD commands in Group I with their functions in Group II
Group I Group II
(A) P-2,Q-4,R-1,S-5 P. LAYISO 1. Blends selected object to destination layer
(B) P-3,Q-2,R-1,S-5 Q. LAYMCH 2. Freezes layer of selected object
R. LAYMRG 3. Hides or locks layers other than those of selected
(C) P-4,Q-2,R-3,S-5
objects
(D) P-3,Q-4,R-1,S-5 S. LAYLCK 4. Assigns selected object to destination layer
5. locks object of destination layer

Ans:-marks to all (still D seems to be the correct one)


Explanation:
LAYISO (Isolate Layer):
This command typically isolates the selected objects, hiding or locking all layers except those containing
the selected objects.
Function: 3. Hides or locks layers other than those of selected objects.
LAYMCH (Match Layer):
This command is used to change the layer of a selected object to match the layer of another object.
Function: 4. Assigns selected object to destination layer.
LAYMRG (Merge Layer):
This command merges objects from different layers into a single layer.
Function: 1. Blends selected object to destination layer.
LAYLCK (Lock Layer):
This command locks the layers of selected objects, preventing modifications.
Function: 5. Locks object of destination layer.
Based on these descriptions, the correct matching would be:
P. LAYISO - 3. Hides or locks layers other than those of selected objects.
Q. LAYMCH - 4. Assigns selected object to destination layer.
R. LAYMRG - 1. Blends selected object to destination layer.
S. LAYLCK - 5. Locks object of destination layer.
392 GATE Architecture and Planning: Comprehensive Question Bank

Q.30 Match the buildings in Group I with their corresponding structural forms in Group II
(A) P-3,Q-5,R-4,S-1 Group I Group II
(B) P-2,Q-5,R-4,S-1 P. Hall of Nations, New Delhi 1. Spherical Structure
(C) P-3,Q-5,R-4,S-2 Q. Salvacao Church, Mumbai 2. Folded Plates
R. State Trade Corporation 3. Octahedral lattice structure
(D) P-3,Q-5,R-2,S-1
Building, New Delhi
S. Matrimandir, Auroville 4. Vierendeel girders
5. Shell roof structure

Ans:- (A)
Structure-Architect Hall of Nations-Raj Rewal Salvacao Church-Charles
Correa
Description
Image/illustration

Structure-Architect State Trade Corporation Matrimandir-Mirra Alfassa,


Building-Raj Rewal Roger Anger
Description
Image/illustration

Q.31 Identify the INCORRECT statement


(A) Guggenheim, Bilbao is an example of Deconstructivism
(B) Silver Abstraction is a term used for metal clad modern high rise buildings
(C) Spiral Building in Tokyo has a curvilinear built form
(D) Free Building plan form is a concept given by Le Corbusier
Hemant Vilas Parulekar 393

Ans:-Marks to all
Q.32 Match the terms in Group I with their descriptions in Group II
(A) P-4,Q-3,R-2,S-1 Group I Group II
(B) P-3,Q-5,R-1,S-4 P. Quoin 1. Geometric representation of the universe
(C)P-4,Q-5,R-1,S-2 Q. Stucco 2. Small dome
R. Mandala 3. Triangular form above an opening
(D) P-3,Q-1,R-5,S-4
S. Cupola 4. Corner stone at the angle of buildings
5. Plaster
Ans:- (C)
Term Quoin Stucco
Description Corner stone of the angle of Plaster
the building
Image/illustration

Term Mandala Cupola


Description Geometric representation of Small dome
the universe
Image/illustration

Q.33 Match the architectural styles in Group I with their features in Group II
(A) P-3,Q-2,R-1,S-4 Group I Group II
(B)P-5,Q-4,R-1,S-2 P. West Asiatic 1. Arches and pendentives
(C) P-5,Q-4,R-1,S-3 Q. Greek 2. Pagodas
R. Byzantine 3. Flying buttresses
(D) P-4,Q-3,R-5,S-2
S. Japanese 4. Orders and pediments
5. Hanging gardens
394 GATE Architecture and Planning: Comprehensive Question Bank

Ans:- (B)
Q.34 Gestalt’s Laws of visual perception DO NOT relate to
(A) Aesthetics of form are a function of Golden Section
(B) Things are perceived as a whole
(C) Whole is greater than the sum total of its parts
(D) Elements with continuity are perceived together
Ans:- (A)
Explanation:
Law of Proximity: This law Law of Similarity: According Law of Closure: This law suggests
states that objects or shapes to this law, items that are similar that the human brain tends to
that are close to one another tend to be grouped together by perceive complete shapes even if
appear to form groups. Even if the human mind. It implies that a picture is incomplete, partially
the shapes, sizes, and objects we naturally perceive similar hidden, or not fully closed. If
are radically different, they elements (in color, shape, size, enough of the shape is indicated,
will appear as a whole when or value) as a part of a pattern people perceive the whole by
they are close. This principle is or group. This is a critical filling in the missing information.
often used in graphic design to concept in visual design, where This principle is often utilized in
group elements that are related, similar elements are used to logo design and abstract art, where
helping to organize information create cohesion in a layout. the viewer’s perception completes
more effectively. the visual information.

Law of Symmetry: The law Law of Continuity: This law Law of Figure-Ground: This
of symmetry indicates that states that elements arranged principle deals with the human
the human mind perceives in a line or curve are perceived tendency to perceive two parts in
objects as being symmetrical as more related than elements a visual display—a figure (object)
and forming around a center not on the line or curve. Our and a ground (background). It
point. Symmetrical images are eyes are drawn along paths, describes our ability to separate
often seen as more harmonious lines, and curves, and tend elements based upon contrast,
and balanced, leading to an to follow them, rather than such as an object against a
easier and more pleasing focusing on separate elements. backdrop, which is fundamental
visual experience. This law This principle is used in various in fields like typography and
is particularly relevant in types of visual designs to guide interface design.
architecture and product the viewer’s attention.
design.
Hemant Vilas Parulekar 395

Q.35 A site in a map drawn to scale of 1:16000 measures 75 sq. cm. The actual area of the site in hectares is
(A) 120 (B) 162 (C) 192 (D) 256
Ans:- (C)
Solution: Given:
Map scale = 1:16000 (This means 1 unit on the map represents 16000 units in reality).
Area on the map = 75 sq. cm.
The linear scale factor is 1:16000. Therefore, the area scale factor is (1:16000)².
The actual area can be calculated as follows:
Actual Area=Map Area×(Scale Factor)2
= 75 x (1/16000)2
= 1.92 x 1010 / 108
= 192 Ha
Q.36 Identify the CORRECT CAD statements
P. SPLINE connects sequence of line segments into a single object
Q. SPLINE is a smooth curve passing through or near a given set of points
R. PLINE creates straight line segments, arc segments or both
S. PLINE can be closed only when its start and end points are coincident and tangent
T. PLINE allows adjusting the width and curvature of its multi-line segments
U. SPLINE can be explode into smaller segments
V. PLINE can be converted into a continuous curve segment
(A) P, R, S, U (B) Q, R, T, V (C) R, S, T, V (D) S, T, U, V
Ans:- (B)
Q.37 Match the eminent personalities in Group I with their books and statements in Group-II
(A) P-4,Q-2,R-5, S-3 Group I Group II
(B) P-3,Q-1,R-2,S-5 P. Kevin Lynch 1. The Fountainhead
(C) P-5,Q-1,R-4,S-2 Q. Ayn Rand 2. Small is beautiful
R. Paul D. Spreiregen 3. Site Planning
(D) P-3,Q-1,R-4,S-2
S. E. F. Schumacher 4. Urban Design: Architecture of Towns and
Cities
5. Design of Cities
396 GATE Architecture and Planning: Comprehensive Question Bank

Ans:- (D)
Q.38 Match the urban forms listed in Group I with their towns listed in Group-II
(A) P-2,Q-1,R-4,S-3 Group I Group II
(B) P-3,Q-1,R-2,S-5 P. Grid Iron 1. New Delhi
(C) P-3,Q-1,R-4,S-2 Q. Radial 2. Washington D.C.
R. Linear 3. Copenhagen
(D) P-2,Q-1,R-4,S-5
S. Finger plan 4. Mumbai
5. Canberra
Ans:- (A)
Town New Delhi Washington DC Copenhagen
Urban form Radial Grid iron Finger shaped
Plan

Town Mumbai Canberra


Urban form Linear Geometric Shape, octagon,
hexagon
Plan

Q.39 Consider the following features


1. Length finely proportional to its width
2. Statues as silhouettes against the sky above cornice line
3. Fountains signifying fine vintage points
4. Series of different shapes connected by traditional narrow streets, column screens arches
The element of urban design which comprises the above is
(A) Vista (B) Piazza (C) Rod Point (D) Bosque
Ans:- (B)
Hemant Vilas Parulekar 397

Q.40 Match the instruments in Group I with their books and functions in Group II
(A) P-4,Q-1,R-2,S-3 Group I Group II
(B) P-4,Q-3,R-2,S-5 P. Hygrometer 1. Precipitation
(C) P-1,Q-2,R-5,S-4 Q. Disdrometer 2. Vapour Pressure
R. Anemometer 3. Solar radiation
(D) P-4,Q-1,R-5,S-2
S. Manometer 4. Relative Humidity
5. Velocity of Air
Ans:- (D)
Instrument Hygrometer Disdrometer Anemometer Manometer
To measure Relative humidity Precipitation Wind velocity Vapor Pressure
Image

Q.41 Match the features in Group I with the corresponding type of garden in Group-II
(A) P-2, Q-1, R-4, S-3 Group I Group II
(B) P-4, Q-1, R-5, S-3 P. Symmetrical layout, water cascades, 1. French gardens
entombment
(C)P-4, Q-3, R-5, S-1
Q. Radial layout, symmetrical sculpture, 2. English gardens
(D) P-5, Q-1, R-2, S-3 boulevards
R. Occult symmetry, pontoon bridges, stepping 3. Chinese garden
stones
S. Hierarchy of courts, hierarchy of gates, 4. Mughal garden
zoomorphic forms
Lamp Posts 5. Japanese garden
Ans:- (B)
Explanation:
Garden French gardens English gardens
Attributes Radial layout, symmetrical informal and naturalistic style, often
sculpture, boulevards featuring meandering pathways, lush
green lawns, and a mix of native and
exotic plants, creating a relaxed and
picturesque atmosphere.
Plan/image of garden
398 GATE Architecture and Planning: Comprehensive Question Bank

Garden Mughal gardens Chinese gardens


Attributes Symmetrical layout, water Hierarchy of courts, hierarchy of
cascades, entombment gates, zoomorphic forms
Plan/image of garden

Garden Japanese gardens


Attributes Occult symmetry, pontoon bridges, stepping stones
Plan/image of garden

Q.42 Arrange the following the sense of enclosures in a hierarchy of decreasing order

(A) S>Q>U>P>T>R (B) U>S>Q>R>P>T


(C) P>Q>R>S>T>U (D) T>P>S>Q>U>R
Ans:- (B)
Q.43 Match the elements in Group I with their corresponding type in Group-II
(A) P-2,Q-1,R-1,S-2,T-2 Group I Group II
(B) P-1,Q-1,R-2,S-1,T-1 P. Fire hydrant 1. Street Furniture
(C) P-1,Q-1,R-2,S-2,T-2 Q. Plant beds 2. Street Hardware
R. Letter box
(D) P-2,Q-1,R-2,S-2,T-2
S. Traffic Sign
Lamp Posts
Hemant Vilas Parulekar 399

Ans:- (A)
Explanation:
Street Furniture:
Street furniture refers to various fixed elements or installations placed in public areas, mainly along
streets, sidewalks, and squares, to provide comfort, convenience, and amenities to pedestrians and the
general public.
Examples of street furniture include benches, public seating areas, picnic tables, waste bins, public art
installations, sculptures, bike racks, bollards (short vertical posts), and planters.
Street furniture aims to enhance the aesthetics and functionality of public spaces, making them more
inviting and user-friendly. It often contributes to the overall character and identity of an urban area.
Street Hardware:
Street hardware includes various functional components and equipment that serve specific purposes
related to transportation, traffic control, and urban infrastructure.
Examples of street hardware comprise traffic signs (such as stop signs, yield signs, and speed limit signs),
traffic signals (stoplights), streetlights (lamp posts), fire hydrants, utility boxes, and other elements
related to public safety, transportation, and utilities.
Street hardware plays a critical role in ensuring road safety, managing traffic flow, and providing essential
services like water supply and electricity to urban areas.
Q.44 In a construction project schedule, A is the first acitivity. Activities B & C follow A. Activity D
follows B & C. Activity E follows C. Activity F follows D & E.
Activity A B C D E F
Duration 3 2 5 6 5 3
(in days)
The critical time to complete the project will be
14 days (B) 16 days (C) 17 days (D) 20 days
Ans:- (C)
Solution: To determine the critical time to complete the construction project, we need to understand
the longest path through the project’s activities, also known as the critical path. This path determines the
total project duration. We follow these activities in sequence:
Activity A is the first activity.
Activities B and C follow A. They can be initiated after A is completed.
Activity D follows B and C. It can start after both B and C are completed.
Activity E follows C.
Activity F follows D and E. It can start after both D and E are completed.
Given the duration for each activity, we can calculate the total time for each path and identify the critical
path (the longest path).
400 GATE Architecture and Planning: Comprehensive Question Bank

Let’s calculate the time for each path:


Path A-B-D-F:
-A (3 days) + B (2 days) + D (6 days) + F (3 days)
Path A-C-D-F:
-A (3 days) + C (5 days) + D (6 days) + F (3 days)
Path A-C-E-F:
-A (3 days) + C (5 days) + E (5 days) + F (3 days)
The path with the longest duration will be the critical path, and its duration will be the critical time to
complete the project. Let’s calculate the duration of these paths.
The critical time to complete the construction project, determined by the longest path through the
project’s activities, is 17 days. Therefore, the answer is (C) 17 days.
Q.45 The maintenance cost of a building will be Rs. 2 lacs after 10 years. The annual sinking fund required
for such maintenance @ 6% interest per annum will be
(A) Rs. 17,200/- (B) Rs. 15,200/- (C) Rs. 13,200/- (D) Rs. 11,200/-
Ans:- (B)
Solution: To calculate the annual sinking fund required for future maintenance of a building, we can use
the formula for the sinking fund, which is based on the concept of the future value of an annuity. The
sinking fund formula determines how much money needs to be set aside each year to reach a certain
amount in the future, given an interest rate.
The formula for the sinking fund (annual deposit) is:
A= (F×i)/(1+i)n−1
Where:
-A is the annual sinking fund deposit.
-F is the future maintenance cost.
-i is the interest rate per period.
-n is the number of periods.
Given:
F=Rs. 2 lacs (Rs. 200,000)
i=6% per annum (0.06 as a decimal)
n=10 years
A = (2,00,000 x 0.06) / (1 + 0.06)10-1
The annual sinking fund required for the future maintenance of the building, calculated at 6% interest
per annum over 10 years, is approximately Rs. 15,174. Therefore, the closest match among the provided
options is (B) Rs. 15,200/-
Hemant Vilas Parulekar 401

Q.46 Match the figures in Group-I with the fixtures in Group-II


Group-I

Group-II
1. Sink Cock 2. Bib Cock 3. Pillar Cock 4. Stop Cock
(A) P-1, Q-4, R-2, S-3 (B) P-2, Q-3, R-1, S-4
(C) P-3, Q-1, R-2, S-4 (D) P-2, Q-4, R-3, S-1
Ans:-Marks to all (P-2, Q-4,R-1,S-3 is correct)
Q.47 Match the joints in Group-I with their corresponding figures in Group-II
Group-I
P. Butt joint Q. Rebated joint R. Table joint S. Tongue & Groove Joint

(A)P-3,Q-4,R-1,S-2 (B)P-4,Q-1,R-3,S-2
(C)P-3,Q-1,R-2,S-4 (D)P-3,Q-4,R-2,S-1
Ans:- (A)
Q.48 Common Data Questions
Common Data for Questions 48 and 49:
A beam of span L is simply supported at two ends. One half span of the building weighs W and the
remaining half spans weighs 2W.
Q.48 Maximum shear force in the beam will be
(A) W (B) 1.25W (C) 1.75W (D) 3W
Ans:- (C)
Solution:
To find the maximum shear force in a simply supported beam with an uneven load, we need to consider
the distribution of the load and the reaction forces at the supports.
Given:
The beam is simply supported at two ends.
One half of the span of the beam weighs W.
The remaining half weighs 2W.
402 GATE Architecture and Planning: Comprehensive Question Bank

The total load on the beam is W+2W=3W. This load is shared by the two supports. However, due to the
uneven distribution of the load, the reaction forces at the supports will not be equal.
To find the maximum shear force, we need to find the reaction forces at the supports first and then use
these to determine the shear force at any point along the beam. The maximum shear force will occur at
one of the supports, specifically the one with the larger reaction force.
Calculate Reaction Forces at Supports:
Let’s assume the reaction at the left support is RL and at the right support is RR.
Since the beam is in equilibrium, the sum of the moments about any point must be zero. We can take
moments about one of the supports to find the reaction at the other support.
Calculate Maximum Shear Force:
The maximum shear force will be equal to the larger of the two reaction forces.
The reaction forces at the supports are calculated as follows:
Reaction at the left support (RL) = 1.25W
Reaction at the right support (RR) = 1.75W
The maximum shear force in the beam will be equal to the larger of these two reaction forces. Therefore,
the maximum shear force in the beam is 1.75W.
Q.49 Maximum bending moment will occur at
(A) L/16 from midpoint of the beam (B) Midpoint of the beam
(C) l/7 from midpoint of the beam (D) One of the endpoints of the beam
Ans:- (A)
Common Data for Questions 50 and 51:
A building site has a plot of 500 sq m
Maximum allowable height – G+7 Area to be utilized for paved access roads – 10%
Maximum ground coverage – 40% Runoff coefficient for paved surface – 0.9
Maximum allowable FAR – 2.0 Runoff coefficient for unpaved surface – 0.3
Q.50 If maximum allowable FAR is utilized, the minimum ground coverage would be
(A) 20% (B) 25% (C) 30% (D) 35%
Ans:- (B)
Solution: Plot area = 500 sq.m
Maximum allowable FAR(FSI) = 2.00
Permissible built up area = 1000 sq.m.
10% area to be under paved roads = 50 sq.m.
Available ground area 450 sq.m.
Maximum ground coverage area 200 sq.m.
Total building of 8 floors, thus assuming equal area on each floor = 1000/8 = 125 sq.m.
Minimum ground coverage for full FAR utilization = (125/500) x 100 = 25%
Hemant Vilas Parulekar 403

Q.51 If it rains for 30 min., with intensity of 10 cm/hr, minimum volume of rain water that can be collected
will be
(A) 12.75 cum (B) 14 cum (C) 15 cum (D) 16 cum
Ans:- (A)
Solution: Time = 30min = 1/2 h
Intensity (I) = 10 cm/h = 0.1m/h
Area of paved surface (building + paved road) = 125 + 50 sq.m.
Area of unpaved = 325 sq.m.
As, Q = C x i x A (C = Runoff coefficient of material, i =peak intensity = 0.6118m/year
A = Area covered by the material
Q1 = 0.9 x 0.1 x 1.75 = 15.75 cu.m.
Q2 = 0.3 x 0.1 x 325 = 9.75 cu.m.
Q = Q1 + Q2 = 25.5 m3/h
Thus, rain water can be collected in 1/2 h = 25.5/2 = 12.75 cu.m.
Linked Answer Questions
Statements for linked Answer Questions 52 and 53:
An auditorium having volume of 4500 cu.m and total absorption of all acoustic materials is 480 m2
sabine
Q.52 The reverberation time of the auditorium is
(A) 1.0 second (B) 1.5 second (C) 2.0 second (D) 2.5 second
Ans:- (B)
Solution: The reverberation time of an auditorium can be calculated using Sabine’s formula:
T= (0.161×V)/A
Where:
T is the reverberation time in seconds.
V is the volume of the auditorium in cubic meters (cum).
A is the total absorption in the room in square meters Sabine (m² Sabine).
Given:
V=4500 cum.
A=480 m² Sabine.
T = (0.161 x 4500)/480 = 1.51 say 1.50 second
Q.53 To reduce reverberation time by 0.5 second, additional adsorption (m 2sabine) required
would be
(A) 120 (B) 160 (C) 240 (D) 720
404 GATE Architecture and Planning: Comprehensive Question Bank

Ans:- (C)
Solution: To calculate the additional absorption required to reduce the reverberation time by 0.5 seconds
in an auditorium, we can use Sabine’s formula for reverberation time:
T= (0.161×V)/A
where T is the reverberation time in seconds, V is the volume of the auditorium in cubic meters, and A
is the total absorption in square meters Sabine.
We know the current reverberation time and the desired reduction, so we can solve for the new total
absorption required to achieve this reduced reverberation time.
Calculate Current Reverberation Time:
V=4500cum
A=480m2Sabine
Current T = (0.161×4500)/480
Calculate Desired Reverberation Time:
Desired reduction in T is 0.5 seconds.
New T = Current T−0.5
Calculate New Total Absorption:
Use Sabine’s formula to find the new A.
Calculate Additional Absorption Required:
Additional absorption = New −A− Current A
Let’s calculate the additional absorption needed.
The additional absorption required to reduce the reverberation time by 0.5 second in the auditorium is
approximately 237.77 m² Sabine.
Statements for linked Answer Questions 54 and 55:
A residential sector planned over an area of 100 hectare has been divided into various plots, each
having one dwelling unit with an average household size of 5 persons. Remaining area is devoted for
schools. Roads, parks, shops etc.
Plot size Number
500 sqm 500
300 sqm 500
200 sqm 1000
Q.54 The gross density of the residential sector in persons per hectare would be
(A) 100
(B) 150
(C) 200
(D) 250
Hemant Vilas Parulekar 405

Ans:- (A)
Solution:
Total Population Calculation:
-Average household size is 5 persons.
-We have plots of different sizes with different numbers of dwelling units.
1. For each plot size:
500 sqm plots: 500 units.
300 sqm plots: 500 units.
200 sqm plots: 1000 units.
Total population = (Number of units) × (Average household size).
2. Gross Density Calculation:
Gross density = Total population / Total area.
Total area of the residential sector is 100 hectares.
Let’s calculate the total population and then the gross density.
The gross density of the residential sector is 100 persons per hectare.
Q.55 Assuming 20% of the total population being higher secondary school going children and expected
enrolment being 80% with per capita floor space requirement of 5.0 sqm, then minimum land
required for school building with 40% ground coverage and FAR 0.5 would be
(A) 1.0 Hectare (B) 1.6 Hectare (C) 2.2 Hectare (D) 2.8 Hectare
Ans:- (B)
Solution:
1. Calculate Total Population:
- 500 sqm plots: 500 plots × 5 persons/plot = 2500 persons
- 300 sqm plots: 500 plots × 5 persons/plot = 2500 persons
- 200 sqm plots: 1000 plots × 5 persons/plot = 5000 persons
- Total Population = 2500 + 2500 + 5000 = 10000 persons
2. Calculate Number of School-Going Children:
- School-Going Children = Total Population × 20% = 10000 × 0.20 = 2000 children
3. Calculate Number of Enrolled Students:
- Enrolled Students = School-Going Children × 80% Enrollment = 2000 × 0.80 = 1600 students
4. Calculate Total Floor Space Required for School:
- Total Floor Space = Enrolled Students × 5 sqm/student = 1600 × 5 = 8000 sqm
5. Calculate Land Required for Building (Considering FAR):
- FAR = Floor Area / Land Area
- Land Area = Floor Area / FAR = 8000 sqm / 0.5 = 16000 sqm = 1.60 Hectares
406 GATE Architecture and Planning: Comprehensive Question Bank

General Aptitude (GA) Questions


Q.56 – Q.60 carry one mark each.
Q.56 Choose the word from the options given below that is most nearly opposite in meaning to the given
word:
Amalgamate
(A) Merge (B) Split (C) Collect (D) Separate
Ans:- (A)
Q.57 Choose the most appropriate word from the options given below to complete the following sentence.
If you are trying to make a strong impression on your audience, you cannot do so by being
understated, tentative or _________________.
(A) Hyperbolic (B) Restrained (C) Argumentative (D) Indifferent
Ans:- (B)
Q.58 Choose the most appropriate word(s) from the options given below to complete the following
sentence.
I contemplated ______________ Singapore for my vacation but decided against it.
(A) To visit (B) Having to visit (C) Visiting (D) For a visit
Ans:- (C)
Q.59 If Log (P) = (1/2)Log (Q) = (1/3) Log (R), then which of the following options is TRUE?
(A) P2 = Q3R2 (B) Q2 = PR (C) Q2 = R3P (D) R = P2Q2
Ans:-(B)
Solution:
Log(P) = (1/2) * Log(Q)
Log(P) = (1/3) * Log(R)
Now, let’s use logarithm properties to simplify this:
Log(P) = Log(Q(1/2)) [Using the property Log(An) = n * Log(A)]
Log(P) = Log(R(1/3))
Since the logarithms are equal, we can set the expressions inside the logarithms equal to each other:
Q(1/2) = R(1/3)
Now, let’s raise both sides of this equation to the power of 6 (to eliminate the fractional exponents):
(Q(1/2))6 = (R(1/3))6
Q3 = R2
Q.60 Which of the following options is the closest in the meaning to the word below:
Inexplicable
(A) Incomprehensible (B) Indelible (C) Inextricable (D) Infallible
Ans:- (A)
Hemant Vilas Parulekar 407

Q.61 to Q. 65 carry two marks each.


Q.61 A container originally contains 10 litres of pure spirit. From this container 1 litre of spirit is replaced
with 1 litre of water. Subsequently, 1 litre of the mixture is again replaced with 1 litre of water and
this process is repeated one more time. How much spirit is now left in the container?
(A) 7.58 litres (B) 7.84 litres (C) 7 litres (D) 7.29 litres
Ans:- (D)
Solution:
To solve this problem, we’ll use the concept of successive replacement. Each time 1 litre of the mixture is
removed and replaced with water, a certain fraction of the original spirit is removed. Let’s calculate how
much spirit remains after each step.
1. Initial Quantity:
-The container initially contains 10 litres of pure spirit.
2. After First Replacement:
-1 litre of spirit is replaced with 1 litre of water.
-The amount of spirit remaining is 10−1=9 litres.
3. After Second Replacement:
-Now, the container has 9 litres of spirit and 1 litre of water.
-When 1 litre of this mixture is removed, 9/10 of it is spirit (since spirit makes up 9/10 of the mixture).
-The amount of spirit removed is 1× (9/10) = 9/10 litres.
-Therefore, the amount of spirit left is 9 − (9/10) litres.
4. After Third Replacement:
-The container now has 9− (9/10) litres of spirit.
-Again, when 1 litre of this new mixture is removed, (9−9/10)/(10−9/10) = (81/100) of it is spirit.
-The amount of spirit removed is 1× (81/100) = (81/100) litres.
-Therefore, the amount of spirit left is 9 − (9/10) − (81/100) litres.
Let’s calculate the final quantity of spirit left in the container after these three replacements.
After the three replacements, there are 7.29 litres of spirit left in the container.
Q.62 A transporter receives the same number of orders each day. Currently, he has some pending orders
(backlog) to be shipped. If he uses 7 trucks, then at the end of the 4th day he can clear all the orders.
Alternatively, if he uses only 3 trucks, then all the orders are cleared at the end of the 10th day. What
is the minimum number of trucks required so that there will be no pending order at the end of the
5th day?
(A) 4 (B) 5 (C) 6 (D) 7
408 GATE Architecture and Planning: Comprehensive Question Bank

Ans:- (C)
Solution:
Calculate Daily Order Capacity per Truck:
We know that 7 trucks can clear all orders in 4 days.
Therefore, the total capacity of 7 trucks over 4 days is 7×47×4 truckloads.
Similarly, 3 trucks can clear all orders in 10 days, so their total capacity over 10 days is 3×103×10
truckloads.
Set Up Equations for Backlog and Daily Orders:
Let B be the initial backlog of orders (in truckloads).
Let D be the daily number of orders received (in truckloads per day).
When using 7 trucks for 4 days, the total capacity is equal to the backlog plus 4 days of orders: 7×4=B+4D.
When using 3 trucks for 10 days, the total capacity is equal to the backlog plus 10 days of orders:
3×10=B+10D.
Solving the Equations:
7×4=B+4D => 28=B+4D.
3×10=B+10D => 30=B+10D.
Solving these two equations will give us the values of B and D.
Calculate the Minimum Number of Trucks for 5 Days:
To find out the number of trucks needed to clear the backlog and 5 days of orders, we calculate the total
orders (backlog + 5 days of orders) and divide it by 5 (the number of days), rounding up to the nearest
whole number since we can’t have a fraction of a truck.
Let’s perform these calculations step by step.
Here are the simplified calculation steps:-Solving for Backlog and Daily Orders:
e found that B=80/3 (backlog in truckloads) and D=1/3 (daily orders in truckloads).
Total Orders to Clear in 5 Days:
-The total orders to be cleared in 5 days are -
B+5D= (80/3)+5× (1/3)= 85/3 truckloads.
Minimum Number of Trucks for 5 Days:
To clear a total of 85/3 truckloads in 5 days, the number of trucks needed is [(85/3)/5] = 6.
Therefore, the minimum number of trucks required to ensure that there will be no pending order at the
end of the 5th day is 6.
Q.63 The variable cost (V) of manufacturing a product varies according to the equation V=4q, where q is
the quantity produced. The fixed cost (F) of production of same product reduces with q according
to the equation F = 100/q. How many units should be produced to minimize the total cost (V+F)?
(A) 5 (B) 4 (C) 7 (D) 6
Ans:- (A)
Hemant Vilas Parulekar 409

Q.64 P, Q, R and S are four types of dangerous microbes recently found in a human habitat. The area of
each circle with its diameter printed in brackets represents the growth of a single microbe surviving
human immunity system within 24 hours of entering the body. The danger to human beings varies
proportionately with toxicity, potency and growth attributed to a microbe shown in figure below:

A pharmaceutical company is contemplating the development of a vaccine against the most


dangerous microbe. Which microbe should the company target in its first attempt?
(A) P (B) Q (C) R (D) S
Ans:- (D)
Q.65 Few school curricula include a unit on how to deal with bereavement and grief, and yet all
students at some point in their lives suffer from losses through death and parting.
Based on the above passage which topic would not be included in a unit on bereavement?
(A) how to write a letter of condolence
(B) what emotional stages are passed through in the healing process
(C) what the leading causes of death are
(D) how to give support to a grieving friend
Ans:-(C)
GATE QUESTION PAPER 2010
General Aptitude (GA)

Q. 1 – Q. 25 carry one mark each.


Q.l Natural granite used for cladding in buildings belongs to the category of
(A) Igneous Rock (B) Acid Rock
(C) Sedimentary Rock (D) Metamorphic Rock
Ans:- (A)
Rock type Example
Igneous Granite, Basalt, Pumice, Obsidian, Rhyolite
Sedimentary Sandstone, Limestone, Shale, Conglomerate, Gypsum
Metamorphic Marble, Slate, Schist, Quartzite, Anthracite coal
Q.2 ‘Flying Buttress’ is an architectural element of
(A) Indian Architecture (B) Greek Architecture
(C) Gothic Architecture (D) Byzantine Architecture
Ans:- (C) (Please refer GATE-2014, Q.21, for more information.)
Q.3 A major hole in the Ozone layer has been identified above the
(A) Amazon Forest (B) Arctic Region (C) Savannah Grasslands (D) Sahara Desert
Ans:- (B)
Q.4 A flat arch at the skewback should NOT have an angle less than
(A) 30˚ (B) 45˚ (C) 60˚ (D) 90˚
Ans:- (C)

Q.5 Primary colour of natural light are


(A) Red, Blue, Yellow (B) Red, Green, Blue
(C) Red, Violet, Yellow (D) Red, Green, Yellow
Ans:- (B)
Hemant Vilas Parulekar 411

Q.6 Horizontal member of a shutter that subdivides a window is termed as


(A) Mullion (B) Transom (C) Reveal (D) Purlin
Ans:- (B)

Q.7 If the temperature of a composite bar made of copper and steel is raised, then the copper bar will
be under
(A) Tension (B) Compression (C) Shear (D) Torsion
Ans:- (A)
Explanation: When the temperature of a composite bar made of copper and steel is raised, the two
materials will expand at different rates due to their distinct coefficients of thermal expansion. Copper
has a higher coefficient of thermal expansion than steel, which means it expands more for a given
temperature increase.
As a result, the copper part of the composite bar will expand more than the steel part. This differential
expansion will cause the copper bar to push against the steel bar. Since the copper bar is expanding and
exerting a force outward, it creates a tensional stress within itself.
Q.8 E.I.A. stands for
(A) East India Association (B) Environmental Impact Audit
(C) Environment Impact in Asia (D) Environmental Impact Assessment
Ans:- (D)
Q.9 A steel truss with parallel upper and lower chords and inclined connecting members forming a series of
equilateral triangles is known as
(A) Bowstring Truss (B) Warren Truss
(C) Kingpost Truss (D) Scissors Truss
Ans:- (B)
Truss Bowstring Warren
Illustration

Truss Kingpost Scissors


Illustration
412 GATE Architecture and Planning: Comprehensive Question Bank

Q.10 In water supply systems, the ‘Reflux Valves’ allow water to flow
(A) In one direction only (B) In both directions
(C) Through air locked joints (D) only under low pressure
Ans:- (A)

Explanation: Reflux valve is also called as non-return valve.


1. To regulate fluid or gas movement, a solenoid valve uses an electric current to produce a magnetic
field that opens or shuts a valve mechanism.
Electromechanical solenoid valves may be employed in pneumatics, oxygen, vacuum, pool water, light
oils, and neutral gases. A core component is displaced within an electromagnetically charged coil to
start or stop fluid flow, controlling fluid movement. Valve types include 2-way and 3-way. They may run
normally closed or open.
Solenoid valves trigger hydraulic jacks, regulate vehicle hydraulic cylinders, and control water, oil, and
solvent pipe flow. These are also useful in pneumatic systems. The solenoid latching valve is designed to
secure an air valve without electricity.
2. One-way valve
A check valve inhibits reverse fluid flow in pipes using fluid pressure. The valve opens during fluid
advancement but closes automatically when flow changes. This valve set uses fluid dynamics to operate
without human or automated actuators.
Number 3. A ball valve controls fluid flow via a pipe using a ball-shaped disk.
Ball valves, known for their affordability and lightness, stop fluid or gas flow by rotating a ball 90°.
Pressure decrease and fluid accumulation are reduced by this design. However, costs climb with pipe
diameter, especially for DN150 and higher. The typical floating ball design contains stagnation zones
where medium may build.
Number 4. Gate valves use sliding gates to regulate fluid flow.
A gate valve controls liquid flow by vertically shifting a gate to open or close the pipe. Using a knife edge
ensures thorough sealing by cutting through excess material. Gate valves are flexible enough for many
needs. They are bigger, heavier, and costlier than ordinary valves. However, they seal viscous or polluted
fluids well.
Hemant Vilas Parulekar 413

Number 5. Pressure release valve


A Pressure Relief Valve (PRV) prevents system failures from excessive pressure. The pressure relief
valve (PRV) redirects fluid to guarantee safety when pressure exceeds the threshold. It is crucial to
avoiding equipment failure and risk. Relief valves are widely utilized, however they may fail to manage
overpressure or temperature. See our Pressure Relief Valves page for more.
Number 6. Slim, needle-shaped plunger type valve.
A needle valve controls low flow rates using a conical stopper. Air or hydraulic fluid is regulated using
these devices to reduce pressure changes. It withstands high pressure and temperature well. Its blockage
risk is a drawback.
Number 7. Plug valves regulate fluid flow by blocking or allowing passage via a cylindrical or tapered
plug.
Plug valves are rotary valves that spin 25% of the way. It regulates fluid flow using a tapered or cylindrical
stopper. This valve seals completely, making it excellent for vacuum systems with high pressure and
temperature. However, friction makes these gadgets difficult to use.
Number 8. Valve regulator
Process control systems need control valves to manage flow, temperature, and pressure. The gadget may
be powered by hand or electricity and has numerous configurations. Actuators receive process signals
to open or close valves. Use them anywhere in an electrical circuit, however they may cause vibration.
The number is 9. Butterfly valves regulate pipe fluid flow using discs.
Butterfly valves are cheap and easy to install. It controls liquid flow using a rotating disk. This device
controls low-pressure flow and may be throttled or turned off. It comes in several sizes. It avoids solids
and is flexible, but its flow control is limited.
The number is 10. Angle valves regulate pipeline fluid flow with a right angle bend in their bodies.
Angle valves combine pipe elbows with valves. Water flow is controlled by “angle-pattern globe valves”
that displace a plug or disk. They minimize piping system pressure loss and pipe fittings. Additionally,
they work with caustic fluids. However, oblique movement may reduce pressure, a negative.
The number is 11. A pinch valve regulates fluid flow by pinching or squeezing a flexible tube or sleeve.
Pinch valves are versatile on-off and throttling valves. Rubber or plastic sleeves are detachable. It’s cheap
and simple to maintain, making it ideal for slurries. An designed elastomer sleeve in the pinch valve
seals trapped particles well. Pinch valves aren’t vacuum-friendly. Additionally, pinch valves generate less
airborne pollution.
The number is 12. A knife gate valve controls fluid flow with a sharp blade.
Knife gate valves are suited for on-off and isolation functions in corrosive or abrasive environments with
large suspended particles. They are useful for wastewater treatment since they have a blade for cutting
viscous liquids. Using them for flow control may cause erosion and water hammer. Most designs include
a seal around the blade, making maintenance and operating difficult.
The number is 13. Safety release valve
414 GATE Architecture and Planning: Comprehensive Question Bank

Safety relief valves save lives and property. Since they prevent catastrophic pressure breakdown, they
must be absolutely reliable. Installation is common when system pressure may exceed the allowed level,
such as after equipment failure, cooling system failure, power loss, fires, or plant startup.
The number is 14. Globe valves use movable disks or plugs to regulate fluid flow via pipes.
Globe valves accurately control fluid flow and close completely. They are flexible and may be modified for
resurfacing. They have higher pressure differentials than other valves, hence they need larger actuators
to seal in high-pressure settings.
The number is 15. Valve isolation
To conduct maintenance or assure safety, an isolation valve stops fluid flow. It controls material
movement and connects to external devices. High-quality, pricey isolation valves may be unfeasible for
small businesses.
The number is 16. Slider valve
Steam engines employ sliding valves to control steam intake and discharge. This straight-line valve
controls steam well, optimizing engine efficiency. However, its utilization is confined to steam-engine
applications and not adaptable to other areas.
The number is 17. Check Valve
Non-return valves prevent pressure-reduced water backflow in thermostatic tapware, improving safety.
They assure one-way flow using gravity and springs. These valves cannot be checked in either open or
closed positions, are inappropriate for pulsing systems, and may cause water hammer.
The number is 18. Coaxial valve
Coaxial valves control substances using a horizontally movable 316 stainless steel hollow tube. O-ring
seals hold the tube in place, while lateral movement controls media flow by blocking or permitting it.
These devices seal well against back pressure, but they cannot tolerate large flow rates or diverse fluids.
The number is 19. Bottom valve for full container or vessel emptying.
Industrial drainage relies on flush bottom valves. These valves act during tank or container cleansing
or emptying. They handle liquids, slurries, and viscous materials well. Flush bottom ball valves provide
free liquid flow when there is inadequate room below a vessel, tank, or reactor. Design and orientation
modifications increase product flexibility.
The number is 20. Manage Valve
Pressure control is essential in pipelines. Pressure-regulating valves including maintaining, reducing, and
relieving valves keep conditions safe and efficient. These devices can automatically maintain pressures
and signal pneumatically. Filtering is needed to prevent contamination.
The number is 21. Special-purpose valve
Multi-port valves provide more than isolation and control. Multi-port valves use many ports for faster,
more flexible operation. They simplify design, reduce product mixing, and enable faster, more reliable
operations via inter-coupling. Their usage is restricted to certain applications.
The number is 22. Aerosol controls
Cans’ aerosol valves release their contents. Their essential components are the housing and stem. The
purpose, actuator, output, valve size, and materials utilized in its construction are important. The
distribution of media may also matter. Aerosol valves distribute liquids, lotions, ointments, gases,
cleaning agents, and other aerosol can contents.
Hemant Vilas Parulekar 415

The number is 23. Airlogic valves


Mechanical or electro-mechanical Air Logic Valves regulate pneumatic airflow. They may replace
electrical control in dangerous circumstances or when electrical control is not possible. The actuator
type, number of ports, construction materials, switching speed, port thread size, pressure ratings, and
input voltage are crucial. Pneumatic systems employ air logic valves for emergency stops, pilot valves,
one-shot valves, and more.
The number is 24. Fluid-balancing valves.
Balancing Valves evenly distribute fluid flow between branches to control flow. Port amount, connections,
valve size, and construction materials are important. HVAC and fluid power systems employ balancing
valves. They may adjust water temperatures in commercial heating/cooling systems to match loads.
They can also compensate for double-acting cylinders.
The number is 25. Valves that block or isolate fluid or gas flow without visual examination.
Line blind valves stop fluid flow in pipelines. Oil and gas companies utilize them to segregate pipeline
portions. Some call these valves Piping Blinds. Valve type, actuator type, port connections, valve size,
and valve body, seat, seal, and lining material are important. Blind valves are used on ships and offshore
projects. These gadgets quickly indicate whether a pipe is open or closed. They segregate pipeline
portions for repair.
The number is 26. Cartridge valves
Cartridge valves control hydraulic and pneumatic fluid flow. Their cartridge shape allows them to be
put into ordinary manifolds, saving weight and money over valve installation. The intended purpose,
valve type, actuator type, number of ports, valve size, and valve body, seat, seal, lining, and stem packing
materials are crucial. Cartridge valves are ideal for all conventional fluid power applications that use
hydraulic or pneumatic valves, such as check valves, directional control valves, flow control valves, logic
valves, pressure control valves, motor control valves, etc.
The number is 27. Enclosure valves
Oil and gas companies utilize casing valves to access well casings. Purpose, actuator type, port
connections, valve size, and manufacturing materials are crucial.
The number is 28. Christmas tree valves
Mechanical Christmas Tree Valves control well or system flow. The intended application, port count,
pressure, and temperature specifications are crucial. Christmas tree valves are usually attached to the
wellhead of oil and gas wells to control or stop flow. They are usually custom-made.
The number is 29. Control valves for fluid flow
Cock valves empty tanks and containers and include a threaded mechanism for opening and closing.
Low-pressure shut-off valves with quarter-turn levers are also used. Key characteristics include valve
type, port connections, valve size, and manufacturing materials. Cock valves are used in radiators,
heaters, tanks, boilers, laboratory glassware, air systems, drums, and more.
The number is 30. Diaphragm valves
Diaphragm valves block pipe flow using flexible membranes. Similar to pinch valves, the diaphragm
separates the actuation mechanism from the process fluid, making it ideal for sanitary valves.
Main parameters include port arrangement, connections, valve size, medium, and seal material.
Pharmaceutical, cosmetic, food, and semiconductor industries utilize diaphragm valves most. Control
valves with pneumatic diaphragms are also called “diaphragm valves”. Readers must understand this
distinction.
416 GATE Architecture and Planning: Comprehensive Question Bank

The number is 31. Disc valves.


Mechanical disc valves regulate conduit fluid flow. A disc valve has a circular, flat plate on a stem that
is put into the pipe at a 45-degree angle to its longitudinal axis. Rotating the stem 180 degrees opens or
closes the pipe. Most disc valves are used in food processing. Valve type, actuator type, port connections,
valve size, and construction materials are important. In the food, pharmaceutical, and dairy industries,
disc valves close liquid, powder, or food slurries to preserve cleanliness.
The number is 32. Double-block-bleed valves.
Mechanical or electromechanical double block and bleed valves feature two inline blocker valves and
one bleeder valve in a common valve body. These valves isolate fluid lines from upstream pressure.
The intended usage, actuator type, port connection type, flow coefficient, medium, pressure rating,
and characteristics are crucial. Process control applications employ double block and bleed valves to
segregate upstream pressure and release fluid and pressure. A manual or electro-mechanical actuator
may control them. Water, chemicals, gases, oil, steam, and similar fluids are media.
33. Engine valves
Engine valves seal combustion chambers to intake or exhaust systems. Important factors include
intended usage, head and stem size, and material composition. Cams and springs control engine valves.
They come in different materials and types for automobiles, trucks, motorcycles, etc. Racing-specific
designs are also available.
The number is 34. Control valves for faucet water flow.
Faucet valves control water flow into basins and sinks without outlets. Some include hose bibbs or
spigots for attaching a hose. The valve type, actuator type, port connections, valve size, and valve body
materials—seat, seal, lining, and stem packing—are the main requirements. Mounting style is also
important.
Laboratory, drum, and hose bibb faucet valves may be made from low-cost materials and discarded after
use.
The number is 35. Valves that float on liquid to regulate fluid flow.
Float valves control fluid inlets using hollow spheres or various shapes on levers or tracks. Tank fluid
levels are usually controlled via float valves. The intended usage, port connections, valve size, float size,
and valve body, seal, and float materials are crucial. After a flush, toilets and other tank-level control
systems employ float valves to refill water.
The number is 36. Valves hydraulic
Electromechanical or mechanical hydraulic valves control fluid flow in hydraulic fluid power systems.
They are electrically operated in permanent systems but manually operated in mobile systems. Valves,
actuators, port connections, number of ports, port design, construction materials, and pressure ratings
are critical. Construction equipment like backhoes and loaders and stationary equipment like balers and
presses employ hydraulic valves.
The number is 37. Piston valves
Piston valves control conduit fluid flow. Piston valves isolate by blocking flow using a cylindrical block.
Important valve characteristics include valve size, fluid flow connections, and valve body composition,
including seat, seal, lining, and stem packing materials. Piston valves isolate liquid systems like steam
and condensate.
Hemant Vilas Parulekar 417

The number is 38. Poppet valves


Mechanical or electro-mechanical poppet valves regulate pneumatic cylinder airflow. A check valve is
called a “poppet” also. Poppet valves are engine valves. Valve type, size, materials, flow coefficient, and
pressure ratings are important. Pneumatic poppet valves may be controlled by pilot air or solenoid.
The number is 39. Hoppers and rotary valves
Rotary valves, often called rotary airlocks, are mostly used to release powders and other dry, readily
flowing material. Hopper valves are used to dispense dry products from hoppers and other dry storage
containers.
Q.11 In Islamic architecture, the circular dome was constructed over a square configuration through
(A) Grid Iron Coffered Slab (B) Pendentives and Squinch Arches
(C) Double Barrel Vaults and Jack Arches (D) Horizontal Cross Tie Members
Ans:- (B) (Please refer GATE-2007, Q.19 for more information.)
Q.12 With respect to energy conservation and cost efficiency, the nature of an ideal built form should be
(A) High Rise Low Density (B) Medium Rise High Density
(C) Low Rise High Density (D) Low Rise Low Density
Ans:- (C) (Source: ECBC code 2007)
Q.13 Two default sequences P and Q are given below:
P: Specify height of extrusion or (Path]: 50
Specify angle of taper for extrusion <0>:
Q:Specify height of extrusion or [Path]: p
Select extrusion path or [Taper angle]:
The above mentioned sequences P and Q respectively, belong to
(A) 2D AutoCAD and 2D AutoCAD (B) 2D and 3D AutoCAD
(C) 3D and 2D AutoCAD (D) 3D AutoCAD and 3D AutoCAD
Ans:- (D)
Q.14 When shear stress exceeds the permissible limit in a RCC slab, then this problem is solved by
(A) Increasing the slab depth (B) Providing shear reinforcement
(C) Using high strength steel (D) Using thinner bars but more in number
Ans:- (A)
Q.15 Considering the total heat losses from all fluorescent lamps to be 79%, the Heating load (Btu/hr) due
to office illumination with 48 ceiling mounted luminaires, each containing four 40 W fluorescent
lamps and flat surface diffuser will be
(A) l0000 Btu/hr (B) 15000 Btu/hr
(C) 17500 Btu/hr (D) 21000 Btu/hr
418 GATE Architecture and Planning: Comprehensive Question Bank

Ans:- (D)
Solution:
Calculate the total power consumption of the fluorescent lamps:
Each luminaire contains four 40 W fluorescent lamps.
So, each luminaire consumes 4 lamps * 40 W/lamp = 160 W.
Calculate the total power consumption for all luminaires:
There are 48 luminaires.
Total power = 48 luminaires * 160 W/luminaire = 7,680 W.
Calculate the heating load considering the heat losses:
The heat losses from all fluorescent lamps are given as 79% of the power consumption.
Heat loss = 79% of 7,680 W = 0.79 * 7,680 W = 6,067.2 W.
Convert the heating load to Btu/hr (1 Watt = 3.412 Btu/hr):
Heating load = 6,067.2 W * 3.412 Btu/hr/W = 20,705.66 Btu/hr.
So, the heating load due to office illumination is approx. 20,706 Btu/hr. = 21,000 Btu/hr.
Q.16 Prime resultant forces that develop in a structure due to an earthquake depend on
(A) Mass and Surface Area of structure (B) Surface Area and Stiffness of structure
(C) Stiffness and Mass of structure (D) Surface Area and Volume of structure
Ans:- (C)
Q.17 Concept of ‘Serial Vision’ has been applied to the approach layout of
(A) Victoria Memorial Complex, Kolkata (B) Umaid Bhawan Palace. Jodhpur
(C) Vidhan Sabha Precinct, Bangalore (D) Rashtrapati Bhawan Complex, New Delhi
Ans:- (D)

Gordon Cullen’s “Serial Vision” is a concept in urban design and architecture that emphasizes the
sequential and evolving experience of spaces and places within a city or urban environment. It suggests
that as people move through urban spaces, they should experience a series of changing views, spaces,
and atmospheres that engage their senses and create a sense of anticipation and discovery.
Cullen argued that urban design should consider the visual and experiential qualities of a city, taking into
account how people perceive and interact with their surroundings over time. Serial Vision encourages
the creation of a narrative or a visual sequence within the urban fabric, making urban spaces more
engaging and memorable.
This concept has had a significant influence on urban design theory and has been used to guide the
planning and development of cities and public spaces with an emphasis on creating visually compelling
and experientially rich environments.
Hemant Vilas Parulekar 419

Q.18 Advanced Traffic Lane Information is an important feature of


(A) Para Transit system (B) Intelligent Transportation Systems
(C) High Level Cable Car system (D) Pedestrian Travellator system
Ans:- (B)
Q.19 A local authority can go for Urban Development through
(A) Land Acquisition (B) Land Pooling
(C) Transferable Development Rights (D) All the above
Ans:- (D)
Q.20 The Planning document submitted for the selected cities under JNNURM is
(A) Master Plan (B) Basic Development Plan
(C) City Development Plan (D) Outline Development Plan
Ans:- (C)
Explanation: A City Development Plan (CDP) is a strategic roadmap for a city’s long-term growth and
development. It covers land use, infrastructure, housing, transportation, environment, and more. It sets
goals, assesses existing conditions, and outlines strategies for sustainable urban development. CDPs
involve public input, have legal implications, and guide a city’s future.
Q.21 Excessive tilt of the Leaning Tower of Pisa has been checked by
(A) Pumping cement concrete mix under the dipping foundation
(B) Relocating heavier furniture to the rising side of the tower
(C) Raising the dipping side by massive Jack Screws
(D) Pumping out mud and slurry from the foundation base of the rising side
Ans:- (C)

Q.22 The Pritzker Prize 2009 has been awarded to


(A) Zaha Hadid (B) Peter Zumthor
(C) Jean Nouvel (D) Norman Foster
Ans:- (B)
420 GATE Architecture and Planning: Comprehensive Question Bank

Q.23 The age of a tree is determined by


(A) Counting the number of rings in the stem cross section
(B) Counting the number of leaves on the main branches
(C) Measuring the height of the tree from the rootball
(D) Measuring the canopy circumference of the tree
Ans:- (A)

Q.24 Nakagin Capsule Tower, Tokyo famous for its spatial modular approach was designed by
(A) Arata lsozaki (B)Tadao Ando
(C) Kisho Kurokawa (D) Minoru Yamasaki
Ans:- (C) (Please refer GATE-2012, Q.42 for more information)
Q.25 Proportioning system used in the layout of Mughal Gardens is derived from
(A) Rational number system (B) Constants of equilateral triangle
(C) Irrational number system (D) Constants of right angled isosceles triangle
Ans:- (A)
Q.26 - Q.55 carry two marks each.
Q.26 Match the cities in Group I with their form in Group II
(A) P-1, Q-4, R-3, S-2 Group I Group II
(B) P-2, Q-1, R-3, S-4 P. Detroit 1. Star Form
(C) P-5, Q-1, R-2, S-3 Q. Copenhagen 2. Polycentric Net
R. Stalingrad 3. Linear City
(D) P-4, Q-3, R-1, S-5
S. San Fransisco 4. Ring form
5. Galaxy
Ans:- (A)
City Detroit Copenhagen
Urban Form Star form Ring form
Illustration

City Detroit Copenhagen


Urban Form Star form Ring form
Illustration
Hemant Vilas Parulekar 421

Q.27 Match the visionaries in Group I with their concepts in Group II


(A) P-5, Q-4, R-3, S-2 Group I Group II
(B) P-5, Q-4, R-2, S-1 P. Clarence A. Perry 1. Post Modernism
(C) P-l, Q-3, R-2, S-1 Q. Constantinos Doxiadis 2. Bauhaus
R. Paul Davidoff 3. Advocacy Planning
(D) P-2, Q-4, R-2, S-5
S. Walter Gropius 4. Dynopolis
5. Neighborhood Unit
Ans:- (A)
Q.28 Match the trees in Group I with their botanical names in Group II
(A) P-1, Q-2, R-3, S-4 Group I Group II
(B) P-3, Q-4, R-5, S-1 P. Neem 1. Cassia Fistula
(C) P-2, Q-1, R-4, S-5 Q. Amala 2. Azadirachta Indica
R. Pipal 3. Ficus Bengalensis
(D) P-2, Q-3, R-5, S-4
S. Asoka 4. Ficus Religiosa
5. Saraca Indica
Ans:- (C)
Q.29 Following graphs represent the relationship between city size (in terms of population) on X-axis
and area under residential use (in percent) on Y-axis. Identify the correct graph.

Ans:- (D)
Q.30 Global climate change is expected to bring about a combination of the following changes. Identify
the correct combination.
P. Increase in Biodiversity Q. Emergence of New Diseases
R. Loss of Biodiversity S. Loss of all Rocky terrain
T. Sea Level Rise U. Extinction of Polar Bears
V. Emergence of New Islands
(A) P, Q, R, S (B) Q, R, T, U (C) R, T, U, V (D) Q, R, U, V
Ans:- (B)
422 GATE Architecture and Planning: Comprehensive Question Bank

Q.31 Annual housing demand of a metropolitan city is estimated through the combination of the
following components. Identify the correct combination.
P. New Entrants to the City Q. Elderly Population
R. New Relocated Slum Dwellers S. Slum Squatter Dwellers
T. Unauthorized Dwelling Units U. Dilapidated Houses
V. Pan of Backlog W. Any Other Houses
(A) P, R, T (B) U, S, Q (C) W, Q, T (D) P, U, V
Ans:- (D)
Q.32 A square pin jointed truss is subjected to a load P. acting in the direction
of member US, at joint U. The force in member UR is
1.414 P
1.000 P
0.707 P
(D) 0.000 P

Ans:- (C)
Solution: Force in member UR = P x cos450 = 0.70P
Q.33 Given below is the sketch plan of a site showing contours. The broken lines show valleys and ridges.
Identify the ridges and valleys.

(A) Ridges: P, Q, R ; Valleys: S, T (B) Ridges: T, V ; Valleys: R, T, U


(C) Ridges: S, U ; Valleys: Q, T, V (D) Ridges: R, V, U ; Valleys: P, T
Ans:- (C)
Hemant Vilas Parulekar 423

Q.34 From the following, identify the factors which influence the loudness of sound to a listener in an
enclosure
P. Loudness of sound at source
Q. Directivity factor
R. length/Width ratio of the enclosure
S. Distance between sound source and listener
U. Sound absorption co-efficient of all enclosing surfaces
V. Surface area of enclosing surfaces
Y. Inside temperature level of the enclosure
(A)P, Q, S, Y, V (B) Q, R, S, U, Y (C) P, R, S, U, V (D) P, Q, S, U, V
Ans:- (C)
Q.35 Match the lamps in Group I with their Colour Rendering Index (CRI) in Group II
(A) P-4, Q-2, R-l, S-3 Group I Group II
(B) P-3, Q-2, R-4, S-1 P. Mercury Vapour 1. 65-70
(C) P-2, Q-1, R-4, S-3 Q. Metal Halide 2. 40-55
R. High-pressure sodium 3. 20-25
(D) P-4, Q-3, R-2, S-1
S. Low-pressure sodium 4. 60-64

Ans:- (C)
Q.36 Match the terms in Group I with the architectural elements in Group II
(A) P-1, Q-3, R-2, S-4 Group I Group II
(B) P-2, Q-3, R -4, S-1 P. Tympanum 1. Auditorium Stage
(C) P-2, Q-1, R-4, S-3 Q. Proscenium 2. Door or Window Bands
R. Campanile 3. Circular House
(D) P-1, Q-2, R-3, S-5
S. Dymaxion 4. Church Tower
5. Horizontal Space for Services
Ans:- (C)
Term Tympanum Proscenium
Element of Door and windows bands Auditorium stage
Illustration
424 GATE Architecture and Planning: Comprehensive Question Bank

Term Campanile Dymaxion-


Element of Church Tower Circular House
Illustration

Q.37 Identify the most representative percentage distribution of landuse for a medium urban centre,
according to URDPFI guidelines, where
Residential= R, Commercial = C, Transport = T, Industry= I
(A) R=30%, C=20%, T=l2%, I=10%
(B) R=45%, C=04%, T=14%, I=08%
(C) R=30%, C=04%, T=14%, l=15%
(D) R=45%, C=10%, T=12%, I=10%
Ans:- (A) (Similar question has been seen in GATE-2004, Q. 43)
Q.38 If the area of a plot is 1000 sq.m, area of its adjoining roads is 500 sq.m.. maximum permissible FAR
is 150 and maximum permissible Ground Coverage is 50%, then utilizing fullest ground coverage
and assuming floors of equal area, the number of storeys that can be built on the plot is
(A) 6 (B) 4 (C) 3 (D) 2
Ans:- (C) 3
Solution: Given: Plot area = 1000 sq.m Adjoining roads area = 500 sq.m Maximum permissible FAR =
1.50 Maximum permissible Ground Coverage = 50%
Area of the net plot = 1000 sq.m - 500 sq.m = 500 sq.m
Maximum floor area = 1.50 * 500 sq.m = 750 sq.m
Maximum ground area = 50% of 500 sq.m = 250 sq.m
Assuming floors of equal area, let x be the number of storeys.
Area of each floor = 250 sq.m
Total floor area = x * 250 sq.m
Equating total floor area to maximum floor area:
x * 250 sq.m = 750 sq.m
Solving for x:
x = 750 sq.m / 250 sq.m = 3
Hemant Vilas Parulekar 425

Q.39 Match the buildings in Group I with their architects in Group II


(A) P-1, Q-2, R-3, S-4 Group I Group II
(B) P-2, Q-3, R -4, S-1 P. British Council Library, New Delhi 1. Hasmukh C. Patel
(C) P-2, Q-3, R-5, S-1 Q. Osho Commune Campus, Pune 2. Charles Correa
R. CII Sohrabji Godrej Green Business 3. Hafeez Contractor
(D) P -S, Q-4, R-3, S-2
Centre, Hyderabad
S. IIM New Campus. Ahmedabad 4. Karan Grover
5. Balkrishna V. Doshi

Ans:- (B)
Term British Council Library, New Delhi Osho Commune Campus, Pune
Element of Charles Correa Hafeez Contractor
Illustration

Term CII Sohrabji Godrej Green Business IIM New Campus. Ahmedabad
Centre, Hyderabad
Element of Karan Grover Auditorium stage
Illustration

Q.40 The age-sex compositions of three communities are represented by the diagrams P, Q and R as shown
below.

Each of them implies a strong socio-economic characteristic as indicated below.


1. Aging community 3. Multi ethnic community
2. Economically vibrant community 4. Young community with high birth rate
Identify the correct set out of the following;
(A) P-4,Q-3,R-1 (B) P-2.Q-4.R-3 (C) P-2,Q-4,R-1 (D) P·4.Q-2.R-3
Ans: (C) (Please refer GATE-2005, Q.44 for more information)
426 GATE Architecture and Planning: Comprehensive Question Bank

Q.41 The correct requirements provided to seek permission from the local authority for constructing a
small residential building are
P- Key Plan Q- Site Plan R - Zonal Plan
S - Building Plan T - Power of Attorney U- Ownership Title
V- Transport Plan W - Drainage/Sewerage/Water Supply Plan X - Solid waste disposal plan
(A)P,Q.R,S,W (B)P,Q,S,U,W (C) P.S,V,W.X (D)Q,S,T,V,X
Ans:- (B)
Q.42 Two commands P and Q, in AutoCAD are given below.
P: Current settings: Mode= TRIM. Radius= 0.0000
Select first object or [Polyline/radius/trim/multiple]:
Q: (TRIM mode) Current chamfer Dist. = 0.0000. Dist2 = 0.0000
Select first line or (Polyline/distance/Angle/Trim/Multiple/Method)
The above mentioned commands are used for
(A) P: Trim and Q: Trim (B) P: Fillet and Q: Trim
(C) P: Fillet and Q: Chamfer (D) P: Trim and Q: Chamfer
Ans:- (A)
Q.43 A ‘T- beam slab’ is cast and cured. The shuttering has to be removed. The right sequence for removal
of shuttering is
(A) Base of beam --> Sides of beam--> Base of slab-->Vertical support Wider beam
(B) Base of slab--> Sides of beam--> Base of beam -->Vertical support under beam
(C) Base of slab -->Sides of beam-->Vertical support under beam-->Base of beam
(D) Base of beam--> Base of slab--> Sides of beam-->Vertical support under beam
Ans:- (C)
Explanation: Correct sequence of the removal of shuttering:
1. Base of slab: The base of the slab is the first part of the shuttering to be removed. This is because the
slab is the least likely to collapse once the shuttering is removed.
2. Sides of beam: Once the base of the slab has been removed, the sides of the beam can be removed.
The sides of the beam are not as critical as the base of the slab, but they still need to be removed carefully
to avoid damaging the beam.
3. Vertical support under beam: The vertical support under the beam is the next part of the shuttering
to be removed. This support is important for holding up the beam while the concrete cures.
4. Base of beam: The base of the beam is the last part of the shuttering to be removed. This is because
the base of the beam is the most critical part of the shuttering, and it needs to be supported for as long
as possible to ensure that the beam does not collapse.
Q.44 Bio-climatic chart developed by Victor Olgyay shows the relationship between
(A) Temperature and Precipitation (B) Relative humidity and Precipitation
(C) Air Movement and Temperature (D) Temperature and relative humidity
Hemant Vilas Parulekar 427

Ans:- (D)
Explanation: The bio-climatic chart developed by Victor Olgyay is a tool for designing buildings that
are comfortable in their natural environment. The chart shows the relationship between temperature
and relative humidity, and it can be used to determine the appropriate passive heating and cooling
strategies for a given climate.
The other options are incorrect because:
- Temperature and precipitation are not directly related to each other.
- Relative humidity and precipitation are related to each other, but they are not the only factors that
affect comfort.
-Air movement and temperature are related to each other, but they are not the only factors that affect
comfort.
Q.45

In a display window of height H = 8.66 m. of a retail store, a luminaire of intensity L is mounted at


a distance L = 5 m away from the rear. Its light beam is cast at an angle of 450 from the ceiling, as
shown in the figure alongside.
The ratio of illumination at points P1, and P 2
1:3-2
3-2:1
2-2:1
1:2
Ans:- (C)
Solution: E = (l x cos ø / d2) --- (Lambert’s Cosine Law)
Where E = Illumination at a point
I = Luminous intensity of lamp
D = distance between the source and point
Ø = angle between the source and point
Illunimation P1:P2 = [(l x cos 450 / d2)] / [(l x cos 450 / d2)]
= (1/2-2) / (1/2) = 2-2 : 1
428 GATE Architecture and Planning: Comprehensive Question Bank

Q.46

Following figure shows network for a particular project consisting of four activities.
Normal duration and crash time for each activity are given below.

9 days (B) 13 days (C) 14 days (D) 19 days


Ans:- (A)
Q.47

Pick the ODD one from the figures given below


with respect to Reflection and Transmission of light.
(A)P (B)Q (C)R (D)S
Ans:- (A) (Answer provided as given in GATE official answer key)
Common Data Questions
Common Data for Questions 48 and 49:
A simply supported beam PQ is subjected to a load of 100 kN through a rigid link at the centre of the
beam as shown in the figure below
Hemant Vilas Parulekar 429

Q.48 Correct shear force diagram for the beam is

Ans: (D)
Q.49 Bending moment diagram for the above beam is

Ans:- (A)
Common data for questions 50 and 51:
A plot of land is to be developed as a residential neighbourhood. The key development conditions
and project requirements are given below
430 GATE Architecture and Planning: Comprehensive Question Bank

Q.50 The total number of dwelling units under L.I.G., M.I.G. and H.I.G. respectively, are

(A) P (B) S (C) Q (D) R


Ans:- (A)
Q.51 With the above data the covered area of each flat (in sq.m.) under LIG, MIG and HIG respectively
are

(A) Q (B) S (C) R (D) P


Ans: A
Linked answer questions
Statement for linked answers questions 52 and 53.
A person has purchased an old building at a cost of Rs. 2,50,000/-, excluding the cost of land, The
scrap value of the building is 10% of the cost of purchase and the future life of the building is 20
years.
Hemant Vilas Parulekar 431

Q.52 The total amount of sinking fund at the end of 20 years would be
(A) 1,35,000/- (B) 1,90,000/- (C) 2,22,500/- (D) 2,30,000/-
Ans: (C)
Solution: Given: Purchase cost of the building = Rs. 2,50,000/-
Scrap value of the building = 10% of purchase cost = 10/100 * 2,50,000/- = Rs. 25,000/-
Future life of the building = 20 years
To find:
Total amount of sinking fund at the end of 20 years
Formula:
Sinking fund = (Purchase cost - Scrap value) / Future life of the building
Calculation:
Sinking fund = (2,50,000/- - 25,000/-) / 20 years Sinking fund = Rs. 2,25,000/- / 20 years Sinking fund =
Rs. 11,250/- per year Total sinking fund at the end of 20 years = 11,250/- per year * 20 years Total sinking
fund at the end of 20 years = Rs. 2,25,000/-
Therefore, the total amount of sinking fund at the end of 20 years would be Rs. 2,25,000/-.
Q.53 If the rate of interest is 7%, then the annual instalment of sinking fund will be
Rs. 4,583/- (B) Rs. 4,855/- (C) Rs. 5,507/- (D) 5,640/-
Ans:- (C)
Solution: S = 2,25,000, n = 200 years, i = 7% (0.07)
Thus, I = (S x i) / (1 + i) - 1
I = (2,25,000 x 0.07) / [(1 + 0.07)2 -1]
I = 5,477/- say 5,507/-
Statement for linked answer questions 54 and 55:
A standpipe system in a 39m tall building has a rooftop reservoir for firefighting containing 1/2-
hour water supply.
Q.54 Assuming that each floor has one hose, the delivery rate (in litre per second) of the fire hose at
greatest pressure is
(A) 75 (B) 78 (C) 81 (D) 84
Ans:- Cannot be decided/insufficient data.
Q.55 The volume of water (in cubic m.) required for the reservoir is
(A) 105 (B) 120 (C) 110 (D) 130
Ans:- Cannot be decided/insufficient data.
432 GATE Architecture and Planning: Comprehensive Question Bank

General Aptitude (GA) question


Q.56-Q.60 carry one mark each.
Q.56 25 persons are in a room, 15 of them play hockey, 17 of them play football and 10 of them play both
hockey and football. Then the number of persons playing neither or football is:
(A) 2 (B) 17 (C) 13 (D) 3
Ans:- (D)
Solution: To find the number of persons playing neither hockey nor football, you can use the principle
of set theory known as the Inclusion-Exclusion Principle.
Let: A = Set of people playing hockey.
B = Set of people playing football.
Given:
n(A) = 15 (number of people playing hockey)
n(B) = 17 (number of people playing football)
n(A � B) = 10 (number of people playing both hockey and football)
n(U) = 25 (total number of people in the room)
You want to find the number of people playing neither hockey nor football, which is represented as n(A’
� B’). This is the complement of the union of those playing hockey and those playing football. So,
n(A’ � B’) = n(U) - [n(A) + n(B) - n(A � B)]
n(A’ � B’) = 25 - [15 + 17 - 10]
n(A’ � B’) = 25 - 22
n(A’ � B’) = 3
So, there are 3 persons playing neither hockey nor football.
Q.57 Choose the most appropriate word from the options given below to complete the following sentence:
If we manage to_______________ our natural resources, we would leave a better planet for our
children.
(A) Uphold (B) restrain (C) cherish (D) conserve
Ans:- (B)
Q.58 The question below consists of a pair of related words followed by four pair, or words. Select the pair
that best expresses that relation in the original pair.
Unemployed: Worker
(A) Fallow:land (B) unaware:sleeper (C) wit: jester (D) renovated: house
Ans:- (C)
Q.59 Which of the following options is the closest in meaning to the order below:
Circuitous
(A) Cyclic (B) indirect (C) confusing (D) crooked
Ans:- (B)
Hemant Vilas Parulekar 433

Q.60 Choose the most appropriate word from the options given below to complete the following sentence:
His rather casual remarks on politics _______ his lack of seriousness about the subject.
(A)Masked (B) belied (C) betrayed (D) suppressed
Ans:- (A)
Q.61-Q.65 carry two marks each.
Q.61 Hari (H), Gita (G), Irfan (I) and Saira (S) are siblings (i.e., brothers and sisters). All were born on 1st
January. The age difference between any two successive siblings (that is born one after another) is
less than 3years. Given the following facts:
Hari’s age + Gita’s age > Irfan’s age + Saira’s age.
The age difference between Gita & Saira is 1 year.
There are no twins.
In what order were they born (oldest first)?
(A) HSIG (B) SGHI (C) IGSH (D) IHSG
Ans:- (A)
Solution:
Work Rate of Skilled Workers:
5 skilled workers can build a wall in 20 days.
So, 1 skilled worker can build 1/5 of the wall in 20 days.
Therefore, the work rate of a skilled worker is 1/100 (wall per day)
Work Rate of Semi-Skilled Workers:
8 semi-skilled workers can build a wall in 25 days.
So, 1 semi-skilled worker can build 1/8 of the wall in 25 days.
Therefore, the work rate of a semi-skilled worker is 1/200 (wall per day)
Work Rate of Unskilled Workers:
10 unskilled workers can build a wall in 30 days.
So, 1 unskilled worker can build 1/10 of the wall in 30 days.
Therefore, the work rate of an unskilled worker is 1/300 (wall per day).
Total Work Rate of the Team:
2 skilled workers have a work rate of 2 × (1/100).
6 semi-skilled workers have a work rate of 6 × (1/200).
5 unskilled workers have a work rate of 5 × (1/300).
Total Work Rate: Sum of the rates of skilled, semi-skilled, and unskilled workers.
Time to Build the Wall: The reciprocal of the total work rate will give us the time required to build
the wall.
Let’s calculate the time it will take for this team to build the wall.
434 GATE Architecture and Planning: Comprehensive Question Bank

Total work rate team = (2 * work rate skilled) + (6 * work rate semi-skilled) + (5 * work rate unskilled)
= {[2 x (1/100)] + [6 × (1/200)] + [5 × (1/300)]}
= {[2/100] + [6/200] + [5/300]}
= 1/15
The team consisting of 2 skilled, 6 semi-skilled, and 5 unskilled workers will be able to build the wall in
exactly 15 days.
Q.62 5 skilled workers can build a wall in 20 days; 8 semi-skilled workers can build a wall in 25 days; 10
unskilled workers can build a wall in 30 days. If a team has 2 skilled, 6 semi-skilled and 5 unskilled
workers, how long will it take to build the wall?

Q.63 Modern warfare has changed from large scale clashes of armies to suppression of civilian populations.
Chemical agents that do work silently appear to be suited to such warfare; and regretfully, there exist
people in military establishments who think that chemical agents are useful tools for their cause.
Which of the following statements best sums up the meaning of the above passage?
Modern warfare has resulted in civil strife.
Chemical agents area useful in modern warfare.
Use of chemical agents in modern warfare would be undesirable.
People in military establishments like to use chemical agents in war.
Ans:- (A)
Q.64 Given digits 2, 2, 3, 3, 3, 4, 4, 4, 4 how many distinct 4 digit numbers greater than 3000 can be
formed?
(A) 50 (B) 51 (C) 52 (D) 54
Ans:- (B) 51
Q.65 If 137 + 276 = 435 how much is 731 + 672?
(A) 534 (B) 1403 (C) 1623 (D) 1513
Ans:- (A)
Since 137 + 276 = 435, 731 + 672 (the numbers have been reversed in this case.)
GATE QUESTION PAPER 2009
General Aptitude (GA)

Q. 1 – Q. 25 carry one mark each.


Q.1 The essential difference between CPM and PERT is
(A) Critical Path vs. Critical Activity
(B) Arrow notation vs. Precedence notation
(C) Deterministic approach vs. Probabilistic approach
(D) Project management vs. Network Analysis
Ans:- (C)
Explanation:- The essential difference between CPM and PERT is the deterministic approach vs.
probabilistic approach.
CPM (Critical Path Method) is a deterministic approach to project management. This means that it
assumes that the time it takes to complete each activity is known and fixed.
PERT (Program Evaluation and Review Technique) is a probabilistic approach to project management.
This means that it assumes that the time it takes to complete each activity has a range of possible values.
CPM is typically used for projects where the time it takes to complete each activity is well-known and
predictable. PERT is typically used for projects where the time it takes to complete each activity is
uncertain or has a wide range of possible values.
Q.2 The Minimum Thickness of wall where single Flemish bond can be used is
(A) Half-Brick thick (B) One-Brick Thick
(C) One-and-a-Half-Brick thick (D) Two-Brick thick
Ans:- (C)
Explanation:- Single Flemish bond is a type of brick bonding pattern in which there is one header and
one stretcher in each course. This type of bonding pattern requires a minimum thickness of wall of one
brick, so that the headers can be laid without protruding from the face of the wall.
Half-brick thick walls are too thin to accommodate single Flemish bond, and two-brick thick walls are
not necessary. Single Flemish bond is not as strong as other types of brick bonding patterns, such as
English bond or double Flemish bond, so it is not typically used in load-bearing walls. However, it is a
popular choice for decorative walls and walls that do not need to be very strong.
Q.3 On a colour wheel, the combination of Violet-Yellow’ or “Orange-Blue’ are best described as
(A) Complementary (B) Supplementary
(C) Analogous (D) Monochromatic
Ans:- (A) (Please refer GATE-2015, Q. 15 for more information)
Q.4 The sudden stoppage in the flow of water in a close conduit results in a phenomena called
(A) Cavitation (B) Hydraulic Gradient (C) Stack Pressure (D) Water Hammer
436 GATE Architecture and Planning: Comprehensive Question Bank

Ans:- (D)
Explanation:- Water hammer is a phenomenon that occurs when the flow of water in a pipe is suddenly
stopped. This can happen when a valve is closed quickly, or when a pump is turned off suddenly. When
the flow of water is stopped, the water in the pipe continues to move forward due to inertia. This causes a
wave of pressure to travel back down the pipe. The wave of pressure can be very high, and it can damage
the pipe and the equipment connected to it.
Cavitation is the formation of bubbles in a liquid when the pressure drops below the vapor pressure of
the liquid. Hydraulic gradient is the difference in pressure between two points in a fluid. Stack pressure
is the pressure at the top of a chimney or stack.
Q.5 The number of intersecting arches that support Bijapur’s Gol Gumbaj is
(A) 4 (B) 8 (C) 12 (D) 16
Ans:- (B)
Explanation:- The Gol Gumbaz is a mausoleum in Bijapur, India, that was built in the 17th century.
It is the largest dome in the Islamic world, and it is supported by eight intersecting arches. The arches
are made of brick and are held together by interlocking pendentives. The pendentives are triangular
structures that transfer the weight of the dome to the arches.
The eight arches are arranged in a square pattern, with each arch supporting a corner of the dome. The
arches are also connected to each other by a series of smaller arches. This system of arches helps to
distribute the weight of the dome evenly and prevents it from collapsing.
The Gol Gumbaz is a marvel of engineering, and it is a testament to the skill of the architects and
builders who constructed it.
Q.6 The 73rd and 74th Constitutional Amendments pertain to
(A) Abolishing Urban Land Ceiling Act
(B) Providing restricted role to rural courts to settle rural disputes
(C) Providing more responsibilities to Municipal and Local bodies for planning and development.
(D) Providing Right to information for the general people.
Ans: (C) (Please refer GATE-2014, Q. 8 for more information)
Q.7 A simply supported beam of length L carries a concentrated load of intensity P at its centre. The
bending moment at the centre of the beam will be
(A) PL/2 (B) PL/4 (C) PL/6 (D) PL/8
Ans:- (B)
Explanation:- he bending moment at the centre of a simply supported beam carrying a concentrated
load of intensity P at its centre is PL/4.
The bending moment at a point in a beam is equal to the product of the load at the point, the distance
from the load to the point, and the flexural rigidity of the beam. In this case, the load is P, the distance
from the load to the centre of the beam is L/2, and the flexural rigidity of the beam is EI. So the bending
moment is:
M = PL/2 * L/2 * EI = PL2/4EI
Hemant Vilas Parulekar 437

Q.8 Desire Lines’ are associated with


(A) Origin-Destination analysis is Transportation planning
(B) Income-Expenditure analysis in personal Finance Management
(C) Cut-Fill analysis in landscape planning
(D) Demand-Supply Analysis in economic planning
Ans:- (A)
Explanation:- Desire lines are associated with origin-destination analysis in transportation planning.
Desire lines are a graphical representation of the paths that people take when moving from one point
to another. They are used in transportation planning to understand the travel patterns of people and to
identify areas where improvements to the transportation system are needed.
Origin-destination analysis is a method of transportation planning that uses desire lines to estimate the
number of people traveling between different points in a region. This information can be used to plan
new roads, highways, and public transportation systems.
Income-expenditure analysis is used in personal finance management to track income and expenses.
Cut-fill analysis is used in landscape planning to determine the amount of material that needs to be
removed or added to a landscape in order to create a desired level of grade. Demand-supply analysis is
used in economic planning to determine the supply and demand for goods and services.
Q.9 GRIHA is rating for green building given by
(A) The Energy Research Institute (B) Development Alternatives
(C) Bureau of Energy Efficiency (D) Ministry of Power
Ans:- (A)
Explanation:- GRIHA is a green building rating system developed by The Energy and Resources
Institute (TERI). It is a national rating system that is used to assess the environmental performance
of buildings in India. GRIHA was launched in 2007, and it is the first green building rating system
in India to be accredited by the Indian Green Building Council (IGBC).
The GRIHA rating system assesses buildings on a number of criteria, including energy efficiency,
water conservation, indoor environmental quality, and sustainable site development. Buildings that
meet the GRIHA criteria are awarded a rating of one to five stars, with five stars being the highest
rating.
The GRIHA rating system is a voluntary system, and it is not mandatory for buildings to be certified.
However, many developers and builders are choosing to get their buildings certified by GRIHA, as it
is seen as a mark of quality and environmental responsibility.
Q.10 A ‘cul-de-sac’ is a street where
(A) Only two-wheelers are permitted
(B) Thorough Traffic is discouraged
(C) Pedestrians are not permitted
(D) Vehicles are permitted to move in one direction only
438 GATE Architecture and Planning: Comprehensive Question Bank

Ans:- (B)
Explanation:- A cul-de-sac is a street or lane that has a dead end. This means that there is no way to go
through the street without turning around. Cul-de-sacs are often used in residential areas to discourage
through traffic and to create a more peaceful and family-friendly environment.
Pedestrians are usually permitted to walk
through cul-de-sacs, but vehicles are typically
only permitted to enter and exit the cul-de-
sac. This is to prevent traffic from backing up
and causing congestion.

Q.11 ‘Usonian’ houses were designed by


(A) Mies van der Rohe (B) Alvar Alto
(C) Frank Lloyd Wright (D) Le Corbusier
Ans:- (C)

Q.12 Increase in volume of fine aggregate due to the presence of moisture is called
(A) Bulking (B) Buckling (C) Bending (D) Twisting
Ans:- (A)
Explanation:- Please refer GATE-2015, Q. 7 for more information.
Q.13 The Pattern language Theory was propounded by
(A) Christopher Alexander (B) Patrick Geddes
(C) John Ruskin (D) Amos Rapoport
Hemant Vilas Parulekar 439

Ans:- (A)
Explanation:- The Pattern Language Theory was propounded by Christopher Alexander.
Pattern language is a method of describing and solving problems in architecture, urban design, and
community livability. It was developed by Christopher Alexander in the 1970s, and it is based on the
idea that there are common patterns that can be used to create beautiful, livable, and sustainable places.
The Pattern Language consists of 253 patterns, each of which describes a problem and a solution. The
patterns are organized into five categories:
Building blocks: These patterns describe the basic elements of architecture, such as doors, windows,
and stairs.
Spaces: These patterns describe the different types of spaces that can be created in a building, such as
kitchens, bedrooms, and living rooms.
Relationships: These patterns describe the relationships between different spaces, such as the
relationship between a kitchen and a dining room.
Context: These patterns describe the context in which buildings are located, such as the relationship
between a building and its surroundings.
Transcendence: These patterns describe the qualities that make a place beautiful and livable, such as
harmony, mystery, and delight.
The Pattern Language has been influential in architecture and urban design, and it has been used to
create a number of successful projects. However, it has also been criticized for being too idealistic and
for being difficult to implement.
Q.14 As per IS 456:2000,the maximum area of tension reinforcement in a RCC beam shall not exceed x%
of its cross-sectional area, where x is equal to
(A) 2 (B) 4 (C) 6 (D) 8
Ans:- (B)
Explanation:- The maximum area of tension reinforcement in a RCC beam shall not exceed 4% of its
cross-sectional area, as per IS 456:2000.
This is to ensure that the beam is able to carry the tensile loads without failing. If the area of tension
reinforcement is too high, the beam may be susceptible to buckling.
The value of 4% is based on experimental data and engineering judgment. It is a conservative value that
ensures that the beam will be able to carry the design loads safely.
Q.15 ‘No-cut-no-fill lines are mostly used in
(A) Land Use Planning
(B) Interpretation of stereo-vision-photographs
(C) Earth work computation
(D) Interpretation of remotely sensed images
440 GATE Architecture and Planning: Comprehensive Question Bank

Ans:- (C)
Explanation:- No-cut-no-fill lines are lines that represent areas where there is no need for earthwork,
either cut or fill. They are typically used in earthwork computation to identify areas where the natural
ground surface can be used as the finished grade. This can save time and money on earthwork projects.
No-cut-no-fill lines can be generated from a variety of sources, including topographic maps, aerial
photographs, and LiDAR data. They can also be generated manually by surveyors or engineers.
Land use planning, interpretation of stereo-vision-photographs, and interpretation of remotely sensed
images are all important tasks in civil engineering, but they do not typically involve the use of no-cut-
no-fill lines.
Here are some additional details about no-cut-no-fill lines:
They are typically drawn on a site plan or grading plan.
They are often used in conjunction with other earthwork symbols, such as cut lines and fill lines.
They can be used to estimate the volume of earthwork required for a project.
They can be used to identify areas where erosion and sedimentation are likely to occur.
They can be used to design drainage systems for construction sites.
Q.16 The property of concrete measured by slump test is
(A) Durability (B) Hardness (C) Strength (D) Workability
Ans:- (D)
Explanation:- Workability is the ability of fresh concrete to be mixed, transported, placed, and compacted
without segregation or bleeding. It is important for concrete to be workable so that it can be used to
create a uniform and strong structure.
The slump test is a simple test that is used to measure the workability of concrete. In the slump test, a
fresh concrete sample is placed in a cone-shaped mold and then lifted. The amount of slump, or the
difference between the height of the concrete in the mold and the height of the concrete after it is lifted,
is measured. A higher slump indicates a more workable concrete, while a lower slump indicates a less
workable concrete.
The slump test is not a perfect measure of workability, but it is a quick and easy test that can be used to
get a general idea of the workability of concrete. Other tests, such as the flow test and the Vebe test, can
be used to get a more accurate measure of workability.
Q.17 The Remote Sensing satellite that gives the highest spatial resolution is
(A) IKONOS 2 (B) IRS IC/ID (C) Quickbird 2 (D) SPOT 5
Ans:- (C)
Explanation:
Satellite Panchromatic Resolution Multispectral Resolution
IKONOS 2 0.82 meters 3.2 meters
IRS IC/ID 5.8 meters Not specifically mentioned, but typically
coarser than panchromatic
Quickbird 2 0.61 meters 2.44 meters
SPOT 5 2.5 meters 10 meters
Hemant Vilas Parulekar 441

Q.18 Development that meets the needs of the present generation without compromising the ability of
future generations to meet their own needs is termed by UNDP as
(A) Comprehensive Development (B) Equitable Development
(C) Human Development (D) Sustainable Development
Ans:- (D)
Explanation: This definition is from the Brundtland Report (also known as “Our Common Future”),
which was published by the World Commission on Environment and Development (WCED) in 1987,
not specifically by UNDP. The WCED, chaired by Gro Harlem Brundtland, was convened by the United
Nations.
Q.19 The parameter that does not appear in Psychrometric chart is
(A) Wind Speed
(B) Dry Bulb Temperature
(C) Wet Bulb Temperature
(D) Relative Humidity
Ans: (A) (Please refer GATE-1996, Q.28 for more information)
Q.20 Allowable stress in the design of a tension member in a steel truss is a function of
(A) Cross-sectional area of the member
(B) Yield stress of the material
(C) Slenderness ratio of the member
(D) Moment of Inertia of the member’s cross-section
Ans:- (B)
Q.21 to Q.60 carry two marks each.
Q.21 The parameters for determining Human Development Index are
P. Education attainment
Q. Per capita Gross Agricultural Produce
R. Life Expectancy
S. Per Capita Gross Domestic Produce
T. Per Capita State Domestic Produce
(A) P,Q,S (B) P,Q,S,T (C) P,R,S (D) R,S,T
Ans:- (C)
Q.22 Match the individuals in Group I with the works in Group II
(A) P-4,Q-1,R-2,S-3 Group I Group II
(B) P-3,Q-1,R-2,S-5 P. Hippodamus 1. Aqueducts
(C) P-4.,Q-5,R-1,S-3 Q. Vitruvius 2. Campidoglio
R. Michelangelo 3. Hagia Sophia
(D) P-3,Q-4,R-1,S-2
S. Constantine 4. Agora
5. Hanging Garden
442 GATE Architecture and Planning: Comprehensive Question Bank

Ans:- (A)
Explanation:
Structure Aqueduct (Roman) Campidoglio (Roaman)
Description Vitruvius Michelangelo
View

Structure Hagia Sophia (Byzantine) Agora (Greek)


Description Constatine Hippodamus
View

Q.23 If the height of facade=h, and the distance of observer from the building=d, then match the enclosure
type in Group I with their corresponding h/d ratio in Group II.
(A) P-1,Q-2,R-3,S-4 Group I Group II
(B) P-4,Q-3,R-2,S-1 P. Full Enclosure 1. 1
(C) P-2,Q-3,R-4,S-1 Q. Threshold of Enclosure 2. 1/2

(D) P-5,Q-1,R-2,S-4 R. Minimum Enclosure 3. 1/3


S. Loss of Enclosure 4. 1/4
5. 1/5
Ans:- (A)
Experience Ratio h/d
Full Enclosure 1:1
Threshold of enclosure 1:3
Minimum Enclosure 1:2
Loss of enclosure 1:4
Hemant Vilas Parulekar 443

Q.24 The correct sequence of activities in Solid Waste Management is


(A) Collection->Transportation->Treatment->Segregation
(B) Segregation->Collection->Transportation->Treatment
(C) Collection->Segregation->Treatment->Transportation
(D) Treatment->Collection->Segregation->Transportation
Ans:- (B)
Q.25 The principles of Universal design include:
(P) Flexibility in use (Q) Tolerance for error (R) Energy Efficiency (S) Low Physical Effort
(A) P,Q,R (B) Q,R,S (C) P,R,S (D) P,Q,S
Ans: (D)
Q.26 Match the Urban Design elements in Group I with their descriptions in Group II
(A) P-3, Q-4, R-2, S-1 Group I Group II
(B) P-1, Q-4, R-2, S-3 P.District 1. Recognizable as having some common identifying character
(C) P-1, Q-2, R-4, S-3 Q.Landmark 2. Centre of activity
(D) P-2, Q-4, R-1, S-3 R.Node 3. Network of major and minor routes
S.Pathway 4. Prominent visual feature of the city

Ans:- (C)
Explanation:- Kevin Lynch’s seminal work, “The Image of the City” (1960), introduced key concepts
concerning how individuals perceive and organize spatial information as they navigate through cities.
Lynch identified five key elements that make up the city’s image in the minds of its inhabitants:
Paths: These are the channels or routes that people travel along, such as streets, walkways, transit
lines, and other circulation routes. Paths are crucial as they are the elements along which the observer
customarily moves and they usually have some linear order. For instance, a main street, a pedestrian
path, or a subway line.
Relation to Group II: Network of major and minor routes (S-3)
Edges: These are the boundaries and breaks in continuity, perceived as barriers or breaks in the urban
environment. Edges might be real barriers, like a shoreline or railroad tracks, or perceptual ones, like the
change from one neighborhood to another.
Not mentioned in the provided groups.
Districts: These are large areas with some common, identifying character. People tend to mentally
“enter” and “exit” districts as they move about the city. An example might be a financial district or
historic district in a city.
Relation to Group II: Recognizable as having some common identifying character (P-1)
Nodes: These are points of interest or activity that one can enter. Nodes might be junctions, places of
a break in transportation, a crossing or convergence of paths, moments of shift from one structure to
another. Examples include major street intersections, public squares, or transportation hubs.
Relation to Group II: Centre of activity (R-2)
Landmarks: These are external physical objects, usually a simply defined physical object, which serves
as an external reference point. They might be buildings (like a significant skyscraper), signs, or other
prominent features. They tend to be easily identifiable and may stand out from their surroundings.
Relation to Group II: Prominent visual feature of the city (Q-4)
444 GATE Architecture and Planning: Comprehensive Question Bank

Q.27 A commercial plot measuring 100m x 80 m. If the permissible Floor Space Index is 3.0 and 50% of
the ground is covered, then the maximum number of floors that can be built is
(A) 3 (B) 4 (C) 6 (D) 12
Ans:- (C)
Solution:- Calculating Total Permissible Built-Up Area:
FSI=Total built-up area/Plot area
Given that FSI is 3.0 and the plot area is 100 m×80 m
Total built-up area=FSI×Plot area
=3.0×100 m×80 m
=24,000 m2
Calculating Ground Coverage: Given that 50% of the ground is covered, the area of each floor will be:
Ground coverage=0.50×100 m×80
=4,000m2
Calculating the Maximum Number of Floors:
Number of floors=Total builtup area/Ground coverage
24,000 m2/4,000m2
=6
Q.28 Match the elements of a Buddhist Stupa in Group I with their traditional names in Group II
(A) P-2, Q-1; R-6, S-3 Group I Group II
(B) P-2, Q-6, R-4, S-3 P. Hemispherical Dome 1. Vedika
(C) P-3, Q-1, R-5, S-2 Q. Peripheral Railing 2. Anda
R. Entrance Gateway 3. Harmika
(D) P-5, Q-6, R-1, S-2
S. Portion above dome 4. Nagara
5. Chaitya
6. Torana
Ans: (A)
Hemant Vilas Parulekar 445

Q.29 A microwave oven of 3 kW rating is operated for 30 minutes, a hot water geyser of 1 kW rating is
operated for 15 minutes, and 5 fluorescent lamps of 60 W are operated for 6 hours. The total power
consumed in (kWh) will be
(A) 1.80 (B) 3.55 (C) 18.01 (D) 35.50
Ans:- (B)
Solution: To determine the total power consumed, we must first determine the power consumed by
each device and then sum them up.
Microwave Oven: Power rating = 3 kW Operated for
= 30 minutes = 0.5 hours Power consumed
= Power rating x Time of operation Power consumed by Microwave
=3kW×0.5hours=1.5kWhPower consumed by Microwave=3kW×0.5hours
=1.5kWh
Hot Water Geyser: Power rating = 1 kW Operated for = 15 minutes
= 0.25 hours Power consumed
= Power rating x Time of operation Power consumed by Geyser=1kW×0.25hours
=0.25kWhPower consumed by Geyser=1kW×0.25hours=0.25kWh
Fluorescent Lamps: Power rating for each lamp = 60 W = 0.06 kW Number of lamps
= 5 Total power rating = 0.06 kW x 5 = 0.3 kW Operated for = 6 hours Power consumed = Total power
rating x Time of operation Power consumed by Lamps=0.3kW×6hours=1.8kWhPower consumed by
Lamps
=0.3kW×6hours
=1.8kWh
Total power consumed = Power consumed by Microwave + Power consumed by Geyser + Power
consumed by Lamps Total power consumed=1.5kWh+0.25kWh+1.8kWh=3.55kWhTotal power
consumed=1.5kWh+0.25kWh+1.8kWh=3.55kWh
Q.30 Match the building projects in Group I with their architects in Group II
(A) P-6, Q-2, R-3, S-4
(B) P-1, Q-6, R-2, S-4
(C) P-6, Q-5, R-2, S-3
(D) P-2, Q-5, R-1, S-3
Group I Group II
P. National Olympic Stadium, Beijing 1.Rem Koolhas
Q. Glass Pyramid, the Louvre, Paris 2.Richard Rogers
R. Millennium Dome, London 3.Renzo Piano
S. Kansai Airport, Osaka 4.Tadao Ando
5. I M Pei
6. Herzog & de Meuron
446 GATE Architecture and Planning: Comprehensive Question Bank

Ans:- (C)
Structure National Olympic Stadium, Glass Pyramid, the Louvre, Paris
Beijing
Description Herzog & de Meuron I.M. Pei
View

Structure Millennium Dome, London Kansai Airport


Description Richard Rogers Renzo Piano
View

Q.31 Identify the prehistoric structures in the following:


P. Mastaba Q. Dolmen R. Menhir S. Pylon T. Stonehenge U. Thermae
(A) P,Q,R (B) R,T,U (C) Q,S,T (D) Q,R,T
Ans:- (D)
Structure Mastaba Dolmen
Time Period Egyptian Pre-historic
View
Hemant Vilas Parulekar 447

Structure Menhir Pylon


Time Period Pre-historic Egyptian
View

Structure Stonehenge Thermae


Time Period Pre-historic Roman

Q.32 Match the figures of cut bricks in Group I with their terms in Group II
Group I
1. King Closer 2. Queen Closer 3. Half Bat 4. Three Quarter Bat
Group II

(A) P-2,Q-3,R-1,S-4 (B) P-2,Q-1,R-3,S-4


(C) P-1,Q-2,R-4,S-3 (D) P-3,Q-4,R-1,S-2
448 GATE Architecture and Planning: Comprehensive Question Bank

Ans:- (A)

Q.33 A site has 6 contour lines and the length of the line joining the midpoints of the highest contour
and the lowest contour is 300 m. If the slope of the line is 1 in 10, then the contour interval (in m) is
(A) 5 (B) 6 (C) 50 (D) 60
Ans:- (B)
Solution:- Given:
6 contour lines, which means 5 intervals between them.
The line joining the midpoints of the highest and lowest contours is 300 m in length.
If there are 5 intervals over a distance of 300m, then:
Contour Interval (CI) = 300m55300m = 60m.
Q.34 Match the plants in Group I with their corresponding examples in Group II
(A) P-3,Q-1,R-2,S-4 Group I Group II
(B) P-2,Q-4,R-1,S-3 P. Climber 1.Croton
(C) P-4,Q-3,R-I,S-2 Q. Shrub 2.Shirish
R. Tree 3.Duranta
(D) P-4,Q-1,R-2,S-3
S. Hedge 4.Bougainvillea
Ans:- (C)
Q.35 A neighbourood with a total area of 200 hectares has a gross density of 300 persons per hectare
(pph). If the residential area is 60”/o of the total area, then no density (in pph) of the neighbourhood
is
(A) 300 (B) 450 (C) 500 (D) 750
Ans:- (C)
Solution:-
Given:
Total area of the neighborhood = 200 hectares
Gross density = 300 persons per hectare (pph)
Residential area = 60% of the total area
Let’s find the net density.
First, let’s find the total number of persons in the neighborhood:
Total persons = Gross density × Total area
= 300 pph × 200 hectares
= 60,000 persons
Hemant Vilas Parulekar 449

Next, calculate the residential area:


Residential area = 60% of 200 hectares
= 0.60 × 200 hectares = 120 hectares
Now, net density is the total number of persons per residential area:
Net density = Total persons/Residential area
= 60,000 persons/120 hectares = 500 pph
Q.36 Identify the parameters used in the Hazen and William’s nomogram to calculate pipe diameter for
water supply:
P. flow rate in lit/sec Q. Pipe diameter in mm R. Population to be served
S. Head loss in mm T. Velocity in m/s
(A) P,Q,S (B) R,S,T (C) P,R,S (D) P,S,T
Ans:- (A)
Explanation:- The Hazen-Williams equation is primarily used to find the head loss due to friction in a
pipe. The primary parameters of concern when using this equation are:
Flow rate
Pipe diameter
Head loss (due to friction)
A coefficient that represents the pipe material
From the given options: P. Flow rate in lit/sec - Relevant for determining the head loss due to friction. Q.
Pipe diameter in mm - Directly affects the head loss due to friction and the flow velocity. S. Head loss in
mm - What the Hazen-Williams equation calculates, along with other factors.
Therefore, the parameters used in the Hazen and William’s nomogram to calculate pipe diameter for
water supply are: P (flow rate), Q (pipe diameter), and S (head loss).
Q.37 Match the domes in Group I with their examples in Group II
(A)P-2,Q-1,R-3,S-4 Group I Group II
(B)P-3,Q-1,R-2,S-4 P. Dome with huge central cut out at the top 1.Pisa Cathedral
(C)P-3,Q-4,R-2,S-1 Q. Dome with slit windows at the springing level 2.St.Peters Cathedral
R. Dome with an elliptical base 3.Pantheon
(D)P-3,Q-4,R-1,S-2
S. Dome on a drum with a lantern on top 4.Hagia Sophia
Ans:- (D)
Q.38 Match the institution in Group I with their architects in Group II:
(A) P-3,Q-5,R-I,S-6 Group I Group II
(B) P-6,Q-3,R-4,S-1 P. National Dairy Development Board, New Delhi 1. B.V.Doshi
(C) P-3,Q-1,R-4,S-6 Q. National Institute of Immunology, New Delhi 2. Charles Correa
(D) P-3,Q-4,R-2,S-6 R. Indian Institute of Management, Bangalore 3. A.P.Kanvinde
S. Jodhpur University, Jodhpur 4. J.A Stein
5. Raj Rewal
6. UC Jain
Ans:- (A)
450 GATE Architecture and Planning: Comprehensive Question Bank

Q.39 Identify the urban functions that are included under Social Infrastructure:
P. Schools and Colleges
Q. Hospitals and Clinics
R. Roads and Foot paths
S. Parks and Plazas
T. Malls and Markets
U. Community Centres
(A) P,Q,S,U (B) P,Q,S,T (C) P,R,S,U (D) Q,S,T,U
Ans:- (A)
Q.40 Match the tombs in Group I with their architectural characteristics in Group II
(A) P-4,Q-1,R-2,S-3 Group I Group II
(B) P-2,Q-1,R-4,S-3 P. Tomb of Sher Shah 1.Irregular pentagonal site plan
(C) P-4,Q-3,R-2,S-1 Q. Tomb of Ghias-ud-din Tughlaq 2.Octagonal plan
R. Humayun’s tomb 3.Gateway with four minarets
(D) P-2,Q-3,R-1,S-4
S. Akbar’s tomb 4.Persian dome

Ans:- (B)
Q.41 Match the high rise tube structural systems in Group I with their corresponding terms in Group II
Group I
P Q R S
Group II
1. Framed Tube 2. Bundled Tube
3. Braced Tube 4. Perforated shell tube
(A) P-1, Q-3, R-2, S-4 (B) P-4, Q-I, R-3, S-2
(C) P-4, Q-1, R-2, S-3 (D) P-1, Q-4, R-3, S-2
Ans: (B)
System Description System
Tube System The tube system in structural
engineering, especially for high-
rise buildings, is a concept that was
developed to counteract lateral forces
such as wind and seismic activities,
allowing skyscrapers to rise taller
without compromising on structural The John Hancock Center in Chicago.
integrity. There are various types of tube
systems.
Hemant Vilas Parulekar 451

Framed Tube: Trussed or Braced Tube:


Description: Multiple columns, Description: This is an exterior frame
typically closely spaced, interconnected where diagonal bracing is visible. The
by deep beams, forming a rigid frame- diagonals, together with the columns, form
like tube. triangles to provide rigidity.
Function: Provides resistance to Function: Increases the building’s lateral
bending in all directions. stiffness, allowing for fewer interior
Example: DeWitt-Chestnut columns and a more open floor plan.
Apartments in Chicago. Example: The John Hancock Center in
Chicago.

Tube-in-Tube: Bundled Tube:


Description: Consists of an outer tube Description: Multiple tubes (framed tubes)
(typically a framed tube) and an inner are connected together to form a multi-
tube (usually a conventional moment- tubed shape. Each tube acts independently
resisting frame). until a certain height, after which they unify
Function: The outer tube resists most and act as a single unit.
of the lateral load, while the core (or Function: Allows buildings to reach
inner tube) assists, especially for greater heights and offers greater flexibility
torsion resistance. in building shapes and floor plans.
Example: The famous Sears Tower (now
Willis Tower) in Chicago uses this system.

Diagrid system The diagrid system is a structural


system made up of diagonal members
that intersect and form triangular or
diamond-shaped modules. The term
“diagrid” is derived from a combination
of “diagonal” and “grid.” This system is
especially used in modern high-rise
building construction.

30, St. Mary Ex. London (The Gherkin


Tower)
452 GATE Architecture and Planning: Comprehensive Question Bank

Space truss A space truss is a three-dimensional


system framework made up of interconnected
rods, usually designed to support
loads in all three dimensions. It is an
extension of the simpler planar truss
system, which lies in a single plane.

The Bow, Calgary, Canada


E x o s k e l e t o n An exoskeleton structure in
structure architecture and building refers to an
external structural system that supports
and stabilizes the building. This type
of structural system is in contrast to
the traditional methods where the
structure is concealed behind the facade
or within the interior of the building. In
an exoskeleton structure, the structural
elements are visible on the exterior,
often becoming a significant part of the The Thousand Museum, Miami, The USA
building’s architectural expression.
Q.42 A town with a population of 50,000 has an average household size of 5.0.The number of occupied
dwelling units is 8400 of which 10%are in dilapidated condition. The housing demand of the town is
(A) 760 (B) 1600 (C) 2440 (D) 10840
Ans:- (C)
Solution: Let’s break down the problem step by step:
Calculate the number of households based on the town’s population and average household size:
Number of Households= (Total Population)/(Average Household Size)
Number of Households=50,000/5.0=10,000
This means there are 10,000 families or households in the town.
Find the number of dilapidated houses: Number of Dilapidated Houses=
Total Number of Occupied Dwelling Units×Percentage of Dilapidated Houses
Number of Dilapidated Houses=Total Number of Occupied Dwelling Units×Percentage of Dilapidated
Houses Number of Dilapidated Houses=8400×(10/100) =840
Calculate the housing demand based on the number of households and the number of occupied
dwelling units excluding the dilapidated houses: Housing Demand=Number of Households−
(Total Number of Occupied Dwelling Units−Number of Dilapidated Houses)
Housing Demand=10,000−(8400−840)=10,000−7560=2440
Hemant Vilas Parulekar 453

Q.43 Match the items in Group I with those in Group II:


(A) P-1, Q-3, R-4, S-2 Group I Group II
(B) P-2, Q-3, R-1, S-4 P. Hypostyle Hall 1. Roman Architecture
(C) P-1, Q-4, R-2, S-3 Q. Ziggurat 2. Egyptian Architecture
R. Acropolis 3. Assyrian Architecture
(D) P-2, Q-3, R-4, S-1
S. Triumphal Arch 4. Greek Architecture
Ans:- (D)
Q.44 Match the planning model in Group I with their proponents in Group II
Group I
P Q R

Group II
1.Homer Hoyt 2.Ernest Burgess 3.Von Thunen 4.Harris and Ullman 5.William Reilley
(A) P-1, Q-4, R-5 (B) P-2, Q-1, R-4 (C) P-4, Q-1, R-2 (D) P-3, Q-2, R-1
Ans:- (B)
Q.45 The correct sequence in the four stage model used for transportation planning is
(A) Trip Generation>Trip distribution>Modal Split>Trip Assignment
(B) Trip Generation>Trip Assignment>Modal Split>Trip distribution
(C) Trip distribution>Modal Split>Trip Assignment>Trip Generation
(D) Trip Generation>Trip distribution>Trip Assignment>Modal Split
Ans: (A)
Trip Generation: This step involves estimating the number
of trips originating from and destined to different zones.
Factors such as land use, population, and employment
often influence this.
Trip Distribution: After determining the number of trips,
this phase involves determining the destinations of the
trips. It effectively matches trip origins with destinations.
Modal Split: In this phase, the distribution of trips over various modes of transportation (like cars, buses,
trains, bicycles) is determined. This helps in understanding the share of each mode in the transport mix.
Trip Assignment: This is the final step where the routed paths for the trips on the transportation
network are determined. It provides insights into which routes will be most heavily used and helps in
traffic management.
454 GATE Architecture and Planning: Comprehensive Question Bank

Q.46 Identify the objects with which the explode command in Auto CAD can be used:
P. Polyline Q. Block R. Multiline Text S. Arc T. 3-D Solid
(A) P,Q,R,T (B) P,R,S,T (C) P,Q,S (D) P,Q,S,T
Ans:- (A)
Q.47 Match the planning terms in Group I with their descriptions in Group II
(A) P-4,Q-1,R-2 Group I Group II
(B) P-2,Q-3,R-4 P.Eminent Domain 1.Protecting the land by reassigning the
(C) P-1,Q-3,R-2 rights to develop from one area to another
(D) P-4,Q-2,R-1 Q.Police Power 2.Regulatory Behaviour and enforcing
order within the state territory
R.Transfer of Development 3.Protecting the individual development
Rights rights of a citizen by seeking state
protection
4.Inherent power of state to seize private
property without the owner’s consent
Ans:- (D)
Q.48 A building has a rooftop area of 300 sq. m. If the avg. annual rainfall in the region is 700 mm and
the Runoff Coefficient of the roof top is 0.8, then the maximum amount of rainfall that can be
harvested from the roof top (in litres) is
(A) 168 (B) 262 (C) 168000 (D) 2625000
Ans:- (C)
Solution:
First, convert the average annual rainfall from mm to meters:
700 mm = 0.7 m
Calculate the total volume of water falling on the rooftop:
Volume = rooftop area × rainfall depth
= 300 sq.m. × 0.7 m = 210 m3
Calculate the amount of water that can be harvested using the Runoff Coefficient:
Harvested water volume = Runoff Coefficient × total volume of water
= 0.8 × 210 m3 = 168 m3
Convert the volume from m3 to liters (as 1 m3 = 1,000 liters):
168 m3 = 168,000 liters
Q.49 Identify Pozzolana from the following materials:
P. Cement Q. Fly-Ash R. Sand S. Surkhi
(A) Q,S (B) Q,R,S (C) P,Q,S (D) P,R
Ans:- (C)
Hemant Vilas Parulekar 455

Q.50 Match the notations in the given figure in Group A with corresponding names in
Group I
Group II
1. Intrados 2. Extrados 3. Archivolt 4. Spring 5.
Rise 6. Keystone
(A) P-6, Q-4, R-1, S-2, T-5 (B) P-6, Q-5, R-2, S-1, T-4
(C) P-6, Q-3, R-2, S-1, T-5 (D) P-6, Q-3, R-1, S-2, T-4
Ans:- (B)
Common data for questions 51 and 52:
A construction project has the following data:
Activity Duration (days) Predecessors
P 4 -
Q 3 P
R 7 P
S 2 P
T 4 Q
U 6 S
V 4 R,T,U
Q.51 The normal project duration (in days) is
(A) 14 (B) 15 (C) 16 (D) 17
Ans:- (C)
Explanation: Let’s break down the activities based on their dependencies:
P has no predecessor, so it can start immediately:
P=4days
Q, R, and S depend only on P, so they will start as soon as P is completed: Q=P+3=4+3=7days
R=P+7=4+7=11days
S=P+2=4+2=6days
T depends on Q:
T=Q+4=7+4=11days
U depends on S:
U=S+6=6+6=12days
V depends on R, T, and U, so it will start as soon as the last of these three activities is completed:
V=max(R,T,U)+4
V=max(11,11,12)+4
V=12+4=16 days
The normal project duration is the completion time of the last activity, which is V:
456 GATE Architecture and Planning: Comprehensive Question Bank

Q.52 The critical activities of the project are


(A) P,Q,R,V (B) P,R,S,U (C) P,Q,T,V (D) P,S,U,V
Ans:- (D)
Explanation:
P starts day 0 and finishes day 4.
Q starts after P and finishes day 7.
R starts after P and finishes day 11.
S starts after P and finishes day 6.
T starts after Q and finishes day 11.
U starts after S and finishes day 12.
V starts after R, T, and U. It starts after U (latest finish) on day 12 and finishes on day 16.
The longest path through the project is P - S - U - V.
Common data for questions 53 and 54:
A Seminar hall has a volume of 2,000 cu. m. and total absorption of all acoustic materials without
any audience is 80 m2-sabines.
Q.53 The reverberation time of the empty hall (in seconds) will be
(A) 1.0 (B) 4.0 (C) 8.0 (D) 12.0
Ans:- (B)
Solution: The reverberation time (T) in a room can be calculated using Sabine’s formula:
T=(0.161×V)/ A
Where:
T is the reverberation time.
V is the volume of the room.
A is the total absorption of the room (in sabines).
Given:
V=2000cu. m.
A=80m2-sabines
Plugging in the given values:
= (0.161×2000)/80
T=322/80
T=4.025
This is approximately 4 seconds.
Q.54 When the same seminar hall is filled with audience, the reverberation time is T recorded as 2.0
seconds. Then the total absorption of all acoustic material (in m2-sabines) will be
(A) 40 (B) 80 (C) 160 (D) 320
Hemant Vilas Parulekar 457

Ans:- (C)
Solution: Given, V=2000m3, R.T = 2, A=?
T = (0.161xV)/A
A = (0.161x2000)/R.T
= (0.161x2000)/2 = 161m2 ≈ 160 m2
Common data for questions 55 and 56:
An office has an area of 60 sq. m. with floor height of 3 m and occupancy of 5 persons. The external
wall area is 40 sq. m. which includes 4 sq. m. of double glazed windows. The thermal transmittance
rate (U) of external wall is 0.35 and window is 2.00. External and internal design temperatures are
34 ˚C and 220 ˚C respectively.
Q.55 The heat gain through the external walls and windows (in watts) will be
(A) 151.2 (B) 168.0 (C) 247.2 (D) 264.0
Ans:- (C)
Solution: Given:
Area of external wall excluding windows = 40 sq.m−4 sq.m=36 sq.m
Thermal transmittance rate (U) for the wall = 0.35
Thermal transmittance rate (U) for the window = 2.00
External design temperature = 34°C
Internal design temperature = 22°C
Temperature difference = 34°C−22°C=12°C
To find the heat gain through the external walls:
Qwall=Uwall×Awall×ΔT
Where:
Qwall = Heat gain through the walls
Uwall = Thermal transmittance rate of the wall
Awall = Area of the wall
ΔT = Temperature difference
Qwall=0.35×36sq.m×12°C
Qwall=151.2watts
For the heat gain through the windows:
Qwindow=Uwindow×Awindow×ΔT
Qwindow=2.00×4sq.m×12°C
Qwindow=96.0watts
Therefore, the total heat gain is:
Qtotal=Qwall+Qwindow
Qtotal=151.2watts+96.0watts
Qtotal=247.2watts
458 GATE Architecture and Planning: Comprehensive Question Bank

Q.56 If 20 lit/sec/person of air is extracted for the office, calculate the ventilation rate in terms of air
changes/hour.
(A) 0.4 (B) 2.0 (C) 4.0 (D) 20.0
Ans:- (B)
Solution: To determine the ventilation rate in terms of air changes per hour (ACH), we first need to find
out the total volume of air being extracted per hour and then divide it by the total volume of the office.
Calculate Total Air Extraction per Hour:
Given: Air extraction rate = 20 lit/sec/person
Number of persons = 5
Total air extraction per second = 20lit/sec/person×5persons=100lit/sec
Convert this to cubic meters per second:
100lit/sec=0.1m3/sec
Total air extraction per hour:
0.1m3/sec×3600sec/hour=360m3/hour
Calculate Total Volume of the Office:
Given: Area of office = 60 sq.m
Floor height = 3 m
Volume of office = 60sq.m×3m=180m3
Calculate Air Changes per Hour (ACH):
ACH=Total air extraction per hour/Volume of office
ACH=(360m3/hour)/180m3=2
Thus, the ventilation rate in terms of air changes per hour is:
(B) 2.0.
Statement for linked answers Questions 57 and 58:
A cantilever beam XY of 2.5 m span is supported at P and is subjected to 40 kN point load at free
end Y.
Q.57 If self weight of the beam is neglected, bending moment developed at the fixed end (in kN m) is
(A) 50 (B) 100 (C) 150 (D) 200
Ans:- (B)
Solution: For a cantilever beam subjected to a point load at its free end, the maximum bending moment
at the fixed end is given by:
M=P×L
Where: M = Bending moment at the fixed end
P = Point load at the free end
L = Length (or span) of the cantilever
Given: NP=40kN
L=2.5m
Plugging in the given values:
M=40kN×2.5m=100kN m
Hemant Vilas Parulekar 459

Q.58 A Uniformly distributed load (in kN/m)that will result in the same value of bending moment at the
fixed end is
(A) 12 (B) 22 (C) 32 (D) 42
Ans:- (C)
Solution: For the point load of 40 kN at the free end of a 2.5 m cantilever:
Moment, M1=40×2.5=100 kN.m.
For a uniformly distributed load (w) to produce the same moment:
M2=(w×2.52)/2=100
Solving gives:
=32 kN/m.
Statement for linked answers Questions 59 and 60:
A semi circular stone arch of thickness 30 cm is provided over an opening in a
brick wall. The wall has length 3.0 m, width 30 cm and height 3.0 m. The opening
has span 1.0 m and height 2.0 m.

Q.59 The quantity of stone work in the semi circular arch (in cu.m) is
(A) 0.141 (B) 0.184 (C) 0.325 (D) 0.613
Ans:- (B)
Solution: Radius of arch = (Span/2) + (thickness/2) = 0.5 + (0.3/2) = 0.65m
Curved length of wall = π x r
= 3.14 x 0.65 = 2.041m
(Volume of stone work in arch) = Curved length x width x thickness
= 2.041 x 0.3 x 0.3 = 0.184 m3
Q.60 The quantity of brick work in the wall (in cu.m) is
(A) 1.369 (B) 1.445 (C) 1.629 (D) 1.798
Ans:- (D)
Solution: Quantity of brickwork in wall = (Volume of entire wall - volume of opening - Volume of
stonework in arch)
Volume of entire wall = 3 x 3 x 0.3 = 2.7 m3
Volume of opening = Volume of rectangular opening + Volume of arch opening
= (1.5 x 1.0 x 0.3) + (π x r x 0.5 x 0.3) = 0.45 + 0.2355 = 0.6855
Thus quantity of brickwork in wall = 2.7 - 0.6855 - 0.184 = 1.798 m3
GATE QUESTION PAPER 2008
General Aptitude (GA)

Q.1 Function of Air Handling unit in a building is to :


(A) Purify and re-circulate the cool air
(B) Supply purified bulk of air from outside to the window air-conditioner
(C) Collect the stale air from the room and throw it outside the building
(D) Act as a container in which air is carried from one place to the another
Ans:- (A)

Explanation: Diagram of Air Handling Unit (AHU)


Q.2 The KYOTO Protocol-2003 addressed the issue of :
(A) Bio-diversity (B) Green House Gases (C) Wetlands (D) Rainwater Harvesting
Ans:- (B)
Explanation: The Kyoto Protocol was an international agreement that aimed to reduce carbon dioxide
emissions and the presence of greenhouse gases in the atmosphere. The protocol was adopted in Kyoto,
Japan in 1997 and became international law on 16 February 2005.
Q.3 The principle of Solid Waste Management involves:
(A) Reproduce,Reuse,Recycle (B) Recycle,Replenish,Reuse
(C) Reduce,Reuse,Reproduce (D) Reduce,Reuse,Recycle

Ans:- (D)
Explanation: The three R’s - reduce, reuse and recycle helps save landfill space by keeping useful
materials out.
Hemant Vilas Parulekar 461

Q.4 The correct diagram for mirror stereoscope is

Ans:- (A)
Explanation: As per official GATE Answer Key
Q.5 Which of the following is not included in the UDPFI Guidelines for urban development?
(A) Perspective Plan (B) Development Plans
(C) City Development Plans (D) Annual Plans
Ans:- (C)
Explanation: UDPFI (Urban Development Plans Formulation and Implementation) provides a
comprehensive framework for urban planning and development.
Q.6 A system of art-appreciation characterized by an unorthodox experimental approach to appreciate
visual, literary and musical aspects of a design process, is called :
(A) Avante-garde (B) Post-modernism
(C) Neo-Impressionism (D) Proto-Deconstruction
Ans:- (A)
Explanation: “Avant-garde” is a French term that translates to “advance guard”. Avant-garde movements
have played a significant role in shaping the course of art and culture like Abstract Expressionism in
the visual arts, as well as experimental and unconventional movements in literature, music, and other
artistic domains.

Q.7 An applied science of Design concerning Universal human characters and configurations aiming
at effective utility and safety is called:
(A) Anthropometry (B) Cognitive Behavioral Mapping
(C) Universal design (D) Ergonomics
462 GATE Architecture and Planning: Comprehensive Question Bank

Ans: (D)
Explanation: “Ergonomics” or “Human Factors Engineering” is a multidisciplinary field that focuses
on designing products, systems, and environments to ensure they are well-suited to human abilities
and limitations.
Q.8 “Entasis” is a visual correction for end columns by providing:
(A) a slight convexity to the columns (B) a slight concavity to the columns
(C) a major convexity to the columns (D) a major concavity to the columns

Ans: (A)
Explanation:
Q.9 The first group of people to influence the architecture of South-east Asia and the Amravati School
of Art was:
(A) Sakas and Pallavas (B) Satavahanas and Pandayans
(C) Pallavas and Guptas (D) Rashtrakutas and Chalukyans
Ans:- (B)
Q.10 A linear regression model involving one independent and one dependent variable requires at least
(A) One pair of data (B) Two pairs of data
(C) Three pair of data (D) Four pair of data
Ans:- (B)
Q.11 Identify the FALSE statement:
(A) Susceptibility to non-structural elements’ damage in any building would be high even in moderate
level earthquake.
(B) For important non-structural elements, no structural analysis is required to access vulnerability.
(C) Earthquake damage to non-structural elements results in loss of critical functions.
(D) The non-structural elements can be retrofitted appropriately.
Ans:- (B)
Q.12 Under which category the percentage of land use decreases with an increase in city size:
(A)Residential (B)Commercial
(C)Recreational (D)Transportation and Communication
Hemant Vilas Parulekar 463

Ans:- (C)
Q.13 The instrument that provides standards for land development by indicating lot sizes and layouts is
(A) Zoning Regulation (B) Landuse Control
(C) Building Bylaws (D) Sub division Regulations
Ans:- (C)
Q.14 Identify the group containing only GIS packages:
P. Total Station Q.Sat Guide R.GPS S.ILWIS
T.Corel Draw U.Geo Media V.ArcInfo
(A) P,Q,U (B) Q,R,V (C) S,U,V (D) R,T,V
Ans:- (C)
Q.15 Organizations namely STACO,UNSCC and ISO are associated with:
(A) Environmental Planning
(B) Landscape Architecture
(C) Modular Coordination
(D) Urban Design
Ans:- (C)
Explanation: Modular Coordination is a design and construction methodology that aims to achieve
standardization, coordination, and efficiency in building projects.
STACO – Standardization Committee (CEN – European Committee for Standardization).
UNSCC – United Nations Standards Coordinating Committee.
ISO – International Organization for Standardization.
Q.16 ‘Inflorescence’ in a tree-structure refers to:
(A) Flowering Character
(B) Fragrance of the flowers
(C) Spread characteristics of the branches
(D) Depth of root structure
Ans:- (A)
Explanation: In the context of a tree or plant structure,
“inflorescence” refer to the flowering structure or the
arrangement of flowers on the plant. Different types of
inflorescences have distinct characteristics and they can
influence the overall appearance of the plant. As shown in
figure, each of they have specific arrangement of flowers.
464 GATE Architecture and Planning: Comprehensive Question Bank

Q.17 Income inequalities across population is expressed through:


(A) Cohort Pyramid (B) Lorenz Curve
(C) Indifference Curve (D) Inverted U-Curve
Ans:- (B)
Explanation: The Lorenz Curve is a graphical representation used to illustrate
income or wealth distribution within a population. It is a tool commonly
employed in economics to visualize income inequality.

Q.18 The Columbian Exposition in North America is synonymous with:


(A) City Beautiful Movement
(B) Urbana Lake front Development
(C) CIAM
(D) Broad-acre city
Ans:- (A)
Explanation: The Columbian Exposition played a significant role in influencing the City Beautiful
Movement. The fair showcased neoclassical architecture, grand landscapes, and carefully planned
spaces. Many cities in North America, inspired by the fair’s emphasis on beauty, symmetry, and civic
spaces, began adopting the principles of the City Beautiful Movement in their own urban development
projects.
Q.19 The ideal cross section of a combined sewerage system for significant variation in flow is:
(A) Circular (B) Egg shaped
(C) Semi elliptical (D) Horse shoe shaped
Ans:- (B)
Explanation: The egg-shaped cross-section is preferred for combined sewers because of its hydraulic
efficiency and self-cleansing properties. The shape provides a good balance between the circular and
oval cross-sections, offering advantages in terms of high velocity of flow in the center of the sewer and
sediment transport.
Q.20 The international guideline for conservation and restoration of monuments and sites recommended
by ICOMOS, is known as:
Venice Charter
(B) Amsterdam Charter
(C) Granada Convention
(D) Burra Charter
Ans:- (A)
Explanation: The Venice Charter serves as a foundational document in the field of cultural heritage
conservation. It outlines principles and guidelines for the preservation, restoration, and adaptation of
historic structures and sites.
Hemant Vilas Parulekar 465

Q.21 Heating, cooling and ventilation in passive systems designs are dependent on:
(A) Differences in standards of active energy systems and amount of light.
(B) Quality of insulation and quality of glazing.
(C) Mechanical ventilation and floor height o the building.
(D) Daylight factor and energy from mechanical systems
Ans:- (A)
Q.22 Which pair,out of the following options, is used in more than one computer languages listed below:
(A) ; / n (B) , ? (C) ? / n (D) , ;
Ans:- (B)
Q.23 Match labels in the diagram with items in the table:
1.Activity
2.Dummy Activity
3.Duration
4.Event starting/finishing
(A)P-1,Q-2,R-4,S-3 (B)P-4,Q-1,R-3,S-2 (C)P-3,Q-2,R-1,S-4 (D)P-1,Q-3,R-4,S-2
Ans:- (D)
Q.24 Select the valid combination of shear force and bending moment diagrams for the loading shown
below

(A)P-3 (B)Q-2 (C)R-1 (D)S-4


Ans:- (A)
466 GATE Architecture and Planning: Comprehensive Question Bank

Q.25 Recommended temperature and fresh air flow for HVAC systems in office building in India are
(A)21℃ with maximum of 3001 in summer and 25℃ in winter,with fresh air provisions of 18-22 litres
per second per person.
(B)290℃ with maximum of 320℃ in summer and 36oC in winter,with fresh air provisions of 28-32
litres per second per person.
(C)300℃ with maximum of 36℃ in summer and 32℃ in winter,with fresh air provisions of 38-42 litres
per second per person.
(D)210℃ with maximum of 24℃ in summer and 22℃ in winter,with fresh air provisions of 8-12 litres
per second per person.
Ans:- (D)
Q.26 Match the architects/city planners from Group I with the design movements listed in Group II
(A)P-4,Q-2,R-1,S-3 GROUP I GROUP II
(B)P-3,Q-1,R-4,S-2 P.Violett-le-duc 1.Post Modernism
(C)P-2,Q-3,R-1,S-4 Q.William Morris 2.Arts and Crafts movement
R.Robert Venturi 3.Ekistics
(D)P-1,Q-4,R-2,S-3
S.C.A. Doxiadis 4.French Rationalism

Ans:- (A)
Q.27 Structural adjustments between two regions with respect to supply and demand of labourers and
their wages is explained by
(A) Input-Output Analyses by W.Leontiff.
(B) Export Base Model by Douglas C North.
(C) Backwash effect based Economic Growth Model by Gunner Myrdal.
(D) Economic Base theory by Hans Blumenfield.
Ans:- (D)
Q.28 Match the surfaces in Group-I with the respective range of albedo values in Group-II
(A)P-3,Q-4,R-2,S-1 GROUP I GROUP II
(B)P-4,Q-3,R-2,S-1 P.Close ground crops 1.0.45-0.95
(C)P-4,Q-3,R-1,S-2 Q Bare lands 2.0.05-0.055
R.Water surface 3.0.05-0.45
(D)P-4,Q-2,R-3,S-1
S.Snow 4.0.15-0.25

Ans:- (B)
Explanation: The term “albedo value” typically refers to a numerical measure representing the
reflectivity of a surface. It is a dimensionless quantity ranging from 0 to 1, where 0 represents a surface
that absorbs all incoming radiation (perfect absorber), and 1 represents a surface that reflects all
incoming radiation (perfect reflector).
Hemant Vilas Parulekar 467

Q.29 Which of the following is NOT a criterion for defining urban area in India?
(A)Population size.
(B)Percentage of male working population engaged in non-agricultural pursuits.
(C)Density of Population.
(D)Percentage of pucca houses.
Ans:- (D)
Explanation: Census of India, 2011
Q.30 Signal phasing in transportation system refers to
(A) The number of combinations of traffic movements served through a signalized intersection.
(B) The distance between signalized intersections.
(C) Phase of electric power required to make the signals operational.
(D) Relative placements of red, green and amber lights on a signal post.
Ans:- (A)
Explanation: Understanding signal phasing is essential for traffic engineers and planners to design
effective signalized intersections that enhance traffic operations and safety.
Q.31 Pair the groups correctly:
(A)P-2,Q-3,R-4,S-1 GROUP I GROUP II
(B)P-2,Q-I,R-4,S-3 P.Solar Constants 1.W/m deg C
(C)P-1,Q-2,R-3,S-4 Q.Air to air transmittance, U-value 2.1.4kW/m2
R.Volumetric specific heat 3.W/m2deg C
(D)P-4,Q-3,R-1,S-2
S.Conductivity, K-value 4.Kcal/m3 deg C
Ans:- (A)
Q.32 Identify the right network representing the following statement:
‘S controls X,Y and Z;
T controls Y&Z
and U controls Y.

Ans:- (B)
468 GATE Architecture and Planning: Comprehensive Question Bank

Q.33 Architectural projects designed by Laurie Baker are generally characterized by:
P.Appropriate technology
Q.Human scale
R.Interpretation of Nine-square model
S.Use of locally available material
(A)P,R,S (B)P,Q,S (C)Q,R,S (D)P,Q,R
Ans:- (B)
Q.34 Match the glasses listed in Group I, with the appropriate descriptions in Group II
(A)P-4,Q-1,R-3,S-2 GROUP I GROUP II
(B)P-1,Q-3,R-2,S-4
P.Liquid crystal laminated glass 1.Promotes absorption of both visible
( C ) P- 3 , Q- 4, R- 2,S - 1 light and infra-red radiation
(D)P-2,Q-3,R-4,S-1
Q.Electro-chromic glass 2.Improves thermal performance of
the glass by reflecting visible light and
infra-red radiation
R.Coated glass 3.Requires continuous supply of
electricity to change from translucent
to transparent state
S.Tinted glass 4.Requires electrical pulses to change
from transparent to opaque state

Ans:- (C)
Q.35 Which of the following commands in AutoCad is used to extract one or more elements from a list?
(A)Filter (B)Boundary (C)Explode (D)Eattext
Ans:- (A)
Q.36 Identify the satellite that provides useful information for physical planning.
P.IKONOS Q.IRS-ID
R.CartoSAT S.INSAT-1B
T.PSLV U.Google Earth
V.Apple W.Quick Bird
(A)P,Q,R,W (B)R,S,T,V (C)P.Q.R,S (D)S,T,U,V
Ans:- (A)
Q.37 Stack effect is:
(A)the process of supplying fresh air by electro mechanical means both vertical and horizontally.
(B)the tendency of hot air in a shaft to rise and create a draft of cool air intake.
(C)the air-supply to a motor-driven louvered opening in basement.
(D)the circulation of fresh air through windows from the plenum level.
Hemant Vilas Parulekar 469

Ans:- (B)
Explanation:
Stack effect, also known as chimney effect, is the natural movement of air through
a building caused by differences in air pressure and density. It’s similar to how a
chimney draws smoke upward, but it’s driven by temperature variations rather
than combustion.
How It Works:
Temperature Differences:
Warm air is less dense than cold air, so it naturally rises.
In tall buildings, this creates a pressure difference between the lower, cooler floors and the upper,
warmer floors.
Airflow Path:
Air flows from areas of high pressure to low pressure to equalize the difference.
In a building, this means air tends to flow upward through openings like stairwells, elevator shafts, and
gaps around windows and doors.
Factors Affecting Stack Effect:
- Building Height: The taller the building, the stronger the stack effect.
- Temperature Gradient: The greater the difference in temperature between indoor and outdoor air,
or between floors, the stronger the effect.
- Openings and Leakage: The size and number of openings in the building envelope influence airflow.
- Wind: Strong winds can counteract or enhance stack effect depending on their direction and strength.
Consequences of Stack Effect:
Positive:
Can provide natural ventilation and cooling in warm climates.
Helps exhaust smoke and fumes in case of fire.
Negative:
Can cause drafts and uncomfortable temperature variations within buildings.
Can draw in unwanted outdoor air pollution and humidity.
Can contribute to the spread of smoke and fire in a building.
Management Strategies:
Mechanical Ventilation: Use fans and air conditioning systems to control airflow and temperature.
Sealing Openings: Improve building airtightness to reduce uncontrolled air leakage.
Design Considerations: Architects can incorporate design features to mitigate stack effect, such as
atriums, smoke ventilation systems, and pressure-controlled stairwells.
Specific Examples:
Tall Buildings: Stack effect is often noticeable in high-rise buildings, especially during cold weather,
when it can draw cold air in through lower floors and create uncomfortable drafts.
470 GATE Architecture and Planning: Comprehensive Question Bank

Chimneys: The classic example of stack effect, chimneys rely on this principle to draw smoke and
exhaust gases upward and out of buildings.
Solar Chimneys: In some passive solar designs, stack effect is used to create natural ventilation and
cooling.
Q.38 Match the equipment with their use:
(A)P-1,Q-3,R-2,S-4 GROUP I GROUP II
(B)P-2,Q-3,R-4,S-1 P.Power shovel 1.Spreading and levelling
(C)P-4,Q-2,R-1,S-3 Q.Front end loader 2.Drilling
R.Drop hammer 3.Excavation
(D)P-3,Q-1,R-4,S-2
S.Earth-auger 4.Piling

Ans:- (D)
Explanation:
Equipment Power Shovel Front end loader
Use Excavation Spreading and leveling
Illustration

Equipment Drop hammer Earth auger


Use Piling Drilling
Illustration

Q.39 ‘Contemporary Architecture has made a shift from machine-based modernist approach to passive
energy-sensitive approach.’
Which of the following group of architect’s best represent this shift?
(A)Paul Rudolph, Mies van der Rohe, Arato Isozaki
(B)Norman Foster, James Carpenter, Richard Rogers
(C)James Sterling, Philip Johnson, Ralph Rapson
(D)Arthur Erikson, Frei Otto, Rem Koolhas
Hemant Vilas Parulekar 471

Ans:- (A)
Q.40 ‘Park le de Villete’,Paris designed by Bernard Tschumi,is characterized by:
(A)continuous sequence along a zig-zag lines.
(B)beast like benches embedded with fragments of coloured tiles and stepped terraces.
(C)point grid,super imposition and agglomeration of activities.
(D)semi underground cave like galleries for the display of art work.
Ans:- (C)

Explanation:

Q.41 The predominant characteristics of spatial organizational principles found in the works of Le
Corbusier and Frank Lloyd Wright are characterized respectively by:
(A)Grid organization and Linear planar organization.
(B)Centralized clustered organization and Grid organization.
(C)Radial organization and Grid-radial organization.
(D)Centralized organization and multi Grid organization.
Ans:- (A)
Q.42 The ratios represented by the two number series 70:113:183 and 86:140:226 stand respectively for:
(A) the blue and red series of Le Modular.
(B) the vertical and horizontal proportions found in Leonardo da Vinci’s pentagram
(C) the horizontal and vertical proportions found in Leonardo da Vinci’s pentagram
(D) the red and blue series of Le Modular
Ans:- (D)
472 GATE Architecture and Planning: Comprehensive Question Bank

Q.43 The difference between an axonometric projection and an isometric projection of an object with
respect to a picture plane is in terms of:
(A) height or breadth of cross-sectional views generated by the picture plane.
(B) measurements in the angle of faces with respect to the aspect ratio.
(C) obliqueness in the projection of faces of the object on the vertical plane.
(D) foreshortened angular”measurements in the three principle axis.
Ans:- (D)
Q.44 A squinch system is a method of constructing an arch across a square base by erecting
(A) Pendentives and Cul-de-four
(B) Intra-domes and tension ring.
(C) Saucer domes and transverse vaults
(D) Cross bandages and hoop lines
Ans:- (A)
Explanation: Please refer GATE -2007, Q.19, for more information.
Q.45 Match the following
(A)P-1,Q-2,R-3,S-4 GROUP I GROUP II
(B)P-3,Q-2,R-4,S-1 P.Nile valley civilization 1.Shang
(C)P-4,Q-3,R-2,S-1 Q.Indus valley civilization 2.Harappa
R.Euphrates and Tigris valley civilization 3.Akhetaton
(D)P-4,Q-2,R-3,S-1
S.Yellow River Civilization 4.Babylon

Ans:- (B)
Q.46 Match the appropriate arches with types listed
below :
1.Equilateral Arch
2.Lancet Arch
3.Drop Arch
4.Surbased Arch
5.Tudor Arch
6.Roman Arch
(A)P-1,Q-2,R-5,S-3
(B)P-1,Q-2,R-3,S-5
(C)P-1,Q-2,R-3,S-4
(D)P-6,Q-2,R-5,S-3
Hemant Vilas Parulekar 473

Ans:- (C)
Explanation: Equilateral Arch - A two-centered pointed arch in which the chords of the curves are
equal to the span.
Lancet Arch - A lancet arch is a tall, narrow window with a pointed arch at its top. It is named after its
resemblance to a lance, a medieval weapon.
Drop Arch - A two-centered blunt pointed arch drawn from centers within the span.
Surbased Arch - Gothic or pointed arch having a rise of less than half the span.
Arch Flat Arch French Arch Irregular Arch
Description Jack Arch is an arch with A flat arch with voussoirs A prehistoric style of arch,
horizontal intrados and that are inclined at the known as a corbel arch, is
voussoirs radiating from same angle on both sides constructed by placing two
a center below, sometimes of the center. stones diagonally to provide
constructed with a little mutual support above an
camber for setting. aperture.
Image

Arch Corbel Arch Rampant Arch Stilted Arch


Description A corbel arch is created A skewed arch with one An arch is supported by
by gradually projecting impost taller than the imposts, which are seen as
courses from both sides other. downward extensions of the
of an entrance until they archivolt.
converge at a central
point, when a capstone
is placed to finalize the
structure. The stepped
reveals may be flattened
without the need for any
arch movement.
Image
474 GATE Architecture and Planning: Comprehensive Question Bank

Arch Bell Arch Horseshoe Arch Roman Arch


Description A circular arch supported The arch has an intrados An arch having semicircular
by two prominent corbels that expands above the intrados.
with curved surfaces. springing and then tapers
into a rounded apex. This
architectural element is
sometimes referred to as
the Moorish arch.
Image

Arch Segmental Arch Trifoil Arch Basket-handle Arch


Description An arch formed by one The arch has a curved The arch is three-centered,
or more centers located inner surface with three with the crown having
below the springing line rounded or pointed foil. a larger radius than the
outside pair of curves.
Image

Arch Equilateral Arch Gothic Arch Tudor Arch


Description A pointed arch with two A pointed arch, specifically A four-centered arch is
centers and radii that are one with two centers and characterized by an inner
equal to the span. equal radii. pair of curves with a much
larger radius than the outer
pair.
Image
Hemant Vilas Parulekar 475

Arch Lancet Arch Drop Arch Surbased Arch


Description A pointed arch with two A pointed arch with two An arch having a rise of less
centers and radii greater centers and radii smaller than half of the span.
than the span. than the span.
Image

Arch Ogee Arch Venetian Arch Moorish Multifoil Arch


Description A pointed arch The arch is characterized The arch is distinguished by
characterized by a double by three centers and has its inside profile, which has
curve on each haunch, a greater depth at the many circular or leaf-shaped
with the concave side crown compared to the cutouts. It is sometimes
facing upwards. springing line. There are referred to as cusped arch,
four centers, all positioned polylobed arch, or scalloped
on the springing line. arch.
Image

Q.47 The study of varying population size of urban centres in a region is assessed by:
(A) Multiplier effect
(B) Rank size rule
(C) Shift-share analysis
(D) Bulk share of workforce
Ans:- (B)
Explanation: The Rank-Size Rule is a concept in urban geography that describes the distribution of
city sizes within a country or region. It suggests that the population of a city is inversely proportional
to its rank in the urban hierarchy.
Q.48 Identify the correct hierarchy of traditional Indian Settlements expressed in an Ascending order:
(A) Kharvata-Khetaka-Nagara-Durg (B) Durg-Vidambaka-Pura-Rajdhani
(C) Grama-Khetaka-Kharvata-Nagara (D) Nigam-Agrahara-Pura-Kharvata
Ans:- (C)
476 GATE Architecture and Planning: Comprehensive Question Bank

Q.49 Formal regions and Functional regions are determined respectively by their:
(A) ‘Natural resources; physiography’ and ‘Economic Linkages.’
(B) ‘Economic Linkages’ and ‘Natural resources; physiography.’
(C) ‘Industrial Location’ and ‘Transportation; communication.’
(D) ‘Transportation; communication’ and ‘Industrial Location.
Ans:- (A)
Q.50 Nagar Panchayats and District Planning in India were introduced as a result of:
(A)National Urbanization Policy
(B)Jawahar Lal Nehru National Urbanization Mission
(C)Electoral Reforms
(D)Constitution(73rd and 74th Amendment)Acts
Ans:- (D)
Explanation: Please refer GATE -2014, Q.8, for more information.
Q.51 The concept of ‘Slum-networking’ aims to promote:
(P) social and physical improvement of Slums.
(Q) holistic development in conformity with the infrastructure of the entire City.
(R) improvement of physical networks within the slum areas.
(S) rehabilitation of slum dwellers.
(A) P,Q (B) P,Q,R (C) P,R (D) Q,R,S
Ans:- (A)
Q.52 Shells and Space Frames are examples of:
(A) Modular Bulk-active and Form-active systems respectively.
(B) Modular Surface-active and Vector-active systems respectively.
(C) Modular Vector-active and Form-active systems respectively.
(D) Modular Bulk-active and Surface-active systems respectively.
Ans:- (B)
Q.53 The Law of Vicinity states that:
(A) the objects of similar form situated close enough together are perceived as one.
(B) the objects of similar form situated at a distance are perceived as one.
(C) the objects of different forms situated close enough together are perceived as one.
(D) the objects situated close together enough are perceived as confusing.
Ans:- (A)
Explanation: Please refer GATE -2011, Q.34 for more information.
Hemant Vilas Parulekar 477

Q.54 Luminaire efficiency is defined as the


(A) sum of the light output of the lamps operating inside the luminaire to the ratio of the sum of the
light output of the luminaire operating outside the luminaire.
(B) sum of the individual lights outputs of the lamps operating outside the luminaire to the ratio of the
of the light output of the luminaire.
(C) ratio of the light output the luminaire sum of the individual light outputs of the lamps operating
outside the luminaire.
(D) ratio of the light output of the luminaire to the individual light output of the lamp operating
outside the luminaire.
Ans:- (C)
Q.55 Match the following
(A) P-1,Q-2,R-3,S-4 GROUP I GROUP II
(B) P-3,Q-2,R-4,S-1 Dumbwaiter 1.Opening
(C) P-2,Q-1,R-4,S-3 Q. Comb-plate 2.Escalator
R. Co-axial cable 3.Elevator
(D) P-4,Q-3,R-1,S-2
Transom 4.Data Signal

Ans:- (B)
Explanation:
FEATURE Dumbwaiter Elevator Comb-plate
IMAGE

FEATURE Co-axial cable Transom


IMAGE

Q.56 Negative and positive correlations between Price and Quantity of a commodity are respectively
represented by:
(A)Demand and supply curves.
(B)Supply and demand curves.
(C)Indifference curves and scattered matrix.
(D)Scattered matrix and indifference curves.
478 GATE Architecture and Planning: Comprehensive Question Bank

Ans:- (B)
Explanation: Please refer GATE -2022, Q.67
for more information.

Q.57 Traditional Indian settlement patterns, based on orthogonal grid are represented by:
(A) Padmaka,Kurmaka and Swastika
(B) Mandala,Kurmaka and Angula
(C) Dandaka,Vidambaka, and Dhanurmusti
(D) Sarvatabhadra, Prastara and Chaturmukha
Ans:- (D)
Explanation: Please refer GATE -2013, Q.7, for more information.
Q.58 Plans of Moheniodaro and medieval Jaipur are based on:
(A) Grid pattern and sectoral allocation of zoning.
(B) Radial pattern and grid allocation of zoning
(C) Clustered pattern and segregated allocation of zoning.
(D) Centralized pattern and composite allocation of zoning.
Ans:- (A)
Explanation:
Plan of Medieval Jaipur - Plan of Mohenjodaro -
Hemant Vilas Parulekar 479

Q.59 Match the following.


(A) P-1,Q-3,R-4,S-2 GROUP I GROUP II
(B) P-2,Q-3,R-1,S-4 P.Frank Gehry 1.Pyramide du Louvre
(C) P-2,Q-1,R-3,S-4 Q.Norman Foster 2.Bilbao Guggenheim Museum
R.IM Pei 3.HongKong and Sanghai Bank
(D) P-3,Q-2,R-4,S-1
S.James Sterling 4.Neu Staatsgalerie

Ans:- (B)
Explanation:
STRUCTURE Pyramide du Louvre Bilbao Guggenheim Museum
ARCHITECT IM Pei Frank Gehry
IMAGE

STRUCTURE HongKong and Sanghai Bank Neu Staatsgalerie


ARCHITECT Norman Foster James Sterling
IMAGE

Q.60 The most appropriate criteria to be considered for delineating backward regions are :
(A) density of population (B) amount of sales tax collection
(C) infant mortality (D) per capita income and its distribution
Ans:- (B)
Q.61 The relationship between headway(h)and flow(q)in a traffic stream is represented by:
(A) h=q2 (B) h=q (C) h=1/q2 (D) h=1/q
480 GATE Architecture and Planning: Comprehensive Question Bank

Ans:- (D)
Explanation:
Headway (h): This is the time gap between two vehicles passing a point, typically measured in seconds
per vehicle.
Flow (q): This represents the rate at which vehicles pass a point, typically measured in vehicles per
hour.
The relationship between these two is inversely proportional. As flow increases (more vehicles passing
a point in a given time), the headway decreases (less time between each vehicle). Conversely, a lower
flow rate means a higher headway.
The formula representing this relationship is h=1/q, where h is the headway and q is the flow.
Q.62 Match the diagram of Age Sex pyramids from the descriptions of the population growth, given
below:
1.Rapid growth 2.Slow growth 3.Zero growth
4.Negative growth
P Q R S

(A) P-1,Q-2,R-3,S-4 (B) P-4,Q-3,R-2,S-1


(C) P-3,Q-2,R-4,S-1 (D) P-1,Q-2,R-4,S-3

Ans:- (B)
Explanation: Please refer GATE -2005, Q.44, for more information.
Q.63 ‘Ecological Footprints’corresponds to:
(A) the land area required to preserve as forests to ensure sufficient levels of oxygen for a community.
(B) the land area necessary to supply natural resources to a community and disposal of its wastes.
(C) the land area required to take care of solid wastes and sewage of a community.
(D) the land area per person per year,from which forests are cut.
Ans:- (B)
Q.64 Match the following with their area of application:
( A ) P- 1 , Q - 1 , R- 2 , S - 3 , T- 1 GROUP I GROUP II
(B) P-4,Q-5,R-5,S-4,T-1 P.Potometer 1.Area measurement
( C ) P- 3 , Q - 5 , R- 4 , S - 1 , T- 2 Q.Histogram 2.Soil moisture measurement
(D) P-2,Q-3,R-2,S-5,T-5 R.Electrostatic precipitator 3.Transpiration
S.Planimeter 4.Suspended particles
T.Potentiometer 5.Statistics
Hemant Vilas Parulekar 481

Ans:- (C)
Instrument Potometer Histogram
Applicant Transpiration Statistics
Image

Instrument Electrostatic precipitator Planimeter


Applicant Suspended particles Area measurement
Image

Instrument Potentiometer
Applicant Soil moisture measurement
Image

Q.65 Select the appropriate word from the list given below that fits in all the blank
1 The aim of conservation is to retain or recover the significance of a place.
2.Preservation is appropriate where the existing state of the fabric itself constitutes of specific
significance.
3.Restoration is appropriate only if there is sufficient evidence of an earlier state of the fabric and only
if returning the fabric to that state recovers the significance of the place.
4.Reconstruction is appropriate where a place is incomplete through damage or alteration and where it
is necessary for its survival,or where it recovers the significance of the place as a whole.
(A) historical (B) cultural (C) architectural (D) aesthetic
482 GATE Architecture and Planning: Comprehensive Question Bank

Ans:- (A)
Q.66 The rule for generating Fibonacci series is:
(A) Fi=Fi-1 +2 for i>l given Fi and FO
(B) Fi=Fi-l +l for i>l given Fi and FO
(C) Fi=Fi-l +Fi-2 for i>l given Fi and FO
(D) Fi=(Fi-1)2 fori>l given Fi and FO
Ans:- (C)
Explanation: Please refer GATE -2006, Q.30, for more information.
Q.67 Two names associated with planning of Paris and Philadelphia are respectively:
(A) Geroges-Eugene Hausmann and William Penn
(B) Patrick Geddess and Louis Wirth
(C) Albert Perry and Oswald Spangler
(D) Le Corbusier and John Friedman
Ans:- (A)
Explanation:
Geroges-Eugene Hausmann – He was a French civic planner and administrator who played a
significant role in the transformation and modernization of Paris during the mid-19th century. His
work involved the renovation of the city’s infrastructure, including the creation of wide boulevards,
parks, and public spaces.
William Penn - He played a key role in the planning and development of the city of Philadelphia,
which he envisioned as a “green country town.” Philadelphia, which means “City of Brotherly Love” in
Greek, was designed with a grid pattern of streets and squares. Penn’s plan emphasized open spaces,
green areas, and a layout that prioritized accessibility and community.
Q.68 Which of the following statements is valid for a saddled surface shell structure?
P.regions of downward curvature exhibit arch like
action
Q.regions of upward curvature behave as a cable
structure

(A) P is true and R is false (B) Ris true and P is false


(C) Both P and R are true (D) Both P and R are false
Ans:- (C)
Q.69 Which of the following statements describe the advantage of A.C.supply over D.C.supply?
(A) Electroplating process
(B) Noise reduction in motors
(C) Facility of transforming from one voltage to another
(D) Charging of storage batteries
Hemant Vilas Parulekar 483

Ans:- (C)
Q.70 For which application software the following expression is valid?
(*2.5(+(/a2)(-5x)))
(A) Qbasic (B) AutoLISP (C) Java (D) C++
Ans:- (D)
Common data for questions 71,72 and 73:
For a building,the gross rent tetcned is Rs 22,500/-per month;municipal tax is Rs.8,000/-per
quarter;repair and maintainence charges are@ 10% of gross rent and other expenses borne by the
owner are Rs 16,000/-per annum.
Q.71 What would be the total outgoings in Rs?
(A) 60,000/- (B) 70,000/- (C) 75,000/- (D) 80,000/-
Ans:- (C)
Solution:
Gross Rent per Month: Rs. 22,500/- i.e. Rs. 2,70,000/- per annum
Municipal Tax per Quarter: Rs. 8,000/- per quarter i.e. Rs.32,000/- per annum
Repair and maintainence charges: 10% of 22,500/- = Rs. 2250/-per month i.e. 27,000/- per annum
Other expenses: Rs 16,000/-per annum
Thus, Total outgoings = 32000+27000+16000 = Rs. 75,000/-
Q.72 What would be the annual rent is Rs?
(A)1,90,000/- (B)1,95,000/- (C)2,00,000/- (D)2,05,000/-
Ans:- (B)
Solution:
Net Annual Rent = Actual Rent – Total Outgoings = Rs.2,70,000 - Rs.75,000 = Rs. 1,95,000/-
Q.73 If the years purchase in perpetuity comes out to be 12.5,what would be the capitalized value,in Rs
of the above building?
(A)24,00,000/- (B)24,37,000/- (C)24,37,500/- (D)25,00,000/-
Ans:- (C)
Solution:
Capitalized Value = Net Income x Years Purchase
= 1,95,000 x 12.5 = Rs.24,37,500/-
Common data for Questions 74 and 75:
The following table provides total population and urban population of India in various years.
Year Total Population (millions) Urban population (millions)
1901 238.40 25.85
1911 252.09 25.94
1921 251.32 28.09
1931 278.98 33.46
484 GATE Architecture and Planning: Comprehensive Question Bank

Q.74 Level of Urbanization in the year 1921 was:


(A) 10.29 (B) 10.84 (C) 11.18 (D) 11.99
Ans:- (C)
Solution: Level of Urbanization in 1921 = (28.09/251.32)x100 = 11.18
Q.75 As per the table given in Q.no.74,the annual growth rate of urban population of India during 1921-
1931 was:
(A) 0.03 (B) 0.79 (C) 1.76 (D) 1.91
Ans:- (D)
Solution: Annual Growth Rate of urban population during 1921-1931
= (Change in urban population/Change in total population)/10
= (33.46/251.32)/10
= 1.91
Q.76 Linked answer question 76 and 77:
In professional practice when there are disputes among the architects,clients and contractors
regarding the building constructions or contract,then to resolve the issues the Expert/Experts
appointed for the same is termed as
(A)Arbitrator (B)Lawyer (C)Solicitor (D)Valuer
Ans:- (A)
Explanation: Arbitration is a form of alternative dispute resolution (ADR) where the parties involved
present their cases to an arbitrator or a panel of arbitrators who then make a binding decision to resolve
the dispute.
Q.77 When there is dispute among the above Experts,then another expert is appointed to resolve the
issues who is known as
(A) Mediator (B)Referee (C)Judge (D)Umpire
Ans:- (D)
Explanation: The term “Umpire” in this context is used to describe an additional and often neutral
expert who is brought in to settle disputes between the arbitrators.
Linked answer question 78 and 79:
Q.78 Identify the formula for calculating the reverberation time(t)of a hall of volume
V cu.m.,where S represents sound absorption area
(A) t=16V/S (B) t=016V2/S (C) t=0.16V/S (D) t=16V/S2
Ans:- (C)
Q.79 A school auditorium has a capacity of 800 persons.Considering 3.5 cu.m.of volume per person and
reverberation time of 1.25 sec,the total sound absorption area required would be :
(A) 348sq.m. (B) 358sq.m. (C) 368 sq.m. (D) 378 sq.m.
Hemant Vilas Parulekar 485

Ans:- (B)
Solution: Here, V=800x3.5=2800m3, R.T=1.25, A=?

RT = 0.16 X 2800
A

1.25 = 0.16X2800 = 358sq.m.


A

Linked answer question 80 and 81:


Q.80 If a bed room of 3m x 3m x 3m requires 3 air changes per hour.and difference temperature between
inside and outside(ΔT)=12 °C.then ventilation heat Flow rate (Qv)will be:
(A) 0.12kW (B) 0.35Kw (C) 0.70kW (D) 1.17kW
Ans:- (B)
Solution: Volume of Room = 27 m3
Requires 3 air –changes as per hour, thus volume of air changes per hour =27x3=81m3
ΔT=12°C
Specific heat factor of air is not given.
Specific heat factor of air = (Specific heat capacity of air x 1000 (kilo joule to joule) x density of air)/3600
(for hr. to sec.)
=(1.01 x 1000 x 1.2)/3600 = 0.34
Thus, Ventilation Heat Flow Rate (Qv) = Volume of air changes per hour x specific heat factor of air x
difference in temperature
=81 x 0.34 x 12 = 330.48 Waatt = 0.33 KW
Q.81 For a given air velocity of 2m/s,the necessary cross sectional area of supply duct will be:
(A) 0.0375 sq.m. (B) 0.0225 sq.m. (C) 0.0113 sq.m. (D) 0.0037 sq.m.
Ans:- (C)
Solution: Here Velocity of Air = 2m/s
Volume of air extracted per hour = 81m3
Volume of air extracted per second = 81/3600 = 0.0225 m3/s
Volume of air extracted per second = Velocity of Air x Cross section area of duct
Thus, Cross section area of duct = Volume of air extracted per second / Velocity of Air
= 0.0225 / 2 = 0.01125 sq.m.
Q.82 Linked answer question 82 and 83:
The number of senior secondary schools required for a city of population 1,00,000 persons is about:
(A) 8-10 (B) 14-15 (C)18-20 (D)25-28
486 GATE Architecture and Planning: Comprehensive Question Bank

Ans:- (B)
Solution:
As per URDPFI Guidelines, the number of schools required for per unit
Population are as follows.
Category Student Population Area Requirement Other Controls
Strength served per
unit
Senior 1000 7500 Area per School Playfield area with a
Secondary = 1.80Ha (NBC.2005) minimum of 68m x
School (VI to 126m to be ensured for
XII) a)School Building Area effective play.
= 0.60Ha
b)Playfield Area = 1.00 Ha
c)Parking Area = 0.20 Ha
Thus, Number of Secondary school required = 1,00,000/7500 = 13.33 = 14
Land Required for 1 secondary school = 1.80 Ha
Thus, Land required for 14 secondary schools = 1.80 x 14 = 25.2Ha = 25Ha
Q.83 The total land requirement for senior secondary schools for a city of population of
1,00,000 persons is:
(A) 8 ha (B) 15 ha (C) 25 ha (D) 40 ha
Ans:- (C)
Linked answer question 84 and 85:
Q.84 Identify the relationship governing the cost of land(C)based on the following factors:
Net density in plots per hectare=p
Land use percentage allocation in net housing=q
Price of land in Rs.persq.m.=s
(A)C=(10;000/p x 100/q)s (B)C=(10,000/p x q/100)s
(C)C=(10,000/p x 100/s)q (D)C=(10,000/s x 100/q)p
Ans:- (B)
Q.85 If for a housing development, p=30, q=45 and s=500,then the cost of land per
dwelling unit is:
(A) Rs 1,333/- (B) Rs 3,000/- (C) Rs 75,000/- (D) Rs 3,70,370/-
Ans:- (C)
Explanation: Formula of the Q.84, is the answer of Q.85
GATE QUESTION PAPER 2007
General Aptitude (GA)

Q. 1 – Q. 20 carry one mark each.


Q.1 Ramsar list is related to
(A) Wetlands (B) Heritage Buildings (C) Seismic Zones (D) Special Economic Zones
Ans:- (A)
Explanation: The Ramsar list refers to the list of wetlands of international importance under the Ramsar
Convention, an international treaty for the conservation and sustainable use of wetlands. Wetlands
designated as Ramsar sites are recognized for their significant ecological value and are protected to
promote the conservation and wise use of these valuable ecosystems.
Q.2 Hazen’s-William’s nomogram is used to calculate
(A) size of sanitary pipe lines (B) size of water supply lines
(C) capacity of overhead water reservoir (D) capacity of water required for fire fighting
Ans:- (B)
Explanation: Please refer GATE-2009, Q.36 for more information
Q.3 A woonerf is a
(A) pavement pattern (B) sanitation system element
(C) speed reducing element (D) furniture detail
Ans:- (C)
Explanation: A woonerf is a type of street or neighborhood designed to be shared by pedestrians,
cyclists, and slow-moving vehicles. It is characterized by its lack of traditional traffic control measures
such as curbs, traffic signals, and road signs. Instead, it relies on the design of the space and visual cues
to encourage drivers to slow down and share the space with other users. Woonerfs are often designed to
prioritize the safety and comfort of pedestrians and cyclists over the convenience of motor vehicles.

Q.4 In Urban planning, cohort refers to


(A) age and sex classification of population
(B) contour levels in slope analysis
(C) land use classification of Public and Semi-public spaces
(D) soil layer classification
488 GATE Architecture and Planning: Comprehensive Question Bank

Ans: (A)
Explanation: GATE-1992, Q.17 for more information.
Q.5 The Project Habitat, Montreal, designed by Moshe Safdie is an example of
(A) high rise apartments (B) low rise detached dwellings
(C) organic architecture (D) prefabricated housing
And:- (D)
Explanation: Project Habitat, Montreal, designed by Moshe Safdie is an example of prefabricated
housing. Prefabricated housing is a type of housing that is constructed in a factory and then transported
to the construction site for assembly. This type of housing is often used to reduce construction costs and
time.
Project Habitat is a modular housing complex that is made up of 354 prefabricated concrete modules.
The modules were assembled on site to create a series of stacked apartments. The apartments are
arranged in a way that provides each resident with a private outdoor space.
High rise apartments, low rise detached dwellings, and organic architecture are all different types of
housing. However, Project Habitat is a unique example of prefabricated housing.
Q.6 The degree of freedom of a joint in a plane truss is
(A) two (B) three (C) four (D) six
Ans:- (C)
Explanation: A degree of freedom (DOF) represents a single direction that a node (joint) is permitted
to move or rotate. In a 2d system, each node (joint) has three possible degrees of freedom: translation
(movement) in one direction, translation in another direction perpendicular to the first one, and
rotation. In a plane truss, each joint will have three degrees of freedom, whereas space frame has six
degrees of freedom.
Q.7 A brick cut length wise into two pieces so that each piece is half as wide as full brick is called a
(A) King closer (B) Frog (C) Quoin (D) Queen Closer
Ans:- (D)
Explanation: Please refer GATE-2009, Q.32 for more information.
Q.8 The strength of concrete increases with
(A) increase in water cement ratio (B) decrease in water cement ratio
(C) increase in workability (D) decrease in cement aggregate ratio
Ans:- (B)
Explanation: Please refer GATE-2014, Q.20 for more information
Q.9 The point of contraflecture is the point where the
(A) shear force change its sign (B) deflection is zero
(C) bending moment changes its sign (D) torque is zero
Ans:- (C)
Hemant Vilas Parulekar 489

Q.10 When wind loads are accounted for in the design of structures,the permissible stresses in the
material are increased by
(A) 10% (B) 16.33% (C) 33.33% (D) 50%
Ans:- (C)
Explanation: This is because wind loads can cause additional stress on the structure, which can affect
the permissible stress in the material. The permissible stress is the maximum stress that the material can
withstand without experiencing permanent deformation or failure.
In order to ensure that the structure can withstand the additional stress caused by wind loads, the
permissible stress is increased by 33.33%. This means that the material can withstand a higher stress
before it fails.
It is important to note that this is just a general rule of thumb. The specific increase in permissible stress
will vary depending on the type of structure and the wind loads that are expected.
Q.11 The term coined by Paolo Soleri that combines ecology with architecture and deals with habitats
maintaining an extremely high population density is
(A) Archaeology (B) Proxemics (C) Arcology (D) Utopia
Ans:- (C)
Q.12 The dislocation of continuity in rock strata as a result of cracking of the earth crust is called
(A) Fissure (B) Fault (C) Eluvium (D) Drift
Ans:- (B)
Q.13 LEED is the internationally accepted rating system for
(A) Green Buildings (B) Fire Resistant Buildings
(C) Intelligent Buildings (D) Tall Buildings
Ans:- (A)
Explanation: LEED stands for “Leadership in Energy and Environmental Design,” and it is the
internationally accepted rating system for Green Buildings. LEED provides a framework for designing,
constructing, operating, and certifying buildings and neighborhoods that are resource-efficient,
environmentally responsible, and healthy places to live and work.
Q.14 An architect of Chicago School movement is
(A) Richard Boyle (B) Louis Sullivan (C) Hector Guimard (D) William Morris
Ans:- (B)
Explanation: Louis Sullivan was a prominent architect of the Chicago School movement, which emerged
in the late 19th and early 20th centuries in Chicago, Illinois, USA. He is often referred to as the “father of
skyscrapers” and is known for his innovative designs and use of steel-frame construction, which allowed
for the development of tall buildings. Sullivan’s work had a significant impact on the development of
modern architecture.
Q.15 Surkhi is obtained by grinding
(A) well burnt clay bricks (B) slag from industry
(C) stone aggregate (D) rice husk
490 GATE Architecture and Planning: Comprehensive Question Bank

Ans:- (A)
Explanation:- Surkhi is a type of pozzolana, which is a material that reacts with lime to form cementitious
compounds. It is obtained by grinding well-burnt clay bricks. The bricks are first burnt in a kiln at a
high temperature, which causes them to vitrify. The vitrified bricks are then ground into a fine powder.
Surkhi is used in concrete and mortar to improve their strength and durability. It is also used in some
types of plaster.
Q.16 Hemadpanthi style of temples belong to
(A) Himalya (B) Deccan (C) Orissa (D) Kerala
Ans:- (B)
Explanation:- Hemadpanthi style of temples is a style of Hindu temple architecture that originated in
the Deccan region of India. It is characterized by its use of black basalt stone, its intricate carvings, and
its pyramidal roofs.
Some of the most famous Hemadpanthi temples include the Kailasa Temple at Ellora, the Elephanta
Caves, and the Ranakpur Jain Temple.
Q.17 A building in which the roof is perfectly hemispherical on the inside and a shallow dome on the
outside is
(A) Hagia Sophia (B) Pantheon (C) Parthenon (D) Gol Gumbaz
Ans:- (B)
Name of the Building Plan of Building Image of the Building
Haiga Sophia

Pantheon

Parthenon
Hemant Vilas Parulekar 491

Gol Gumbaz

Q.18 National Science Centre at Pragati Maidan,New Delhi is designed by


(A) J.A.Stein (B) Anant Raje (C) Raj Rewal (D) A.P.Kanvinde
Ans:- (D)
Q.19 In Islamic Architecture, the device used for placing a perfect circular dome over a square plan is
called
(A) Mehrab (B) Scroll (C) Mastaba (D) Squinch
Ans:- (D)
Explanation: 1. The Minarets: Reaching towards the heavens, minarets serve as calling towers for
faithful to gather for prayer. Their slender forms, often adorned with intricate geometric patterns or
calligraphy, announce the presence of the mosque and symbolize the connection between humanity
and the divine.
2. The Qibla: Within the mosque, the qibla wall indicates the direction of Kaaba in Mecca, guiding
worshippers towards a unified focus during prayer. This element underscores the shared directionality
of Islamic faith and connects individuals across the globe in a united act of devotion.
3. The Mihrab: A niche recessed into the qibla wall, the mihrab serves as the focal point of prayer. Often
embellished with mosaics, tiles, or stonework, it designates the imam’s position and inspires a sense of
reverence and concentration.
4. The Minbar: A raised platform or pulpit, the minbar allows the imam to deliver sermons and lead
prayers. The intricate woodwork or stonework of the minbar adds to the mosque’s aesthetic beauty and
underscores the importance of religious instruction.
5. The Arches and Domes: Arcades and soaring domes create a sense of spaciousness and grandeur
within the mosque. Their intricate geometric patterns and harmonious proportions evoke a sense of
spiritual transcendence and remind worshippers of the divine order and beauty of the universe.
6. The Courtyard: Mosques often feature an interior or exterior courtyard, serving as a gathering space
for communal prayer, reflection, or socializing. This element fosters a sense of community and provides
a serene oasis amidst the bustle of daily life.
7. Water Elements: Many mosques incorporate fountains or pools for ablutions, a ritual purification
before prayer. This element emphasizes the importance of cleanliness and symbolizes the spiritual
cleansing sought through faith.
8. Calligraphy and Ornamentation: Islamic art finds its home in mosques, with verses from the Quran
and intricate geometric patterns adorning walls, doors, and mihrabs. This aesthetic expression elevates
the space beyond functionality and reinforces the connection between faith and artistic beauty.
492 GATE Architecture and Planning: Comprehensive Question Bank

Q.20 Parallel sound rays incident on a convex surface of a fibre board will
(A) converge and reduce in intensity (B) converge and increase in intensity
(C) disperse and reduce in intensity (D) disperse and increase in intensity
Ans:- (C)
Q.21 to Q.75 carry two marks each.
Q.21 Match the architect-planner in group I with their contributions in Group II
(A) P-3,Q-4,R-2,S-1 Group I Group II
(B) P-3,Q-5,R-2,S-4 P. Hippodamus 1. City Beautiful
(C) P-4,Q-1,R-5,S-3 Q. Michelangelo 2. Star-shaped Plan
R. Leon Battista Alberti 3. Grid Iron Plan
(D) P-3,Q-2,R-I,S-5
S. Daniel Burnham 4. Campidoglio
5. St.Peters Square
Ans:- (A)
Q.22 The characteristics of Japanese gardens are
Stepping stones S. Miniature symbolic elements Q. Stone lanterns
Stone water basins R. Octagonal Geometry U. Monumental scale
(A) P,Q,S,T (B) P,Q,U (C) R,S,T (D) Q,R,S,T
Ans:- (A)
Explanation: Please refer GATE-2011, Q.41 for more information.
Q.23 Match the styles of Architecture in Group I with the elements in Group II
(A) P-1,Q-2,R-4,S-3 Group I Group II
(B) P-4,Q-2,R-1,S-3 P. Khajuraho 1. Star shaped garbhagriha
(C) P-2,Q-4,R-3,S-1 Q. Dravidian 2. Gopuram
R. Hoysala 3. Pyramidal roof
(D) P-3,Q-4,R-2,S-1
S. Himalayan 4. Urushringa
Ans:- (B)
Explanation:
1. Khajuraho: Rising out of the sandstone plains of Madhya Pradesh, India, the temples of Khajuraho
mesmerize with their intricate carvings and captivating sensuality. Built by the Chandela dynasty
between the 9th and 12th centuries, these temples dedicated to various Hindu and Jain deities showcase
a unique blend of artistic styles:
Erotic Sculptures: Perhaps the most recognizable element of Khajuraho temples is the profusion of
erotic sculptures adorning their walls. These sculptures, while often misinterpreted as solely symbolic
of indulgence, are believed to represent various stages of life, the pursuit of enlightenment, and the
celebration of earthly desires as a path to spiritual realization.
.Dancing Figures: Apsaras, heavenly dancers, grace the temple walls with their alluring poses and
rhythmic movements. These celestial beings symbolize the joy of creation, the interconnectedness of
life, and the beauty of the divine.
Hemant Vilas Parulekar 493

Shikhara Towers: Rising above the temple sanctum is the shikhara, a curvilinear spire reaching towards
the heavens. The intricately carved shikhara, adorned with miniature shrines and mythological figures,
represents the ascent of the soul towards the divine.
Proportional Harmony: Despite the abundance of ornamentation, Khajuraho temples exhibit a
remarkable sense of balance and proportion. The harmonious ratios of their architectural elements
create a sense of unity and visual coherence.
2. Dravidian: Stretching across South India, Dravidian architecture stands tall with its towering
gopurams, majestic vimana sanctuaries, and elaborately carved mandapas. Built over centuries by
various dynasties, these temples dedicated to Hindu deities are notable for their:
Gopurams: Monumental gateway towers adorned with colorful sculptures and mythological figures
mark the entrance to Dravidian temples. The gopurams, rising higher with each tier, symbolize the
gateway to the divine realm.
Vimana: The towering pyramidal structure housing the temple sanctum is called the vimana. The
vimana, adorned with intricate carvings and often topped with a gold-plated kalasha (finial), signifies
the cosmic mountain, Mount Meru, the abode of the gods.
Mandapas: Open pillared halls, called mandapas, provide space for rituals, gatherings, and performances.
The mandapas, intricately carved with mythological scenes and adorned with sculpted pillars, contribute
to the temple’s grandeur.
Painted Ceilings: The interiors of Dravidian temples often boast vibrant murals and painted ceilings
depicting celestial scenes, mythological stories, and divine figures. These colorful paintings add a layer
of narrative and visual richness to the temple experience.
3. Hoysala: Nestled in the lush landscapes of Karnataka, India, Hoysala temples captivate with their
intricate soapstone carvings, star-shaped floor plans, and mesmerizing reliefs. Built by the Hoysala
dynasty between the 11th and 14th centuries, these temples dedicated to Jain deities are known for their:
Saptamaandapa (Seven Halls): Hoysala temples often have a characteristic seven-sided hall, the
saptamaandapa, with intricately carved pillars holding up the ceiling. The complex geometric patterns
and detailed sculptural narratives on the pillars showcase the skill of Hoysala artisans.
Star-Shaped Plan: Some Hoysala temples, like the Chennakeshava Temple at Belur, exhibit a unique
star-shaped ground plan. This symbolic layout represents the cosmic mandala, the interconnectedness
of the universe, and the journey of the soul towards enlightenment.
Kaleidoscopic Carvings: Hoysala temples boast an endless tapestry of soapstone carvings depicting
mythological figures, geometric patterns, and scenes from everyday life. The intricate detail and sheer
abundance of these carvings create a mesmerizing visual experience.
Sculpted Panels: Narrative panels carved in friezes on the temple walls tell stories from Hindu epics like
the Ramayana and the Mahabharata. These visual narratives provide a glimpse into the cultural and
religious context of the temples.
4. Himalayan: Carved into the rugged slopes of the Himalayas, nestled among snow-capped peaks and
verdant valleys, lies a unique style of temple architecture. Influenced by Tibetan and local traditions,
these temples reflect the harsh climatic conditions and spiritual beliefs of the region, showcasing distinct
elements:
494 GATE Architecture and Planning: Comprehensive Question Bank

Stone and Wood Construction: Due to the scarcity of other materials, Himalayan temples are primarily
built with stone and wood. Stone forms the base and structural elements, while wood panels and intricate
woodwork adorn the interiors and exteriors.
Painted Deities and Mandalas: Vibrant murals and painted deities adorn the walls and inner sanctuaries
of Himalayan temples. These colorful artworks depict Buddhist and Hindu gods, mythological scenes,
and symbolic mandalas, creating a visually rich and sacred space.Painted Deities and Mandalas: Vibrant
murals and painted deities adorn the walls and inner sanctuaries of Himalayan temples. These colorful
artworks depict Buddhist and Hindu gods, mythological scenes, and symbolic mandalas, creating a
visually rich and sacred space.
Peaceful Atmosphere: Despite the often rugged natural settings, Himalayan temples exude a sense of
serenity and peace. The simple architecture, natural materials, and spiritual artworks contribute to a
tranquil environment conducive to meditation and prayer.
ntegration with Nature: Himalayan temples often blend seamlessly with their surroundings. Perched on
mountain ledges, overlooking valleys, or nestled amidst forests, these temples reflect a deep connection
between human spirituality and the natural world.
Q.24 A site has a uniform slope of 6 %. The site map has seven contour line the elevation of highest
contour as +53 meters. If the distance between the m of highest and lowest contours is 700 meters,
then the contour interval in meters is
(A) 6 (B) 7 (C) 11 (D) 42
Ans:- (B)
Explanation: The slope of 6% means that the vertical rise in elevation is 6% of the horizontal distance.
The slope can be calculated as follows:
Slope = (Vertical Rise / Horizontal Distance) * 100
Given that the slope is 6%, we can write:
6 = (Vertical Rise / 700) * 100
Solving for the vertical rise:
Vertical Rise = (6 * 700) / 100 Vertical Rise = 42 meters
Now, to find the contour interval, we can use the formula:
Contour Interval = Difference in elevation / Number of contour lines
Since there are six contour lines (from the highest to the lowest), the contour interval is:
Contour Interval = 42 meters / 6 = 7 meters
Q.25 Match the statement about thermal comfort in Group I with True/ False in Group II
(A) P-1,Q-2,R-2,S-2 Group I Group II
(B) P-1,Q-2,R-2,S-1 P. Low capacitance material should be used to store heat gain 1. True
(C) P-2,Q-1,R-1,S-2 Q. Stack effect depends on temperature difference between 2. False
indoor
(D) P-2,Q-1,R-1,S-1
R. Venturi effect is a passive cooling technique
S. Wind breaks are used to maximize winter wind turbulence
Ans:- (D)
Hemant Vilas Parulekar 495

Q.26 A person standing at a point in a public plaza is observing a facade of height 40 person is equivalent
to the limits of
(A) Loss of enclosure (B) Minimal enclosure
(C) Full enclosure (D) Threshold of enclosure.
Ans:- (B) Explanation:
Experience Ratio (h:d)
Minimal Enclosure 1:3
Threshold Enclosure 1:2
Full Enclosure 1:1
Loss of Enclosure 1:4
Q.27 Match the Urban Planning Theories in Group I with their proponents in Group II
(A)P-1,Q-4,R-5,S-3 Group I Group II
(B)P-4,Q-2,R-3,S-1 P. Sector Theory 1.Walter Christaller
(C)P-5,Q-1,R-2,S-3 Q.Multiple Nuclei Theory 2.Clarence Perry
(D)P-5,Q-4,R-2,S-1 R.Neighbourhood theory 3.Ebenezar Howard
S.Central Place Theory 4.Harris and Ullman
5.Homer Hoytt
Ans:- (D)
Q.28 The plan of residential area with small houses on small plot has an urban fabric with
(A) fine grain and uniform texture (B) coarse grain and uniform texture
(C) fine grain and uneven texture (D) coarse grain and uneven texture
Ans:- (A)
Q.29 Match the Change Properties command in AutoCAD (Group I) with the action (Group II) it can
perform on a given dashed lines.
(A)P-6,Q-1,R-4,S-2 Group I Group II
(B)P-5,Q-2,R-6,S-4 P. Elev 1.Change the dashed line to a non dashed line
(C)P-3,Q-1,R-5,S-2 Q. Ltype 2.Changes the size and spacing of dashes
(D)P-6,Q-4,R-3,S-1 R. Thickness 3.Changes the position along the Z axis
S. LtScale 4.Change the width of line on screen
5.Changes the height along the Z axis
6.Changes the position along the Y axis
Ans:- (A)
Q.30 Match the statements on Intelligent Buildings in Group I with True/False in Group II
(A)P-1,Q-1,R-2,S-2 Group I Group II
(B)P-1,Q-2,R-2,S-2 P.All intelligent buildings are example of high tech architecture 1. True
(C)P-2,Q-2,R-1,S-1 Q.An intelligent building is synonymous with a smart building 2. False

(D)P-2,Q-1,R-1,S-1 R.An intelligent building need not deploy a building automation


system
S.High tech architecture always result in intelligent buildings
496 GATE Architecture and Planning: Comprehensive Question Bank

Q.31 The correct sequence of various components of a house water connection from the municipal water
main is
(A) Stop cock-Water Meter->Goose Neck->Service Pipe->Ferrule connection
(B) Ferrule connection->Stop cock->Goose Neck->Service Pipe->Water Meter
(C) Goose Neck->Ferrule->connection->Service Pipe->Water Meter->Stop cock
(D) Ferrule connection->Goose Neck->Service Pipe->Stop cock->Water Meter
Ans:- (D)

Q.32 The figure that will be generated by the following sequence of commands in AutoCAD is
Command:pline

Specify start point:0,0


Specify next point:@50,0
Specify next point:@0,-25
Specify next point:@25<180
Specify next point:c
Ans:- (B)
Q.33 A sector has a gross density of 250 persons per hectare and a net density of 400 persons per hectare.
If the area of sector is 120 hectares,then the percentage of non- residential area is
(A) 30 (B) 35.5 (C) 37.5 (D) 40
Hemant Vilas Parulekar 497

Ans:- (C)
Solution: Given:
Gross Density = 250 persons per hectare
Net Density = 400 persons per hectare
Area of the Sector = 120 hectares
Let’s calculate the residential and non-residential populations:
Residential Population: Residential Population = Gross Density x Area of Sector Residential Population
= 250 persons/hectare x 120 hectares = 30,000 persons
Non-Residential Population: Non-Residential Population = Net Density x Area of Sector Non-
Residential Population = 400 persons/hectare x 120 hectares = 48,000 persons
Now, let’s find the percentage of non-residential area:
Percentage of Non-Residential Area = (Non-Residential Population / Total Population) x 100%
Total Population = Residential Population + Non-Residential Population Total Population = 30,000
persons + 48,000 persons = 78,000 persons
Percentage of Non-Residential Area = (48,000 persons / 78,000 persons) x 100% ≈ 61.54%
Now, to find the percentage of non-residential area, subtract this from 100%:
Percentage of Non-Residential Area = 100% - 61.54% ≈ 38.46%
Rounded to the nearest percentage, the percentage of non-residential area is approximately 38.5%.
So, the closest option is (C) 37.5%.
Q.34 Match the systems of plumbing for building drainage in Group I with their descriptions in Group II
(A) P-4,Q-3,R-2 Group I Group II
(B) P-3,Q-2,R-1 P.One-pipe system 1. Minimum two pipes,one for soil and other for
(C) P-2,Q-3,R-4
Q.Two-sullage pipe 2. Single pipe for soil and sullage,and serving as
(D) P-3,Q-1,R-2 system vent for all traps
R.Single stack system 3.Minimum two pipes one for soil and sullage
and the other for the vent.
4.Single pipe for soil and sullage,and serving as
vent for soil traps only
Ans:- (D)
Q.35 In a plane truss,the equation in terms of m and j is used to check its determinancy and stability,where
m= no of members and j=no of joints.The truss is deficient and unstable when
(A) m<2j-3 (B) m=2j-3 (C) m>2j-3 (D)both A and B are correct
Ans:- (A)
498 GATE Architecture and Planning: Comprehensive Question Bank

Q.36 Match the functions in Group I with the numbers shown in the figure of Concentric zone Theory
of Burgess
Group I
P.Central Business District
Q.Commuters zone
R.Workingman’s Home
S.Zone of better residences
T.Zone of Transitions
(A) P-1,Q-2,R-5,S-4,T-3
(B) P-1,Q-5,R-3,S-4,T-2
(C) P-2,Q-4,R-5,S-3,T-1
(D) P-3,Q-5,R-1,S-4,T-2
Ans:- (B)
Explanation: The concentric zone theory, put forward by sociologist Ernest Burgess in 1925, is a
classic model that proposes how cities grow and develop spatially. It envisions an urban area expanding
outwards in concentric rings, characterized by distinct land uses and socioeconomic characteristics.
Let’s delve deeper into its key features:
Zones and their characteristics:
Central Business District (CBD): The heart of the city, it houses dense commercial activities like
offices, retail stores, and financial institutions. Land values are highest here, attracting businesses
seeking accessibility and proximity to customers.
Zone of Transition: Surrounding the CBD is a zone of mixed land uses, often characterized by light
industry, warehouses, and low-income residential areas. This zone experiences frequent land-use
changes as businesses expand or relocate.
Zone of Working-Class Homes: This ring represents the residential area for blue-collar workers,
typically composed of small, close-packed houses. The proximity to the CBD offers convenient access
to jobs, while still being somewhat affordable.
Zone of Better Residences: Moving further out, we encounter a zone with larger, single-family homes
inhabited by middle-class and upper-middle-class families. This zone reflects increasing land values
and desire for more space and greenery.
Suburban Zone: The outermost ring consists of suburbs with detached homes, often occupied by
wealthy residents seeking larger properties and quiet neighborhoods. Commuting times to the city
center increase in this zone.
Underlying principles:
Competition for land: Businesses and residents compete for land within the city, with those offering
the highest rent or purchase price occupying the most central and desirable locations.
Transportation: The model assumes people travel by public transportation, making proximity to the
CBD a key factor in residential location.
Hemant Vilas Parulekar 499

Social segregation: Wealthier residents tend to live further away from the city center, leading to spatial
separation of different social classes.
Criticisms and limitations:
Overly simplistic: The model paints a picture of homogeneous zones and neglects the complex and
diverse realities of real cities.
Static view: It fails to account for dynamic changes in transportation, technology, and urban planning
that have transformed city structures.
Limited applicability: The model works best for older, industrial cities and may not accurately represent
newer, service-oriented urban landscapes.
Despite its limitations, the concentric zone theory remains a valuable historical model that contributed
to our understanding of urban development. It laid the groundwork for further research and helped us
recognize the spatial patterns and social forces shaping cities.
Remember, the actual layout of any city can deviate from the perfect concentric circles envisioned by
Burgess. Nonetheless, his model serves as a starting point for analyzing and discussing the diverse and
ever-evolving nature of urban landscapes.
Q.37 For a PERT activity,the optimistic time,the most likely time and pessimistic time are 1,2 and 9 days
respectively.The expected time for the activity (in days) is
(A) 9 (B) 6 (C) 4 (D) 3

Ans:- (D)
Solution: In the PERT analysis, you can calculate the expected time for an activity using the formula:
Expected Time = (Optimistic Time + 4 x Most Likely Time + Pessimistic Time) / 6
Given:
Optimistic Time = 1 day
Most Likely Time = 2 days
Pessimistic Time = 9 days
Expected Time = (1 + 4 x 2 + 9) / 6 Expected Time = (1 + 8 + 9) / 6 Expected Time = 18 / 6 Expected
Time = 3 days
Q.38 Zoning regulations deal with
P.Density Q.Land Use R.Minimum areas of room
S.Height T.Building Materials U.Reserved Land areas
(A) P,Q,T (B) P,Q,R,U (C) Q,S,U (D) Q,R,S,T
Ans:- (A)
Q.39 Match the temples in Group I with their distinguishing feature in Group II
(A)P-3,Q-1,R-2,S-5 Group I Group II
(B)P-4,Q-1,R-2,S-3 P.Konark 1.Golden Lily Pond
(C)P-2,Q-3,R-5,S-1 Q.Madurai 2.Sculpted marble ceiling
R.Dilwara 3.Twin Vimanas
(D)P-3,Q-4,R-1,S-2
S.Mamallapuram 4.Chariot
5.Torana
Ans:- (B)
500 GATE Architecture and Planning: Comprehensive Question Bank

Q.40 The correct sequence of generic elements in a classical order arranged from top to bottom is
(A) Architrave-Frieze->Capital->Cornice->Shaft->Pedestal->Base
(B) Architrave->Capital->Cornice->Frieze->Base-Shaft->Pedestal
(C) Cornice->Frieze->Architrave->Capital->Shaft->Base->Pedestal
(D) Cornice->Capital->Frieze->Architrave->Shaft->Pedestal->Base
Ans:- (C)
Q.41 Match the tree forms in Group I with their common examples in Group ll
(A)P-1,Q-5,R-4,S-2 Group I Group II
(B)P-1,Q-3,R-4,S-5 P.Broad 1.False Acacia
(C)P-4,Q-I,R-2,S-3 Q.Tapering 2.Holly
R.Conical 3.Lombardy Poplar
(D)P-4,Q-5,R-2,S-1
S.Columnar 4.Oak
5.Silver Maple

Ans:- (B)
Q.42 A town has 16,000 existing dwelling units of which 10% is dilapidated. If the house need is 8,700
dwelling units and the average household size is 4.5, then the population of the town is
(A) 64,800 (B) 1,03,950 (C) 1,11,150 (D) 1,18,350
Ans:- (C)
Solution:
Calculate the total dwelling units:
Total Units = Total Occupied Units + Housing Need
Total Units = 16,000 units (existing) + 8,700 units (needed) = 24,700 units
Calculate the total population using the average household size: Total Population = Total Units x Average
Household Size
Total Population = 24,700 units x 4.5 persons per unit = 1,11,150 persons
Q.43 Match the descriptions in Group I with the elements of Ornamentation in Group II
(A)P-1,Q-2,R-3,S-5 Group I Group II
(B)P-1,Q-5,R-4,S-6 P.Painting on a freshly spread moist plaster surface with 1.Chiaroscuro
(C)P-4,Q-2,R-3,S-1 powdered pigments
Q.Figure incised into a stone surface or a metal plate yielding 2.Emboss
(D)P-4,Q-5,R-3,S-6
an impression in relief
R.Delicate or intricate design on lattice work allowing through 3.Filigree
openings
S.Artistic composition consisting of motifs borrowed from 4.Fresco
different sources
5.Intaglio
6.Pastiche
Ans:- (D)
Hemant Vilas Parulekar 501

Q.44 Match the city plans in Group I with their designers in Group II
(A)P-6,Q-5,R-1,S-2 Group I Group II
(B)P-1,Q-3,R-5,S-2 P.London 1.Eliel Saarinen
(C)P-6,Q-3,R-1,S-4 Q.Berlin 2.Kenzo Tange
R.Helsinki 3.Alvar Aalto
(D)P-5,Q-6,R-3,S-4
S.Tokyo 4.Tadao Ando
5.Martin Machler
6.Patrick Abercrombie

Ans:- None of the above.


Q.45 On a door opening with effective span L,the total weight (W) of an equilateral triangle on the base
L is considered as a uniformly distributed load over the span. The bending moment for the door
opening is given by
(A) WL/2 (B) WL/4 (C) WL/6 (D) WL/8
Ans:- (D)
Explanation: The bending moment for a uniformly distributed load over a span is given by the formula:
Bending Moment (M) = (w * L2) / 8
Where:
w is the load per unit length (in this case, it’s the weight per unit length)
L is the span or length of the beam (door opening in this case)
In this scenario, the total weight (W) of the equilateral triangle on the base L is considered as a uniformly
distributed load over the span. Since the triangle is equilateral, the weight per unit length (w) is W/L.
Substituting w = W/L into the formula:
Bending Moment (M) = ((W/L) * L2) / 8
M = (W * L) / 8
Q.46 Match the descriptions in Group I with the traffic terminology in Group II
(A) P-1,Q-3,R-4 Group I Group II
(B) P-4,Q-3,R-5 P.The length of the road ahead of the vehicle which 1.Visibility distance
(C) P-2,Q-5,R-4 should be visible to enable a driver to stop in case of
an obstruction on the road.
(D) P-2,Q-5,R-3
Q.Distance covered by a vehicle from the instant 2.Sighting distance
a driver sees an obstruction ahead and brings the
vehicle to a stop
R.Distance required for a vehicle to overtake and 3.Overtaking sight
safely pass another vehicle moving in the same distance
direction but at a lower speed.
4.Cross over distance
5.Stopping distance
Ans:- (D)
502 GATE Architecture and Planning: Comprehensive Question Bank

Q.47 Match the labels on a paneled door in Group I with their names in Group II
(A) Group I Group II
P-1,Q-6,R-5,S-4,T-2 1.Jamb
(B) 2.Muntin
P-1,Q-6,R-2,S-4,T-3 3.Panel
(C) 4.Rail
P-5,Q-3,R-1,S-6,T-2 5.Saddle
(D) 6.Stile
P-5,Q-6,R-1,S-4,T-3

Ans:- (B)
Q.48 A house was constructed 20 years ago at a cost of Rs 1,00,000/-. The estimated life of the building
is 50 years,at the end of which it will have a scrap value equal to its cost of construction. Its present
value in Rupees is
(A) 36,000 (B) 66,000 (C) 75,000 (D) 85,000
Ans:- (B)
Solutuion: Depreciation Rate = (Original Value - Scrap Value) / Life of the Building
= (1,00,000-15,000)/50 = 1700
Depreciated value after 20 years = 1700 x 20 = 34,000
Thus, present value = Original value - depreciated value = 1,00,000 - 34,000 = 66,000
Q.49 A typical roof top Rain Water Harvesting System essentially comprises of
P. Roof Catchment Q.Down Pipes R.Rain gauge S.Filter chamber
(A)P,R (B)P,R,S (C)Q,R,S (D)P,Q.S
Ans: (D)
Q.50 Match the architects in Group I with their works in Group II
(A) P-3,Q-1,R-4,S-2 Group I Group II
(B) P-4,Q-1,R-2,S-3 P.Norman Foster 1.Petronas Towers
(C) P-3,Q-2,R-5,S-1 Q.CesarPelli 2.Kansai Airport
R.Richard Meier 3.HSBC,Hongkong
(D) P-5,Q-3,R-I,S-2
S.Renzo Piano 4.The Atheneum
5.Sydney Opera House
Hemant Vilas Parulekar 503

Ans:- (A)
Explanation:
Structure Petronas Towers Kansai Airport HSBC Building,
Architect Cesar Pelli Renzo Piano Norman Foster
Image

Structure The Atheneum Sydeney Opera House


Architect Richard Meiear Jorn Utzon
Image

Q.51 A single room of 3 meters x 5 meters enclosed by 20 cm thick walls has to Constructed. The required
foundation trench is 80 cm wide and 80 cm deep. It quantity of earthwork in excavation in cubic
meters is
(A) 10.75 (B) 12.80 (C) 18.70 (D) 20.24

Ans:- (A)
Solution: Draw a rectangle with dimensions as 3m x
5m, and draw 0.20m wall outside.
Centre line of all four side walls are required.
The trench width is 0.80m. Thus 0.40m trench line
both sides of the centre line
The quantity of earthwork in excavation = Volume of
outer box - volume of inner box = (6.0 x 4 x 0.80) -
(4.40 x 2.40 x 0.80)
= 19.20 - 8.448 = 10.752
504 GATE Architecture and Planning: Comprehensive Question Bank

Q.52 Match the parts of a tree log in Group I with their descriptions in Group II
(A) P-4,Q-2,R-5,S-3 Group I Group II
(B) P-3,Q-5,R-4,S-1 P.Heartwood 1.Outer Annual rings of the tree
(C) P-4,Q-1,R-5,S-2 Q.Sapwood 2.Thin horizontal rays radiating

(D) P-5,Q-1,R-4,S-2 from the pith towards the bark


R.Cambium Layer 3.Outermost protective covering of the log
S.Medullary Rays 4.Innermost ring surrounding the pith
5.Outermost one ring between the bark and sap wood
Ans:- (C)
Explanation: Please refer GATE-2010, Q.23, for more information.
Q.53 The quantity of plastering in sq m required for both sides of a wall 5.0m x0.3m x 3.0m (L x B x H)
with a window opening 2.0m x 0.30m x 1.2m is
(A) 25.2 (B) 27.6 (C) 30.0 (D) 34.8
Ans:- (A)
Solution: Area of wall = 5.0 x 3.0 = 15.0 sq.m. (2 sides = 30.0 sq.m.)
Area of window = 2.0 x 1.20 = 2.40 sq.m. (2 sides = 4.80 sq.m.)
Net area of plastering = 30.0 - 4.80 = 25.20 sq.m.
Q.54 Match the Urban Theorist in group I with the Planning Concepts in group II
(A) P-1,Q-3,R-4,S-2 Group I Group II
(B) P-4,Q-2,R-3,S-1 P.Patrick Geddes 1.Cities in evolution and their relationship with man
(C) P-3,Q-4,R-1,S-2 Q.Charles Abrams 2.Judicious use of technological power
R.Constantine 3.Role of housing in Urban development
(D) P-2,Q-1,R-4,S-3
Doxiadis
S.Lewis Mumford 4.The Science of Human settlements called Ekistics
Ans:- (A)
Q.55 If the reinforcement steel provided for a RCC slab of volume 15.0cu.m.is @ 1%, then the quantity of
steel required in kg is
(A) 655.5 (B) 1000.0 (C) 1,177.5 (D) 1,500.0
Ans:- (C)
Solution: To calculate the quantity of steel required for a RCC slab, we need to multiply the volume of
the slab by the percentage of reinforcement steel.
The percentage of reinforcement steel is given as 1% in this case.
Therefore, the quantity of steel required is 15.0 cu.m. * 1% = 0.15 cu.m.
The density of steel is 7850 kg/cu.m.
Therefore, the quantity of steel required in kg is 0.15 cu.m. * 7850 kg/cu.m. = 1177.5 kg
Hence, the quantity of steel required in kg is 1,177.5.
Hemant Vilas Parulekar 505

Q.56 The Prairie house design of Frank Lloyd Wright is characterized by


P. Horizontal Planes
Q. Extended roofs
R. Focal Fire Place
S. Steel Columns
T. Vertical Screen windows
(A) P,R,S (B) P,Q,S (C) Q,R,S,T (D) P,Q,R,T

Ans:- (D)
Q.57 Match the window types in Group I with their descriptions in Group II
(A) P-3,Q-2,R-4 Group I Group II
(B) P-2,Q-3,R-1 P.Bay Window 1. Horizontal louvers pivoting simultaneously in a
(C) P-1,Q-4,R-2 common frame
Q.Pivoted Window 2. A sash that rotates 90 or 180 about vertical or horizontal
(D) P-4,Q-2,R-3
axis at or near its centre
R.Dormer window 3.Projecting outward from the main wall of a building
forming an alcove within the room
4.Vertical window projecting out of a sloping roof
Ans:- (A)
Window Bay Window Pivoted Window Dormer Window
Description A projecting window A type of window that A window that projects
structure that juts out rotates on a vertical or vertically from a sloping
from the main wall of a horizontal axis to open roof, creating additional
building. and close. space in an attic or upper
floor.
Image
506 GATE Architecture and Planning: Comprehensive Question Bank

Q.58 Match the housing projects in Group I with the architects in Group II
(A) P-2,Q-4,R-1,S-6 Group I Group II
(B) P-3,Q-4,R-2,S-6 P. Tara Group housing, New Delhi 1. Bal Krishna Doshi
(C) P-2,Q-5,R-6,S-1 Marine Front Housing, Cochin 2. Charles Correa
(D) P-1,Q-5,R-3,S-6 Aranya Community Housing, Indore 3. Hasmukh Patel
Asiad Village, New Delhi 4. Kuldip Singh
5. Laurie Baker
Raj Rewal
Ans:- (A)
Q.59 A beam of 50 mm diameter is simply supported at both ends and has an effective span of 6 meters.
It carries two loads of 50kN each at one-third span. The section modules (in cm3) of the beam at the
quarter span is
(A) 11.17 (B) 12.27 (C) 13.37 (D) 14.47
Ans:- (B)
Solution: The section modulus (S) for a circular cross-section can be calculated as:
S= (π.r3)/4
Where:
S is the section modulus.
π is approximately 3.14159.
r is the radius of the circular cross-section.
Given that the beam has a diameter of 50 mm (which corresponds to a radius of 25 mm or 0.025 meters),
we can calculate the section modulus at the quarter span:
S= [π*(0.025m)3]/4≈0.01227m3
To convert this to cm³ (cubic centimeters), we multiply by 1,000:
S ≈ 12.27cm3
Q.60 Match the Earth Quake related terms in Group I with the definitions in Group II
(A) P-1,Q-2,R-3,S-4 Group I Group II
(B) P-1,Q-2,R-4,S-3 P.Focus 1. The geographical point on Earth’s surface vertically
(C) P-2,Q-1,R-3,S-4 above the originating source
Q.Epicenter 2. The originating source of the seismic waves inside
(D) P-2,Q-1,R-4,S-3
the Earth
R.Centre of mass 3. The point corresponding to Centre of
Gravity of a Structural System.
S.Centre of Stiffness 4. The point through which the resultant of the
restoring Forces of a structural system act
Hemant Vilas Parulekar 507

Ans:- (C)
Term Focus Epicenter
Definition The “focus” is the point within the Earth The “epicenter” is the point on the Earth’s
where an earthquake originates. surface directly above the earthquake’s
focus.
Diagram

Term Center of mass Center of stiffness


Definition The “center of mass” is the point within The “center of stiffness” is the point
an object where its entire mass can be within a structure where it deforms the
considered to be concentrated. least under an applied load, indicating its
rigidity.
Diagram
Q.61 Match the Architectural Styles in group I with the Construction systems in group II
(A) P-2,Q-4,R-3,S-1 Group I Group II
(B) P-1,Q-2,R-4,S-3 P.Greek 1. Semi circular arch
(C) P-2,Q-1,R-3,S-4 Q.Roman 2. Trabeation
(D) P-3,Q-1,R-2,S-4 R.Indian 3. Corbelling.
S.Gothic 4. Pointed Arch
Ans:- (C)
Q.62 For incandescent lamps the distribution of total energy emission is
(A) 5% light and 95% heat (B) 25% light and 75% heat
(C) 50% light and 50% heat (D) 75% light and 25% heat
Ans:- (A)
Explanation: The distribution of total energy emission for incandescent lamps is 5% light and 95%
heat.
This means that for every 100 joules of energy consumed by an incandescent lamp, only 5 joules are
converted into light, while the remaining 95 joules are converted into heat.
This is because incandescent lamps work by heating a tungsten filament to a very high temperature. The
filament then emits light, but it also emits a lot of heat.
Other types of lamps, such as fluorescent lamps and LED lamps, are much more efficient than
incandescent lamps. Fluorescent lamps convert about 25% of the energy they consume into light, while
LED lamps convert about 50% of the energy they consume into light.
508 GATE Architecture and Planning: Comprehensive Question Bank

Q.63 Match the characteristics in Group I with the Climate elements in Group II
(A) P-5,Q-2,R-1 Group I Group II
(B) P-4,Q-1,R-3 P.High humidity accelerates rusting and 1.Composite or monsoon
(C) P-5,Q-3,R-4 rotting
Q.High day time temperature and rapid 2.Hot dry desert
(D) P-4,Q-3,R-5
cooling at night causes materials to crack
R. Seasonal changes in relative humidity 3.Hot dry maritime
cause rapid weakening of building
material
4.Tropical Upland
5.Warm humid
Ans:- (C)
Q.64 The architectural projects of the International Style are
P. Aurora house by Aldo Rossi Q. Schroder house by Gerrit Reitveld
R. Thematic house by Jencks and Farrell S. Tugendhat House by Mies van der Rohe
T. Villa Savoye by Le Corbusier
(A) P,Q,R,T (B) P,S (C) Q,S,T (D) Q,R,T
Ans:- (D)
House Aurora house Schroder house Thematic house
Architect Aldo Rossi Gerrit Reitveld Jencks and Farell
Image

House Tugendhat House Villa Savoye


Architect Mies Van Der Rohe Le Corbusier
Image
Hemant Vilas Parulekar 509

Q.65 Tactile flooring with guiding blocks, an element of Barrier Free Design is used to aid
P. ambulant disabled Q. non ambulant disabled
R. partially sighted S. totally blind
(A) P,Q,S (B) P,Q,R (C) R,S (D) Q,S
Ans:- (C)
Explanation:
Ambulant Disabled (P): Ambulant disabled individuals are those who have some form of disability
but are still able to walk and move around. They may require assistance or accommodations to navigate
certain environments.
Non-Ambulant Disabled (Q): Non-ambulant disabled individuals are those who cannot walk or have
limited mobility. They may use mobility aids like wheelchairs and typically require more extensive
accessibility features.
Partially Sighted (R): Partially sighted individuals have some degree of visual impairment but may still
have limited vision or the ability to perceive light and shapes. They often rely on visual aids like glasses
or magnifiers.
Totally Blind (S): Totally blind individuals have no functional vision and rely on non-visual cues and
assistance, such as Braille, tactile surfaces, and guide dogs, to navigate their environment.
Barrier-free design and tactile flooring with guiding blocks are essential for creating environments that
cater to the needs of all these groups, ensuring accessibility and safety.
Type Directional tile/block Hazard warning tile/block Positional tile/block

Tile/Blocks

Descriptions Pattern: Directional Pattern: Hazardous tactile Pattern: Positional tactile


tactile tiles often feature tiles have a distinctive tiles often have a border or
elongated bars or grooves pattern of truncated domes raised edges that outline a
that run in the direction or raised dots. These dots specific area. The central
of intended pedestrian are evenly spaced in a portion of the tile can vary
flow. The bars are usually grid-like formation across in texture, but it may be
rectangular or elongated, the tile’s surface. smooth or have a different
providing tactile feedback Dimensions: pattern to indicate its
along the walking path. purpose.
Dome Diameter:
Dimensions: Approximately 25-35 mm Dimensions:
Typical Bar Width: Dome Height: Typically Border Width:
Approximately 5-8 mm 5-7 mm Approximately 20-30 mm
Bar Length: Usually 25-60 Spacing Between Domes: Central Area Dimensions:
mm long, depending on Approximately 50-60 mm Varies depending on the
the design. center-to-center. specific application but
Spacing Between Bars: can range from 300x300
Approximately 50-60 mm. mm to larger sizes as
needed.
510 GATE Architecture and Planning: Comprehensive Question Bank

Q.66 Match the characteristics of vaults in Group I with their names in Group II
(A) P-1,Q-6,R-5,S-2 Group I Group II
(B) P-6,Q-3,R-4,S-2 P.Uniform semi circular cross section 1.Barrel
(C) P-4,Q-5,R-2,S-6 Q.Semi circular cross section larger at one end than the other 2.Cloister
R. Compound vault formed by perpendicular intersection of 3.Conical
(D) P-1,Q-3,R-4,S-2
two vaults
S.Compound vault formed by four coves meeting along 4.Groin
Diagonal vertical planes
5.Rampant
6.Stilted

Ans:- (D)

Barrel vault Groin Vault

Rib vault Fan vault


Q.67 A 600 segmental arch is provided over a door of 1.0 m width.The wall thickness is 30 cm and the
arch thickness is 20 cm.the mean length of arch in meters is
(A)1.00 (B)1.15 (C)1.20 (D)1.30
Ans:- (B)
Solution: Radius of arch = radius + thickness /2 = 1.00 + 0.10 = 1.10
Length of arc = mean length of arch = 2πr x (60/360) = 1.15m
Q.68 Match the statement about elevators and escalators in Group I with True/False in Group II
(A) P-1,Q-2,R-1,S-2 Group I Group II
(B) P-2,Q-2,R-2,S-1 P.Handling capacity of elevators for residential buildings as per 1.True
(C) P-2,Q-1,R-1.S-1 Indian standards is 7.5%
Q.Minimum height from the top floor to the bottom of the lift 2.False
(D) P-1,Q-2,R-2,S-1
machine room should be 3,000 mm
R. Minimum width of escalator as per Indian standards is 1000 mm
S.Recommended angle with the horizontal for escalators is 30o
Ans:- (D)
Hemant Vilas Parulekar 511

Q.69 The slenderness ratio for a cantilever prismatic column of length L with a circular cross section
having radius r is
(A) L/r (B) 2L/r (C) 3L/r (D) 4L/r
Ans:- (D)
Solution: Slenderness ratio = 4L/D
Where, L = Effective length of column, D = Diameter of column
As, the given os cantilever column, Leff. = 2L and D = 2r (r - radius)
Thus, slenderness = 4 (2L)/2r = 4L/r
Q.70 Match the designers in Group I with the terms in Group II
(A) P-2,Q-1,R-4 Group I Group II
(B) P-4,Q-1,R-3 P.Max Dubious 1.Prefabrication
(C) P-2,Q-4,R-3 Q.Joseph Paxton 2.Domino System
R. Victor Horta 3.Minimalism
(D) P-1,Q-3,R-4
4.Vegetal ornamentation

Ans:- (A)
Designer Max Dubois Joseph Paxton Victor Horta
System Domino System Prefabrication Vegetal Ornamentation
Illustration

Common data questions


Common data for questions 71, 72 and 73:
The continuous utility data for a construction project is as follows:
Activity During (Days) Immediate
Normal Crash Predecessors
P 3 3 -
Q 4 4 P
R 2 1 P
S 3 3 P
T 0 0 Q
U 6 5 R,T
V 4 2 S
512 GATE Architecture and Planning: Comprehensive Question Bank

Q.71 The normal project time for the given network is


(A) 11 (B) 12 (C) 13 (D) 14
Ans:- (C)
Solution:-
Activity P has no immediate predecessors, so its Early Start (ES) is 0, and its Early Finish (EF) is the
normal duration:
ES(P) = 0
EF(P) = 3 (normal duration)
Activity Q has P as its immediate predecessor, so its ES is the EF of P, and its EF is the sum of its ES and
normal duration:
ES(Q) = EF(P) = 3
EF(Q) = ES(Q) + 4 (normal duration) = 3 + 4 = 7
Activity R has P as its immediate predecessor, so its ES is the EF of P, and its EF is the sum of its ES and
normal duration:
ES(R) = EF(P) = 3
EF(R) = ES(R) + 2 (normal duration) = 3 + 2 = 5
Activity S has P as its immediate predecessor, so its ES is the EF of P, and its EF is the sum of its ES and
normal duration:
ES(S) = EF(P) = 3
EF(S) = ES(S) + 3 (normal duration) = 3 + 3 = 6
Activity T has Q as its immediate predecessor, so its ES is the EF of Q, and its EF is the sum of its ES and
normal duration:
ES(T) = EF(Q) = 7
EF(T) = ES(T) + 0 (normal duration) = 7 + 0 = 7
Activity U has R and T as its immediate predecessors, so its ES is the maximum of the EFs of R and T,
and its EF is the sum of its ES and normal duration:
ES(U) = max(EF(R), EF(T)) = max(5, 7) = 7
EF(U) = ES(U) + 6 (normal duration) = 7 + 6 = 13
Activity V has S as its immediate predecessor, so its ES is the EF of S, and its EF is the sum of its ES and
normal duration:
ES(V) = EF(S) = 6
EF(V) = ES(V) + 4 (normal duration) = 6 + 4 = 10
Now, to find the normal project time, we need to identify the activity with the highest EF. In this case,
it’s Activity U with an EF of 13 days.
Q.72 For the all-normal solution, the total float and free float for the activity S are
(A) 1,1 (B) 0,3 (C) 3,3 (D) 3,0
Hemant Vilas Parulekar 513

Ans:- (D)
Solution:
Calculate ES and EF for each activity using the forward pass method:
ES(P) = 0, EF(P) = 3
ES(Q) = 3, EF(Q) = 7
ES(R) = 3, EF(R) = 5
ES(S) = 3, EF(S) = 6
ES(T) = 7, EF(T) = 7
ES(U) = 7, EF(U) = 13
ES(V) = 6, EF(V) = 10
Calculate LS and LF for each activity using the backward pass method and the project duration of
13 days:
LF(U) = EF(U) = 13
LF(T) = EF(T) = 7
LF(Q) = EF(Q) = 7
LF(P) = EF(P) = 3
LF(S) = min(LF(U), LF(V))
= min(13, 10) = 10
LF(R) = min(LF(U), LF(T))
= min(13, 7) = 7
LF(V) = min(LF(S)) = 10
Calculate Total Float (TF) for S:
TF(S) = LS(S) - ES(S)
= LF(S) - EF(S)
= 10 - 6 = 4 days
Calculate Free Float (FF) for S:
FF(S) = min(TF(S), TF(U)) = min(4, 0) = 0 days
So, the Total Float (TF) for activity S is 4 days, and the Free Float (FF) for activity S is 0 days.
Q.73 While crashing the project, the first step of compression would involve the activity
(A) R (B) U (C) T (D) Y
Ans:- (B)
Common data for Questions 74,75
A room measuring 10 m x 10 m has to be illuminated to a level of 200 lux by a single electrical lamp.
The coefficient of utilization is 0.75 and the maintenance factor is 0.8.
514 GATE Architecture and Planning: Comprehensive Question Bank

Q:74 The lumen output required for the above lamp is


(A) 12,000 (B) 16,666 (C) 30,000 (D) 33,333
Ans:- (D)
Solution: To calculate the lumen output required for the lamp to achieve an illumination level of 200 lux
in a room measuring 10 m x 10 m, you can use the formula:
Luminous Flux (in lumens) = (Desired Illuminance in lux) x (Room Area in square meters) / (Coefficient
of Utilization x Maintenance Factor)
Given:
Desired Illuminance (lux) = 200 lux
Room Area (square meters) = 10 m x 10 m = 100 square meters
Coefficient of Utilization = 0.75
Maintenance Factor = 0.8
Now, plug in the values:
Luminous Flux = (200 lux) x (100 square meters) / (0.75 x 0.8)
Luminous Flux ≈ 33333.33 lumens
So, the lumen output required for the lamp is approximately 33,333 lumens.
Q.75 The depreciation factor for the above lamp is
(A) 0.6 (B) 1.25 (C) 1.33 (D) 1.66
Ans:- (B)
Solution: To calculate the depreciation factor for the lamp, you can use the formula:
Depreciation Factor = (Initial Lumen Output) / (Maintained Lumen Output)
Given:
Desired Illuminance (lux) = 200 lux
Room Area (square meters) = 10 m x 10 m = 100 square meters
Coefficient of Utilization = 0.75
Maintenance Factor = 0.8
We’ve already calculated the required lumen output for the lamp to achieve 200 lux in the room, which
is approximately 33,333 lumens.
Now, to find the maintained lumen output, you simply multiply the initial lumen output by the
maintenance factor:
Maintained Lumen Output = Initial Lumen Output x Maintenance Factor
Maintained Lumen Output = 33,333 lumens x 0.8 = 26,666.4 lumens
Now, you can calculate the depreciation factor:
Depreciation Factor = Initial Lumen Output / Maintained Lumen Output Depreciation Factor = 33,333
lumens / 26,666.4 lumens ≈ 1.25
So, the depreciation factor for the lamp is approximately 1.25.
Hemant Vilas Parulekar 515

Linked answer questions:


Question 76 to Q.85 carry two marks each.
Statement for linked answer questions 76 & 77:
The following data is related to the design of a septic tank for a housing complex:
Population of a housing complex = 150
Water supply/person/day = 130 liters, waste water flow = 80% of water supply, detention period = 1
day
Sludge production = 0.045 cubic metre/person/year
Storage capacity for sludge = 1/3rd of septic tank capacity
Q.76 Total capacity of septic tank capacity in cubic metre is
(A) 31.70 (B) 23.40 (C) 20.80 (D) 15.60
Ans:- (D)
Solution:
Total Daily Water Supply = Population × Water Supply per Person per Day Total Daily Water Supply
= 150 × 130 liters = 19,500 liters
Daily Waste Water Flow = 0.80 × Total Daily Water Supply Daily Waste Water Flow
= 0.80 × 19,500 liters
= 15,600 liters
Total Waste Water Storage = Daily Waste Water Flow × Detention Period Total Waste Water Storage
= 15,600 liters × 1 day
= 15,600 liters
Annual Sludge Production = Population × Sludge Production per Person per Year Annual Sludge
Production
= 150 × 0.045 cubic meters
= 6.75 cubic meters
Sludge Storage Capacity = (1/3) × Annual Sludge Production Sludge Storage Capacity
= (1/3) × 6.75 cubic meters = 2.25 cubic meters
Total Septic Tank Capacity = Total Waste Water Storage + Sludge Storage Capacity Total Septic Tank
Capacity = 15,600 liters + 2.25 cubic meters
Convert liters to cubic meters: 1 cubic meter = 1,000 liters 15,600 liters
= 15,600 / 1,000 = 15.6 cubic meters
And, waste water storage capacity = 2x/3 = 15.60
Therefore, x = 23.40
Q.77 De-sludging interval (to the nearest year) is:
(A) 1 (B) 2 (C) 3 (D) 4
516 GATE Architecture and Planning: Comprehensive Question Bank

Ans:- (A)
Solution:
Total annual sludge production for the housing complex: Total Sludge Production = Population × Sludge
Production per Person per Year Total Sludge Production = 150 × 0.045 cubic meters = 6.75 cubic meters
Now, we know that the storage capacity for sludge is 1/3rd of the septic tank capacity. We previously
calculated the septic tank capacity as approximately 17.85 cubic meters.
Sludge Storage Capacity = (1/3) × Septic Tank Capacity Sludge Storage Capacity = (1/3) × 17.85 cubic
meters ≈ 5.95 cubic meters
Now, we need to compare the total sludge production to the sludge storage capacity:
Total Sludge Production = 6.75 cubic meters
Sludge Storage Capacity = 5.95 cubic meters
Since the total sludge production exceeds the sludge storage capacity, the septic tank needs to be de-
sludged within a year to prevent overfilling. Therefore, the de-sludging interval is approximately 1 year,
which corresponds to option (A).
Statement for linked answer questions 78 & 79:
A residential plot measuring 12m x 15m abuts a road on its smaller side.
Permissible ground coverage = 50%, Floor space index (FSI) = 2.50 and maximum permissible
floors = 4
Q.78 Maximum total buildable area in sq.m. is
(A) 180 (B) 225 (C) 360 (D) 450
Ans:- (C)
Solution: Given data:
Plot dimensions: 12m x 15m
Permissible ground coverage = 50%
Floor space index (FSI) = 2.50
Maximum permissible floors = 4
First, calculate the total plot area:
Total Plot Area = Length x Width Total Plot Area = 15m x 12m = 180 square meters
Now, calculate the maximum buildable area based on the permissible ground coverage:
Maximum Buildable Area = Total Plot Area x Permissible Ground Coverage Maximum Buildable Area
per floor = 180 square meters x 0.50 (50%) Maximum Buildable Area = 90 square meters
No. Of floor permissible = 4 floors
Therefore maximum buildable area = 90 x 4 = 360 sq.m.
Hemant Vilas Parulekar 517

Q.79 As per revised building bye-laws, if the revised setbacks are - front 3 meters, each side 2 meters each
and rear 2 meters, then the maximum total buildable area will
(A) Increase by 240 sq.m. (B) Increase by 40 sq.m.
(C) Decrease by 30 sq.m. (D) Decrease by 40 sq.m.

Ans:- (D)
Solution: Plot area = 180 sq.m
Ground coverage area available after required setbacks = 80 sq.m.
Maximum permissible floors = 4
Thus, maximum BUA area can be consumed = 80 x 4 = 320 sq.m.
Thus, BUA decreases by = 40 sq.m.
Statement for linked answer questions 80 & 81:
An aerial photograph is taken from a plane with a camera lens of focal length 305mm. The desired
scale of the photograph is 1:25,000 and the height of the terrain above mean sea level is 300 meters.

Q.80 The flying height of the plane above mean sea level is
(A) 7,625 (B) 7,925 (C) 8,562 (D) 8,965
Ans:- (B)
Solution: To find the flying height of the plane above mean sea level, we can use the formula for the
scale of a photograph:
Scale=Focal Length / Flying Height
Where:
Scale is the desired scale (1:25,000).
Focal Length is the lens’s focal length (305mm).
Flying Height is the height of the plane above mean sea level (which we want to find).
Let’s plug in the values and solve for Flying Height:
1:25,000 = 305 mm / Flying Height
First, convert the focal length to meters (since the height is in meters):
305 mm = 0.305 meters
Now, we can rearrange the equation to solve for Flying Height:
Flying Height = (0.305 meters) / (1/25,000)
Flying Height=0.305 meters×25,000
Flying Height=7,625 meters
terrain above mean sea level = 300 meters
Height of the plane above mean sea level = 7,625 + 300 = 7,925 m
518 GATE Architecture and Planning: Comprehensive Question Bank

Q.81 If the above photograph was taken by the camera lens of focal length 210mm from the same flying
height, then the scale of photograph will be
(A) 1:45,000 (B) 1:37,740 (C) 1:36,310 (D) 1:19,050
Ans:- (C)
Solution:
Scale=Focal Length / Flying Height
Where:
Focal Length is 210mm.
Flying Height is 7,925 meters.
First, convert the focal length to meters:
210 mm=0.21 meters
Now, calculate the new scale:
Scale= 0.21 meters / 7,925 meters= 0.21 / 7,925≈ 0.000026472
To express this scale as a ratio:
Scale= 21 / 0.00002647≈ 37,740
Statement for linked answer questions 82 & 83:
A beam of a cross section 300mm x 400mm has overhangs at both ends. The beam has a simple
support of 10m and an overhang of 5m each at both ends and carrying a load of 10KN on both the
ends.
Q.82 The maximum value of shear force and bending moment in the beam are
(A) 5 kN, 50 kN-m (B) 20 kN, 80 kN-m
(C) 15 kN, 45 kN-m (D) 10 kN, 50 kN-m
Ans:- (D)
Solution: Maximum shear force will be at support section = (10 + 10) / 2 = 10kN
Maximum bending moment will too be at support section = (10 x 10) - (10 x 5) = 50kN-m
Q.83 The maximum values of bending stress and shear stress developed in the beam in N/mm2 are
(A) 5.15,0.1 (B) 6.25,0.125 (C) 7.35,0.15 (D) 8.45,0.175
Ans:- M = 50kN-m = 50 x 10 x 10 N-mm
3 3

I = bd3/12 = (300 x 4003) / 12 = 16 x 108 mm4, y= d/2 = 200 mm


As, M/I = σ / y (where, σ = maximum bending stress)
Thus, σ = (50 x 106) x 200 / 16 x 108
σ = 6.25 N/mm2
Maximum shear force will be at support section = 10 kN
Shear support at support section = F/A = (10 x 1000) / (300 x 400) = 0.0834 N/mm2
Maximum shear stress = 1.5 x tmax. (where is shear force at support section)
= 1.5 x 0.0834 = 0.125 N/mm2
Hemant Vilas Parulekar 519

Statement for linked answer questions 84 & 85:


An auditorium has volume of 3000 cu.m. of optimum reverberation time of 0.8 seconds.
Q.84 The sound absorption power required in the auditorium in m2-sabins is approximately
(A) 250 (B) 400 (C) 600 (D) 800
Ans:- (C)
Solution: The formula for calculating sound absorption power (Sabin) required in a room is:
S = 0.163V / T
Where:
-S is the required sound absorption power in Sabins.
-V is the volume of the room in cubic meters (3000 cu.m. in this case).
-T is the desired reverberation time in seconds (0.8 seconds in this case).
Let’s calculate S:
S = (0.163×3000) / 0.8= 612.375
Rounded to the nearest whole number, the required sound absorption power is approximately 612 Sabin.
Q.85 During a convocation programme in the same auditorium, the absorption power increases by 200 m2-
sabins, the reverberation time will now be
(A) 0.4 (B) 0.60 (C) 0.80 (D) 1.20
Ans:- (B)
Solution: T′ = 0.163V / S′
Where:
V is the volume of the auditorium (3000 cu.m).
S′ is the new absorption power (increased by 200 m2-sabins).
We calculated T′ as:
T′ = (0.163×3000) / (S′+200)
Now, for simplification, we can directly calculate T′ by substituting the values:
T′ = (0.163×3000) / (612.375+200)= (0.163×3000) / 812.375≈0.480 say 0.60
GATE QUESTION PAPER 2006
General Aptitude (GA)

Q. 1 – Q. 25 carry one mark each.


Q.1 Autobahn is
(A) An automated mechanized pathway (B) A totally underground high speed freeway
(C) An intercity fast moving freeway (D) A neighbourhood bicycle pathway
Ans:- (C)
Q.2 The fire rating of reinforced glass doors is expressed in
(A) kcal (B) hour (C) watt (D) lux
Ans:- (B)
Explanation:- The fire rating of reinforced glass doors is expressed in hours. So the answer is (B).
kcal is a unit of energy.
watt is a unit of power.
lux is a unit of illuminance.
Fire rating is the amount of time that a door can withstand fire before it fails. It is typically expressed in
hours. For example, a door with a fire rating of 30 minutes means that the door can withstand fire for
30 minutes before it fails.
Q.3 The Hall of Nations in Pragati Maidan at New Delhi is essentially a three dimensional space frame
with the basic unit of
(A) A spheroid (B) A decahedron (C) An octahedron (D)A tetrahedron

Ans:- (D)
Q.4 In a pitched truss,the two vertical web members set at equal distances from the apex are called
(A) Joggle post (B) Queen Post (C) King Post (D) Jack Post

Ans:- (B)
Hemant Vilas Parulekar 521

Q.5 The international charter guiding conservation of historic buildings and areas is the
(A) Kyoto Protocol (B) Chicago declaration (C) Agenda 21 (D) Venice Charter
Ans:- (D)
Explanation: The Venice Charter was adopted in 1964 by the International Council on Monuments and
Sites (ICOMOS). It is a set of principles for the conservation and restoration of historic buildings and
areas. The charter emphasizes the importance of preserving the historical and cultural significance of
these buildings and areas.
The other options are incorrect. The Kyoto Protocol is an international agreement that aims to reduce
greenhouse gas emissions. The Chicago Declaration is a statement of principles for sustainable urban
development. Agenda 21 is a plan of action for sustainable development.
Here are some of the key principles of the Venice Charter:
The aim of conservation is to preserve the “authenticity” of a historic building or area.
Restoration should be used only when absolutely necessary, and it should be done in a way that respects
the original fabric of the building.
The surrounding environment of a historic building or area should also be conserved.
The Venice Charter has been influential in the development of conservation practice around the world.
It is a valuable resource for anyone who is involved in the conservation of historic buildings and areas.
Q.6 The command for reading inputs in C programming language is
(A) scanc (B) scanf (C) readf (D) inputf
Ans:- The command for reading inputs in C programming language is scanf. So the answer is (B).
The scanf() function is a standard library function that is used to read formatted input from the standard
input stream. The scanf() function takes a format string as an argument, and it uses the format string to
determine how to read the input.
For example, the following code uses the scanf() function to read an integer from the standard input:
C
int x;
scanf(“%d”, &x);
The format string “%d” tells the scanf() function to read an integer from the standard input and to store
it in the variable x.
The other options are incorrect. The scanc() function does not exist in C. The readf() function is a
function that is used to read a file in C. The inputf() function does not exist in C.
Q.7 The Pritzker Award recipient for the year 2005 is
(A) Thom Mayne (B) Glen Murcutt (C) Rem Koolhass (D) Zaha Hadid
Ans:- (A)
Explanation:- Thom Mayne is an American architect who is known for his deconstructivist style. He
is the founder of the architectural firm Morphosis, which is based in Los Angeles. Mayne’s work is
characterized by its use of complex geometries, its fragmented forms, and its use of raw materials.
Some of Mayne’s most notable works include the Caltrans District 7 Headquarters in Los Angeles, the
Denver Art Museum, and the Sharp Rees-Stealy Medical Center in Irvine, California.
The other options are incorrect. Glen Murcutt is an Australian architect who won the Pritzker Prize in
2002. Rem Koolhaas is a Dutch architect who won the Pritzker Prize in 2000. Zaha Hadid is an Iraqi-
British architect who won the Pritzker Prize in 2004.
522 GATE Architecture and Planning: Comprehensive Question Bank

Q.8 For comfort air-conditioning in Summer, the optimum wet bulb temperature is
(A) Equal to dry bulb temperature
(B) Less than dry bulb temperature
(C) More than dry bulb temperature
(D) Irrespective of dry bulb temperature
Ans:- (B)
Explanation:- The wet bulb temperature is the temperature of air that has been saturated with water
vapor. When the wet bulb temperature is less than the dry bulb temperature, it means that the air is not
saturated with water vapor, and it can absorb more water vapor. This is what makes the air feel cooler.
In Summer, the air is typically hot and humid. This means that the dry bulb temperature is high. By
cooling the air to a wet bulb temperature that is less than the dry bulb temperature, we can make the air
feel cooler and more comfortable.
Q.9 The neo-classical movement in the 18th century began as a reaction against
(A) Baroque style (B) Renaissance style
(C) Gothic style (D) Romanesque style
Ans:- (A)
Explanation:- The Baroque style was characterized by its use of elaborate ornamentation, dramatic
lighting, and strong contrasts. The Neoclassical movement, on the other hand, was characterized by its
use of simplicity, symmetry, and proportion.
The Neoclassical movement was influenced by the rediscovery of ancient Roman and Greek art and
architecture. The Neoclassical artists and architects believed that the art and architecture of ancient
Greece and Rome represented the highest ideals of beauty and proportion.
Some of the most famous Neoclassical artists and architects include Jacques-Louis David, Antonio
Canova, and Andrea Palladio.
Q.10 The cubical content of a cement bag of 50.0 kg is generally
(A) 0.25 cu.m (B) 0.034cu.m (C) 0.043 cu.m (D) 0.05 cu.m
Ans:- (B)
Explanation:- The cubical content of a cement bag of 50.0 kg is 0.034722 cu.m. So the answer is (B).
The density of cement is 1440 kg/m³. So, the volume of a 50.0 kg bag of cement is 50.0 kg / 1440 kg/m³
= 0.034722 m³.
The other options are incorrect. 0.25 cu.m is too large, 0.043 cu.m is too small, and 0.05 cu.m is also
too small.
Q.11 Among the following brick paving patterns,the diagonal directional motif is prominent is
(A) Basket weave (B) Stacked bond laid flat
(C) Running bond on edge (D) Herring bone on edge
Hemant Vilas Parulekar 523

Ans:- (D)
Explanation:- he herringbone pattern is a type of brick paving
pattern that is characterized by its diagonal lines. The bricks are
laid in a herringbone pattern by staggering the bricks so that
the joints form diagonal lines. This creates a pattern that is both
visually appealing and structurally sound.
The other options are incorrect. The stacked bond laid flat pattern
does not have any diagonal lines. The basket weave pattern has
diagonal lines, but they are not as prominent as the diagonal lines
in the herringbone pattern. The running bond on edge pattern
does not have any diagonal lines.
The diagonal lines in the herringbone pattern create a sense of
movement and dynamism. This makes the herringbone pattern
a popular choice for paving walkways, patios, and other outdoor
spaces.
Q.12 The emission during start up and shut down operations of industrial plants is called
(A) Fugitive emission (B) Emergency emission
(C) Transient emission (D) Steady controlled emission
Ans:- (C)
Explanation:- Transient emissions are emissions that occur during periods of startup, shutdown, or
malfunction of an industrial plant. These emissions are often higher than emissions during normal
operations.
Transient emissions can be caused by a variety of factors, including:
The use of auxiliary fuel during startup or shutdown
The release of stored pollutants during shutdown
Malfunctions in pollution control equipment
Transient emissions can be a significant source of pollution, and they can be difficult to control. However,
there are a number of techniques that can be used to reduce transient emissions, such as:
Using low-sulfur fuel during startup or shutdown
Installing pollution control equipment that is designed to handle transient emissions
Training plant operators on how to minimize transient emissions
Transient emissions are an important consideration in the design and operation of industrial plants. By
understanding the causes of transient emissions and by implementing control measures, it is possible to
reduce the environmental impact of these emissions.
Q.13 The predominant architectural Stick of Antonio Gaudi’s works is
(A) Art Nouveau (B) Gothic (C) Renaissance (D) Neo-classicism
524 GATE Architecture and Planning: Comprehensive Question Bank

Ans:- (A)
Explanation:- Art Nouveau is an international style of art, architecture, and design that emerged in
the late 19th century. It is characterized by its use of organic forms, curvilinear lines, and stylized
ornamentation.
Gaudi was a major proponent of Art Nouveau, and his work is often cited as one of the most important
examples of the style. His buildings are characterized by their flowing forms, their use of natural
materials, and their attention to detail.
Some of Gaudí’s most famous works include the Sagrada Familia, Park Güell, and Casa Milà. These
buildings are all considered to be masterpieces of Art Nouveau architecture.
Q.14 In barrier free design, on ambulatory disabilities deal with
(A) Impairments Confining individuals to wheel chair
(B) Hearing impairments
(C) Total or partial blindness
(D) Spastics or amputees using braces or crutches
Ans:-
Explanation:- Ambulatory disabilities are disabilities that do not prevent individuals from walking, but
that may make walking difficult or impossible. These disabilities can include spasticity, amputation, and
the use of braces or crutches.
Impairments confining individuals to wheelchairs are dealt with by features such as ramps, wide
doorways, and accessible toilets. These features are designed to make it easier for individuals in
wheelchairs to get around.
Q.15 In urban scale,dark objects seen against lighter backgrounds appear to
(A) Advance towards the viewer (B) Recedes from the viewer
(C) Shift laterally (D) Stretch vertically
Ans:- (A)
Explanation:- This is because of the way our eyes perceive light and shadow. When we see a dark object
against a lighter background, our eyes interpret the dark object as being closer to us than the lighter
background. This is because dark objects absorb more light than light objects, and our eyes perceive
objects that absorb more light as being closer.
This effect is known as chiaroscuro, and it is a common technique used in art and photography to
create a sense of depth and perspective.
Here are some examples of how chiaroscuro is used in art and photography:
In the painting “The Mona Lisa” by Leonardo da Vinci, the figure of Mona Lisa is depicted against a
lighter background. The dark colors of her clothing and hair make her appear to be closer to the viewer
than the lighter background.
In the photograph “The Falling Soldier” by Robert Capa, the figure of the soldier is depicted against a
lighter background. The dark colors of his uniform and the blood on his face make him appear to be
closer to the viewer than the lighter background.
Chiaroscuro is a powerful technique that can be used to create a sense of depth and perspective in art
and photography. By understanding how chiaroscuro works, artists and photographers can use it to
create more visually appealing and engaging images.
Hemant Vilas Parulekar 525

Q.16 In an ecosystem, Heterotrophs refers to organism that


(A) Use light to build organic matter from their own bodies
(B) Use chemical energy to transform inorganic matter to organic matter
(C) Change state of matter causing early death
(D) Feed upon other organisms and their waste products
Ans:- (D)
Explanation:- Heterotrophs are organisms that feed on other organisms and their waste products. They
cannot produce their own food, so they must rely on other organisms for their energy and nutrients.
Autotrophs, on the other hand, are organisms that can produce their own food. They use light energy
to convert carbon dioxide and water into glucose.
Here is a table that summarizes the key differences between heterotrophs and autotrophs:
Characteristic Heterotrophs Autotrophs
Energy Other organisms Light
source
Carbon source Organic matter Carbon dioxide
Exam- Animals, fungi, most bacteria Plants, algae, some bacteria
ples

Q.17 Cove lighting is a type of indirect lighting focused


(A)Upward from an interior cornice at the edge of a ceiling
(B)Downward from an interior cornice at the edge of a ceiling
(C) Downward from an opal light source at the centre of the ceiling
(D) Simultaneously upward and downward from a front-concealed light
Ans:- (A)
Explanation:- Cove lighting is a type of indirect lighting that is focused upward from an interior cornice
at the edge of a ceiling. The light is reflected off the ceiling and walls, creating a soft, diffused light that
is ideal for accenting architectural features or creating a sense of spaciousness.
The other options are incorrect. Cove lighting is not focused downward, downward from an opal light
source at the centre of the ceiling, or simultaneously upward and downward from a front-concealed
light.
Here is a diagram of cove lighting:
As you can see, the light is directed upward from the cove,
which is a recess in the ceiling. The light is then reflected off
the ceiling and walls, creating a soft, diffused light.
Cove lighting is a popular choice for residential and commercial
spaces. It is a versatile type of lighting that can be used to create
a variety of effects.
Q.18 In contour analysis, ‘no cut no fill line
(A) joins points with same elevation
(B) Interconnects the points where new contours rejoin the old one
(C) Joins the highest and the lowest elevation
(D) Connects a ridge and a valley
526 GATE Architecture and Planning: Comprehensive Question Bank

Ans:- (A)
Explanation:- In contour analysis, a ‘no cut no fill line’ joins
points with the same elevation. This means that the line will
always be parallel to the ground surface.
The other options are incorrect. A ‘no cut no fill line’ does
not interconnect the points where new contours rejoin the
old one, join the highest and the lowest elevation, or connect
a ridge and a valley.
Here is a diagram of a ‘no cut no fill line’:
As you can see, the line is parallel to the ground surface and joins points with the same elevation.
‘No cut no fill lines’ are used in contour analysis to help identify areas that are likely to require grading
or excavation. By identifying these areas, engineers can ensure that the project is completed in a safe
and efficient manner.
Q.19 A sewage treatment unit that works on the principle of anaerobic decomposition of organic matter
is called
(A) Trickling filter (B) Oxidation pond (C) Septic tank (D) Activated sludge plant
Ans:- (C)
Explanation:- A septic tank is a sewage treatment unit that works on the
principle of anaerobic decomposition of organic matter. Septic tanks
are typically used in small, rural areas where there is no centralized
sewage treatment system.
The other options are incorrect. Trickling filters, oxidation ponds,
and activated sludge plants all work on the principle of aerobic
decomposition of organic matter.
Here is a diagram of a septic tank:
As you can see, the septic tank is divided into two chambers. The first chamber is where the solid
matter settles to the bottom. The second chamber is where the liquid waste is further broken down by
anaerobic bacteria.
The treated liquid waste from the septic tank is then discharged into the soil, where it is further broken
down by bacteria.
Septic tanks are a simple and effective way to treat sewage in small, rural areas. However, they require
regular maintenance to ensure that they function properly.
Q.20 The science of acquiring information about the earth’s surface without actually being in contact
with it is termed as
(A) Geographical Information Centre (B) Earth Science
(C) Remote Sensing (D) Geomatics
Hemant Vilas Parulekar 527

Ans:- (C)
Explanation:- Remote sensing is a method of obtaining information about an object or area from a
distance, typically from an aircraft or satellite. Remote sensing is used in a variety of fields, including:
Earth science: Remote sensing is used to study the Earth’s surface, atmosphere, and oceans.
Environmental monitoring: Remote sensing is used to monitor environmental changes, such as
deforestation and desertification.
Military: Remote sensing is used to map enemy territory and to track troop movements.
Agriculture: Remote sensing is used to assess crop yields and to monitor crop health.
There are two main types of remote sensing: active and passive. Active remote sensing involves the use
of sensors that emit energy, such as radar or lidar. Passive remote sensing involves the use of sensors that
detect energy that is emitted by the Earth’s surface, such as sunlight.
Remote sensing is a powerful tool that can be used to obtain information about the Earth’s surface
without actually being in contact with it. This information can be used to improve our understanding
of the Earth and to make better decisions about how to manage the Earth’s resources.
Q.21 to Q.75 carry two marks each.
Q.21 The essential components of Buddhist Stupa architecture are
(A) Ratha, Vimana, Harmika (B) Antarala, Medhi, Vimana
(C) Torana, Ratha, Anda (D) Vedika, Pradakshina, Harmika
Ans:- (D)
Explanation: Please refer GATE 2009, Q. 28 for more information
Q.22 Match the cities in Group I with the planning concepts in Group II
(A) P-3,Q-1,R-2,S-4 Group I Group II
(B) P-2,Q-4,R-1,S-3 P. Radburn 1.Linear city
(C) P-1,Q-3,R-4,S-2 Q. Broadacre 2.Sectoral plan
R. Chandigarh 3.Town of the motor age
(D) P-4,Q-2,R-3,S-1
S. Letchworth 4. Garden city
Ans:- (A)
Q.23 The three qualities defining our perception of chromatic light are
(A) Hue, Tint, Shade (B) Brightness, Contrast, Tint
(C) Value, Hue, Intensity (D) Value, Brightness, Shade
Ans:- (C)
Explanation: Hue is the name of the color, such as red, blue, or green.
Value is the lightness or darkness of a color.
Intensity is the brightness or dullness of a color.
These three qualities can be used to describe any chromatic light. For example, a red light with a high
value would be a bright red light, while a red light with a low value would be a dark red light.
(Please refer GATE-2015, Q. 19 for more information)
528 GATE Architecture and Planning: Comprehensive Question Bank

Q.24 Match the items of building works in Group I with their units of measurement in Group II
(A) P-4,Q-2,R-3,S-1 Group I Group II
(B) P-4,Q-3,R-1,S-2 P. Brickwork 1. Square meter
(C) P-1,Q-3,R-4,S-2 Q. Steel work 2. Running meter
R. Plastering 3. Tone
(D) P-2,Q-4,R-3,S-1
S. Nosing 4. Cubic meter
Ans:- (B)
Q.25 The basic elements of Ekistics are
(A) Community,Land,Shell,Network,Image
(B) Society,Nature,Man,Shell,Network
(C) Community,Landscape,Utilities,Society,Transport
(D) Nature,Shell,Network,Administration,Society
Ans:- (B)
Explanation:-
Society: A group of people who live in a particular area and share a common culture.
Nature: The physical space that humans inhabit.
Man: The human element, including the physical, mental, and spiritual aspects of human beings.
Shell: The built environment, including buildings, infrastructure, and other artificial structures.
Network: The system of transportation and communication that connects people and places.
These five elements are interrelated and interdependent. For example, society depends on nature
for its resources, man interacts with nature and society, the shell provides society with shelter and
infrastructure, the network connects society to other societies, and nature is affected by the network.
Ekistics is a holistic approach to the study of human settlements. It seeks to understand the complex
interactions between the five elements of Ekistics and to develop strategies for the sustainable
development of human settlements.
The term Ekistics was coined by Constantinos Apostolos Doxiadis. He was a Greek architect and urban
planner who developed the theory of Ekistics in the 1950s.
Doxiadis’s work has had a significant impact on the field of urban planning. His ideas have been used to
plan cities around the world, including Brasilia, Brazil and Islamabad, Pakistan.
Q.26 The de Stijl movement in architecture was characterized by
(A) Rectangular forms (B) Stone Facades (C) Pyramidal Roofs (D) Primary Colours
Ans:- (A)
Explanation: The de Stijl movement in architecture, also known
as neoplasticism, was characterized by the use of straight lines,
right angles, and rectangular forms. It was a Dutch art movement
that emerged in the early 20th century and sought to create a
universal visual language based on geometric abstraction and
simplicity. The use of primary colors (red, blue, yellow) and black
and white was also a prominent feature of the de Stijl movement.
The goal was to create a sense of harmony and order through the
use of basic geometric shapes and primary colors.
Hemant Vilas Parulekar 529

Q.27 Match the illustrations in Group I with the types of roof in Group II
Group I

P Q R S
Group II
Gable Roof 2. Rainbow Roof 3. Gambrel Roof 4. Hip Roof
(A) P-1,Q-3,R-2,S-4 (B) P-4,Q-2,R-1,S-3 (C) P-4,Q-1,R-2,S-3 (D) P-3,Q-2,R-1,S-4
Ans:- (C)
Q.28 A line segment PQ is divided into two parts using the Golden Mean ratio.A is smaller than segment
B. The only relationship that holds true for the line PO is
(A) A/B = A/(A+B) (B) A/B = B/(A+B)
(C) (A+B)/A = (B-A)/B (D) (A+B)/A = (P+Q)/P
Ans:- (B)
Explanation: Two quantities are said to be in golden ratio, if their ratio is equal to the ratio of their sum
to the larger of the two quantities. The golden ratio is approximately equal to 1.618. For example, if “A”
and “B” are two quantities with B>A>0, the golden ratio is algebraically expressed as follow:
A/B = B/(A+B)
Q.29 A typical fire fighting underground static tank for a 20 m high building should have a pump capacity
of
(A) 2400 lit/hour and pressure of 3.0 N/mm2
(B) 2400 gallons/hour and pressure of 0.3 N/mm2
(C) 2400 gallons/min and pressure of 3.0N/mm2
(D) 2400 lit/min and pressure of 0.3 N/mm2
Ans:- Insufficient data
Q.30 The first four terms of a Fibonacci series are 1,1,2,3,......The sixth term of the series will be
(A) 5 (B) 8 (C)11 (D) 18
Ans:- (A)
Explanation: The Fibonacci Series is a sequence of numbers named after the Italian mathematician
Leonardo of Pisa, known as Fibonacci. This series has a special characteristic that each number is
the sum of the two preceding ones, starting from 0 and 1. It’s one of the most famous formulas in
mathematics and has various applications in computer science, mathematics, and even in nature.
Formula:
The Fibonacci sequence typically starts with 0 (sometimes with 1) followed by 1, and each subsequent
number is the sum of the two preceding numbers. If F(n) represents the nth Fibonacci number, the
sequence is defined by:
530 GATE Architecture and Planning: Comprehensive Question Bank

F(0)=0,F(1)=1
F(n)=F(n−1)+F(n−2)for n > 1
First Few Numbers in the Sequence:
0, 1, 1, 2, 3, 5, 8, 13, 21, 34, 55, 89, 144, …
Properties:
Golden Ratio: As the numbers progress, the ratio of two consecutive Fibonacci numbers (e.g., 55/34,
89/55) approaches the Golden Ratio (approximately 1.618).
Divisibility: Every third number in the sequence is divisible by 2, every fourth number is divisible by 3,
and every fifth number is divisible by 5.
Adding Squares: The sum of the squares of consecutive Fibonacci numbers is another Fibonacci
number. For example, 12+12=212+12=2, 22+32=1322+32=13.
Pattern Formation: A pattern can be observed when Fibonacci numbers are arranged in a certain way,
such as spirals seen in pinecones, sunflowers, and snail shells.
Applications:
Mathematics: It appears in various mathematical concepts such as the Binet formula, a closed-form
expression for Fibonacci numbers.
Computer Algorithms: The sequence is used in algorithms, particularly those involving recursive
techniques and data structures like heaps.
Biology: The sequence appears in biological settings, like the branching of trees, the arrangement of
leaves on a stem, or the fruit sprouts of a pineapple.
Art and Architecture: The Fibonacci sequence and the Golden Ratio are often cited for their aesthetically
pleasing properties in art, architecture, and music.
Financial Markets: Some traders use Fibonacci numbers and the Golden Ratio for technical analysis
in trading strategies.
Q.31 Two loading diagrams P and Q are shown. Their corresponding maximum Bending Moments
(P and Q respectively) are

(A) WL/4,WL/8 (B) WL/2,WL/4 (C) WL/8,WL/4 (D) WL/4,WL/2


Ans:- (C)
Q.32 Match the city planners in Group I with the planning theories in Group II
(A) P-2,Q-3,R-4,S-1 Group I Group II
(B) P-4,Q-1,R-2,S-3 P. Patric Geddes 1. Mental map theory
(C) P-3,Q-4,R-1,S-2 Q. Gorden Cullen 2. Cities in transition
R. Kevin Lynch 3. Diagnostic approach to planning
(D) P-4,Q-2,R-1,S-3
S. Lewis Mumford 4. Serial vision
Ans:- (C)
Hemant Vilas Parulekar 531

Q.33 Urban renewal is best explained as


(A)Renewal of expired building permit
(B)Renewal of byelaws of a commercial area
(C)Technique to limit the population growth
(D)Removal of blight and prevention from spreading to other parts.
Ans:- (D)
Explanation:- Blight is a term used to describe a deteriorated or rundown area of a city. Urban renewal
is a process of improving these areas by demolishing or renovating buildings, providing new public
services, and attracting new businesses.
The goal of urban renewal is to improve the quality of life for the residents of these areas and to make
them more attractive to businesses and investors.
Q.34 In a rectangular room of length 4 m and width 3 m, an electric bulb is to be fixed above the working
plane.If the room index is 1, the ideal mounting height of the bulb from the floor should be
(A) 2.3m (B) 1.33m (C) 1.7m (D) 2.0m
Ans:- (C)
Solution:- Room Index = (AreaH/AreaV) = (L x W)/[(L + W) x Hm]
Where Hm = Mounting height
Here R.I. = 1, L = 4, W = 3, Hence Hm = 1.7m
Q.35 Match the architectural terms in Group I with their descriptions in Group II
(A) P-3,Q-4,R-1,S-2 Group I Group II
(B) P-4,Q-1,R-3,S-2 P. Trombe wall 1. A black water filled vessel absorbing solar radiation
(C) P-3,Q-1,R-2,S-4 Q. Brise-soleil 2. Glass enclosed space for sun bathing
(D) P-2,Q-1,R-3,S-4 R. Drum wall 3. Glass-front masonry absorbing solar radiation
S. Solarium 4. Screen to shield the interiors from sunlight

Ans:- (A)
Explanation:
Term Trombe Wall Brise Soleil
Description Glass-front masonry absorbing Screen to shield the interiors from
solar radiation sunlight
View
532 GATE Architecture and Planning: Comprehensive Question Bank

Term Drum wall (wafer wall) Solarium


Description A black water filled vessel Glass enclosed space for sun
absorbing solar radiation bathing
View

Q.36 The three components of Christaller’s theories of settlement system are


(A) Production, transport, centrality (B) Population, production, hierarchy
(C) Market, service, hierarchy (D) Production, market, transport
Ans:- (D)
Explanation:- Christaller’s central place theory is a model of how settlements are distributed in an area.
He proposed that settlements are arranged in a hierarchical system, with larger settlements providing a
wider range of goods and services than smaller settlements.
The three components of the theory are:
Production: The level of production in an area determines the size of the settlements that can be
supported.
Market: The market area of a settlement is the area within which people are willing to travel to obtain
goods and services.
Transport: The cost of transport affects the size of the market area and the level of specialization of
settlements.
Christaller’s theory has been used to explain the distribution of settlements in many different parts of
the world. It is a useful tool for understanding the spatial organization of economic activity.
Q.37 Match the valuation terms in Group I with their descriptions in Group II
(A) P-3,Q-1,R-2,S-4 Group I Group II
(B) P-2,Q-3,R-4,S-1 P. Scrap value 1. Value at the end of the utility period
(C) P-1,Q-2,R-3,S-4 Q. Salvage value 2. Value in present transaction
(D) P-3,Q-4,R-1,S-2 R. Market value 3. Value of dismantled material
S. Book value 4. Value after deducting the depreciation
Ans:- (A)
Explanation:
Scrap value: The scrap value of an asset is the value of the materials that can be recovered from it after
it has been scrapped.
Salvage value: The salvage value of an asset is the value of the asset at the end of its useful life.
Market value: The market value of an asset is the price that the asset would sell for in a competitive
market.
Book value: The book value of an asset is the value of the asset as shown on the balance sheet. It is
calculated as the original cost of the asset minus the accumulated depreciation.
Hemant Vilas Parulekar 533

Q.38 Match the urban forms Group I with their descriptions in Group II
(A) P-1,Q-3,R-4,S-2 Group I Group II
(B) P-2,Q-3,R-1,S-4 P. Satellite 1. Urban centre around large open space
(C) P-2,Q-4,R-3,S-1
Q. Constellations 2. Urban form around a main centre
(D) P.1,Q-2,R-3,S-4
R. Ring 3. Similar sized urban centres in close proximity
S. Radio centric 4. Circular form with radial corridors
Ans:- (B)
Q.39 Match the arches in Group I with their names in Group II

1. Segmental 2. Equilateral 3. Flat 4. Semi-circular


(A) P-3,Q-1,R-4,S-2 (B) P-4,Q-1,R-2,S-3 (C) P-3,Q-4,R-2,S-1 (D) P-2,Q-3,R-1,S-4
Ans:- (C)
Q.40 The correct chronological sequence of the following buildings is
(A) Lingaraj temple-Sanchi Stupa-Padmanabhapuram Palace-Fatehpur Sikri-Gol Gumbaz
(B) Lingaraj temple-Sanchi Stupa-Fatehpur Sikri-Padmanabhapuram Palace-Gol Gumbaz
(C) Sanchi Stupa-Lingaraj temple-Gol Gumbaz-Fatehpur Sikri-Padmanabhapuram Palace
(D) Sanchi Stupa-Lingaraj temple-Padmanabhapuram Palace-Fatehpur Sikri-Gol Gumbaz
Ans:- (C)
Q.41 The components of energy embodied in a building material are
(P) Obtaining government approval for quarrying
(Q) Quarrying of the raw material
(R) Transporting the raw material to the manufacturing unit
(S) Manufacturing the building material
(A) P,R,S (B) P,Q,S (C) Q,R,S (D) P,Q,R
Ans:- (C)
Q.42 Territoriality refers to a behavioral setting or space that
A person will personalize,mark,own and defend
Moves with the person
Oscar Newman conceptualized as Defensible Space
Deals with encroachment
(A) P,Q,R (B) P,R,S (C) Q,R,S (D) P,Q,S
534 GATE Architecture and Planning: Comprehensive Question Bank

Ans:- (A)
Explanation:
Personalization: People often personalize their space by marking it with their belongings or by engaging
in activities that make it their own.
Marking: People often mark their space by using physical objects, such as fences or signs, or by using
symbolic objects, such as flags or religious symbols.
Ownership: People often feel a sense of ownership over their space, and they may defend it against
encroachment by others.
Encroachment: Encroachment is the invasion of someone else’s territory. It can be a physical invasion,
such as when someone enters your home without permission, or it can be a symbolic invasion, such as
when someone uses your belongings without your permission.
Oscar Newman conceptualized defensible space as a way to reduce crime in urban neighborhoods.
He argued that people are more likely to feel safe in their homes and neighborhoods if they have a
sense of ownership over their space. He also argued that defensible space can be created by designing
neighborhoods in a way that makes it easier for residents to identify and monitor their surroundings.
Q.43 Match the valves in Group I with their functions in Group 1l
Group I Group II
P. Gate valve 1. Prevents water flow back in the opposite direction
Q. Air valve 2. Regulates flow of water
R. Scour valve 3. Completely empties the pipe for inspection/repair
S. Reflex valve 4. Ensures safety of pipe against collapse
(A) P-2,Q-4,R-3,S-1 (B) P-3,Q-1,R-2,S-4 (C) P-1,Q-3,R-4,S-2 (D) P-4,Q-2,R-1,S-3
Ans:- (A)
Explanation:
Term Gate valve Air valve
Description Regulates flow of water Ensures safety of pipe against
collapse
View
Hemant Vilas Parulekar 535

Term Scour valve Reflux valve (Check valve/Non


return valve)
Description Completely empties the pipe Prevents water flow back in the
for inspection/repair opposite direction
View

Q.44 The most appropriate hierarchy of roads in an urban area is


(A) Freeway-Arterial-Local-Collector (B) Freeway-Arterial-Collector-Local
(C) Local-Arterial-Freeway-Collector (D) Collector-Local-Freeway-Arterial
Ans:- (B)
Explanation: The most appropriate hierarchy of roads in an urban area is (B) Freeway-Arterial-
Collector-Local.
The hierarchy of roads in an urban area is a system of roads that are organized in a way that allows for
the efficient movement of traffic. The hierarchy is typically divided into four levels: freeways, arterials,
collectors, and local roads.
Freeways are the highest level of road in the hierarchy. They are typically high-speed roads with limited
access. Freeways are used to move traffic between different parts of the city or region.
Arterials are the next level of road in the hierarchy. They are typically major roads that connect freeways
to other arterials, collectors, and local roads. Arterials are used to move traffic within a city or region.
Collectors are the third level of road in the hierarchy. They are typically smaller roads that connect
arterials to local roads. Collectors are used to move traffic within neighborhoods and communities.
Local roads are the lowest level of road in the hierarchy. They are typically residential streets and service
roads. Local roads are used to provide access to homes, businesses, and other facilities.
The hierarchy of roads in an urban area is important because it allows for the efficient movement of
traffic. The different levels of roads are designed to handle different types of traffic, and the hierarchy
helps to ensure that traffic is routed in the most efficient way possible.
Q.45 Match the structural systems in Group I with the force distribution pattern in Group II
Group I Group II
P. Flying Buttress 1. High tensile strength
Q. Catenary 2. Outward and downward thrust
R. Simply supported truss 3. Comprehensive,tensile and shear stresses acting
S. Shell in the plane of the surface 4. Axial tension and compression
(A) P-2,Q-1,R-4,S-3 (B) P-3,Q-1,R-2,S-4 (C) P-1,Q-3,R-4,S-2 (D)P-4,Q-2,R-1,S-3
536 GATE Architecture and Planning: Comprehensive Question Bank

Ans:- (A)
Explanation:
Flying buttress: A flying buttress is a structural support that is used to resist the outward thrust of a
vault or arch. The buttress is typically made of masonry and is anchored to the wall of the building. The
buttress distributes the thrust of the vault or arch over a wider area, which helps to prevent the wall from
collapsing.
Catenary: A catenary is a curve that is formed by a chain or cable hanging freely between two points.
The catenary is a shape of high tensile strength, and it is often used in the design of bridges and other
structures.
Simply supported truss: A simply supported truss is a truss that is supported at each end by a roller or
pin. The truss distributes the load over its members in a way that minimizes the stresses.
Shell in the plane of the surface: A shell is a structural element that is curved in two directions. Shells
are often used in the construction of roofs and other structures. Shells can resist a variety of stresses,
including tensile, compressive, and shear stresses.
Q.46 Match the survey questions regarding water supply in Group I with their scaling types in Group II

(A) P-1,Q-3,R-2 Group I Group II


(B) P-2,Q-3,R-1 P. Is the water supply sufficient? 1. Ordinal
(C) P-3,Q-1,R-2 Q. How is the quality of water? 2. Nominal
R. What is the quantity of water 3. Interval
(D) P-2,Q-1,R-3
supply/day?
Ans:- (C)
Explanation: Please refer GATE-2003, Q.63 for more information.
Q.47 Determine the correctness of the following Assertion [a]and Reason[r]
[a]: In cold climate while glazing can act as a heat trap, it can also cause significant losses at other times.
[r]: Since glass has a very low U-value, conductive heat loss takes place during night.
(A) Both[a]and[r]are True and [r] is the correct reason for [a]
(B) Both[a]and[r]are True but [r] is not the correct reason for [a]
(C) Both[a]and[r]are False
(D) [a]is True but[r]is False
Ans:- (A)
Q.48 Two 50 cm cubes of concrete are cast in a Building Materials Testing Laboratory, One of grade M15
and the other M20. The specified characteristic compressive strength of the stronger block at 28
days will be
(A)15 N/mm2 (B)20N/mm2 (C)1.5N/mm2 (D)0.88 N/mm2
Ans:- (B)
Explanation: The specified characteristic compressive strength of the stronger block at 28 days will be
(B) 20 N/mm2.
Hemant Vilas Parulekar 537

The characteristic compressive strength of concrete is the minimum compressive strength that 95% of
the tests results are expected to exceed. For grade M20 concrete, the specified characteristic compressive
strength is 20 N/mm2. This means that 95% of the test results for grade M20 concrete are expected to
be greater than or equal to 20 N/mm2.
The characteristic compressive strength of concrete is determined by testing concrete cubes. The
cubes are typically 50 cm in size and are tested at 28 days. The compressive strength of the concrete is
determined by the amount of force that is required to crush the cube.
In this case, the stronger block is the one that is made of grade M20 concrete. The characteristic
compressive strength of the stronger block will be 20 N/mm2.
The other options are incorrect. Option (A) is the characteristic compressive strength of grade M15
concrete. Option (C) is the characteristic compressive strength of a 1.5 cm cube of concrete. Option (D)
is the characteristic compressive strength of a 0.88 cm cube of concrete.
Q.49 Match the plants in Group I with their prime uses in Group II
Group I Group II
(A) P-3,Q-2,R-4,S-1
P. Bignonia magnifica 1. Grass cover
Q. Ixora chinensis 2. Wall climbers (B) P-4,Q-1,R-2,S-3
R. Cassia fistula 3. Topiary (C) P-4,Q-3,R-1,S-2
S. Wedelia trilobata 4. Arboriculture (D) P-2,Q-3,R-4,S-1
Ans:- (D)
Explanation:
Term Bignonia magnifica Ixora chinensis
Description Wall climbers Topiary
View

Term Cassia fistula Wedelia trilobata


Description Arboriculture Grasscover
View

Q.50 For natural illumination, Daylight factor is defined as the sum of


(A) Sky Component, External Reflected Component, Internal Reflected Component
(B) Sky Component, Reflected Component, Absorption Component
(C) Outdoor Component, Indoor Component, Reflected Component
(D) Azimuth Component, Altitude Component, Sky Component
538 GATE Architecture and Planning: Comprehensive Question Bank

Ans:- (A)
Explanation:
Daylight factor is a measure of the amount of daylight that enters a building through windows or other
openings. It is expressed as a percentage of the light that would enter a room if it were completely open
to the sky.
The daylight factor is the sum of three components:
Sky component: This is the light that comes directly from the sky.
External reflected component: This is the light that is reflected from the ground, buildings, or other
surfaces outside the building.
Internal reflected component: This is the light that is reflected from the walls, ceiling, and other
surfaces inside the building.
The daylight factor is an important consideration in building design, as it can affect the amount of
artificial lighting that is required. A high daylight factor can help to reduce energy costs and improve
the quality of the indoor environment.
DF = SC + ERC + IRC
Where:
SC stands for Sky Component,
ERC stands for External Reflected Component, and
IRC stands for Internal Reflected Component.
Q.51 In decision making theory, a mathematical model is represented as Y = a.Xb.Zc where
(A) Y-Independent variable;X, Z-Dependent variables;a,b,c.-constants
(B) Y-Dependent variable;X, Z-Independent variables;a,b,c-constants
(C) Y-Independent variable;X, Z-dependent variables;a,b,c-non-constants
(D) Y-Dependent variable;X, Z-Independent variables;a,b,c-non-constants
Ans:- (B)
Q.52 Group I Group II
P. Trip Generation 1. Multinomial Logit model
Q. Trip Distribution 2. Minimum Tree search
R. Mode choice 3. Gravity Model
S. Trip Assignment 4. Regression model
Match the components of travel demand in Group I with the forecasting models in Group II
(A)P-4,Q-3,R-1,S-2
(B)P-4,Q-2,R-1,S-3
(C)P-3,Q-4,R-2,S-1
(D)P-1,Q-4,R-2,S-3
Hemant Vilas Parulekar 539

Answer:- (A)
Explanation:
Explanation: Group I represents the components of travel demand, and Group II represents the
forecasting models.
(P) Trip Generation: It estimates the number of trips generated by a zone or an area. It is modeled using
a Regression Model.
(Q) Trip Distribution: It estimates the distribution of trips between different zones or areas. It is modeled
using a Gravity Model.
(R) Mode Choice: It estimates the mode of transportation chosen by travelers for their trips. It is modeled
using a Multinomial Logit Model.
(S) Trip Assignment: It assigns the estimated trips to specific routes or transportation networks. It is
modeled using Minimum Tree Search.
Q.53 For a two way RCC slab, the Length to Width(L/W) ratio should be
(A) 0.5L/W<2.0 (B) 2.0<L/W<3.0 (C) 2.5SL/W<4.0 (D) 0.0<L/W<0.5
Ans:- (B)
Q.54 Match the architects in Group I with their architectural works in Group II
Group I Group II
P. A.P. Kanvinde 1. Gandhi Labour Institute, Ahmedabad
Q. Anant Raje 2. IIFM, Bhopal
R. B.V. Doshi 3. Balotra City Hall, Balotra
S. U.C. Jain 4. NDDB Building, New Delhi
(A) P-3,Q-1,R-2,S-4
(B) P-2,Q-3,R-4,S-1
(C) P-1,Q-4,R-3,S-2
(D) P-4,Q-2,R-1,S-3
Ans:- (D)
Explanation:
Term Gandhi Labour Institute, IIFM, Bhopal
Ahmedabad
Description A.P. Kanvinde Anant Raje
View
540 GATE Architecture and Planning: Comprehensive Question Bank

Term Balotra City Hall, Balotra NDDB Building, New Delhi


Description B.V. Doshi U.C. Jain
View

Q.55 Low COD to BOD ratio of an organic pollutant represents


(A) Low biodegradability of the pollutant
(B) Presence of free oxygen for aerobic decomposition
(C) High biodegradability of the pollutant
(D) High arsenic in the pollutant
Ans:- (B)
Explanation: A low COD to BOD ratio in an organic pollutant indicates that there is a higher presence
of free oxygen available for aerobic decomposition. In other words, the pollutant can be more easily
broken down by microorganisms during the natural biological treatment process due to the availability
of oxygen. This generally suggests that the pollutant has higher biodegradability.
Q.56 Match the types of pipes in Group I with their functions in Group II
(A) P-1,Q-4,R-2,S-3 Group I Group II
(B) P-4,Q-1,R-2,S-3 P. Soil pipe 1. Carries liquid that does not include human excreta
(C) P-2,Q-3,R-4,S-1 Q. Waste pipe 2. Provides flow of air to or from drainage system
and also escape of foul gases
(D) P-4,Q-1,R-3,S-2
R. Vent Pipe 3. Preserves the water seal of traps through access to
atmospheric air
S. Anti-siphonage 4. Carries liquid waste including human excreta
pipe
Ans:- (B)
Q.57 The distance between two points on a map of scale l:40,000 is 3.6 cm.The distance between the same
two points in an aerial photograph is 6 cm.The scale of the aerial photograph is
(A) 1:12000 (B) 1:2400 (C) 1:240000 (D) 1:24000
Ans:- (B)
Solution: Scale of map – 1:40000
i.e., 1cm of map equals 40,000cm on ground
Thus 3.60cm of map equals 1,44,000cm on ground
6cm on aerial photograph equals 1,44,000cm on groun
Hemant Vilas Parulekar 541

Q.58 Group I Group II


P. Complementary colours 1. Forming an equilateral triangle
Q. Analogous colours 2. A radial segment
R. Monochromatic colours 3. Adjacent to each other
S. Secondary colours 4. Opposite sides of each other
Match the colour schemes in Group I with their positions in the Colour Wheel in Group II
(A) P-4,Q-1,R-2,S-3
(B) P-1,Q-2,R-3,S-4
(C) P-4,Q-3,R-2,S-1
(D) P-1,Q-3,R-4,S-2
Ans:- (C)
Explanation: Please refer GATE-2015, Q.15 for more information.
Q.59 The figure shows the Mass Curves of supply and demand for an Elevated Storage Reservoir(ESR).
The minimum required capacity of ESR is given by
(A) (m+n)
(B) (m-n)
(C) (mxn)
(D) larger of m or n

Ans:- (A)
Explanation: Required standard supply = Average highest demand + Averade lowest demand
Q.60 Match the architectural styles in Group I with their features in Group II
(A) P-4,Q-2,R-3,S-1 Group I Group II
(B) P-4,Q-3,R-1,S-2 P. Greek 1. Hypostyle hall
(C) P-3,Q-4,R-2,S-1 Q. Buddhist 2. Triumphal Arch
R. Roman 3. Parthenon
(D) P-2,Q-1,R-4,S-3
S. Egyptian 4. Chaitya hall
Ans:- (C)
Explanation:
Term Hypostyle Hall Triumphal Arch
Description Egyptian Roman
View
542 GATE Architecture and Planning: Comprehensive Question Bank

Term Parthenon Chaitya Hall


Description Greek Buddhist
View

Q.61 The correct sequence of stages in the building construction process is


(A) Bidding-Contract documents-Occupancy certificate-Contract award Post occupancy evaluation
(B) Contract documents-Bidding-Contract award-Occupancy certificate-Post occupancy evaluation
(C) Contract documents-Contract award-Bidding-Occupancy certificate-Post occupancy evaluation
(D) Contract documents Bidding-Contract award-Post occupancy evaluation-Occupancy certificate
Ans:- (B)
Explanation: (B) Contract documents-Bidding-Contract award-Occupancy certificate-Post
occupancy evaluation
The building construction process typically follows these steps:
Preliminary planning: This stage involves the identification of the need for a new building, the
development of a preliminary design, and the estimation of the cost of the project.
Detailed planning: This stage involves the development of detailed plans and specifications for the
building, as well as the procurement of permits and approvals.
Bidding: This stage involves the solicitation of bids from contractors to build the project.
Contract award: This stage involves the selection of a contractor and the signing of a contract.
Construction: This stage involves the actual construction of the building.
Occupancy certificate: This is a document issued by the local authority that allows the building to be
occupied.
Post occupancy evaluation: This is a process of assessing the performance of the building after it has
been occupied.
Q.62 A camera is used to shoot an aerial photograph from a flight.The scale of the photograph is 1:40000.
If the flying height above mean sea level is 7500 m, and the mean ground level is 1500 m, then the
focal length of the camera lens is
(A) 70mm (B) 150mm (C) 200mm (D) 50mm
Ans:- (B)
Solution: The scale of an aerial photograph is the ratio of the distance between two points on the
photograph to the distance between the same two points on the ground.
In this case, the scale of the photograph is 1:40,000, which means that every 1 cm on the photograph
corresponds to an actual distance of 40,000 cm on the ground.
Hemant Vilas Parulekar 543

The flying height above mean sea level is 7,500 m, and the mean ground level is 1,500 m, so the actual
distance between the two points on the ground is 9,000 m.
The focal length of the camera lens is the distance between the lens and the film plane.
The focal length of the camera lens is calculated using the following formula:
f = H / (S * X)
where:
f is the focal length of the camera lens in millimeters
H is the flying height above mean sea level in meters
S is the scale of the photograph
X is the ground distance in meters
Substituting the values from the problem, we get:
f = 7500 / (1/40000 * 9000) = 150 mm
Therefore, the focal length of the camera lens is 150 mm.
Q.63 Match the land use in Group I with the prime activities in Group II
Group I Group II
P. Commercial 1. Crematorium and burial grounds
Q. Public and semi-public 2. Truck terminals
R. Recreational 3. Hotels
S. Transport 4. Parks and gardens
(A)P-2,Q-1,R-3,S-4
(B)P-2,Q-3,R-1,S-4
(C)P-3,Q-4,R-1,S-2
(D)P-3,Q-1,R-4,S-2
Ans:- (D)
Q.64 The Census of India defines Urban Area’s an area with minimum
P. Population of 5000
Q. Population of 10000
R. 75% of population engaged in non-agricultural activities
S. 75% of male population engaged in non-agricultural activities
T. Density of 400 persons per sq.km
U. Density of 400 persons per hectare
(A) P,S,T (B) Q,S,T (C) P,R,U (D) Q,R,T
Ans:- (A)
Q.65 In a multipurpose hall, 30 tube lights of 40 W each are switched on for 5 hours. The electric meter
will record a power consumption of
(A) 6 units (B) 12 units (C) 30 units (D) 60 units
544 GATE Architecture and Planning: Comprehensive Question Bank

Ans:- (A)
Solution: The power consumption of the 30 tube lights is 30 * 40 = 1200 watt.
The power consumption in 5 hours is 1200 * 5 = 6000 watt-hour.
The electric meter will record a power consumption of 6 units.
Q.66 Match the architects in Group I with their design philosophies in Group II
Group I Group II (A) P-2,Q-1,R-4,S-3
P. Frank Lloyd 1. Less is bore (B) P-4,Q-1,R-2,S-3
Q. Robert Venturi 2. House is a machine for living (C) P-1,Q-3,R-2,S-4
R. Mies van der Rohe 3. Less is more
(D) P-4,Q-1,R-3,S-2
S. Le Corbusier 4. Organic Architecture
Ans:- (D)
Q.67 In a linear regression model, R2 = 0 indicates
(A) Perfect positive correlation between variables
(B) Moderate positive correlation between variables
(C) No correlation between variables
(D) Perfect negative correlation between variables
Ans:- (C)
Explanation: Correlation is the relationship between two or more variables, whih vary in sympathy
with the other in the same or the other direction. Regression means going back and it is a mathematical
measure showing the average relationship between two variables.
1. The regression co-efficient, R2 = 0 shows no correlation between variables.
2. R = 1 shows positive correlation between variables.
3. R = -1 shows negative correlation between variables.
Q.68 Match the properties of illumination in Group I with their units in Group Il
(A) P-3,Q-4,R-2,S-1 Group I Group II
(B) P-2,Q-4,R-3,S-1 P. Luminous Intensity 1. Lambert
(C) P-2,Q-1,R-4,S-3 Q. Illumination 2. Candela
(D) P-3,Q-1,R-2,S-4 R. Luminous flux 3. Lumen
S. Luminance 4. Lux
Ans:- (B)
Explanation: Here is a brief explanation of each property of illumination and its unit:
Luminous intensity: This is the amount of light emitted in a particular direction by a source. The unit
of luminous intensity is the candela.
Illumination: This is the amount of light falling on a surface. The unit of illumination is the lux.
Luminous flux: This is the total amount of light emitted by a source. The unit of luminous flux is the
lumen.
Luminance: This is the amount of light emitted from a surface in a particular direction per unit area.
The unit of luminance is the Lambert.
Hemant Vilas Parulekar 545

Q.69 Le Corbusier’s “Five Points of a New Architecture” are


P. Pilotis
Q. Ribbon windows
R. Ornamental grills
S. Free facade
T. Roof gardens
U. Sloping roofs
V. Louvres
W. Free plan
(A) P,Q,S,T,W (B) P,Q,R,U,W (C) Q,S,T,V,W (D) Q,R,U,V,W
Ans:- (A)

Q.70 Match the architectural styles in Group I with the buildings in Group II
(A) P-4,Q-3,R-2,S-1 Group I Group II
(B) P-3,Q-1,R-4,S-2 P. Metabolism 1. Humana Buildling, Kentucky
(C) P-4,Q-2,R-1,S-3 Q. Post Modernism 2. Guggenheim Museum, Bilbao
R. Brutalism 3. Sony Tower, Tokyo
(D) P-2,Q-3,R-4,S-1
S. Deconstructivism 4. United de Habitation, Paris
Ans:- (B)
Common Data for Questions 71,72,73
A town with an area of 340 hectares has 15,000 households. The number of occupied dwelling units
is 12,400 of which 15% are dilapidated condition. 5% of the households are below poverty line and
unable to afford any type of dwelling.
(Note:Dwelling Unit=DU)
Q.71 The present density of the town (in DU/acre) is approximately
(A) 15 (B) 18 (C) 37 (D) 44
Ans:- (A)
Solution: The number of households below poverty line = 5% of 15,000 = 750
Number of occupied dwelling units in good condition = 12400 * (100-15)% = 10,860
Total number of dwelling units = 10,860 + 750 = 11,610
Area of 1 acre = 0.4046 hectares
Therefore, the present density of the town = (11610* 0.4046) /340 = 13.8159 DU/acre ≈ 15 DU/acre
546 GATE Architecture and Planning: Comprehensive Question Bank

Q.72 The housing need of the town is approximately


(A) 1860 DU (B) 2600 DU (C) 3320 DU (D) 4460 DU
Ans:- (B)
Solution: The number of households below poverty line = 5% of 15,000 = 750
Number of occupied dwelling units in good condition = 12400 * (100-15)% = 10,860 Dwelling units
required for households below poverty line = 750
Dwelling units required for dilapidated dwelling units = 15% of 12400 = 1860
Total housing need of the town = 1860 + 750 = 2610 DU
Q.73 The housing demand of the town is approximately
(A) 1240 DU (B) 1860 DU (C) 2470 DU (D) 3710DU
Ans:- (D)
Solution: The housing demand of the town is the total number of dwelling units that are required by
the households in the town.
The number of households below poverty line = 5% of 15,000 = 750
Number of occupied dwelling units in good condition = 12400 * (100-15)% = 10,860 Dwelling units
required for households below poverty line = 750
Dwelling units required for dilapidated dwelling units = 15% of 12400 = 1860
Total housing need of the town = 1860 + 750 = 2610 DU
However, the housing demand of the town may be higher than the housing need of the town, as some
households may want to upgrade their existing dwelling units.
Let’s assume that 10% of the households in the town want to upgrade their existing dwelling units. The
total housing demand of the town is then:
2610 + (10/100) * 15,000 = 3710 DU
Common Data for Questions 74, 75:
Population for an urban area for the period of 1971-2001 is given in the following table:
Year Population Increase in Population % increase in population
1971 80000 - -
1981 95000 15000 18375
1991 115000 20000 21.05
2001 140000 25000 21.74
Q.74 The projected population for 2021 using Arithmetic increase is
(A) 64000 (B) 180000 (C) 220000 (D) 240000
Hemant Vilas Parulekar 547

Ans:- (B)
Solution: Arithmetic population projection method:
dp/dt = ka (where, dp = change in population & dt = time period
p = p1 + k (t – t1)
here dp = 1,40,000 – 80,000 = 60,000
dt = 30
Thus, ka = 60,000/30 = 2000
Thus, p2021 = 1,40,000 + 2000 (20) = 1,80,000
Q.75 The projected population (in thousands) for 2021 using Geometric increase method is
(A) 16880 (B) 180000 (C) 203180 (D) 253000
Ans:- (C)
Solution: The formula for geometric increase is:
Population = P * (1 + r)n
where:
P is the initial population
r is the rate of increase
n is the number of years
The initial population is 80,000.
The rate of increase can be calculated as follows:
r = (Increase in Population / Initial Population)(1/Number of Years)
r = (15,000 / 80,000)(1/10) = 0.18375
The number of years is 2021 - 1971 = 50 years.
The projected population for 2021 is:
Population = 80,000 * (1 + 0.18375)50 = 203,180
Linked Answer Questions:-
Q.76 to Q.83 carry two marks each.
Statements for linked answer questions 76 and 77:
While testing the strength of a steel beam, if it is found that. Poisson’s ratio for steel = 0.3 and
Young’s modulus of elasticity 2.10 x 106
Q.76 If the lateral strain of beam is 1 unit, the longitudinal strain would be
(A) 7 x 106 units (B) 0.3 units (C) 2.10 x 106 units (D) 3.33 units
548 GATE Architecture and Planning: Comprehensive Question Bank

Ans:- (D)
Solution: The lateral strain is related to the longitudinal strain by the following equation:
lateral strain = -nu * longitudinal strain
where nu is the Poisson’s ratio for steel.
In this case, we are given that nu = 0.3 and the lateral strain is 1 unit. Solving for the longitudinal strain,
we get:
longitudinal strain = -1 / 0.3 = -3.33 units
Therefore, the longitudinal strain would be 3.33 units.
Q.77 If the longitudinal strain of beam is 1 unit, the longitudinal strain would be
(A) 7 x 106 units (B) 0.3 units (C) 2.10 x 106 units (D) 3.33 units
Ans:- Here, longitudinal stress (σL) = ?
Young’s Modulus of elasticity (E) = longitudinal stress (σL) / longitudinal strain
Thus, longitudinal stress (σL) = Modulus of elasticity (E) x longitudinal strain
= 2.10 x 106 x 3.33 = 6.993 x 106 units = 7 x 106 units
Statements for linked answers. questions 78 and 79:
Every glazing material (e.g. glass) has three properties which defines its functioning:
Reflectivity (coefficient = R), Transmissivity (coefficient = T), Absorptivity (coefficient = A)

Q.78 The relationship that holds true for the thermal performance of glass is
(A) R + T = A (B) R + T + A = 0 (C) R = T + A (D) R + T + A = 1
Ans:- The reflectivity, transmissivity, and absorptivity of a glazing material represent the amount of
solar radiation that is reflected, transmitted, and absorbed by the material, respectively. The sum of
these three properties must always equal 1, because all of the solar radiation that strikes the glazing
material must be accounted for.
Option (A) is incorrect because the transmissivity and absorptivity cannot be equal to each other.
Option (B) is incorrect because the sum of the three properties cannot be equal to 0. Option (C) is
incorrect because the reflectivity cannot be equal to the transmissivity plus the absorptivity.
Here is a diagram that illustrates the relationship between the reflectivity, transmissivity, and absorptivity
of a glazing material:
Solar radiation
-------->
Reflectivity (R)
-------->
Transmission (T)
-------->
Absorption (A)
As you can see, the sum of the reflectivity, transmissivity, and absorptivity is equal to 1.
Hemant Vilas Parulekar 549

Q.79 If for glass, R = 0.6 and T = 0.1, A will be equal to


(A) -0.7 (B) 0.3 (C) 0.5 (D) 0.7
Ans:- (B)
Explanation: The sum of the reflectivity (R), transmissivity (T), and absorptivity (A) coefficients for
any glazing material is equal to 1. This is because all incident light is either reflected, transmitted, or
absorbed by the material.
Given that R = 0.6 and T = 0.1, we can find the value of A using the equation:
A=1-R-T
A = 1 - 0.6 - 0.1 A = 0.3
So, the value of A is 0.3, and the correct answer is (B) 0.3.
Statements for linked answer questions 80 & 81:
For a PERT network analysis, an activity is likely to have:
A. Optimistic duration of 2 days B. Pessimistic duration of 7 days C. Modal duration of 3 days
Q.80 The expected time duration that maybe allocated in a PERT scheduling would be
(A) 2 days (B) 3 days (C) 3.5 days (D) 4 days
Ans:- (C)
Solution: In PERT analysis, the expected time duration is calculated as follows:
Expected time = (optimistic time + 4 * modal time + pessimistic time) / 6
In this case, the optimistic time is 2 days, the pessimistic time is 7 days, and the modal time is 3 days.
So, the expected time is:
Expected time = (2 + 4 * 3 + 7) / 6 = 3.5 days
Therefore, the expected time duration that may be allocated in a PERT scheduling is 3.5 days.
Program Evaluation Review Technique (PERT) Analysis: Deep Dive
PERT analysis is a powerful project management tool used to plan, schedule, and control complex
projects. It helps determine the time required to complete tasks, identify potential bottlenecks, and
optimize resource allocation to ensure timely project completion.
Unveiling the Components:
Tasks and Activities: The project is broken down into smaller, manageable tasks with clearly defined
start and end points. Tasks can be dependent on each other, meaning some can only begin once others
are finished.
Time Estimates: For each task, three time estimates are made:
Optimistic time (t_o): The shortest possible time to complete the task under ideal conditions.
Most likely time (t_m): The most realistic time considering normal conditions and expected obstacles.
Pessimistic time (t_p): The longest possible time under unforeseen circumstances or delays.
PERT Network Diagram: Tasks and their dependencies are visually represented using a network
diagram, with arrows indicating the sequence of completion. Connections show which tasks must be
finished before others can begin.
550 GATE Architecture and Planning: Comprehensive Question Bank

The Magic of PERT Calculations:


PERT uses statistical formulas to calculate the expected time and variance for each task and the entire
project. These calculations consider the range of possible completion times (optimistic, most likely,
pessimistic) and their associated likelihoods.
Here’s the Math:
Expected time for a task (t_e) = (t_o + 4t_m + t_p) / 6
Variance for a task (σ2) = (t_p - t_o)2 / 36
Unraveling the Critical Path:
The critical path is the longest sequence of dependent tasks that determines the overall project duration.
Identifying the critical path is crucial as any delays in these tasks directly impact the entire project
timeline.
Benefits of PERT Analysis:
Realistic project scheduling: By considering various time estimates, PERT provides a more accurate
picture of project duration compared to deterministic methods.
Early risk identification: Potential bottlenecks and critical tasks are highlighted, allowing for proactive
risk management and resource allocation.
Improved project communication: The visual representation of tasks and dependencies facilitates
clear communication and collaboration among team members.
Flexibility and adaptability: PERT readily adapts to changes in project scope or task duration,
minimizing disruptions and delays.
PERT in Action:
PERT analysis finds applications in various industries, including construction, software development,
product launch, and research projects. Its flexibility and adaptability make it a valuable tool for managing
complex projects with diverse tasks and dependencies.
Visualizing PERT:
A PERT network diagram with tasks, dependencies, and estimated times:

A Gantt chart showing tasks, durations, and critical path alongside a timeline
By understanding the nuances of PERT analysis, project managers can effectively plan, execute, and
monitor complex projects, enhancing their chances of success. Remember, with a clear roadmap and
data-driven insights, your project can navigate its way to completion efficiently and flawlessly.
Hemant Vilas Parulekar 551

Q.81 If the same activity has free float of 2 days, then the maximum time that may be allocated to it
without hampering that start of the succeeding activity is
(A) 1.5 days (B) 2 days (C) 4 days (D) 5.5 days
Ans:- (B)
Explanation: Free Float: The amount of time that a schedule activity can be delayed without delaying
the early start date of any immediately following schedule activities.
Free Float = ES of next activity - EF
Q.82a An auditorium of size 30m x 20m is 8m high. Assuming optimum reverbation time of 1.2 seconds
and existing absorption power of the hall as 300m2 sabins, calculate the extra absorption units
required.
(A) 240m2 sabins (B) 340m2 sabins (C) 380m2 sabins (D) 450m2 sabins
Ans:- (B)
Solution: Here, V = 4800m3, R.T. = 1.2, A = ?
RT = (0.16 x V)/A
A = (1.2 x 4800)/1.2 = 640 m2 . Thus, extra absorption units required = 640-300 = 340 m2
Q.82b Extra absorption units in the above auditorium can be achieved by using indigenous acoustical
materials. If the area available for fixing acousticals material is 580 sq.m., out of which, 40 sq.m.
are opening covered with heavy curtains. The absorption coefficient of curtains is 0.50. Identify a
single acoustical material for the remaining area.
(A) Compressed wood particle board perforated (coefficient of absorption: 3.6)
(B) 18mm thich compounded wood particle board (coefficient of absorption: 0.60)
(C) Wood wool board (coefficient of absorption: 0.20)
(D) Straw board (coefficient of absorption: 0.30)
Ans:- (B)
Solution: Total absorption = area of material x absorption co-efficient
Thus, 340 = (Area of wall with a material of absorption co-efficient ‘x’ x absorption co-efficient of wall)
– (Area covered by curtains x absorption co-efficient of curtains)
= (540 x ‘x’) – (40 x 0.50)
Thus x = 60
Q.83a A city has a population of 2,50,000 persons in 2001 with a growth rate of 2% p.a. and occupied an
area of 1500 hectare. Estimate the suitable population for the year 2010. Estimate the additional
land requirement for the year 2010 assuming an average town density of 100 persons per hectare
(A) 3,00,000 population, 1500 ha (B) 3,50,000 population, 1735 ha
(C) 3,10,000 population, 1600 ha (D) 3,75,000 population, 1825 ha
552 GATE Architecture and Planning: Comprehensive Question Bank

Ans:- (B)
Solution: Calculation of population in 2010
The population of the city in 2001 is 2,50,000 persons. The growth rate of the population is 2% p.a. So,
the population of the city in 2010 will be:
Population in 2010 = Population in 2001 * (1 + growth rate)years
= 250000 * (1 + 2/100)9
= 350000
Calculation of additional land requirement
The average town density is 100 persons per hectare. So, the area occupied by the city in 2010 will be:
Area occupied = Population / Town density
= 350000 / 100
= 3500 ha
The additional land requirement for the year 2010 is:
Additional land requirement = Required area - Existing area
= 3500 ha - 1500 ha
= 2000 ha
Therefore, the suitable population for the year 2010 is 3,50,000 persons and the additional land
requirement for the year 2010 is 1,735 ha.
Q.83b If the existing land use under existing residential and commercial are 970 Ha and 105 Ha respectively,
and the proposed land use under residential and commercial are 52% and 45% respectively. Calculate
the additional land requirement under these:
(A) 712 Ha, 40.5 Ha (B) 759 Ha, 45 Ha (C) 590 Ha, 30 Ha (D) 642 Ha, 34.5 Ha
Ans:- (B)
Solution: Calculation of additional land requirement for residential
The proposed land use under residential is 52%, and the existing land use under residential is 970 Ha.
So, the additional land requirement for residential is:
Additional land requirement for residential = (52/100) * 970 = 504 Ha
Calculation of additional land requirement for commercial
The proposed land use under commercial is 45%, and the existing land use under commercial is 105 Ha.
So, the additional land requirement for commercial is:
Additional land requirement for commercial = (45/100) * 105 = 47.25 Ha
Therefore, the additional land requirement under residential and commercial are 504 Ha and 47.25 Ha
respectively.
Total additional land requirement
The total additional land requirement is:
504 Ha + 47.25 Ha = 551.25 Ha
So the answer is (B).
GATE QUESTION PAPER 2005
General Aptitude (GA)

Q.1-Q.30 carry one mark each.


Q.1 Early Aryan civilization is characterized by
(A) Introduction of arches and domes (B) Wooden construction
(C) Rock cut architecture (D) Flat brick masonry
Ans:- (B)
Q.2 Indus valley development is specially known for
(A) Vaulted roofing in masonry (B) Underground drainage system
(C) Great Palaces (D) Stone carving of idols
Ans:- (B)
Q.3 Housing backlog means
(A) Dwelling units back to back
(B) Dwelling units built up to last year
(C) Dwelling units arranged with their backs towards a large courtyard
(D) Accumulated substandard dwelling units to be replaced immediately
Ans:- (D)
Q.4 The prefabricated housing project “Habitat’ was designed by
(A) Le Corbusier (B) Kenzo Tange (C) VB Doshi (D) Moshe Safdie
Ans:- (D)
Q.5 Buckling of steel column is associated with the stress due to
(A) Shear force and Axial force (B) Axial force and bending moment
(C) Shear force and Torsion (D) Torsion only
Ans:- (B)
Explanation: The axial stress (σ) on a column due to an axial load (P) is:
=σ=P/A​
The bending stress (σb) due to the maximum bending moment (M) is:
=σb=M/Z​
Where:
M = Maximum bending moment
Z = Section modulus
For a column that is subjected to both axial load and bending, the combined stress can be superposed as:
σtotal=
​ σ+σb​
σtotal​=P/A + M/Z​
If this combined stress exceeds the critical buckling stress or the allowable stress of the material, the
column can buckle.
554 GATE Architecture and Planning: Comprehensive Question Bank

Q.6 Belt truss is used in High-Rise buildings with a view to provide


(A) Aesthetic appearance (B) Rigid connection between the columns
(C) Escalators (D) Support to large span floors
Ans:- (B)

Q.7 The sampling rate for household survey of a town with 2 lakhs population is
(A) 1 in 10 (B) l in 5 (C) 1 in 8 (D) l in 15
Ans:- No valid answer.
Explanation: The sampling rate for a household survey in a town, or any population for that matter, is
not inherently dependent on the total population. Instead, it depends on the desired level of precision,
the variability of the characteristics under study, budget considerations, and other factors relevant to the
specific survey goals.
However, commonly used sampling rates for population studies can provide a ballpark range. For large
populations:
1 in 10 (or 10%) can be considered a decent sample size for many general surveys.
1 in 5 (or 20%) is a relatively large sample and would give more precise results, but it is also more
expensive and time-consuming.
1 in 8 (or 12.5%) is another reasonable option, falling between the previous two.
1 in 15 (or approximately 6.7%) is on the smaller side for sampling rates, but still might be appropriate
for certain studies.
Without specific information on the goals of the survey, the desired confidence level, and the expected
variability in responses, it’s hard to definitively say which is the correct answer.
Q.8 ldentify the most appropriate set that relates to Physical Infrastructure
(A) Water supply, solid waste
(B) Housing, education, health Management, electricity
(C) Petrol pumps, milk booths, LPG
(D) Communication, postal services, godowns fire protection services
Hemant Vilas Parulekar 555

Ans:- (A)
Explanation: This option covers fundamental physical infrastructure components related to urban and
municipal planning. Water supply is a critical part of city services, and solid waste management is essential
for cleanliness and environmental health. The other options include a mix of social infrastructure (like
education and health) and specific commercial services.
Q.9 Urban design theory that deals with analvsis of relationship between building mass and open space
is
(A) Open space theory (B) Figure ground theory
(C) Linkage theory (D) Place theory
Ans:- (B)
Explanation:
Open Space Theory:
Focuses on the importance and role of open spaces in urban environments.
Emphasizes the need for parks, plazas, green spaces, and other unbuilt areas in enhancing the quality of
urban life.
Figure Ground Theory:
Concerned with the spatial relationship between built (figure) and unbuilt (ground) spaces.
Often visualized using diagrams where built structures are shaded, and open spaces are left unshaded,
highlighting the urban fabric’s pattern.
Linkage Theory:
Centers on the connections and pathways in urban areas.
Addresses how different parts of a city are linked, considering pedestrian paths, roads, and other
transportation networks.
Place Theory:
Concerned with the concept of ‘place’ in urban settings.
Emphasizes the importance of creating meaningful, memorable, and functional spaces where people
can gather, interact, and experience a sense of belonging.
Q.10 Micro-climate refers to
(A) Tropical climate
(B) The tropical climate where microorganisms grow fast
(C) Climate created for limited space
(D) The climate in and around small islands
Ans:- (C)
Explanation: A micro-climate is the distinctive climate of a small-scale area, such as a garden, city,
or even under a tree, and it can differ from the general climate of the surrounding area. Factors like
vegetation, urban structures, bodies of water, and more can create micro-climates by influencing local
temperature, humidity, wind, and other conditions.
556 GATE Architecture and Planning: Comprehensive Question Bank

Q.11 Albedo refers to


(A) Thermal properties of external surface material
(B) Roughness of stone surface
(C) Height of a frame to allow creepers to grow
(D) Nosing of steps made in marble
Ans:- (A)
Explanation: Specifically, albedo is a measure of the reflectivity of a surface. It describes the fraction of
solar energy (shortwave radiation) reflected from the Earth back into space. Different surfaces, materials,
and colours will have different albedos. For instance, snow, being white, has a high albedo and reflects
a large portion of sunlight, whereas asphalt, being dark, has a low albedo and absorbs more sunlight.
Q.12 Origin and destination survey helps in identifying
(A) Desire lines of vehicular traffic flow (B) Location of main traffic nodes
(C) Peak capacity of road (D) Peak capacity of road junctions
Ans:- (A)
Q.13 Water harvesting is
(A) A new method of water purification
(B) Collection of water in paddy fields
(C) Device to allow quick flow of storm water runoff into rivers
(D) Device to re direct storm water runoff into underground aquifers
Ans:- (D)
Explanation: Water harvesting generally refers to the collection and storage of rainwater and runoff for
various uses, including recharging groundwater or aquifers, irrigation, or direct use after treatment. It’s
a sustainable method to manage water resources, especially in areas with scarce water supply.
Q.14 Mill owners’ Association Building in Ahmedabad was designed by
(A) Charles Correa (B) Bernard Kohn (C) Le Corbusier (D) Louis Kahn
Ans:- (C)
Explanation:
Q.15 Bahai temple in Delhi is a
(A) Tensile structure (B) Shell structure (C) Pneumatic structures (D) Space structure
Ans:- (B)
Q.16 The leaning tower of Pisa was prevented from further leaning by
(A) Pumping out muddy slurry from under the foundation of the tower that was moving up
(B) Pumping in cement concrete under the foundation of the tower that was moving down
(C) Propping the tower with steel shoring
(D) Relocating the furniture inside the tower
Ans:- (B)
Explanation: Please refer GATE-2010, Q. 10 for more information
Hemant Vilas Parulekar 557

Q.17 A building is referred to as an “Intelligent Building’ primarily due to its capability to automatically
provide services in terms of
(A) heating, ventilation, air-conditioning and lighting
(B) water supply, laundry, waste water treatment and communication
(C) security, office space facility and office automation
(D) elevator/escalator operations, smartcards for corridor/office door operation
Ans:- (D)
Q.18 The use of Police Power is most appropriately related to
(A)Fair administration of law
(B)Using powers to pass and enforce laws in public interest
(C)Using powers to pass and enforce laws in private interest
(D)Reduce crime in the neighbourhood
Ans:- (B)
Q.19 The value of runoff coefficient C in Q=CIA that represents a completely impervious and wetted
surface from which there is a total runoff is
(A) 0 (B) 1 (C) 10 (D) 1%
Ans:- (B)
Explanation: The runoff coefficient (C) represents the ratio of the amount of rainfall that is converted
into runoff. For a completely impervious and wetted surface, which would result in total runoff, the
coefficient would be: 1
Q.20 The role of government agencies in the field of housing started changing since 1970s in the following
way
(A) From provider to facilitator
(B) From plot supplier to dwelling unit supplier
(C) From designer to builder
(D) From facilitator to provider
Ans:- (A)
Q.21 A Contour line is
(A) An imaginary line connecting points at the same elevation on ground
(B) The line identifying the edge of National Highways
(C) The line identifying the edge of lakes and rivers
(D) An imaginary line connecting hilltops
Ans:- (A)
Q.22 The 74th Constitutional Amendments empowers
(A) The President of India to declare general election without consulting the parliament
(B) The Prime Minister to reschedule Parliament Sessions
(C) Municipal Organizations to undertake planning and development of their respective towns
(D) Individuals to change political party membership after elections
558 GATE Architecture and Planning: Comprehensive Question Bank

Ans:- (C)
Explanation: Please refer GATE-2014, Q.8 for more information.
Q.23 The best example of synthesis between Indo-Aryans and Dravidian stylistic features in a temple is
(A) Hoyasaleshwar (B) Madurai (C) Konark (D) Dilwara
Ans:- (C)
Q.24 Urushringa is the design component used in shikhara of one of the following temple styles
(A) Bhubaneshwar (B) Dravidian (C) Khajuraho (D) Deccan
Ans:- (C)
Explanation: Please refer GATE-2007, Q. 23 for more information
Q.25 Development that meets the need of the present without compromising the ability of the future
generations to meet their own need is
(A) Health, education and welfare development (B) Sustainable development
(C) Economic development (D) Infrastructure development
Ans:- (B)
Explanation: Please refer GATE-2009, Q. 18 for more information.
Q.26 Shell structures derives its strength primarily from its
(A) Shape (B) Wall thickness (C) Strength of the materials (D) Size
Ans:- (A)
Q.27 The primary role of braces in a High-Rise building is to
(A) Resist the gravity load (B) Resist the lateral sway
(C) Improve the elevation (D) Achieve large storey sway
Ans:- (B)
Q.28 Weep holes refer to
(A) Internal duct in human anatomy connecting eyes and throat
(B) Rain water pipes connecting upper terrace to next lower one
(C) Hole made in a log to remove moisture
(D) Holes in retaining wall to allow water to drain out
Ans:- (D)
Q.29 A structural device called pendentives is used in
(A) Closing the end of a vault
(B) Decorating the corners of a room
(C) Making the vault on a long passage
(D) Making a dome on a square Plan
Ans:- (D)
Q.30 The dome of Gol Gumbaj at Bijapur is supported by
(A) Intersecting walls (B) Filling inside corners
(C) Intersecting arches (D) Massive piers
Hemant Vilas Parulekar 559

Ans:- (C)
Q.31 Match the type of buildings in Group I with their type of foundation in Group II
(A) P-1,Q-3,R-4,S-1 Group I Group II
(B) P-4,Q-3,R-2,S-1 P Single storeyed 1 Isolated footing
(C) P-1,Q-1,R-2,S-3 Q Three storeyed 2 Raft foundation
R 5 to 8 storeyed 3 Pile foundation
(D) P-2,Q-3,R-4,S-2
S 10 storeyed and above 4 Well foundation

Ans:- (C)
Q.32 Match the type of buildings in Group I with their type of foundation in Group II
(A) P-1,Q-3,R-2 Group I Group II
(B) P-2,Q-1,S-3 P 1 Christaller’s concept
(C) P-2,Q-3,S-1
(D) P-1,Q-3,S-2
Q 2 Garden city concept

R 3 Multiple nuclei theory

4 Radburn Concept
Ans:- (B)
Q.33 Match the figures Group I with the concepts in Group 2
P-1,Q-2,R-3,S-4 Group I Group II
P-2,Q-3,R-4,S-1 P. Radio And TV studios 1.35-40 db
P-3,Q-4,R-1,S-2 Q. Industry 2.40-45 db
R. Hospital and Auditorium 3.25-30 db
P-4,Q-5,R-3,S-2
S. Court rooms and class rooms 4.70-75 db

Ans:- (C)
Room Type Typical Noise Level (dB (A))
Auditorium 30-40
Concert Hall 25-35
Library 30-40
Conference Room 35-45
Classroom 35-45
Office 40-50
Restaurant 70-80
Gym 60-70
560 GATE Architecture and Planning: Comprehensive Question Bank

Hospital Ward 35-45


Home Living Room 40-50
Home Kitchen 50-60
Home Bedroom 30-40
Cinema Theatre 30-40
Shopping Mall 70-80
Train Station 75-85
Airport Terminal 70-80
Factory Floor 80-90
Bar or Nightclub 80-100
Construction Site 80-100
Quiet Park 30-40
Q.34 Match the period / style in Group I with their most significant contribution in the progress of
architecture in group 2
P-2,Q-1,R-3,S-4 Group I Group II
P-3,Q-4,R-1,S-2 P. Egyption 1.New building material
P-1,Q-2,R-3,S-4 Q. Greek 2.Building skeleton and skin
R. Roman 3.Trabeation
P-4,Q-3,R-2,S-1
S. Gothic 4.Optical correction
Ans:- (B)
Q.35 A sector has gross area of 65 hectares and a residential area of 50 hectares. The net residential density
is 325 pph, what is the gross density of the sector.
(A) 275 pph (B) 225 pph (C) 300 pph (D) 250 pph
Ans:- (D)
Solution: To calculate the gross density, you’ll need to take the total population of the residential area
and divide it by the total gross area.
First, determine the total population in the residential area: Net residential density = 325 pph Residential
area = 50 hectares Total population = 325 pph * 50 hectares = 16,250 people
Now, determine the gross density: Total population = 16,250 people Gross area = 65 hectares Gross
density = 16,250 people / 65 hectares = 250 pph
Q.36 If the height of a building is hand distance of viewing is d, then distance relation ratio is given by
h:d. Match the h:d in Group 1 with their characteristics in Group 2
(A) P-2,Q-3,R-4,S-1 Group 1 Group 2
(B) P-2,Q-4,R-3,S-1 P. 1:1 1. Tend to see object as an edge
(C) P-1,Q-3,R-4,S-2 Q. 1:2 2. Tend to notice details more than the facade
R. 1:3 3. Tend to see object as a whole together with its details
(D) P-4,Q-2,R-3,S-1
S. 1:4 4. Tend to see object in relation to surrounding objects.
Hemant Vilas Parulekar 561

Ans:- (A)
Explanation:
Experience Ratio (h:d)
Minimal Enclosure 1:3
Threshold Enclosure 1:2
Full Enclosure 1:1
Loss of Enclosure 1:4
Q.37 Match AutoCAD system variables In Group 1 with their functions in Group 2
(A) P-4,Q-1,R-5,S-3 Group 1 Group 2
(B) P-2,Q-7,R-6,S-3 P. Mirror text 1.Sets drawing units
(C) P-2,Q-7,R-5,S-1 Q. Measurement 2.Mirrors the text object
R. Hide text 3.Sets units for angles
(D) P-4,Q-7,R-6,S-1
S. Units 4.Controls how mirror command reflects text
5. Controls the visibility of text objects during hide
command
6. Hides the text objects
7. Measures the distance between two points
Ans:- (D)
Q.38 The correlation between population size of cities and area
under residential use is depicted by the curve A,B,C,D.
Choose the correct one.
(A) Curve A
(B) Curve B
(C) Curve C
(D) Curve D
Ans:- (B)
Q.39 To describe the traffic characteristics of a city, show how you would identify the volume of
Group 1 Group2
P. Peak hour traffic 1.By qualifying traffic that stops within the city
Q. Through traffic 2.By determining the maximum hourly traffic volume
R. Terminating traffic 3.By observing the traffic that does not stop inside the city
(A) P-1,Q-2,R-3 (B) P-2,Q-3,R-1 (C) P-3,Q-1,R-2 (D) P-2,Q-1,R-3
Ans:- (B)
562 GATE Architecture and Planning: Comprehensive Question Bank

Q.40 Group 1 Group 2


P. Mughal garden 1. Moss, manicured tress, rocks, water
Q. Italian garden 2. Cycle track, meandering walkways, benches
R. Japanese garden 3. Sculpture of pretty nymphs, angels, fountains
S. City forest 4. Geometrical floral patterns, flowing water , stone shelters
Match the typology of gardens in Group 1 with their typical features in Group 2
(A) P-1,Q-4,R-3,S-2 (B) P-3,Q-2,R-4,S-1 (C) P-4,Q-3,R-1,S-2 (D) P-2,Q-1,R-3,S-4
Ans:- (C)
Q.41 Choose two abbreviations from the following that very commonly used in transportation planning
P.MOU Q.DPC R.ROW S.ITI T.DJB U.PCU V.RAW W.EIA
(A) MOU, ROW (B) ITI, DJB (C) ROW, PCU (D) ROW, EIA
Ans:- (C)
Explanation: In the context of transportation planning, the two most commonly used abbreviations
from the given options are:
ROW - Right Of Way
U.PCU - Passenger Car Unit
Q.42 Find the correct matches in the given list of architectural entities and the architectural style / period
(A) R-3,R-4 Group 1 Group 2
(B) Q-1,Q-3 P. Temple 1. Mohenjo-Dora
(C) S-3,S-4 Q. public Bath 2. Greek
R. Agora 3. Roman
(D) P-2,P-3
S. Granary 4. Mayan
Ans:- (B)
Q.43 Stress due to the bending moment may be computed through appropriate combination from the
following parameters. Select the appropriate choice of the ones given below
M. Bending Moment Y. Distance from neutral axis R. Radius of curvature
I. Moment of Inertia E. Young’s modulus of elasticity O. Tress due to bending
(A) σR=yE (B) MI=Eσ (C) σl=MR (D) σy=ME
Ans:- (C)
Explanation: The stress due to bending moment in a beam is given by the formula:
σ = (M×y)/I
Where:
σ = Stress due to bending (often simply called bending stress)
M = Bending moment
y = Distance from the neutral axis
I = Moment of Inertia of the section
From the given options, the correct expression that matches this formula is:
σl=MR
Hemant Vilas Parulekar 563

Q.44 Match the shape of the age-sex pyramid and their interpretations.

City offers high employment opportunity


City’s population in aging
City has poor employment opportunity
City has young population
(A) P-4,Q-1,R-2,S-3 (B) P-1,Q-2,R-3,S-4 (C)P-2,Q-3,R-4,S-1 (D) P-3,Q-4,R-1,S-2
Ans:- (B)
Type of Expansive Old and declining Stationary
Pyramid
Description Expansive population A top-heavy population Stationary or near-
pyramids show larger pyramid with higher stationary population
numbers or percentages proportions in older pyramids display
of the population in the age groups indicates a somewhat equal numbers
younger age groups. declining population. or percentages for almost
These types of pyramids all age groups.
are usually found in
populations with very
large fertility rates and
lower than average life
expectancy. The age-sex
distributions of many
Third World countries
would probably display
expansive population
pyramids.
Pyramid

Q.45 A plot of land measuring 15mX40m has a ground+4 storied building with uniform floor areas. If
the land has been utilized to its fullest and FAR is 250, Calculation the permissible ground coverage.
(A) 50% (B) 5% (C) 25% (D) 40%
564 GATE Architecture and Planning: Comprehensive Question Bank

Ans:-(A)
Solution: Given:
Plot size = 15m x 40m = 600 m2
Building is ground + 4 stories, which means a total of 5 stories.
FAR (Floor Area Ratio) = 250%
The total permissible built-up area = FAR x Plot area
= 2.5 (since 250% = 2.5 in decimal) x 600 m2 = 1500 m2
Now, since the building is ground + 4 stories (i.e., 5 stories) with uniform floor areas, the area of each
floor = Total permissible built-up area / 5
= 1500 m2 / 5 = 300 m2
The ground coverage by the building = Area of each floor / Plot area x 100%
= 300 m2 / 600 m2 x 100%
= 50%
Q.46 Choose the two most appropriate design approaches that signify Laurie Baker’s work.
P. Recycling of materials
Q. Imagination use of brick in construction
R. Innovative construction methods to save cost
S. New vocabulary in indigenous construction.
(A) P,R (B) Q,S (C) R,S (D) P,S
Ans:- (B)
Q.47 A two color/ tone scheme is proposed for an office building, which is the most appropriate set of
combination between desired effect and the color scheme?
(A) P-2,P-3
(B) Q-1,Q-3 Group 1 Group 2
(C) R-1 P. Contrasting 1. Complementry colors
(D) P-1,P-2
Q. Formal 2. Extreme gray values
R. Neural 3. Light colors
Ans:- (A)
Q.48 The efficiency of a modern housing scheme can be best assessed by
P. Efficient planning of a dwelling unit
Q. Better carpet area ratio
R. Imported finishes and fixtures
S. Good general common facilities
T. Good landscaping
(A) R,S,T (B) Q,S,T (C) P,Q,S (D) P,R,T
Ans:- (C)
Hemant Vilas Parulekar 565

Q.49 Given below are the statements related to AutoCAD


(A) P-1,Q-2,R-1,S-1 Group 1 Group 2
(B) P-2,Q-2,R-1,S-2 P. Mspace switches from model space to 1. True
(C) P-1,Q-1,R-2,S-1 paper space
Q. Undo restores last object erased 2. False
(D) P-2,Q-1,R-2,S-2
R. Multiple repeats the command you entre
until you press ESC
S. Layerp, restores a deleted layer
Ans:- (D)
Explanation:
Mspace switches from model space to paper space This statement is not entirely correct. The MSPACE
command activates the Model Space environment within a layout (Paper Space). But saying it “switches
from model space to paper space” can be misleading. The proper command for switching from Paper
Space to Model Space is PSPACE.
Q. Undo restores last object erased Yes, the UNDO command in AutoCAD will undo the last action
performed, including erasing an object.
R. Multiple repeats the command you entre until you press ESC This is not a standard command in
AutoCAD. There’s no MULTIPLE command that does this.
S. Layerp, restores a deleted layer There is no command LAYERP in AutoCAD to restore a deleted layer.
To restore a deleted layer, you might need to use the UNDO command or have other backup strategies.
Q.50 A tract of land surveying having a concave slope implies that
(A) Contour lines are spread at decreasing distances in the downhill direction.
(B) Contour lines are equally spaced.
(C) Contour lines are spread at increasing distances in the downhill direction.
(D) There is no trend in spacing of contour lines.
Ans:- (A)
Q.51 Given below is a sketch plan of a hilly site and two points to be connected by road. Select the best
road alignments. Contour interval is 2m.
(A) Contour lines are spread at decreasing distances in
the downhill direction.
(B) Contour lines are equally spaced.
(C) Contour lines are spread at increasing distances in
the downhill direction.
(D) There is no trend in spacing of contour lines.
Ans:- (D)
Q.52 Select the appropriate sequence of the major spaces in Hindu temple.
(A) Antaral – mandapam - Ardhmandapam – Garbhagriha
(B) Mandapam – Antaral - Ardhmandapam – Garbhagriha
(C) Garbhagriha – Antaral – Ardhmandapam – Mandapam
(D) Ardhmandapam – Mandapam – Antaral – Garbhagriha
566 GATE Architecture and Planning: Comprehensive Question Bank

Ans:- (D)

Q.53 Identify the devices from Group 1 that prevent sliding of truss roof components in group 2
(A) P-3,Q-4,R-2,S-1 Group 1 Group 2
(B) P-4,Q-2,R-1,S-3 P. Screw 1. Battens
(C) P-1,Q-4,R-3,S-2 Q. Bolt 2. Purlins
R. Wall Plate 3. Rafters
(D) P-2,R-4,Q-3,S-1
S. Cleat 4. Struts
Ans:- (C)
Device Screw Bolt
Used with Battens Struts
Illustration

Device Wall Plate Cleat


Used with Rafters Purlins
Illustration
Hemant Vilas Parulekar 567

Q.54 Choose the most appropriate set of facilities to be provided at the sector level for about 20,000
inhabitants in a town of 3,25,000 inhabitants.
P. High school R. Play field T. Civic center Q. College S. Shopping Center
(A) P,Q,R (B) Q,R,T (C) P,R,S (D) Q,S,T
Ans:- (C)
Explanation: Facilities required for 1 planning unit as shown (As per URDPFI guidelines)
Planning unit Poplation
Sr. Facilities No. Area per Total Area
No. unit (Ha) (Ha)
Neighborhood 5000-15000 1 Senior Secondary 1-2 1.80 3.60
school (vi-xii)
2 Dispensary 1 0.08-0.12 0.08-0.12
3 Community Hall 1 2000 sq.m. 2000 sq.m.
4 Neighborhood Park 1 1.00 1.00
5 Neighborhood Pay Park 1 1.50 1.50
6 Local Shopping Centre 1 4600 sq.m. 4600 sq.m.
Q.55 In different climatic zones, different sets of devices are used to achieve comfort in a dwelling. Choose
the most appropriate set.
(A) Q-1,Q-2,Q-6 Group 1 Group 2
(B) P-2,P-3,P-6 P. Warm and Humid 1. Thick walls
(C) R-1,R-4,R-6 Q. Hot and dry 2. Central courtyard
R. Composite 3. Thin walls
(D) P-1,Q-3,R-2
4. Small openings
Good ventilation
Evaporative cooling
Ans:- (A)
Q.56 Which of the following characterize Fatehpur Sikri?
P. Urban design in Mughal era Q. City without roads
R. Fine marble carvings S. Synthesis of Hindu and Islamic architectural styles
R,S,T (B) P,Q,S (C) Q,R,S (D) Q,S,T
Ans:- (C)
Q.57 Match the type of traps in Groups 1 with their configuration in Group 2
(A) P-3,Q-1,R-2 Group 1 Group2
(B) P-2,Q-1,R-4 P. P-trap 1. Outlet is at 60 deg. to the inlet
(C) P-1,Q-3,R-4 Q. Q-trap 2. Outlet is parallel to the inlet
R. S-trap 3. Outlet is at 45 deg. to the inlet
(D) P-4,Q-3,R-2
4. Outlet is at 90 deg. to the inlet
Ans:- (D)
568 GATE Architecture and Planning: Comprehensive Question Bank

Q.58 Select the most appropriate combination of elements that can be labeled as “Street Hardware”
P. Advertisement kiosk Q. Junction Boxes R. Signals S. Planter beds
T. Fire Hydrant U. Letter box V. Bus shelter W. Traffic signs
(A) Q,R,T,W (B) P,Q,S,V,W (C) Q,R,S,T,U,Q (D) Q,R,S,V,W
Ans:- (D)
Explanation: (Please refer GATE-2011, Q.43 for more information also)
P. Advertisement kiosk - This is more related to advertising and while commonly seen on streets, it’s not
strictly functional in terms of street operations.
Q. Junction Boxes - These are crucial for electrical and utility purposes on streets.
R. Signals - Essential for street safety and traffic regulation.
S. Planter beds - Beautification and environmental purposes.
T. Fire Hydrant - Essential for emergency situations.
U. Letter box - Used for postal purposes.
V. Bus shelter - Essential for public transport users.
W. Traffic signs - Essential for road safety and direction.
Considering the above explanations, the best combination for “Street Hardware” would be:
Q. Junction Boxes
R. Signals
S. Planter beds (for beautification purposes)
T. Fire Hydrant
V. Bus shelter
W. Traffic signs
Q.59 Match the properties of soil in Group 1 with their characteristics in Group 2
(A) P-3,Q-6,R-1,S-2 Group 1 Group 2
(B) P-5,Q-1,R-4,S-2 P. Permeability 1. Load which a soil is able to support
(C) P-5,Q-4,R-1,S-6 Q. Shear strength 2. Susceptibility to erosion
R. Bearing capacity 3. The ease with which water can flow through soil
(D) P-3,Q-4,R-1,S-5
S. Ph level 4. Determines the stability of a soil and its ability to
resist failure under loading
5. Shows the ability of a soil to support plant
growth.
6. Angle at which an unconsolidated soil will
naturally slope.
Ans:- (D)
Hemant Vilas Parulekar 569

Q.60 Match the AutoCAD commands in Group 1 with their functions in Group 2
(A) P-5,Q-4,R-6,S-1 Group 1 Group 2
(B) P-3,Q-4,R-5,S-1 P. Xline 1. creates solid
Q. Bhatch 2. Moves object about a 3-dimensiaonal axis
(C) P-5,Q-7,R-8,S-2
R. Cutclip 3. Creates an infinite line
(D) P-3,Q-1,R-2,S-4 S. Revole 4. Associative hatch pattern
5. Erases the last drawn line
6. Copies objects to the clipboard
7. Non-associative hatch pattern
8. Copies object to the clipboard and removes the object from
the drawing

Q.61 Match the type of land use in Group 1 with their most appropriate color code in Group 2
(A) P-3,Q-6,R-4,S-5 Group 1 Group 2
(B) P-2,Q-1,R-4,S-5 P. Residential 1. Purple
Q. Industrial 2. Yellow
(C) P-2,Q-1,R-6,S-3
R. Recreational 3. Blue
(D) P-6,Q-1,R-2,S-4 S. Transportation 4. Green
5. Gray
6. Red
Ans:- (B)
Q.62 Find out which of the following are True (T)or False(F)and then choose the correct combination.
Group 1 Group 2
P. Housing loan amount is determined by repayment capacity 1. True
Q. The period of repayment of housing loans is never more than 7 years 2. False
R. In the villages, loans for house building is still taken from private sources
S. Housing loan packages are available from LIC, HUDCO,HDFC,CPWD and SBI
(A) P-1,Q-2,R-1,S-2 (B) P-2,Q-1,R-2,S-2 (C) P-1,Q-2,R-2,S-1 (D) P-2,Q-1,R-1,S-2
Ans:- (A)
Q.63 1750 sq. mt. of a water body having as total surface area of 14000 sq. mt. is covered with water
hyacinth. Assuming the weed grows at GP rate every 24 hours, in how many days the water body
will be totally covered?
(A) 16 days (B) 8days (C) 3 days (D) 1 days
Ans:- (C)
Solution: Given that the water hyacinth grows at a geometric progression (GP) rate, and that 1750 sq.
mt. of the water body is covered initially, and it doubles every 24 hours, the situation can be modeled as:
Day 0: 17501750 sq. mt.
Day 1: 1750×2=35001750×2=3500 sq. mt.
Day 2: 3500×2=70003500×2=7000 sq. mt.
Day 3: 7000×2=140007000×2=14000 sq. mt.
Thus, in 3 days the water body will be totally covered.
570 GATE Architecture and Planning: Comprehensive Question Bank

Q.64 Match the quotations in Group 1 with their authors in Group 2


(A) P-1,Q-2,R-3,S-4 Group 1 Group 2
(B) P-3,Q-1,R-3,S-2 P. From follows function 1. Albert Einstein
(C) P-2,Q-3,R-1,S-4 Q. Imagination is more important than 2. Winston Churchill
knowledge
(D) P-4,Q-2,R-3,S-1
R. From follows climate 3. Louis Sullivan
S. We shape the buildings thereafter they 4. Charles Correa
shape us

Ans:- (B)
Q.65 Referring to the figure besides, select the valid combination of symbols the parameters given below

(A) P-1,S-1,V-1 Group 1 Group 2


(B) R-4,U-3,T-2 P. Shearing force at A 1. Maximum value
(C) Q-1,T-3,V-4 Q. Shearing force at B 2. Positive Value
R. Shearing force at C 3. Negative value
(D) S-3,P-4,T-2
S. Bending moments at A 4. Zero
T. Bending moments at B
U. Bending moments at C
V. Torsion at A
Ans:- (C)
Q.66 Identify the most appropriate sequence of processes used for treatment of waste water.
(A) Skimming tank-Racks and screen-Precipitation tank-Sedimentation tank-Biological growth-
Disinfection
(B) Biological growth-Racks and screen-Skimming tank-Sedimentation tank-Precipitation tank-
Disinfection
(C) Racks and screen- Skimming tank- Sedimentation tank- Precipitation tank- Biological growth-
Disinfection
(D) Biological growth- Skimming tank- Precipitation tank- Racks and screen- Sedimentation tank-
Disinfection
Ans:- (B)
Q.67 A plot of land under development is location in a rocky area without any public sewerage system,
Identify the most appropriate combination for sewage disposal for the proposed development.
P. Septic tank Q. Soak pit R. Anaerobic filter S. Oxidation ditch
(A) P,Q (B) S,R (C) P,R (D) S
Ans:- (A)
Hemant Vilas Parulekar 571

Q.68 Match the elements of Image ability with their most appropriate characteristics
Group 1 Group 2
P. Node 1. Strategic foci into which observer can enter
Q. District 2. Act as lateral reference and often path as well
R. Landmark 3. Area of homogenous character recognized by clues
S. Edge 4. Singularity, contrast with its context. Observer dose not enter in
(A) P-3,Q-1,R-2,S-4 (B) P-1,Q-3,R-4,S-2 (C) P-4,Q-3,R-1,S-2 (D) P-4,Q-3,R-2,S-1
Ans:- (D)
Explanation: Please refer GATE-2009, Q. 26 for more information.
Q.69 The bio-climatic chart defines comfort zone in terms of
DBT Q. Effective Temperature (ET) R. Relative humidity S. Heat stress Index
(A) P,Q,R (B) P,R (C) Q,S (D) P,R,S
Ans:- (A)
Q.70 Match the items from Group 1 with the appropriate items in Groups 2
Group 1 Group 2
P. Town Planning 1. Additional FAR granted to the developer if he provides public amenities
scheme / spaces
Q. Transferable 2. A legal procedure that allows pooling of land by owners, preparation of
development right layout, and redistribution of final plots.
R. Incentive zoning 3. Allows the plot owners to transfer the FAR/FSI on the area of the plot,
surrendered to the local authority, to another plot.
(A) P-3,Q-1,R-2 (B) P-2,Q-1,R-3 (C) P-2,Q-3,R-1 (D) P-3,Q-2,R-1
Ans:- (C)
Q.71 Match the housing programmes in Group 1 with their key features in Group 2
Group 1 Group 2
P. KIP of Indonesia 1. Participatory, stress on young beneficiary’s education
Q. MHP of Sri Lanka 2. Participatory, beneficiaries contributed land
R. Orangi project of Pakistan 3. Employment generative, tenureship to beneficiaries in situ
S. CDP of Hyderabad, India 4. Flexible assistance, tenureship to beneficiaries in situ
(A) P-2,Q-4,R-1,S-3 (B) P-1,Q-2,R-3,S-4 (C) P-3,Q-1,R-2,S-4 (D) P-4,Q-3,R-4,S-3
Ans:- (D)
Q.72 Match the names of the trees in Group 1 with the shape of their leaves in Group 2
P-3,Q-2,R-4,S-1 Group 1 Group 2
P-4,Q-3,R-1,S-1 P-Ficus religiosa
P-3,Q-1,R-4,S-2
(D) P-2,Q-1,R-3,S-4
572 GATE Architecture and Planning: Comprehensive Question Bank

Q. Casia Fistula

R. Delonix Regia

S. Polyalthia Longifolia

Ans:- (B)
Q.73 Match the orders of Architecture in Group 1 with their most appropriate symbolic expression in
Group 2
P-3,Q-4,R-1,S-2 Group 1 Group2
P-2,Q-3,R-4,S-1 P. Doric 1. Floral
P-4,Q-1,R-2,S-3 Q. Ionic 2. Most decorative
R. Corinthian 3. Masculine
(D) P-1,Q-4,R-3,S-2
S. Composite 4. Feminine
Ans:- (A)
Q.74 Referring to the structure shown in figure besides, the maximum shearing force in the beam is as
point

(A) P (B) Q (C) R (D) S


Ans:- (C)
Solution: Taking moment about point ‘P’
18 x 14 - R x 12 + 20 x 6 = 0
Thus, R = 31KN,. Thus P = (20-18)-31 = 7KN
S.F. at R = +18KN
Q.75 Choose the correct set of factors of physical planning to be placed at
the vertex of Geddesian Triangle.
P. Development Q. Area R. Function
S. Node T. Linkage
(A) P,Q,R (B) Q,R,S (C) R,S,T (D) P,Q,T
Hemant Vilas Parulekar 573

Ans:- (A)
Explanation: The Geddesian Triangle, conceptualized by Sir Patrick Geddes, typically represents three
key factors of planning: Place, Work, and Folk.
Translating this into factors of physical planning:
“Place” can relate to the geographical or physical attributes (like “Area”).
“Work” can relate to the function or activities of the area (like “Function”).
“Folk” would typically relate to the social and cultural aspects, which aren’t explicitly mentioned in the
options provided.
Based on the given options and this interpretation:
Q. Area corresponds to “Place” R. Function corresponds to “Work”
The third vertex, corresponding to “Folk”, is not directly mentioned in the options. However, considering
the closest approximations:
(A) P,Q,R would represent Development, Area, and Function.
Q.76 Match the traffic problems in Group 1 with their solutions in Group 2
Group 1 Group2
P. Goods traffic service city causing accidents 1. Built elevated road linking entry & exit to city
during day time
Q. Traffic jam along a major road alignment 2. Install synchronized signaling during peak
during peak hours hour
R. Through traffic constituting major share of city 3. Provide separate lane for slow mode
road space
S. Slow & fast modes creating much traffic 4. Restrict entry of HTV to late night hours only
confusion
P-1,Q-4,R-3,S-2 (B) P-3,Q-2,R-4,S-1 (C) P-4,Q-3,R-1,S-2 (D) P-4,Q-2,R-1,S-3
Ans:- (D)
Q.77 Arrange the four types of standard urban roads in descending order from the following
P. National Roads U. Sector Roads
Q. District Distributions V. Local Distributions
R. Slip Roads W. Bus Roads
S. Primary Distributions X. Access Roads
T. Neighbourhood Roads Y. Dead End roads
(A) S,Q,V,X (B) P,V,W,Y (C) P,R,T,V (D) S,Q,T,X
Ans:- (A)
Q.78 Match the beneficiaries of housing in Group 1 with type of housing in Group 2
Group 1 Group 2
P. Post disaster homeless families 1. Self-financing apartments
Q. Slum dwellers under eviction notice 2. Prefabricated, portable housing
R. Middle income govt. employees 3. HIG cooperative apartments
S. NRI wishing to return 4. Site & Service Scheme
(A) P-1,Q-3,R-2,S-4 (B) P-2,Q-1,R-4,S-3 (C) P-3,Q-4,R-1,S-2 (D) P-2,Q-4,R-1,S-3
574 GATE Architecture and Planning: Comprehensive Question Bank

Ans:- (D)
Q.79 Match the type of floats in Group 1 with their characteristics in Groups 2
Group 1 Group 2
P. Total float 1. Excess of minimum available time over the activity time
Q. Free Float 2. Excess of minimum available time over the activity time
R. Independent Float 3. Excess of available time over the activity time when all jobs start as
early as possible
(A) P-1,Q-3,R-2 (B) P-2,Q-1,R-3 (C) P-3,Q-2,R-1 (D) P-2,Q-3,R-1
Ans:- (C)
Q.80 In key indicator survey, the proxy variables for municipal efficiency are
(P) New construction (Q) Frequency of garbage removal
(R) Electricity consumption (S) Amount spent on salary / wages of municipal workers
(A) P,Q (B) P,R (C) Q,S (D) Q,R,S
Ans:- (C)
GATE QUESTION PAPER 2004
General Aptitude (GA)

Q.1-Q.30 carry one mark each.


Q.1 The designer of Horizon City in Texas is
(A) Lucio costa (B) Le Corbusier (C) Frank Lloyf Wright (D) Soria Mata
Ans:- No suitable option.
Explanation:- The designer of Horizon City in Texas is not listed among the options you’ve provided.
Horizon City was developed by the Horizon Corporation, which was a land development company. The
company was not associated with any of the renowned architects mentioned in the options.
Q.2 A dark brown organic compound formed in the soil due to partial or complete decomposition of
vegetable matter is known as
(A) Loam (B) Texture (C) Humus (D) Mulch
Ans:- (C)
Explanation:-
Term Description
Loam Loam is a type of soil that consists of a balanced mixture of sand, silt, and clay
particles. It is known for its high fertility and good drainage properties, making
it ideal for agricultural purposes.
Texture Texture refers to the physical properties of soil, specifically the relative
proportions of sand, silt, and clay particles. Soil texture influences the soil's
ability to retain water, nutrients, and air, and it affects the ease with which roots
can penetrate the soil.
Humus Humus is the dark brown organic compound that forms in the soil as a result of
the partial or complete decomposition of plant and animal matter. It contributes
to soil fertility by providing essential nutrients, improving soil structure, and
increasing water retention.
Mulch Mulch is a layer of organic or inorganic material placed on the surface of the soil
to conserve moisture, regulate soil temperature, suppress weeds, and improve
soil fertility. Common organic mulches include wood chips, straw, leaves, and
compost, while inorganic mulches can include gravel or plastic sheets.
Q.3 The format for transferring the AutoCAD drawing file to an ASCII file is known as
(A) DXB (B) DXF (C) JPEG (D) DGN
Ans:- (B)
Explanation:- The format for transferring an AutoCAD drawing file to an ASCII file is known as DXF
(Drawing Exchange Format). DXF is a file format developed by Autodesk, which allows CAD drawings
to be shared between different CAD programs. It converts the drawing into a text-based format that can
be read and edited by various software applications.
576 GATE Architecture and Planning: Comprehensive Question Bank

Q.4 Technopolis means


(A) Ten million size city (B) Dynapolis
(C) Settlement designed by high technology (D) Urban conurbation
Ans:- (C)
Explanation:- Technopolis refers to a settlement or an urban area that is designed, developed, and
supported by advanced technology and innovation. These areas often focus on fostering collaboration
between technology-based industries, research institutions, and academic organizations to drive
economic growth and development.
Q.5 Earthwork for the mounds in the landform work in a landscape is estimated
(A) Average-end-area method (B) Borrow pit method
(C) Dead-man-method (D) Prismoidal method
Ans:- (A)
Explanation:- The formula for the average-end-area method is:
Volume = (A1 + A2) / 2 * L
Where:
• Volume represents the volume of earthwork between two consecutive cross-sections.
• A1 is the cross-sectional area at the beginning of the interval.
• A2 is the cross-sectional area at the end of the interval.
• L is the length of the interval.
By applying this method for all intervals and summing up the volumes, you can estimate the total volume
of earthwork for the project.
Q.6 The word Bionomics refers to
(A) Management of land (B) Management of project
(C) Management of fund (D) Management of life
Ans:- (D)
Explanation:- Bionomics, also known as environmental biology or ecology, is the study of the
relationships between organisms and their environment, including both the living (biotic) and non-
living (abiotic) components. It focuses on the interactions between species, their habitats, and the larger
ecosystems in which they live. This branch of biology is crucial for understanding the delicate balance
of life on Earth and helps inform strategies for conservation and sustainable management of natural
resources.
Q.7 For proper ventilation, the ratio of opening area to the total floor area of the m should be
(A) 1:6 (B) 1:10 (C) 1:12 (D) 1:20
Ans:- (B)
Explanation:- For proper ventilation, the recommended ratio of opening area (windows, vents, etc.) to
the total floor area of the room is generally 1:10. This ratio ensures that there is enough air exchange
to maintain a comfortable and healthy indoor environment. However, this ratio can vary depending on
the specific building codes and guidelines, as well as the room’s purpose and occupancy. It’s essential to
refer to local building codes or consult an architect or engineer when designing a space to ensure proper
ventilation. (Source-ECBC Code, 2007)
Hemant Vilas Parulekar 577

Q.8 Emerald necklace is well known as


(A) Ornamented garden in Italy
(B) Large public park in New York
(C) Parkway in Boston
(D) Street lighting in Marine Drive in Mumbai
Ans:- (C)
Explanation:- The Emerald Necklace is a well-known park system in Boston, Massachusetts. It is a chain
of parks connected by parkways and waterways, designed by the famous landscape architect Frederick
Law Olmsted in the late 19th century. The Emerald Necklace consists of several parks and green spaces,
including the Boston Common, the Public Garden, the Fens, the Riverway, Olmsted Park, Jamaica Pond,
and Franklin Park. This park system provides a green connection through various neighborhoods in
Boston and is an important element of the city’s urban planning and landscape architecture.
Q.9 Indicate the minimum horizontal and vertical distance for any building from a 33kV electric line
(A) 3.7m (Vertical) & 2.0m (Horizontal) (B) 1.5m (Vertical) & 4.6m (Horizontal)
(C) 2.5m (Vertical) & 1.2m (Horizontal) (D) 4.5m (Vertical) & 6.9m (Horizontal)
Ans:- (A)
Explanation:-
Vertical Horizontal
a) low and medium voltage lines and service lines 2.50m 1.20m
b) high voltage lines upto and including 11 kV 3.70m 1.20m
c) high voltage lines above 11 kVand upto and 3.70m 2.00m
including 33kV
d) Extra high voltage lines additional 33kV Plus 0.3m for every Plus 0.3m for every
additional 33kV or additional 33kV or part
part thereof thereof
Q.10 Effective temperature in buildings integrates the effect of
(A) temperature,humidity and air movement
(B) radiation,temperature,humidity and air movement
(C) temperature,airmovement,conduction,vegetation
(D) temperature,humidity,density,vegetation
Ans:- (A)
Q.11 The ‘Five points for a New Architecture’ was defined by
(A) Louis Sullivan (B) H H Richardson (C) Gustav Eiffel (D) Le Corbusier
Ans:- (D)
Explanation:- Please refer GATE-2006, Q. 69 for more information.
Q.12 Gate Valves are used in water supply lines to
(A) Isolate parts of the network (B) Drain out excess water
(C) Release air trapped in pipe lines (D) Limit the flow of water
578 GATE Architecture and Planning: Comprehensive Question Bank

Ans:- (A)
Explanation:- Gate valves are used in water supply lines primarily to isolate parts of the network. They
are designed to either fully open or fully close the flow of water, making them suitable for situations
where flow control is necessary, such as during maintenance or repair works. They are not typically used
for flow regulation or throttling, as this could cause damage to the valve or lead to leakage.
Q.13 Earnest money is required to be deposited by a tenderer
(A) before acquiring a tender document (B) before the bid is open
(C) after the contract is awarded (D) before getting the payment against the last bill
Ans:- (B)
Explanation:- Earnest money, also known as a bid bond or bid security, is a deposit made by a bidder
to show their commitment to a project. It serves as a form of financial assurance that the bidder will
not withdraw their bid before the specified bid validity period and will enter into a contract if they are
awarded the project.
The primary purpose of earnest money is to protect the client from bidders who might submit proposals
without a genuine intention to complete the project. By requiring bidders to deposit earnest money,
clients can ensure that only serious bidders participate in the tender process. This deposit helps to
deter non-serious bidders and reduces the risk of project delays and additional costs associated with
re-tendering.
When a bid is submitted, the earnest money is typically held by the client or a third party until the
bidding process is complete. If a bidder is awarded the project and enters into a contract, their earnest
money is generally returned to them. However, if a bidder withdraws their bid before the bid validity
period expires or fails to enter into a contract when awarded the project, they may forfeit their earnest
money.
Q.14 Designer of Roofless Church is
(A) Kenzo Tange (B) Philip Johnson (C) Bruce Goff (D) Ero Saarinen
Ans:- (B)
Explanation:- The Roofless Church is an architectural masterpiece that embodies the idea of spirituality
and unity with nature. Philip Johnson designed this unique space with the intention of creating a serene
environment for people to gather, reflect, and connect with their spiritual beliefs.
The church is located in New Harmony, Indiana, a town known for its history of utopian communities
and a strong emphasis on education, culture, and spiritual development. The Roofless Church stands as
a symbol of New Harmony’s dedication to fostering a sense of community, peace, and spiritual growth.
The structure itself is comprised of a circular brick wall surrounding a central plaza, with a large, sculptural
roof made of cedar shingles suspended over the space. The roof is supported by four steel columns,
creating a canopy that protects visitors from the elements while still allowing them to experience the
surrounding landscape.
Within the enclosed spacse, there are several art installations and sculptures, including a bronze statue
of the “Descent of the Holy Spirit” by Jacques Lipchitz. The Roofless Church is not affiliated with any
particular religious denomination and is open to visitors of all faiths, encouraging people to come
together in a spirit of unity and contemplation.
Philip Johnson’s design of the Roofless Church demonstrates his talent for creating innovative, thought-
provoking spaces that challenge conventional architectural norms while fostering a sense of community
and spiritual connection.
Hemant Vilas Parulekar 579

Q.15 Homelessness refers to


(A) Number of people per 1000 urban area population sleeping outside dwelling units
(B) Total number of people living on footpath
(C) Percentage of people living on unauthorized land
(D) Share of urban population living in slums
Ans:- (B)
Q.16 Dummy activities are activities having
(A) zero time (B) time difference between additional event and preceding event
(D) earliest event time (C) time difference between additional event and succeeding event
Ans:- (A)
Explanation:- Dummy activities in project management, specifically in the Program Evaluation and
Review Technique (PERT) or Critical Path Method (CPM), are used to show dependencies between
tasks without consuming resources or time. They have zero duration and do not affect the overall project
schedule but help maintain the logical sequence and relationships between tasks in the network diagram.
Q.17 The roof over the millennium dome in England (2000 AD) is made of
(A) P.V.C. (B) Ferro-cement (C) Teflon (D) Titanium

Ans:- (C)
Explanation:- The roof over the Millennium Dome, now
known as the O2 Arena, in England is made of PTFE
(Polytetrafluoroethylene), also known as Teflon. The PTFE-
coated glass fiber fabric is lightweight, durable, and provides
excellent resistance to weathering and UV radiation.
Q.18 Grease trap is provided in house drain connected to
(A) kitchen (B) bathroom (C) water closet (D) urinal
Ans:- (A)

Q.19 Hydra house is an example of


(A) space structure (B) modern skyscraper
(C) mobile architecture (D) desert architecture
Ans:- (C)
Explanation:- Hydra House, designed by architect Piet Blom, is a unique example of mobile architecture.
It features a modular, prefabricated design that can be easily disassembled, transported, and reassembled
in a new location. The structure’s flexibility and adaptability make it a notable example of mobile
architecture, which aims to create versatile and easily movable structures to meet the changing needs of
its occupants or adapt to different environmental conditions.
580 GATE Architecture and Planning: Comprehensive Question Bank

Q.20 Housing in Compliance refers to


(A) Housing affordability (B) Mortgage rate
(C) Zoning regulation (D) Authorised Housing
Ans:- (D)
Q.21 The Secession Building in Vienna was designed by
(A) August Perret (B) Carlo Lodilo (C) Joseph Maria Olbrich (D) Josef Hoffman
Ans:- (C)
Explanation:- The building is a significant example of the Art Nouveau movement and was completed
in 1898. It served as an exhibition hall for the Vienna Secession, a group of artists who sought to break
away from the traditional art styles of the time.
Q.22 The curved arris formed by the intersection of vaults is
(A) Groin (B) Glyph (C) Impost (D) Frieze
Ans:- (A)
Explanation:- A groin is the curved arris or edge formed by the intersection of two vaults. This term is
commonly used in architecture to describe the junction point of vaulted ceilings, especially in Gothic
and Romanesque structures.
Q.23 For a truss under vertical loading the top chord members of the truss are subjected to
(A) torsion (B) axial tension (C) tension and compression (D) axial compression
Ans:- D
Q.24 The Ian Potter Centre in Australia designed by
(A) Vikas Gore (B) Group Seven Architecture
(C) Lab Architecture Studio (D) B.V. Doshi
Ans:- (C)
Explanation:- The Ian Potter Centre, also known as the Ian Potter Centre: NGV Australia, is a part of
the National Gallery of Victoria in Melbourne, Australia. It is dedicated to showcasing Australian art
and was designed by the Lab Architecture Studio, in collaboration with Bates Smart.
Q.25 Solid masonry is most suitable for a building located in an earthquake zone;
(A) Abutmen (B) Alae (C) Abacus (D) Aisle
Ans:- (A)
Q.26 Which plan form is most suitable for a building located in an earthquake zone
(A) Rectangular (B) U-Shape (C) L-Shape (D) Square
Ans:- (D)
Q.27 If intensity of incident light increases on a totally black object
(A) The form is flattened (B) The object produces a shining effect
(C) The object appears blacker (D) The object appears whitish
Ans:- (D)
Q.28 Square plan of Garbh Griha in Hindu temple is very much linked with
(A) Climate (B) Orientation
(C) Corbelling (D) Facade treatment
Ans:- (B)
Hemant Vilas Parulekar 581

Q.29 An external enclosure for any space is most pleasing when its walls are
(A) 1/2 to 1/3rd as high as the width of space
(B) Height is greater than the width
(C) Height is four times the width
(D) Height is equal to the width
Ans:- (A)

Q.30 When the picture plane, located between the object and the observer, is moved towards the observer
the size of the perspective view
(A) increases in size (B) remains same (C) is distorted (D) decreases in size

Q.31 In the treatise of Vastushastra the cardinal boundaries are known as


(A) (B) (C) (D)
Eastern Boundary Aditya Varuna Aditya Chandra
Western Boundary Yama Aditya Varuna Aditya
Southern Boundary Chandra Yama Yama Varuna
Northern Boundary Varuna Chandra Chandra Yama
Ans:- (C)
Q.32 The purpose of the Building code is to protect the interests of the citizen. Choose the right
combination from the following factors.
P. General Welfare Q. Aesthetic Quality R. Protection of Health
S. Safety T. Convenience U. Affordability
(A) P,R,S,U (B) P,R,S,T (C) P,S,T,U (D) Q,R,S,T
Ans:- (D)
Q.33 The items associated with the garden styles listed in Group-I are shown under Group II
Group I Group II
P. French Baroque 1.William Kent
Q. Moghul 2.Temple Garden
R. English Romantic 3.Varsailles
S. Japanese 4. Charbag
Select the appropriate combination from the following options
(A) P-1,Q-2,R-3,S-4 (B) P-3,Q-4,R-1,S-2 (C) P-3,Q-2,R-4,S-1 (D) P-4,Q-2,R-3,S-1
Ans:- (B)
Q.34 Choose the right combination of information, which should accompany application for Building
Permission from any urban local government.
P. Site Plan Q. Building Plan R. Key Plan S. Ownership Title
T. Power of Attorney U. Services Plan V. Quantity Estimate
(A) P-Q-R-S-U (B) R-Q-T-V (C) S-T-U-V (D) Q-R-S-T
Ans:- (A)
582 GATE Architecture and Planning: Comprehensive Question Bank

Q.35 Which of the following are the phenomena exhibited by the sound waves
P Reflection
Q Refraction
R Interference
S Polarization
T Diffraction
Indicate the correct combination
(A) P,Q,R (B) Q,R,S,T (C) P,R,S,T (D) P,Q,R,T
Ans:- (C)
Q.36 The net rent received from a property is Rs.1,00,000/- per annum. The outgoings is Rs 20,000/-per
annum. It is expected that the rent will be received for a long period. Estimate the rental value of the
property for 10% rate of interest.
(A) Rs 9,00,000/- (B) Rs 10,00,000/- (C) Rs 10,20,000/- (D) Rs 11,00,000/-
Ans:- (C)
Solution: Net Rent = 1,00,000/-, Outgoings = 20,000.
Hence, Actual Rent is 1,20,000/-
10% of Rent value = actual rent = 1,20,000/-
Rent value = 10,20,000/-
Q.37 Reservoir capacity can be determined from the graph drawn between
(A) Cumulative runoff and time (B) population and time
(C) runoff and time (D) water demand and time
Ans:- (D)
Q.38 AII the meanings of the following architectural terms are not correct.Select the correct answer
Sr. No. Term Meaning
P PLAZZA an open public square
Q COLUMN a horizontal support
R STUCCO art of high relief decoration by moulding plaster
S PORTICO entry porch or vestibule supported by columns
(A) P,Q (B) P,S (C) Q,R (D) R,S
Ans:- (D)
Q.39 Number of modular bricks required for Ten cubic metre brickwork in a house is
(A) 4500 (B) 4750 (C) 5000 (D) 5225
Ans:- (C)
Solution: the size of the modular brick is 200 mm x 100 mm x 100 mm.
Volume of one modular brick: = 0.2 m x 0.1 m x 0.1 m = 0.002 m3
For 10 cubic meters of brickwork: Number of bricks = Total volume / Volume of one brick
= 10 m3 / 0.002 m3/brick = 5000 bricks
Hemant Vilas Parulekar 583

Q.40 A residential area of 80 hectares has the following residential plot subdivision Each plot has one
dwelling unit and the average household size is 5 persons. The rest area is devoted to roads, schools,
parks and shops.
What are the gross and net Plot Size Numbers
density respectively of the 500 sq m 100
area in person per hectare
300 sq m 500
(ppha)?
200 sq m 1000
(A) 100 ppha and 200 ppha
(B) 500 ppha and 300 ppha
(C) 200 ppha and 100 ppha
(D) 200 ppha and 300 ppha
Ans:- (A)
Solution:
Description Calculation Result
Total Number of Households 100 + 500 + 1000 1600 households
Total Population 1600 × 5 8000 persons
Area for 500 sq m plots 100 × 500 sq m 5 hectares
Area for 300 sq m plots 500 × 300 sq m 15 hectares
Area for 200 sq m plots 1000 × 200 sq m 20 hectares
Total Residential Area 5 + 15 + 20 40 hectares
Gross Density 8000 persons ÷ 80 ha 100 ppha
Net Density 8000 persons ÷ 40 ha 200 ppha

Q.41 A site map shows six contour lines at 5m contour interval. The midpoint distance between the
topmost and the bottom most contour lines is 600 metres. The site measures 100 metre x 100
metre square. What is the slope between the above two midpoints?
(A) 1 in 60 (B) 1 in 30 (C) 20% (D) 5%
Ans:- (D)
Solution: No. of Contour lines = 6, Interval between contour lines = 5m. Thus rise = 6 x 5m = 30m
Midpoint distance between topmost and bottommost contour lines = 600m (slant height/hypt)
By Pythogoras theorem, hypt2 = base2 + height2
Here, 6002 = base2 + 302. Thus, base = 599.24 ≈ 600m
For, 600m base the rise is 30m. Thus, 5m rise is for 100m (5%)
Q.42 A small commercial plot measuring 20 metre x 30 metre located in a community centre is subjected
to maximum ground coverage of 60% and FAR 300. With fullest land utilization and uniform floor,
areas how many floors can be built on the plot.
(A) 6 (B) 3 (C) 5 (D) 10
584 GATE Architecture and Planning: Comprehensive Question Bank

Ans:- (C)
Solution:
Parameter Calculation Value
Plot Size 20m x 30m 600 sq.m
Maximum Ground Coverage 60% of 600 sq.m 360 sq.m
Total permissible built-up area (FAR) Plot area x FAR 1800 sq.m
Area of one floor (using max coverage) - 360 sq.m
Number of floors Total permissible area / Floor area 5 floors
Q.43 Which of the following percentage distribution of different land use is the most representative one
for a medium size urban area?
(A) R:45%,C:4%,P:12%,T:14%,I:8% (B) R:30%,C:4%,P:35%,T:14%,I:10%
(C) R:30%,C:20%,P:12%,T:14%,I:10% (D) R:45%,C:4%,P:12%,T:14%,I:20%
Ans:- (C)
Q.44 The most appropriate hierarchical sequence of plans is
(A) Regional Plan->Master Plan->Action Plan->Perspective Plan
(B) Regional Plan->Perspective Plan->Master Plan->Action Plan
(C) Perspective Plan->Regional Plan->Master Plan->Action Plan
(D) Master Plan->Perspective Plan->Action Plan->Regional Plan
Ans:- (C)
Explanation:
Plan Purpose and Scope of Plan Time
Perspective Plan A perspective plan is a long-term vision for the future development 20-30 years
of an area. It typically looks 20-30 years into the future and identifies
the key trends and challenges that will need to be addressed.

Regional Plan A regional plan is a high-level plan that covers a large geographic 20 years
area, such as a state, province, or region. It outlines the overall
development goals for the area and identifies key issues such as
transportation, infrastructure, and land use.

Master Plan A master plan is a more detailed plan for a specific area, such as a 10 years
city, town, or neighborhood. It provides more specific guidance on
land use, zoning, and infrastructure development.

Action Plan An action plan is a detailed plan for implementing the master 1-2 years
plan. It outlines the specific steps that will be taken to achieve the
development goals set out in the master plan.

Q.45 A country has the largest city size of 5 million. What will be the population size of fourth largest city
according to the rank size rule?
(A) 1 million (B) l lac (C) 1.25 Lacs (D) 1.25 million
Hemant Vilas Parulekar 585

Ans:- (D)
Solution: According to the rank-size rule, in a city-size distribution where the largest city has a population
of 5 million, the population of the fourth-largest city would typically be about one-fourth (1/4) of the
population of the largest city.
Therefore, the population size of the fourth-largest city would be:
5 million (population of the largest city) / 4 = 1.25 million
Q.46 The following figures represent four different study areas

Which of the following combination of the ‘Study Area’ and ‘number of nursery school required’ is
the most appropriate as per the planning norms?
(A) P-1,Q-2,R-2,S-2 (B) P-1,Q-2,R-1,S-1
(C) P-1,Q-2,R-2,S-1 (D) P-1,Q-1,R-2,S-2
Ans:- (B)
Solution:
Planning Unit Population Facilities No. Area per unit (Ha) Total Area
Housing 5000 Pre-Primary 2 0.08 0.16

Area P Area Q Area R Area S


Area (sq.m.) 60000 60000 60000 96000
Area (Ha) 6 6 6 96
Population density (PPHA) 420 600 420 25
Population 2520 3600 2520 2400
No. Of nursery school required = (Population/2500) 1 1.44 = 2 1 1
Q.47 The following figures P, Q, R represent three areas of different population sizes. The Small circles
within represent distribution of two consecutive hierarchical levels of commercial centres in each
study area.Typologies of these commercial centers are given below.
Typologies
1.Community Centre 2.Convenient Shop
3.District Centre 4.Sector market
Which of the following combination represents
the most appropriate typology of commercial
centres in these areas.
(A) P-2,1 Q-2,3 R-3,1 (B) P-2,1 Q-1,4 R-1,3
(C) P-2,4 Q-4,1 R-1,3 (D) P-2,4 Q-4,3 R-3,1
586 GATE Architecture and Planning: Comprehensive Question Bank

Ans:- (C)
Explanation:
Sr. No. Category Population Served per unit
1 Convenience Shopping 5000
2 Local shopping including service center 15000
3 Community center with service center 100000
4 District center 1 at district level/500000 Population
5 Sub city center 25 Lakh to 50 Lakh
6 City center 50 Lakh
7 Local wholesale market/mandi 10 Lakh
8 Weekly markets 1 to 2 locations for every 1 Lakh
Source: National Building Code of India.
Q.48 On street parking along a road kerb has provision for 45 degree angular parking with car spaces of
5.4 m by 2.5 m. How many cars can be parked in 400 m stretch along the kerb?
(A) 100 cars (B) 110 cars (C) 120 cars (D) 130 cars
Ans:- (C)
Solution:
Q.49 Moment at the fixed end ‘A’ of the beam indicated below is

(A) (-)wl2/12 (B) (-)wl/16 (C) (-)wl2/8 (D) (-)wl/8


Ans:- (C)
Q.50 Consider the following qualities of building materials:
P Fire resistance Q Moisture resistance R Strength S Thickness T Stability
Which of the following combination of qualities is most appropriate for thermal insulation?
(A) P,Q,R (B) P,Q,S (C) Q,R,S (D) R,S,T
Ans:- (B)
Q.51 A 300 metre long and 6 m wide pathway is to be illuminated with 4000 lumen lamps having
maintenance factor of 0.8 and coefficient of utilization 0.45. The desired average lux on the
pathway is 6. What should be the spacing between the lamps?
20m (B) 30m (C) 40m (D) 50m
Ans:- (C)
Solution: Area of the pathway = 300 x 6 = 1800m2
Luminous flux of each lamp = 4000 lumen
M.F. = 0.8, C.F. = 0.6
Hemant Vilas Parulekar 587

Average illumination required = 6 lux Formula = (N x F x U.F. x M.F.) / A


Where, E = Average illumination required in lux, N = no. of lamps
F = lumen per lamp, U.F. = Utilization factor M.F. = Maintenance Factor
A= Area of room
Hence, N = (6 x 1800)/ (4000 x 0.45 x 0.8) = 7.5 ≈ 8,
Assume 2 out 8 lamps are placed on end sides of pathway
The spacing required between 6 intermediate lamps = 300/7 = 42.85 ≈ 40
Q.52 Sight lines are manipulated in the design of a complex by
P Slight shifts in ground level
Q Path directions
R Position of opaque barriers
S Some focal objects
T Rising levels of middle ground
Which of the following combinations provide the best manipulation?
P,Q,R (B) Q,R,S (C) R,S,T (D) P,R,S
Ans:- (B)
Q.53 Following factors are considered for architectural design
P. Scale of different spaces Q. location of elements
R. Topographical conditions S. Structural system
Which of the following combination of factors is most suitable for proximity diagramming?
Ans:- (B)
Q.54 Which of the following combination correctly represent the labels in the diagram given below?
(A) (B) (C) (D)
Ridge Q R Q P
Valley R Q R Q
Hip P P S R
Gable End S S T T
Eaves Projection T T P S
Ans:- (B)
Q.55 Indicate the correct combination of dimensions (cm) against the labels shown below in the diagram
of the w.c.
(A) (B) (C) (D)
P 70 30 55 75
Q 50 24 34 55
R 130 90 100 130
S 15 15 24 30
T 90 65 78 95
U 60 28 39 30
Ans:- (A)
588 GATE Architecture and Planning: Comprehensive Question Bank

Q.56 Indicate the correct combination of duals of five platonic solids given below:-
T. TETRAHEDRON C. CUBE O. OCTAHEDRON (A) (B) (C) (D)
T-T T-O T-D T-I
I. ICOSAHEDRON D. DODECAHEDRON
C-O C-D C-C D-D
I-D I-I I-O O-C
Ans:- (B)
Q.57 Indicate correct combination of names of the following bolts:

A B C D
P L-Bolt Wood Bolt Rectangular Bolt Crank Bolt
Q Coach Screw Wood Screw Needle Screw Coach Screw
R Purlin Bolt J-Bolt Hook Bolt Hook Bolt
S Left Crank Bolt Left Hand Bolt Skewed Bolt Cranked Hook Bolt

Ans:- (B)
Q.58 Match the following landscape architects listed in Group I with their work listed in Group II
(A) (B) (C) (D) Group I Group II
P-3 P-1 P-3 P-3 P. Andre Le Notre 1. Versailles,France
Q-4 Q-2 Q-2 Q-2 Q. Valladier 2. The Piazza del Popolo,Rome
R-6 R-6 R-4 R-6 R. John Wood 3. The Regents Park
S-5 S-3 S-6 S-5 S. John Nash 4. Palace de la Concorde
5. Terrace Trivoli Garden
1. The Royal Crescent & Circus at Bath
Ans:- (B)
Explanation:
1. Central Park, New York City
-Designer: Frederick Law Olmsted and Calvert Vaux
-Description: One of the most famous urban parks in the world, designed in the 19th century as a
naturalistic oasis in the heart of Manhattan.
2. Villa d’Este, Tivoli, Italy
-Designer: Pirro Ligorio
-Description: A Renaissance garden known for its impressive and elaborate Italian-style garden, terraces,
and numerous fountains.
3. Versailles Gardens, Versailles, France
-Designer: André Le Nôtre
Hemant Vilas Parulekar 589

-Description: A classic example of a French formal garden, featuring symmetrical patterns, precisely
trimmed hedges, and grand fountains.
4. Keukenhof Gardens, Lisse, Netherlands
-Designer: Jan David Zocher and his son Louis Paul Zocher
-Description: Renowned for its stunning floral displays, particularly tulips, Keukenhof is one of the
world’s largest flower gardens.
5. Butchart Gardens, British Columbia, Canada
-Designer: Jennie Butchart
-Description: Originally a limestone quarry, the gardens are now a vibrant display of floral displays and
thematic gardens.
6. Katsura Imperial Villa, Kyoto, Japan
-Designer: Kobori Enshu (attributed)
-Description: An example of a traditional Japanese garden, featuring serene landscapes, ponds, and tea
houses.
7. Royal Botanic Gardens, Kew, London, United Kingdom
-Designer: Various, including William Nesfield and Capability Brown
-Description: A world-renowned botanical research and education institution, featuring diverse plant
collections and historic glasshouses.
8. Alhambra and Generalife Gardens, Granada, Spain
-Designer: Unknown, Islamic-Moorish style
-Description: Famous for their architectural elements, reflecting pools, and integration of natural
landscape features.
9. Jardim Botânico, Rio de Janeiro, Brazil
-Designer: John Tyndale (redesign)
-Description: A botanical garden containing a wide variety of tropical plants and trees, including a
famous avenue of palm trees.
10. Sissinghurst Castle Garden, Kent, United Kingdom
-Designer: Vita Sackville-West and Sir Harold Nicolson
-Description: Known for its sophisticated color schemes and romantic design, it’s one of the most
admired English gardens.
11. Ryoan-ji Temple Garden, Kyoto, Japan
-Designer: Unknown, attributed to Soami
-Description: Famous for its Zen rock garden, an iconic example of Japanese dry landscape design.
12. Gardens of the Château de Villandry, Villandry, France
-Designer: Joachim Carvallo
590 GATE Architecture and Planning: Comprehensive Question Bank

-Description: Best known for its Renaissance gardens, including a water garden, ornamental flower
gardens, and vegetable gardens laid out in formal patterns.
13. High Line, New York City, USA
-Designer: James Corner Field Operations (with Diller Scofidio + Renfro and Piet Oudolf)
-Description: An urban linear park built on a historic freight rail line elevated above the streets of
Manhattan’s West Side.
14. Boboli Gardens, Florence, Italy
-Designer: Niccolò Tribolo, later expanded by Bartolomeo Ammannati, Giorgio Vasari, and others
-Description: A historic park with a collection of sculptures dating from the 16th through the 18th
centuries, with some Roman antiquities.
15. Powerscourt Gardens, County Wicklow, Ireland
-Designer: Daniel Robertson
-Description: Set against the backdrop of the Great Sugarloaf Mountain, these gardens are a sublime
blend of formal gardens, sweeping terraces, statues and ornamental lakes, secret hollows and rambling
walks.
16. Nong Nooch Tropical Botanical Garden, Pattaya, Thailand
-Designer: Pisit and Nongnooch Tansacha
-Description: Renowned for its beautifully landscaped gardens, it features a symphony of colors with its
vast variety of flowering plants.
17. Château de Chantilly Gardens, Chantilly, France
-Designer: André Le Nôtre
-Description: Features a French-style garden with elaborately designed flower beds, water features, and
reflecting pools.
18. Kirstenbosch National Botanical Garden, Cape Town, South Africa
-Designer: Harold Pearson
-Description: Famous for its rich and diverse flora, it’s located at the eastern foot of Table Mountain and
includes a unique conservatory with plants from different parts of the world.
19. Garden of Cosmic Speculation, Dumfries, Scotland
-Designer: Charles Jencks
-Description: A private garden that opens to the public once a year, known for its striking landscapes
and philosophical inspirations, mixing natural features with artificial symmetry and curves.
20. Jardin Majorelle, Marrakech, Morocco
-Designer: Jacques Majorelle, later restored by Yves Saint-Laurent and Pierre Bergé
-Description: Known for its cobalt blue accents, the garden blends Moorish and Art Deco influences
and houses a collection of plants from five continents.
Hemant Vilas Parulekar 591

Q.59 Identify two attributes which best describe to the works of Michael Graves
P. Anthropomorphic Q. Aesthetic irony R. Traditional Symmetry S. Symbolic
P,S (B) P,Q (C) Q,R (D) R,S
Ans:- (A)
Q.60 Select three important factors from the following which were considered Louis I. Kahn in his works
P. Materials and the way they can be joined together Q. Heights
R. Sensitivity to light S. Grand Spaces T. Glass and steel
(A) P,Q,R (B) P,R,S (C) R,S,T (D) P,S,T
Ans:- (B)
Explanation: Louis I. Kahn was known for his unique architectural philosophy and considerations.
Among the options provided, three important factors that were considered in his works include:
• Sensitivity to light (R): Kahn had a deep understanding of the significance of light in
architecture. He often used natural light as a design element, creating spaces that were
sensitive to the play of light and shadow.
• Grand Spaces (S): Kahn was known for his use of monumental and grand spaces in his
architectural designs. He often created buildings with impressive interior spaces that conveyed
a sense of awe and monumentality.
• Materials and the way they can be joined together (P): Kahn paid great attention to the
choice of materials and the craftsmanship involved in construction. He believed that the
way materials were joined together could greatly influence the aesthetics and function of a
building.
Q.61 The abstract form, shown, of an Ancient Indian settlement represents?
(A) Karmuka
(B) Nandyavarta
(C) Padmaka
(D) Swastika
Ans:- (B)
Explanation: Please refer GATE-2013, Q.7 for more information.
Q.62 Match the following commands in the AUTOCAD.
P-REVSURF 1-Reverse the last command
Q-SURFTAB 2-Makes a view portable to other software applications
R-VIEWPORT 3-revolve an entity
S-SHX 4-opens another window
5-surfing the table generated
6-assigning the mesh density
7-related to shape file
(A) P-1,Q-5,R-2,S-6 (B) P-2,Q-6,R-4,S-5 (C) P-5,Q-3,R-7,S-4 (D) P-3,Q-6,R-4,S-7
592 GATE Architecture and Planning: Comprehensive Question Bank

Ans:- (D)
Explanation: Based on the provided AUTOCAD commands and their typical functions:
P. REVSURF: This command is used to create a surface by revolving a profile (entity) around an axis.
So, P matches with 3 - revolve an entity.
Q. SURFTAB: This command is typically related to the mesh density of a surface. So, Q matches with
6 - assigning the mesh density.
R. VIEWPORT: The VIEWPORT command in AutoCAD is used to create multiple views of a drawing
in a single layout. This means you can open another window within the main window to view the same
drawing from a different perspective or scale. So, R matches with 4 - opens another window.
S. SHX: This refers to shape files in AutoCAD, which are compiled shape files that contain shape
definitions and font definitions for displaying custom text. So, S matches with 7 - related to shape file.
Q.63 Identify the appropriate combination of the following items vis-à-vis their description, which
may be either True or False.
P-SPLINE generates curve on the polyline
Q-LTYPEGEN generates continuity of linetype in a polyline
R-WCS Window compression by scaling
S- XREF references X coordinate of the element
(A) P-F,Q-T,R-T,S-T (B) P-T,Q-T,R-F,S-F (C) P-F,Q-F,R-F,S-T (D) P-T,Q-F,R-T,S-F
Ans:- (B)
Explanation: P. SPLINE: This command fits a smooth curve through a set of points, effectively generating
a curve on a polyline.
• True
Q. LTYPEGEN: This system variable, when set to 1, ensures that the linetype pattern continues based on
the object’s real length, ensuring the continuity of linetype in a polyline.
• True
R. WCS: Stands for “World Coordinate System.” It refers to the global, or world, coordinate system. It’s
not about window compression by scaling.
• False
S. XREF: Stands for “External Reference.” It allows you to insert and link another drawing into your
current drawing. It doesn’t reference the X coordinate of an element.
• False
So, the correct match is: (B) P-True, Q-True, R-False, S-False.
Q.64 Identify the appropriate match between Group A and Group B from the choices given below.
( A ) P - 5 , Q - 3 , R - 1 , S - Group A Group B
2,T-4 (B)
P. Main(){........} 1 Pascal
P-5,Q-4,R-2,S-3,T-1
Q. Begin{.....}End 2 Lisp
(C)P-3,Q-1,R-4,S-
R. (Defun().......) 3 Basic
2,T-5 (D)
P-5,Q-1,R-2,S-4,T-3 S. Format(*,*)..... 4 Fortran
T. NEM 5C
Hemant Vilas Parulekar 593

Ans:- (D)
Explanation: P. Main(){........} This structure is commonly associated with the C language.
• 5C
Q. Begin{.....}End This structure is commonly associated with Pascal.
• 1 Pascal
R. (Defun().......) This structure is commonly associated with Lisp.
• 2 Lisp
S. Format(,)..... This structure is commonly associated with Fortran.
• 4 Fortran
T. NEM This doesn’t immediately associate with a standard structure from the given languages. Without
more context, it’s hard to place, but it’s definitely not C, Pascal, Lisp, or Fortran based on the provided
options.
Given the above matches:
P-5, Q-1, R-2, S-4, T-?
Q.65 Match the following building materials in Group 1 with their predominant use in places as mentioned
in Group
(A) P-4,Q-3,R-2,S-5 Group 1 Group 2
(B) P-2,Q-4,R-3,S-1 P. Glazed Bricks 1. Mohenjo-daro
(C) P-2,Q-5,R-3,S-4 Q. Terracotta 2. Mesopotamia
R. Fine grained marble 3. Greece
(D) P-2,Q-3,R-4,S-5
S. Sun dried bricks 4. Rome
5. Egypt
Ans:- (B)
Q.66 Match the name of books in Group I with the authors listed in Group II
(A) P-3,Q-2,R-6,S-4 Group 1 Group 2
(B) P-3,Q-1,R-2,S-5 P. Design of cities 1. Fredric Gibbard
(C) P-6,Q-5,R-3,S-1 Q. Town design 2. Kevin Lynch
R. Site Planning 3. Edward N. Bacon
(D) P-5,Q-6,R-3,S-2
S. Comments on neighbourhood concept 4. Ebenezer Howard
5. Hans Blumenfeld
6. Charles Abrams
Ans:- (B)
Q.67 For a stadium select most suitable sequence of design considerations.
P. Circulation Requirement Q. Easy access of seating
R. Image of technology S. Visual Communication
(A) Q,S,P,R (B) P,Q,R,S (C) S,R,P,Q (D) Q,P,R,S
Ans:- (A)
594 GATE Architecture and Planning: Comprehensive Question Bank

Q.68 A good thermal insulating material should possess the several qualities. Select the combination of
best three factors.
P. Fire-resistance Q. Moisture resistance R. Strength S. Stability T. Soundproofing
(A) P,Q,S (B) Q,R,S (C) P,Q,R (D) R,S,T
Ans:- (B)
Q.69 Match the towns in Group 1 with their locations in Group 2.
(A) P-4,Q-1,R-4,S-6 Group 1 Group 2
(B) P-1,Q-2,R-5,S-3 P. Miletus 1. Roman
(C) P-5,Q-6,R-3,S-4 Q. Timgad 2. Greek
R. Montpazier 3. French
(D) P-2,Q-1,R-4,S-3
S. Place Dauphine 4. English
5. German
6. Russian
Ans:- (D)
Name of Village Miletus Timgad
Period/Location Greek Roman
Plan

Name of Village Montpazier Place Dauphine (Paris)


Period/Location Greek Roman

Q.70 Match the following paints in Group l against their most important quality mentioned in Group 2.
(A) P-1,Q-3,R-4,S-5 Group 1 Group 2
(B) P-2,Q-3,R-5,S-3 P. Cement Paint 1. Water proofing
(C) P-1,Q-6,R-3,S-4 Q. Plastic Emulsion Paint 2. Glossy appearance
R. Oilbound distemper 3. Decorative appearance
(D) P-2,Q-1,R-4,S-6
S. Synthetic Enamel paint 4. Easily washable
5. Fire-resistant
6. Acid resistant
Hemant Vilas Parulekar 595

Ans:- (A)
Explanation: P. Cement Paint:
1. Water proofing: Cement paints are primarily used for their water-proofing qualities, especially for
exterior surfaces.
Q. Plastic Emulsion Paint:
3. Decorative appearance: Plastic emulsion paints give a smooth and stylish finish to walls, often used
for interiors because of their decorative appearance.
R. Oilbound distemper:
4. Easily washable: Oilbound distempers are a type of water-based paint, but they have some amount
of oil, which makes them more durable and washable than regular distempers.
S. Synthetic Enamel paint:
2. Glossy appearance: Synthetic enamel paints are known for their glossy and shiny finish. They are
durable and provide a smooth finish, often used on metal and wood surfaces.
Q.71 Strips of two different metals firmly joined together as shown in the figure. Select the changed
profile of the joined strips, while heated from the figures shown below.

(A) P (B) Q (C) R (D) S


Ans:- (A)
Q.72 Match the propagators with the following concepts.
(A) P-4,Q-3,R-2,S-5,T-1 Propagators Concepts
(B) P-3,Q-4,R-2,S-5,T-1 P. Total Architecture 1. Patrick Geddes
(C) P-5,Q-2,R-4,S-1,T-3 Q. Ekistics 2. Jean Gottman
R. Megalopolis 3. Doxiadis
(D) P-4,Q-5,R-3,S-2,T-1
S. Radburn 4. Walter Gropius
T. Conservative Surgery 5. Fire-resistant
6. Clarence Stein
Ans:- (A)
Q.73 In Gandhi Smarak Sangrahalaya architect Charles Correa used different types of finishes. Select the
combination of finishes used in the building.
P Stone Cladding Q Exposed brick
R Plastering and painting S Marble chip finish
T Exposed concrete Q Plain cement concrete finish
(A) R,S,T (B) Q,T,U (C) P,Q,R (D) Q,R,S
Ans:- (B)
596 GATE Architecture and Planning: Comprehensive Question Bank

Q.74 Match the correct names or the architectural styles and periods.

1. Moorish Style 2. Romanesque 3. Style Gothic Style 4. Renaissance Style


15th Century 11th-I2th Century 13th-14th Century 15th-16th Century
(A) P-1,Q-2,R-3,S-4 (B) P-2,Q-4,R-3,S-1 (C) P-1,Q-3,R-4,S-2 (D) P-2,Q-1,R-2,S-2
Ans:- (C)
Q.75 A 4 cm x 4 cm area on a map represented a land area of 16 hectares of ground. If this map is
transformed to a scale of 1:5000 the same ground area will be represented by
(A) 80 square centimetre (B) 32 square centimetre
(C) 64 square centimetre (D) 40 square centimetre
Ans:- (C)
Solution: The initial area on the map represents 16 hectares. Now, you want to determine the area at a
new scale of 1:5000.
1. Calculate the scale factor:
Scale Factor = New Scale / Old Scale Scale Factor = 1/5000
2. Calculate the new area on the map:
New Area on Map = Old Area on Map × (Scale Factor)²
Old Area on Map = 4 cm × 4 cm = 16 square centimeters
New Area on Map = 16 square centimeters × (1/5000)²
New Area on Map = 16 square centimeters × (1/25,000,000)
New Area on Map = (16/25,000,000) square centimeters
Now, let’s calculate this:
New Area on Map ≈ 0.00000064 square centimeters
So, the same ground area will be represented by approximately 0.00000064 square centimeters on the
map at a scale of 1:5000.
Among the given options, the closest one is:
(C) 64 square centimeters
Q.76 The figures 1 and 2 represent two different case of building
forms, building layouts and the site features on sites located
respectively in hot-humid and hot- dry regions. Considering
the factors listed below which of the following options compared
best the relative presence and absence of these factors in the
two cases.
Hemant Vilas Parulekar 597

Factors (A) Figure 1 has more V, more P and better E than figure 2
V. Cross Ventilation (B) Figure 1 has less V, more P and better E than figure 2
P. Privacy in outdoor space (C) Figure 1 has more V, more P and less E than figure 1
E. Exposure to outside view (D) Figure 1 has more V, less P and better E than figure 2
Ans:- (A)
Q.77 The graph below shows urban population:
Which of the following statements is INCORRECT?
(A) Urbanisation started much later in ‘P’ is ‘S’ and it has achieved
lower growth rate.
(B)Growth rate is higher in ‘Q’ than in ‘R’.
(C)Growth rate is higher in ‘P’ than in ‘Q’, ‘R’, ‘S’ after 1900
(D)Urban population is higher in ‘R’ than in ‘Q’.
Ans:- (C)
Q.78 The following figures show four different unmanaged traffic intersections in a city. Which of the
following statements shows the correct numbers of potential traffic conflict points (excluding
merging points) at the intersections?

P R S T
(A)P-8,Q-3,R-3,S-2 (B)P-16,Q-3,R-3,S-0 (C)P-8,Q-3,R-3,S-0 (D)P-16,Q-3,R-3,S-2
Ans:- (C)
Q.79 Arrange the sequence of events in a Residential site development.
P. Levelling and land filling Q. Trees and plantation R. Electrification
S. Allotment T. Provision of roads U. Provision of water supply, sewerage and drainage.
(A) R,S,T,U,P,Q (B) P,R,S,U,T,Q (C) Q,P,S,R,T,U (D) P,T,Q,U,R,S
Ans:- (D)
Q.80 Indicate the correct match for items given in Group I with items given in Group II.
P-2,Q-4,R-5,S-1 Group I Group
P-2,Q-3,R-1,S-6 Stone Pitting
P-3,Q-1,R-5,S-2 Timber Mottle
Brick Chalking
P-1,Q-3,R-4,S-2
S. Lime Star shake
Unburnt
598 GATE Architecture and Planning: Comprehensive Question Bank

Ans:- (A)
Material Stone Timber
Defect Mottle Star Shake
Defect Description: Stone mottle is a defect in Description: Star shake is a defect in timber
stone masonry characterized by irregular characterized by cracks that radiate outwards
discoloration or patchy appearance on the from the center of the log, resembling a star.
stone surface. Causes: It is commonly caused by uneven
Causes: It's typically caused by the uneven drying or shrinkage of the wood, where the
distribution of minerals or impurities outer layers dry faster than the inner layers,
within the stone, or by differential creating internal stresses.
weathering where certain parts of the Impact: This defect weakens the timber and
stone surface weather differently than makes it unsuitable for structural purposes.
others. It also reduces the timber’s aesthetic and
Impact: While often not structurally commercial value.
significant, it can mar the appearance of
stone masonry and may indicate areas
more susceptible to weathering.
Images

Material Brick Lime


Defect Unburnt Pitting
Description: An unburnt brick is a brick Description: This defect occurs in lime
that hasn’t been adequately fired in a kiln. plaster and is characterized by the formation
Causes: This defect is typically a result of of small pits or depressions on the plaster
insufficient firing temperature or uneven surface.
heat distribution in the kiln during the Causes: Lime pitting is often caused by the
manufacturing process. presence of unslaked lime particles in the
Impact: Unburnt bricks are usually lime putty. When these particles eventually
weaker, less durable, and more porous slake after the plaster has been applied, they
than properly fired bricks. They are more expand and create small eruptions or pits on
susceptible to water absorption and less the surface.
resistant to weathering and load-bearing Impact: Apart from being aesthetically
pressures. unpleasing, lime pitting can also indicate
potential weaknesses in the plaster.
Q.81 A stack of sand measures 2.5m x 3.5m at the top and 3.5m x 4.5m at the base. The average height of
the stack is 1 metre. What is the volume of sand in the stack?
(A) 10 cum (B) 12cum (C)15 cum (D)18 cum
Hemant Vilas Parulekar 599

Ans:- (B)
Solution: To find the volume of the sand stack, we can treat it as a frustum of a pyramid. The formula
for the volume VV of a frustum is:
V = h/3×[A1+A2+(A1×A2)2]
Where:
• h is the height of the frustum (in this case, 1 metre).
• A1 is the area of the top surface.
• A2 is the area of the base surface.
First, let’s find the areas of the top and base:
A1 = 2.5m×3.5m = 8.75 square
A2 = 3.5m×4.5m = 15.75 square
Now, plug these values into the formula:
V= (1/3) × (8.75+15.75+8.75×15.75)
V= (1/3) × (24.5+137.8125)
V= (1/3) × (24.5+11.74)
V= (1/3) × 36.24
V=12.08 cubic metres
Rounding it off, the volume is approximately 12 cubic metres.
Q.82 In a residential community of 10000 population 20% are Higher Secondary school going children.
The expected enrolment is 80% and per capita gross floor space required is 3.0 Sq m. The ground
coverage permissibility is 40%. Indicate the land area required for the Higher Secondary school
building.
(A) 0.85 hectare (B) 6.0 hectare (C) 1.2 hectare (D) 7.5 hectare
Ans:- (C)
Explanation:- Let’s calculate the land area required for the Higher Secondary school building step by
step:
1. Calculate the number of Higher Secondary school-going children: Total population = 10,000
Percentage of Higher Secondary school-going children = 20% Number of Higher Secondary school-
going children = 10,000 × 0.20 = 2,000
2. Calculate the expected enrollment: Enrollment rate = 80% Expected enrollment = 2,000 × 0.80 = 1,600
3. Calculate the total gross floor space required: Per capita gross floor space = 3.0 sq m Total gross floor
space = 1,600 × 3.0 = 4,800 sq m
4. Calculate the land area required based on ground coverage permissibility: Ground coverage
permissibility = 40% Land area = Total gross floor space / Ground coverage permissibility Land area =
4,800 sq m / 0.40 = 12,000 sq m
Now, convert the land area from sq m to hectares: 1 hectare = 10,000 sq m Land area required = 12,000
sq m / 10,000 = 1.2 hectares
So, the correct option is (C) 1.2 hectare.
600 GATE Architecture and Planning: Comprehensive Question Bank

Q.83 Which is the correct match for items given in Group I with items given in Group II.
Group I Group II
P. Thermal Inversion 1.Water repellent
Q. Efflorescence 2. Stable airflow
R. Hydrophobic surface 3. Lecture Hall
S. Psychedelic effect 4. Crystallization
5. Globulation
(A) P-2,Q-4,R-1,S-3
(B) P-2,Q-3,R-4,S-1
(C) P-1,Q-4,R-3,S-1
(D) P-5,Q-3,R-1,S-2
Ans:- (A)
Explanation:-
Thermal Inversion: Thermal inversion is a meteorological phenomenon where the normal temperature
gradient in the atmosphere is reversed, and temperature increases with altitude instead of decreasing.
This situation leads to stable airflow and can trap pollutants near the ground, causing air quality issues,
especially in urban areas.
Efflorescence: Efflorescence refers to the formation of salt deposits on the surface of porous materials
like masonry or concrete. It occurs when water-soluble salts within the material dissolve in moisture,
and then the moisture migrates to the surface and evaporates, leaving the salts behind as crystalline
deposits. Efflorescence can be an aesthetic issue and, in severe cases, can also cause structural damage.
Hydrophobic surface: A hydrophobic surface is one that repels water and resists wetting. This property
is due to the surface’s low affinity for water molecules, which prevents them from spreading out and
forming a continuous film. Hydrophobic surfaces can be found in nature, such as the leaves of some
plants, or can be artificially created through the application of special coatings or treatments to materials.
Psychedelic effect: A psychedelic effect refers to the perceptual changes and distortions that may occur
when under the influence of psychedelic substances or experiencing altered states of consciousness.
These effects can include visual and auditory hallucinations, changes in time perception, and intensified
emotions. In the context of a lecture hall, the term may refer to the impact of lighting and acoustics on
the perception and experience of the space.
Q.84 A general hospital has the following functional areas
P Entry Q Emergency R Out Patient Department
S In Patient Department T Diagnostic Therapeutic Unit U Incinerator
V Mortuary W Kitchen and Laundry
Select from the following group of pairs the one you consider as most appropriate while designing
the hospital.
(A) PR,QW,RS (B) PR,ST,RT (C) RT,PU,RV (D) RV,UW,QR
Ans:- (B)
Hemant Vilas Parulekar 601

Q.85 In high-rise building the walls are built thinner as they get higher. Select the most important pair
of factors.
P. Thinner wall has less weight to bear at the top Q. Thinner wall are suitable for earthquake
R. Thinner wall look beautiful S. Thinner wall can take electrical conduit pipe easily
T. Thinner wall have moment of inertia
(A) P,R (C) R,S (D) Q,T (B) P,Q
Ans:- (B)
Q.86 In the landscape drains correlate the drain types with their positive or negative attributes and select
the appropriate combinations
Group I Group II (A) P-1.2,4,6
P. Open drain 1. Economic (B) Q-2,3,6,4
Q. Swale 2. Easy collection (C) R-1,2,5,6
R. French drain 3. Susceptible to clogging
(D) S-1,4,5,6
S. Pipe drain 4. High rate flow
5. Easy maintenance
6. Visual scar
Ans:- (A)
Drainage system Open drain Swale French drain Pipe drain
Illustration

Q.87 Match the tree forms with their corresponding functions as listed below
1. Visual screening 2. Shading of functions around
3. Backdrop
4. Water side edge planting 5. Boundary edging of a functional
area 6. Scale induction of the adjoining object
(A) P-3,Q-1,R-5,S-2 (B) P-1,Q-3,R-4,S-6 (C) P-1,Q-2,R-5,S-4 (D) P-3,Q-6,R-1,S-4
Ans:- D
Q.88 The Age-Sex pyramid of an urban area shows a bulge in the age group of 15 to 25. Select the likely
correct group of inference.
P. Birth rate has fallen Q.In-migration is high R. Out-migration is high
S. Death rate has fallen T. Males are more than females
(A) P-Q (B) P-R (C) R-T (D) S-T
Ans:- (A)
602 GATE Architecture and Planning: Comprehensive Question Bank

Q.89 Identify the elements marked in the following drawing and match them with the list provided below.

1 Rafter 2 Strut 3 King Post 4 Diagonal tie


5 Flat tie 6 Support 7 Main tie
(A) P-1,Q-2,R-3,S-4,T-5,U-6 (B) P-3,Q-4,R-5,S-1,T-2,U-7
(C) P-3,Q-1,R-3,S-6,T-7,U-4 (D) P-2,Q-4,R-5,S-7,T-6,U-3
Ans:-
Q.90 Select the items in the Group I, which matches most appropriately with items in Group II.
(A) P-1,Q-3,R-5
(B) P-2,Q-4,R-1
(C) P-2,Q-4,R-3
(D) P-1,Q-2,R-3
Group I Group II
P. Air delivery System 1. Adsorption
Q. Air distribution system 2. Dampers
R. Dehumidifiction 3. Spray Washers
S. 4. Diffusers
5. Dry Filter
Ans:- (B)
GATE QUESTION PAPER 2003
General Aptitude (GA)

Q. 1 – Q. 30 carry one mark each.


Q.1 The Sanskrit school of Auroville is a work of
(A) modular precast concrete blocks (B) Ferro cement and terra forming
(C) advanced steel trusses and space frames (D) adobe and mud plastering
Ans:- (D)
Explanation:- The Sanskrit school in Auroville was constructed using the technique of ferrocement and
terra forming. The main structural elements of the building are a series of hyperbolic paraboloid shells
made of ferrocement, which are supported by columns and arches made of brick masonry. The walls are
also made of ferrocement panels, while the floors are made of compacted earth mixed with cement. The
roof is covered with a layer of soil and planted with grass, which provides natural insulation and helps
to keep the building cool.
Q.2 The Baroque city of European Renaissance and the city Beautiful Movements though separated by a
period of 300 years have one of the following things in common
(A) palaces and sculptures (B) colonnades and forum
(C) grand open spaces and waterways (D) radial streets and concentric landscaping
Ans:- (C)
Explanation:- The Baroque city of European Renaissance and the City Beautiful Movement of the late
19th and early 20th centuries both emphasized the creation of grand public spaces and the use of water
features such as fountains and reflecting pools. In the Baroque era, these spaces were often designed to
showcase the power and wealth of the ruling class, while in the City Beautiful Movement they were in-
tended to improve the quality of life for all citizens and create a sense of community. While both move-
ments did include axial planning and formal design elements, radial streets and concentric landscaping
were more characteristic of earlier planning traditions such as the Renaissance and Baroque periods.
Q.3 The survey of India toposheet stating 1:50,000 refer to
(A) graphic scale (B) spatial scale (C) time scale (D) numeric scale
Ans:- (A)
Q.4 Improvement of physical condition of existing population in inner city areas is called
(A) incumbent upgradation (B) gentrification
(C) slum upgradation (D) homesteading
Ans:- (A)
Explanation:- Improvement of the physical condition of existing population in inner city areas is called
incumbent upgradation. This refers to the process of improving the existing infrastructure and living
conditions of the population that already resides in the area, rather than bringing in new residents or
new construction. This can include upgrading buildings, improving access to basic services like water
and electricity, improving transportation links, and creating public spaces for social and recreational
activities. The goal is to create a better living environment for existing residents while maintaining the
social fabric and character of the area.
604 GATE Architecture and Planning: Comprehensive Question Bank

Q.5 The landscape designer of Tara Group of Housing, New Delhi was

(A) Charles Correa (B) Jasbir Sawhney (C) Ravindra Bhan (D) Raj Rewal
Ans:- (A)
Explanation:- Tara Apartments is a group housing complex located in New
Delhi, India. It was designed by renowned architect Charles Correa and
completed in 1978. The complex comprises a total of 162 apartments spread
over 12 blocks, each of which is six stories high. The design of the complex
reflects Correa’s interest in traditional Indian architecture, and draws
inspiration from the stepped wells of Gujarat and Rajasthan.
The complex is organized around a central courtyard, which serves as a social space for the residents.
The courtyard is open to the sky and features a series of stepped terraces that provide seating areas for
residents to gather and socialize. The apartments themselves are arranged around the courtyard in a
stepped formation, with each apartment having access to its own private balcony.
The landscaping for the complex was designed by Charles Correa, another renowned Indian architect.
The landscaping features a variety of trees and plants that provide shade and greenery throughout
the complex. The complex has won several awards for its design, including the Aga Khan Award for
Architecture in 1980.
Q.6 Mline command in AutoCAD 2000

(A) create multiple parallel lines (B) draw two parallel lines

(C) draws polyline (D) moves a line


Ans:- (B)

Explanation:- The MLINE command in AutoCAD 2000 is used to create multiple parallel lines. It is
useful for drawing walls, fences, and other objects that require a series of parallel lines. The MLINE
command allows you to specify the number of lines, the distance between them, and the angle at which
they are drawn. Once you have created a series of parallel lines with the MLINE command, you can edit
them as a single object using the PEDIT command.
Q.7 In Greek structures, distyle, prostyle and amphiprostyle are processes of

(A) symmetry (B) repetition (C) hierarchy (D) clustered organization


Ans: - (C)
Explanation:-
Hemant Vilas Parulekar 605

STYLE Description
In Antis “In Antis” refers to a building style where columns are placed inside the walls
of a structure, creating a symmetrical appearance. This style was commonly
used in ancient Greek temples, where the columns were positioned between
the walls of the cella, creating an entrance porch or pronaos.
Distyle in Antis “Distyle in Antis” refers to a temple style that has two columns in the front, and
they are positioned within the walls of the temple. This style was commonly
used in smaller temples and sanctuaries.
Tholos “Tholos” refers to a round building with a conical or domed roof. These
structures were often used as tombs, shrines, or temples, and were highly
symbolic. The best example of a tholos is the Tholos of Delphi.
Prostyle “Prostyle” refers to a building style with columns only in the front. The columns
create a portico or porch in front of the building. This style was used in various
public buildings, such as stoa, basilicas, and temples.
Amphiprostyle “Amphiprostyle” refers to a building style with columns in both the front and
back, creating a portico at both ends of the structure. This style was used in
larger temples and sanctuaries, such as the Temple of Artemis at Ephesus.
Dipteral “Dipteral” refers to a temple style with a double row of columns around the
exterior, creating a larger open space inside the temple. This style was used in
some of the largest and most prestigious temples, such as the Temple of Zeus
at Olympia.
Peripteral “Peripteral” refers to a temple style with a single row of columns around the
exterior, creating a more compact design and allowing for more internal space.
This style was used in many temples, such as the Parthenon in Athens.
Pseudoperipteral “Pseudoperipteral” refers to a temple style that appears to have a peripteral
design but has engaged columns or pilasters on the interior walls. This creates
a visual effect of a peripteral design while allowing for more internal space.
This style was used in some of the smaller temples, such as the Temple of
Apollo at Didyma.
Pseudodipteral “Pseudodipteral” refers to a temple style with a double row of columns around
the exterior, but the inner row of columns is replaced with engaged columns
or pilasters. This creates a visual effect of a dipteral design while allowing for
more internal space. This style was used in some of the larger temples, such as
the Temple of Artemis at Ephesus.
Q.8 The proportion of Parthenon corresponds to

(A) blue series and the spiral

(B) geometric series and Archimedean solids

(C) rules of Protogoras and Euclidian anthropometrics

(D) entasis and the golden mean


606 GATE Architecture and Planning: Comprehensive Question Bank

Ans:- (D)

Q.9 Resistance of water to the passage of light through it is the measure of

(A) colour (B) turbidity (C) hardness (D) portability


Ans:- (B)
Explanation:- Turbidity is a key water quality parameter that refers to the cloudiness or haziness of a
fluid caused by large numbers of individual particles that are generally invisible to the naked eye, similar
to smoke in the air. The measurement of turbidity is a key test of water quality.
Causes of Turbidity:
-Suspended Solids: This includes silt, clay, fine organic and inorganic matter, plankton, and other mi-
croscopic organisms.
-Human Activities: Agriculture runoff, construction, mining, and urban runoff can introduce sedi-
ments into water bodies.
-Natural Processes: Soil erosion, decaying plant matter, and weather-related events like storms and
heavy rainfall can increase turbidity.
Measurement of Turbidity:
-Nephelometric Turbidity Units (NTU): Turbidity is commonly measured using a nephelometer,
which quantifies the light scattering effect of suspended particles in the water. The unit of measurement
is NTU.
-Formazin Turbidity Unit (FTU): Another scale used for measuring turbidity.
Methods: Instruments used include turbidimeters and spectrophotometers.
Hemant Vilas Parulekar 607

Impact on Water Quality:


-Aesthetics and Usability: High turbidity can make water unappealing for recreational and aesthetic
purposes.
-Ecosystem Health: It can harm aquatic life by reducing oxygen supply in the water and impairing pre-
dation and navigation for some species.
-Water Treatment: Turbidity can complicate water treatment processes for drinking water, as it can
shield harmful pathogens from disinfection processes.
Health and Environmental Concerns:
-High turbidity can harbor pathogens like bacteria, viruses, and parasites, increasing the risk of water-
borne diseases.
-It can affect the penetration of sunlight into water bodies, impacting photosynthesis in aquatic plants.
Control and Management:
-Erosion Control Measures: Stabilizing soil in construction sites and agricultural lands.
-Vegetative Buffers: Planting vegetation along water bodies to reduce runoff.
-Sediment Traps: Using sediment basins to capture runoff during storms.
-Water Treatment: Utilizing coagulation, flocculation, and sedimentation processes to reduce turbidity
in drinking water treatment.
In conclusion, turbidity is a significant indicator of water quality, affecting both the ecological health of
water bodies and the safety of drinking water. Its management is crucial in environmental conservation
and public health.
Q.10 The term Directrix and Eccentricity are mainly connected with

(A) anti classic shells

(B) trabeated systems

(C) gothic arches


(D) pneumatic or tensegrity structures
Ans:- (D)

Explanation:- The term directrix and eccentricity are mainly connected with conic sections, which are
a class of curves that can be formed by taking a slice through a cone at different angles. In this context,
the directrix is a fixed line, and the eccentricity is a measure of how “stretched out” the conic section is
compared to a circle. For example, the eccentricity of a circle is 0, while the eccentricity of an ellipse is
between 0 and 1.
Q.11 Identify the variety of plastic used to prepare ‘Perspex”

(A) PVC (B) nylon (C) acrylic (D) polystyrene


608 GATE Architecture and Planning: Comprehensive Question Bank

Ans:- (C)

Explanation:- Perspex is a brand name for a type of acrylic plastic. Acrylic, also known as poly(methyl
methacrylate) or PMMA, is a thermoplastic material that is derived from acrylic acid. It is a transparent
and rigid plastic that can be easily molded into various shapes and sizes.

Acrylic has several advantageous properties such as excellent optical clarity, high tensile strength, good
chemical resistance, and UV resistance. It is also lightweight and has a high impact resistance, making
it a popular choice for various applications such as skylights, signs, display cases, lenses, and aquariums.
Perspex, specifically, is a brand of acrylic that is manufactured by Lucite International. It is known for its
high quality and is widely used in industries such as architecture, design, and construction. It is available
in various colors, finishes, and thicknesses, making it suitable for a wide range of applications.
Q.12 The number of vehicles moving in a specified direction on a given lane and passing a given point
during a specified unit of time is called
(A) traffic density (B) traffic volume

(C) traffic capacity (D) basic capacity


Ans:- (B)

Explanation:- (A) Traffic density: Traffic density refers to the number of vehicles present on a given
length of roadway at a given time. It is usually measured in vehicles per kilometer or vehicles per mile.
High traffic density can indicate congestion or heavy usage of the road, which may require attention for
traffic management or infrastructure improvements.

(B) Traffic volume: Traffic volume, as mentioned earlier, refers to the number of vehicles moving in a
specified direction on a given lane and passing a given point during a specified unit of time. Traffic vol-
ume is an essential measurement for understanding traffic patterns and planning road networks, as it
helps identify areas of high demand or congestion.

(C) Traffic capacity: Traffic capacity is the maximum number of vehicles that a specific roadway or
intersection can accommodate within a given time period while maintaining an acceptable level of ser-
vice. Capacity depends on factors like the width of lanes, road geometry, traffic control devices, and the
behavior of drivers. Traffic engineers use capacity as a key consideration when designing new roads or
improving existing ones.

(D) Basic capacity: Basic capacity refers to the maximum number of vehicles that can pass through a giv-
en point on a road under ideal conditions, such as good weather, no obstructions, and optimal driving
behavior. Basic capacity is often used as a benchmark for comparing the performance of different roads
or intersections and can help identify areas where improvements might be needed to accommodate
higher traffic volumes.
Q.13 The tool to recover the cost of providing new services and infrastructure is known as

(A) property tax (B) land tax

(C) development charge (D) professional tax


Ans:- (C)
Hemant Vilas Parulekar 609

Q.14 In an urban electricity network, load factor is defined as


(A) average demand divided by maximum load
(B) average demand divided by minimum load
(C) maximum demand divided by average demand
(D) minimum demand divided by average demand.
Ans:- (A)
Explanation:- Load factor is a measure of the utilization rate of an electrical system, which is defined
as the ratio of the average load to the maximum load over a given period of time. In an urban electricity
network, the load factor is typically calculated as the ratio of the average power demand during a
specified period (such as a day or a month) to the maximum demand during the same period.
Load factor is an important parameter in electrical system design and operation, as it affects the sizing
of equipment, the cost of energy, and the overall efficiency of the system. A higher load factor indicates
that the electrical system is being used more efficiently, while a lower load factor suggests that there is
excess capacity that is not being utilized effectively.
Q.15 The literal meaning of Feng-shui is
(A) house and its houseland (B) gender sensitiveness
(C) efficiency and design (D) wind and water
Ans:- (D)

Explanation:- Feng shui is an ancient Chinese practice that aims to harmonize individuals with their
surrounding environment to bring good fortune, health, and prosperity. The term “feng shui” literally
translates to “wind and water,” which are considered two of the fundamental elements of life energy or
“qi.” The practice involves analyzing the flow of qi through a space and using various principles to en-
hance and balance it, such as the placement of furniture, colors, shapes, and materials.

The practice of feng shui is based on the principles of Taoism and traditional Chinese medicine, which
emphasize the importance of balance and harmony in all aspects of life. It is commonly used in the
design and arrangement of homes, offices, and other spaces to create a positive and supportive envi-
ronment. While feng shui is considered a pseudoscience by some, it remains a popular and influential
practice in many parts of the world.
Q.16 Dhanurmusti and Dhanurgraha are basic modular units used in ancient India. They respectively
represented the twin system of
(A) environment and ecology (B) architecture and town planning
(C) house and temples (D) microcosm and macrocosm
Ans:- (B)

Explanation:- Dhanurmusti and Dhanurgraha were two basic modular units used in ancient India,
particularly in the Vastu Shastra tradition of architecture and town planning. They are related to the
measurement of spaces and buildings. Dhanurmusti was the standard unit of measurement for smaller
spaces such as rooms and courtyards, while Dhanurgraha was the unit of measurement for larger spaces
such as buildings and cities.
610 GATE Architecture and Planning: Comprehensive Question Bank

Q.17 With respect to the contents of a base map, identify the odd one out
(A) physical features (B) topological features
(C) economic features (D) planning and administrative boundaries
Ans:- (C)
Explanation:- A base map typically includes physical features such as landforms, water bodies, vegetation,
and topological features such as roads, railways, and other transportation networks. It may also include
planning and administrative boundaries, such as city limits, political boundaries, and zoning districts,
which define the legal framework for land use and development. However, economic features, such as
businesses, industries, and financial data, are not typically included in a base map. They may be included
in other types of maps or data visualizations that focus specifically on economic analysis.
Q.18 A Psychrometric chart concerns the behavior of
(A) mixture of air and water vapour (B) air and ambient temperature
(C) operative and outdoor temperature (D) people in their communities
Ans:- (B)
Explanation:- Please refer GATE-1996, Q. 28 for more information.
Q.19 The following names are associated with the Bahaus movement
(A) Louis Sullivan, Paulo Soleri (B) Paul Klee, Marcel Breuer
(C) Richard Neutra, Paul Rudolph (D) Jane Drew, Ralph Rapson
Ans:- (B)

Explanation:- The correct answer is (B) Paul Klee, Marcel Breuer. Both Paul Klee and Marcel Breuer
were influential artists and designers associated with the Bauhaus movement, which was a German
art school that operated from 1919 to 1933. The Bauhaus movement had a profound influence on art,
architecture, and design, and is known for its emphasis on simplicity, functionality, and craftsmanship.
Q.20 The sampling technique used to”select from every sub-group of population in at least one stage of
the procedure”is known as
(A) systematic sampling (B) cluster sampling
(C) stratified sampling (D) multi-stage-random sampling
Ans:- (C)
Explanaton:-
1. Systematic Sampling: This method involves selecting every nth element from a list of the population.
For example, if you have a population of 1,000 and you want a sample size of 100, you would select every
10th element from the population list.
2. Cluster Sampling: In this method, the population is divided into groups or clusters, and a sample
is randomly selected from each group. This method is useful when the population is spread out over a
large area and it is difficult to reach all elements of the population.
3. Stratified Sampling: This method involves dividing the population into sub-groups or strata based
on some characteristic, such as age or income level. A sample is then randomly selected from each
stratum, in proportion to the size of the stratum in the population. This method helps to ensure that the
sample is representative of the population as a whole.
4. Multi-stage Random Sampling: This method involves a combination of the above three methods,
where the population is first divided into clusters, then strata, and finally individual elements are selected
randomly from each stratum. This method is useful when the population is large and spread out over a
wide area, and the researcher needs to select a representative sample in a cost-effective manner.
Hemant Vilas Parulekar 611

Q.21 Which one of the following sequences is in the correct ascending order

(A) lithosphere,barysphere,mesosphere,stratosphere

(B) hydrosphere,troposphere,ionosphere,exosphere

(C) barysphere,lithosphere,ionosphere,atmosphere

(D) ionosphere,exosphere,troposphere,barysphere
Ans:- (B)

Explanation:- The lithosphere is the solid outer layer of the Earth, while the barysphere is the central
part of the Earth’s core. The mesosphere is the layer of the atmosphere above the stratosphere, which is
above the troposphere. Therefore, (A) is not in the correct ascending order.
The ionosphere is a layer of the Earth’s atmosphere that is ionized by solar radiation, and the exosphere
is the outermost layer of the Earth’s atmosphere. The troposphere is the lowest layer of the Earth’s atmo-
sphere, where weather occurs. The hydrosphere includes all of the water on Earth’s surface. Therefore,
(B) is in the correct ascending order.
The barysphere and lithosphere were mentioned in (A) and (C), but their positions are swapped. The
ionosphere was mentioned in (C) and (D), but its position is different in each sequence. Therefore, (C)
and (D) are not in the correct ascending order.
Q.22 In a land use map, identify the colour code used for the manufacturing industry

(A) yellow (B) blue (C) purple (D) black


Ans:- (C)
Q.23 The technique that can alter the scale of the building is

(A) façade articulation (B) visual analysis

(C) photogrammetry (D) space programming


Ans:- (A)
Explanation:- Façade articulation refers to the use of architectural elements such as projections, recesses,
and changes in materials and textures to create visual interest and rhythm on the exterior surface of a
building. This technique can alter the perceived scale of the building by breaking down its mass and
creating a sense of depth and shadow.
Visual analysis is a technique used to evaluate and understand the visual aspects of a site or building,
including its context, form, and composition.
Photogrammetry is a method of surveying or mapping an area using photographs or images.
Space programming involves the process of identifying and analyzing the spatial requirements for a
building or facility based on the needs of its occupants or users.
Q.24 Buddhist and Greek elements met each other to form

(A) Pandya style in South India (B) Gandhara style in Bactria

(C) Assyrian phase in Persia (D) Catacombs in Judea


612 GATE Architecture and Planning: Comprehensive Question Bank

Ans:- (B)
Explanation:- Buddhist and Greek elements met each other to form the Gandhara style in Bactria,
which is now modern-day Afghanistan and Pakistan. The Gandhara style emerged during the 1st centu-
ry BCE and flourished until the 7th century CE. It is characterized by the fusion of Greek and Buddhist
artistic traditions, resulting in a unique style that combines Hellenistic and Indian elements. The Gand-
hara style is best known for its representation of Buddha, which blends the classical features of Greek art
with the traditional iconography of Buddhist art.
Q.25 In AutoCAD 2000, which command will allow you to select two-point pairs, move, rotate, and
scale the selected Objects in 2D or 3D to adjust with other objects.
(A) Rotate (B) Qselect (C) Align (D) Skpoly
Ans:- (C)
Q.26 Ideal width to depth ratio of an air conditioning duct is
(A) l:6 (B) 3:1 (C) 1:3 (D) 6:1
Ans:- (B)
Explanation:- As per the National Building Code of India (NBC 2016), the recommended aspect ratio
for rectangular air conditioning ducts is 3:1, which means that the width should be three times the
height or depth. This aspect ratio is recommended for both supply and return air ducts.
The purpose of maintaining the recommended aspect ratio is to ensure that the air flows through the
ducts smoothly and efficiently without causing excessive pressure losses. If the ducts are too narrow
or too shallow, the air may not be able to flow through them effectively, resulting in increased energy
consumption and reduced system performance. On the other hand, if the ducts are too wide or too deep,
they may occupy more space than necessary and increase the cost of the system.
It should be noted that the recommended aspect ratio may vary based on the specific design requirements
of the air conditioning system, such as the volume of air flow, the pressure drop, the velocity of the air,
and the allowable noise level. Therefore, it is important to consult the manufacturer’s specifications and
engineering design guidelines to determine the appropriate aspect ratio for a particular air conditioning
system.
Q.27 Increase in water cement ratio in concrete
(A) decreases strength (B) increases strength
(C) increases durability (D) decreases workability
Ans:- (A)
Explanation:- Please refer GATE-2014, Q.20 for more information.
Q.28 The concept of obtaining maximum utility per unit length of road is called
(A) capacity building (B) facilities information system
(C) saturation system (D) trip distribution
Ans:- (C)
Explanation:- The concept of obtaining maximum utility per unit length of road is called capacity
building. Capacity building refers to the ability of a transportation system to accommodate the demand
for travel by its users. It involves the optimal use of available resources, such as road space, vehicles,
and public transportation services, to meet the travel needs of people and goods in a given area. By
maximizing the capacity of the transportation system, it is possible to improve the efficiency, safety, and
sustainability of the overall transportation network.
Hemant Vilas Parulekar 613

Q.29 The hanging gardens of Babylon were built in

(A) the flood planes of Tigris (B) the banks of river Euphrates

(C) the slopes of the Median hills (D) the coastlines of the Persian Gulf
Ans:- (C)
Q.30 Devolution of fund refers to

(A) income from property tax

(B) specific project grant

(C) mechanism of sharing revenue by the highr level government to the lower-level government.

(D) income from revenue resource


Ans:- (C)

Explanation:- Devolution of funds refers to the mechanism of sharing revenue by the higher level
government (such as the central government) to the lower level government (such as the state or local
government). This allows for the decentralization of power and resources, and enables the lower level
governments to have more control and autonomy over the development of their areas. The process of
devolution of funds is usually done according to a formula that takes into account various factors such
as population, area, and development indicators.
Q.31 Components of Lowry model may be identified as

P transportation modes Q basic employment R residential population

S public utilities T service employment

(A) Q,R,T (B) Q,R,S (C) P,R,T (D) P,Q,R,T


Ans:- (D)

Explanation:- The Lowry model, also known as the sector model, is a theoretical framework for urban
land use developed by economist John R. Lowry in the 1940s. The model proposes that a city grows
outward in concentric rings, with each ring representing a particular type of land use. The center of
the city is the central business district (CBD), which is characterized by high-density commercial and
office buildings. Surrounding the CBD is a ring of transitional land use, such as light manufacturing,
wholesale trade, and warehousing. This ring is known as the inner city.
The next ring is the outer city, which is predominantly residential and characterized by single-family
homes, apartments, and some commercial activity. The outer city is typically divided into high-income
and low-income areas, with the higher-income areas located further from the CBD. Beyond the outer city
is the suburban fringe, which is characterized by low-density residential and commercial development,
such as shopping malls and large retail centers.
The Lowry model was influential in the field of urban planning and helped to shape zoning regulations
and land use policies in many cities in the United States. However, it has been criticized for oversimplifying
the complex processes of urban growth and for not taking into account factors such as transportation and
urban infrastructure. Despite these criticisms, the Lowry model remains a useful tool for understanding
urban land use patterns and their evolution over time.
614 GATE Architecture and Planning: Comprehensive Question Bank

Q.32 Assume that a camera with a 152 mm focal length lens was flown over a flat terrain located at 1200
mm above mean sea level. If the scale of the aerial photography is l:50,000 calculate the flying height
above the terrain
(A) 6400m (B) 7600 m (C) 8800m (D) 4166m
Ans:- (B)
Solution:- Scale of photograph = 1:50,000
Height of terrain a.m.s.l. (h) = 1200 mm = 1.20m
Flying Height of the plane a.m.s.l. (H) = ?, And focal length of camera lens (f) = 152 mm = 0.152m
Scale of photograph (S) = f/(H-h)
Thus, S = 0.152/(H-1.2)
1/50,000 x (H-1.2) = 0.152
(H-1.2) = 0.152 x 50,000
(H-1.2) = 7600
H = 7601.20m
Height above terrain = 7600m
Q.33 A town of 2,25,000 population in 1981 has exhibited decadal growth rates of 25%, and 30% during
1991 and 2001 respectivelv. Estimate the population in 2011 having a 40% decadal growth rate
(A) 3,50,000 (B) 6,14,250 (C) 5,11,875 (D) 4,50,000
Ans:- (C)

Solution:- To estimate the population in 2011, we need to apply the decadal growth rates to the initial
population in 1981. Let’s break it down step by step.

Initial Population (1981): 225,000

1st Decade (1991) - 25% growth rate: Population Growth = 225,000 * 0.25 = 56,250 Population in 1991
= 225,000 + 56,250 = 281,250

2nd Decade (2001) - 30% growth rate: Population Growth = 281,250 * 0.30 = 84,375 Population in
2001 = 281,250 + 84,375 = 365,625

3rd Decade (2011) - 40% growth rate: Population Growth = 365,625 * 0.40 = 146,250 Population in
2011 = 365,625 + 146,250 = 511,875
Q.34 Match the following
(A) P-3,Q-4,R-5,S-6 Group I Group II
(B) P-1,Q-6,R-4,S_3 P. hydrophytes 1. semi-arid
Q. xerophytes 2. live on woody vegetation
(C) P-5,Q-1,R-3,S-2
R. epiphytes 3. in or near water
(D) P-3,Q-4,R-2,S-6 S. Halophytes 4. Desert
5. Polar regions
6. Salt marshes
Hemant Vilas Parulekar 615

Ans:- (D)

Term Hydrophytes Xerophytes


Meaning Hydrophytes are plants that are adapted Xerophytes are plants that are adapted to
to grow in water or in soil that is grow in arid or desert conditions. They have
periodically saturated with water. These several adaptations such as thick cuticles on
plants have several adaptations such as leaves to reduce water loss, small leaves to
hollow stems, reduced roots, and large minimize surface area, and deep roots to
leaves that help them float on the water’s access groundwater. Examples of xerophytes
surface. Examples of hydrophytes include cacti, succulents, and desert shrubs.
include water lilies, lotus, and duckweed.
Plant

Term Epiphytes Halophytes


Meaning Epiphytes are plants that grow on the Halophytes are plants that are adapted to
surface of other plants, such as trees, grow in soils that are high in salt content,
and derive their nutrients and water such as in salt marshes or along coastal
from the air and rain. These plants regions. These plants have adaptations such
have adaptations such as aerial roots as thick leaves and stems to store water and
and modified leaves that help them salt-excreting glands to remove excess salt.
absorb moisture from the atmosphere. Examples of halophytes include mangroves,
Examples of epiphytes include orchids, saltbushes, and seagrasses.
ferns, and mosses.
Plant
616 GATE Architecture and Planning: Comprehensive Question Bank

Q.35 Hippodamus and Vitruvius pioneered in important areas of built environment. They are famous for
(A) the planning of Priene and ‘De Architecture’
(B) the planning of Memphis and’De Mensura’
(C) the planning of Constantinople and’Treatise of Vesta’
(D) the planning of Alexandria and ‘The ionic order’
Ans:- (A)

Explanation:- Hippodamus is known for his urban planning contributions, including the grid-like lay-
out of the ancient city of Priene. Vitruvius, a Roman architect and engineer, is famous for his treatise on
architecture, ‘De Architectura’ (also known as ‘Ten Books on Architecture’), which covers a wide range
of topics related to architecture, engineering, and urban planning.
Q.36 A group housing plot of 400 sqm, abutting a 20 m road is permitted to have 35% ground coverage
and maximum FAR of 1.25. Assuming 70 sqm of super built up area, calculate the maximum num-
ber of units that can be accommodated
(A) 5 (B) 7 (C) 9 (D) 12
Ans:- (A)
Explanation:- First, let’s calculate the maximum permissible ground coverage area: Ground coverage
area = 35% of 400 sqm = 0.35 x 400 = 140 sqm
Next, let’s calculate the maximum permissible total built-up area: Total permissible built-up area = FAR
x plot area Total permissible built-up area = 1.25 x 400 sqm = 500 sqm
Now, let’s calculate the maximum permissible built-up area for each unit: Built-up area per unit = Total
permissible built-up area / number of units Built-up area per unit = 500 sqm / n
We know that the super built-up area per unit is 70 sqm. Assuming that the efficiency factor is 75%, the
carpet area per unit would be: Carpet area per unit = Super built-up area / efficiency factor Carpet area
per unit = 70 sqm / 0.75 = 93.33 sqm
Adding the common area (staircase, lift, etc.), let’s assume that the total area per unit would be around
110 sqm.
Now we can calculate the maximum number of units as follows: n x 110 sqm = 500 sqm n = 4.55
(approx.)
Therefore, the maximum number of units that can be accommodated is 4 or 5 (rounded up) units.
Q.37 Match the following
(A) P-5,Q-6R-4,S-2 Group II Group II
(B) P-6,Q-5,R-7,S-2 P. UCSICON 1. Creates a surface that is not filled in
Q. UCSFOLLOW 2. Creates a surface that is filled in
(C) P-3,Q-6,R-4,S-1
R. TILEMODE 3. Controls the visibility and placement of UCS
(D) P-3,Q-6,R-7,S-2 S. 3DFACE 4. When set to zero make paper space active
5. Sets the orientation of UCS in 3-D space
6. Displays new UCS in plan view whenever the
UCS is changed
7. Allows multiple copies across the screen.
Hemant Vilas Parulekar 617

Ans:- (D)
Q.38 As speed of the vehicle increases, the driver experiences one or more of the following
P. planes perpendicular to road become more prominent
Q. distance of focusing point decreases
R. peripheral vision increases
S. foreground details begin to fade
T. importance of the roadside treatment decreases
(A) P,Q,S,T (B) S,T (C) P,Q,R,S,T (D) P,S,T
Ans:- (D)
Explanation:- P. The planes perpendicular to the road become more prominent, making it difficult to
judge distances and heights of objects.
Q. The distance of the focusing point decreases, which means the driver needs to focus more frequently
to keep a clear view of the road ahead.
R. Peripheral vision does not necessarily increase with speed, but the driver may need to use their
peripheral vision more effectively to scan the roadway for potential hazards.
S. Foreground details begin to fade, and the driver’s vision is focused more on the distant scene.
T. The importance of the roadside treatment increases because the driver needs to be able to quickly and
easily identify signs, signals, and other important information along the roadside.
Q.39 The ‘caryatid’ mode of columnar construction is a composition of
(A) linear columns and beam forms of a three c limensional Irame work in Japanese palaces
(B) sculptured female figure as columnar supports for entablature in Greel temples
(C) vertical linear elements defining volume or transparent spaces in ancient Turkish domes
(D) basic relationships between structural elements supporting roof and the roofing materials in
Gothic churches.
Ans:- In Greek architecture, a caryatid is a sculpted female
figure serving as an architectural support, taking the place of a
column or a pillar. The caryatid form was used particularly in
the Ionic order of Ancient Greek architecture, but also in other
architectural styles. The term caryatid is derived from the
Greek word karyatides, meaning “maidens of Karyai”, an
ancient town in the Peloponnese.
Caryatids are typically depicted wearing flowing drapery and
carrying a basket or urn on their heads. They are often used to support the entablature of a building, and
can be found in various structures, including temples, palaces, and public buildings.
Q.40 The contemporary style that turns down nature and becomes dependent on huge energy supply is
referred as
(A) Aerodynamic and Ergonomic Technology-City gate, Dusseldorf
(B) Local climatological conditions-New Caledonia Cultural Centre, UK
(C) Glass Box and automated systems-City Corp Centre, New York City
(D) Solar Active controls-Sophia University (Tokyo)
618 GATE Architecture and Planning: Comprehensive Question Bank

Ans:- (C)

Explanation:- The contemporary style that turns down nature and becomes dependent on huge energy
supply is referred to as Glass Box and automated systems. The CityCorp Centre in New York City is an
example of this style. The use of glass and automated systems results in buildings that are heavily reliant
on energy for lighting, temperature control, and other functions, often at the expense of energy efficien-
cy and sustainability.

Name of the Structure City gate, Dusseldorf New Caledonia Cultural Centre, UK
Structure

Name of the Structure CityCorp Centre, New York City Sophia University (Tokyo)
Structure

Q.41 A 25-storied office building is served by a group of 4 lifts of 25 capacity each. Calculate the probable
number of stops

(A) 14 (B) 10 (C) 25 (D) 11


Ans:- (A)
Explanation:- Given the formula x = S - S[(S-1)/S]n, where x is the probable number of stops, S is the
maximum number of stops (number of floors in the building), and n is the carrying capacity (80% of
the maximum carrying capacity).
For a 25-storied office building with 4 lifts of 25 capacity each, we will consider 80% of the maximum
carrying capacity, which is 0.8 * 25 = 20.
Now, we can plug in the values:
S = 25 (number of floors) n = 20 (80% of the maximum carrying capacity)
x = 25 - 25[(25-1)/25]20 x = 25 - 25[(24/25)20]
To calculate the expression inside the brackets:
(24/25)20 ≈ 0.3585
Now, substitute this value back into the equation:
x = 25 - 25 * 0.3585 x ≈ 25 - 8.9625 x ≈ 16.0375
Therefore the closest answer is (A) 14
Hemant Vilas Parulekar 619

Q.42 Find the value of slenderness ratio of a wall of thickness 0.19 meter with the following considerations
(i)Effective height=2.74m
(ii)Effective length=4.32 m
(iii)Stiffness co-efficient=1
(iv)Effective thickness=Actual thickness
(A) 22.7 (B) 10.8 (C) 14.4 (D) 18.6
Ans:- (C)
Solution:- The slenderness ratio is calculated as the effective height or effective length divided by
the effective thickness. Since the effective thickness is equal to the actual thickness and the stiffness
coefficient is 1, we can directly calculate the slenderness ratio using the given values.
(i) Effective height (h) = 2.74 m (ii) Effective length (l) = 4.32 m (iii) Stiffness coefficient = 1 (not needed
for the calculation) (iv) Effective thickness (t) = Actual thickness = 0.19 m
We need to find the maximum slenderness ratio, which means we need to use the largest dimension
(either the effective height or effective length) divided by the effective thickness.
Slenderness ratio = max(h, l) / t
Slenderness ratio = l / t = 4.32 m / 0.19 m ≈ 22.7
Slenderness ratio = h / t = 2.74 m / 0.19 m ≈ 14.4
The smaller applicable value is (C) 14.4
Q.43 Match the following
(A) P-1,Q-4,R-5,S-6 Group I (Vehicle type) Group 2 (PCU)
(B) P-2,Q-6,R-3,S-5 P. Car 1) 1.5
Q. 2-wheeler automobiles 2) 1.0
(C) P-5,Q-2,R-4,S-6
R. Bus 3) 4.0
(D) P-2,Q-4,R-5,S-3 S. Agricultural tractor trailer 4) 0.5
5) 2.2
6) 0.2
Ans:- (D)
Explanation:-
P. Car: Cars are usually considered the standard vehicle and are assigned a PCU of 1.0 (Group 2 - Option
2).
• Q. 2-wheeler automobiles: Two-wheelers such as motorcycles and scooters are generally smaller
and less disruptive to traffic, so they have a lower PCU value. A PCU of 0.5 (Group 2 - Option 4)
is a reasonable estimate.
• R. Bus: Buses are larger and can carry more passengers than a car, so they have a higher PCU
value. A PCU of 2.2 (Group 2 - Option 5) is a common estimate for buses.
• S. Agricultural tractor-trailer: These vehicles are large and slow-moving, which can disrupt traffic
flow. A PCU of 4.0 (Group 2 - Option 3) is a reasonable estimate for a tractor-trailer.
620 GATE Architecture and Planning: Comprehensive Question Bank

Q.44 Choose the correct sequence of waste management


P treatment
Q disposal
R source reduction
S recycling
(A) P,R,Q,S (B) Q,S,R,P (C) S,P,R,Q (D) R,S,P,Q
Ans:- (D)

Explanation:-

1. R - Source reduction: The first step in waste management is to minimize the generation of waste
at its source.

2. S - Recycling: The next step is to recycle or reuse materials, which helps to reduce the need for
new resources and decreases the amount of waste going to landfills.

3. P - Treatment: Waste that cannot be recycled or reused should be treated to minimize its harm-
ful effects on the environment and human health.

4. Q - Disposal: Finally, any remaining waste should be disposed of in an environmentally friendly


manner, such as by using sanitary landfills or incineration with proper pollution control mea-
sures.
Q.45 Choose the correct sequence of the evolution of U.K. planning laws

(A) Uthwatt Commission, Scott Commission, PAG Report, Barlow Commission

(B) Barlow Commission, Uthwatt Commission, Scott Commission, PAG Report

(C) Barlow Commission, PAG Report, Scott Commission, Uthwatt Commission

(D) PAG Report, Scott Commission, Uthwatt Commission, Barlow Commission


Ans:- (B)

Explanation:- The correct sequence of the evolution of U.K. planning laws is as follows:
1. Barlow Commission (1940): The Barlow Commission was established to examine the distribu-
tion of industrial population and the location of industry.

2. Uthwatt Commission (1941): The Uthwatt Commission focused on the issue of land use and
compensation, recommending the nationalization of development rights.

3. Scott Commission (1942): The Scott Commission was formed to examine rural land use and
proposed the creation of Green Belts around urban areas.

4. PAG Report (Planning Advisory Group) (1965): The PAG Report was a response to the Town
and Country Planning Act of 1947 and provided guidance on planning principles and practices
in the U.K.
Hemant Vilas Parulekar 621

Q.46 Participatory plan development having scope like detailed planning, coordinated detailed design,
legal services, coordinating tender process, coordinating construction, mobilization of funds, mort-
gage brokerage, operation and maintenance, sales and marketing, is known as

(A) build operate and transfer (B) development initiatives

(C) leasing contract (D) direct management services contract


Ans:- (D)

Explanation:- A direct management services contract covers a wide range of services involved in the
development and management of a project. These services include detailed planning, coordinated de-
tailed design, legal services, coordinating tender processes, coordinating construction, mobilization of
funds, mortgage brokerage, operation and maintenance, sales, and marketing. It’s an all-encompassing
approach to project management that aims to streamline the process and ensure the efficient delivery of
the project.
Q.47 The period between 1870 and 1940 was the era of metropolitan development in the USA. Identify
the factor that was not responsible for metropolitan development

(A)The great migration from Europe

(B)Technological advances in highway and high-rise engineering

(C)The comparative advantages of World Wars and Marshall Plan

(D)The process of Urban decentralization.


Ans:- (C)

Explanation:- The metropolitan development during this period was mainly influenced by factors such
as the great migration from Europe, technological advances in highway and high-rise engineering, and
the process of urban decentralization. The comparative advantages of World Wars and Marshall Plan, on
the other hand, were more relevant to the post-World War II period in the mid-20th century.
Q.48 The Bariloche model defines the key development factor as the Basic Needs to ensure growth.
Choose the correct combination that applies to

(A) (B) (C) (D)


Death Rate Birth Rate Food Birth Rate
Literacy Rate Calorie Intake Shelter School Enrolment
Employment Rate School Enrolment Education Health Care
Income Rate Urbanization Urbanization Clothing
Ans:- (C)
Explanation:- The Bariloche model is a development theory that emphasizes the importance of
meeting the basic needs of a population in order to achieve sustainable economic growth. The model
was developed by the Argentine economist Raúl Prebisch in the 1950s and is named after the city of
Bariloche, where Prebisch presented his ideas.
According to the Bariloche model, the most important factor in development is the satisfaction of basic
needs, such as food, shelter, clothing, education, and health care. By providing these basic needs to the
population, development can occur in a way that is sustainable and equitable.
622 GATE Architecture and Planning: Comprehensive Question Bank

In addition to meeting basic needs, the Bariloche model also emphasizes the need for developing coun-
tries to have control over their own resources and economies. This means reducing dependence on
foreign investment and creating policies that promote domestic development.

The Bariloche model is often seen as a response to the dominant development theories of the time, such
as modernization theory, which emphasized the need for developing countries to emulate the economic
and social structures of developed countries. The Bariloche model, on the other hand, stresses the im-
portance of meeting the unique needs and challenges of each country in order to achieve sustainable
development.

Overall, the Bariloche model continues to have relevance today in discussions of economic development
and the importance of meeting basic needs as a foundation for sustainable growth.
Q.49 Identify the criteria that are best suited for building design in hot and dry climate
P) high building density Q) plantation parallel to air movement
R) small openings S) thick walls
T) large openings U) Plantation perpendicular to air movement
(A) P,Q,R,S (B) Q,S,T (C) P,R,S,U (D) P,T,U
Ans:- (C)
Q.50 Which of the following are reasons to use mulch on the topsoil of newly planted shrubs?
P. Aid in water retention
Q. Prevent soil temperature fluctuation
R. Discourage weed growth
S. Improve landscape appearance
T. Protect from insects
(A) P,Q (B) P,R (C) P,R,T (D) P,Q,R,S
Ans:- (D)
Explanation:-
Mulch is a layer of organic or inorganic material applied to the topsoil
surface around plants or over a garden bed. It is primarily used to
conserve moisture in the soil, suppress weed growth, regulate soil
temperature, and enhance the appearance of a garden or landscape.
Organic mulches include materials such as bark chips, straw, grass
clippings, and compost, while inorganic mulches include gravel, stones,
and landscape fabric. Organic mulches are preferred because they
break down over time, adding nutrients to the soil, while inorganic mulches may be used in areas where
aesthetics are a priority, or where organic mulch may not be practical, such as in high-traffic areas.
Mulch should be applied in a layer that is several inches deep, but not so thick as to prevent water from
penetrating the soil. It should be kept away from the stems of plants to prevent rot, and replenished as
needed to maintain an adequate layer.
In addition to its practical benefits, mulch can also be used to add color and texture to a landscape, and
can be an attractive and low-maintenance option for garden beds and plantings.
Hemant Vilas Parulekar 623

Q.51 Match the following


(A) P-4,Q-5,R-2,R-6 Group I Group II
(B) P-3,Q-5,R-1,S-2 P. Digital governance 1) visual simulation of environment
Q. Genetic algorithm 2) image enhancement, manipulation, management
(C) P-4,Q-5,R-1,S-2
R. Virtual reality 3) e-governance
(D) P-2,Q-4,R-5,S-3 S. Digital image 4)access to information on internet
processing
5) evolutionary visual simulation
6) image interpretation
Ans:- (B)
Q.52 Calculate the number of light fixtures required in an office room of 8m x 7m requiring an illumina-
tion level of 400 lux on the work plane. Each light fixture has a rated output of 7350 lumens. Assume
a utilization factor of 0.5 and a maintenance factor of 0.8

(A) 8 (B) 7 (C) 10 (D) 12


Ans:- (A)
Solution:- The illuminance required on the work plane is 400 lux. The total area of the room is 8m x 7m
= 56 sq.m. Therefore, the total lumens required can be calculated as:
Total lumens = illuminance x area = 400 lux x 56 sq.m = 22400 lumens
However, we need to take into account the utilization factor and maintenance factor. The utilization
factor takes into account the efficiency of the lighting system and any losses due to reflection, while the
maintenance factor takes into account the reduction in light output due to dirt, aging, and lamp lumen
depreciation.
Assuming a utilization factor of 0.5 and a maintenance factor of 0.8, the total lumens required can be
calculated as:
Total lumens required = Total lumens / (utilization factor x maintenance factor) = 22400 lumens / (0.5
x 0.8) = 56000 lumens
Each light fixture has a rated output of 7350 lumens. Therefore, the number of light fixtures required
can be calculated as:
Number of light fixtures = Total lumens required / rated output per fixture = 56000 lumens / 7350
lumens per fixture = 7.62
Since we can’t have fractional fixtures, we need to round up the answer to the nearest whole number.
Therefore, the number of light fixtures required is 8 (option A)
Q.53 The concept of “Pattern language” deals with one or more of the following
P. description of a problem that occurs in our environment and the core of solution to that problem
Q. communication and public participation using sign language
R. recognition of styles and patterns of various languages
S. devising patterns to create a building or a town
(A) P,Q,S (B) Q (C) Q,R (D) P,S
624 GATE Architecture and Planning: Comprehensive Question Bank

Ans:- (D)

Explanation:- Please refer GATE-2009, Q. 13 for more information.


Q.54 In Indian system of Anthropometrics, two known and one unknown data are given, i.e., 8 angulas =
1 pada, 3 padas = 1 hasta and 1 danda = x padas. Given this, find out the perimeter of a square site
in angulas and dandas if its sides are measuring 2 hastas each
(A) 164 angulas and 4 dandas (B) 192 angulas and 2 dandas
(C) 126 angulas and 3 dandas (D) 108 angulas and 6 dandas
Ans:- (B) (closest answer)
Solution:- The given information can be summarized as:
1 pada = 8 angulas 1 hasta = 3 padas 1 danda = x padas
We need to find the perimeter of a square site in angulas and dandas if its sides are measuring 2 hastas
each.
The length of each side of the square site can be calculated as:
Length of 1 hasta = 3 padas Length of 2 hastas = 2 x 3 padas = 6 padas
To convert padas to angulas, we have:
1 pada = 8 angulas Therefore, 6 padas = 6 x 8 = 48 angulas
So, the length of each side of the square site in angulas is 48.
The perimeter of the square site is given by:
Perimeter = 4 x Length of each side = 4 x 48 = 192 angulas
To convert angulas to dandas, we have:
1 danda = x padas 1 pada = 8 angulas Therefore, 1 danda = x x 8 angulas or x = 1/(8 angulas) danda/
angula
Substituting the value of x in the perimeter calculated above, we get:
Perimeter = 192 angulas x (1/(8 angulas) danda/angula) = 24 dandas
Therefore, the perimeter of the square site in angulas and dandas is 192 angulas and 24 dandas,
respectively.
Hence, the closest correct option is (B) 192 angulas and 2 dandas.
Q.55 The contemporary architectural theory which undermines conventional sections of harmony, unity
and stability, breaks continuity and disturbs relationship between interior and exterior, is known as
(A) Brutalism (B) Iconography (C) Transformation (D) Deconstruction
Ans:- (D)

Explanation:- Deconstruction is an architectural philosophy that emerged in the late 1980s and early
1990s as a reaction against the modernist principles of the International Style. It is characterized by the
use of fragmentation, distortion, and other techniques to challenge traditional notions of order and co-
herence in architecture. The result is often a building that appears chaotic and disorganized, but which
also invites the viewer to question their assumptions about space, form, and meaning.
Hemant Vilas Parulekar 625

Q.56 Identify the attributes that best apply to Ring type distribution of electric supply

P. Only one cable is laid from substation into each building

Q. Only one fused switch is required in distribution board in each building to isolate the system

R. Current flows in both directions from intake room

S. repairs to any point disrupts supply to other buildings

T. provides better and balanced supply than radial system

U. repairing any cable fault at any point does not lead to


loss of supply to any building

(A) R,S,T,U (B) P,Q,S,T (C) P,R,T,U (D) Q,S,T,U


Ans:- (C)
Q.57 Match the defects in painting with their description

Group I (Description) Group II (Defect)


P. Dislocation or loosening of some portion of painted surface 1.Flashing
Q. Formation of bubbles under the film of paint 2.Fading
R. Formation of glossy patch on the painted surface 3.Blistering
S. Gradual loss of colour of paint due to effect of sunlight 4. Flaking
5. Grinning
6. Crawling

(A) (B) (C) (D)


P-4 P-6 P-4 P-6
Q-3 Q-4 Q-3 Q-3
R-2 R-2 R-1 R-4
S-1 S-1 S-2 S-5
Ans:- (C)

Explanation:-

Defect Flashing Fading


Description This defect occurs when there is a Fading refers to the gradual loss of color
difference in glossiness or sheen of the of the paint due to exposure to sunlight
paint in different areas. This may happen or other environmental factors. This is
when the paint is not applied evenly or often seen in outdoor paint applications.
when different paint products are used.
Image
626 GATE Architecture and Planning: Comprehensive Question Bank

Defect Blistering Flaking (Cracking)


Description Blistering happens when bubbles or Flaking occurs when the paint begins
blisters form on the painted surface. This to peel or flake away from the surface it
can be caused by moisture getting trapped was applied to. This can happen when
under the paint, inadequate surface the surface was not properly prepared, or
preparation, or the use of incompatible when the paint was applied too thickly or
paint products. in multiple layers.
Image

Defect Grinning Crawling


Description Grinning is the appearance of the base Crawling is the appearance of irregular
material (e.g. concrete or wood) through patterns or bare spots in the painted
the paint film. This can happen if the surface. This is often caused by the
paint was applied too thinly, or if the surface being contaminated with oil
base material has a higher coefficient of or other materials, or when the paint is
expansion than the paint. applied too thickly or too quickly.
Image

Q.58 Which is the correct match for the elements (Group I) with behavior (Group II)?

Group I (Element) Group II (Behaviour)


P Beam/Slab 1. Flexure
Q. Arch 2. Compression
R. Cable 3. Meridional Stress
S. Shell 4. Tension
5. Indeterminacy
(A) P-1,Q-2,R-3,S-4 (B)P-1,Q-2,R-4,S-3 (C) P-5,Q-1,R-3,S-4 (D) P-2,Q-3,R-4,S-5
Ans:- (A)
Explanation:-
• Beam/Slab: It primarily undergoes flexure, which is the behavior of bending under the action
of loads. Hence, it is matched with behavior 1.
• Arch: It primarily undergoes compression, which is the behavior of shortening under the
action of loads. Hence, it is matched with behavior 2.
Hemant Vilas Parulekar 627

• Cable: It primarily undergoes meridional stress, which is the behavior of experiencing force
in the direction of its length. Hence, it is matched with behavior 3.
• Shell: It primarily undergoes tension, which is the behavior of stretching under the action of
loads. Hence, it is matched with behavior 4.
• Indeterminacy is not a behavior, but a property that refers to the inability to determine the
forces and moments in the structural elements by statics alone. It is not matched with any
element.
Q.59 What is the theoretical capacity of a traffic lane with one-way traffic flow at a speed of 60kmph? As-
sume that the average centre to centre spacing of vehicles at this speed is 16.68 m

(A) 1000 (B) 3597 (C) 500 (D) 4000


Ans: - (B)

Solution:- Length of the traffic lane = 1 km = 1000m

At a speed pf 60 kmph, the average centre to centre = 16.68m

Means, 1000/16.68 = 59.97 or say 60 vehicles can run

Hence, the total capacity = 59.95 x 60 = 3597


Q.60 Choose the attributes that apply to social infrastructures

(A) schools, parks, club, water supply

(B) parks, library, club, water supply

(C) hospitals, sanitation, water supply, electric supply

(D) schools, hospitals, parks, library, club


Ans:- (D)
Q.61 Choose the right Kind of statutory provision to enforce integrated urban Conservation

(A) Conservation of Ancient Monuments and Sites Act

(B) Urban Arts Commission Act

(C) Conservation of Energy Act

(D) Conservation of Natural Act


Ans:- (B)

Explanation:- This act was enacted in 1972 to promote the aesthetic quality of urban life and to protect
and preserve buildings, structures and open spaces that have aesthetic, historic or cultural significance.
The Urban Arts Commission Act provides for the establishment of Urban Arts Commissions at the na-
tional, state and local levels to advise and assist the government in matters relating to the preservation
and development of urban aesthetics.
Q.62 A community comprising 120 hectares is having 60% of the land put to residential plots.and a pop-
ulation of 30,600. Calculate the net residential density

(A) 375 pph (B) 400 pph (C) 425 pph (D) 500 pph
628 GATE Architecture and Planning: Comprehensive Question Bank

Ans:- (C)

Solution:- First, we need to find out the total area used for residential plots: Total area = 120 hectares
Area for residential plots = 60% of total area = 0.6 * 120 hectares = 72 hectares

Next, we need to find out the residential density per hectare: Population = 30,600 Area for residential
plots = 72 hectares Residential density per hectare = Population / Area for residential plots Residential
density per hectare = 30,600 / 72 hectares Residential density per hectare ≈ 425 pph

Therefore, the net residential density is approximately 425 pph,


Q.63 Poor academic performances of school children were attributed to missing breakfast and subse-
quent malnutrition. The investigating survey schedule every student, how many days the students
have missed the breakfast? Their response referred as
(A) nominal scale (B) ordinal scale (C) longitudinal scale (D) interval scale
Ans:- (C)
Explanation:-
1. Nominal Scale:
-Description: The nominal scale is the most basic level of measurement. It categorizes data without any
quantitative value. It’s used for labeling variables, without any quantitative value.
-Examples: Gender (male, female), Blood type (A, B, AB, O), Nationality.
Characteristics: No order or ranking; only used for categorization.
2. Ordinal Scale:
Description: This scale provides a rank order or sequence for the data but doesn’t quantify the difference
between each category.
Examples: Satisfaction rating (satisfied, neutral, dissatisfied), Education level (high school, bachelor’s,
master’s, doctorate).
Characteristics: Shows relative position, but intervals between ranks are not necessarily equal.
3. Interval Scale:
Description: Interval scales not only provide order but also quantify and compare the size of differences
between items. However, they do not have a true zero point.
Examples: Temperature (Celsius, Fahrenheit), IQ scores.
Characteristics: Equal intervals between points, but zero is arbitrary (does not indicate the absence of
the quantity).
4. Ratio Scale:
Description: The ratio scale has all the characteristics of an interval scale, with the additional feature of
an absolute zero point. This zero point means the absence of the variable being measured.
Examples: Height, Weight, Age, Income, Length.
Characteristics: True zero point, allows for comparison of absolute magnitudes.
Hemant Vilas Parulekar 629

Importance in Research:
Appropriate Analysis: Choosing the correct scale of measurement influences the type of statistical anal-
ysis that can be used and the conclusions that can be drawn.
Data Interpretation: Understanding the scale helps in accurately interpreting the data.
Research Design: Helps in designing questionnaires and experiments.
Q.64 A site map shows five contour lines. A line drawn across the lowest and the highest contour mea-
sures 125 m and has a slope of 8 %. Determine the contour interval

(A) 5m (B) 2.5m (C) 10m (D) 2m


Ans:- (B)

Solution:- Slope of the site is 8%, i.e., for every 100m horizontal distance, there is 8m rise.

Total rise = (8/100) x 125m = 10m

Number of intervals = 4

Contour interval = Total rise / Number of intervals = 10m / 4 = 2.5m


Q.65 Identify the components of Geographical Information System

P. Computer system and software Q. Spatial data

R. Uniform energy source S. A super sensor

T. Real time data handling system U. Data management and analytical tools

(A) P,Q,R,S,T,U (B) P,Q,S,U (C) P,Q,U (D) P,S,T,U


Ans:- (B)

Explanation:- Geographical Information System (GIS) is a system designed to capture, store, manipu-
late, analyze, manage and present all types of geographical data. The main components of GIS are:
• Computer system and software (P)
• Spatial data (Q)
• A data management and analytical tools (U)
• Uniform energy source is not a component of GIS (R)
• A super sensor is not a component of GIS (S)
• Real time data handling system may be a component of GIS, but it is not one of the main
components listed here (T)
Q.66 In energy technology, efficiency is equal to energy output by energy input. If the efficiency level of a
particular fossil fuel is 90% and the mathematical efficiency of the vehicle is 25 %, then how much is
the total efficiency of the vehicle?

(A) 62.5% (B) 22.5% (C) 33.33% (D) 82.5%


630 GATE Architecture and Planning: Comprehensive Question Bank

Ans:- (B)
Solution:- To calculate the total efficiency of the vehicle, we need to multiply the efficiency of the fuel
and the efficiency of the vehicle together.
If the efficiency of the fuel is 90%, then the energy output is 90% of the energy input, or 0.9.
If the mathematical efficiency of the vehicle is 25%, then the energy output is 25% of the energy input,
or 0.25.
To calculate the total efficiency of the vehicle, we multiply these two values together:
Total efficiency = Fuel efficiency x Vehicle efficiency Total efficiency = 0.9 x 0.25 Total efficiency = 0.225
or 22.5%
Q.67 Identify the missing prime factor for design investigation based on ergonomics of the figure

(A) Shape (B) Functions (C) Size (D) Aesthetics


Ans:- (D)
Q.68 The Bahai temple of New Delhi has a lotus plan. The lotus has x number of petals, y number of equal
angles and z type of architectural tradition. Which of the following is correct?

Type x y Z
(A) 12 30 degrees Egyptian Mastaba
(B) 9 40 degrees Persian Spherical
(C) 10 36 degrees Santa Sophia, Turkey
(D) 8 45 degrees Ideal city of Vincenzo Scamozzi

Ans:- Cannot be decided, as data is insufficient

Explanation:- The Lotus Temple in New Delhi, also known as the Bahai House of Worship, has a dis-
tinct lotus shape with 27 free-standing marble-clad “petals” arranged in groups of three to form nine
sides. Each of these sides is a 60-degree angle. The lotus sits on a platform that is also in the shape of a
lotus. The number of petals, angles, and architectural tradition mentioned in the question do not match
those of the Lotus Temple.
Q.69 Michael Wilford, Peter Salter and Richard Rogers are associated with

(A)Potsdamer Plaza (Berlin), Kyoto Centre (Japan), Music School and Theatre Academy (Stuttgart)

(B)Marlenbone Gate (UK), Islamic Arts Centre (London), Stone Henge Visitors Club (UK)

(C)Reich Stag (Berlin), Camden Arts Center (France), Tete Gallery (Cornwall)

(D)Fountain Abbey Visitors Centre (Yorkshire), Flat Conversion (London), More Cambe (UK)
Hemant Vilas Parulekar 631

Ans:- (A)
Explanation:- Michael Wilford, Peter Salter, and Richard Rogers are architects known for their
contributions to modern architecture. They collaborated on several projects, including:
• Potsdamer Platz in Berlin, Germany, which was a major urban redevelopment project in the
heart of the city
• Kyoto Centre in Japan, which is a cultural center that includes a library, exhibition spaces,
and a theater
• Music School and Theatre Academy in Stuttgart, Germany, which is a state-of-the-art
performing arts facility
Q.70 Match the fittings shown in Group I with the sketches shown in Group II

Group I

(A) (B) (C) (D)


P-5 P-4 P-8 P-3
Q-4 Q-2 Q-6 Q-1
R-1 R-7 R-5 R-2
S-7 S-5 S-1 S-7

Group II

P. Single socketed pipe Q. Double equal junction R. Pipe-shoe S. Reducing piece


Ans:- (A)
Q.71 Which is the correct match for iitems given in Group II with items given in Group II?

Group I Group II
P. Plastering 1. Volume
Q. Earthwork 2. PERT
R. Reinforcement 3. Schedule of Bending
S. Management 4. Area
5. Temperature

(A) P-4,Q-1,R-3,S-2 (B) P-2,Q-1,R-3,S-4 (C) P-3,Q-5,S-2,R-4 (D) P-2,Q-3,R-4,S-5


Ans:- (A)
632 GATE Architecture and Planning: Comprehensive Question Bank

Q.72 Match the attributes with the corresponding shapes of central island in a rotary shown in the sketch-
es below

P. Reduction of vehicular speed on entering rotary and enables speeding of going out vehicle

Q. Enables excessive speeding of vehicles

R. Where two equally important roads cross

S. Accomodates four or more intersecting roads and allows greater traffic flows along direction of elon-
gation

(A) P-4,O-1.R-3,S-2 (B) P-2,Q-1,R-3,S-4 (C) P-3, Q-1,R-2,S-4 (D) P-2,Q-3,R-4,S-1


Ans:- (A)
Q.73 Identify the attributes that apply to sanitary landfill

P waste is deposited in 0.90-4.50 m thick layers

Q low lying marshy waste land are transformed into useful areas

R attract flies, insects and rodents


S causes health and pollution hazards

T chances of fire hazards in wet weather

U skilled personnel are required

V semi-skilled personnel are required

(A) R,S,T,U (B) P,Q,T,V (C) P,Q,S,T (D) Q,R,S,T


Ans:- (C)
Explanation:- P: Waste is deposited in 0.90-4.50 m thick layers - In sanitary landfills, waste is typically
placed in layers and compacted to reduce its volume.
Q: Low lying marshy waste land is transformed into useful areas - Sanitary landfills can be constructed
on low-lying or marshy land, which can eventually be converted into useful areas such as parks or other
public spaces once the landfill is closed and properly capped.
Hemant Vilas Parulekar 633

S: Causes health and pollution hazards - Although sanitary landfills are designed to minimize
environmental impacts, there is still a potential for health and pollution hazards if the landfill is not
properly managed and maintained.
T: Chances of fire hazards in wet weather - Although sanitary landfills are designed to minimize
environmental impacts, there is still a risk of fire hazards due to the presence of combustible materials
in the waste. However, wet weather would reduce the risk of fires.

Q.74 Match the urban forms in Group I with their corresponding urban areas in Group II

Group I (Urban Form) Group II (Urban Area)


P. Sheet 1. Delhi
Q. Star 2. Tokyo
S. Ring 3. Copenhagen
T. Linear 4. Ohio
5. Bombay
6. Chandigarh

(A) (B) (C) (D)


P-3 P-2 P-1 P-4
Q-2 Q-3 Q-2 Q-1
R-4 R-4 R-3 R-2
S-6 S-5 S-6 S-3
Ans:- (B)
Q.75 Four sequences of development are given below. Choose the one that is appropriate for slums

(A) land-services-house-people (B) land-services-people-house

(C) people-land-house-services (D) land-people-house-services


Ans:- (C)
Explanation:- In the case of slums, people typically settle on vacant or underutilized land, often without
formal permission or ownership. They construct makeshift housing using available materials, and over
time, as the population increases and the settlement becomes more established, they may seek access to
basic services like water, sanitation, and electricity. This sequence represents the informal and organic
nature of slum development, where people first occupy the land and then housing and services follow.
634 GATE Architecture and Planning: Comprehensive Question Bank

Q.76 The neoclassical school and the modern ecological school of environment Studies differ on

(A) historical evaluation of environmental resources

(B) their relative emphasis on cost-benefit test vis-a-vis resource development.

(C) their variable interpretation of local and global ecosystems.

(D) their variable interpretation of Malthusian model of sustainability explaining environmental


disaster
Ans:- (B)

Explanation:- The neoclassical school and the modern ecological school of environmental studies differ
on:
(B) their relative emphasis on cost-benefit test vis-a-vis resource development.

The neoclassical school focuses on the economic aspects of environmental resources and relies heavily
on cost-benefit analysis as a decision-making tool. It seeks to maximize the efficient allocation of re-
sources by considering the trade-offs between economic growth and environmental impacts.

In contrast, the modern ecological school places greater emphasis on the interconnectedness of ecosys-
tems, the importance of sustainability, and the need for a more holistic approach to resource manage-
ment. While it may consider cost-benefit analysis as one of the tools, it is not the primary focus. Instead,
this school of thought highlights the need to maintain ecological balance and minimize negative im-
pacts on the environment. This approach often involves considering social and environmental factors
that are difficult to quantify in monetary termsTop of FormBottom of Form.
Q.77 The term Imperial Garden, Viewing hill, Middle Sea, South Sea, Gate Of Heavenly Peace, Land and
god of Grain are associated with

(A) Mongolian Garden-Hukado Garden, Ulan Bator (B) Japanese Garden-le Garden at Kobe

(C) Chinese Garden-Forbidden city at Peking (D) Persian Garden-Palace Court at Persopolis
Ans:- (C)

Explanation:- These terms are related to the Chinese gardening tradition and elements found within
the Forbidden City, which was the imperial palace for the Ming and Qing dynasties in Beijing, China.
Q.78 Find the thermal transmittance (U) of a 50 cm thick wall with 5 cm cavity, 1.8cm thick plaster inter-
nally and having a normal exposure.

Given: thermal resistivity of brickwork = 0.96 kcal per m2h0C

thermal resistivity of plaster = 2.02 kcal per m2 h0C

Thermal resistivity O13 Cm cavity = 0.205 kcal per m2h0C

Internal Surface resistance = 0.1440 kcal per m2h0C

External Surface resistance = 0.0620kcal per m2h0C

(A) 1.672 kcal per m2hoC (B) 1.078 kcal per m2hoC

(C) 0.935 kcal per m2hoC (D) 2.015 kcal per m2hoC
Hemant Vilas Parulekar 635

Ans:- (B)
Solution:- To find the thermal transmittance (U) of the wall, we need to consider the different layers
and their respective thermal resistances (R). The total thermal resistance of the wall is the sum of the
individual resistances, and U is the inverse of the total thermal resistance.
1. Calculate the thermal resistance of each layer: R_brickwork = Thickness_brickwork / Thermal_
resistivity_brickwork = 0.50 m / 0.96 (kcal/m²h°C) R_plaster = Thickness_plaster / Thermal_re-
sistivity_plaster = 0.018 m / 2.02 (kcal/m²h°C) R_cavity = Thermal_resistivity_cavity = 0.205
(kcal/m²h°C)
2. Add internal and external surface resistances: R_internal = 0.1440 (kcal/m²h°C) R_external =
0.0620 (kcal/m²h°C)
3. Calculate the total thermal resistance (R_total): R_total = R_brickwork + R_plaster + R_cavity
+ R_internal + R_external
4. Calculate the thermal transmittance (U): U = 1 / R_total
Using the provided data, we can calculate:
R_brickwork = 0.50 / 0.96 ≈ 0.5208 R_plaster = 0.018 / 2.02 ≈ 0.0089 R_total = 0.5208 + 0.0089 + 0.205
+ 0.1440 + 0.0620 ≈ 0.9407
U = 1 / 0.9407 ≈ 1.063 kcal/m²h°C
The thermal transmittance (U) of the wall is approximately 1.063 kcal/m²h°C, which is closest to option
(B) 1.078 kcal/m²h°C. Note that there might be minor discrepancies due to rounding.
Q.79 Domes of Islamic period are found in three distinct phases. Given below are four phases and their
respective examples. Identify the incorrect one.

Phases Type
(A) Imperial style Lodhi Tomb Dome, Delhi
(B) Provincial style Bijapur Dome
(C) Early style Samarkand Dome
(D) Mughal style Sultan Ghori Dome
Ans:- (B)

Explanation:-

Name Lodhi Tomb Dome, Delhi Bijapur Dome


Style Mughal style Provincial style
Image
636 GATE Architecture and Planning: Comprehensive Question Bank

Name Samarkand Dome Sultan Ghori Dome


Style Imperial style Early style
Image

Q.80 Identify the building that has open passageways, inner outer courts, influences from Lodhi gardens,
use of local stones for horizontal elements and precast concrete slabs for vertical elements, in -situ
concrete frames and burnt clay jalis.

(A) French Embassy staff quarters in New Delhi

(B) India International Center in New Delhi

(C) Akbar Hotel in New Delhi

(D) YMCA Guests House Complex and staff quarters in New Delhi
Ans:- (D)

Explanation:-

Structure French Embassy staff quarters, New India International Center, New Delhi
Delhi
Architect Raj Rewal Joseph Allen Stein
Image

Structure Akbar Hotel in New Delhi YMCA Guests House Complex and staff
quarters in New Delhi
Architect Prasad Shivnath Ranjit Sabikhi
Image

Q.81 Find the total volume of liquid in a septic tank with the following criteria.
i.Numbers of users=20
ii Cleaning intervals=365 days
iii The capacity required for sludge digestion=0.033 m3 per capita at 250C
iv Volume of digested sludge= 0.0002 m3 per capita per day
(A) 4080 litres (B) 2580 litres (C) 5250 litres (D) 3289 litres
Hemant Vilas Parulekar 637

Ans:- (B) (Closest answer)


Explanation:- To find the total volume of liquid in a septic tank, we need to consider the number of
users, cleaning intervals, capacity required for sludge digestion, and the volume of digested sludge. We
can follow these steps:
1. Calculate the volume of sludge digestion per capita: Volume of sludge digestion = Number of
users × Capacity required for sludge digestion = 20 × 0.033 m³ = 0.66 m³
2. Calculate the daily volume of digested sludge for all users: Daily volume of digested sludge =
Number of users × Volume of digested sludge per capita per day = 20 × 0.0002 m³ = 0.004 m³
3. Calculate the total volume of digested sludge over the cleaning interval: Total volume of digested
sludge = Daily volume of digested sludge × Cleaning interval (in days) = 0.004 m³ × 365 days =
1.46 m³
4. Calculate the total volume of liquid in the septic tank: Total volume = Volume of sludge diges-
tion + Total volume of digested sludge = 0.66 m³ + 1.46 m³ = 2.12 m³
5. Convert the total volume from cubic meters (m³) to liters: Total volume = 2.12 m³ × 1000 (1 m³
= 1000 liters) = 2120 liters
The total volume of liquid in the septic tank is approximately 2120 liters, which is not among the given
options. The closest option is (B).
Q.82 Safe axial load for a short R.C. square column having cross section as 300 mm x 300 mm and 4 lon-
gitudinal bars of diameter 20 mm (using M 20 concrete and Fe 415 steel) is near to
(A)100 KN (B) 10 KN (C) 1000 KN (D) 10000 KN
Ans:- (C)
Explain:- To calculate the safe axial load for a short reinforced concrete square column, we need to
consider the compressive strength of the concrete and the contribution of the steel reinforcement. Given
that the column uses M20 concrete and Fe415 steel, we can follow the process below:
1. Determine the cross-sectional area of the concrete (Ac) and steel (As): Ac = (300 mm x 300 mm)
- (4 x π x (20 mm / 2)2) As = 4 x π x (20 mm / 2)2
2. Calculate the compressive strength of the concrete and steel: For M20 concrete, the characteris-
tic compressive strength (fck) is 20 MPa or 20 N/mm². For Fe415 steel, the yield strength (fy) is
415 MPa or 415 N/mm².
3. Apply a partial safety factor for the concrete and steel: For the concrete, a partial safety factor
(γc) of 1.5 is commonly used: fck’ = fck / γc For the steel, a partial safety factor (γs) of 1.15 is
commonly used: fy’ = fy / γs
4. Calculate the safe axial load (Pu) for the column: Pu = 0.4 x fck’ x Ac + 0.67 x fy’ x As
Using the provided data, we can calculate:
Ac = (300 x 300) - (4 x π x (20 / 2)2) = 90000 - 1256 = 88744 mm² As = 4 x π x (20 / 2)2 = 1256 mm² fck’
= 20 / 1.5 = 13.33 N/mm² fy’ = 415 / 1.15 = 360.87 N/mm²
Pu = 0.4 x 13.33 x 88744 + 0.67 x 360.87 x 1256 Pu = 474043.36 + 305443.84 Pu ≈ 779487.2 N
Convert the safe axial load from newtons (N) to kilonewtons (kN): Pu ≈ 779.5 kN
The safe axial load for the given short R.C. square column is approximately 779.5 kN, which is closest to
the option (C) 1000 KN.
638 GATE Architecture and Planning: Comprehensive Question Bank

Q.83 Given below is a table of five types of roads and their characteristics. Which of the following types
represent the collector and the urban arterial street?

Type Width (meters) Recommended speed (km/h)


P 10-20 30
Q 20-30 50
R 30-40 60
S 50-60 80
T 80-100 80-100

(A) R, S (B) Q, S (C) Q, R (D) S, T


Ans:- (A)

Road Type Design Speed (km/h) Space Standards (meters)


Expressway 100-120 80-120 (depending on number of lanes)
Arterial 50-60 18-30
Sub-arterial 40-50 12-18
Collector 30-40 6-12
Local street 20-30 4-6
Q.84 Match the hierarchy of Chandigarh Road pattern.
Group I (Hierarchy) Group II (Connection)
(A) P-2,Q-4,R-1,S-3 P. V1 1. Sector connection
Q. V2 2. Cross country
(B) P-1,Q-2,R-5,S-3
R. V3 3. to building
(C) P-3,Q-4,R-5,S-2 S. V4 4. Sector divider
5. Pedestrain connection
(D) P-4,Q-1,R-3,S-5
Ans:- (A)

Explanation:- Please refer GATE-2013, Q. 3 for more information.


Q.85 Public interest determinants of urban development may be stated to comprise of
(A) health, infrastructure, entertainment, and safety
(B) economy, amenity, convenience, and entertainment
(C) health and safety, convenience, economy and amenity
(D) entertainment, economy, health and infastructure
Ans:- (C)
Explanation:- Public interest determinants of urban development refer to the factors that are important
for the well-being and benefit of the public as a whole. These determinants are often taken into consid-
eration by urban planners and policymakers when making decisions related to urban development.
The four key public interest determinants of urban development are:
1. Health and safety: Urban development should prioritize the health and safety of residents, with
a focus on creating healthy and safe living environments, including access to clean water and air,
proper sanitation, and emergency services.
Hemant Vilas Parulekar 639

2. Convenience: Urban development should provide convenient access to essential services and
facilities, such as transportation, schools, healthcare facilities, markets, and other amenities.
3. Economy: Urban development should support economic growth and development, including
job creation, income generation, and overall economic prosperity for residents.
4. Amenity: Urban development should provide a high quality of life and a pleasant living envi-
ronment for residents, with access to parks, green spaces, cultural and recreational facilities, and
other amenities that enhance quality of life.
Q.86 Which housing layout does not follow a strict geometric (orthogonal) pattern?

(A) Prastara variation (B) Cluster variation

(C) Finger variation (D) Finger and cluster variation


Ans:- (D)
Q.87 Choose the appropriate concept propagated in the urban land use theory based on economic deter-
minants.

(A) Concentric Zone theory (B) Sector theory

(C) Axial development theory (D) Multiple nuclei theory


Ans:- (D)

Explanation:- The Multiple Nuclei Theory is a concept in urban land use theory that suggests that
instead of one central business district, there are several nuclei or centers of development within a city
that act as focal points for different activities. This theory proposes that cities are formed by the coming
together of various centers of activity, each with their own distinct character, and that these centers grow
and develop over time, giving rise to a complex and diverse urban landscape.

The Multiple Nuclei Theory was first proposed by Harris and Ullman in 1945 and was a departure from
the earlier Concentric Zone Theory and Sector Theory which suggested that urban growth was based
on concentric circles or sectors of development around a central business district. The Multiple Nuclei
Theory is more flexible and better able to account for the complexity and diversity of modern cities, as it
recognizes that different parts of a city may have different functions and characteristics that contribute
to the overall urban fabric.
Q.88 Identify the characteristic of a ‘district’ as an element of imageability.

P. Has two-dimensional extent

(Q) Is channel of movement

(R)Used as lateral references, boundaries or breaks

(S) Observer enters ‘inside of ‘

(T) Presence of a common theme

(A) P,S,T (B) P,R,S (C) Q,R,S (D) P,R,S,T


Ans:- (D)

Explanation:- Please refer GATE-2009, Q. 26 for more information.


640 GATE Architecture and Planning: Comprehensive Question Bank

Q.89 A settlement pattern has a 4x4 matrix with a cross like principal axis of main roads and four small-
er crosses like networks of arterial roads, opening upto 4 gateways cardinally. The pattern is called

(A) Dandaka (B) Nandyavarta (C) Chaturmukha (D) Sarvatobhadra


Ans:- (D)

Explanation:- Please refer GATE-2013, Q. 7 for more information.


Q.90 If the expected duration of a project is 30 months along with a standard deviation of 6 months,
what is the probability that the project will be completed in 26 months?

(A) 0.100 (B) 0.500 (C) 0.253 (D) 0.340


Ans:- No proper answer
Explanation:- To calculate the probability that a project will be completed in 26 months, we need to use
the standard normal distribution and convert the 26-month value to a z-score using the formula:
z = (x - μ) / σ
where x is the value of interest (26 months), μ is the expected duration (30 months), and σ is the standard
deviation (6 months).
So, we have:
z = (26 - 30) / 6 = -0.67
GATE QUESTION PAPER 2002
General Aptitude (GA)

Q. 1 – Q. 25 carry one mark each.


Q.1 The colour code for the storage of objects with radiation hazards according to the Occupational
Safety Hazards Act (OSHA) is
(A) red with black (B) purple with yellow
(C) orange with blue (D) green with white
Ans:- (B)
Explanation:-
Colour Implies Application
Red Danger Safety cans and signs.
Stop emergency stop buttons and fire extinguishers.
Fluorescent Orange, Biosafety Labels and containers for biomedical waste (warning
Orange-Red must be in Fluorescent Orange, Orange-Red with the
biosafety symbol in a contrasting colour).
Orange Warning used to indicate dangerous or energized equipment, such
as electrical panels and switches.
Yellow Caution tripping hazards and fall hazards
Green Safety used to indicate safety or first aid equipment, such as
eyewash stations and first aid kits.
Blue Information used to indicate information or mandatory actions, such
as signs indicating the location of emergency equipment.
Purple Radiation used to indicate radiation hazards, such as radioactive
materials or equipment that emits radiation.
Black and white Boundaries used to indicate traffic areas, such as aisles and pathways.
Q.2 Terra-cotta is a kind of earthen work processed by
(A) drying in the sun (B) burning at high temperature
(C) mixing with lime (D) compacting under pressure
Ans:- (B)
Explanation:- Terra-cotta is a kind of earthenware made from a clay-based unglazed ceramic material.
The term “terra-cotta” comes from the Italian words for “baked earth.” The process of making terra-cotta
involves molding clay into the desired shape, drying it, and then firing it in a kiln at high temperatures
until it becomes hard and durable. The firing process gives terra-cotta its characteristic reddish-
brown color and hard, slightly porous texture. Terra-cotta has been used for centuries in architectural
ornamentation, pottery, and sculpture.
Q.3 Sand blasting is a technique used for cleaning the surface of
(A) stone work (B) concrete work (C) wood work (D) earth work
642 GATE Architecture and Planning: Comprehensive Question Bank

Ans:- All choices are correct.


Explanation:- Sand blasting is a technique used for cleaning the surface of various materials such as
metal, concrete, and masonry. It involves directing a high-pressure stream of abrasive material (usually
sand) against the surface to remove dirt, paint, rust, and other coatings. The abrasive material is propelled
by compressed air or water. Sand blasting is commonly used to prepare surfaces for painting or other
surface treatments, or to clean and restore historical buildings or monuments.
Q.4 Series of column in a straight row arrangement is termed as
(A) Orthostyle (B) Peristyle (C) Tracery (D) Colonia
Ans:- (B)
Explanation:- Please refer GATE-1999, Q.1 for more information.
Q.5 When the picture plane moves towards the observer, the size of the picture in perspective
(A) does not change (B) increases (C) decreases (D) gets elongated
Ans:- (B)
Explanation:- When the picture plane moves towards the observer, the size of the picture in perspective
increases. This is because as the picture plane moves closer to the observer, the angle of view widens and
more of the scene is visible. As a result, objects in the scene appear larger on the picture plane. This is
known as foreshortening and is a common effect in perspective drawing.
Q.6 The term “Vedika’ in Buddhist architecture is used for
(A) railing around stupa (B) crown umbrella of stupa
(C) decorative entrance of stupa (D) niche in the wall of stupa
Ans:- Please refer GATE-2009, Q.28 for more information.
Q.7 Capital value of a property is equal to
(A) net rent value per year multiplied by sinking fund
(B) annual rent minus outgoings
(C) net annual rent multiplied by years purchase
(D) gross annual rent plus overhead costs
Ans:- (C)
Explanation:- The capital value of a property is equal to the net annual rent multiplied by years purchase.
Q.8 ‘Demac gauge’ is a device for
(A) measuring mechanical strain in structures
(B) measuring the dampness of inner walls
(C) checking the thermal conductivity of walls
(D) gauging air temperature
Ans:- (A)
Explanation:- The ‘Demac gauge’ (Demountable Mechanical Strain Gauge) is a device used for
measuring mechanical strain in structures, especially in concrete structures. It is a type of deformation
gauge that is used to measure the changes in the shape or size of the structure due to applied loads or
environmental factors such as temperature or moisture. The device consists of a metal foil that is bonded
to the surface of the structure and connected to an electrical circuit. As the structure deforms, the foil
also deforms, causing a change in the electrical resistance of the circuit, which can be measured and
recorded.
Hemant Vilas Parulekar 643

Q.9 Funicular polygons are


(A) Stress diagrams (B) Force diagrams
(C) Regular shapes (D) Irregular shapes
Ans:- (B)
Explanation:- Funicular polygons are force
diagrams that represent the magnitude and direction
of forces acting on a structure or system of forces
in equilibrium. They are named after the funicular
state, which is the state of a structure under the
influence of a system of forces where all the forces
are concurrent and parallel to the direction of
gravity. Funicular polygons are used in structural
engineering and architecture to analyze and design
structures that are in equilibrium under the influence
of external loads and internal forces.

Q.10 The duct size for an air conditioning system is dependent on amount of air flow and its
(A) velocity (B) temperature (C) relative humidity (D) latent heat
Ans:- (A)
Q.11 The botanical name of “Neem’ tree, used for roadside landscaping is
(A) Lagerstroemia speciosa (B) Ficus benghalensis
(C) Klein hama hospital (D) Azadirachata Indica
Ans:- (D)
Q.12 Symbolic representation of water by sand is frequently found in
(A) Mughal Garden (B) English Garden
(C) Japanese Garden (D) Moorish Garden
Ans:- (C)
Explnation:- Please refer GATE-2016, Q.4 for more information.
Q.13 Kenzo Tange developed the concept of
(A) Deconstructivism (B) Arcology (C) Metabolism (D) Anamorphosis
Ans:- (C)
Explanation:- Kenzo Tange developed the concept of “Metabolism” in architecture. Metabolism is a
Japanese architectural movement that emerged in the post-war period, particularly in the 1960s. The
term “metabolism” refers to the idea of architecture as a living, growing, and changing organism that
can adapt to its environment and the needs of its inhabitants over time. This movement was influenced
by ideas from biology, technology, and science fiction, and was characterized by its focus on modularity,
flexibility, and the use of prefabrication and industrial materials. Tange was one of the leading figures of
the Metabolist movement and his work helped to shape the modern architecture of Japan.
Q.14 The maximum slope in a for wheel chair movement is
(A) 1:8 (B) 1:12 (C) 1:20 (D) 1:25
644 GATE Architecture and Planning: Comprehensive Question Bank

Ans:- (B)
Explanation:- According to the Handbook on Barrier-Free Environment and Accessibility, published by
the Central Public Works Department (CPWD) of India, the maximum slope for wheelchair movement
should not exceed 1:12 (8.33%) for a continuous run of ramp, and 1:8 (12.5%) for a short length of ramp.
It is important to maintain these maximum slopes to ensure that wheelchair users can easily and safely
access different levels of a building.
Q.15 Preserving timber by Charring is carried out for
(A) Wooden piles (B) Wooden partitions
(C) Wooden doors (D) Wooden truss
Ans:- (A)
Explanation:- Preserving timber by charring is carried out for wooden
piles. Charring, also known as carbonization, involves burning the
surface of the timber to create a layer of charcoal. This layer of charcoal
protects the wood from rot, insects, and decay, making it a common
method of preserving timber for use in construction, particularly for
piles that are intended to be driven into the ground. The charring
process also makes the wood fire-resistant.
Q.16 In construction work, Hoes are used for
(A) transporting materials (B) excavating trenches
(C) mixing concrete (D) compacting sand
Ans:- (B)
Explanation:- Hoes are typically used for excavating trenches in construction work. They are handheld
tools with a flat, pointed blade that is used to break up soil and other materials, such as clay or gravel,
and move them from one location to another. Hoes can also be used for other tasks such as weeding
and digging shallow holes, but in construction work they are primarily used for excavating trenches for
foundations, pipes, and other underground utilities.
Q.17 Swastika form of settlement layout in ancient Indian town planning is basically
(A) a grid iron pattern (B) a radial pattern
(C) a ring radial pattern (D) an informal pattern
Ans:- (A)
Explanation:- Please refere GATE-2013, Q.7 for more information.
Q.18 Pompidou Centre in Paris, designed by Renzo Piano, illustrates the concept of
(A) Biology analogy (B) Mechanical analogy
(C) Romantic analogy (D) Pattern Language analogy
Ans:- (B)
Explanation:- The Pompidou Centre in Paris, designed by Renzo
Piano and Richard Rogers, illustrates the concept of mechanical
analogy. The building’s mechanical systems, such as air conditioning
ducts, water pipes, and escalators, are expressed on the exterior of
the building, giving it a unique and recognizable appearance. This
approach is often referred to as “inside-out” architecture, where the
building’s systems and structure are visible on the exterior, rather than being hidden behind walls and
facades. This idea of expressing a building’s inner workings on its exterior is an example of mechanical
analogy, where the building is likened to a machine or mechanism.
Hemant Vilas Parulekar 645

Q.19 The Kandariya Mahadeo Temple in Khajuraho is an example of


(A) Rock Cut architecture (B) Nagara architecture
(C) Dravida Architecture (D) Indo Saracenic architecture
Ans:- (B)
Explanation:- The Kandariya Mahadeo Temple in Khajuraho is an example of Nagara architecture.
Nagara style of architecture is characterized by a curvilinear spire, known as shikhara, which is topped
by an amalaka (circular ribbed stone) and kalasha (water-pot). It originated in the northern regions
of India and was particularly popular in the medieval period. The Khajuraho temples, including the
Kandariya Mahadeo temple, were built between the 10th and 12th centuries CE, during the Chandela
dynasty, and are some of the finest examples of Nagara architecture.
(Please refer GATE-2001, Q.1.11 for more information)
Q.20 Decision on phasing is required in the design of
(A) channelized intersection (B) priority intersection
(C) grade separated intersection (D) signalized intersection
Ans:- (D)
Explanation:- Decision on phasing is required in the design of a signalized intersection. Phasing refers
to the specific time intervals during which different movements of traffic (such as through traffic, left
turns, and right turns) are given the right-of-way at the intersection. The proper phasing of traffic signals
is critical to ensure the safe and efficient movement of traffic through the intersection. It also plays an
important role in managing traffic congestion and reducing delay. The phasing plan for a signalized
intersection is typically developed based on traffic volume and patterns, as well as safety considerations.
Q.21 For street lighting maximum lumen per watt is obtained from
(A) sodium vapour lamps (B) mercury vapour lamps
(C) incandescent lamps (D) fluorescent lamps
Ans:- (A)
Explanation:- or street lighting, the maximum lumen per watt is obtained from sodium vapour lamps.
Sodium vapour lamps are energy-efficient and have a high luminous efficacy, meaning they produce
more light per watt of energy consumed compared to other types of lamps such as mercury vapour
lamps, incandescent lamps, or fluorescent lamps. Sodium vapour lamps also have a longer lifespan and
lower maintenance costs compared to other types of lamps. These properties make them popular for
street lighting applications. However, the light they produce has a yellowish hue, which may not be
suitable for all types of outdoor lighting applications.
Q.22 ‘Logit Model’ for discrete choice analysis has been developed by
(A) A.O. Hirshman (B) D. McFadden (C) P. Nijkamp (D) T. Scitovsky
Ans:- (B)
Explanation:- The Logit Model is a statistical model used for analyzing and modeling the behavior of
individuals or groups in choosing among a set of discrete alternatives. It was developed by economist
Daniel McFadden in the early 1970s and has since become a widely used tool for analyzing choices in a
variety of fields, including transportation, marketing, and public policy.
646 GATE Architecture and Planning: Comprehensive Question Bank

The Logit Model assumes that individuals make choices based on a set of underlying attributes or
characteristics associated with each alternative. These attributes can be either observable or unobservable,
and may include factors such as price, quality, convenience, and personal preferences.
The Logit Model estimates the probability that an individual will choose a particular alternative based
on the attributes associated with that alternative. The model assumes that the relationship between
the attributes and the choice probabilities is linear in the form of a logit function, which is a logistic
transformation of the ratio of the probability of choosing one alternative to the probability of choosing
all alternatives.
The Logit Model has several advantages, including its ability to model both continuous and discrete
variables, its flexibility in accommodating different types of data, and its ability to estimate the effect
of individual attributes on the probability of choosing a particular alternative. The Logit Model is also
relatively easy to estimate and interpret, making it a popular tool for modeling and predicting choice
behavior.
Q.23 ET index can be obtained from nomograms showing
(A) DBT, WBT and air velocity (B) DBT, AH and RH
(C) AH, RH and air velocity (D) WBT and air velocity
Ans:- (A)
Explanation:- The ET (evapotranspiration) index can be obtained from nomograms showing DBT
(dry bulb temperature), RH (relative humidity), and wind speed. The ET index is used to estimate the
amount of water lost through evapotranspiration from a crop or vegetation. The nomogram uses the
DBT, RH, and wind speed values to calculate the ET index, which is then used to determine the amount
of water needed for irrigation to replace the water lost through evapotranspiration.
Q.24 SKPOLY command in AutoCad helps in
(A) sticking together two polylines (B) drawing street kerbs with polyline
(C) creating polylines while sketching (D) creating shortest path along polylines
Ans:- (C)
Explanation:- The SKPOLY command in AutoCAD is used to create a polyline that is offset from an
existing polyline at a specified distance. It is a useful tool for creating a parallel copy of a polyline. The
command works by asking the user to specify the distance and direction of the offset from the original
polyline, and then creates a new polyline at the specified distance. Therefore, option (C) “creating
polylines while sketching” is the correct answer.
Q.25 Class II towns as defined in the Indian Census relate to a minimum population Size of
(A) 2,00,000 (B) 1,50,000 (C) 1,00,000 (D) 50,000
Ans:- (D)
Explanation:- The Indian Census classifies different classes of towns based on their population size. The
classes and the corresponding minimum population sizes are:
1. Class I: Population of 100,000 and above
2. Class II: Population of 50,000 to 99,999
3. Class III: Population of 20,000 to 49,999
4. Class IV: Population of 10,000 to 19,999
5. Class V: Population of less than 10,000
These classifications are used for administrative and planning purposes by the Indian government.
Hemant Vilas Parulekar 647

Q.26-Q.50 carry two marks each


Q.26 A town having a population of 88,236 (in the year 1971) has shown an arithmetic increase in its
population. The growth rate was 12 % per decade. The population in 2001 was
(A) 97,122,000 (B) 1,03,007 (C) 1,20,000 (D) 2,59,000
Ans:- (C)
Solution:- To calculate the population in 2001, we can use the formula:
Population in 2001 = Population in 1971 x (1 + growth rate)(number of decades)
Here, the population in 1971 is given as 88,236, and the growth rate is 12% per decade. We need to
calculate the number of decades between 1971 and 2001.
Number of decades = (2001 - 1971) / 10 = 3
Substituting the values in the formula, we get:
Population in 2001 = 88,236 x (1 + 0.12)3 = 120,042.45
Rounding off to the nearest integer, we get:
Population in 2001 = 120,042
Therefore, the population in 2001 was 120,042.
Q.27 The series 1/1, 1/3, 1/5, 1/7, 1/9.........is known as
(A) geometric series (B) harmonic series (C) Fibonacci series (D) golden mean series
Ans:- (B)
Explanation:- The series 1/1, 1/3, 1/5, 1/7, 1/9, ... is known as the harmonic series.
(A) A geometric series is a series of numbers where each term after the first is found by multiplying the
previous term by a fixed number called the common ratio. The general form of a geometric series is:
a, ar, ar2, ar3, ...
where a is the first term, r is the common ratio, and each term after the first is obtained by multiplying the
previous term by r. Geometric series can be used to model many natural phenomena, such as population
growth or compound interest.
(B) A harmonic series is a series of numbers where each term is the reciprocal of a positive integer. The
general form of a harmonic series is:
1, 1/2, 1/3, 1/4, 1/5, ...
Harmonic series can be used to model some natural phenomena, such as the distribution of particles in
a fluid.
(C) A Fibonacci series is a series of numbers where each term is the sum of the two preceding terms. The
general form of a Fibonacci series is:
1, 1, 2, 3, 5, 8, 13, 21, 34, 55, ...
Fibonacci series can be found in many natural phenomena, such as the branching of trees, the
arrangement of leaves on a stem, and the spirals on a seashell.
(D) The golden mean series is a special case of the Fibonacci series, where each term is obtained by
dividing the previous term by the golden ratio, which is approximately equal to 1.618. The general form
of the golden mean series is:
1, 1/1.618, (1/1.618)2, (1/1.618)3, ...
The golden mean series can be found in many natural phenomena, such as the spiral patterns in
sunflower heads and the branching of trees. The golden ratio is also considered aesthetically pleasing
and has been used in art and architecture for thousands of years.
648 GATE Architecture and Planning: Comprehensive Question Bank

Q.28 For a Split Complementary colour scheme, the primary red colour should be associated with
(A) blue violet-red violet (B) red orange-yellow orange
(C) blue green-yellow green (D) red violet-blue green
Ans:- (C)
Explanation:- For a Split Complementary colour scheme, the primary colour is paired with two colours
that are adjacent to its complementary colour. So, for primary red, the split complementary colours
would be yellow-green and blue-green.
Q.29 The fixed end moment of a cantilever beam with a u.d.l. of 20 KN/m and span of 3 m is
(A) 60 kNm (B)90kNm (C) 120KNm (D) 150kNm
Ans:- (B)
Solution:- The fixed end moment of a cantilever beam with a uniformly distributed load (UDL) of 20
kN/m and span of 3 m is given by:
M = wL2/2
where w is the UDL, and L is the span.
Substituting the given values, we get:
M = 20 x 32/2 = 90 kNm
Therefore, the fixed end moment of the cantilever beam is 90 kNm. Answer: (B) 90 kNm.
Q.30 The buildable area available on a plot of one hectare with a FAR of 2.25 is
(A) 15500 sqm (B) 20500 sqm (C) 22500 sqm (D) 25500 sqm
Ans:- (C)
Solution:- The buildable area available on a plot of one hectare with a FAR (Floor Area Ratio) of 2.25
can be calculated as follows:
Buildable area = FAR x plot area Buildable area = 2.25 x 10,000 sqm (1 hectare = 10,000 sqm) Buildable
area = 22,500 sqm
Therefore, the answer is (C) 22,500 sqm.
Q.31 For an open drain if the ratio or the are due to wetted perimeter is tripled A the velocity of water in
the drain increases by
(A) two times (B) three times (C) four times (D) five times
Ans:- (A)
Explanation:- As per Manning’s equation,
V = (1/n) x R2/3 x S1/2
Where, V= Velocity, n=constant, S=Slope of drain, R= (Area/Wetted Perimeter). This implies, V � R2/3
If Ratio of R is increased thrice, V2 = (1/n) x (3R)2/3 x S1/2
= (3)2/3 x 1/n x R2/3 x S1/2
= 2.08 x 1/n x R2/3 x S1/2
V2 = 2 x V
Hence, if the ratio of Wetted Perimeter/Area of the drain is tripled, the velocity increases two times.
Hemant Vilas Parulekar 649

Q.32 In transportation planning Link-Node diagrams are required for


(A) estimation of zonal trips
(B) determination of trip characteristics
(C) trip assignment along roads
(D) determination of mode characteristics
Ans:- (C)
Explanation:-
Link-Node diagrams are typically used in transportation planning for the determination of trip
characteristics.
They are a graphical representation of the transportation network, with links representing physical
transportation routes (such as roads or transit lines) and nodes representing points of intersection or
transfer between different links.
These diagrams are used to analyse traffic flow, identify congestion points, and plan for improvements
to the transportation network.
C
|
|
A-------------1------------ |-------------3--------------D
|
|
B
In transportation planning and network analysis, various types of diagrams and models are used to
represent and analyze transportation systems. One of the common diagram types is the Link-Node
diagram. Here’s an overview:
1. Link-Node Diagrams:
- Links: In transportation planning, “links” typically represent the physical segments of transportation
infrastructure like roads, highways, railways, or even pedestrian paths. Each link has attributes like
length, capacity, and travel time.
- Nodes: Nodes represent locations where these links intersect or connect. Nodes can be intersections,
junctions, traffic circles, or any point where travelers can change direction or mode.
- Connectivity: Link-node diagrams illustrate the connectivity of the transportation network, showing
how links intersect and how travelers can move from one location (node) to another via different routes
(links).
- Analysis: These diagrams are fundamental for various transportation analyses, including route
planning, traffic flow modeling, congestion management, and more.
650 GATE Architecture and Planning: Comprehensive Question Bank

2. Flow Diagrams:
- Flow diagrams are used to depict the movement of people, vehicles, or goods within the transportation
network. They help visualize the volume and direction of traffic between nodes.
- Common types of flow diagrams include origin-destination (OD) matrices, flow maps, and flowcharts.
OD matrices show the movement of travelers between specific origins and destinations, while flow maps
provide a visual representation of traffic patterns.
3. Schematic Diagrams:
- Schematic diagrams simplify complex transportation networks into more understandable
representations. These are often used in transit systems, showing routes, stops, and transfer points.
- Transit diagrams, like subway or bus maps, are a common example.
They provide a simplified view of a transit system to help passengers understand routes and connections
easily.
4. Network Topology Diagrams:
- Network topology diagrams focus on the structure of transportation networks.
They show how links and nodes are connected and may include information about the hierarchy of
roads or the presence of major transportation hubs.
- These diagrams help planners understand the overall layout of the transportation network and identify
critical nodes or bottlenecks.
5. Tree Diagrams:
- Tree diagrams are used to illustrate hierarchy or branching within a transportation system.
For instance, they can depict how major highways branch into smaller roads or how a transit system
connects to various neighborhoods.
- These diagrams can be useful for understanding how different parts of the transportation network are
organized.
6. Phase Diagrams:
- Phase diagrams are often used in traffic signal design and traffic engineering.
They show how traffic signal phases (e.g., green, red, yellow) are timed at intersections to optimize
traffic flow.
- These diagrams, along with various modeling and simulation techniques, help transportation planners
and engineers analyze, design, and optimize transportation systems to meet the needs of communities
and efficiently move people and goods.
The choice of diagram or model depends on the specific goals and characteristics of the transportation
project.
Hemant Vilas Parulekar 651

Q.33 Water while falling down from a horizontal surface with an edge lip creates
(A) glassy sheet flow (B) vertical angular rhythm
(C) agitated frothy surface in multiple layers (D) none of the above
Ans:- (A)
Explanation:- Water falling down from a horizontal surface with an edge lip creates a glassy sheet flow.
Q.34 The R.L.at the basement floor of a building and the R.L.at the road surface are 93.00 and 94.85
respectively. If the plinth height from the road is 600 mm then the depth of the basement is
(A) 1.25 (B) 1.85 (C) 2.35 (D) 2.45
Ans:- (D)
Explanation:- Difference between road level = 94.85 – 93.00 = 1.85m
Plinth height of the basement = 1.85 + 0.60 = 2.45m
Q.35 Along a critical path crashing is carried out for the activity that has
(A) minimum cost-time slope (B) maximum cost-time slope
(C) minimum cost slope (D) maximum time slope
Ans:- (A)
Explanation:- Please refer GATE-1993, Q.20 for more information.
Q.36 Archimedean solids have
(A) regular faces (B) right angles (C) irregular faces (D) none of the above
Ans:- (C)
Explanation:- Archimedean solids have regular faces, but they
may not have right angles. The Archimedean solids are a class
of polyhedra that are composed of regular polygons meeting in
identical vertices, but they may have faces that are not identical
to each other. The vertices of an Archimedean solid may also
have different arrangements, resulting in various symmetries.
Examples of Archimedean solids include the truncated
icosahedron (the shape of a soccer ball) and the rhombicosidodecahedron.
Q.37 If the scale of the map is 1:30,000, then 1 sq cm area of the map would represent
(A) 9.00 Hectare (B) 15.00 Hectare (C) 17.30 Hectare (D) 30.00 Hectare
Ans:- (A)
Solution:- Scale map = 1:30,000
Thus, 1cm2 = 1cm x 1cm = 30,000 x 30,000 (on ground)
= 90,00,00,000 cm2 = 9.00 Ha
Q.38 The oxygen demand in the initial stage of the biological decomposition Of sewage is due to the
presence of
(A) nitrogenous matter (B) phosphate matter
(C) carbonaceous matter (D) calcareous matter
652 GATE Architecture and Planning: Comprehensive Question Bank

Ans:- (C)
Explanation:- The oxygen demand in the initial stage of the biological decomposition of sewage is
primarily due to the presence of carbonaceous matter. Carbonaceous matter is made up of organic
compounds, such as sugars, starches, and proteins, that are broken down by microorganisms during the
biological decomposition process. As these compounds are broken down, they consume oxygen, leading
to the demand for oxygen in the water. Nitrogenous and phosphate matter may also contribute to the
oxygen demand during the decomposition process, but their contribution is typically smaller than that
of carbonaceous matter.
Q.39 Number of modular bricks required for 10 cum brickwork is
(A) 4500 (B) 4750 (C) 5000 (D) 5225
Ans:- (C)
Explanation:- To calculate the number of modular bricks required for 10 cum brickwork, we need to
first determine the volume of one modular brick.
The volume of one modular brick is:
0.20 m x 0.10 m x 0.10 m = 0.002 cubic meters
Next, we need to calculate the total volume of bricks required for 10 cubic meters of brickwork:
Volume of bricks = 10 cubic meters / 0.002 cubic meters per brick Volume of bricks = 5000 bricks
Therefore, the answer is (C) 5000 bricks.
Q.40 GPS instrument is used for determining
(A) Temperature of a place (B) Lat-Long of a place
(C) Rainfall of a place (D) Wind velocity of a place
Ans:- (B)
Q.41 An ideal example of a tensile roofing system is found in
(A)Sydney Opera House by J Utzon
(B)Olympic stadium in Tokyo by Kenzo Tange
(C)TWA airport terminal by Eero Saarinen
(D) Bahai Temple in New Delhi by Fairburz Sahba
Ans:- (B)
Explanation: - Please refer GATE-2016, Q.32 for more information.
Q.42 In measuring traffic noise, the noise level L90 represents
(A) background noise level (B) lowest noise level
(C) average noise level (D) peak noise level
Ans:- (A)
Explanation:- In measuring traffic noise, the noise level L90 represents the background noise level. L90 is
the sound level exceeded for 90% of the measurement period, meaning that only 10% of the measurement
period is expected to have noise levels higher than L90. The background noise level is the sound level
that is consistently present in an environment, even in the absence of any other sounds or noises. It is
typically measured over an extended period to obtain a representative value.
Hemant Vilas Parulekar 653

Q.43 The process of acquiring excess land for selling at a higher price after the completion of the project
is known as
(A) Encroachment (B) Enforcement (C) Betterment (D) Recoupment
Ans:- (C)
Q.44 Cadastral map shows
(A) cropping pattern (B) land subdivisions
(C) land ownership (D) depth of water bodies
Ans:- (C)
Explanation:- A cadastral map is a type of map that shows the boundaries
of land ownership, as well as other details such as the location of buildings,
roads, watercourses, and other features. Essentially, a cadastral map is a
detailed map of the land within a particular area, and it is often used by
government agencies, land surveyors, and others to help manage land use,
property taxation, and other related activities. So, option (C) land ownership
is the correct answer.
Q.45 Participation ratio is obtained from
(A) work force and population in work age group (B) work force and total population
(C) male work force and female work force (D) work force and in-migration
Ans:- (B)
Explanation:- The participation ratio is obtained from the work force and the total population. It is a
measure of the percentage of the population that is in the labor force, or in other words, the percentage of
the population that is either employed or actively seeking employment. The formula for the participation
ratio is:
Participation ratio = (Labor force / Total population) x 100
Q.46 To perceive the backdrop of a building more than its facade detail and the total façade simultaneously,
the height distance relationship should be at least
(A) 1:1 (B) 1:2 (C) 1:4 (D) 1:6
Ans:- (C)
Explanation:- The height distance relationship you are referring to is called the “foreshortening effect”.
Foreshortening is the visual distortion that occurs when an object is viewed at an angle, such that one
dimension appears shorter than the others. In architecture, this effect is important to consider when
designing buildings in relation to their surrounding context, particularly in urban settings where
buildings may be viewed from a distance.
The appropriate height distance relationship for perceiving the backdrop of a building more than its
facade detail and the total façade simultaneously may depend on several factors, including the height
of the building, the distance from the viewer, and the surrounding context. However, a general rule of
thumb is that the height distance relationship should be at least 1:4. This means that for every 1 unit of
height, the viewer should be at least 4 units of distance away.
Q.47 Marble stone used in buildings, is a type of
(A) Metamorphic rock (B) Calcerous rock (C) Igneous rock (D) Sedimentary rock
654 GATE Architecture and Planning: Comprehensive Question Bank

Ans:- (A)
Explanation:- Please refer GATE-2010, Q.1 for more information.
Q.48 IRDP in India is associated with
(A) regional development (B) rural development
(C) riverfront development (D) road development
Ans:- (B)
Explanation:- IRDP stands for Integrated Rural Development Programme, which was a poverty
alleviation program launched by the Government of India in 1978. Therefore, option (B) rural
development is the correct answer. The program aimed to improve the overall quality of life of rural
people by providing various types of assistance, such as income-generation activities, infrastructure
development, and social services. It was one of the largest rural development programs in the world,
and it aimed to reduce poverty, generate productive employment opportunities, and raise the standard
of living in rural areas of India.
Q.49 Incentive Zoning is related to
(A) provision of extra advantages to developers
(B) provision of extra advantage to common people
(C) provision of extra advantage to traffic
(D) provision of extra advantage to handicapped citizens
Ans:- (B)
Explanation:- Incentive zoning is a planning tool that provides incentives or bonuses to developers
in exchange for certain public benefits or amenities that they provide in their development projects.
The incentives typically take the form of increased density or height allowances, reduced setback
requirements, or other zoning relaxations that allow developers to build more than they would be
allowed under standard zoning regulations.
In exchange for these incentives, developers are required to provide certain public benefits or amenities,
such as affordable housing units, public open spaces, transit improvements, or historic preservation.
The idea behind incentive zoning is to use the private sector’s interest in maximizing profits to achieve
public goals and objectives, such as affordable housing or improved public access to waterfront areas.
Q.50 When the arrival rate of bank customers is lower than the service rate at the counters
(A) queues ae quite often formed (B) queues are never formed
(C) queues of infinite length are formed (D) queues of particular length are always formed
Ans:- (A)
GATE QUESTION PAPER 2001
General Aptitude (GA)

Only MCQs have been covered for this paper


Q. 1 – Q. 25 carry one mark each.
Q.1.1 According to Aristotle, the ideal range of population for a city or polis is
(A) 2000-30,000 (B) 1,000-10,000 (C) 10,000-20,000 (D) 10,000-1,00,000
Ans: - (B)
Q.1.2 Crash Time’ of a project indicates the
(A) optimum time in which a project is completed
(B) shortest time in which a project can be completed
(C) maximum delay that the project can undergo
(D) point of time at which the project will fail
Ans: - (B)
Q.1.3 Architrave’ is the
(A) slab of stone on top of classical order
(B) lowest part of entablature
(C) wall supporting the weight of an arch or vault
(D) topmost part of a classical Greek column
Ans: - (B)
Q.1.4 The first garden city, Letchworth was designed by
(A) Ebenezar Howard (B) Antonio Sant Elia
(C) Raymond Unwin (D) Clarence Perry
Ans: - (A)
Q.1.5 The minimum water supply requirement per head per day for residential purpose including
drainage and sanitation is
(A) 70 litre (B) 135 litre (C) 180 litre (D) 210 litre
Ans:- All are correct
Q.1.6 The term ‘Conservative Surgery’ was coined by
(A) Patrick Geddes (B) Le Corbusier (C) Edwin Lutyens (D) Frank Lloyd Wright
Ans: - (A)
Q.1.7 Level of Services (LOS) for a signalized intersection is defined in terms of
(A) traffic flow characteristics (B) mix of modes
(C) delay (D) profile of intersection
Ans: - (A)
656 GATE Architecture and Planning: Comprehensive Question Bank

Q.1.8 The designer of the garden of Versailles, France was


(A) Andre Le Notre (B) Ebenezer Howard
(C) Andrea Palladio (D) Patrick Geddes
Ans: - (A) (Please refer GATE-2004, Q. 58 for more information)
Q.1.9 The noise level for lecture rooms should be kept at
(A) 30dB (B) 40dB (C) 50dB (D) 60dB
Ans: - (B)
Explanation: - Please refer GATE-2005, Q. 33 for more information.
Q.1.10 The architectural movement, which was also popularly referred to as ‘Jazz Moderna’ is
(A) Arts and Crafts movement (B) Art Nouveau movement
(C) Art Deco movement (D) Ecole de Beaux Arts movement
Ans: - (B)
Explanation: - Art Nouveau was a decorative and architectural style that originated in the late 19th
century and reached its peak in the early 20th century. It is characterized by its use of flowing, organic
forms inspired by nature, as well as an emphasis on craftsmanship and decoration.
The movement emerged as a reaction against the industrialization and mass production of the time,
with its practitioners seeking to create more decorative and ornate works that celebrated individual
craftsmanship and creativity. The style is often associated with the Art Nouveau movement in France,
but it had many other names and variations in other countries.
Art Nouveau was heavily influenced by the Arts and Crafts movement, as well as Japanese art, which
was becoming increasingly popular in Europe at the time. Its motifs include curving lines and stylized
flowers, plants, and animals, often arranged in asymmetrical patterns. The use of new materials such
as glass, wrought iron, and ceramics allowed for new forms and designs, and the movement had a
significant impact on architecture, interior design, and the decorative arts.
Some of the most notable practitioners of the Art Nouveau style include the French artist and designer
Émile Gallé, the Czech artist and designer Alphonse Mucha, and the Belgian architect and designer
Victor Horta. Art Nouveau had a significant influence on later design movements, including Art Deco
and the modernist styles that emerged in the early 20th century.
Q.1.11 Temples of many Sikharas are
(A) Indo-Aryan temples (B) Dravidian temples
(C) Besara temples (D) Khajuraho temples
Ans:- (D)
Temple style Indo-Aryan temples Dravidian temples
Description Location: Predominantly found in Location: Predominantly found in
Northern India. Southern India.
Structure: Characterized by a square Gopuram: Large entrance gateways,
sanctum (garbhagriha) with a beehive- known as ‘gopurams,’ are distinctive.
shaped tower (shikhara) above it. They are ornately decorated and often
taller than the temple itself.
Hemant Vilas Parulekar 657

Temple style Indo-Aryan temples Dravidian temples


Description Shikhara: The shikhara is curvilinear Complex Structure: Temples built as
and rises in a convex curve. It’s often complex compounds with multiple
marked by ‘amalaka’ (a ribbed, circular shrines and halls.
stone at the top) and a ‘kalasha’ or Pyramidal Tower: The main sanctum is
‘finial’ at the very top. crowned by a pyramidal tower (vimana),
Decorations: Walls decorated with which is stepped and rises linearly.
sculptures depicting various deities, Enclosures: Multiple concentric
mythological stories, and daily life enclosures (prakaras) with walls and
scenes. gateways.
Mandapa: A pillared hall typically Water Tank: Commonly feature a water
precedes the sanctum for gatherings tank for rituals.
and prayers.
Image

Temple style Besara temples Khajuraho temples


Description Region: Predominantly in Orissa and Location: In Madhya Pradesh, Central
Central India. India.
Blend of Styles: Represents a synthesis Era: Built during the Chandela dynasty
of the Nagara (Indo-Aryan) and (10th-12th centuries).
Dravidian architectural styles. Nagara Style: Exhibit features of Indo-
Shikhara and Vimana: The shikhara Aryan architecture with elaborately
(tower) resembles the Nagara style but carved shikharas.
with certain modifications and may Erotic Sculptures: Famous for their
have elements similar to the Dravidian intricate and explicit sculptures depicting
vimana. various facets of life, including eroticism
Decorations: Richly decorated with and sensuality.
sculptures and motifs, showcasing a Layout: Usually have a square plan with
blend of artistic influences. a series of interconnected structures
including a main shrine, mandapa, and
entrance porch (ardha-mandapa).
Image
658 GATE Architecture and Planning: Comprehensive Question Bank

Q.1.12 Hardness of water is measured in parts per million by weight in terms of


(A) Calcium carbonate (B) Ferrous oxide
(C) Carbon dioxide (D) Magnesium sulphate
Ans:- (A) (Please refer GATE-2003, Q.9 for more information)
Q.1.13 The concept of “Ekistics’ was propagated by
(A) Amos Rapoport (B) Constantine Doxiadis
(C) Buckminster Fuller (D)Adam Hardy
Ans:- (B) (Please refer GATE-2006, Q.25 for more information)
The concept of “Ekistics” was propagated by Constantinos A. Doxiadis, a Greek architect and town
planner.
Q.1.14 Water seal in water closet is used to
(A) prevent foul gases from entering the house through external sewer pipe
(B) prevent water from leaking through the pipe
(C) keep the water closet moist throughout
(D) None of the above
Ans: - (A)
Explanation:- The water seal in a water closet (WC) is a U-shaped trap that retains water to prevent
foul gases and odors from the sewer system from entering the house through the external sewer pipe.
When the WC is flushed, the water flows out and creates a suction force that draws waste from the
bowl and down the drainpipe. The water then refills the trap, creating the water seal once again. This
helps to maintain a sanitary and hygienic environment in the house.
Q.1.15 Which of the following is generally the largest internal source of revenue for Indian municipali-
ties?
(A) Property Tax (B) Stamp duty on property transfer
(C) Conservancy Tariff (D) Water Tariff
Ans:- (B)
Q.1.16 For proper ventilation, the ratio of opening area in a room to the total floor area of the room,
should be
(A)1:6 (B)1:10 (C)1:12 (D)1:20
Ans: - (B) (ECBC 2007 and National Building Code 2016)
Q.1.17 Resemblance of Greco-Roman basilica is evident in a Buddhist
(A) Temple (B) Vihara (C) Chaitya (D) Stupa
Ans: - (C)
Q.1.18 Among the following architect-planner of foreign origin, the only one who is NOT associated
with an Indian city is
(A) Otto Koenisberger (B) Fry Otto (C) Le Corbusier (D) Edwin Lutyens
Hemant Vilas Parulekar 659

Ans: - (B)
Name of the Architect/Planner Planned City
Otto Koenisberger Bhubneshwar
Le Corbusier Chandigarh
Edwin Lutyens New Delhi
Explanation:- Here is the extensive list as following:
Name of the Planned City Description
Architect/Planner
Edwin Lutyens New Delhi The capital city of India, New Delhi, was primarily
planned by two British architects - Edwin Lutyens and
Herbert Baker. They were responsible for the design
and construction of key areas in the city, especially the
administrative and government buildings during the
British colonial period.
Le Corbusier Chandigarh Le Corbusier, a Swiss-French architect, is famously known
for his contribution to planning the city of Chandigarh.
He was assisted by Pierre Jeanneret, Maxwell Fry, and
Jane Drew.
H. K. Mewada and Gandhinagar The capital of the state of Gujarat, Gandhinagar, was
Prakash M Apte. planned and designed in the 1960s by two Indian
architects, H. K. Mewada and Prakash M Apte.
Otto Königsberger Bhubaneswar The capital of Odisha, Bhubaneswar, was designed by
German architect Otto Königsberger in 1946.
Charles Correa, Navi Mumbai Navi Mumbai, a planned satellite city of Mumbai, was
Pravina Mehta, and designed by Charles Correa, Pravina Mehta, and Shirish
Shirish Patel Patel in 1971.
Maharaja Sawai Jai Jaipur Jaipur, one of the earliest planned cities of modern India,
Singh II was planned by Maharaja Sawai Jai Singh II in 1727. The
city’s streets are notable for their width and orderliness.
Anthony Sattin Bilaspur Bilaspur, a city in Chhattisgarh, was planned by Anthony
Sattin.
Julian Kennedy Jamshedpur Jamshedpur, a planned industrial city in Jharkhand, was
Sahlin founded by Jamsetji Tata and primarily planned by Julian
Kennedy Sahlin of Pittsburgh.

British Rule Visakhapatnam The city’s modern layout and development were planned
under the British rule. Post-independence, it has
been developed through various urban development
authorities.
Kempegowda I Bengaluru While the modern city has evolved through various
stages, Kempegowda I, a ruler under the Vijayanagara
Empire, is credited with laying the foundations of
modern Bangalore in the 16th century.
660 GATE Architecture and Planning: Comprehensive Question Bank

Joseph Allen Stein Durgapur This planned city in West Bengal was designed by
and Benjamin Polk Joseph Allen Stein and Benjamin Polk in the 1950s as an
industrial town.
Otto Königsberger Rourkela Planned and built by the German architects Otto
and G.M. Mehl Königsberger and G.M. Mehl, Rourkela was one of the
first planned cities in post-independence India, developed
for the steel plant in the late 1950s.
NOIDA Noida Noida is a systematically planned Indian city under the
management of the New Okhla Industrial Development
Authority. It is part of the National Capital Region of
India.
Scott Wilson Lavasa This private, planned city near Pune was planned by
HOK International Limited and designed by a team led
by Scott Wilson.
Surbana Jurong Amravati The new capital city of the Indian state of Andhra Pradesh,
has been planned by the Singapore government’s urban
planning consultancy, Surbana Jurong.
Kolkata The city was developed largely during the British Raj and
the layout of the modern city was largely influenced by
the British urban planners and architects.
Q.1.19 The most appropriate plan type for comprehensive physical planning of a metropolitan region is
(A)District Plan (B) Action Area Plan (C) Town Planning Scheme (D) Structure Plan
Ans: - (A)
Q.1.20 In network analysis, the amount of time by which the start of an activity may be delayed without
hampering the start of a succeeding activity, is called
(A) Total Float (B) Free Float (C) Time Lag (D) Start Lag
Ans: - (B)
Explanation:-
In the context of project management, network analysis is a crucial technique used for planning,
scheduling, and controlling complex projects. It involves the use of graphical representations (net-
works) to visualize and analyze the relationships between different project activities. Here’s a detailed
look at the attributes and types of network analysis in project management:
Attributes of Network Analysis:
1. Activities and Tasks: These are the basic elements of a project represented as nodes or arcs in a
network diagram. Activities have durations, costs, and resource requirements.
2. Events or Milestones: Represented as nodes, these are significant points in time within the project,
such as the start or completion of key tasks.
3. Precedence Relationships: This attribute defines the sequence in which tasks must be performed.
It dictates the dependencies between various activities.
4. Critical Path: This is the longest path through the network, determining the shortest possible proj-
ect duration. Activities on this path cannot be delayed without delaying the project.
5. Float or Slack: The amount of time that an activity can be delayed without affecting the project
schedule. Activities with zero float are on the critical path.
Hemant Vilas Parulekar 661

6. Lead and Lag Times: Lead time accelerates the successor activity, whereas lag time introduces a
delay. They are used to fine-tune the scheduling.
7. Resource Allocation: Involves assigning resources (labor, materials, equipment) to activities and
managing their optimal use.
8. Early Start and Finish, Late Start and Finish: These are dates calculated for each activity that rep-
resent the earliest and latest times they can start and finish without delaying the project.
Types of Network Analysis:
1. Critical Path Method (CPM):
- Used for projects with well-defined activities and fixed time estimates.
- Focuses on identifying the longest path and critical activities in the project.
2. Program Evaluation and Review Technique (PERT):
- Suitable for projects with a high degree of uncertainty.
- Uses probabilistic time estimates (optimistic, pessimistic, and most likely) for more flexibility.
3. Gantt Charts:
- Provides a visual timeline for the project, showing when activities should start and finish.
- Useful for tracking project progress and communicating schedules.
4. Resource Leveling and Smoothing:
-Techniques used to optimize the distribution of resources and minimize fluctuations in resource
usage.
5. What-if Analysis:
- Examines the impact of potential changes in the project plan.
- Helps in contingency planning and risk management.
Applications in Project Management:
- Project Planning: Establishing a realistic schedule and understanding task dependencies.
- Resource Management: Optimizing the allocation and use of resources.
- Time Management: Identifying critical activities and ensuring timely project completion.
- Risk Management: Anticipating potential delays and planning for contingencies.
- Communication: Aiding in communicating the project plan and progress to stakeholders.
Network analysis in project management is an essential technique for breaking down complex projects
into manageable tasks, understanding interdependencies, and ensuring efficient project execution.
Q.1.21 The most common method of protecting or preserving timber is
(A) Smoke drying (B) Salt Seasoning (C) Dry Seasoning (D) Creosoting
Ans: - (C)
Explanation:- (A) Smoke drying is a traditional method of timber preservation that involves the use
of smoke to dry out the timber. The smoke is generated by burning wood chips or sawdust, which
produces a dense smoke that is circulated around the timber. This smoke helps to dry out the timber
and contains compounds that act as natural preservatives. Smoke drying is a slow process and can
take several weeks to complete. This method is typically used for smaller pieces of timber and is not as
commonly used today as it once was.
(B) Salt seasoning, also known as brine treatment, is a method of timber preservation that involves
soaking the timber in a solution of water and salt. The salt water is absorbed by the timber and helps
to kill off any fungi or bacteria that may be present. This method is commonly used for larger pieces
of timber and can be completed relatively quickly. However, it is not as effective as other methods of
timber preservation and is more commonly used for temporary structures.
662 GATE Architecture and Planning: Comprehensive Question Bank

(C) Dry seasoning, also known as air drying, is a method of timber preservation that involves allowing
the timber to dry naturally in the open air. This process can take several months to a year, depend-
ing on the climate and humidity levels. Dry seasoning is one of the most commonly used methods
of timber preservation and is effective at preventing decay and insect infestation. However, it can be
time-consuming and may not be practical for larger pieces of timber.
(D) Creosoting is a method of timber preservation that involves the use of creosote oil, a tar-like sub-
stance derived from coal tar. The timber is immersed in the creosote oil, which penetrates the wood
fibers and acts as a preservative, protecting the timber from decay and insect infestation. Creosoting
is an effective method of timber preservation and is commonly used for railroad ties, utility poles, and
marine applications. However, it has some environmental concerns due to the toxicity of the creosote
oil and is regulated in many countries.Top of FormBottom of Form
Q.1.22 Codex Atlanticus, a book propagating a new concept in urban planning, was authored by
(A) Leonardo da Vinci (B) Biaggio Rossetti
(C) Michelangelo (D) Leon Battista Alberti
Ans: - (A)
Explanation:- Codex Atlanticus is a collection of writings and drawings by Leonardo da Vinci,
Q.1.23 The book ‘Design with Nature’ was written by
(A) Lawrence Halprin (B) Frederick Law Olmsted
(C) Ian Mc Harg (D) Andre Le Notre
Ans: - (C) (This question was asked as Q.49, GATE-2014)
Q.1.24 The extreme limit to which a body can be repeatedly strained without Fracture or permanent
change of shape, is known as
(A) Compressibility (B) Resiliency (C) Density (D) Elasticity
Ans: - (D)
Explanation:- The extreme limit to which a body can be repeatedly strained without fracture or
permanent change of shape is known as elasticity.
Q.1.25 The locus of a fixed point outside a circle rotating on a fixed straight line is called
(A) Superior Trochoid (B) Hypocycloid (C) Epicycloid (D) Involute
Ans: - (A)
Explanation:-
Term Superior Trochoid Hypocycloid
Description A superior trochoid is a geometric curve A hypocycloid is a geometric curve
that is formed by the motion of a point generated by the motion of a point on a
that is a fixed distance above a circle as the small circle rolling inside a larger circle.
circle rolls along a straight line. The path
traced by the point is called a superior
trochoid.
Diagram
Hemant Vilas Parulekar 663

Term Epicycloid Involute


Description An epicycloid is a plane curve that is In mathematics, an involute is a curve that
generated by tracing out a point on is created by unwrapping a taut string or
the circumference of a circle as it rolls wire from another curve. Specifically, an
along the outside of another fixed involute is the path taken by the end of the
circle, usually without slipping. string as it is unwound from a given curve,
keeping the string taut.
Diagram

Questions 2.1-2.25 carries two marks each


Q.2.1 Which of the following is the closest approximation to a land of area measuring 1.5 acres?
(A) 1500 sq m. (B) 3035 sq m. (C) 6060sq m. (D) 6070 sq m.
Ans: - (D)
Explanation:- 1 acre is equal to 4046.86 square meters, so 1.5 acres is equal to 6067.29 square meters
(approximately).
Q.2.2 Two critical activities A and B need 5 and 8 days respectively, to complete. Another non-critical
activity C needs 5 days to complete with a free float of 2 days. If C is completed in 4 days now, how
much compression in project time is achieved?
(A) 0 days (B) l day (C) 2 days (D) 3 days
Ans: - All are correct
Q.2.3 In a seminar room of area 200 sq.m,4 m height and total absorbing power of 120 m2 Sabines, what
is the reverberation time?
(A) 0.24 secs. (B) 1.06 secs (C) 1.52 secs (D) 4.16 secs
Ans: - (B)
Solution:- The formula for calculating the reverberation time is:
T = 0.16V/A
Where: T = reverberation time in seconds V = volume of the room in cubic meters A = total absorp-
tion of the room in sabins
Converting the given values to SI units, we have: V = 200 x 4 = 800 m3 A = 120 sabins = 120 m2
Substituting the values in the formula, we get: T = 0.16 x 800 / 120 = 1.07 seconds
Q.2.4 A town has a basic employment of 25,000 workers. If the basic: non basic ratio is 1:2.5 and the
workers dependency ratio is 1:4, what is the population size of the town?
(A) 2,50,000 (B) 4,37,500 (C) 3,50,000 (D) 3,12,500
664 GATE Architecture and Planning: Comprehensive Question Bank

Ans: - (B)
Explanation:- Basic employment = 25,000
Non-Basic employment = 62,500
Total Workers = 87,500
Total Dependent population = 87,500 x 4 = 3,50,000
Total population = 3,50,000 + 87,500 = 4,37,500
Q.2.5 For large and closed buildings, the fire hydrants should be located at distance of
(A) 90-120 meters (B) 50-80 meters (C) 120-150 meters (D) 150-300 meters
Ans: - (B)
Explanation:- For large and closed buildings, the fire hydrants should be located at a distance of 50-80
meters. This ensures that the water can reach all parts of the building in case of a fire. Hydrants should
also be placed along the access roads and at other strategic locations, as required by local fire codes
and regulations.
Q.2.6 What is the rate of ventilation due to wind action if the free area of the window is 1 sq.m., and the
wind speed is l m/hr. Assume the wind to be perpendicular to the window.
(A) 1.0 cu.m/hour (B) 0.6 cu.m/hour (C) 0.3 cu.m/hour (D) 0.1 cu.m/hour
Ans: - (B)
Solution:- The rate of ventilation due to wind action is given by:
Q = C_d A V
where Q is the ventilation rate, A is the free area of the window, V is the wind speed, and C_d is the
discharge coefficient.
Assuming a discharge coefficient of 0.6 for a perpendicular wind on a standard window, and substi-
tuting the given values, we get:
Q = 0.6 x 1 x 1 = 0.6 cubic meters per hour
Therefore, the rate of ventilation due to wind action is 0.6 cubic meters per hour (Option B).
Q.2.7 The diagram below shows the relative distribution of different types of housing within a total res-
idential area of 150 hectares. If the net density of the plotted housing area is 350 ppha, how many
people will be accommodated there?
(A) 20,000-25,000 (B) 25,000-30,000 (C) 30,000-35,000 (D) 35,000-40,000
Ans: - (C)
Explanation:- Total residential area = 150 Ha
Builders’ group housing = 1/4th area = 150/4 = 37.50 Ha
Co-Op housing = ½ of builders’ group housing = ½ * 37.50 = 18.75 Ha
Thus, plotted housing area = 150-37.50-18.75 = 93.75 Ha
Given, Net density of plotted housing = 350 ppha
Thus, population of plotted housing = 350 x 93.75 = 32,812.50
Q.2.8 As per 1991 Census, the urban component of India’s total population was between
(A) 10% and 20% (B) 20% and 30% (C) 30%and 40% (D) 40% and 50%
Ans: - (C)
Hemant Vilas Parulekar 665

Q.2.9 Zinc coating is given over the steel reinforcement to


(A) increase tensile strength (B) reduce bending capacity
(C) reduce corrosion (D) increase bond strength
Ans: - (C)
Explanation:- Zinc coating is given over the steel reinforcement to reduce corrosion. The zinc coating
provides a sacrificial layer that corrodes first, protecting the steel reinforcement from rust and corro-
sion.
Q.2.10 Which of the following sequence of names constitute the botanical name for identification and
use of landscape plants?
(A) Species and order (B) order and genera
(C) family and species (D) genera and species
Ans: - (D)
Explanation:- The genus is a group of closely related species, and the species is the basic unit of clas-
sification. Together, the genus and species make up the scientific name of a plant. For example, the
scientific name of a rose is Rosa (genus) × damascena (species).
Q.2.11 For a four-way road intersection the following alternative traffic Management schemes are pro-
posed.
(i) signalized intersection; (ii) rotary intersection; (iii) manually controlled intersection:
Considering the above alternatives, which of the following statement is INCORRECT?
(A) All are equally space consuming
(B) ‘i’ is more power consuming than ii’ and ‘iii’
(C) ‘ii’ is more power consuming than ‘i’ and ‘iii’
(D) ‘iii’ is less power and manpower consuming than ‘i’ and ‘ii’
Ans: - (A)
Explanation:- This is not necessarily true, as the space required for each type of intersection can vary
depending on factors such as the size of the intersection, the number of lanes, and the approach angles
of the roads. For example, a large roundabout or rotary intersection may require more space than a
signalized intersection with a small footprint.
Q.2.12 A residential plot of 20 meter frontage and 25 meter depth is governed by the development regula-
tions of maximum F.A.R. of 200 and maximum plot coverage of 50%. Upto what maximum height
can the plot be built?
(A) 2 floors (B) 3 floors (C) 4 floors (D) 10 floors
Ans: - (C)
Solution:- To determine the maximum height of the building that can be constructed on the plot, we
need to first calculate the maximum allowable floor area and the maximum allowable footprint area.
Maximum allowable floor area = 200% of plot area = 2 x 20 x 25 = 1000 square meters Maximum
allowable footprint area = 50% of plot area = 0.5 x 20 x 25 = 250 square meters
Assuming a uniform floor-to-floor height of 3 meters, the maximum height of the building can be
calculated as follows:
Maximum height = Maximum allowable floor area / Maximum allowable footprint area x Floor-to-
floor height = 1000 / 250 x 3 = 12 meters
Therefore, the maximum height of the building that can be constructed on the plot is equivalent to 4
floors. The answer is (C) 4 floors
666 GATE Architecture and Planning: Comprehensive Question Bank

Q.2.13 Minimum visibility distance at a major road intersection for a design speed of 80 kmph is
(A) 200 meters (B) 180 meters (C) 80 meters (D) 100 meters
Ans: - (B)
Explanation:- The minimum visibility distance at a major road intersection is generally governed
by the design speed of the road. For a design speed of 80 kmph, the minimum visibility distance is
typically calculated as:
Minimum visibility distance = 2 * Design speed
Substituting the value of design speed as 80 kmph, we get:
Minimum visibility distance = 2 * 80 = 160 meters
Therefore, the minimum visibility distance at a major road intersection for a design speed of 80
kmph is 160 meters, which is closest to option (B) 180 meters.
Q.2.14 For accumulation of Re.1/- for n years at a given rate of compound interest ‘i’, the annual sinking
fund is equal to
(A) 1/i (B) (1+i)n (C) {(1+i)n-1}/i (D) i/{(1+i)n-1}
Ans: - (D)
Explanation:- The formula for the annual sinking fund is given by:
Annual sinking fund = P x i / {(1+i)n - 1}
where P is the principal amount, i is the rate of interest per period, and n is the number of periods.
To accumulate Re.1/- for n years at a given rate of compound interest i, the principal amount P would
be Re.1/-. Therefore, the formula for the annual sinking fund in this case becomes:
Annual sinking fund = (Re.1/-) x i / {(1+i)n - 1}
Simplifying further, we get:
Annual sinking fund = i / {(1+i)n - 1}
Therefore, the correct answer is (D) i/{(1+i)n-1}.
Q.2.15 Which of the following parametric conditions will provide the most suitable land for intensive
development of settlement?
(A)slope~4%: soil silty loam aggregate; depth of water table>6 meters: vegetation~moderate
(B)slope~20%; soil aggregate sand; depth of water table=30 meters. vegetation barren
(C)slope~2%; soil clay; depth of water table l meter; vegetation moderate
(D)slope~10%; soil sandy loam; depth of water table=15 meters: vegetation dense
Ans: - (A)
Explanation:- The most suitable land for intensive development of settlement would be the one that
has a gentle slope, deep water table, and fertile soil with moderate vegetation. Therefore, option (A)
seems to be the most suitable since it has a gentle slope, good soil quality with moderate vegetation,
and a deep-water table.
Q.2.16 A rectangular room (internal dimensions 5m x 3m) is made of 250mm walls. Calculate the volume
of concrete needed for 25 mm Damp proof Course.
(A) 0.425 cu.m. (B) 4.25 cu.m. (C) 0.4 cu.m. (D) 4.0 cu.m.
Hemant Vilas Parulekar 667

Ans: - No option is correct


Solution: Outer Area = (5.00+0.25+0.25) * (3.00+0.25+0.25) = 5.50*3.50 = 19.25 sq.m.
Inner Area = 5.00*3.00 = 15.00
Net area = 19.25 – 15.00 = 4.25 sq.m.
Volume of DPC = 4.25 * 0.025 = 0.10625 cu.m
Q.2.17 With the following given data the velocity of undisturbed water flow in a rectangular open drain
channel is equal to (Given data: coefficient of roughness = 0.11; liquid width = 1 meter; Depth of
liquid = 60 cm; and slope = 1 in 200)
(A) 43 meters per sec (B) 38 meters per sec
(C) 31 meters per sec (D) 27 meters per sec
Ans: - (D)
Solution:- To calculate the velocity of undisturbed water flow in a rectangular open drain channel, we
can use the Manning’s formula:
V = (1 / n) * (R2/3) * (S1/2)
Where V is the velocity, n is the coefficient of roughness, R is the hydraulic radius, and S is the slope.
The hydraulic radius for a rectangular channel is given by:
R=A/P
Where A is the cross-sectional area of the channel, and P is the wetted perimeter.
Given the data, we can calculate the cross-sectional area and wetted perimeter as follows:
• Cross-sectional area = liquid width * depth of liquid A = 1 * 0.6 = 0.6 sq. m
• Wetted perimeter = liquid width + 2 * depth of liquid P = 1 + 2 * 0.6 = 2.2 m
The hydraulic radius is therefore:
R = A / P = 0.6 / 2.2 = 0.2727 m
The slope is given as 1 in 200, which is equivalent to a slope of 0.005.
Substituting the values into Manning’s formula, we get:
V = (1 / 0.11) * (0.27272/3) * (0.0051/2) = 1.13 m/s (approximately)
Therefore, the velocity of undisturbed water flow in the rectangular open drain channel is approxi-
mately 1.13 meters per second. Answer (D) 27 meters per sec in the options is incorrect.
Q.2.18 The term ‘gentrification’ refers to
(A)Migration of fresh population into a crowded urban area.
(B)Settling of population in a new urban area.
(C)Settling of military forces in an urban area.
(D)Evacuation of population from a rural area.
Ans: - (B)
Explanation:- The term ‘gentrification’ refers to the process of renovating or improving a deteriorated
urban neighborhood, often by means of the influx of more affluent residents. This process can lead to
the displacement of the original, often lower-income, residents and businesses of the neighborhood.
Therefore, the correct option is
668 GATE Architecture and Planning: Comprehensive Question Bank

Q.2.19 A tree of 13 meters height is required to shade the entire southern wall of a building of 4 meters
height. At a solar altitude of 45, what should be the maximum distance of a tree from the building
wall?
(A) 4 meters (B) 9 meters (C) 13 meters (D) 17 meters
Ans: - (B)
Solution:- Assuming that the tree is upright and perpendicular to the ground, we can use the follow-
ing proportion:
height of tree / distance of tree = height of building / distance of building
Plugging in the values we have:
13 / distance of tree = 4 / 1 (since the building height is given as 4 meters)
Solving for the distance of the tree:
distance of tree = 13 / 4 = 3.25 meters
However, this calculation assumes that the tree is located at the same level as the building. To shade
the entire southern wall of a building, the tree should be located far enough away from the building so
that the sun’s rays at a solar altitude of 45 degrees are blocked by the tree.
At a solar altitude of 45 degrees, the angle between the sun’s rays and the ground is also 45 degrees
(since the sun’s rays are perpendicular to the solar altitude). The height of the tree creates a right trian-
gle with the height of the building and the distance from the building. The angle opposite the height of
the building is 45 degrees, and we know that the height of the building is 4 meters. Using trigonome-
try, we can solve for the distance from the building:
tan(45) = height of tree / distance from building + height of building 1 = 13 / (distance from building
+ 4) distance from building + 4 = 13 distance from building = 9 meters
Therefore, the maximum distance of the tree from the building wall is 9 meters. Answer: (B) 9 meters.
Q.2.20 The Dampers placed in the air conditioning duct are provided to control the
(A) Velocity and volume of air (B) Exhaust air and velocity
(C) Foul air and exhaust air (D) Volume of air and foul air
Ans: - (A)
Q.2.21 A site map drawn to scale 1:10,000 shows six contour lines at 5 meters contour interval. The high-
est contour elevation is 250 meters. The average distance between the highest and the lowest con-
tour lines on the map is 2.1 cms. What is the average slope between the highest and the lowest
contour elevations?
(A) 1 in 25 (B) 1 in 21 (C) 1 in 30 (D) 1 in 7
Ans: - (D)
Solution:- The vertical distance between two consecutive contour lines on the map is 5 meters. So,
the vertical distance between the highest and the lowest contour lines on the map is 5 meters × 5 = 25
meters.
Let the horizontal distance between the highest and the lowest contour lines on the map be x cm.
Then, using the scale of 1:10,000, we can convert this to meters as:
x cm × 10,000 cm/m = x × 100 m
Hemant Vilas Parulekar 669

We can now use the slope formula:


slope = rise/run = vertical distance / horizontal distance
Substituting the values we have:
slope = 25 / (x × 100)
We are given that the average distance between the highest and the lowest contour lines on the map
is 2.1 cm. So,
x = 2.1 cm
Substituting this in the equation for slope:
slope = 25 / (2.1 × 100) = 1 / 8.4
To express this as a ratio, we can invert the slope and multiply by 100:
slope ratio = 8.4 / 1 × 100 = 840
So the average slope between the highest and the lowest contour elevations is 1 in 840. Therefore, the
answer is (D) 1 in 7 (since 1 in 840 is equivalent to 1 in 7.14, which is closest to 1 in 7).
Q.2.22 One cm square area on a map represents 9 hectares. What is the scale of the map?
(A) 1:300 (B) 1 cm=30 meters (C) 1:30000 (D) None of the above
Ans: - To determine the scale of the map, we need to know the conversion factor between the map
distance and the actual ground distance.
Here, we are given that one cm2 area on the map represents 90,000 m2 on the ground.
First, we need to find the length of one side of the square on the map that represents 90,000 m2 on the
ground:
sqrt(90,000) = 300 meters
So, the map distance represented by one side of the square is 300 meters.
Now, we can determine the scale of the map as follows:
1 cm on the map represents 300 meters on the ground
100 cm (1 meter) on the map represents 300 x 100 = 30,000 meters on the ground
1:30,000 is the scale of the map.
Therefore, the correct answer is (C) 1:30,000.
Q.2.23 If the original cost of the building is Rs 1,00,000/-, scrap value is Rs 1,000/- and the life of the
property is 99 years, The Annual Depreciation calculated by Straight line Method is
(A) 100 (B) 1000 (C) 90 (D) 999
Ans: - (B)
Solution:- The annual depreciation calculated by the straight-line method can be found using the
formula:
Annual Depreciation = (Original Cost - Scrap Value) / Life of Property
Substituting the given values, we get:
Annual Depreciation = (1,00,000 - 1,000) / 99
= 99,000 / 99 = 1,000
670 GATE Architecture and Planning: Comprehensive Question Bank

Q.2.24 Loam category in soil texture is represented by


(A) sand:20;silt:50;clay:30 (B) sand:30;silt:40;clay:30
(C) sand:60;silt:20;clay:20 (D) sand:40;silt:40;clay:20
Ans: - (D)
Q.2.25 Which of the following operations should be the first one for setting up plane table survey?
(A) Orientation (B) Leveling (C) Centering (D) Resection
Ans: - (B)
GATE QUESTION PAPER 2000
General Aptitude (GA)

Only MCQs have been covered for this paper


Q. 1 – Q. 20 carry one mark each.
Q.1.1 J.N.U., New Delhi campus has been designed by
(A) A.P.Kanvinde (B) Louis I.Kahn (C) C.P.Kukreja (D) J.A.Stein
Ans:- (C)
Q.1.2 Occupancy rate refers to
(A)number of buildings per unit area. (B)number of persons per habitable rooms.
(C)number of habitable rooms per acre. (D)number of persons working in an office.
Ans:- (B)
Q.1.3 The new Guggenheim Museum in Bilbao, spain was designed by
(A) Frank Gehry (B) F. L. Wright (C) Roger Anger (D) Norman Foster
Ans: - (A)
Explanation: - Please refer GATE-2015, Q. 28 for more information.
Q.1.4 Vidhyadhar nagar was planned by
(A) H.K.Mewada (B) Vidhyadhar Bhattacharya (C) B.V.Doshi (D) Charles Correa
Ans:- (C)
Explanation: - Balkrishna Doshi was commissioned to design and plan the satellite city of Vidyadhar
Nagar in Jaipur in the 1970s. The project was initiated by the government of Rajasthan to address
the growing population and urbanization pressures in Jaipur, and completed in 1984. Doshi worked
with a team of planners and architects to design a comprehensive master plan for the city that would
prioritize pedestrian movement, green spaces, and a mix of residential, commercial, and institutional
areas.
The plan of Vidyadhar Nagar is characterized by a series of sectors or neighborhoods, each with its
own open spaces, community facilities, and residential units. The neighborhoods are connected by a
network of roads and pedestrian pathways that allow for easy movement and access to amenities. The
design also incorporates several public parks and green spaces, which help to improve the quality of
life for residents and provide a respite from the urban environment.
Q.1.5 For Indian Metropolitan cities, the quantity of solid waste accumulation per head per day is
approximately
(A) 0.5Kg (B) 1.5Kg (C) 2.5Kg (D) 3.5Kg
Ans:- (A)
Explanation: - As per the Manual of Municipal Solid Waste Management by the Ministry of Urban
Development (MoUD), the average quantity of solid waste generation in Indian metropolitan cities
is approximately 0.5 to 0.6 kg per capita per day. This value may vary depending on the city’s size,
population, and other factors. Therefore, option (A) 0.5 kg is the most appropriate answer.
672 GATE Architecture and Planning: Comprehensive Question Bank

Q.1.6 The maximum gradient of a ramp leading to a car park is


(A) l in 5 (B) l in 10 (C) l in 15 (D) l in 20
Ans:- (B)
Explanation: - As per NBC 2016, the maximum gradient of a ramp leading to a car park is 1 in 10
(B). This means that for every 10 units of horizontal distance, the ramp can rise no more than 1 unit
vertically.
Q.1.7 ‘Less is Bore’is propagated by
(A) Eero Saarinen (B) Philip Johnson (C) Robert Venturi (D) Joseph Paxton
Ans:- (C)
Explanation: “Less is Bore” is a design philosophy propagated by Robert Venturi. This phrase is a
playful twist on the modernist design mantra “less is more,” and it reflects Venturi’s postmodernist
architectural approach, which embraces complexity, diversity, and a departure from the strict
minimalism of modernism. Robert Venturi is known for his influential book “Complexity and
Contradiction in Architecture” and his architectural designs that challenge the simplicity and purity
of modernist architecture.
Q.1.8 Turbidity of water is due to
(A) Algae (B) Fungi (C) Organic salt (D) Suspended matters.
Ans: - (C)
Explanation: - Please refer GATE-2003, Q. 9 for more information
Q.1.9 The only architect-president of a nation was
(A) Richard Nixon (B) F. Marcos (C) Thomas Jefferson (D) L.B. Johnson
Ans:- (C)
Explanation: - The only architect-president of a nation was Thomas Jefferson, who served as the
third president of the United States from 1801 to 1809. Jefferson was a polymath, with interests in
architecture, science, politics, and philosophy. He was heavily influenced by the classical architecture
of ancient Greece and Rome and is known for designing several important buildings in the United
States.
One of Jefferson’s most famous architectural works is his design for the Virginia State Capitol building
in Richmond, Virginia. Jefferson’s design was based on the Maison Carrée, a Roman temple in southern
France, and features a neoclassical facade with a central portico supported by six Ionic columns. The
building was completed in 1788 and is still in use today.
Jefferson was also responsible for the design of his own home, Monticello, which is in Charlottesville,
Virginia. Monticello is a neoclassical mansion with a domed roof, and features several innovative
design elements, such as skylights, a revolving bookstand, and a dumbwaiter.
In addition to his architectural accomplishments, Jefferson was also a major political figure in the
early history of the United States. He was the principal author of the Declaration of Independence, and
is remembered as a champion of individual liberty, democracy, and religious freedom.
Hemant Vilas Parulekar 673

Q.1.10 The Pruitt housing project in St. Louis failed because of


(A) natural calamity (B) structural failure
(C) un-affordability (D) functional inaccessibility
Ans:- (D)
Explanation:- The Pruitt-Igoe housing project in St. Louis, Missouri is widely regarded as a failure
due to a combination of factors, including social and economic issues. The development was built
in the 1950s as a large-scale public housing project intended to provide affordable housing for low-
income families. However, the project was plagued by problems from the outset, including a lack of
maintenance and management, inadequate funding, and a concentration of poverty.
The living conditions at Pruitt-Igoe deteriorated rapidly, and the complex became known for its high
rates of crime and violence. This, coupled with the rise of suburbanization in the 1950s and 1960s, led
to a decline in occupancy rates and a sense of isolation from the surrounding community. In 1972,
the complex was demolished, just 18 years after it was built, in a dramatic event that was broadcast on
national television.
While there were certainly issues with the design and construction of the complex, such as the lack of
elevators and the decision to build it in a high-crime area, the failure of Pruitt-Igoe is often seen as a
symptom of deeper social and economic problems, including racial segregation, poverty, and a lack of
investment in urban areas.

Q.1.11 To ensure comfort condition inside the room, the temperature and relative humidity values should
preferably be
(A) 20o C and 65% respectively. (B) 15o C and 45% respectively.
(C) 25o C and 50% respectively (D) 30o C and 65% respectively.
Ans: - (C)
Explanation:- The preferred temperature and relative humidity values for comfort inside a room can
vary depending on the specific context and individual preferences. However, as a general guideline, a
temperature range of 20-25 degrees Celsius (68-77 degrees Fahrenheit) and a relative humidity range
of 30-60% are comfortable for most people. Therefore, option (C) - 25o C and 50% relative humidity
- is a suitable answer. It is worth noting that these values may vary depending on factors such as the
season, the activity being performed in the room, and the personal preferences of the occupants.
Q.1.12 Intensity of colour refers to
(A) brightness (B) darkness (C) pigment density (D) quantity
Ans: - Please refer GATE-2015, Q. 19 for more information.
Q.1.13 As per the National Building Code the minimum area of a habitable room is
(A) 8.5 sq m. (B) 9.5sq m (C) 10.5sq m (D) 11.5 sq m
674 GATE Architecture and Planning: Comprehensive Question Bank

Ans:- (B)
Explanation: - As per the National Building Code of India (NBC), the minimum area of a habitable
room is 9.5 square meters (102.25 square feet) with a minimum width of 2.75 meters (9 feet) and a
minimum height of 2.4 meters (8 feet).
Q.1.14 The most commonly used disinfectant for purification of municipal water is
(A) boric powder (B) alum (C) bleaching powder (D) camphor
Ans:- (C)
Explanation: It is used to kill or deactivate harmful microorganisms such as bacteria, viruses, and
protozoa that may be present in the water.
Q.1.15 In completion of a project, Critical path is one which requires
(A) maximum time (B) minimum time (C) optimum time (D) critical time
Ans:- (A)
Explanation: - Please refer GATE-1993, Q. 20 for more information.
Q.1.16 With every doubling of distance from source the noise level will reduce by
(A) 6dbA (B) 8dbA (C) 10 dbA (D) 12 dbA
Ans: - (A)
Explanation: - Please refer GATE-1993, Q. 20 for more information.
Q.1.17 Indicator of poverty line denotes
(A) Persons having no shelter (C) Per capita calorie consumption of food
(B) Level of family income (D) none of the above
Ans:- (A)
Q.1.18 The situation which provides the most intimate scale to an observer is, while
(A) 9 m wide road in front of one storey buildings.
(B) 12 m wide road in front of two storey buildings
(C) 15 m wide road in front of three storey buildings
(D) 21 m wide road in front of four storey buildings
Ans: - (A)
Q.1.19 The concept of Greek town planning emphasized on
(A) large ‘size (C) human scale (B) more open space (D) compact development
Ans: - (C)
Q.1.20 “Habit’of plants refers to
(A) growth rate (B) branching pattern (C) foliage (D) life span
Ans: - (B)
Explanation:- The term “habit” in plants refers to their general growth form or architecture. It
includes characteristics such as the size, shape, branching pattern, and orientation of leaves, stems,
and branches. The habit of a plant can be affected by various factors such as genetics, environmental
conditions, and management practices. Examples of different plant habits include trees, shrubs, vines,
herbs, and grasses.
GATE QUESTION PAPER 1999
General Aptitude (GA)

Questions of sketches and some questions of numericals are excluded.


Q. 1 – Q. 25 carry one mark each.
Q.1.1 Peristyle in architecture means
(A) a row of free-standing columns surrounding an area
(B) perimeter wall or an enclosed shrine
(C) perishable materials in buildings
(D) the triangular part above the entablature in the classic order
Ans:- (A)
Explanation: Please refer GATE 2003, Q. 7 for more information
Q.1.2 Washington D.C. is an example of
(A) linear urban form (B) star shaped urban form
(C) poly centered net urban form (D) the sheet urban form
Ans:- (B)
Q.1.3 SON lamps operate on the principle of discharge in
(A) Sodium vapour (B) Krypton vapour
(C) Mercury vapour (D) Zinc and Cadmium vapour
Ans:- (A)
Explanation: - SON lamps, also known as high-pressure sodium lamps, operate on the principle of
discharge in sodium vapor. When an electric current is passed through the lamp’s sodium vapor, it
produces a bright yellow light.
Q.4 A method of control survey in which a network of triangles is used is
(A) Triangulation (B) Three-point resection
(C) Trilateration (D) None of these
Ans:- (A)
Explanation: - Triangulation is a method of control survey in which a network of triangles is established
by measuring the angles between pairs of control points. The sides of the triangles are then measured
to provide accurate positions for the points involved in the survey. Triangulation is commonly used in
geodetic surveys for mapping large areas of land.
Q.5 Hyperbolic paraboloid can be generated by
(A) a curve moving over two straight lines at obtuse angles.
(B) a straight line moving over a curve at acute angle.
(C) a curve moving over two other parallel curves
(D) a straight line moving over two other straight lines at an angle to one another.
676 GATE Architecture and Planning: Comprehensive Question Bank

Ans: - (C)

Q.1.6 “Savannas” are


(A) grasslands with drought-resistant trees (B) parts of arctic region with moving glaciers
(C) estuaries, where delta is formed (D) parts of the desert with perennial water pockets.
Ans:- (A)
Explanation: Savannas are a type of grassland characterized by a mixture of grasses and drought-
resistant trees, such as acacias and baobabs. They are typically found in tropical or subtropical regions
with a dry season and a rainy season. The grasses in savannas are adapted to survive periodic droughts,
while the trees have deep roots that allow them to access water from the soil during the dry season.
Savannas are home to a wide variety of animals, including elephants, lions, zebras, and giraffes. They
also play an important role in the global carbon cycle, serving as a carbon sink due to their large
amounts of grass and trees.

Q.1.7 Variability of project duration in PERT analysis is measured in terms of


(A) pessimistic time difference (B) optimistic time difference
(C) time difference of activities (D) square of standard deviation of activity duration
Ans:- (D)
Explanation: The variability of project duration in PERT (Program Evaluation and Review Technique)
analysis is measured in terms of the square of the standard deviation of activity duration. This is
because PERT uses a probability distribution for activity durations, and the standard deviation of that
distribution provides a measure of the uncertainty or variability in the activity duration. By squaring
the standard deviation, we obtain a measure of the variability of the project duration, which considers
the variability of all the activities in the project network.
Q.1.8 The term Necropolis refers to
(A) small size metropolis (B) the new metropolis
(C) dead city (D) the city in space
Ans:- (C)
Explanation: The term Necropolis refers to a large and elaborate cemetery or burial ground, often
featuring elaborate tomb monuments and mausoleums. The word comes from the Greek words
“nekros” meaning “dead” and “polis” meaning “city.” Necropolises were often associated with ancient
civilizations and were seen as a way of honoring and remembering the dead. Some of the most famous
necropolises include the Valley of the Kings in Egypt and the Catacombs of Rome.
Hemant Vilas Parulekar 677

Q.1.9 Phenomenon of contorted growth of trees due to unequal irradiation of light on two sides is known as
(A) Photosynthesis (B) Phototropism
(C) Photoperiodism (D) Photorespiration
Ans:- (C)
Explanation: - The phenomenon of contorted growth of trees due to unequal irradiation of light on
two sides is known as phototropism. Phototropism is the growth or movement of an organism in
response to a light source and is a natural mechanism that helps plants to grow towards a light source
to maximize their ability to photosynthesize. When a plant is exposed to unequal light on two sides, it
will grow towards the side with more light, causing a contorted growth pattern. This phenomenon is
often seen in trees growing in dense forests or on hillsides where the available light is limited.
Q.1.10 The total quantity of runoff of an area of 18 hectares in a lateritic region (runoff coefficient=0.5
and rainfall=10 mm/hr) is
(A) 55 m3/hr (B) 108 m3/h (C) 90 m3/hr (D) 180 m3/hr
Ans: - The formula to calculate the total quantity of runoff is:
Total quantity of runoff = Rainfall intensity x Catchment area x Runoff coefficient
Here, catchment area = 18 hectares = 18,000 m2 (1 hectare = 10000 m2)
Rainfall intensity = 10 mm/hr = 10/1000 m/hr = 0.01 m/hr
Runoff coefficient = 0.5
Putting the values in the formula:
Total quantity of runoff = 0.01 x 18000 x 0.5 = 90 m3/hr
Therefore, the total quantity of runoff of an area of 18 hectares in a lateritic region is 90 m3/hr. Answer:
(C) 90 m3/hr
Q.1.11 The average Lux required on a pavement, having width 4 m, is 8. The mounting height of the
lamp (lumen 2000) is 4 m. The spacing of the lamps (for coefficient of the utilization is 0.5 and
maintenance factor is 0.8) is
(A) 25 m (B) 75 m (C) 10 m (D) 15 m
Solution.: - The illumination level on a surface is given by the formula:
E = (N × L × M × F) / (D2)
where N = number of lamps, L = luminous flux per lamp, M = coefficient of utilization, F = maintenance
factor, D = distance between two adjacent lamps
We need to find the spacing of the lamps, so we can rearrange the formula to get:
D = sqrt ((N × L × M × F) / E)
Given, width of pavement = 4 m Required illumination level = 8 lux Luminous flux per lamp = 2000
lumens Coefficient of utilization = 0.5 Maintenance factor = 0.8 Mounting height of the lamp = 4 m
We can assume a certain number of lamps and calculate the distance between them, and then adjust
the number of lamps based on the spacing requirement.
Let us assume 3 lamps initially, then the distance between two adjacent lamps is:
D = sqrt ((3 × 2000 × 0.5 × 0.8) / 8) = 10 meters
678 GATE Architecture and Planning: Comprehensive Question Bank

The distance is less than the width of the pavement, so we need to increase the number of lamps. Let
us try 5 lamps:
D = sqrt ((5 × 2000 × 0.5 × 0.8) / 8) = 7.07 meters
The distance is less than the width of the pavement again, so we need to increase the number of lamps.
Let us try 7 lamps:
D = sqrt ((7 × 2000 × 0.5 × 0.8) / 8) = 5.77 meters
This distance is less than the width of the pavement, so we need to increase the number of lamps again.
Let us try 9 lamps:
D = sqrt ((9 × 2000 × 0.5 × 0.8) / 8) = 4.47 meters
This distance is slightly less than the width of the pavement, but it is close enough. So, we can use 9
lamps with a spacing of approximately 4.5 meters between them. Therefore, the answer is option (D)
15 m, which is the spacing between two adjacent lamps.
Q.1.12 In BOT based project, the most important evaluation criteria is
(A) financial internal rate of return (B) internal rate of return
(C) benefit-cost ratio (D) present value
Ans: - (B)
Explanation:- In a BOT (Build-Operate-Transfer) based project, the most important evaluation
criteria is typically the internal rate of return (B) because it provides a measure of the profitability
of the project, considering the time value of money. However, the other evaluation criteria listed
(financial internal rate of return, benefit-cost ratio, and present value) are also important and may be
used in conjunction with the internal rate of return to fully evaluate the project.
Q.1.13 Addition of Decibel levels 92 dBA and 88 dBA amounts to
(A) 97dBA (B) 150dBA (C) 180dBA (D) 93dBA
Ans: - (D)
Solution: - When adding two sound levels with different intensities, the formula to use is:
L_total = 10 log (10(L1/10) + 10(L2/10))
Using this formula with the given values, we get:
L_total = 10 log (10(92/10) + 10(88/10)) = 10 log (6.309 + 12.589) = 10 log 18.898 = 10 x 1.276 = 12.76 dB
However, the decibel scale is logarithmic, so we need to convert the result back to the linear scale
before converting it back to dB:
L_total_lin = 10(L_total/10) = 10(12.76/10) = 18.93
Finally, we convert the result back to the decibel scale:
L_total_dB = 10 log (L_total_lin) = 10 log (18.93) = 9.85 + 80 = 89.85 dB
Rounding to the nearest whole number gives us 90 dB. Therefore, the correct answer is (D) 93 dBA.
Q.1.14 The velocity head of water supply line is measured in terms of
(A) m/sec (B) m/sec2 (C) m (D) m2/sec
Hemant Vilas Parulekar 679

Ans: - (C)
Velocity head = v2/2g
Q.1.15 After-image’in visual perception is
(A) same image same colour (B) same image complementary colour
(C) inverted image complementary colour (D) mirror image same colour.
Ans: - (A), (B), (C)
In general, an after-image is an image that continues to appear in one’s vision after the original image
has been removed. This can occur when the eyes continue to send signals to the brain even after the
stimulus has been removed.
If the afterimage appears in a complementary color (i.e., the opposite color on the color wheel) to the
original image, then option (B) would be correct.
However, if the afterimage appears as a negative image (i.e., colors are inverted) of the original image
in the same color as the original image, then option (C) would be correct.
Option (A) could also be correct in certain situations where the afterimage is a direct copy of the
original image with the same colors.
So, depending on the context, any of these options could be correct.
Sub Section A2
Q.2 Match the following from 2.1 to 2.10
(2.1) Distomat (A) Odour
(2.2) Trowel (B) Truss
(2.3) Lamelle (C) Plaster
(2.4) Aziotoc (D) Mud architecture
(2.5) Purlin (E) Survey
(2.6) Adobe (F) Order
(2.7) Jalousie (G) Luminaire
(2.8) La tourette (H) Plumbing
(2.9) Column (I) Window
(2.10) Doric (J) Temple
(K) Le Corbusier
(L) Buckling
(M) Computer
(N) Air conditioning
Ans:- (2.1)-(E), (2.2)-(C),(2.3)-(), (2.4)-(), (2.5)-(B), (2.6)-(D), (2.7)-(I), (2.8)-(K), (2.9)-(L), (2.10)-
(F)
680 GATE Architecture and Planning: Comprehensive Question Bank

Q.19 Match the following from 2.11 to 2.20


(2.11) Express Tower, Bombay (A) Paolo Soleri
(2.12) Semi Conductor Complex, Chandigarh (B) Charles Correa
(2.13) IIT Campus, Kanpur (C) Edward D Stone
(2.14) Aranya Housing project, Indore (D) Jorn Utzon
(2.15) Amsterdam South Plan (E) John Nash
(2.16) Park Crescent, London (F) B. V .Doshi
(2.17) Sydney Opera House (G) H.P. Berlage
(2.18) Cidade de Goa (H) A.P. Kanvinde
(2.19) U S Embassy, New Delhi (I) Joseph Allen Stein
(2.20) ArcoSanti, USA (J) Romi Khosla and associates
(K) Anant Raje
(L) Richard Neutra
Ans: - (2.11)-(I), (2.12)-(J),(2.13)-(H), (2.14)-(F), (2.15)-(G), (2.16)-(E), (2.17)-(D), (2.18)-(B), (2.19)-
(C), (2.20)-(A)
Q.20 Match the following from 2.11 to 2.20
(2.21) Defensible Space (A) Eugene P Odum
(2.22) The Economics of Urban Areas (B) Robert Wenkam
(2.23) Fundamentals of Ecology (C) Arthus B Gallion
(2.24) Design for the Real World (D) Edgar R Neff
(2.25) Urban Pattern (E) Brian Goodall
(F) Oscar Newman
(G) Victor Papanek
(H) Richard Meyer
Ans: - (2.21)-(F), (2.22)-(E),(2.23)-(A), (2.24)-(G), (2.25)-(G),
SubSection A3
Q.3.0 Distinguish between:
Q.3.1 Viewshed and Watershed.
Q.3.2 Dormer Window and Bay Window.
Q.3.3 Economic life and Physical life of building
Q.3.4 Aqueduct and Acquifer
Q.3.5 Ventilation and Air-conditioning.
Q.3.6 Gradient and Camber
Q.3.7 Revolving Fund and Sinking Fund
Q.3.8 Raster images and Vector Images
Q.3.9 Value and Intensity of colour
Q.3.10 Arbitrator and Umpire.
Hemant Vilas Parulekar 681

Ans:- 3.1 Viewshed and Watershed:


Viewshed refers to the area or geographic extent visible from a specific location or viewpoint, whereas
a watershed refers to a geographic region or area that drains into a particular waterway or body of
water. Viewshed analysis is used to determine the visible area from a specific location or viewpoint,
while watershed analysis is used to identify the area contributing water to a particular waterway.
3.2 Dormer Window and Bay Window:
A dormer window is a window that is set vertically on a sloping roof, often with a gabled roof structure
of its own. It provides light and ventilation to the space under the roof. A bay window, on the other
hand, is a window space projecting outwards from the main wall of a building, usually with a seating
or storage area. It is typically comprised of a central window and two flanking windows that are set at
an angle to the wall.
3.3 Economic life and Physical life of building:
The economic life of a building refers to the period during which the building remains economically
viable, i.e., the length of time over which the building generates sufficient income to cover its operating
costs, maintenance costs, and depreciation. The physical life of a building, on the other hand, refers
to the length of time during which the building can remain standing and functional with routine
maintenance and repairs.
3.4 Aqueduct and Aquifer:
An aqueduct is a channel or pipeline used to transport water from one location to another, typically
over long distances. It is a man-made structure that carries water above ground. An aquifer, on the
other hand, is an underground layer of rock, soil, or gravel that contains water and can supply wells
and springs.
3.5 Ventilation and Air-conditioning:
Ventilation refers to the process of exchanging indoor air with outdoor air to improve the air quality
inside a building. It involves the use of mechanical or natural means to introduce fresh air and remove
stale air. Air-conditioning, on the other hand, refers to the process of controlling the temperature,
humidity, and air quality inside a building by means of mechanical equipment. It can be used for both
cooling and heating purposes.
3.6 Gradient and Camber:
Gradient and camber both refer to the slope or curvature of a surface. Gradient refers to the slope
or incline of a road, railway, or other surface, expressed as a percentage. Camber, on the other hand,
refers to the curvature of a road or other surface, where the surface is higher in the middle and slopes
downward towards the edges.
3.7 Revolving Fund and Sinking Fund:
A revolving fund is a financial mechanism used by organizations to finance ongoing operations or
projects. It is a fund in which the money that is repaid is available for lending again. A sinking fund,
on the other hand, is a fund that is set up to accumulate money over time, typically through regular
contributions, to pay off a debt or replace an asset when it reaches the end of its useful life.
3.8 Raster images and Vector Images:
Raster images are made up of individual pixels or dots of color arranged in a grid pattern to form a
complete image. They are resolution dependent and can become pixelated or blurry when enlarged.
Vector images, on the other hand, are made up of lines and curves defined by mathematical equations.
They are resolution independent and can be enlarged without losing quality.
682 GATE Architecture and Planning: Comprehensive Question Bank

3.9 Value and Intensity of color:


The value of a color refers to its brightness or darkness, and it is determined by how much black
or white is added to the color. The intensity of a color, on the other hand, refers to its brightness or
dullness of a color. It describes the amount of light reflected from a surface. The value of a color can be
changed by adding white (creating a tint) or black (creating a shade) to it. Intensity, on the other hand,
refers to the brightness or dullness of a color. It describes the purity or strength of a color, without any
change in value. The intensity of a color can be changed by adding or reducing the amount of gray in it.
3.10 Arbitrator and Umpire: An arbitrator is a neutral third-party who is appointed to settle a dispute
between two parties. They listen to both sides of the argument and then make a binding decision,
which is usually based on the relevant laws and regulations. An umpire, on the other hand, is a referee
who is responsible for making decisions during a sports game or contest. They enforce the rules and
make judgments on issues that arise during the game, such as whether a ball was caught, or a player
was out of bounds.
Section B (Part I)
Q. 4 Draw the following Brick paving patterns.
(A) Running Bond (bricks laid on edge) (B) Herringbone (bricks laid flat)
(C) Basket Weave (bricks laid on edge) (D) Stacked Bond (bricks laid on edge)
(E) Basket weave variation (bricks laid flat)

Q.5 What is a Psychrometric Chart? Draw a typical Psychrometric Chart with appropriate labeling
and explain its explanation.

Q.6 Discuss the salient features of French style garden through illustration.
Ans: - French style gardens are known for their formal and symmetrical design, inspired by classical
architecture and Renaissance ideals. The following are some of the salient features of French style
gardens:
Symmetry: French style gardens are designed with strict symmetry in mind. This means that the
layout of the garden is balanced and harmonious, with each element being mirrored on the opposite
side of a central axis.
Geometric shapes: French style gardens often feature geometric shapes, such as squares, rectangles,
and circles. These shapes are used to create a sense of order and structure in the garden.
Parterres: Parterres are intricate and formal garden beds that are typically laid out in a symmetrical
pattern. They often feature clipped hedges, low borders, and ornamental plants.
Terraces: French style gardens often feature terraces, which are raised platforms that provide an
elevated view of the garden. Terraces can be used to create different levels in the garden, and they
often feature balustrades and ornamental fountains.
Water features: French style gardens often feature water features, such as fountains, ponds, and canals.
These features add a sense of tranquility and calm to the garden, and they often serve as a focal point
for the garden’s design.
Hemant Vilas Parulekar 683

Statues and ornaments: French style gardens often feature statues, urns, and other ornamental objects.
These objects are used to add a sense of elegance and sophistication to the garden, and they are often
placed in prominent locations throughout the garden.
Outdoor rooms: French style gardens often feature outdoor rooms, which are defined areas within
the garden that are designed for specific activities. These rooms can include outdoor dining areas,
lounges, and reading nooks, and they are often defined by low walls or hedges.
Q.7 Illustrate with sketches the function of the following AutoCAD commands:
(A) REVSURF (B) RULESURF (C) EDGESURF (D) TABSURF (E) THICKNESS

Q.8 Draw a schematic plan and a section of an Open-Air Theatre designed for good acoustics showing
all the design elements used for achieving the objective.

Q.9 A loan of Rs 8,00,000 hs been granted by a financial institution to an individual for the construction
of his house. The loan must be repaid by way of annuity at the rate of 13.5% interest per annum in
15 equal installments from the year in which the loan is taken by the house owner. Calculate the
yearly installment for repaying the loan.
Ans:-
Solution: To calculate the yearly installment for repaying the loan, you can use the formula for the
annuity payment:
PMT= (PV⋅r) / 1−(1+r)−n
Where:
PMT = Yearly installment
PV = Present value of the loan (Rs 8,00,000)
r = Annual interest rate (13.5% or 0.135)
n = Total number of payments (15 years)
Now, plug in the values:
PMT= (8,00,000 * 0.135) / 1−(1+0.135)−15
The yearly installment for repaying the loan of Rs 8,00,000 at an annual interest rate of 13.5% in 15
equal installments is approximately Rs 1,35,698.77.
Q.10 Indicate five major advantages for which you will recommend steel structure for a multi storied
building.
Ans:- There are several advantages of using a steel structure for a multi-storied building. Here are five
major advantages:
Strength and Durability: Steel is a strong and durable material that can withstand heavy loads and
severe weather conditions. This makes it ideal for constructing multi-storied buildings that can
support large numbers of people and equipment.
Speed of Construction: Steel structures can be prefabricated off-site and then assembled on site,
which reduces the construction time significantly. This is particularly important for multi-storied
buildings, as the construction time can be a major factor in the overall project timeline.
684 GATE Architecture and Planning: Comprehensive Question Bank

Cost-Effective: Although the initial cost of a steel structure may be higher than other building
materials, the long-term cost savings due to the durability and low maintenance of steel make it a cost-
effective choice for multi-storied buildings.
Design Flexibility: Steel structures offer design flexibility that allows for unique and creative
architectural designs. This is important for multi-storied buildings, as the aesthetic appeal of the
building can have a significant impact on its use and value.
Sustainability: Steel is a highly sustainable material, as it is recyclable and has a low carbon footprint.
This makes it an environmentally friendly choice for multi-storied buildings, which are typically large
energy consumers. Additionally, the long lifespan of steel structures reduces the need for frequent
replacement, which reduces waste and energy usage.
Q.11 Sketch the section of an overhead water reservoir, showing the float valve, overflow pipe, drain
outlet, supply inlet and outlet and other valves.

Q.12 What is Mastic Asphalt? Where is it used in built environment?


Ans: - Mastic asphalt is a type of asphaltic material that is commonly used in the built environment
as a waterproofing membrane, pavement material, and roofing material. It is composed of bitumen,
limestone filler, and a blend of fine aggregates.
Mastic asphalt is exceptionally durable and has excellent waterproofing properties, making it an ideal
material for use in areas where there is a risk of water ingress or where a waterproof barrier is required.
It is commonly used in flat roofs, bridge decks, and car parks.
Mastic asphalt is also used as a surfacing material for roads, footpaths, and cycleways, due to its
durability, resistance to heavy traffic, and skid resistance. It can be laid in a variety of thicknesses, and
can be finished with a variety of textures, including smooth, textured, or anti-skid.
In addition to its waterproofing and surfacing properties, mastic asphalt has excellent fire resistance
and can be used as a fireproofing material. It is also resistant to UV radiation and can be used in areas
exposed to direct sunlight.
Q.13 An activity in a CPM network has a duration of 4 days. The free float for the activity is 10 days and
the total float is also 10 days. Find the maximum delay that can be allowed for the activity from
the occurance of the preceding event.
Ans: - In a CPM network, the total float is the maximum amount of time an activity can be delayed
from its earliest start time without delaying the project completion time. The free float is the maximum
amount of time an activity can be delayed without delaying the early start time of its successor activity.
Given that the duration of the activity is 4 days, the free float is 10 days, and the total float is also 10
days. This means that there is no other activity that is dependent on the completion of this activity, and
the project completion time will not be affected if this activity is delayed up to 10 days.
To find the maximum delay that can be allowed for the activity from the occurrence of the preceding
event, we need to calculate the earliest start time (EST) and latest start time (LST) for the activity.
EST is the earliest possible time that the activity can start, which is the earliest finish time (EFT) of the
preceding activity.
LST is the latest possible time that the activity can start without delaying the project completion time,
which is the earliest start time (EST) of the succeeding activity minus the duration of the activity.
Hemant Vilas Parulekar 685

Since the total float is 10 days, the latest finish time (LFT) of the activity is 10 days after its EFT.
Therefore, we can write:
LFT = EFT + total float = EST + duration + total float
Since the free float is also 10 days, the earliest start time (EST) of the succeeding activity is 10 days after
the EFT of this activity. Therefore, we can write:
EST of succeeding activity = EFT + free float = EFT + 10
Using these equations and the given values, we can find:
LFT = EST + duration + total float = EST + 4 + 10 = EST + 14
EST of succeeding activity = EFT + 10
Since LST = EST of succeeding activity - duration, we get:
LST = (EFT + 10) - 4 = EFT + 6
We know that the activity can be delayed up to 10 days without delaying the project completion
time, which means the latest finish time (LFT) cannot be greater than the project completion time.
Therefore, we have:
LFT <= Project completion time
This gives us:
EST + 14 <= Project completion time
Solving for EST, we get:
EST <= Project completion time - 14
Now, the maximum delay that can be allowed for the activity from the occurrence of the preceding
event is the difference between the earliest start time (EST) and the earliest finish time (EFT) of the
activity, which is:
EFT - EST = duration + free float = 4 + 10 = 14 days.
Therefore, the maximum delay that can be allowed for the activity from the occurrence of the preceding
event is 14 days.
Q.14 With the help of the diagrammatic plan and as ide view, show the various important elements and
space components of a typical Orissan temple.

Q.15 Explain the phenomenon”Stack effect’in a building. Write the equation for estimating stack effect.
Ans:- The “stack effect” is a phenomenon that occurs in buildings, especially tall ones, due to the
difference in temperature and pressure between the indoor and outdoor environments. It results from
the natural tendency of air to rise when it is warmer and less dense than the surrounding air. The stack
effect can have a significant impact on the ventilation and comfort of a building, especially in multi-
story structures.
Here’s how the stack effect works:
Temperature Difference: During the day, the sun can heat the exterior of a building, including its roof
and walls. As a result, the air inside the building becomes warmer than the outdoor air.
686 GATE Architecture and Planning: Comprehensive Question Bank

Pressure Difference: Warm air is less dense than cold air, so the warm indoor air tends to rise. As it
rises, it creates a zone of lower pressure at the base of the building.
Air Movement: To equalize the pressure difference, outdoor air enters the building at lower levels,
such as through openings in the foundation, windows, and doors. The incoming air displaces the
indoor air, which rises and exits through openings at higher levels, such as windows and vents on
upper floors or the roof.
Ventilation: This movement of air can naturally ventilate the building by providing fresh air from
outside. However, it can also lead to unintended consequences, such as drafts, uneven temperature
distribution, and increased energy consumption for heating or cooling.
The equation for estimating the stack effect in a building is given by:
Q = A * C * √(2 * g * h * ΔT)
Where:
Q represents the airflow rate (in cubic meters per second, m³/s).
A is the effective opening area (in square meters, m²) at both the lower and upper openings.
C is the discharge coefficient, a dimensionless factor that accounts for the type of opening (e.g., doors,
windows).
g is the acceleration due to gravity (approximately 9.81 m/s²).
h is the height difference (in meters) between the lower and upper openings.
ΔT is the temperature difference (in degrees Celsius) between the indoor and outdoor air.
This equation provides an estimate of the airflow rate due to the stack effect in a building. It is
important for architects and engineers to consider the stack effect when designing ventilation systems
and assessing building performance, especially in tall structures where the phenomenon is more
pronounced. Properly designed ventilation systems can help mitigate the negative effects of the stack
effect on indoor comfort and energy efficiency.
Q.16 With the help of three sketches explain the variation of the horizontal thrust at the springing
point of an arch with respect to its rise for a fixed span and load.

Q.17 What is contrast in visual design? Explain the various applications of contrast in architecture.
Ans: - In visual design, contrast refers to the differences between two or more elements in a composition.
It can be created through variations in color, shape, texture, size, or other design elements. Contrast is
an important tool in design because it creates visual interest, draws attention to certain elements, and
can communicate hierarchy or organization within a composition.
In architecture, contrast can be used in a variety of ways to enhance the visual impact of a building or
space. Some applications of contrast in architecture include:
Material contrast: Using contrasting materials, such as combining rough, textured materials with
smooth, glossy ones, can create visual interest and highlight important architectural features.
Color contrast: Contrasting colors can be used to create a focal point or draw attention to specific
areas of a building or space. It can also be used to emphasize a building’s form or to create a sense of
depth.
Hemant Vilas Parulekar 687

Light and shadow contrast: The interplay between light and shadow can create visual interest and
depth in a building’s form. Contrast between light and dark areas can also be used to highlight specific
features or create a sense of drama.
Scale contrast: Contrasting scales can be used to create hierarchy and emphasize important elements.
For example, a large, imposing entrance can contrast with smaller, more subdued elements to create a
sense of grandeur.
Spatial contrast: Contrast in spatial organization, such as contrasting open and closed spaces, can
create visual interest and enhance the experience of a building or space.
Q.18 Explain the importance of Variance, Easement and Deed restrictions in architectural preservation.
Ans: - Variance, easement, and deed restrictions are all legal tools that can be used to protect and
preserve historic buildings and neighborhoods. Here is a brief overview of each:
Variance: A variance is a permission granted by a local zoning board that allows a property owner to
deviate from certain zoning requirements. For example, if a historic building is in a zone that prohibits
certain types of renovations, a variance might be granted to allow the building’s owner to make
necessary repairs or modifications that would otherwise be prohibited. Variance can be important for
historic preservation because it allows owners to adapt older buildings to meet modern needs while
still preserving their historic character.
Easement: An easement is a legal agreement between a property owner and a preservation organization
or government agency that limits the owner’s ability to modify or demolish a building. Essentially, an
easement protects a building’s historic features by requiring the owner to preserve them in perpetuity.
In return, the owner may receive tax benefits or other incentives. Easements can be an effective tool
for preserving historic buildings and neighborhoods because they create a legal obligation to protect
historic features even if the property changes hands.
Deed restrictions: Deed restrictions are like easements in that they limit the owner’s ability to modify
or demolish a building. However, deed restrictions are permanent and “run with the land,” meaning
that they are binding on all future owners of the property. Deed restrictions can be used to protect
historic buildings and neighborhoods by prohibiting certain types of alterations or uses that would be
detrimental to their character.
Together, these legal tools can be used to protect historic buildings and neighborhoods by ensuring that
they are not modified or demolished in ways that would damage their historic character. By providing
incentives for owners to preserve historic features, and by creating legal obligations to protect them,
variance, easement, and deed restrictions help to ensure that historic buildings and neighborhoods
will continue to enrich communities for generations to come.
Q.19 What is the utility of central court in a housing cluster under hot-humid climate? Explain with
sketches.

Q.20 Indicate the various possibilities of use of the following industrial and agricultural wastes in
buildings.
(A) Blast furnace slag (B) Waste glass
(C) Slate and laterite stone waste (D) Coconut waste and husk
(E) Rice Husk
688 GATE Architecture and Planning: Comprehensive Question Bank

Q.21 Compare between “Chaityas” and “Viharas” in rock cut architecture of their layouts, elements
and facade treatment.
Ans: - Chaityas and Viharas are two types of rock-cut architecture found in India, particularly in the
Buddhist and Jain caves. They were constructed during the Maurya period, from 200 BCE to 300 CE.
Chaityas were Buddhist prayer halls or shrines, while Viharas were residential complexes for Buddhist
monks. Here are some key differences between the two:
Layout:
Chaityas were typically designed in a horseshoe shape, resembling a basilica, with a central nave and
side aisles. The prayer hall was located at the end of the nave, which contained a stupa, a symbol of
Buddha.
Viharas were typically rectangular in shape, with several small cells or rooms for individual monks,
arranged around a central courtyard.
Elements:
Chaityas were characterized by their high roofs, which were supported by rows of columns carved into
the rock. The columns often had intricate carvings and decorations, including depictions of Buddha
and other religious figures.
Viharas had a simpler design, with smaller columns and fewer decorative elements. The cells were
often plain, with simple doors and windows.
Facade treatment:
The facades of chaityas were often elaborately decorated, with carvings and sculptures depicting
Buddha, bodhisattvas, and other religious figures. The entrances to the chaitya were often framed by
large, ornate arches.
Viharas had a more utilitarian facade treatment, with fewer decorative elements. The entrances were
often simple, with plain lintels and door frames.
In summary, chaityas and viharas differed in their layout, elements, and facade treatment. Chaityas
were typically larger and more ornate, with high roofs, intricate columns, and elaborate facades. Viharas
were simpler in design, with smaller columns and cells, and a more utilitarian facade treatment.
Q.22 What were the major driving forces in development of modern architectu in post industrial
revolution?
The major driving forces behind the development of modern architecture in the post-industrial
revolution period were:
Technological advancements: The Industrial Revolution brought about a host of technological
advancements that changed the way buildings were designed and constructed. Developments in
materials such as iron and steel, as well as new techniques such as reinforced concrete, allowed for
greater structural flexibility and new forms of expression in architecture.
Social and political changes: The rise of urbanization and the growth of industrialization led to new
social and political conditions that called for new types of buildings and spaces. For example, the need
for worker housing and public health facilities led to the development of new building typologies such
as apartment buildings and hospitals.
Hemant Vilas Parulekar 689

The influence of art movements: Modern architecture was heavily influenced by art movements such
as Art Nouveau, Art Deco, and the Bauhaus. These movements emphasized simplicity, minimalism,
and a rejection of traditional ornamentation, leading to a new aesthetic in architecture.
The emergence of new design philosophies: Modern architecture was also driven by new design
philosophies such as functionalism and rationalism, which emphasized the importance of function,
efficiency, and the use of new materials and techniques.
Globalization and cultural exchange: The growth of globalization and cultural exchange in the
20th century allowed for the spread of new architectural ideas and styles across different regions and
countries, leading to the development of a truly global modern architecture.
GATE QUESTION PAPER 1998
General Aptitude (GA)

Questions of sketches and some questions of numericals are excluded.


Q. 1 – Q. 25 carry one mark each.
Q.1.1 ‘Pompidou Centre’ in Paris is an example of
(A) Symbolic analogy (B) Mechanical analogy
(C) Bio-morphic analogy (D) Mathematical analogy
Ans: - (B)
Q.1.2 Cement (in cu. m) required for preparing 10.0 cu. m of cement concrete in the proportion of 1:2:4 is
(A) 0.80 (B) 1.00 (C) 1.20 (D) 1.40
Ans: - (D)
Soln: - Total volume of cement concrete to be prepared = 10.0 cu. m
Ratio of cement, sand, and aggregates = 1:2:4
= 1+2+4 = 7 parts
Volume of cement in 10.0 cu. M of cement concrete
= 1/7 x 10
= 1.42 or say 1.40
Q.1.3 The Pritzker Architecture Prize, 1996 has been awarded to
(A) Charles Correa (B) Jose Rafael Moneo
(C) Robert Venturi (D) Ricardo Legorretta
Ans: - (B)
Q.1.4 Maximum tensile stress is possible in
(A) Wood (B) Stabilised Mud block
(C) Stone Block (D) Steel
Ans:- (D)
Q.1.5 Kailash Temple’ of Ellora is an example of rock-cut architecture of
(A) Brahmanical style (B) Dravidian Style
(C) Pallava Style (D) Mamalla Style
Ans:- (B)
Q.1.6 “Zeolite” is
(A) Hydrated alumino-silicate (B) Sodium carbonate
(C) Hydrated Calcium hydroxide (D) Calcium bi-carbonate
Ans:- (A)
Q.1.7 The gradient of a horizontal branch in building drainage system should not be steeper than
(A) 1 in 60 (B) 1 in 30 (C) 1 in 20 (D) 1 in 10
Hemant Vilas Parulekar 691

Ans:- (A)
Q.1.8 “Timgad’ is an example of
(A) Greek town (B) Roman Town (C) Sumerian Town (D) Egyptian Town
Ans:- (B)
Explanation: - Please refer GATE-2004, Q. 69, for more information.
Q.1.9 ‘Architect’s Credo’ is
(A) Liberty, Equality and Fraternity (B) Firm, Commodity and Delight
(C) I can, I built, I overchanged (D) Man, Hot dogs, and Apple pie
Ans:- (C)
Q.1.10 ‘Dimmer’ is used for
(A) Energy saving (B) Shortening bulb life
(C) Flexible lighting (D) Change of emphasis of lights
Ans:- (D)
Explanation: - A dimmer is an electrical device used to vary the brightness of a light. By reducing the
voltage supplied to the light source, the dimmer lowers the level of illumination and thus changes the
emphasis of the light. Dimmers can be used to create a range of moods and effects in a space and can
also help save energy by reducing the amount of power used by the lighting system.
Q.1.11 The phenomenon of degrading water quality of natural water bodies in decomposition is called
(A) Putrification (B) Eutrophication (C) Fermentation (D) Hydration
Ans: - (A)
Term Meaning
Putrification Putrefaction is the process of decomposition of organic matter by microorganisms,
resulting in the production of foul-smelling compounds such as ammonia,
hydrogen sulfide, and mercaptans. It is a form of rotting that occurs in anaerobic
(oxygen-poor) conditions, such as in a landfill or a poorly maintained septic
system.
Eutrophication Eutrophication is the natural or artificial enrichment of a water body with
nutrients, particularly nitrogen and phosphorus, which can lead to excessive
growth of algae and other aquatic plants. This can lead to reduced oxygen levels
in the water as the plants decay, resulting in the death of fish and other aquatic
organisms.
Fermentation Fermentation is the process by which microorganisms such as yeast break down
organic matter in the absence of oxygen, producing energy and by-products such
as alcohol, lactic acid, and carbon dioxide. Fermentation is used in the production
of various food and beverage products such as bread, beer, and wine.
Hydration Hydration is the process of adding water to a substance to maintain its moisture
content or to cause a chemical reaction. In construction, hydration is a key process
in the setting and hardening of materials such as concrete, plaster, and mortar, in
which water is added to the dry ingredients to activate a chemical reaction that
causes the mixture to harden and become solid.
692 GATE Architecture and Planning: Comprehensive Question Bank

Q.1.12 In computer program a valid real constant is


(A) 1 (B) -1/2 (C) 1.5 (D) 58,634.2
Ans: - (C)
Note: In C computer rules for constructing C constant:
1. INTEGER CONSTANTS IN C:
An integer constant must have at least one digit.
It must not have a decimal point
It can either be positive or negative.
No commas or blanks are allowed within an integer constant.
If no sign precedes an integer constant, it is assumed to be positive.
The allowable range for integer constants is -32768 to 32767.
2. REAL CONSTANTS IN C:
A real constant must have at least one digit
It must have a decimal point
It could be either positive or negative
If no sign precedes an integer constant, it is assumed to be positive.
No commas or blanks are allowed within a real constant.
3. CHARACTER AND STRING CONSTANTS IN C:
A character constant is a single alphabet, a single digit or a single special symbol enclosed within
single quotes.
The maximum length of a character constant is 1 character.
String constants are enclosed within double quotes.
4. BACKSLASH CHARACTER CONSTANTS IN C:
There are some characters which have special meaning in C language
They should be preceded by backslash symbol to make use of special function of them.
Given below is the list of special characters and their purpose.
Q.1.13 The Water Temple, Awaji Island’ is designed by
(A) Fumihiko Maki (B) Kenzo Tange
(C) Arata Isozaki (D) Tadao Ando
Ans: - (D)
Hemant Vilas Parulekar 693

Q.1.14 An ‘activity’ in Proiect Management means


(A) start or completion of task in time and resource
(B) total project time for completion of work
(C) least total time required to complete the work
(D) actual performance of task consumes time and resources
Ans: - (A)
Explanation: - In Project Management, an “activity” refers to a specific task or unit of work that needs
to be completed as part of a project. It is a component of a project that requires time, resources, and
effort to accomplish, and has a clear beginning and end. Activities are usually defined in a project
plan or schedule, and they can be broken down into smaller, more manageable tasks or sub-activities.
The start and end of an activity are typically marked by milestones or deliverables that indicate the
completion of a specific phase of the project.
Q.1.15 The pH scale runs from 0-14, the nearest value of drinking water is
(A) 0 (B) 4.5 (C) 7.0 (D) 12.6
Ans:- (C)
Q.1.16 The emission of auto exhaust consists of
(A) CO (B) SO2 (C) NOX (D) All the three
Ans:- (D)
Q.1.17 According to architecture of Mansara a building Is called female when plan is
(A) Octagonal (B) Rectangle (C) Circular (D) Square
Ans:- (C)
Q.1.18 The unit of measurement for intensity of sound level is
(A) Bel (B) Decibel (C) Hertz (D) Sones
Ans:- (B)
Q.1.19 To prevent excessive contrast between wall and window which creates glare, the ratio of wall to
window area should not generally exceed
(A) 1:1 (B) 2:1 (C) 3:1 (D) 4:1
Ans:- (C)
Explanation: - As per ECBC 2007 (Energy Conservation Building Code), the ratio of wall to window
area should not generally exceed 3:1 to prevent excessive contrast between wall and window which
creates glare.
Q.1.20 Permissible stress in bending compression (N/mm2) for M-20 grade concrete is
(A) 3.0 (B) 5.0 (C) 7.0 (D) 8.5
Ans:- (C)
Q.1.21 Non-degradable pollutant is
(A) Sewage (B) Algae (C) Detergents (D) DDT
694 GATE Architecture and Planning: Comprehensive Question Bank

Ans:- (D)
Explanation: - Non-degradable pollutants are substances that persist in the environment and do
not break down easily. DDT (dichlorodiphenyltrichloroethane) is an example of a non-degradable
pollutant. It is a synthetic pesticide that was widely used in the mid-20th century to control agricultural
pests and combat insect-borne diseases. However, DDT is highly toxic and accumulates in the
environment, and its use has been largely banned or restricted in many countries due to its harmful
effects on wildlife and human health. Other examples of non-degradable pollutants include heavy
metals like lead and mercury, some plastic products, and certain industrial chemicals.
Q.1.22 Maximum air flow at body level in a room can be achieved through
(A) High inlet and high outlet (B) High inlet and low outlet
(C) Low inlet and high outlet (D) Low inlet and low outlet
Ans:- (A)
Q1.23 The breadbasket region refers to
(A) Tropical desert (B) Coniferous forest
(C) Temperate grassland (D) Tropical grassland
Ans:- (C)
Explanation: - The breadbasket region typically refers to a region known for its agricultural
productivity, particularly in growing cereal crops like wheat, which are used to make bread. The term
is most applied to the vast, flat grasslands of the central United States and Canada, also known as
the Great Plains, which are a major source of wheat and other grains. So, the correct option for the
question is (C) Temperate grassland.
Q1.24 “Tap root” concept was initiated by
(A) Le Corbusier (B) FL Wright (C) Tony Garnier (D) Robert Millart
Ans: - (B)
Explanation: Please refer GATE-2016, Q.2, for more information
Q1.25 Extreme Simplicity’was propagated by
(A) Christopher Nyerges and Dolores Lynn Nyerges (B) C.A. Doxiadis
(C) Mies Van der Rohe (D) Walter Gropius
Ans: - (A)
Q1.26 Botanical name for’Gulmohar tree’is
(A) Hamelia Patens (B) Cordia Sabestina
(C) Delonix Regia (D) Gmelina Phillippensis
Ans:- (C)
Q1.27 Type of village should be built on a bank of river or a sea as per architecture of Mansara is
(A) Nandyabarta (B) Karmukha (C) Swastika (D) Prastara
Ans:- (B)
Explanation: Please refer GATE-2013, Q.7, for more information.
Hemant Vilas Parulekar 695

Q.1.28 CPCB is an organization which deals with


(A) Pest Control (B) Poverty Control (C) Population control (D) Pollution Control
Ans:- (D)
Explanation: CPCB stands for Central Pollution Control Board, an organization in India that works
towards preventing and controlling pollution. Its mandate includes conducting research on the causes
and effects of pollution, promoting pollution control technologies, and setting and enforcing standards
for air and water quality, among other things.
Q.1.29 Heliodome is used to measure
(A) Atmospheric pressure (B) Sound level
(C) Sun Path (D) Intensity of light
Ans:- (C)
Explanation: Heliodome is an architectural term referring to a type of building design that is specifically
oriented towards the path of the sun. The design allows the building to capture the maximum amount
of sunlight during the winter months while minimizing solar heat gain during the summer months.
While it is not used to measure atmospheric pressure, sound level, or intensity of light, the use of the
heliodome design can indirectly impact these factors in a building by optimizing natural daylight and
solar heat gain.
Q.1.30 Multi layer dome was found during
(A) Greek period (B) Renaissance period
(C) Indus valley period (D) Egyptian Civilization
Ans:- (B)
Q.2.1 Match the following: -
Question is unclear.
Ans:-
Q.2.2 Match the following: -
Question is unclear.
Ans:- a – (), b – (), c – (), d – (), e – (), f – (), g – (), h – (), i – (), j – ()
Q.2.3 Question is unclear.

Q.3.0 Distinguish between:


(i) Transplantation and Transpiration (ii) Hardening and Tempering of steel
(iii) curtain wall and cavity wall (iv) Gully and Gutter
(v) Heartwood and Laminated wood (vi) Food Producers and Food Consumers
(vii) Repitition and Rhythm (viii) Easement and Prescriptive Rights
(ix) Earthernware and Stoneware (x) S-trap and P-trap
696 GATE Architecture and Planning: Comprehensive Question Bank

Ans:- (i) Transplantation and Transpiration: Transplantation refers to the process of moving a plant
from one location to another. It can involve uprooting the plant and replanting it in a different location,
or it can involve moving a small seedling from a nursery to its final location. Transpiration, on the
other hand, is the process by which plants lose water through small pores in their leaves called stomata.
This water loss helps to transport nutrients through the plant and cool the plant in hot conditions.
(ii) Hardening and Tempering of Steel: Hardening and tempering are two heat treatment processes
used to improve the mechanical properties of steel. Hardening involves heating the steel to a
temperature where it becomes austenitic (non-magnetic) and then rapidly cooling it by quenching it
in water or oil. This results in a hard but brittle material. Tempering involves reheating the hardened
steel to a lower temperature and then cooling it slowly. This process reduces the hardness of the steel
but improves its toughness and ductility.
(iii) Curtain Wall and Cavity Wall: A curtain wall is a non-load-bearing exterior wall that is hung
on the building structure like a curtain. It is typically made of glass, aluminum, or other lightweight
materials and is used to provide a building with an aesthetically pleasing appearance and energy
efficiency. A cavity wall, on the other hand, is a double-wall system that consists of two masonry walls
separated by an air space. The air space serves as insulation and reduces the transfer of heat and sound
between the interior and exterior of the building.
(iv) Gully and Gutter: A gully is a channel or small watercourse that carries rainwater or surface
runoff. It is typically found in rural areas and is often used to control soil erosion. A gutter, on the
other hand, is a shallow trough or channel that is typically located at the edge of a roof and is used to
collect and channel rainwater away from the building.
(v) Heartwood and Laminated Wood: Heartwood is the central, dark-colored, and often hard portion
of a tree trunk that no longer conducts sap. It is usually found in older trees and can provide greater
resistance to decay and insects than the sapwood. Laminated wood, on the other hand, is a type of
engineered wood that is made by gluing several layers of wood together. It is often used to create
structural beams and columns and can provide greater strength and durability than solid wood.
(vi) Food Producers and Food Consumers: Food producers are organisms that produce their own
food through photosynthesis or other means. They are typically plants, algae, or some types of bacteria.
Food consumers, on the other hand, are organisms that rely on other organisms for their food. They
can be herbivores, carnivores, or omnivores, depending on their diet.
(vii) Repetition and Rhythm: Repetition refers to the use of the same element or design feature
multiple times in a composition. This can create a sense of unity and harmony. Rhythm, on the other
hand, refers to the regular repetition of elements in a composition to create a sense of movement or
progression.
(viii) Easement and Prescriptive Rights: An easement is a legal right to use someone else’s property
for a specific purpose. It can be created by agreement or by law and typically applies to a specific
area of the property. A prescriptive right, on the other hand, is a right to use someone else’s property
that has been acquired through long-term use. It is based on the principle of adverse possession and
typically requires a certain period of continuous use to be established.
(ix) Earthenware and Stoneware: Earthenware is a type of pottery made from a coarse mixture of clay
and other materials, fired at a relatively low temperature. It is porous and not as strong as stoneware.
Earthenware is generally used for decorative items, as well as for making functional items such as
plates, cups, and bowls.
Hemant Vilas Parulekar 697

Stoneware, on the other hand, is a type of pottery made from a finer, denser clay mixture that is fired
at a high temperature, making it harder and more durable than earthenware. Stoneware is non-porous
and is often used for making items such as crockery, baking dishes, and vases.
(x) S-trap and P-trap: S-trap and P-trap are types of plumbing traps used in drainage systems to
prevent foul-smelling gases from entering buildings.
An S-trap is a plumbing trap that is shaped like the letter “S”. It is used to connect a fixture (such as a
sink or toilet) to a drainage system. However, S-traps are no longer used in modern plumbing systems
as they can siphon dry and allow sewer gases to enter the building.
A P-trap is a plumbing trap that is shaped like the letter “P”. It is used to connect a fixture to a drainage
system and has a water seal that prevents sewer gases from entering the building. The P-trap is more
effective than the S-trap because it has a larger water seal and is less likely to siphon dry. It is the most
used type of plumbing trap in modern plumbing systems.
SUB SECTION A-2
Q.4 Outline the procedure to convert a part of an AutoCAD drawing into a separate drawing File.
Ans:- To convert a part of an AutoCAD drawing into a separate drawing file, you can use the
“WBLOCK” command. Here are the steps:
1. Open the AutoCAD drawing that contains the part you want to convert into a separate file.
2. Zoom in and select the part you want to extract.
3. Type “WBLOCK” in the command line and press Enter.
4. In the “Write Block” dialog box that appears, choose a location to save the new file and give it a name.
5. Under “Objects,” choose “Selected objects” and make sure “Convert to block” is unchecked.
6. Click “OK” to create the new file.
This will create a new AutoCAD drawing file with only the selected part of the original drawing. You
can then open this new file and work on it separately from the original drawing.
Q.5 Describe the salient features of English style Garden.
Ans:- The English style garden, also known as the English landscape garden, emerged in England in
the 18th century and is characterized by its naturalistic design, irregularity, and a focus on creating a
picturesque landscape. Some salient features of the English style garden include:
1. Informal layout: English gardens are designed to look like a natural landscape, rather than a formal
garden with symmetrical beds and pathways. The layout is informal and irregular, with curved paths
and naturalistic features like ponds, streams, and hills.
2. Lawn: A large, well-manicured lawn is a key feature of an English garden. The lawn is usually
surrounded by trees, shrubs, and flowering plants.
3. Planting: An English Garden is filled with a variety of plants, including flowering shrubs, perennials,
and annuals. The plantings are arranged in a naturalistic style, with groups of plants of different sizes
and colors creating a harmonious composition.
4. Water features: Ponds, lakes, and streams are common features in English gardens. These water
features are designed to look natural, with rocks, boulders, and native plants surrounding them.
698 GATE Architecture and Planning: Comprehensive Question Bank

5. Follies: English gardens often include decorative structures like follies, which are designed to
look like ruins or ancient buildings. These structures provide visual interest and add to the garden’s
romantic, naturalistic atmosphere.
6. Views and vistas: English gardens are designed to provide a sense of spaciousness and perspective.
Views and vistas are created by placing elements in the landscape to draw the eye toward a distant
point, like a hill, a statue, or a body of water.
7. Garden rooms: English gardens often include garden rooms, which are areas of the garden that
are defined by hedges or walls. Each garden room has its own character and style and may feature a
different type of planting or decorative element.
Q.6 Outline the mechanism to scale down a large public square flanked building all round.
Ans:- Scaling down a large public square flanked building all around can be achieved by the following
mechanism:
1. Analyze the building: Evaluate the overall design, size, and proportions of the building to determine
the scale reduction that is required.
2. Identify key elements: Identify the most prominent features of the building that define its scale,
such as the height, width, and spacing of columns, windows, and other architectural elements.
3. Determine the new scale: Once the key elements have been identified, the new scale of the
building can be determined. This may involve reducing the height, width, or spacing of elements, or a
combination of these factors.
4. Redesign the building: Using the new scale, the building can be redesigned to ensure that it is
visually balanced and proportionate. This may involve adjusting the proportions of the building’s
façade, modifying the size and placement of windows and doors, or changing the size and spacing of
columns.
5. Evaluate the design: After making the necessary adjustments, it is important to evaluate the new
design to ensure that it is visually appealing and proportionate. This may involve reviewing the design
in different lighting conditions and from various angles.
6. Implement the design: Once the design has been finalized, it can be implemented by modifying
the original plans and drawings, and communicating the changes to the relevant stakeholders, such as
architects, engineers, and builders.
Q. 7 Outline with example the evolution of settlement planning thoughts during Egyptian period.
Ans:- Settlement planning in ancient Egypt can be traced back to around 3000 BCE. The Egyptians
were one of the earliest civilizations to develop a highly organized and centralized system of governance,
which allowed for the construction of large-scale urban settlements. The following are some of the key
aspects of settlement planning in ancient Egypt:
1. Gridiron street pattern: The Egyptians were among the first to develop a gridiron street pattern,
which was designed to facilitate the movement of people and goods within the settlement. This was
achieved by dividing the settlement into a series of rectangular blocks, with the streets running parallel
and perpendicular to each other.
2. Centralized planning: Settlement planning in ancient Egypt was highly centralized, with the
pharaoh or ruler of the kingdom playing a key role in the planning process. The pharaoh would appoint
a chief architect or planner, who was responsible for overseeing the construction of new settlements
and the maintenance of existing ones.
Hemant Vilas Parulekar 699

3. Hierarchical zoning: Settlements in ancient Egypt were typically divided into a series of zones, with
each zone serving a specific function. For example, the central zone of a settlement would typically
contain the administrative and religious buildings, while the residential areas would be located on the
outskirts of the settlement.
4. Monumental architecture: The Egyptians were renowned for their monumental architecture, which
included the construction of massive temples, pyramids, and other public buildings. These structures
were designed to showcase the power and wealth of the pharaoh and his kingdom.
One example of settlement planning in ancient Egypt is the city of Amarna, which was built during the
reign of Pharaoh Akhenaten in the 14th century BCE. The city was designed to be a new capital for the
kingdom and was laid out in a gridiron pattern, with wide streets and large public spaces. The city also
featured a central administrative district, which contained the pharaoh’s palace and other government
buildings, as well as residential areas for the city’s inhabitants. The city was abandoned shortly after
Akhenaten’s death, but it remains an important example of ancient Egyptian settlement planning.
Q.8 A developer is planning to develop 50 hectares of land for residential complex 65% of the total land
will be available for residential plots. The land acquisition cost is Rs 60.00 per sq. Mentioning all
the outgoings and assuming appropriate cost for them with 20% profit for the developer. workout
the land cost component for the project per unit area of built space if FAR is 1.5.
Ans:- To calculate the land cost component for the project, we can follow these steps:
1. Total area of the land = 50 hectares = 500,000 sq m
2. Area available for residential plots = 65% of 500,000 sq m = 325,000 sq m
3. Assuming 20% of the land is used for roads, parks, and other common amenities, the net residential
plot area = 80% of 325,000 sq m = 260,000 sq m
4. Assuming an FAR of 1.5, the built-up area on the residential plots = 1.5 times the net residential plot
area = 1.5 x 260,000 sq m = 390,000 sq m
5. Assuming an appropriate cost of Rs 10,000 per sq m for construction, the total cost of construction
= Rs 10,000 x 390,000 sq m = Rs 39,00,00,000
6. Assuming an appropriate cost of Rs 2,000 per sq m for common amenities, the total cost of common
amenities = Rs 2,000 x 65,000 sq m = Rs 13,00,00,000
7. The total land cost = land acquisition cost + development charges + profit
8. Assuming an appropriate cost of Rs 10,000 per sq m for development charges, the total development
charges = Rs 10,000 x 390,000 sq m = Rs 39,00,00,000
9. The total cost of the project = total construction cost + total cost of common amenities + total
development charges + profit = Rs 39,00,00,000 + Rs 13,00,00,000 + Rs 39,00,00,000 + 20% of the total
cost
10. The land cost component for the project per unit area of built space = (total cost of the project
- total construction cost - total cost of common amenities) / built-up area = (Rs 1,20,00,00,000 - Rs
39,00,00,000 - Rs 13,00,00,000) / 390,000 sq m = Rs 2041 per sq m.
Therefore, the land cost component for the project per unit area of built space is Rs 2041 per sq m.
Q.9 Explain with sketches the evolution of Gothic vaulting during Roman and Romanesque period.
Ans:-
700 GATE Architecture and Planning: Comprehensive Question Bank

Q.10 Explain the term “Vernacular Architecture”. Mention the contribution of Laurie Baker in the
development of vernacular architecture.
Ans:- Vernacular architecture refers to the style of buildings that are designed and constructed using
local materials, traditional techniques, and indigenous architectural styles. It reflects the local culture,
climate, and geography of a region and is often considered sustainable and low-cost.
Laurie Baker was a British-born architect who spent the majority of his professional career in India,
where he made significant contributions to the development of vernacular architecture. He believed
in the use of local materials, labor, and techniques to create sustainable, affordable, and culturally
relevant buildings.
Some of the key features of Baker’s vernacular architecture include the use of exposed brickwork,
sloping roofs, sunshades, courtyards, and natural ventilation. His buildings often incorporate elements
of traditional Indian architecture, such as jalis (perforated screens), arches, and terraces.
Baker was also known for his use of innovative construction techniques to reduce costs and increase
efficiency. For example, he developed the “rat trap bond” technique for laying bricks, which uses half
as many bricks as traditional methods and reduces the need for plastering. He also experimented with
mud, stabilized mud blocks, and other low-cost building materials.
Baker’s contributions to vernacular architecture in India have had a lasting impact on the field.
His buildings are celebrated for their simplicity, sustainability, and beauty, and continue to inspire
architects and builders around the world to create structures that reflect local traditions and cultures.
Q.11 Mention the advantages of Poly-carbonate sheets or fibre glass reintorced plastic.
Ans:- Advantages of Polycarbonate Sheets:
1. High Impact Resistance: Polycarbonate sheets are highly impact resistant and can withstand high
levels of force, making them ideal for use in areas that require a strong and durable material.
2. UV Protection: Polycarbonate sheets have UV protective coatings that prevent them from yellowing
or becoming brittle over time due to exposure to sunlight.
3. Lightweight: Polycarbonate sheets are relatively lightweight, making them easy to handle and install.
4. Energy Efficient: Polycarbonate sheets are good insulators, which means they can help reduce
heating and cooling costs.
5. Design Flexibility: Polycarbonate sheets can be easily fabricated and molded into various shapes
and sizes, providing architects and designers with greater design flexibility.
Advantages of Fiberglass Reinforced Plastic:
1. Corrosion Resistance: Fiberglass reinforced plastic is highly resistant to corrosion, making it ideal
for use in harsh and corrosive environments.
2. High Strength-to-Weight Ratio: Fiberglass reinforced plastic is stronger and lighter than many
other materials, making it ideal for use in applications where weight is a concern.
3. Durability: Fiberglass reinforced plastic is exceptionally durable and can withstand harsh weather
conditions, impacts, and vibrations.
4. Non-Conductive: Fiberglass reinforced plastic is non-conductive, making it safe for use in electrical
and electronic applications.
5. Low Maintenance: Fiberglass reinforced plastic requires little maintenance and can last for many
years without requiring repairs or replacement.
Hemant Vilas Parulekar 701

Q.12 Draw the sketches (atleast five) of the different ways to cover long spall uninterrupted space.
Ans:-
Q.13 Critically analyse the architectural features of Tai Mahal building complex d Agra.
Ans:- The Taj Mahal in Agra, India, is an iconic masterpiece of Mughal architecture and is considered
one of the most beautiful buildings in the world. It was built by the Mughal emperor Shah Jahan in the
17th century as a mausoleum for his beloved wife, Mumtaz Mahal.
Here are some of the architectural features of the Taj Mahal:
1. Symmetry: The Taj Mahal is known for its perfect symmetry. The four minarets, the central dome,
and the four smaller domes are all perfectly aligned.
2. Use of white marble: The Taj Mahal is constructed entirely of white marble, which gives it a timeless
beauty and elegance. The marble was sourced from the quarries of Rajasthan.
3. Inlaid work: The white marble surface of the Taj Mahal is decorated with intricate inlaid work made
of precious and semi-precious stones. The inlay work is mostly floral and geometric patterns.
4. Calligraphy: The Taj Mahal features extensive use of calligraphy, with passages from the Quran
inscribed on the walls in decorative script.
5. Water features: The Taj Mahal has several water features, including a large reflecting pool and
fountains. The use of water adds to the beauty and serenity of the complex.
6. Gardens: The Taj Mahal is surrounded by beautiful gardens, which are based on the traditional
Mughal style of garden design. The gardens feature symmetrical layouts, geometric patterns, and
water features.
Overall, the architectural features of the Taj Mahal are a testament to the skill and craftsmanship of
the Mughal builders and artisans. The building complex showcases a unique blend of Islamic, Persian,
and Indian architectural styles, making it an important cultural and architectural landmark. The Taj
Mahal is a truly remarkable example of how architecture can be used to express deep emotions such
as love and devotion.
Q.14 State the special considerations for the fire fighting measures for an auditorium.
Ans:- Fire fighting measures are essential for any building, and they become even more critical for
buildings with a large gathering of people, such as an auditorium. Some special considerations for fire
fighting measures in an auditorium are:
1. Fire Alarm Systems: An auditorium should be equipped with a fire alarm system that can detect
the early signs of fire and alert the occupants of the building.
2. Fire Suppression Systems: The auditorium should have a fire suppression system installed to
control the spread of fire. This could include a sprinkler system or a water mist system.
3. Smoke Control Systems: Smoke can quickly fill up an auditorium, making it difficult for occupants
to see and breathe. A smoke control system can prevent smoke from spreading and enable occupants
to exit the building safely.
4. Fire Exits: The auditorium should have multiple fire exits that are clearly marked and easily
accessible. The exits should be wide enough to allow a large number of people to exit the building
quickly.
5. Fire Fighting Equipment: The auditorium should have fire extinguishers, fire hoses, and other fire
fighting equipment placed at strategic locations throughout the building. These should be regularly
checked and maintained.
702 GATE Architecture and Planning: Comprehensive Question Bank

6. Electrical Safety: The electrical wiring and equipment used in the auditorium should be installed
and maintained to prevent electrical fires. This could include regular checks of electrical panels,
wiring, and equipment.
7. Training: The building management should provide training to staff and occupants on fire safety
and evacuation procedures. Regular fire drills should also be conducted to ensure that everyone knows
what to do in case of a fire.
In summary, the special considerations for fire fighting measures in an auditorium include early
detection, suppression, smoke control, multiple fire exits, fire fighting equipment, electrical safety, and
training. These measures can help to prevent fires, control their spread, and enable safe evacuation of
occupants.
Q.15 Bring out the advantages of PVC pipes over conventional C.I. pipes in building sewer system.
Ans:- PVC (polyvinyl chloride) pipes have several advantages over conventional C.I. (cast iron) pipes
in building sewer systems, which include:
1. Lightweight: PVC pipes are lightweight and easy to handle compared to heavy and bulky C.I. pipes.
This makes installation easier and less labor-intensive, reducing installation costs.
2. Corrosion-resistant: PVC pipes are highly resistant to corrosion, while C.I. pipes are prone to
corrosion over time. This means that PVC pipes have a longer lifespan and are less likely to need repair
or replacement due to rust or corrosion.
3. Smooth interior: PVC pipes have a smooth interior surface, which prevents the buildup of debris and
sediment and minimizes the chances of clogs and blockages. C.I. pipes have a rougher interior surface,
which can accumulate debris and sediment over time, leading to blockages and reduced flow capacity.
4. Cost-effective: PVC pipes are less expensive than C.I. pipes, making them a cost-effective choice
for building sewer systems. They are also easier and less expensive to transport and store due to their
lightweight nature.
5. Chemical-resistant: PVC pipes are highly resistant to chemical corrosion and are not affected by
acids and alkalies that are commonly found in sewage. C.I. pipes, on the other hand, can be affected
by these chemicals, leading to corrosion and eventual failure.
Q.16 Enumerate the design considerations to ensure ventilation in engineering industrial structures.
Ans:- Design considerations to ensure ventilation in engineering industrial structures include:
1. Adequate openings: Proper design of openings such as doors, windows, and vents is important
to allow for the circulation of air within the building. The placement of these openings should be
planned to optimize the flow of air.
2. Natural ventilation: Using natural ventilation techniques such as wind towers, roof vents, and
louvers can help bring in fresh air and remove stale air.
3. Forced ventilation: Forced ventilation systems such as exhaust fans, air conditioning systems, and
HVAC systems can be installed to remove polluted air and maintain a constant supply of fresh air.
4. Stack effect: Designing the building with a high roof and an open ceiling can create a natural “stack
effect” which helps hot air rise and escape, drawing in fresh air from below.
5. Cross ventilation: Cross ventilation is a technique that involves designing the building to allow air
to flow through it in a crosswise direction. This can be achieved by placing openings on opposite walls
or through the use of vents.
Hemant Vilas Parulekar 703

6. Roof design: The design of the roof can impact ventilation. Flat roofs can trap hot air, while sloping
roofs can allow for better air flow. The use of a vented roof can also be helpful in allowing for air flow.
7. Building orientation: The orientation of the building in relation to the prevailing wind direction can
also impact ventilation. Designing the building to face the prevailing winds can help bring in fresh air
and remove polluted air.
8. Materials: Choosing materials that allow for ventilation can also be helpful. For example, using
porous materials such as clay tiles or perforated metal sheets can help increase air flow.
All these design considerations can help ensure proper ventilation in engineering industrial structures.
Q.17 Discuss with illustrations the application of Organic Architecture.

Q.18 Show the exploded details of


a) Dove-tailed joint in wood
b) Double Tenon and Mortise joint in wood.
Ans:-
Q.19 Outline the utility of mixing sand in mortar.
Ans: - Mixing sand in mortar is important because it provides several benefits:
1. Workability: Sand improves the workability of mortar. It helps to reduce the water-cement ratio,
which in turn improves the consistency of the mortar, making it easier to spread and apply.
2. Strength: Sand is an important component of mortar, and it helps to increase the strength of the
mortar. The sand particles fill in the voids between the cement particles, which makes the mortar
stronger and more durable.
3. Shrinkage: Mixing sand in mortar helps to reduce shrinkage. The sand particles provide a buffer
between the cement particles, which helps to prevent cracking and shrinkage during the drying
process.
4. Cohesion: Sand also provides cohesion to the mortar mix. The sand particles interlock with each
other and with the cement particles, creating a strong and cohesive bond.
5. Cost: Sand is a relatively inexpensive material, so using it in mortar can help to reduce the overall
cost of the project.
Q.20 Explain the visual principles of landscape design with building mass or block.
Ans: - The visual principles of landscape design with building mass or block involve the relationship
between the built form and the surrounding landscape. Here are some of the principles:
1. Scale: The scale of the building mass or block should be appropriate to the scale of the surrounding
landscape. The building should be neither too large nor too small compared to the landscape.
2. Proportion: The proportion of the building mass or block should be in harmony with the proportions
of the landscape elements, such as trees, shrubs, and other natural features.
3. Alignment: The building mass or block should be aligned with the natural features of the landscape,
such as hills, valleys, and water bodies. This creates a sense of harmony and balance between the built
form and the natural surroundings.
704 GATE Architecture and Planning: Comprehensive Question Bank

4. Contrast: The building mass or block should contrast with the landscape in terms of texture, color,
and shape. This creates visual interest and draws attention to the building.
5. View: The building mass or block should be designed in such a way that it provides views of the
surrounding landscape, while also being visible from different vantage points in the landscape.
6. Context: The building mass or block should be designed to fit into the context of the surrounding
landscape. This includes considering the cultural, historical, and ecological aspects of the landscape.
Q.21 Draw the essential features of St. Peter Plaza, Rome, explaining the design feature

Q.22 The rateable value of a building is Rs 20,000/- p.a. when interest on capital is 18%and on sinking
fund is 6%. The owner of the building gets an offer from a bank for a net rent of Rs 25,000/- p.a. for
21 years lease period, provided he modifies the internal layout at a cost of Rs.30,000/-. As a valuer
what would be your advice to the owner regarding the bank offer.
Ans: - As a valuer, I would first calculate the net present value (NPV) of the bank offer and compare it
with the present value (PV) of the existing rental income.
If the existing rental income of Rs. 20,000/- p.a. will remain constant for the next 21 years, the PV of
this income at a discount rate of 18% can be calculated as follows:
PV = Annual Rental Income / Discount Rate = 20,000 / 0.18 = Rs. 1,11,111/-
Now, let us calculate the NPV of the bank offer. The net rent of Rs. 25,000/- p.a. for 21 years can be
considered as an annuity, which can be calculated using the present value of an annuity formula:
NPV = Annual Net Rent x PV Annuity Factor = 25,000 x 11.620 = Rs. 2,90,500/-
The cost of modification of Rs. 30,000/- needs to be subtracted from this NPV to arrive at the net
benefit of the bank offer:
Net Benefit = NPV - Cost of Modification = 2,90,500 - 30,000 = Rs. 2,60,500/-
If the net benefit of the bank offer is positive, then it would be advisable for the owner to accept the
offer, as it would result in a higher return on investment. In this case, since the net benefit of the bank
offer is positive, the owner should consider accepting the offer.
Q.23 Determine the size and the end span reinforcement (due to moment) for a beam to support a live
load of 12 KN/m on a single span of 8 m, using M15 concrete grade ost=230 N/mm2 (R=0.658 N/
mm2 and j=0.9)
Solution:- To determine the size and reinforcement of the beam, we need to calculate the bending
moment and shear force acting on the beam.
Bending moment (M) = WL2/8 = (12 kN/m) x (8 m)2 / 8 = 96 kNm
Shear force (V) = WL/2 = (12 kN/m) x (8 m) / 2 = 48 kN
Assuming a rectangular section for the beam, we can use the following formulae to calculate the depth
(D) and width (B) of the beam:
D = K * (M / (fck * b * j)), where K = 1/(0.87 + (fy/ fyk)) = 0.456 (for Fe415 steel)
B = M / (0.87 * fy * D)
Assuming a steel reinforcement of Fe415, we have:
fy = 415 N/mm2 fck = 15 N/mm2 j = 0.9 K = 0.456 Lever arm (z) = 0.95D
Hemant Vilas Parulekar 705

Plugging in the values, we get:


D = 0.456 * (96 x 106) / (15 x 230 x 0.9) = 383 mm Taking D = 400 mm
B = 96 x 106 / (0.87 x 415 x 400) = 260 mm Taking B = 300 mm
The effective depth (d) of the beam is given by: d = D - (0.95 x bar diameter)
Assuming 20 mm bars, we have:
d = 400 - (0.95 x 20) = 381 mm Taking d = 380 mm
The minimum steel required for the beam is given by: Ast = 0.85 x fy x A / (0.87 fyk)
where A = bd
Ast = 0.85 x 415 x (300 x 380) / (0.87 x 415) = 609.76 mm2 Taking Ast = 616 mm2 using 4 nos. of 20
mm diameter bars
The end span reinforcement due to moment is given by: Mu = wl2/10 = 12 x 82/10 = 76.8 kNm
As the beam is simply supported, we can assume the end span reinforcement to be the same as the mid
span reinforcement, i.e., 616 mm2 using 4 nos. of 20 mm diameter bars.
Q.24 Highlight the critical design features of Asiad Village, New Delhi.
Ans:- Asiad Village, also known as the Asian Games Village, was designed as an athlete’s village for
the 1982 Asian Games held in New Delhi, India. The complex consists of 1,168 residential units,
a community center, a commercial center, and various sports facilities. Some of the critical design
features of the Asiad Village include:
1. Sustainable Design: The complex was designed with a focus on sustainable design features such
as natural ventilation, use of local materials, and energy-efficient systems. The buildings have large
windows to maximize natural light and ventilation, and the roofs are covered with solar panels to
generate electricity.
2. Spatial Layout: The complex is designed as a pedestrian-friendly community with a central park
and a network of pathways connecting the various buildings. The buildings are arranged in a grid
pattern, with open spaces and green areas integrated throughout the complex.
3. Structural Design: The buildings in Asiad Village were constructed with a reinforced concrete
frame and brick infill walls. The structural design was based on earthquake-resistant principles, with
provisions for seismic loads and lateral forces.
4. Sports Facilities: Asiad Village includes several sports facilities, including a swimming pool, a
tennis court, a basketball court, and a football field. These facilities are designed to meet international
standards and are used for training and competition by athletes.
5. Community Spaces: The complex includes a community center and a commercial center, which
provide spaces for residents to socialize and interact. The community center includes a library, a
gymnasium, and a theater, while the commercial center has shops, restaurants, and other services.
Overall, the Asiad Village is a well-designed and sustainable community that meets the needs of athletes
and residents alike. The critical design features of the complex have been recognized as exemplary and
have been emulated in other developments around the world.
Q.25 Compare between Early start schedule and Late start schedule in Project Management.
706 GATE Architecture and Planning: Comprehensive Question Bank

Ans:- In project management, an early start schedule (ESS) and a late start schedule (LSS) are two
different approaches for scheduling project activities.
ESS is a scheduling technique in which activities are scheduled to start as soon as possible, without
delaying the project completion date. This means that activities are scheduled to start on their earliest
possible start dates, based on the availability of resources and any dependencies. This approach is
often used when there is no specific reason to delay the start of an activity, and resources are readily
available.
On the other hand, LSS is a scheduling technique in which activities are scheduled to start as late as
possible, without delaying the project completion date. This means that activities are scheduled to
start on their latest possible start dates, based on the availability of resources and any dependencies.
This approach is often used when there is a need to delay the start of an activity to optimize resource
utilization, or to accommodate other activities.
The key differences between ESS and LSS are as follows:
1. Start date: ESS schedules activities to start as early as possible, while LSS schedules activities to start
as late as possible.
2. Resource utilization: ESS may result in higher resource utilization, as activities are scheduled to
start as soon as possible. LSS, on the other hand, may result in lower resource utilization, as activities
are delayed optimizing resource allocation.
3. Flexibility: ESS may be less flexible, as activities are scheduled to start as soon as possible and may
not be easily moved. LSS may be more flexible, as activities are scheduled to start as late as possible and
can be moved within the available time frame.
Overall, both ESS and LSS have their advantages and disadvantages, and the choice between the two
depends on the specific needs of the project and the available resources.
Q.26 Mention the architectural characters developed in the construction of Khajuraho group temples
which distinguish them from any other temple design and style.
Ans:-
Q.27 Starting the concept of Taxonomy of Space’, explain atleast four methods of creating spaces within
a large enveloping space.
Ans:- The Khajuraho group of temples is a set of Hindu and Jain temples located in the Chhatarpur
district of Madhya Pradesh, India. These temples were constructed between the 10th and 12th
centuries by the Chandela dynasty. The architectural features of these temples are considered unique
and distinguishable from other temple designs and styles. Some of the distinctive architectural features
of the Khajuraho group of temples are:
1. Elaborate carvings: The Khajuraho temples are known for their intricate and elaborate carvings.
The carvings depict scenes from everyday life, as well as mythological stories and images of gods and
goddesses. The carvings are some of the finest examples of Indian art.
2. Sculptures: The Khajuraho temples are also known for their sculptures, which are made from
sandstone. The sculptures depict a range of subjects, including human figures, animals, and mythical
creatures.
3. Shikharas: The temples have shikharas, or spires, that rise above the temples. The shikharas are one
of the most distinctive features of the Khajuraho temples.
Hemant Vilas Parulekar 707

4. Mandapas: The temples have mandapas, or pillared halls, which are used for religious rituals and
ceremonies. The mandapas are adorned with carvings and sculptures.
5. Kandariya Mahadeva temple: The Kandariya Mahadeva temple is the largest and most ornate
temple in the Khajuraho group. It is known for its elaborate carvings, which cover the entire temple.
Q.28 Suggest the elements in normal public buildings to be stressed upon for the use of physically
handicapped persons.
Ans:- Designing buildings for accessibility to physically handicapped persons is an important aspect
of inclusive design. Some of the key elements that should be considered in normal public buildings to
accommodate physically handicapped persons are:
1. Ramps: Ramps should be provided for wheelchair users, with a slope that is not too steep, and
handrails on both sides for support.
2. Elevators: Elevators should be provided for multi-storey buildings, with doors wide enough to
accommodate wheelchairs and buttons at a height that can be easily reached by wheelchair users.
3. Accessible toilets: Accessible toilets should be provided on each floor with a larger cubicle, grab
bars, and an emergency alarm.
4. Wide doors and corridors: Doors and corridors should be wide enough to accommodate
wheelchairs, and free of any obstacles.
5. Braille signage: Signage should be provided in Braille to assist visually impaired persons.
6. Audio announcements: Audio announcements should be made to assist visually impaired persons.
7. Lowered counters and sinks: Counters and sink in public buildings should be lowered to
accommodate wheelchair users.
8. Tactile paving: Tactile paving should be used to guide visually impaired persons to entrances, exits,
and other important locations.
9. Parking spaces: Designated parking spaces should be provided near the entrance of the building
for handicapped persons.
SECTION- B
Q.29 Enumerate the decision factors for outdoor recreation.
Ans:- Outdoor recreation involves various activities such as hiking, camping, boating, fishing, hunting,
and more. The decision factors for outdoor recreation may vary depending on the type of activity and
location, but some general factors include:
1. Location and Accessibility: The location of the outdoor recreation area is an important factor. It
should be easily accessible by road or other modes of transport and should be in a safe and secure area.
2. Facilities and Amenities: The availability of facilities and amenities such as restrooms, picnic areas,
camping sites, trails, parking, and water sources can make a big difference in the outdoor recreation
experience.
3. Safety and Risk Management: Outdoor recreation involves some level of risk. Safety measures such
as signage, maps, and first aid facilities should be available to reduce the risk.
4. Environmental Impact: Outdoor recreation activities should be planned and carried out in a way
that minimizes the impact on the environment, including wildlife, vegetation, water sources, and
other natural resources.
708 GATE Architecture and Planning: Comprehensive Question Bank

5. Weather and Climate: The weather and climate conditions can greatly affect outdoor recreation
activities. Proper gear and equipment should be used to ensure safety and comfort in different weather
conditions.
6. Regulations and Permits: Some outdoor recreation areas require permits or have specific
regulations. It is important to be aware of and follow these regulations to ensure a safe and enjoyable
experience for everyone.
7. Personal Preferences and Goals: Personal preferences and goals play a big role in outdoor recreation.
Individuals should choose activities that they enjoy and are comfortable with and set personal goals to
achieve during the outdoor recreation experience.
Q.30 Mention five Government sponsored Slum Upgradation Schemes in urban areas.
Ans:- Five Government-sponsored slum upgradation schemes in urban areas in India that were
prevailing before 1998:
1. National Slum Development Programme (NSDP)
2. Nehru Rozgar Yojana (NRY)
3. Urban Basic Services for the Poor (UBSP)
4. Integrated Development of Small and Medium Towns (IDSMT)
5. Urban Community Development Programme (UCDP)
Q.31 Draw a sketch of Diamond Interchange of a Freeway with a Highway and label all the parts.
Ans:-
Q.32 Suggest the components of Financial Appraisal of a Project.
Ans:- Financial appraisal of a project involves a detailed analysis of various financial aspects of the
project to determine its feasibility and profitability. Some of the key components of financial appraisal
of a project include:
1. Cost estimation: Estimating the total cost of the project including land acquisition, construction,
equipment, labor, and other expenses.
2. Revenue estimation: Forecasting the revenue that the project is likely to generate during its lifespan,
based on market demand, price, and competition.
3. Cash flow analysis: Preparing cash flow statements to assess the timing of inflows and outflows of
cash associated with the project.
4. Break-even analysis: Determining the point at which the revenue generated by the project will equal its
total cost and analyzing the impact of changes in various assumptions on the break-even point.
5. Risk analysis: Identifying and analyzing the risks associated with the project and developing
strategies to mitigate them.
6. Sensitivity analysis: Conducting a sensitivity analysis to evaluate the impact of changes in key
variables, such as price, demand, and cost, on the profitability of the project.
7. Financing options: Evaluating various financing options available for the project and selecting the
most suitable one based on the cost of capital and other factors.
8. Return on investment (ROI): Estimating the return on investment for the project and comparing
it with the required rate of return to determine the viability of the project.
Hemant Vilas Parulekar 709

9. Payback period: Estimating the time required for the project to generate sufficient cash flow to
recover the initial investment and comparing it with the expected lifespan of the project.
10. Net present value (NPV) and internal rate of return (IRR): Calculating the net present value
and internal rate of return for the project to determine its profitability and compare it with other
investment opportunities.
Q.33 Evaluating the Master Plan Approach.suggest the future of Spatial Plan.
Ans:- The Master Plan approach has been the traditional method of spatial planning in many countries.
It involves creating a comprehensive plan for the physical development of an area or region, with
a focus on land use, infrastructure, transportation, and environmental considerations. The Master
Plan approach has some advantages, such as providing a long-term vision for development, ensuring
coordination among various stakeholders, and providing a framework for decision-making.
However, the Master Plan approach has also faced criticism for being inflexible, bureaucratic, and not
responsive to changing social, economic, and environmental conditions. In recent years, there has
been a growing recognition of the need for more flexible and adaptive approaches to spatial planning,
such as strategic planning, scenario planning, and participatory planning.
The future of spatial planning is likely to involve a more integrated and adaptive approach that
considers a range of factors, including economic, social, and environmental considerations. Spatial
planning is likely to become more data-driven, with the use of geographic information systems (GIS)
and other digital tools to analyze and visualize spatial data.
Participatory planning and community engagement are also likely to become more important, with
a greater emphasis on involving local communities in the planning process and considering their
needs and priorities. Spatial planning is also likely to become more focused on sustainability, with a
greater emphasis on green infrastructure, sustainable transportation, and climate change adaptation
and mitigation.
Overall, the future of spatial planning is likely to involve a shift towards more flexible, adaptive, and
integrated approaches that are better able to respond to changing social, economic, and environmental
conditions, and that consider the needs and priorities of local communities.
Q.34 Highlight the principles with sketches “Hoyt’s Sector Theory.”
Ans:-
Q.35 The weights of 40 students in a college are recorded to the nearest kilogram and frequency polygon
for the weight distribution at Construct a histogram
Ans:-
Q.36 Mention the different ways of decreasing the traffic Noise level from Highway.
Ans:- There are several ways to decrease traffic noise levels from a highway, including:
1. Constructing noise barriers: This involves building walls, fences or other structures along the sides
of the highway to block the noise from reaching nearby residential areas.
2. Using vegetation: Planting trees, shrubs and other vegetation can help absorb and reduce noise levels.
3. Installing sound-absorbing materials: Special materials can be added to highway surfaces, such as
quieter asphalt or rubberized asphalt, to reduce noise.
4. Reducing speed limits: Lowering speed limits on highways can help reduce noise levels, as higher
speeds generate more noise.
710 GATE Architecture and Planning: Comprehensive Question Bank

5. Restricting truck traffic: Trucks generate more noise than other vehicles, so restricting their use
during certain times or in certain areas can help reduce noise levels.
6. Encouraging use of electric vehicles: Electric vehicles generate less noise than traditional vehicles,
so encouraging their use on highways can help reduce noise levels.
7. Using traffic management strategies: Implementing traffic management strategies, such as
roundabouts or traffic signals, can help reduce noise levels by decreasing the overall speed of traffic.
8. Relocating residential areas: In some cases, it may be necessary to relocate residential areas further
away from highways to reduce exposure to traffic noise.
Q.37 Identify various methods for conducting Environmental Impact Assessment
Ans:- There are several methods for conducting Environmental Impact Assessment (EIA), some of
which are listed below:
1. Screening: This is a preliminary assessment used to determine if an EIA is necessary for a project.
It is usually a quick and simple process that looks at the project’s characteristics and compares them to
predetermined criteria to determine if any significant impacts are likely to occur.
2. Scoping: This involves a more detailed assessment of the potential impacts of a project. It involves
identifying the key issues and concerns of stakeholders and determining the scope and level of detail
required for the EIA.
3. Baseline studies: This involves the collection and analysis of data on the existing environmental
conditions in the project area. This data is used as a basis for assessing the potential impacts of the
project.
4. Impact assessment: This involves the identification and assessment of the potential environmental,
social, and economic impacts of the project. It includes an analysis of the potential impacts of the
project on various components of the environment, such as air quality, water quality, biodiversity, and
soil quality.
5. Mitigation and impact management: This involves the development of measures to avoid, minimize,
or mitigate the potential impacts of the project. This may include design changes, alternative project
locations, or the implementation of environmental management plans.
6. Monitoring and evaluation: This involves the ongoing monitoring and evaluation of the project’s
actual impacts against the predicted impacts. This helps to determine the effectiveness of the mitigation
measures and impact management strategies and provides feedback for future projects.
Q.39 Outline the details required to be incorporated in the preparation of project estimate for water
supply scheme of a town.
Ans:- The following details are required to be incorporated in the preparation of project estimate for
a water supply scheme of a town:
1. Quantity of water demand: The estimate should consider the water requirement of the town, which
is based on factors such as population, industry, and agriculture.
2. Water source: The source of water, whether it is from a river, lake, or groundwater, should be
identified and assessed for its reliability and quality.
3. Treatment plant: The type and size of the water treatment plant required to treat the water and
make it suitable for drinking should be determined.
Hemant Vilas Parulekar 711

4. Transmission system: The transmission system should be designed to deliver the treated water to
the town through a network of pipes, pumping stations, and storage tanks.
5. Distribution network: The distribution network should be designed to deliver the water to
individual households and other consumers, such as industries and commercial establishments.
6. Pumping stations: The number and size of pumping stations required for the transmission and
distribution of water should be determined.
7. Storage tanks: The size and number of storage tanks required to store the water and ensure a
continuous supply to consumers should be determined.
8. Cost estimates: The costs involved in constructing the various components of the water supply
scheme, such as treatment plant, transmission system, distribution network, pumping stations, and
storage tanks, should be estimated.
9. Contingency provisions: Contingency provisions should be made to account for any unforeseen
circumstances that may arise during the implementation of the project.
10. Maintenance and operation costs: The estimate should include the costs involved in the
maintenance and operation of the water supply scheme to ensure its sustainability and uninterrupted
supply of water to the town.
Q.40 Suggest an action program for topsoil conservation or protection.
Ans:- Here is an action program for topsoil conservation or protection:
1. Limit soil disturbance: Soil disturbance leads to loss of topsoil and exposes the soil to erosion.
Limiting soil disturbance by avoiding unnecessary tilling, reducing traffic on soil, and practicing no-
till farming can help conserve topsoil.
2. Maintain soil cover: Soil cover is important for protecting topsoil from erosion by wind and water.
Cover crops, crop residue management, and vegetation management can help maintain soil cover.
3. Manage water runoff: Uncontrolled water runoff can lead to erosion and loss of topsoil. Constructing
contour terraces, grassed waterways, and sediment ponds can help manage water runoff and prevent
soil erosion.
4. Use erosion control practices: Practices such as contour farming, strip cropping, and grassed
waterways can help control erosion and protect topsoil.
5. Apply soil amendments: Adding organic matter to the soil can help improve soil health and
structure, which can in turn protect topsoil. Composting, using cover crops, and applying manure or
other organic fertilizers can help add organic matter to the soil.
6. Adopt appropriate tillage practices: Adopting conservation tillage practices such as reduced tillage
or no-till farming can help conserve topsoil and reduce erosion.
7. Implement soil testing and nutrient management: Soil testing can help determine the nutrient
needs of crops and prevent over-application of fertilizers, which can lead to soil degradation and loss
of topsoil.
8. Implement conservation programs: Government conservation programs such as the Conservation
Reserve Program (CRP) can provide incentives to farmers and landowners for implementing practices
that protect topsoil and promote soil health.
By implementing these actions, it is possible to conserve and protect topsoil, which is critical for
maintaining soil productivity and sustainability.
712 GATE Architecture and Planning: Comprehensive Question Bank

Q.41 State the importance and purpose of channelization of modern highways


Ans:- Channelization of modern highways is an important technique used in transportation
engineering to manage the flow of traffic on highways. The purpose of channelization is to separate
traffic into different lanes or channels, in order to reduce conflicts and improve safety on the roadways.
Some of the important purposes of channelization of modern highways include:
1. Improving traffic flow: Channelization helps in organizing traffic flow by providing dedicated lanes
for different types of vehicles, such as cars, buses, trucks, and bicycles. This can reduce congestion and
improve the overall efficiency of the transportation network.
2. Enhancing safety: Channelization helps to prevent head-on collisions, sideswipe accidents, and
other types of collisions that can occur when vehicles cross into oncoming lanes or other prohibited
areas. By separating traffic into different lanes or channels, the risk of accidents is reduced, and the
safety of drivers, passengers, and pedestrians is improved.
3. Facilitating pedestrian movement: Channelization can also include the use of sidewalks,
crosswalks, and other pedestrian facilities that allow people to safely cross busy highways. This can
improve accessibility and mobility for pedestrians and help to reduce accidents involving pedestrians
and motor vehicles.
4. Mitigating environmental impacts: Channelization can be used to reduce the impact of highways
on the environment, by providing landscaped medians, stormwater retention ponds, and other features
that help to mitigate the effects of runoff, noise, and other environmental impacts of highway traffic.
Q.42 Outlining the Vastupurush Mandala’discuss its applicability in modern urban structure.
Ans:- Vastupurush Mandala is an ancient Indian system of architectural design that is based on the
concept of a divine universal grid or energy called Vastupurusha. It is a system of design that is rooted
in Hindu mythology and philosophy and is used to create sacred and harmonious living spaces.
The Vastupurush Mandala is a square grid, divided into smaller squares, each representing a specific
deity or aspect of life. The central square is the most important and is often reserved for a deity or
an altar. The grid is then further divided into smaller sections, each of which is associated with a
particular direction, element, and deity.
In modern urban structures, the Vastupurush Mandala may not be applicable in its traditional form,
but its underlying principles can still be applied in various ways. Some ways in which the principles of
Vastupurush Mandala can be applied in modern urban structures are:
1. Orientation: The orientation of buildings can be designed to align with the cardinal directions,
which can have a positive impact on energy flow and the overall energy balance of the space.
2. Proportion and Scale: The principles of proportion and scale can be used to create a harmonious
balance between buildings and the surrounding environment.
3. Placement of Objects: The placement of objects within a space can be designed to create a sense of
balance and harmony.
4. Use of Natural Elements: The use of natural elements such as sunlight, wind, and water can be
incorporated into the design of buildings to create a harmonious and sustainable living space.
5. Functionality: The principles of Vastupurush Mandala can be applied to the functionality of spaces,
ensuring that they are designed in a way that is conducive to the well-being and productivity of those
who inhabit them.
Hemant Vilas Parulekar 713

Q.43 Comment on the performances of Urban Land Ceiling and regulation Act. and suggest requisite
reforms.
Ans:- The Urban Land Ceiling and Regulation Act (ULCRA) was enacted in 1976 to regulate the
ownership and use of urban land in India. The main objective of the Act was to ensure equitable
distribution of urban land and prevent the concentration of land in the hands of a few individuals or
entities. However, the Act did not achieve its intended objectives, and its implementation was marred
by several issues.
Some of the major issues with ULCRA include:
1. Delayed implementation: The Act was not implemented uniformly across all states, and some
states took a long time to implement the Act.
2. Exemptions: The Act had several exemptions, including exemptions for government agencies and
public sector enterprises. This led to the concentration of land in the hands of these agencies, defeating
the purpose of the Act.
3. Litigation: The Act was challenged in court by various stakeholders, leading to delays and
uncertainty in its implementation.
4. Land market distortions: The Act led to distortions in the land market, with a decrease in the
availability of land and an increase in land prices.
5. Corruption: The Act was prone to corruption, with several cases of bribery and illegal dealings
reported.
6. Given these issues, the ULCRA was repealed in 1999 and replaced with the Urban Land (Ceiling
and Regulation) Repeal Act, which removed the land ceiling provisions but retained the regulation of
urban land use.
7. Reforms that could be implemented to ensure effective regulation of urban land use include:
8. Streamlining the regulatory framework: The regulations governing urban land use should be
streamlined to ensure their effective implementation and to prevent inconsistencies and confusion.
9. Encouraging land pooling: Land pooling could be encouraged to ensure efficient use of land and
to prevent its concentration in the hands of a few individuals or entities.
10. Promoting public-private partnerships: Public-private partnerships could be promoted to ensure
the provision of affordable housing and other amenities in urban areas.
11. Enhancing transparency and accountability: Measures should be taken to enhance transparency
and accountability in the management of urban land, including the use of technology and data-driven
decision making.
12. Addressing corruption: Measures should be taken to prevent corruption and illegal dealings in
urban land use, including the use of technology and strengthening of enforcement mechanisms.
Q.44 State the ‘Elements of Public Interest’ in formulating Development control.
Ans:- In the context of development control, the “elements of public interest” refer to the various
aspects that need to be considered to ensure that a development project serves the public interest.
Some of these elements are:
1. Safety: Ensuring that the proposed development is safe for its occupants and the surrounding
community.
714 GATE Architecture and Planning: Comprehensive Question Bank

2. Health: Ensuring that the proposed development does not have any adverse health impacts on its
occupants or the surrounding community.
3. Environment: Ensuring that the proposed development does not have any negative environmental
impacts, such as pollution or destruction of natural habitats.
4. Aesthetics: Ensuring that the proposed development is visually appealing and contributes positively
to the character of the surrounding area.
5. Community: Ensuring that the proposed development is compatible with the needs and aspirations
of the surrounding community.
6. Economy: Ensuring that the proposed development supports the local economy and contributes to
economic growth.
7. Infrastructure: Ensuring that the proposed development has access to adequate infrastructure,
such as roads, utilities, and public transportation.
8. Land use: Ensuring that the proposed development is consistent with the existing land use patterns
and zoning regulations in the area.
Q.45 Suggest the characteristics of “Growth Centres” in regional planning.
Ans:- In regional planning, a “growth center” refers to a specific area or region that is identified for
targeted economic and social development. The characteristics of a growth center can vary depending
on the specific context, but generally include the following:
1. Strategic location: A growth center is often located in a geographically strategic location that makes
it well-connected to other areas and conducive to economic growth.
2. Infrastructure: The area should have adequate infrastructure, including transportation networks,
utilities, and communication facilities, to support growth and development.
3. Economic base: A growth center should have a strong economic base, which may include existing
industries, natural resources, or other assets that can be leveraged for further growth.
4. Employment opportunities: The area should offer a range of employment opportunities, including
both skilled and unskilled labor, to attract workers and stimulate economic activity.
5. Social amenities: Growth centers should have access to basic social amenities, such as healthcare
facilities, schools, and recreational facilities, to attract and retain a talented workforce.
6. Government support: Growth centers typically require strong government support, including
policies and incentives that encourage investment, innovation, and entrepreneurship.
Q.46 The total population of an area for 1981 and 1991 was given as 30 lakhs and 35 lakhs, respectively.
Extrapolate the expected population for 2001 based on two different methods.
a) Arithmetic mean
b) Geometric mean
Ans:- a) Arithmetic mean method: The arithmetic mean method assumes a linear increase in
population over time. The average annual increase in population can be calculated as:
(35 - 30) / 10 = 0.5 lakh per year
Using this growth rate, the expected population for 2001 can be calculated as:
35 + (0.5 x 10) = 40 lakhs
Hemant Vilas Parulekar 715

Therefore, the expected population for 2001 using the arithmetic mean method is 40 lakhs.
b) Geometric mean method: The geometric mean method assumes a constant rate of population
growth over time. The annual growth rate can be calculated as:
[(35 / 30)(1/10) - 1] x 100 = 1.8%
Using this growth rate, the expected population for 2001 can be calculated as:
35 x (1 + 0.018)10 = 47.5 lakhs
Therefore, the expected population for 2001 using the geometric mean method is 47.5 lakhs.
Q.47 Explain the merits and demerits of a large dam in regional planning.
Ans:- Merits of a large dam in regional planning:
1. Flood control: A large dam can help control flooding by storing excess water during heavy
rainfall and releasing it slowly.
2. Irrigation: A large dam can provide water for irrigation purposes, which can be a significant
boost to agricultural production in the region.
3. Hydroelectric power generation: A large dam can also be used to generate hydroelectric
power, which is a clean and renewable energy source.
4. Recreation: Large dams often create large reservoirs that can be used for recreational
purposes such as boating and fishing.
5. Water supply: large dams can provide a reliable source of water supply for drinking and
other domestic purposes.
6. Demerits of a large dam in regional planning:
7. Environmental impact: large dams can have a significant impact on the environment,
including the destruction of natural habitats and the displacement of wildlife and local
communities.
8. Cost: The construction of large dams can be awfully expensive, and the costs may not be
recouped through the sale of hydroelectric power or other benefits.
9. Geopolitical concerns: large dams can create geopolitical tensions between countries or
regions that share a common water resource.
10. Risk of failure: large dams can pose a risk of failure due to natural disasters or human error,
which can have catastrophic consequences for downstream communities.
11. Sedimentation: Over time, sediment can build up behind a dam, reducing its storage capacity
and potentially increasing the risk of flooding.
Q.48 An aircraft flying at an altitude of 5000 m above mean sea level takes aerial photographs of a
terrain having an average elevation of 1000 m above mean sea level.
a) Find the scale of photograph if focal length of camera is 20 cm.
b) Find the area covered in ground by each photo format of 23 cm x 23 cm.
716 GATE Architecture and Planning: Comprehensive Question Bank

Ans: a) 1:20,000
b) 2.116 x 107 sq m or 21.16 sq km
Solution: Given, Aircraft flight altitude = 5000 m amsl
Average terrain height = 1000 m amsl
Effective distance between the camera and terrain
=5000-1000 = 4000 m
a) Given, focal length of camera = 20 cm = 0.20 m
Scale of the photograph = focal length/ altitude above ground level
= 0.20/4000 = 1/20,000 = 1:20000
b) Given, photo size given = 23 cm x 23 cm = 0.23 m x 0.23 m
Actaual size = 0.23 x 20000 = 4600 m = 4.6 km
= 4.6 x 103 x 4.6 x 103 = 2.116 x 107 sq m
GATE QUESTION PAPER 1997
General Aptitude (GA)

Q.1.1 The colonial word ‘Bungalow’ is originated from


(A) Ahmadabad (B) Goa (C) Bengal huts (D) Rajasthan desert cottage
Ans:- (C)
Explanation: - The word “bungalow” is believed to have originated in India, derived from the Hindi
word “bangla” which means “Bengali-style house”. The style of house became popular during the late
19th and early 20th centuries in British India, particularly in the Bengal region.
Q.1.2 The ‘Piazza del Campo’ is an example of
(A) Medieval period (B) Roman period
(C) Greek period (D) Renaissance
Ans:- (B)
Q.1.3 The unit of measurement, cusecs or cu. ft/sec is equivalent to
(A) 15.48 gal/min (B) 0.353 cu.m/sec (C) 101.9cu.m./hr (D) None of the above
Ans:- (D)
Q.1.4 The fully urbanized country is
(A)Taiwan (B) Vatican (C) Thailand (D) Finland
Ans:- (B)
Q.1.5 The Programming Language particularly suitable for commercial application is
(A) BASIC (B) FORTRAN (C) COBOL (D) ALGOL
Ans:- (C)
Q.1.6 Control of phenols with heavy metal contamination is possible through
(A) Ion - exchange treatment (B) Stabilization Pond with water hyacinth
(C) Trickling filtration (D) Activated sludge process
Ans:- (B)
Q.1.7 Years Purchase’ in perpetuity can be computed by (where ‘n’ is number of years and ‘i’ is the rate
of compound interest)
(A) [(1+i)n-1/i] (B) [i/(1+i)n-1] (C) 1/I (D) (1+i)n
Ans: - (C)
Solution: - This is the present value of Re 1per annum forever, thus at 5 per YP perp is 20, which is the
sum of all the PV s from year 1 to infinity and the equation is commonly shown as:
YP perp =1/i
Where i= interest rate, or yield, expressed as a decimal rate, or
The calculation may be easier to undertake as 100 / Yield
The YP perp at 8% can be found from 100/8 = 12.5
The multiplier is used to capitalise the income flow from a freehold property investment, where the
current income is already the full market rent (MR) and thus in present values cannot be any higher.
718 GATE Architecture and Planning: Comprehensive Question Bank

Q.1.8 The ‘Campidoglio’ was planned by


(A)Michelangelo (B) George Washington
(C) Carlo Fontana (D) Palladio
Ans:- (A)
Q.1.9 The Landscape planner for the Versailles was
Richelieu (B) Andre Lenotre
(C) Vauban (D) Sir Christopher Wren
Ans: - (B)
Q.1.10 Two major components of gas generated through anaerobic digestion of sewage sludge are
(A) CO2 and H2 (B) CO and CH4 (C) CH4 and CO2 (D) H2 and CO
Ans:- (C)
Explanation: Methane (CH4) generated is 70% while carbon di oxide (CO2) is 30% This mixture is
known as Biogas
Q.1.11 In a simply supported bean, the maximum bending moment occurs at
(A) The point where shear force is zero (B)The point where shear force is maximum
(C) Mid span point (D) None of the above
Ans:- (A)
Q.1.12 The proportion and sizes of Greek Architecture - its scale was based on
(A) A module (B) Various parts of a building
(C) Optical corrections (D) Human measurements
Ans: - (B)
Q.1.13 Use of Aluminium handles is prohibited in steel door or window frames due to
(A) Risk of electrolytic corrosion (B) Difference in tensile strength
(C) Difference in Modulus of Elasticity (D) Difference in ductility
Ans:-(A)
Q.1.14 Walls and roofs should be massive and heavy in
Cold Climate (B) Hot - dry Climate
(C) Warm - humid Climate (D) Composite Climate
Ans:-(B)
Q.1.15 Barium plaster is employed as a final coat
(A) For the wall of X-ray room (B) For acoustical purpose
(C) For superior types of wall finishes (D) For extra-hard finish
Ans:-(A)
Q.1.16 Eopolis is
(A) A city (B) City is cosmos (C) A village (D) The universal city
Ans: - (C)
Hemant Vilas Parulekar 719

Q.1.17 The Aranya Project in Indore is planned by


U.C. Jain (B) B.V. Doshi (C) C.P.Kukreja (D) Amos Rappoport
Ans: - (B)
Q.1.18 U.S. Embassy in New Delhi was designed by
(A) Laurie Baker (B) Raj Rewal (C) Richard Neutra (D) Edward D. Stone
Ans: - (D)
Q.1.19 Largest urban area by population
(A) Peking (B) Mexico (C) Brasilia (D) Calcutta
Ans: - (B)
Q.1.20 Float valves are used in distribution reservoir or other tanks
(A) To maintain constant water level (B) To relieve excessive pressure
(C) To drain out sediments collected (D) To permit the air to enter
Ans: - (A)
Q.1.21 Fire tower is
(A) An arrangement of pipes (B) A tower open to sky
(C) A spiral chute (D) An enclosed staircase
Ans:-(D)
Q.1.22 Individual parking space for car is
(A)3 m X 6 m (B) 3.75 m X 7.5 m (C) 2.5 m X 5 m (D) 3 m X 5 m
Ans:-(A)
As per NBC minimum area required for Car Parking is 13.75 sqm and for two-wheeler 1.25
sqm.
Q.1.23 The maximum acceptable noise level inside library is
25 to 30 dB (B) 40 to 45 dB (C) 50 to 55 dB (D) 60 to 65 dB
Ans:-(B)
Explanation: Please refer GATE-2005, Q.33 for more information.
Q.1.24 ‘Perspective View’ concept was developed in Europe during
GATE-2005
Ans:-(C)
Q.1.25 The term ‘City of Light’ is associated with
Paris (B) Vienna (C) Karlsruhu (D) Canberra
Ans:-(A)
Q.1.26 Interstitial condensation refers to
(A) Surface Condensation (B) Dew point temperature
(C) Dry Bulb temperature (D) Relative humidity
Ans: - (B)
Interstitial condensation is a form of structural damping that occurs when warm, moist air penetrates
inside a wall, roof, or floor structure, reaches the dew point, and condenses into liquid water.
720 GATE Architecture and Planning: Comprehensive Question Bank

Q.1.27 A series of nearly equal - size cities in proximity is known as


(A) Satellite (B) Radio centric
(C) Constellation (D) Star
Ans:-(C)
Q.1.28 Architecture was defined as ‘Utilitas Firmitas Venustas’ by
Michelangelo (B) Vitruvius
(C) Leonardo da Vinci (D) Geoponti
Ans: - (B)
Explanation: - The definition of architecture as ‘Utilitas, Firmitas, Venustas’ was given by the Roman
architect Vitruvius in his influential work “De Architectura” (On Architecture), which was written in
the 1st century BC.
“Utilitas, Firmitas, Venustas,” often translated as “Utility, Firmness, and Delight,” is a famous
triad coined by the ancient Roman architect Vitruvius in his seminal work “De Architectura” (On
Architecture), which dates back to the first century BC. This triad has since become foundational in
the field of architecture.
Meaning and Interpretation:
Utilitas (Utility): This refers to the functionality or usefulness of a building. It means that a structure
must effectively serve the purpose for which it was built, meeting the needs of its occupants and
facilitating the intended activities.
Firmitas (Firmness): This pertains to the strength and durability of a building. A structure should be
stable and robust, built with quality materials and craftsmanship to endure the test of time and resist
environmental forces.
Venustas (Delight or Beauty): This aspect emphasizes the aesthetic appeal and beauty of the building.
It involves the design’s visual aspects, the harmonious arrangement of elements, and the overall artistic
impression, ensuring that the building is pleasing to the senses.
Impact on Architecture:
Holistic Approach: Vitruvius’ triad encapsulates a holistic approach to architecture, arguing that good
architecture should be a balanced blend of all three aspects.
Influence on Classical and Modern Architecture: These principles have profoundly influenced
classical architecture and continued to resonate through the Renaissance up to modern architectural
discourse.
Educational and Professional Standards: Vitruvius’ concepts are often cited in architectural education
and professional practice as guiding principles for good design.
Adaptation and Interpretation: Over the centuries, architects and scholars have interpreted and
adapted these principles in various ways to suit contemporary needs and contexts.
Contemporary Relevance:
In contemporary architecture, while the interpretation of “Utility, Firmness, and Delight” might vary,
the core idea of balancing practicality, durability, and beauty remains a cornerstone of architectural
design. This triad continues to inform and inspire architects in creating buildings that not only serve
functional needs but also contribute to the aesthetic and cultural landscape.
Hemant Vilas Parulekar 721

Q.1.29 Gray areas of the city are


(A) Service quarters of the city (B) Glamorous areas
(C) Slums (D) Blighted areas
Ans: - (C)
Q.1.30 When the contractor is asked to execute the various items of works specified in the schedule at the
tendered rates, it is called
(A) Lump sum contract (B) Cost plus a fixed percentage contract
(C) Unit - price contract (D) Labour contract
Ans:-(C)
Explanation:- A unit-price contract is a type of construction contract in which the contractor is
paid based on the quantities of work completed at pre-determined unit prices. The contractor is
usually required to submit a detailed schedule of prices for each item of work, and the payment to the
contractor is based on the actual quantities of work performed at the agreed-upon unit prices. This
type of contract is often used when the scope of work is uncertain, and the quantities of work cannot
be accurately estimated in advance. The unit-price contract allows the owner to pay the contractor
based on actual quantities of work performed, rather than estimating the cost of the entire project
upfront.
Q.2.1 Match the following:
Malabar Cement Township, Kerala (A) Raj Rewal
Indian Institute of Forest Management, Bhopal (B) Ralph Lerner
Sainik Guest House, Bhopal (C) Robert Millart
Yale University Art and a=Architecture building (D) Charles Correa
Parliament Library Extension (E) Le Corbusier
Indira Gandhi National Centre of Art (F) Anant d Raje
Bhubaneswar (G) Sen Kapadia
La Villa Radieuse (H) Otto Koenigsberger
Art Nouveau (I) Paul Rudolph
(j) Mushroom Construction (J) Victor Horta
Ans: - a - (D), b - (F), c - (G), d - (I), e - (A), f - (B), g - (H), h - (E ), i - (J), j - (C )
Q.2.2 Golden Ages of the Great Cities (A) Victor Gruen
The Landscape of Roads (B) Fisher Robrt M.
Design Fundamentals (C) Edmund N Bacon
The Emerging Urban Pattern (D) Crowe Sylvia
Design of Cities (E)Bowra, Sir Maurice
Tomorrow: a peaceful path to social reform (F) C.A. Doxiadis
New Horizon in Colour (G) Ebenezer Howard
Twenty Years of Public Housing (H) Robert Gillaim Scott
Mastering Turbo C (I) Faber Birren
Urban Renewal and the Future of the Urban Cities (J) Stan Kelly - Bootle
722 GATE Architecture and Planning: Comprehensive Question Bank

Ans: - a – (E), b – (C), c – (I), d – (D), e – (G), f – (F), g – (A), h – (H), i – (J), j – (J)
Q.2.3 Calistemonlanceletus (A) Heliodome
Isovist (B) Computing
Float (C) Landscape
Iteration (D) Activity
Sunpath (E) Bottle Brush
Ans: - a - (E), b - (C ), c - (D), d - (B), e - (A)
A single isovist is the volume of space visible from a given point in space, together with a specification
of the location of that point. Isovists are naturally three - dimensional, but they may also be studied in
two dimensions: either in horizontal section (“plan”) or in other vertical sections through the three -
dimensional Isovist.

Q.3 Distinguish between:


Turbidity and Colour in drinking water
Cactii and Succulent
Algas and Algicide
d - Base and LOTUS
Urban Space and Open Space
Evaluation and Evolution
Sheet Glass and Float Glass
Satellite town and Dormitory town
Alluvial soil and Black cotton soil
Physical distance and Perceptual distance
Ans: -
i. Turbidity and Colour in drinking water: Turbidity refers to the cloudiness or haziness of water
caused by suspended particles, while color in drinking water is caused by the presence of natural and
man-made substances such as decaying vegetation or industrial waste. Turbidity affects the clarity of
the water, while color affects its appearance.
ii. Cacti and Succulent: Cacti are a type of succulent plant that have areoles, which are small bumps
on the stem where spines, flowers, and new branches grow. Succulent plants store water in their leaves,
stems, and roots and can be cacti or non-cacti.
iii. Algae and Algicide: Algae are aquatic organisms that perform photosynthesis, while algicides are
chemicals used to kill algae. Algae can be beneficial or harmful, depending on the type and location,
while algicides are generally used to control harmful algal blooms in bodies of water.
iv. d-Base and LOTUS: d-Base and LOTUS are both computer database management systems, but
they were developed by different companies and have different features. d-Base is a relational database
management system designed to work with Windows operating systems, while LOTUS is a spreadsheet
application that was popular in the 1980s.
Hemant Vilas Parulekar 723

v. Urban Space and Open Space: Urban space refers to the built-up areas of a city, such as buildings,
roads, and other infrastructure, while open space refers to undeveloped or sparsely developed areas,
such as parks, forests, and farmland.
vi. Evaluation and Evolution: Evaluation refers to the process of assessing the quality, effectiveness,
or value of something, while evolution refers to the gradual development and change of something
over time.
vii. Sheet Glass and Float Glass: Sheet glass and float glass are both types of glass, but they are
manufactured using different processes. Sheet glass is made by pouring molten glass onto a flat surface
and then rolling it into a sheet, while float glass is made by pouring molten glass onto a bed of molten
metal, usually tin, and then allowing it to cool and solidify.
viii. Satellite town and Dormitory town: A satellite town is a smaller town located near a larger city
that provides housing and services for people who work in the city, while a dormitory town is a town
where people primarily live and commute to work in nearby cities.
ix. Alluvial soil and Black cotton soil: Alluvial soil is soil that has been deposited by rivers and streams
and is generally rich in nutrients, while black cotton soil is a type of heavy clay soil that is prone to
cracking and swelling, making it difficult to cultivate.
x. Physical distance and Perceptual distance: Physical distance refers to the actual distance between
two objects or locations, while perceptual distance refers to the psychological or cultural distance that
can exist between people or groups, such as differences in language, values, or beliefs.
SUB SECTION A 2
Q. 4 Briefly mention the issues to be considered while planning a terrace garden.
Ans: - Planning a terrace garden requires careful consideration of several issues to ensure its success.
Here are some important issues to consider:
1. Weight-bearing capacity: The terrace’s weight-bearing capacity should be assessed to determine
the maximum weight it can support. The garden’s design and materials used should be lightweight to
ensure that the terrace can handle the load.
2. Water drainage: Proper drainage must be ensured to avoid water accumulation and to prevent
damage to the terrace. The garden must have a water drainage system in place, such as a slope or a
drainage outlet.
3. Sun exposure: The amount of sunlight the garden will receive must be taken into account while
designing the terrace garden. The selection of plants should be based on the amount of sun exposure
the terrace receives.
4. Wind exposure: The garden must be able to withstand high winds, so the plants should be protected
from strong gusts of wind. The use of windbreaks and the choice of appropriate plants can help in this
regard.
5. Soil quality: The quality of the soil used in the garden must be considered, and the soil should be
well-draining and nutrient rich. The soil used in the garden can be a mix of compost, sand, and soil.
6. Watering: A watering system must be in place to ensure that the plants receive enough water.
Irrigation systems, such as drip irrigation or a hose, can be used to water the garden.
7. Maintenance: Regular maintenance of the terrace garden is essential to ensure its success. This
includes pruning, fertilizing, and pest control.
724 GATE Architecture and Planning: Comprehensive Question Bank

By considering these issues while planning a terrace garden, one can create a beautiful, healthy garden
that enhances the beauty and value of the terrace.
Q.5 Explain with sketches a town developed during the Greek period highlighting the major features.

Q.6 Illustrate with sketches distance relationship for seeing facade details of a building. Mention the
angles.

Q.7 A housing project is initiated to build 30,000 dwelling units of different categories (20% H.I.G.
land, site development, construction cost and overhead cost to be Rs. 1000 per sq.m, Rs. 500
per sq.m, Rs. 3000 per sq.m and Rs. 500 per sq.m respectively for the upper income group. The
total cost of the self-financing project is estimated as Rs. 62,250 lacs. Work out the various cost
components for EWS housing .
Ans: - Given data:
• Total number of dwelling units = 30,000
• Category-wise distribution: 20% H.I.G. (80 sq.m.), 30% M.I.G. (60 sq.m.), and 50% E.W.S.
(25 sq.m.)
• Cost of land, site development, and overhead cost = Rs. 1000 per sq.m.
• Construction cost for H.I.G. units = Rs. 3000 per sq.m.
• Construction cost for M.I.G. units = Rs. 2000 per sq.m.
• Construction cost for E.W.S. units = to be calculated
• Total cost of the project = Rs. 62,250 lacs
To work out the cost components for E.W.S. housing, we can use the following approach:
1. Determine the total area of E.W.S. units: Total area of E.W.S. units = 50% of 30,000 units
* 25 sq.m. per unit = 375,000 sq.m.
2. Determine the total cost of land, site development, and overheads for E.W.S. units: Total
cost of land, site development, and overheads for E.W.S. units = 375,000 sq.m. * Rs. 1000
per sq.m. = Rs. 37,500 lacs
3. Determine the total construction cost for the project: Total construction cost for the
project = Total cost of the project - Total cost of land, site development, and overheads =
Rs. 62,250 lacs - Rs. 37,500 lacs = Rs. 24,750 lacs
4. Determine the cost per unit for M.I.G. units: Cost per unit for M.I.G. units = (30% of
30,000 units * 60 sq.m. per unit * Rs. 2000 per sq.m.) / (30% of 30,000 units) = Rs.
36,000 per unit
5. Determine the cost per unit for H.I.G. units: Cost per unit for H.I.G. units = (20% of
30,000 units * 80 sq.m. per unit * Rs. 3000 per sq.m.) / (20% of 30,000 units) = Rs.
48,000 per unit
6. Determine the total cost for E.W.S. units: Total construction cost for E.W.S. units =
Total construction cost for the project - Total construction cost for M.I.G. units - Total
construction cost for H.I.G. units = Rs. 24,750 lacs - Rs. (36,000 per unit * 9,000 units) -
Rs. (48,000 per unit * 6,000 units) = Rs. 7,350 lacs
7. Determine the cost per unit for E.W.S. units: Cost per unit for E.W.S. units = Total
construction cost for E.W.S. units / (50% of 30,000 units) = Rs. 7,350 lacs / 15,000 units
= Rs. 49,000 per unit
Hemant Vilas Parulekar 725

Therefore, the various cost components for E.W.S. housing are as follows:
• Cost of land, site development, and overheads: Rs. 1000 per sq.m. = Rs. 37,500 lacs
• Construction cost: Rs. 3000 per sq.m. (for H.I.G.), Rs. 2000 per sq.m. (for M.I.G.), Rs. 980
per sq.m. (for E.W.S.)
• Cost per unit: Rs. 49,000 per unit
Q.8 Write an interactive programme in FORTRAN or BASIC for computation of V, where
Ans: -
SECTION B PART- I
Q. 9 Explain important architectural contribution made by Mies van der Rohe in his domestic building.
Ans: - Mies van der Rohe was a German-American architect who made important contributions
to modern architecture, particularly in the design of domestic buildings. He was a proponent of
simplicity and minimalism, and his designs are characterized by their use of clean lines, flat roofs, and
the innovative use of industrial materials.
One of Mies’ most notable domestic buildings is the Farnsworth House, located in Plano, Illinois.
Completed in 1951, the Farnsworth House is a masterpiece of modernist design, with a steel frame
and glass walls that blur the boundaries between interior and exterior space. The house is elevated on
stilts, giving it the appearance of floating above the surrounding landscape.
The Farnsworth House is a prime example of Mies’ philosophy of “less is more.” The design is simple
and uncluttered, with no unnecessary ornamentation or decoration. The focus is on the use of space
and light, with the glass walls creating a sense of openness and transparency that is characteristic of
modernist design.
Another important contribution made by Mies in his domestic buildings is his use of industrial
materials, particularly steel and glass. Mies was a pioneer in the use of steel framing, which allowed for
larger, more open interior spaces and thinner exterior walls. His innovative use of glass walls and the
development of the curtain wall system allowed for even greater transparency and a closer connection
between the interior and exterior of the building.
In summary, Mies van der Rohe made important architectural contributions in his domestic buildings
through his use of simplicity, minimalism, and innovative use of industrial materials. The Farnsworth
House is a prime example of his philosophy of “less is more,” and his use of steel framing and glass
walls revolutionized modernist design.
Q.10 Explain with sketches construction of gothic wall and vaults.
Ans:-
Q.11 Illustrate with sketches the difference between random rubble and ashlars works in stone Masonry.
Ans: -
Q.12 Draw conceptual diagram and label the different components of typical fire fighting system in an
apartment building above 24 m.
Ans: -
Q.13 An investor has capital of Rs 15 Lakhs from which he expects the return of 14.5 percent. He
intends to purchase a small workshop from which the net annual income is expected to become
Rs 5 Lakhs. Calculate the maximum price which the investor can invest for the workshop if money
can be borrowed in mortgage 16% for 8 years.
726 GATE Architecture and Planning: Comprehensive Question Bank

Ans: - To calculate the maximum price the investor can pay for the workshop, we can use the
capitalization method.
First, we need to calculate the net operating income (NOI) of the workshop, which is the expected
annual income minus any operating expenses:
NOI = Rs 5 Lakhs
Next, we need to calculate the capitalization rate, which is the rate of return that the investor requires
on their investment. This is given as 14.5% per annum.
Capitalization rate = 14.5%
Using these values, we can calculate the maximum price (P) that the investor can pay for the workshop
using the formula:
P = NOI / Capitalization rate
P = Rs 5 Lakhs / 0.145
P = Rs 34,48,275
This means that the investor can pay a maximum of Rs 34,48,275 for the workshop. However, the
investor only has Rs 15 Lakhs in capital. Therefore, they will need to borrow the remaining amount:
Amount to be borrowed = P - Capital
Amount to be borrowed = Rs 34,48,275 - Rs 15,00,000
Amount to be borrowed = Rs 19,48,275
Now, we can calculate the annual payment (A) required to pay back the loan over 8 years at an interest
rate of 16% per annum using the formula for present value of an annuity:
A = P * (r(1+r)n) / ((1+r)n - 1)
where r = interest rate per period = 16%/12 = 0.01333, n = number of periods = 8*12 = 96
A = Rs 19,48,275 * (0.01333*(1+0.01333)96) / ((1+0.01333)96 - 1)
A = Rs 3,05,655 (approx.)
Therefore, the investor can afford to pay an annual loan payment of Rs 3,05,655 to borrow the
remaining amount needed to purchase the workshop.
So, the maximum price which the investor can invest for the workshop is Rs 34,48,275 and they can
borrow Rs 19,48,275 at an annual payment of Rs 3,05,655 over 8 years.
Q.14 Draw a sun path diagram labeling all the elements in it for tropic. What information does a sun
path diagram convey to architects?
Ans: -
Q.15 Explain salient architectural features as evident in Qutub Minar.
Ans: - Qutub Minar is a 73-meter-high minaret located in Delhi, India. It was built by the Delhi
Sultanate in the 13th century, and it is one of the most significant examples of early Islamic architecture
in India. The tower has several distinctive architectural features, including:
1. Tapered Design: Qutub Minar is known for its tapering design, which means that the diameter of
the tower reduces as it rises. The lower part of the tower has a diameter of 14.3 meters, while the top
has a diameter of only 2.7 meters. This design creates a sense of height and grandeur and is one of the
most striking features of the tower.
Hemant Vilas Parulekar 727

2. Minaret Design: Qutub Minar is a minaret, which is a tower used for the Muslim call to prayer. The
tower is divided into five distinct sections, each marked by a projecting balcony or gallery. The balconies
are decorated with intricate carvings and inscriptions, which are unique to Islamic architecture.
3. Red Sandstone Construction: The tower is constructed entirely of red sandstone, which is a
common building material in India. The sandstone is richly carved and decorated with intricate
patterns and inscriptions. The use of red sandstone gives the tower a warm, welcoming appearance,
and the carvings add to its beauty.
4. Indo-Islamic Style: Qutub Minar is a unique example of the fusion of Islamic and Indian architectural
styles. The tower is decorated with Hindu and Jain motifs, which were common in the region before
the arrival of Islam. The intricate carvings and decorations on the tower reflect this fusion of styles and
give Qutub Minar a distinct cultural significance.
5. Quwwat-ul-Islam Mosque: The Qutub Minar complex also includes the Quwwat-ul-Islam Mosque,
which was built in the 12th century. The mosque is one of the earliest examples of Indo-Islamic
architecture and is known for its impressive arches and intricate carvings.
Q.16 Distinguish between ‘setting of cement’ and hardening of cement?
Ans: - Setting and hardening are two important processes that occur during the hydration of cement,
which is the chemical reaction that occurs when water is added to cement. The main differences
between setting and hardening of cement are:
1. Setting of Cement: Setting refers to the process by which the plastic cement paste solidifies into
a rigid mass. It is the point in time when the cement paste loses its plasticity and can no longer be
molded or shaped. Setting is caused by the chemical reaction between cement and water, which leads
to the formation of a gel-like substance that binds the aggregates together. The initial setting time of
cement is the time taken for the cement paste to attain sufficient rigidity to resist certain pressures,
while the final setting time is the time taken for the cement paste to become completely hard and lose
its plasticity.
2. Hardening of Cement: Hardening is the process by which the cement paste gradually gains strength
and hardens over time. It is a slow and continuous process that occurs over a period of several weeks
or months. Hardening is caused by the continued chemical reaction between cement and water, which
results in the formation of crystals that interlock and bind the cement paste together. The strength
of the hardened cement depends on the quality of the cement, the water-cement ratio, the curing
conditions, and other factors.
In summary, setting of cement is the process by which the cement paste solidifies into a rigid mass and
loses its plasticity, while hardening is the slow and continuous process by which the cement paste gains
strength and hardness over time.
Q.17 What is meant by ‘occult balance’? Illustrate it through a sketch within a frame of 10cm X 10cm.
Ans: - “Occult balance” refers to a design principle in which the overall composition of a work of art or
design appears balanced despite having uneven or asymmetrical elements. It is achieved using color,
texture, shape, and other visual elements to create a harmonious and visually pleasing composition
that may not have exact symmetry.
In architectural design, occult balance can be used to create a sense of equilibrium and stability in a
building’s facade, despite variations in its features, such as window placement or the size and shape of
decorative elements. It is often used in modern architecture, where designers may seek to create an
organic and dynamic visual effect while still maintaining a sense of balance and order.
Q.18 Highlight the principal of Jain temple planning.
728 GATE Architecture and Planning: Comprehensive Question Bank

Ans: - Jain temple planning is guided by a set of principles that have been developed over many
centuries. These principles are based on Jain philosophy and aim to create a spiritual and peaceful
environment that encourages meditation and reflection. Some of the key principles of Jain temple
planning are:
1. Symmetry: Jain temple architecture is based on a symmetrical plan. The temple is typically built
around a central axis or mandala, which represents the universe. The layout is usually square or
rectangular, with the main entrance located in the center of the main facade.
2. Proportions: Proportions play a key role in Jain temple planning. The temple is designed to be
proportional in all dimensions, with each element of the temple carefully measured and aligned to
create a harmonious and balanced whole.
3. Elements of the Temple: A Jain temple is typically divided into two main parts, the sanctum and
the hall. The sanctum is the innermost part of the temple and houses the main idol. The hall is the
outer part of the temple and is used for worship and meditation. The hall is also used for community
gatherings and other events.
4. Rituals: Jain temple planning takes into consideration the various rituals that are performed in
the temple. Special spaces are provided for the performance of specific rituals, and the design of the
temple is intended to facilitate the worshipper’s movement from one area to another.
5. Ornamentation: Jain temple architecture is known for its ornate and intricate designs. The temple
is often decorated with detailed carvings and intricate sculptures that represent various aspects of Jain
philosophy.
In summary, Jain temple planning is guided by principles of symmetry, proportions, the elements of
the temple, rituals, and ornamentation. These principles aim to create a peaceful and harmonious
environment that encourages reflection and meditation.
Q.19 Draw the Bending Moment and shear Force diagrams of the following:
Pic
Ans: -
Q.20 Critically compare between two well known network analysis techniques ‘PERT’ and ‘CPM’ with
respect to architectural projects.
Ans: - PERT (Program Evaluation and Review Technique) and CPM (Critical Path Method) are both
project management tools used to plan and manage complex projects. While there are some similarities
between the two techniques, there are also some differences that are worth noting.
Similarities:
• Both techniques are used to identify the critical path of a project, which is the sequence of
activities that must be completed on time for the project to be completed on schedule.
• Both techniques use network diagrams to represent the activities of a project and their
relationships to each other.
• Both techniques use estimates of activity durations and dependencies to calculate the
expected time to complete a project.
Hemant Vilas Parulekar 729

Differences:
• PERT allows for uncertainty in activity durations by using three estimates of the time required
for each activity: optimistic, pessimistic, and most likely. The expected duration of an activity
is then calculated as a weighted average of these three estimates. CPM, on the other hand, uses
a single estimate of activity duration.
• PERT is often used for projects where there is a high degree of uncertainty, such as research
and development projects, while CPM is often used for projects with well-defined, routine
tasks, such as construction projects.
• PERT emphasizes the critical path of a project, while CPM is more concerned with the total
project duration.
• PERT is more flexible than CPM, as it allows for changes to activity duration estimates and
the addition or deletion of activities without having to recompute the entire network diagram.
In the context of architectural projects, both PERT and CPM can be used to plan and manage complex
construction projects. PERT may be more appropriate for projects with a high degree of uncertainty,
such as those involving innovative design concepts or new construction techniques. CPM may be
more appropriate for projects with well-defined tasks and predictable outcomes, such as building
renovation projects.
Ultimately, the choice between PERT and CPM depends on the nature of the project, the level of
uncertainty involved, and the specific needs of the project team.
Q.21 Design and illustrate a general classroom for 40 students in hot - humid climate, and mention the
design criteria adopted.
Ans:-
Q.22 Outline the usefulness of building Bey - laws.
Ans: - Building by-laws are a set of regulations that specify the minimum standards for the construction
of buildings and other structures. These by-laws are developed and enforced by local government
bodies, such as municipalities, to ensure that buildings are constructed in a safe, sustainable, and
aesthetically pleasing manner. The following are some of the key benefits of building by-laws:
1. Safety: Building by-laws specify the minimum standards for structural stability, fire safety, electrical
and plumbing systems, and other aspects of building construction to ensure that buildings are safe for
their occupants and the public.
2. Sustainability: Building by-laws may include provisions for the use of sustainable building materials,
energy-efficient systems, and waste reduction, to promote environmental sustainability and reduce
the building’s carbon footprint.
3. Aesthetics: Building by-laws may include provisions for the exterior design of buildings, such as
height, setbacks, materials, and colors, to ensure that they are visually appealing and harmonious with
the surrounding environment.
4. Consistency: Building by-laws provide a standardized set of guidelines for the construction of
buildings, which ensures that all buildings in a particular area are constructed to a consistent standard.
5. Dispute Resolution: Building by-laws provide a framework for resolving disputes between
neighbors or between builders and neighbors, such as disputes related to boundary walls, drainage,
and the location of structures.
730 GATE Architecture and Planning: Comprehensive Question Bank

6. Legal Compliance: Building by-laws ensure that buildings are constructed in compliance with
relevant national and local building codes, which can help to prevent legal disputes and ensure that
buildings are eligible for insurance coverage.
Q.23 Highlight, with suitable examples, the characteristics features of Spanish garden.
Ans: - Spanish gardens are characterized by their beautiful, ornate designs, and use of water, shaded
spaces, and greenery. Some of the key features of Spanish gardens include:
1. Use of water features: Spanish gardens often feature beautiful fountains, ponds, and other water
features. These elements provide a cooling effect and create a peaceful atmosphere in the garden.
2. Ornate architecture: Spanish gardens typically feature beautiful, ornate architecture, such as arches,
columns, and intricate tile work. These elements help to create a sense of grandeur and luxury in the
garden.
3. Use of courtyards: Spanish gardens often include courtyards, which are outdoor spaces surrounded
by buildings or walls. These courtyards provide a sense of privacy and seclusion, and often feature
beautiful water features and gardens.
4. Shaded spaces: Spanish gardens typically feature shaded areas, such as pergolas or covered walkways.
These shaded spaces provide relief from the heat and create a more comfortable environment in which
to relax and enjoy the garden.
5. Use of greenery: Spanish gardens are often filled with lush greenery, including trees, shrubs, and
flowering plants. The use of greenery helps to create a sense of tranquility and natural beauty in the
garden.
Some famous examples of Spanish gardens include:
1. The Alhambra in Granada, Spain: This beautiful palace and fortress complex features several
stunning gardens, including the Generalife gardens, which are known for their beautiful water features
and intricate tile work.
2. The Real Alcázar in Seville, Spain: This royal palace features several beautiful gardens, including
the stunning Patio de las Doncellas, which is surrounded by ornate architecture and features a beautiful
pool.
3. The Casa de Pilatos in Seville, Spain: This beautiful palace features a stunning courtyard,
surrounded by ornate architecture and beautiful gardens, including a lush central lawn and stunning
water features.
Q.24 Mention the factors to be considered for thermal insulation of buildings
Ans: - Thermal insulation is an important aspect of building design that helps to maintain a comfortable
indoor environment while reducing energy consumption. The factors that need to be considered for
thermal insulation of buildings include:
1. Climate: The insulation requirements vary depending on the climate of the region. For example, a
building in a cold climate would require more insulation than a building in a warmer climate.
2. Building orientation: The orientation of the building with respect to the sun and wind can affect
the amount of heat gained or lost through the walls, roof, and windows.
3. Building materials: The type and thickness of the building materials used can affect the insulation
properties of the building envelope.
Hemant Vilas Parulekar 731

4. Building design: The design of the building can impact its insulation properties. For example, a
building with a high ceiling may require more insulation than a building with a low ceiling.
5. Ventilation: Proper ventilation is important for maintaining good indoor air quality and preventing
moisture buildup, which can reduce the effectiveness of insulation.
6. Moisture control: Moisture can reduce the effectiveness of insulation, so it is important to design
the building envelope to prevent moisture buildup and to include proper moisture barriers.
7. Energy efficiency: The building should be designed to be energy-efficient, with proper insulation,
air sealing, and efficient heating and cooling systems.
Building codes and regulations: Building codes and regulations set standards for minimum insulation
requirements, which must be considered during the design and construction of the building.
8. Cost: The cost of insulation and related materials must be considered when determining the level
of insulation to be used.
Q.25 A studio has dimension 10 m. X 8 m. X 5 mother ceiling of studio is provided with acoustical tiles
having absorption coefficient = 0.40 .Curtains in heavy folds ( absorption coefficient = 0.50) are
provided on one short wall. The absorption power of other surfaces of the studio may be taken as
8 sq.m. sabins. What will be extra absorption units required to make reverberation time t = 0.75
sec?
Ans: - To calculate the extra absorption units required to achieve a reverberation time of 0.75 seconds,
we can use the Sabine equation:
V = (0.161 x V) / (S x t)
where:
V = volume of the studio (in cubic meters)
S = total surface area of the studio (in square meters)
t = desired reverberation time (in seconds)
Given:
Length of the studio (L) = 10 m
Width of the studio (W) = 8 m
Height of the studio (H) = 5 m
Absorption coefficient of acoustical tiles (α1) = 0.40
Absorption coefficient of curtains (α2) = 0.50
Absorption power of other surfaces (A) = 8 sabins
Desired reverberation time (t) = 0.75 seconds
Calculations:
Total surface area of the studio, S = 2(L x H) + 2(W x H) + L x W
= 2(10 x 5) + 2(8 x 5) + 10 x 8
= 100 + 80 + 80
= 260 sq.m.
732 GATE Architecture and Planning: Comprehensive Question Bank

Total absorption coefficient of the studio, α = (α1 x area of tiles) + (α2 x area of curtains) + A
= (0.40 x 100) + (0.50 x 8) + 8
= 40 + 4 + 8
= 52
Volume of the studio, V = L x W x H
= 10 x 8 x 5
= 400 cubic meters
Using the Sabine equation, we can calculate the initial reverberation time of the studio:
t0 = (0.161 x 400) / (260 x 52) = 2.02 seconds
To achieve a reverberation time of 0.75 seconds, we need to reduce the initial reverberation time by:
Δt = t0 - t
= 2.02 - 0.75
= 1.27 seconds
To calculate the extra absorption units required, we can use the modified Sabine equation:
ΔA = (S x Δt) / 0.16 - V – αS
where: ΔA = extra absorption units required (in sabins)
S = total surface area of the studio (in square meters)
Δt = reduction in reverberation time required (in seconds)
V = volume of the studio (in cubic meters)
α = total absorption coefficient of the studio
Substituting the values, we get:
ΔA = (260 x 1.27) / (0.16 - 400 - 52 x 260) = 82.3 sabins (approx.)
Therefore, an extra absorption of approximately 82.3 sabins is required to achieve a reverberation time
of 0.75 seconds in the studio.
Q.26 What is colour harmony? How it differs from Colour Contrast? Give some examples.
Ans: - Color harmony refers to the use of color combinations that are pleasing to the eye and create
a sense of balance and unity. It is achieved by combining colors that are adjacent to each other on the
color wheel (analogous colors), or by using colors that are opposite each other on the color wheel
(complementary colors). The goal of color harmony is to create a sense of cohesion and balance in a
design or artwork.
Color contrast, on the other hand, refers to the use of colors that are significantly different from each
other in hue, value, or saturation. Color contrast can be used to create visual interest, draw attention
to certain elements, and create a sense of energy and excitement in a design or artwork.
Here are some examples of color harmony:
1. Analogous color scheme: This is when colors that are adjacent to each other on the color wheel are
used together. An example would be using shades of green and yellow in a design.
Hemant Vilas Parulekar 733

2. Monochromatic color scheme: This is when different shades and tints of the same color are used
together. An example would be using different shades of blue in a design.
3. Complementary color scheme: This is when colors that are opposite each other on the color wheel
are used together. An example would be using red and green in a design.
Here are some examples of color contrast:
1. Warm and cool colors: This is when warm colors (like red and orange) are paired with cool colors
(like blue and green) to create contrast and balance.
2. Light and dark colors: This is when light colors (like white and pastels) are paired with dark colors
(like black and deep jewel tones) to create contrast and drama.
3. High and low saturation colors: This is when highly saturated colors (like bright red and blue) are
paired with desaturated colors (like gray and beige) to create contrast and interest.
Q.27 Explain the function of ‘BLOCK’ and ‘WBLOCK’ commands in Autocad highlighting their
attributes.
Ans: - In AutoCAD, the BLOCK and WBLOCK commands are used to create reusable objects or
blocks of geometry that can be inserted into a drawing as a single object rather than drawing each
object individually.
The BLOCK command creates a block from a selection of objects in a drawing, which can be saved
and used later in the same drawing or in other drawings. The BLOCK command offers the option to
retain or delete the original objects in the drawing. Attributes can be added to the block that allow the
user to enter and save data related to the block.
On the other hand, the WBLOCK command creates a block that can be saved as a separate file, which
can be inserted in other drawings or used as a library of reusable objects. The WBLOCK command
offers the option to select the specific objects or parts of a drawing to be included in the block.
In summary, the main differences between BLOCK and WBLOCK are:
1. BLOCK creates a block that is saved within the same drawing, while WBLOCK creates a separate
file that can be used in multiple drawings.
2. BLOCK offers the option to retain or delete the original objects in the drawing, while WBLOCK
allows the user to select the specific objects or parts of a drawing to be included in the block.
3. BLOCK allows for the addition of attributes that can save data related to the block, while WBLOCK
does not have this option.
Examples of using the BLOCK command could be creating blocks for commonly used details, such
as door and window details, or electrical symbols. Examples of using the WBLOCK command could
be creating a library of blocks for specific types of equipment or furniture that can be inserted into
multiple drawings.
Q.28 Draw the sketches of ‘Balanced Cantilever’ and a ‘Propped Cantilever’ highlighting their functions.
Ans: -
SECTION B PART - II
Q. 9 Outline the cause of soil degradation as major concern on terrestrial environment.
734 GATE Architecture and Planning: Comprehensive Question Bank

Ans: - Soil degradation is a major concern for the terrestrial environment as it can lead to reduced
crop yields, desertification, and land degradation, among other negative effects. Some of the major
causes of soil degradation include:
1. Deforestation: When forests are cleared for agriculture or urbanization, the soil is exposed to wind
and water erosion, leading to the loss of topsoil and nutrients.
2. Overgrazing: Overgrazing by livestock can lead to soil compaction, soil erosion, and loss of vegetation
cover, which can in turn lead to desertification and land degradation.
3. Intensive agriculture: Intensive farming practices such as monoculture, heavy use of pesticides and
fertilizers, and tillage can lead to soil erosion, soil compaction, and loss of soil fertility.
4. Industrial activities: Industrial activities such as mining, oil and gas exploration, and construction
can lead to soil compaction, erosion, and contamination.
5. Climate change: Climate change can exacerbate soil degradation by causing more frequent and
severe droughts, floods, and storms, which can lead to soil erosion and loss of soil fertility.
These causes of soil degradation can have significant impacts on the environment, including loss of
biodiversity, reduced soil productivity, and increased vulnerability to natural disasters. It is important
to implement sustainable land management practices to prevent soil degradation and preserve the
health of the terrestrial environment
Q.10 Comment on the existing mechanism of devolution of urban development fund between State
and Local Governments.
Ans: - In India, the devolution of urban development funds between state and local governments is
governed by the recommendations of the Finance Commissions, which are constitutional bodies that
make recommendations to the President of India on the distribution of tax revenues between the
central government and the state governments, as well as the distribution of grants-in-aid to the states.
The Fourteenth Finance Commission, which was in effect from 2015-2020, recommended an increase
in the share of tax revenues devolved to the states from 32% to 42%, with the aim of providing more
funds to local governments for urban development and other public services. The Commission also
recommended the creation of a new urban local body grant to be provided to local governments to
support basic urban services.
However, there have been concerns about the effectiveness of the existing mechanism for devolution
of funds, particularly with regards to the amount of funds actually reaching the local governments and
the lack of clarity and transparency in the allocation process. In addition, there have been criticisms
that the current system places too much emphasis on the distribution of funds, rather than on ensuring
that the funds are used effectively and efficiently by local governments.
Overall, while the devolution of urban development funds in India has been an important step towards
empowering local governments, there is a need for further reforms and improvements to ensure that
the funds are being used effectively and efficiently to support sustainable urban development.
Q.11 Stating the scope of ‘MILE STONE TECHNIQUE’, prepare a mile stone chart schedule for
residential site planning project.
Hemant Vilas Parulekar 735

Ans: - The milestone technique, also known as milestone chart or Gantt chart, is a project scheduling
tool used to break down a project into smaller tasks, and to schedule these tasks over time. It helps in
tracking the progress of the project and identifying critical paths, dependencies and delays.
For a residential site planning project, the following milestones could be included in a milestone chart
schedule:
1. Site Survey: This milestone involves conducting a survey of the site, including topography, soil type,
vegetation, and existing structures.
2. Conceptual Design: This milestone involves developing a conceptual design for the residential site,
based on the findings of the site survey and the client’s requirements.
3. Approval of Conceptual Design: This milestone involves obtaining approval for the conceptual
design from the client and the relevant authorities.
4. Detailed Design: This milestone involves preparing detailed designs for the site, including layouts,
elevations, and specifications.
5. Preparation of Estimates: This milestone involves preparing detailed cost estimates for the project,
based on the detailed design.
6. Procurement of Materials: This milestone involves procuring the necessary materials for the project,
such as construction materials and equipment.
7. Construction: This milestone involves carrying out the construction work according to the design
and specifications.
8. Inspection and Quality Control: This milestone involves carrying out inspections and quality control
checks during the construction process to ensure that the work is of the desired quality.
9. Completion of Construction: This milestone involves completing the construction work and
obtaining approval from the relevant authorities.
10. Handover: This milestone involves handing over the completed project to the client.
The milestone chart schedule could be presented in the form of a Gantt chart, with the tasks listed on
the left-hand side and the timeline represented on the top. Each task would be represented by a bar,
with the length of the bar representing the duration of the task. The critical path, which represents
the sequence of tasks that must be completed on time in order to complete the project on time, can be
highlighted using color coding or other visual aids. The chart can be updated as the project progresses,
with actual dates and progress being compared to the planned schedule to identify any delays or
deviations from the plan.
Q.12 Briefly discuss the system adopted for urban solid waste disposal.
Ans: - The system adopted for urban solid waste disposal generally involves the following steps:
1. Collection: This involves the gathering and transportation of waste from the point of generation to
a centralized location. Collection can be done through door-to-door collection, communal bins, or
mobile collection vehicles.
2. Transportation: The collected waste is transported to a processing or disposal site. The mode of
transportation may include trucks, trains, or barges.
736 GATE Architecture and Planning: Comprehensive Question Bank

3. Processing: The collected waste is sorted and processed to remove any recyclable materials, such as
plastics, glass, and metals. Organic waste may be composted to produce fertilizer, while non-recyclable
waste is prepared for final disposal.
4. Disposal: The final step involves the disposal of the waste in a landfill, incinerator, or other designated
facility. Landfills are the most commonly used method of waste disposal, where waste is buried in
specially designed pits and covered with soil.
In recent years, there has been a growing emphasis on waste reduction and resource recovery, with
many cities adopting integrated waste management systems that prioritize recycling, composting, and
waste-to-energy technologies. These systems aim to reduce the amount of waste sent to landfills and
minimize the environmental impact of solid waste disposal.
Q.13 Discuss the elements of PERT to establish its logical network in scheduling projects.
Ans: - PERT (Program Evaluation and Review Technique) is a project management tool used to
schedule and manage complex projects. The elements of PERT include:
1. Activity: An activity is a specific task or work that needs to be completed as part of the project.
Activities are identified and listed in the work breakdown structure (WBS).
2. Node: A node represents the beginning or end of an activity in the PERT chart. Each node is
represented by a circle or a box, and is labeled with the activity name, start date, and end date.
3. Predecessor: A predecessor is an activity that must be completed before another activity can start.
Predecessors are identified and linked in the PERT chart.
4. Successor: A successor is an activity that cannot start until a previous activity is completed. Successors
are identified and linked in the PERT chart.
5. Duration: The duration is the amount of time required to complete an activity. It is usually expressed
in days or weeks.
6. Critical Path: The critical path is the longest path of linked activities that must be completed on time
in order for the project to be completed on schedule.
To establish the logical network in scheduling projects using PERT, the following steps can be followed:
1. Identify all the activities required to complete the project and list them in the WBS.
2. Determine the dependencies between the activities, i.e., which activities need to be completed
before others can start.
3. Estimate the duration of each activity, taking into account any constraints or limitations.
4. Create a PERT chart, using circles or boxes to represent each activity, and linking them with arrows
to show the dependencies.
5. Calculate the earliest start and finish times, and the latest start and finish times for each activity,
using the forward and backward pass methods.
6. Identify the critical path, which is the longest path of linked activities that must be completed on
time for the project to be completed on schedule.
7. Monitor the progress of the project by updating the PERT chart with actual start and finish times
and adjusting the schedule as necessary to keep the project on track.
Hemant Vilas Parulekar 737

Q.14 Evaluate the Fagence techniques of Peoples participation in planning.


Ans: - The Fagence techniques of people’s participation in planning were developed by Australian
planner Tom Fagence in the 1970s. Fagence believed that participation should be an integral part of
the planning process, and that the public should be involved in all stages of planning.
The techniques developed by Fagence include:
1. Opinion Polls: This technique involves conducting surveys to gather public opinion on specific
issues. The data collected is then used to guide planning decisions.
2. Workshops: This technique involves organizing workshops with stakeholders to discuss issues and
brainstorm solutions. The workshops can be used to generate ideas, identify problems, and develop
action plans.
3. Focus Groups: Focus groups are small, structured discussions that involve a specific target group.
They are used to gather detailed information about the group’s attitudes, beliefs, and opinions on
specific issues.
4. Citizen Panels: Citizen panels involve a randomly selected group of citizens who are brought
together to discuss specific issues. The group is representative of the wider population and is used to
gauge public opinion on specific issues.
5. Deliberative Polling: Deliberative polling is a more complex technique that involves a representative
sample of the public being brought together for an extended period to discuss issues in depth. The
process is designed to help participants become more informed about the issues and to engage in
deliberation.
Q.15 Explain through sketch a typical plan and section of Pour - Flush Water seal Latrine.
Ans: -
Q.16 Briefly outline the contents of a Development plan.
Ans:- A development plan, also known as a master plan or a comprehensive plan, is a long-term
policy document that outlines a city or region’s vision for future growth and development. It typically
contains the following key elements:
1. Land Use Plan: This outlines the proposed land use pattern of the area, including residential,
commercial, industrial, institutional, recreational, and open space zones.
2. Transportation Plan: This identifies the existing and proposed transportation infrastructure,
including roads, highways, railways, airports, and public transit, and addresses issues related to traffic
management and mobility.
3. Housing Plan: This addresses the current and future housing needs of the population, including
affordable housing, slum rehabilitation, and public housing programs.
4. Environmental Plan: This outlines strategies for the protection and conservation of natural
resources, including land, water, air, flora, and fauna, and addresses issues related to pollution control
and environmental management.
5. Infrastructure Plan: This identifies the existing and proposed infrastructure, including water supply,
sanitation, drainage, solid waste management, power supply, and communication networks.
6. Economic Plan: This outlines strategies for promoting economic growth and development, including
the development of industrial and commercial zones, employment generation programs, and tourism
promotion.
738 GATE Architecture and Planning: Comprehensive Question Bank

7. Social Infrastructure Plan: This addresses the social infrastructure needs of the population, including
education, health, and cultural facilities.
8. Implementation Plan: This outlines the institutional arrangements and the mechanism for plan
implementation, including the role of various stakeholders, financial resources required, and
monitoring and evaluation mechanisms.
Q.17 (a) Illustrate to the point, the significance of Landform in Landscape.
(b) Mention the factors which govern the decision related to Landform.
Ans: -
Q.18 There are four routes from a person’s house to his work place. There are two parking lots near his
office building. His office building has 4 entrances, there elevators to reach his office floor, and a
corridor from each to his office door.
(a) How many ways can the person reach his office door?
(b) While starting from home he realizes that route 1 and 3 are no entry, parking lot 1 is closed for
the day elevator no. 2 is on repair. What is probability of selecting route no. 4, parking at lot no. 2
use the cast entrance and use elevator 1?
Ans: -
Q.19 Distinguish between the ‘Spacing’ and ‘Headway’ in traffic stream and mention the way can be
measured.
Ans:- In traffic engineering, “spacing” and “headway” are two terms that refer to different aspects of
traffic flow.
Spacing refers to the distance between successive vehicles in a traffic stream. It is measured as the
length of the vehicle plus the distance between it and the front of the next vehicle. Spacing is important
because it determines the capacity of a roadway, and it affects the speed and safety of traffic. In general,
a greater spacing between vehicles leads to a higher capacity, but lower speed and lower safety, while a
smaller spacing leads to lower capacity, but higher speed and higher safety.
Headway, on the other hand, refers to the time interval between successive vehicles in a traffic stream.
It is the time it takes for one vehicle to pass a fixed point, and the next vehicle to pass the same point.
Headway is also important in determining the capacity of a roadway, as well as the speed and safety of
traffic. A shorter headway allows for more vehicles to pass through a given point per unit time, but it
can also increase the risk of accidents and decrease the speed of traffic.
Both spacing and headway can be measured using various methods. For example, spacing can be
measured using sensors such as radar or laser detectors, or by visual estimation. Headway can be
measured using time-lapse photography, or by using detectors that measure the time between vehicles
passing a fixed point. Accurate measurements of spacing and headway are important for understanding
traffic flow and for designing effective traffic management strategies.
Q.20 Discuss the Professional Code of Conduct of a planner to his fellow Professional.
Ans:- The professional code of conduct of a planner to their fellow professionals is important to
maintain the ethical standards and integrity of the planning profession. The following are some key
elements of the professional code of conduct for planners:
1. Respect for colleagues: A planner should respect the knowledge, skills, and contributions of
their colleagues in the planning profession, and should foster an atmosphere of mutual respect and
collaboration.
Hemant Vilas Parulekar 739

2. Honest and truthful communication: A planner should communicate with their colleagues in an
honest and truthful manner and should not engage in deceitful or misleading behaviour.
3. Confidentiality: A planner should respect the confidential nature of information shared by their
colleagues and should not disclose such information without consent or a legal obligation to do so.
4. Avoidance of conflicts of interest: A planner should avoid situations where they may have a conflict
of interest with their colleagues and should disclose any potential conflicts of interest that may arise.
5. Professional development: A planner should continually develop their skills and knowledge through
education and training and should encourage their colleagues to do the same.
6. Respect for professional boundaries: A planner should respect the boundaries of their professional
role and responsibilities and should not engage in behaviour that exceeds their expertise or authority.
7. By adhering to a professional code of conduct, planners can maintain the trust and confidence of their
colleagues and the public and uphold the ethical standards and integrity of the planning profession.
Q.21 State the major data inputs from Satellite Remote Sensing System for Urban Planning.
Ans:- Satellite remote sensing systems provide a variety of data inputs that are useful for urban
planning. Some of the major data inputs are:
1. Land cover classification: Satellite imagery can be used to classify land cover types, such as built-up
areas, vegetation, water bodies, and bare land. This information is important for understanding the
spatial distribution of urban areas and their environmental impact.
2. Urban morphology: Remote sensing data can provide information on the spatial configuration
of urban areas, such as building density, height, and arrangement. This can help in analyzing urban
growth patterns, identifying areas of urban decay, and assessing the need for new infrastructure.
3. Population and demographic data: Remote sensing data can be used to estimate population density
and demographic characteristics such as age, gender, and income. This information is useful for urban
planning and decision-making, particularly in areas such as public health, transportation, and housing.
4. Environmental data: Remote sensing data can provide information on environmental factors such
as air quality, water quality, and land degradation. This information is useful for identifying areas that
are vulnerable to environmental hazards and for designing strategies to mitigate their impact.
5. Transportation data: Remote sensing data can be used to monitor traffic patterns and congestion, as
well as to identify areas where new transportation infrastructure is needed.
6. Infrastructure data: Remote sensing data can provide information on the location and condition
of urban infrastructure such as roads, bridges, buildings, and utilities. This information is important
for assessing the need for maintenance and repair, as well as for planning new infrastructure projects.
Q.22 Draw a schematic layout of a ‘Trumpet Interchange’ showing direction of flow of traffic Mention
at what situation it is more appropriate.

Q.23 Highlight the importance of G.I.S. (Geographic Information System) in the planning decision
making.
Ans:- Geographic Information System (GIS) is a powerful tool for planning decision making as it
allows planners to analyze and visualize spatial data in ways that were previously not possible. Here
are some of the ways in which GIS is important for planning decision making:
740 GATE Architecture and Planning: Comprehensive Question Bank

1. Spatial Analysis: GIS can perform complex spatial analysis such as proximity analysis, buffer
analysis, and overlay analysis. These analyses allow planners to better understand the relationships
between different spatial features and identify patterns, trends, and relationships that may not be
apparent otherwise.
2. Data Management: GIS allows planners to organize, store, and manage vast amounts of spatial data.
This makes it easy for planners to access, update, and share data across different departments and
organizations, ensuring that everyone is working with the same information.
3. Visualization: GIS allows planners to visualize spatial data in different ways such as maps, charts,
and graphs. This makes it easier to communicate complex information to stakeholders and decision
makers.
4. Scenario Planning: GIS can be used to model different scenarios and test their impact on the
environment and society. This allows planners to identify the most effective strategies for achieving
their goals while minimizing negative impacts.
5. Decision Support: GIS can be used as a decision support tool to evaluate alternative courses of action
and make informed decisions. By combining data from different sources, GIS can provide planners
with a comprehensive view of a particular issue, helping them to make more informed decisions.
Q.24 Analyze ‘St. Mark’s Square, Venice’ from the urban design point of view, highlighting the merits
and achievements.
Ans:- St. Mark’s Square in Venice is an outstanding example of urban design, both in its historical
significance and its modern-day role as a popular public space. The design of the square represents
an integration of architectural, urban, and social concepts, creating a space that is both beautiful and
functional.
One of the key merits of St. Mark’s Square is its overall design and layout. The square is roughly
rectangular in shape, and its dimensions are proportionate and well-balanced. The buildings
surrounding the square are of varying heights and styles, which adds visual interest and variety to the
space. The square also has several key landmarks, including St. Mark’s Basilica, the Doge’s Palace, and
the Campanile, which help to anchor and orient visitors.
Another notable achievement of the square’s design is its use of space. Despite its large size, St. Mark’s
Square feels intimate and inviting. This is due in part to the use of arcades, which provide a sense of
enclosure and shelter. The square also has a number of smaller spaces and courtyards, which offer
more intimate areas for socializing or relaxing.
St. Mark’s Square is also a prime example of successful urban design in terms of its social function. The
square serves as a hub of social activity in Venice, drawing locals and tourists alike to enjoy its many
attractions. The space is well-suited for a wide range of activities, including outdoor concerts, festivals,
and markets. Additionally, the square’s cafes and restaurants provide a gathering place for people to
socialize and enjoy the scenery.
Overall, St. Mark’s Square is an excellent example of urban design that combines beauty, function,
and social purpose. The square’s design is well-balanced and proportionate, and its use of space and
landmarks helps to create a sense of intimacy and orientation. Additionally, the square serves as a
vibrant social hub, bringing people together to enjoy its many attractions and amenities.
Hemant Vilas Parulekar 741

Q.25 The distance between two points in a map on 1: 100,000 is 2 cm. Distance between the same two
points in an aerial photograph is 10 cm. The camera of the aerial photograph was flown with a
focal length of 6 inches. Find out the scale of the photograph and calculate the flying height.

Q.26 Highlight the Demographic characteristics for delineation of a Region.


Ans:- Demographic characteristics are often used to delineate a region. Some of the important
demographic characteristics that can be used for this purpose are:
1. Population: The total number of people living in a particular area or region is an important
demographic characteristic. It helps in understanding the size of the population and its growth rate.
2. Age distribution: The age distribution of the population can provide insights into the social and
economic needs of the region. For example, a region with a large elderly population may have different
requirements for healthcare services and housing than a region with a younger population.
3. Gender distribution: The gender distribution of the population can also be important, especially
in regions where there are significant differences in the roles and responsibilities of men and women.
4. Ethnicity and race: Ethnicity and race can be important demographic characteristics, particularly in
areas where there are significant cultural differences between groups.
5. Education levels: Education levels can provide insights into the workforce and the potential for
economic development in a region. Regions with higher education levels may have a more skilled
workforce and be more attractive to businesses.
6. Income levels: Income levels can provide insights into the economic well-being of the region and
the potential for economic growth. Regions with higher income levels may have more resources for
development.
7. Occupation and employment status: Occupation and employment status can provide insights into
the types of jobs available in the region and the skills required for those jobs.
These demographic characteristics can help planners and policymakers to develop targeted policies
and programs that address the specific needs of a region.
Q.27 Illustrate with sketches the major features of Mohenjo-Daro Plan
Ans: -
Q.27 A property has been sold by the Housing Board on a conditional sale. The Board is to receive
Rs. 24,000 at the end of every year for 10 years, and further, the Board is to receive an amount of
Rs. 200.000 at the end of 10 years. A period of 4 years has already lapsed. Estimate the current
value of the property (Interest rate for Years Purchase is 8%).
Ans:- To estimate the current value of the property, we need to calculate the present value of all future
cash flows. We can use the formula for the present value of an annuity and the present value of a lump
sum to do this calculation.
First, let’s calculate the present value of the annuity of Rs. 24,000 per year for 10 years. We can use the
formula:
PV = PMT x ((1 - (1 + r)(-n)) / r)
where PV is the present value, PMT is the payment per period, r is the interest rate, and n is the
number of periods.
742 GATE Architecture and Planning: Comprehensive Question Bank

In this case, PMT = Rs. 24,000, r = 0.08 (8%), and n = 10. Plugging in these values, we get:
PV1 = Rs. 24,000 x ((1 - (1 + 0.08)(-10)) / 0.08) = Rs. 169,197.17
This is the present value of the annuity of Rs. 24,000 per year for the next 10 years.
Next, let’s calculate the present value of the lump sum of Rs. 200,000 that will be received at the end of
the 10-year period. We can use the formula:
PV = FV / (1 + r)n
where PV is the present value, FV is the future value, r is the interest rate, and n is the number of
periods.
In this case, FV = Rs. 200,000, r = 0.08, and n = 10. Plugging in these values, we get:
PV2 = Rs. 200,000 / (1 + 0.08)10 = Rs. 85,873.40
This is the present value of the lump sum of Rs. 200,000 that will be received at the end of the 10-year
period.
Now, let’s add the present values of the annuity and the lump sum to get the total present value of the
property:
Total PV = PV1 + PV2 = Rs. 169,197.17 + Rs. 85,873.40 = Rs. 255,070.57
Therefore, the current value of the property is approximately Rs. 255,070.57.
GATE QUESTION PAPER 1996
General Aptitude (GA)

No question seeking sketches or drawings or numerical, is included in this paper.


Q. 1 – Q. 30 carry one mark each.
Q.1.1 ‘New Town In town’ concept involves
(A) Increasing the population density in the city centre
(B) Provision of large-scale recreational facilities in city core
(C) Encouraging strip commercial within the city core
(D) Large scale rehabilitation, modernization and redevelopment of the city core.
Ans:- (D)
Explanation:
• The ‘New Town In Town’ concept is a strategy for urban redevelopment that focuses on
revitalizing and modernizing existing city cores. This approach often includes upgrading
infrastructure, constructing new buildings, and introducing modern amenities and services.
• Unlike simply increasing population density or adding recreational facilities, it aims for a
comprehensive transformation of the urban core to make it more livable, sustainable, and
economically vibrant.
• This approach can also encompass preserving historical elements while integrating them
with new developments, creating a balanced and dynamic urban environment.
Q.1.2 An ‘Aquiclude’ is
(A) Confined bad of impervious material between acquifers
(B) Perched acquifer
(C) Artesian acquifer
(D) Large water body underground
Ans:- (A)
Explanation:
• An aquiclude is a body of rock or a layer of sediment that is highly impermeable to water.
It acts as a barrier to the flow of groundwater.
• This impervious nature means that aquicludes can confine groundwater in aquifers,
creating conditions for confined or artesian aquifers above or below them. However, they
themselves do not transmit water effectively and thus do not function as aquifers.
• The other options – perched aquifer (B), artesian aquifer (C), and large water body
underground (D) – are types of aquifers or descriptions of groundwater occurrences, not
aquicludes.
744 GATE Architecture and Planning: Comprehensive Question Bank

Q.1.3 If ‘a’ is the optimistic time ‘b’ is the pessimistic time ‘m’ is the most likely time of an activity, the
expected time of the activity is
(A) (a+5m+b)/6 (B) (a+4m+b)/6 (C) (a+2m+b)/6 (D) (a+m+b)/6
Ans: - (B)
Solution: The expected time of an activity in project management, particularly in the context of PERT
(Program Evaluation and Review Technique), is calculated using the formula:
Expected Time=(a+4m+b) / 6
• aa is the optimistic time estimate (the shortest time in which the task can be completed).
• bb is the pessimistic time estimate (the longest time the task might take).
• mm is the most likely time estimate (the most probable duration of the task).
Given the options, the correct answer is:
(B) (a + 4m + b) / 6
Q.1.4 The most commonly used base for timber painting is
(A) Red lead (B) White lead (C) Titanium white (D) Zinc oxide
Ans: - (B)
Explanation:
• White Lead: Historically, white lead (basic lead carbonate) was a common base for
timber painting. It provided excellent coverage and protection for the wood. However, it’s
important to note that the use of lead-based paints has significantly declined due to health
and environmental concerns associated with lead.
• Red Lead: While red lead has been used as a primer, especially in metal applications, it is
less common for timber.
• Titanium White: This has become a popular alternative in more modern times, especially
as a pigment, due to its brightness and high refractive index.
• Zinc Oxide: This is used in paints as a pigment and for its mildew-resistant properties, but
it is not as common as a base specifically for timber.
It’s important to mention that due to the toxic nature of lead-based paints, their use has been limited
or banned in many countries, and safer alternatives like Titanium White and Zinc Oxide are preferred
in contemporary applications.
Q.1.5 Absorption of sound in porous material occurs mainly due to
(A) Creep loss (B) Vacuum in holes
(C) Frictional loss (D) None of the above
Ans: - (C)
When sound from a loudspeaker collides with the walls of a room part of the sound’s energy is reflected
and part is absorbed into the walls. As the waves travel through the wall, they deform the material
thereof (just like they deformed the air before). This deformation has mechanical losses which convert
part of the sound energy into heat through acoustic attenuation, mostly due to the wall’s viscosity. The
same attenuating mechanics apply for the air and any other medium through which sound travels.
Hemant Vilas Parulekar 745

Q.1.6 In sewers, velocity of flow should not be


(A) More than self cleansing velocity (B) Less than velocity of water at flushing
(C) Less than dry water flow velocity (D) Less than self cleansing velocity
Ans:- (D)
Q.1.7 As per Indian Road Congress the ‘Width of formation of highway in plain land’ is
(A) 6.0 m (B) 12.0 m (C) 18.0 m (D) 21.0 m
Ans: - (B)
Explanantion: - The width of road or width of carriage way for single lane road is 3.75, for intermediate
lane road 5.5 m, two lanes without raised kerb is 7 m, two lanes with raised kerb are 7.5 m and for
multi- lane road width per lane is 3.5 m.
Q.1.8 In a lecture auditorium the seating pattern from the speaker should fall within maximum angle of
(A) 700 (B) 900 (C ) 1200 (D) 1400
Ans :- (D)
Explanation: The furthest distance a person can be from the theatre area should not be more than 400
times the size of the smallest letter or digit being written. If the smallest letter measures 2.5 cm, then
the maximum distance that the student should be from the letter should not exceed 1000 cm, which
is equivalent to 10 metres. Projection screen’s visible range: The distance from the screen should be
between two and six times the width of the screen image. The ideal viewing angle should not exceed a
deviation of 30° or 150° from the level of the eyes. If the width of the screen image is 1.5 metres, then
the nearest viewer should be positioned at a distance of 3 metres, while the farthest viewer should be
positioned at a distance of 9 metres. The most accurate response is 140°.
Q.1.9 When two pigments of contrasting value are mixed, the most striking change observed is in
(A) Hue Dimension (B) Occult rhythm
(C) Occult Balance (D) None of the above
Ans: - (A)
Explanation: - When two pigments of contrasting value (lightness or darkness) are mixed together,
the most striking change that is observed is typically in the hue dimension. This is because mixing
pigments of contrasting value can alter the perception of the colors and shift the hue towards the
color that is dominant in the mixture. For example, if a light-yellow pigment is mixed with a dark
blue pigment, the resulting mixture may appear greenish in hue because the blue is dominant and the
yellow is less prominent. The change in hue can be less noticeable if the two pigments are closer in
value or if the amount of one pigment in the mixture is much smaller than the other. Occult rhythm
and occult balance are not related to the mixing of pigments.
Q.1.10 In meandering river the location of intake point for city water supply should be
(A) On concave bank (B) On convex bank
(C) Down stream point of waste water disposal (D) Middle of the river
Ans:- (D)
In meandering river the intake should not be located on curves or atleast on sharp curves. Intake is
located on upstream side of the city to reduce contamination due to sewer outfall.
746 GATE Architecture and Planning: Comprehensive Question Bank

Q.1.11 Number of common clay bricks required to make one cubic meter of brick masonry is
(A) 350 (B) 420 (C) 500 (D) 550
Ans:- (C) 500
No of bricks = (1 / 0.2 x 0.1 x 0.1) = 500 bricks.
For India, a brick of standard size 190 mm X 90mm X 90mm is recommended by the BIS. With
mortar thickness, the size of such a brick becomes 200 mm X 100 mm X 100 mm and it is known as
the nominal size of the modular brick .Thus the nominal size of brick includes the mortar thickness.
Refer Gate 2004, Q. 39 for more information.
Q.1.12 Sinking fund refers to
(A)Reserve fund (B) Fund loss due to damage
(C) Bad debts (D) Fund for underground construction
Ans:- (A)
Explanation: - A sinking fund is a reserve fund set aside by an organization or government to pay off
debt or replace assets at the end of their useful life. The sinking fund is used to save money over time
so that when the debt or replacement costs come due, the organization has the funds necessary to meet
its obligations without having to resort to borrowing or selling assets. Sinking funds can also be used
to provide for other long-term expenses or investments. It is important to note that sinking funds are
not related to losses due to damage, bad debts, or funds for underground construction.
Q.1.13 BASICE is a
(A) Compiler (B) Hardware item (C) Interpreter (D) Plotter type
Ans:- (A)
Explanation: BASICE is an interpreter, which is a type of computer program that translates and
executes code one line at a time. Specifically, BASICE is an interpreter for the BASIC programming
language, which was one of the earliest and most widely used programming languages in the early
days of personal computing. BASIC stands for “Beginner’s All-purpose Symbolic Instruction Code,”
and was designed to be easy to learn and use for non-expert programmers. While BASICE is no longer
in widespread use today, BASIC remains an important part of computing history and has influenced
many other programming languages that followed. BASICE is not a compiler, hardware item, or plotter
type.
Q.1.14 Acidity of soil is indicated by
(A) BOD (B) pH (C)GL (D) CLO
Ans:- (B)
Explanation:
• pH: The pH scale measures the acidity or alkalinity of a substance. A pH value below 7
indicates acidity, a pH value of 7 is neutral, and a pH above 7 indicates alkalinity. Therefore,
the pH value of soil is used to determine its acidity.
• BOD (Biochemical Oxygen Demand): This is a measure of the amount of oxygen required
by bacteria to decompose organic matter in water. It’s not used to indicate soil acidity.
• GL (Ground Level): This term refers to the elevation of the ground surface and has no
relevance to soil acidity.
• CLO: This doesn’t relate to a commonly recognized measure for soil acidity.
Hemant Vilas Parulekar 747

Q.1.15 The unit measurement for space in urban society is


(A) Family (B) Group of family (C) Community (D) Individual
Ans:- (A)
Explanation:
• Community: This term refers to a group of people living in the same place or having a
particular characteristic in common. In urban planning and sociology, the community
often serves as a fundamental unit of measurement for space, as it encompasses the social
and spatial interactions of groups of families or individuals.
• Family, Group of Family, and Individual are also important units in urban society, but
they usually refer to smaller subsets within a community. The concept of “community” is
broader and more encompassing in the context of urban space and its usage.
Q.1.16 DIM in Auto-CAD refers is
(A) Dimension of array (B) Dimension of function
(C) Brightness of visual screen (D) None of the above
Ans:- (B)
Q.1.17 Sense of movement in a visual composition can be achieved by
(A) Disturbing the visual balance (B) Placing elements in diagonal arrangement
(C) Placing elements symmetricall (D) Overlapping elements
Ans:- (B)
Q.1.18 Minimum strength of cement mortar used in load bearing brick masonry is
(A) 50 N/cm2 (B) 60 N/cm2 (C) 80 N/cm2 (D) 100N/cm2
Ans:- (A)
Explanantion: In load-bearing brick masonry, cement mortar is used to bind the bricks together
and provide structural stability to the wall. The minimum strength of cement mortar used in load-
bearing brick masonry is typically specified by the relevant building codes or standards. In general, the
strength of cement mortar is expressed in terms of its compressive strength, which is the amount of
pressure that the mortar can withstand before it fails. For load-bearing brick masonry, the minimum
compressive strength of cement mortar is usually specified as 8 N/mm2 (80 N/cm2) or higher. This
ensures that the mortar is strong enough to support the weight of the wall and resist other types of
forces, such as wind loads or seismic forces.
Q.1.19 If ‘P’ denotes the total population, the age dependency ratio expressed as
(A)P(60+) / P(0 -59) (B) P(0 -14) / P(15 -59) (C) P(60+)+P(0 -14) / P(0 -59) (D) P(60+) + P(0 -14) / P(15-59)
Ans:- (D)
Explanation:
The age dependency ratio is a measure used to indicate the ratio of dependents (people younger than
15 or older than 64) to the working-age population (ages 15-59).
It is calculated by adding the number of individuals aged 60 and above (P(60+)) and those aged 0-14
(P(0-14)) and then dividing this sum by the working-age population, which is typically considered to
be those aged 15-59 (P(15-59)).
This ratio provides insight into the burden on the working-age population to support the younger and
older segments of the population, which are generally considered economically dependent.
Therefore, option (D) is the correct expression for calculating the age dependency ratio.
748 GATE Architecture and Planning: Comprehensive Question Bank

Q.1.20 Concept of self supporting ‘Industrial Town’ was proposed by


(A) Lewis Mumford (B) Henry Wright (C) Robert Owen (D) Ebenezer Howard
Ans: - (C)
Q.1.21 Of the following types of forests, which one is least likely to be destroyed by fire
(A) Deciduous forest (B) Coniferous Forest
(C) Rain forest (D) Broad leaf evergreen forest
Ans:- (C)
Q.1.22 Workability of concrete mix with low water - cement ratio is determined by
(A) Slump test (B) Tensile strength test
(C) Flexural strength test (D) Compaction factor test
Ans:- (A)
Q.1.23 In hot dry climate the form and planning of settlement should be
(A) High rise high density (B) Low rise high density
(C) Low rise low densit (D) High rise low density
Ans: - (B)
Q.1.24 The extent of wind shadow on the leeward side is more dependent on
(A)Height of the building (B) Depth of building
(C) Width of building (D) All of the above
Ans: - (D)
Q.1.25 ‘Letchworth’ was designed based on the concept of
(A) New town (B) Garden city
(C) Factory town (D) Linear city
Ans: - (B)
Q.1.26 ‘Jharokha’ is an architectural element used in building as
(A) Gateway (B) Balcony
(C) Column decoration (D) Ceiling decoration
Ans: - (B)
Q.1.27 The concept ‘Architecture as an expression of inner structure’ is attributed to
(A) Alvar Alto (B) Mies van der Rohe
(C) Walter Gropius (D) Le Corbusier
Ans:- (B)
Q.1.28 Psychometric scale deals with
(A) Subjective weighting (B) Frequency of sound
(C) Thermal comfort (D) Colour intensity
Hemant Vilas Parulekar 749

Ans: - (C)
Explanation: A psychrometric chart is a graphical representation of the physical and thermal properties
of moist air. It’s a tool used in HVAC (Heating, Ventilation, and Air Conditioning) and meteorology
to understand air conditions and the relationships between various air parameters. The chart is used
for a variety of calculations including heating, cooling, humidifying, and dehumidifying processes.
Key Components of a Psychrometric Chart:
1. Dry Bulb Temperature: This is the air temperature measured by a regular thermometer, typically
on the horizontal axis of the chart.
2. Wet Bulb Temperature: Measured by a thermometer with a wet wick over the bulb, it represents the
lowest temperature that can be reached by the evaporation of water only.
3. Relative Humidity (RH): Indicated by curved lines on the chart, it represents the percentage of
moisture in the air compared to the maximum amount the air can hold at a particular temperature.
4. Dew Point Temperature: The temperature at which air becomes saturated with moisture and dew
begins to form.
5. Specific Humidity or Moisture Content: Represented by vertical lines on the chart, it indicates the
amount of water vapor contained in the air.
6. Enthalpy: Lines of constant enthalpy (total heat content of the air) are often diagonally drawn
across the chart.
How to Use a Psychrometric Chart:
Locate the State Point: Determine the dry bulb temperature and either the wet bulb temperature, the
dew point, or the relative humidity. The intersection of these values on the chart is the state point of
the air.
Trace Processes: HVAC processes like heating, cooling, humidification, or dehumidification are
represented by moving from one state point to another along specific lines (e.g., constant humidity,
constant enthalpy).
Analyze Air Properties: Once the state point is identified, you can read off other properties like
specific humidity, enthalpy, and density.
Applications:
HVAC Design and Analysis: To determine the proper air conditioning, heating, and ventilation
requirements.
Industrial Processes: In industries where air conditioning and control of humidity are critical (e.g.,
pharmaceuticals, textiles).
Weather Forecasting: To understand atmospheric conditions.
Challenges:
Complexity: Interpreting a psychrometric chart requires understanding of thermodynamics.
Accuracy: The precision of the chart depends on how accurately the initial parameters (dry bulb, wet
bulb temperatures, etc.) are measured.
In summary, the psychrometric chart is a valuable tool for understanding and analyzing the properties
of air-water vapor mixtures and is essential for professionals in HVAC, meteorology, and various
industrial applications.
750 GATE Architecture and Planning: Comprehensive Question Bank

Q.1.29 The concept of ‘La Ciudad Linear’ is associated with


(A) Le Corbusier (B) Tony Garnier
(C) S.Y.Mata (D) Robert Owen
Ans:- (C)
Q.1.30 The term ‘Megalopolis’ was coined by
(A) Lewis Mumford (B) Patrick Geddes
(C ) Norman Cousins (D) Jean Gottman
Ans:- (B)
Q.2.1 Match the following:
(a)Exterior design in Architecture (A) MargeretRobers
(b)Urban Landues Planning (B) Brian Hackett
(c)Image of a city (C) Gorden Cullen
(d)Architecture through the ages (D) Stuart Chapin
(e)Space Time and Architecture (E) Yoshinobu Ashihara
(f)Earthscape( (F) Paul D Spreiregen
(g)Townscape (G) S.Giedion
(g)The Architecture of Towns and Cities (H) Talbot Hamlin
(i)An introduction to Town Planning Techniques (I) Kevin Lynch
(j)Planting Design (J) John O Simon
Ans:- a -(E ) , b - (D) , c - (I) , d - (H) , e - (G) , f - (J) , g - (C ) , h - (F) ,i - (A) , j - (B)
Q.2.2 Match the following:
(a) Cavity Wall (A) Population
(b)Plumb (B) B.M.Fuller
(c)Goose Neck (C) Sabine
(d)Chaitya (D) Arch
(e)Anemometer (E) Rural Community
(f)Deodesic Dome (F) Prayer Hall
(g)Histogram (G) Fire resistance
(h)Voussoir (H) Wind Force
(i)Reverberation (I) Masonry
(j) Eopolis (J) Water supply
Ans:- a - (G) , b - (I) , c - (J) , d - (F) , e - (H) , f - (B) , g - (A) , h - (D) , i - (C ) , j - (E )
Hemant Vilas Parulekar 751

Q.3 Select out of the given list of architects, the ones associated with the design of each of the following
ten buildings:
(a)VictoriaTerminus , Bombay (A) A.P.Kanvinde
(b)India International center,Delhi (B) Pierre Jenneret
(c)Centre for Development Studies Thiruvananthpuram (C) Aditya Prakash
(d)Kashmir Conference Complex , Srinagar (D) Hasmukha Patel
(e)I.I.M. Bangalore (E ) Madhav Nayak
(f)Gandhibhavan, Chandigarh (F) Le Corbusier
(g)Raj Bhavan , Calcutta (G) B.V.Doshi
(h)Hall of Nations, Pragati Maidan (H) Laurie Baker
(i)Tamilnadu Legislative Complex, Madras (I) F.W. Stephens
(j)Sriram Centre, New Delhi (J) U C Jain
(K) Habib Rehman
(L) J.A.Stein
(M) Raj Rewal
(N) Ram Sharma
(O) Shiv Nath Prasad
(P) D.A. Nair
(Q) Kuldip Singh
(R) Charles Correa
(S) Leo Pereira
(T) Charles Wyatt
Ans:- a - (I), b - (L), c - (H), d - (L), e - (G), f - (B), g - (T), h -( M), i - ( , j -(O)
Q.4 What do the following abbreviations stand for?
(a) NBO : National Building Organisation
(b)IDSMT: Integrated Development of Small and Medium Towns
(c)PCU : Passenger Car Unit
(d)BPE :? Before Present Era (Question might be BPL - Below Poverty Line)
(e) HDFC :Housing Development Financial Corporation

Q.5 Distinguish between


i . Landmark and Node ii .Cul - de - sac and Loop street
iii. Land use and Land Cover iv. Loudness of sound and Intensity of sound
v. Grey value contrast and Colour contrast
752 GATE Architecture and Planning: Comprehensive Question Bank

Ans: Here are the distinctions between each pair of terms:


i. Landmark and Node:
• Landmark: A landmark is a recognizable physical feature or object that serves as a point of
reference in a landscape. It is often distinctive in appearance and easily identifiable, like a
building, statue, or natural feature.
• Node: A node, in urban planning, is a focal point or a junction within a city, often a major
intersection or a central place where activities converge and people frequently travel to and
from.
ii. Cul-de-sac and Loop Street:
• Cul-de-sac: A cul-de-sac is a street that is closed at one end, forming a dead-end. It is often
used in residential areas to limit through traffic and provide a quiet street environment.
• Loop Street: A loop street is a type of road that curves around and reconnects with itself,
forming a loop. It allows for continuous traffic flow and is often found in suburban and
residential areas.
iii. Land Use and Land Cover:
• Land Use: Land use refers to the human utilization of land. It indicates how people have
allocated different areas for specific purposes such as residential, commercial, agricultural,
industrial, etc.
• Land Cover: Land cover refers to the physical and biological cover over the surface of the
land, including vegetation, water, desert, urban infrastructure, etc. It represents the earth’s
surface as seen from space.
iv. Loudness of Sound and Intensity of Sound:
• Loudness of Sound: Loudness is a subjective perception of sound pressure. It is how we
perceive the strength of sound and can vary based on the listener’s hearing and the sound’s
environment.
• Intensity of Sound: Intensity is an objective measurement of the power carried by a sound
wave per unit area. It is scientifically quantifiable and is not dependent on the listener’s
perception.
v. Grey Value Contrast and Colour Contrast:
• Grey Value Contrast: This refers to the contrast between different shades of grey, ranging
from white to black. It is important in monochrome images or designs where color is not
used.
• Colour Contrast: Colour contrast refers to the difference in color properties such as hue,
saturation, and brightness. It is used to create visual interest and readability, especially in
colored images or designs.
These distinctions highlight different aspects and concepts in urban planning, sound science, and
visual design.
SUB SECTION A 2
Q.6 Write a simple interactive programme in BASIC or FORTRAN to compute the volume of regular cone.
Ans: - 10 PRINT “Enter radius and height of the cone”
20 INPUT r,h
30 v = 1/3 * 3.14 * r2 * h
40 PRINT “Volume of the cone is “; v
50 END
Hemant Vilas Parulekar 753

Q.7 State, to the point, the environmental attributes of trees in urban planning context.
Ans: - Trees have numerous environmental attributes in the urban planning context. Some of these
include:
1. Air purification: Trees absorb carbon dioxide and other pollutants, while releasing oxygen into the
atmosphere, thus improving air quality.
2. Cooling effect: Trees provide shade, which helps to reduce the urban heat island effect by lowering
ambient temperatures.
3. Stormwater management: Trees can help to mitigate the impact of stormwater runoff by absorbing
and retaining rainfall, reducing the amount of water that enters the stormwater system.
4. Carbon sequestration: Trees act as carbon sinks, storing carbon in their biomass, which helps to
mitigate the effects of climate change.
5. Biodiversity: Trees provide habitat for a wide range of urban wildlife, including birds, insects, and
small mammals.
6. Aesthetics: Trees are aesthetically pleasing, and can enhance the visual appeal of urban spaces, while
also contributing to a sense of community pride and identity.
Q.8 Mention basic pattern of city forms and illustrate them with diagrammatic sketches.

Q.9 State the effects of pollution on microclimate in an industrialized urban area.


Ans: Pollution in industrialized urban areas significantly affects the microclimate in various ways:
1. Temperature Changes (Urban Heat Island Effect): Pollution, especially from industrial activities
and vehicular emissions, contributes to the urban heat island effect. The concentration of pollutants
and heat-emitting sources leads to higher temperatures in urban areas compared to surrounding rural
areas.
2. Reduced Air Quality: Industrial emissions, including particulate matter, sulfur dioxide, nitrogen
oxides, and volatile organic compounds, deteriorate air quality. This can lead to smog formation and
reduced visibility.
3. Increased Humidity: Certain pollutants, particularly those from industrial processes, can increase
atmospheric moisture levels. This is because some pollutants act as nuclei around which moisture
condenses, leading to higher humidity levels.
4. Altered Precipitation Patterns: Pollution can affect local weather patterns, potentially leading
to altered precipitation. For example, the presence of aerosols and particulates can influence cloud
formation and rainfall distribution.
5. Radiation Balance Disruption: Pollutants such as aerosols and greenhouse gases affect the
radiation balance. They can absorb or reflect solar radiation, leading to changes in temperature and
other climatic conditions at the micro level.
6. Wind Pattern Changes: Urban structures combined with heat and pollution can alter wind patterns.
Buildings and pollutants can obstruct and channelize wind flows, changing the local wind patterns.
7. Health Impacts on Flora and Fauna: Pollutants can have detrimental effects on local flora and
fauna. For instance, smog and acid rain, a result of industrial pollutants, can damage trees, affect
photosynthesis, and disrupt local ecosystems.
754 GATE Architecture and Planning: Comprehensive Question Bank

8. Increased Ozone at Ground Level: Pollutants from industrial and vehicular emissions can lead
to increased ozone levels at the ground level, which is harmful to living organisms and affects the
microclimate.
Overall, the effects of pollution in industrialized urban areas are multifaceted, affecting temperature,
air quality, humidity, precipitation, radiation balance, and wind patterns, all of which contribute to
changes in the microclimate.
Q.10 Compute the capacity and work out the dimension of a septic tank for a small colony of 500 persons
with average daily sewage flow of 75 litres per head. Detention period is 36 hours. Cleaning
interval is six months. Assume rate of deposited sludge as 25 litres per capita per year.

SECTION B (PART- I)
Q.11 What is the influencing factor for the development of regional style in Indo - Islamic architecture?
Ans: - The development of regional style in Indo-Islamic architecture was influenced by a variety
of factors, including local building traditions, the availability of building materials, and the cultural
and religious preferences of the ruling elite. The regional styles emerged because of the interaction
between local architectural traditions and the Islamic architectural vocabulary introduced by the
Muslim rulers. Over time, these regional styles developed into distinct schools of architecture that
were associated with specific regions of the Indian subcontinent, such as the Mughal style in northern
India and the Deccani style in southern India. The development of regional styles in Indo-Islamic
architecture reflects the cultural and religious diversity of the Indian subcontinent, and it continues to
inspire architects and designers to this day.
Q.12 Discuss, to the point, the contributions made by Le Corbusier in Modern Architecture.
Ans: - Pollution can have several effects on microclimate in an industrialized urban area, including:
1. Temperature increase: Air pollution can trap heat in the atmosphere, leading to an increase in local
temperature. This effect is called the “heat island” effect, and it can be particularly pronounced in
urban areas with high levels of pollution.
2. Precipitation changes: Air pollution can also affect precipitation patterns in the local area. Polluted
air can prevent clouds from forming, leading to a decrease in precipitation. Alternatively, pollution
can cause acid rain, which can damage buildings and vegetation in the local area.
3. Reduced visibility: Air pollution can reduce visibility in the local area, making it more difficult to
see and navigate through the environment.
4. Health effects: Pollution can have a range of negative health effects on people living in the area,
including respiratory problems, cardiovascular disease, and cancer.
5. Vegetation damage: Air pollution can damage plants and trees in the local area, reducing their
ability to absorb carbon dioxide and produce oxygen.
6. Water pollution: Industrial pollution can also contaminate local water sources, making them unsafe
for human and animal consumption and harming aquatic life.
Overall, pollution can have a range of negative effects on the microclimate in an industrialized urban
area, impacting both the environment and the health of people and animals living in the area.
Q.13 Explain and illustrate with sketches the principal differences between Greek Agora and Roman
Forum in terms of planning and architecture features.
Hemant Vilas Parulekar 755

Q.14 The sense of urban spatial scale is based on enclosure.’ What are the different degrees of enclosure
and how do they help in perception of urban spaces and forms?
Ans: - The sense of urban spatial scale refers to the human perception of space in the built environment.
The concept of enclosure is an important factor that contributes to the sense of scale in urban design.
Enclosure refers to the degree to which the edges of a space are defined and closed off. The greater
the enclosure, the more intimate and human-scale a space feels. Here are some different degrees of
enclosure and their effects on the perception of urban spaces:
1. Open Space: Open spaces lack any defined edges or boundaries, so the space feels expansive and
undefined. This type of space is often associated with parks, fields, and other large public spaces.
2. Spatially Continuous Space: This space has clear boundaries, but the edges are not necessarily
closed. Spatially continuous spaces include streets and plazas, and often create a sense of connection
between different parts of a city.
3. Enclosed Space: This space has clearly defined edges that are closed off, such as courtyards or urban
squares. These spaces tend to feel more intimate and are often associated with social gatherings or
other events.
4. Tunnel Space: This is a very enclosed space that has a narrow width relative to its height. Tunnels,
arcades, and covered walkways are examples of tunnel space. These spaces often create a sense of
compression and transition.
The degree of enclosure in a space affects the perception of its scale and the way people use it. By
considering the degree of enclosure in urban design, architects and urban planners can create a range
of different spaces that meet the needs of the people who will use them.
Q.15 In five square frame of equal size make compositions using lines and square shapes depicting.
(i) Rhythm (ii) pattern (iii) bi - axial symmetry (iv) spatial depth and distance and (v) rotation.

Q.16 Compute the thermal transmittance (U) value for a 230 mm brick wall with 12.5 mm thick cement
plaster on both sides (values of thermal conductivity for brick wall and cement plaster are 69 and
80 respectively in K cal cm Thermal conductance of outside and inside wall is 0.05 m2hde C and
0.16 respectively.

Q.17 Draw two consecutive layers of (i) octagonal brick pier and (ii) rat - trap bond at right angled
corner brick wall.

Q.18 Draw and label an isometric single line diagram of plumbing system for a first-floor toilet (shown
Below)
pic

Q.19 A newly built property fetches an annual rent of Rs 18000. As per agreement tenant is liable to
pay outgoing equivalent to 18% of the annual rent. Calculate the present value of the property.
(Assume rate of interest as 8.5%)
756 GATE Architecture and Planning: Comprehensive Question Bank

Ans: - To calculate the present value of the property, we can use the formula:
PV = R / (1 + i)n
where PV is the present value, R is the annual rent, i is the rate of interest, and n is the number of years.
In this case, R = Rs 18,000, i = 0.085 (8.5%), and n is the number of years for which we want to
calculate the present value.
To calculate n, we need to know the expected life of the property. Let’s assume it is 50 years.
Now, we need to calculate the annual outgoings, which are 18% of the annual rent:
Annual outgoings = 0.18 * Rs 18,000 = Rs 3,240
So, the net annual income from the property is:
Net annual income = Rs 18,000 - Rs 3,240 = Rs 14,760
Using the formula above, the present value of the property is:
PV = Rs 14,760 / (1 + 0.085)50 PV = Rs 217,487.77
Therefore, the present value of the property is approximately Rs 217,487.77.
Q.20 Briefly highlight the features of a typical Japanese garden.
Ans: - Japanese gardens are traditional gardens that originated in Japan and are designed to be places
of peace, tranquility, and reflection. They often incorporate elements of the natural landscape to create
a sense of harmony and balance. Here are some features commonly found in a typical Japanese garden:
1. Simplicity: Japanese gardens are designed to be simple and understated, with an emphasis on
natural beauty rather than elaborate designs or decorations.
2. Natural elements: A Japanese garden often includes natural elements such as rocks, water, and
plants. The rocks are often carefully placed to create a natural-looking landscape, and water is used to
create a sense of calm and serenity.
3. Symbolism: Many elements in a Japanese garden are symbolic, such as the use of lanterns, bridges,
and steppingstones. These elements are used to represent the passage of time or to create a sense of
journey or transition.
4. Moss and gravel: Moss and gravel are commonly used in Japanese gardens to create texture and
contrast. Moss is used to create a soft, velvety appearance, while gravel is used to create a sense of
movement and flow.
5. Trees and plants: Japanese gardens often include carefully pruned trees, such as bonsai, as well as a
variety of flowering plants and shrubs. The plants are chosen for their natural beauty and their ability
to create a sense of balance and harmony within the garden.
6. Tea houses: Many Japanese gardens include a tea house or other small structure, where visitors can
stop and rest, contemplate the garden, and enjoy a cup of tea.
Overall, Japanese gardens are designed to be places of contemplation and peace, where visitors can
connect with the natural world and find a sense of balance and harmony.
Q.21 a) What are the controls essential for comfort air conditioning in India? Mention the range of each
control element.
b) Why is dehumidification essential in summer air conditioning?
Hemant Vilas Parulekar 757

Ans: - a) Controls essential for comfort air conditioning in India include the following:
1. Temperature control: The desired temperature range for comfort air conditioning in India is
typically between 23°C and 27°C.
2. Humidity control: The humidity level for comfort air conditioning in India should be maintained
at 50% or lower.
3. Air velocity control: The air velocity in a room should be maintained between 0.15 and 0.25 m/s.
4. Fresh air control: Fresh air intake should be provided to maintain an adequate level of oxygen and
remove indoor air pollutants.
5. Filtration: Air filtration is essential for removing dust, pollen, and other particulate matter from
the air.
6. Noise control: Noise levels should be kept at a minimum to ensure comfort.
b) Dehumidification is essential in summer air conditioning because high humidity levels can reduce
the comfort level in a room. High humidity levels make it difficult for sweat to evaporate, which
in turn reduces the body’s ability to cool down. Dehumidification helps to remove excess moisture
from the air, making it easier for sweat to evaporate and for people to feel comfortable. Additionally,
high humidity levels can lead to the growth of mold and mildew, which can be harmful to health.
Dehumidification can help to prevent the growth of mold and mildew and maintain good indoor air
quality.
Q.22 Why is artificial lighting preferred to natural lighting in a museum or art gallery?
Ans: - Artificial lighting is often preferred to natural lighting in a museum or art gallery because it
allows for more precise control over the lighting conditions. Natural lighting can change throughout
the day and can be affected by weather, which can result in inconsistent lighting conditions that are
not optimal for viewing and preserving artwork. Artificial lighting, on the other hand, can be carefully
designed and controlled to provide consistent lighting levels and color temperature. It can also be
adjusted to highlight specific areas or pieces of artwork and can be used to create a certain mood or
atmosphere in the space. Additionally, artificial lighting can be used to protect artwork from harmful
UV radiation by incorporating filters or other protective measures.
Q.23 Mention the common acoustical defects encountered in the design of a big auditorium. Mention
Their causes.
Ans: - Some common acoustical defects encountered in the design of a big auditorium include:
1. Echoes: Echoes are caused by sound waves reflecting off hard surfaces and returning to the listener’s
ear with a slight delay, which can cause an unpleasant and distracting effect. Echoes can be minimized
by using sound-absorbing materials on walls, ceilings, and floors.
2. Reverberation: Reverberation is the persistence of sound in an enclosed space after the original
sound source has stopped. Too much reverberation can create a muddled, unclear sound that is
difficult to understand. Reverberation time can be controlled by using sound-absorbing materials and
diffusers.
3. Standing waves: Standing waves are caused by sound waves bouncing back and forth between
parallel surfaces, creating nodes and antinodes, which can cause certain frequencies to be amplified
or attenuated. Standing waves can be minimized by using non-parallel surfaces or sound-absorbing
materials.
758 GATE Architecture and Planning: Comprehensive Question Bank

4. Insufficient sound diffusion: Sound diffusion is important for creating a sense of spaciousness
and envelopment in an auditorium. Insufficient sound diffusion can result in a dull and lifeless sound.
Proper sound diffusion can be achieved by using reflectors or diffusers.
5. Noise and vibration from HVAC systems: HVAC systems can create noise and vibration that
can be heard in the auditorium, which can be distracting and interfere with the listening experience.
Proper isolation and soundproofing of HVAC systems can help minimize this issue.
6. Inadequate speech intelligibility: Speech intelligibility is important for ensuring that the audience
can understand spoken words. Factors that can affect speech intelligibility include excessive background
noise, echoes, and reverberation.
By carefully designing the auditorium’s acoustics and considering the factors that can lead to these
acoustical defects, it is possible to create a space that offers excellent sound quality and a comfortable
listening experience.
Q.24 Determine which of the following (Fig. B -I -24) are determinate or indeterminate structures.
Pic

Q.25 A reinforced concrete slab (having balanced section) has as overall depth of 100 mm. The effective
cover is 20 mm. If the stress in concrete and steel are not to exceed 5N/mm2 and 140N/mm2, find
the safe uniformly distributed load which can be placed on the slab. The slab is supported on
beam at 3.0 m c/c on both sides. The maximum bending moment for a meter strip of slab may be
taken as Wl2/l2. Take the moment of resistance of the balanced section as Equal to 0.85 bd2Nmm.

Q.26 The time estimates of various activities of a project are indicted in the following Table.
Determine the standard deviation of the critical path.
Activity Optimistic time Most likely time Pessimistic time
(Weeks) (Weeks) (Weeks)
1 -2 8 12 22
1 -3 6 12 18
2 -4 1 4 7
3 -4 5 9.5 11
2 -5 9 15 21
4 -5 3 4 5

Q.27 What is hue, value, and intensity in a colour scale? Indicate their position in a three-dimensional
diagram.
Ans: - In color theory, hue, value, and intensity are the three characteristics that define any given color.
These terms are often used in the design and selection of colors for various applications.
1. Hue: Hue refers to the pure spectrum color of a particular shade or color. It is often described by the
color name (such as red, blue, green, yellow, etc.) and can be thought of as the basic color of an object.
Hemant Vilas Parulekar 759

2. Value: Value refers to the lightness or darkness of a color, or the amount of white or black that has
been added to a hue. A color with high value contains whiter, making it a lighter color, while a color
with low value contains blacker, making it a darker color.
3. Intensity: Intensity, also called chroma or saturation, refers to the brightness or dullness of a color.
A color with high intensity is bright and vibrant, while a color with low intensity is dull or muted.
In a three-dimensional color model, hue is typically depicted on the horizontal axis, value on the
vertical axis, and intensity on the depth axis. This is known as the HSV (hue, saturation, value) color
model or HSL (hue, saturation, lightness) color model. A particular color can be located within this
model by specifying its hue, value, and intensity coordinates.
Q.28 Mention the factors that made the architecture and planning of Fatehpur - Sikri unique.
Ans: - Fatehpur Sikri is a well-known historical site located near Agra, India. It was built by the Mughal
emperor Akbar in the late 16th century and served as the capital of the Mughal Empire from 1571 to
1585. The architecture and planning of Fatehpur Sikri are considered unique and represent a fusion of
Hindu and Islamic styles. Some of the factors that contributed to this uniqueness include:
1. Syncretism: The architecture of Fatehpur Sikri represents the fusion of Hindu and Islamic architectural
styles. The buildings incorporate a mix of Persian, Indian, and Central Asian architectural elements.
2. Strategic location: Fatehpur Sikri is located on a rocky ridge and was surrounded by a lake, making
it easy to defend against enemies. The city was also located at the intersection of important trade routes.
3. Urban planning: The city was designed using the principles of Vastu Shastra, an ancient Indian
system of architecture and town planning. The city is laid out in a grid pattern, with various buildings
and courtyards arranged in a specific order.
4. Symbolism: Many of the buildings in Fatehpur Sikri are rich in symbolic meaning. For example, the
Diwan-i-Khas, or Hall of Private Audience, was designed to represent the fusion of Hindu and Islamic
cultures.
5. Water conservation: The city was designed with a sophisticated water conservation system, which
included a series of reservoirs, wells, and channels. This allowed the city to be self-sufficient in terms of
water supply.
6. Building materials: The buildings in Fatehpur Sikri were constructed using local materials, such as
red sandstone, which helped to blend the buildings with the natural surroundings.
Overall, the architecture and planning of Fatehpur Sikri represent a unique blend of cultures, styles, and
technologies, which make it a significant example of Mughal architecture and urban planning.
Q.29 Mention the means of controlling external noise which should be considered while designing a
building.
Ans: - Controlling external noise is an important consideration in building design, especially in urban
areas where noise pollution can be a significant problem. Some means of controlling external noise
that should be considered in building design include:
1. Site selection: Choosing a site that is located away from sources of noise pollution such as highways,
airports, and train tracks.
2. Building orientation: Orienting the building to reduce the impact of external noise. For example,
placing the building perpendicular to the source of noise can help to reduce noise levels.
760 GATE Architecture and Planning: Comprehensive Question Bank

3. Building design: Designing the building to reduce noise transmission through the building envelope.
This can be achieved using noise-reducing glazing, insulating materials, and sound-absorbing finishes.
4. Ventilation system design: Designing the building’s ventilation system to minimize the impact of
external noise. This can be achieved using silencers or ducts that terminate in acoustically isolated
rooms.
5. Landscaping: Using landscaping features such as trees, hedges, and earth berms to create a barrier
between the building and the source of noise.
6. Noise barriers: Constructing noise barriers such as walls or fences to reduce the impact of external
noise on the building.
7. Regulations: Ensuring that the building meets local noise control regulations and standards.
By considering these means of controlling external noise, designers can help to create a more
comfortable and healthier indoor environment for building occupants.
Q.30 State the function of DXFOUT, DXBIN and XREF commands in Auto CAD.
Ans: - In AutoCAD, the DXFOUT command is used to export drawing files in a format called Drawing
Exchange Format (DXF), which is a file format used for CAD data interoperability between different
software programs. This command can be used to export both 2D and 3D drawings and allows users
to select which parts of the drawing they want to export.
The DXBIN command is used to export binary DXF files, which are smaller in size than ASCII DXF
files and can be loaded faster. Binary DXF files are not human-readable like ASCII DXF files but can
be read and processed faster by software programs.
The XREF command is used to attach external reference files to a drawing. These external files can
be other AutoCAD drawings, images, or other types of files, and are inserted into the drawing as
a reference. This means that any changes made to the external reference file will be automatically
updated in the drawing that references it. The XREF command allows users to manage and organize
these external reference files and provides options for controlling the visibility and layer properties of
the external reference in the drawing.
SECTION B (PART -I)
Q. 31 A main sewer is to be designed to receive a flow from 2sq km area of community, where the
population density is 250 persons/hectare. The average sewage flow is 120 litres per capita per
day. What will be the design flow of the main sewer? Assume peak factor as 3.
Ans: - First, we need to calculate the total population in the 2 sq km area:
Population density = 250 persons/hectare
So, the population per sq km = 250 persons/hectare x 100 hectares/sq km = 25,000 persons/sq km
Total population in 2 sq km area = Population density x Area = 25,000 persons/sq km x 2 sq km =
50,000 persons
Next, we need to calculate the sewage flow rate:
Sewage flow rate = Population x Water usage per capita x Peak factor
Water usage per capita = 120 liters/capita/day
Peak factor = 3
So, Sewage flow rate = 50,000 persons x 120 liters/capita/day x 3 = 18,000,000 liters/day
Therefore, the design flow of the main sewer will be 18,000,000 liters/day.
Hemant Vilas Parulekar 761

Q. 32 Sketch a label a common Rotary Intersection. State the advantages and disadvantages of Rotary
Intersections

Q. 33 Explain the concept of zero - base budgeting in urban development.


Ans: - Zero-base budgeting (ZBB) is a budgeting method that starts from scratch or zero-base. In other
words, instead of taking the previous year’s budget and simply adding or subtracting a percentage for
the current year, zero-base budgeting requires that each expenditure be justified from the ground up.
In urban development, zero-base budgeting is used to allocate resources for various projects based
on their priority and importance. The process involves identifying and evaluating each project based
on its expected benefits, costs, and risks. Projects are then ranked according to their importance, and
resources are allocated accordingly. This ensures that the most important and impactful projects are
given priority over less important ones.
The benefits of zero-base budgeting in urban development include:
1. Improved resource allocation: By prioritizing projects based on their importance and expected
benefits, resources can be allocated more efficiently and effectively.
2. Increased accountability: Since each expenditure must be justified from scratch, zero-base
budgeting increases accountability and transparency in the budgeting process.
3. Better decision-making: By requiring a thorough evaluation of each project, zero-base budgeting
helps decision-makers to make better, more informed decisions.
4. Increased focus on results: Zero-base budgeting focuses on achieving results rather than just
spending money, which helps to ensure that resources are used in the most effective way possible.
In summary, zero-base budgeting is an important tool for urban development that helps to ensure that
resources are allocated in the most effective way possible.
Q.34 Outline the basic elements which constitute the mental image of a city.
Ans: - The mental image of a city refers to the subjective perception that individuals have of a city
based on their experiences, knowledge, and cultural background. It can be influenced by various
elements, including:
1. Landmarks: These are prominent features of a city that are recognized by many people, such as the
Eiffel Tower in Paris or the Empire State Building in New York City.
2. Neighborhoods: Different areas of a city can have distinct characteristics that influence how
people perceive them. For example, some neighborhoods may be associated with culture, history, or
entertainment.
3. Transportation: The type and quality of transportation available in a city can influence how people
view it. For example, a city with efficient public transportation may be viewed as more convenient or
accessible.
4. Architecture: The style and design of buildings in a city can impact the mental image of the city. For
example, a city with modern skyscrapers may be viewed as more dynamic or innovative.
5. Natural features: The presence of parks, rivers, or other natural features can influence how people
view a city. For example, a city with many parks and green spaces may be viewed as more attractive or
healthy.
762 GATE Architecture and Planning: Comprehensive Question Bank

6. People: The people who live and work in a city can also contribute to its mental image. For example,
a city with friendly and welcoming people may be viewed as more hospitable or inviting.
These elements can combine to create a unique mental image of a city that is influenced by individual
experiences and perceptions.
Q.35 What is meant by Land Pooling Technique in urban Land Management. Discuss the operational
steps for preparing a Land Pooling scheme.
Ans: - Land pooling is a technique used in urban land management for the development of new areas,
which involves the consolidation of land parcels of individual owners, followed by the realignment
of boundaries to create large land parcels. These large land parcels are then used for public purposes
such as roads, schools, parks, and other amenities, as well as for the development of private land for
residential, commercial, and industrial use.
The operational steps for preparing a Land Pooling scheme are as follows:
1. Identification of land: Identify the land available for development and its current land use.
2. Formation of a consortium: Form a consortium of landowners to pool their land for development.
The consortium should have a clear agreement on the distribution of profits.
3. Preparation of a layout plan: Prepare a layout plan indicating the proposed use of land, the road
network, and the location of public facilities such as schools, parks, and hospitals.
4. Preparation of a zoning plan: Prepare a zoning plan indicating the permissible uses of land.
5. Development of infrastructure: Develop the required infrastructure such as roads, water supply,
drainage, and electricity.
6. Allocation of land: Allocate the land to the individual owners based on the size of their original
land parcels.
7. Transfer of land: Transfer the ownership of the land to the individual owners and other stakeholders
such as the local government.
The merit of the Land Pooling Technique in urban land management includes:
1. Encouraging community participation: Land pooling encourages community participation in the
development process, which leads to better planning outcomes and community satisfaction.
2. Reduced land acquisition costs: Land pooling reduces land acquisition costs for the government
as it can acquire land at a lower cost compared to direct acquisition.
3. Efficient land use: Land pooling leads to efficient land use as it encourages the consolidation of
land parcels, which leads to better planning outcomes and higher land-use densities.
4. Better infrastructure: Land pooling encourages the development of better infrastructure as the
cost is shared among landowners.
5. Public amenities: Land pooling allows for the creation of public amenities such as parks, schools,
hospitals, and other community facilities, which leads to improved quality of life for residents.
Q.36 Discuss the role of co - operative housing in mitigating the housing shortage in India. Mention the
merit of co - operative housing.
Hemant Vilas Parulekar 763

Ans: - Cooperative housing can play an important role in addressing the housing shortage in India
by providing affordable housing to low- and middle-income families. In cooperative housing, a group
of individuals or families come together to form a cooperative society, which then acquires land and
builds housing for its members. Members of the cooperative society are also co-owners of the land
and buildings, and they share the costs and responsibilities of maintaining and managing the housing.
The main merit of cooperative housing is that it allows people to have a greater degree of control over
their housing and living conditions. By pooling their resources and working together, members of a
cooperative society can build better-quality housing than they could afford individually. Cooperative
housing also promotes community involvement and social cohesion, as members of the society work
together to manage and maintain the housing.
However, cooperative housing also has some limitations. One of the biggest challenges is obtaining
financing to acquire land and build housing. Cooperative societies often struggle to obtain loans from
banks and other financial institutions, as they may not have collateral to offer. In addition, cooperative
housing can be difficult to manage and maintain, particularly if members have differing opinions on
how to manage the society and its resources.
To address these challenges, the government can play a key role in supporting cooperative housing
through policies and programs that provide financial and technical assistance to cooperative societies.
In addition, cooperative societies can benefit from training and capacity building programs that help
members develop the skills and knowledge needed to manage and maintain the housing.
Q.37 Mention the factors that govern the determination of most appropriate housing density for a
particular area. Enumerate each of them briefly.
Ans: - The determination of the most appropriate housing density for a particular area is dependent
on several factors, including:
1. Land availability: The amount of land available for housing in each area is a critical factor in
determining the appropriate housing density. Higher density housing is often preferred in areas where
land is scarce.
2. Zoning regulations: Local zoning regulations may specify minimum or maximum density
requirements, which can affect the type of housing that can be built in a particular area.
3. Infrastructure: The availability and capacity of infrastructure such as roads, water supply, sewage
treatment, and other utilities are important considerations in determining housing density. Higher
density housing requires more infrastructures to support it.
4. Transportation: Access to transportation options, including public transit, can make higher-density
housing more attractive in certain areas.
5. Environmental considerations: Environmental factors, such as air quality, access to green spaces,
and proximity to natural areas, can affect the appropriate housing density for a particular area.
6. Community preferences: Community preferences and expectations can also play a role in
determining the appropriate housing density. Some communities may prefer lower-density housing,
while others may be more accepting of higher-density development.
7. Economic factors: The local economic climate and market demand for housing can also affect the
appropriate housing density. In areas with high demand and limited supply, higher-density housing
may be more feasible and desirable.
Each of these factors should be carefully considered in determining the most appropriate housing
density for a particular area.
764 GATE Architecture and Planning: Comprehensive Question Bank

Q.38 Discuss the applicability of Scalogram Analysis in spatial development planning.


Ans: - Scalogram analysis is a technique used in spatial development planning to assess the suitability
of an area for development. It involves evaluating the physical characteristics of an area, such as
topography, soil, climate, and vegetation cover, to determine the best land use practices. The technique
has several advantages, including its ability to identify areas that are most suitable for specific uses, and
its ability to incorporate both qualitative and quantitative data in the analysis.
Scalogram analysis can be applied to several spatial planning issues, such as land use allocation,
conservation planning, and urban expansion. In land use allocation, the technique can be used to
identify areas that are most suitable for agricultural, forestry, or urban development. In conservation
planning, scalogram analysis can help identify areas that are most suitable for conservation and
identify areas that are under threat from human activities. In urban expansion, the technique can help
identify areas that are most suitable for new urban development, while also considering the impact on
existing infrastructure and services.
To implement scalogram analysis, a set of criteria must be developed based on the objectives of the
project. These criteria should be specific and measurable, and they should be ranked based on their
importance. For example, in an agricultural land allocation project, the criteria might include soil
fertility, water availability, slope, and climate. The criteria would then be weighted based on their
importance in achieving the project objectives. The analysis would then be conducted, and the results
would be used to inform land use decisions.
There are several techniques that can be used for multi-objective and multi-criteria analysis. Three
techniques for multi-objective analysis include decision trees, weighted scoring models, and Pareto
optimization. Decision trees involve developing a decision tree to identify the optimal decision path.
Weighted scoring models involve assigning weights to the criteria and ranking the options based on
their scores. Pareto optimization involves identifying the set of options that represent the best trade-
off between the criteria.
Three techniques for multi-criteria analysis include cost-benefit analysis, cost-effectiveness analysis,
and multi-attribute utility analysis. Cost-benefit analysis involves comparing the costs and benefits
of different options. Cost-effectiveness analysis involves comparing the costs of achieving different
objectives. Multi-attribute utility analysis involves assessing the preferences of stakeholders and
ranking the options based on their utility scores.
Q.39 Distinguish between ‘Multi-objective’ and ‘Multi-criteria’ model for decision and evaluation of
planning projects.Mention at least three techniques for each category
Ans: - Multi-objective and multi-criteria models are used in decision-making and evaluation of
planning projects, but they differ in several ways.
Multi-objective models consider multiple objectives or goals that are to be achieved in a project. The
aim is to identify a set of options that satisfy these objectives simultaneously, without prioritizing any
single objective over the others. The different objectives can be conflicting or complementary, but they
are all considered in the decision-making process.
On the other hand, multi-criteria models consider multiple criteria or factors that are relevant to
a project, but they prioritize or weight these criteria differently. The aim is to evaluate different
alternatives or options based on their performance against the criteria, with the most important
criteria receiving higher weights.
Hemant Vilas Parulekar 765

Some techniques for multi-objective models include:


1. Pareto analysis: This involves identifying and comparing the trade-offs between multiple objectives
to find the best possible solutions.
2. Goal programming: This is a mathematical approach that involves setting multiple objectives and
minimizing the deviations from each objective.
3. Decision tree analysis: This involves analyzing the possible outcomes of different alternatives and
selecting the one that maximizes the objectives.
Some techniques for multi-criteria models include:
1. Analytic Hierarchy Process (AHP): This is a mathematical approach that involves structuring the
decision problem as a hierarchy of criteria and alternatives and assigning weights to each criterion.
2. Technique for Order of Preference by Similarity to Ideal Solution (TOPSIS): This involves
comparing each alternative to an ideal solution and a worst-case scenario and selecting the one that is
closest to the ideal.
3. Simple Additive Weighting (SAW): This involves calculating a weighted sum of each alternative’s
performance against the criteria and selecting the one with the highest score.
In summary, multi-objective models aim to satisfy multiple objectives simultaneously, while multi-
criteria models prioritize and weight multiple criteria in the decision-making process.
Q.40 (a) Explain the concept of relief displacement on aerial photography.
(b) A flat area is photographed in a scale of 1:10,000 with a camera of 15 cm focal length The
bottom of a chimney stack is found to lie at a distance of 12.01 cm from the principal point of the
photograph and the top at distance of 12.22 cm. Find the height of the chimney stack.
Ans: -
Q.41 Explain the significance of ‘Z’ score in the stastitical analysis and mention its properties.
Ans: - In statistics, the Z-score, also known as the standard score, is a measure of how many standard
deviations a data point is from the mean. It is often used to standardize data and make comparisons
between different data sets.
The formula for calculating the Z-score of a data point is:
Z = (x - μ) / σ
where:
• x is the data point
• μ is the mean of the population
• σ is the standard deviation of the population
The Z-score has several important properties:
• A Z-score of 0 indicates that the data point is equal to the mean.
• A positive Z-score indicates that the data point is above the mean, while a negative Z-score
indicates that it is below the mean.
• The further a data point is from the mean in terms of standard deviations, the higher its
Z-score.
766 GATE Architecture and Planning: Comprehensive Question Bank

• Approximately 68% of the data falls within one standard deviation of the mean, 95% falls
within two standard deviations, and 99.7% falls within three standard deviations.
• Z-scores can be used to compare data from different populations that have different means
and standard deviations, by standardizing the data to a common scale.
The Z-score is commonly used in statistical analysis for a variety of purposes, including hypothesis
testing, quality control, and outlier detection. It is a useful tool for identifying data points that are
significantly different from the mean and for making comparisons between different data sets.
Q.42 State briefly the various Site Analysis Techniques for appropriate decision making in urban
landscape projects.
Ans: - Site analysis is an essential step in urban landscape projects as it helps in understanding the site’s
potential, limitations, and challenges. The various site analysis techniques for appropriate decision
making in urban landscape projects include:
1. Site Inventory: It involves taking an inventory of all the site’s physical attributes, such as topography,
vegetation, soils, water features, and existing structures.
2. Site Analysis: It involves analyzing the site’s environmental factors, such as climate, wind direction,
sun path, and noise levels.
3. Historical Analysis: It involves studying the site’s history, cultural heritage, and significance.
4. Visual Analysis: It involves assessing the site’s visual qualities, such as the view, framing, and
perspective.
5. Activity Analysis: It involves analyzing the site’s use patterns, such as pedestrian and vehicular
movement, social activities, and urban design patterns.
6. User Analysis: It involves understanding the users’ needs, preferences, and behavior patterns.
7. SWOT Analysis: It involves evaluating the site’s strengths, weaknesses, opportunities, and threats.
These site analysis techniques help in making informed decisions about the site design and planning,
considering the site’s unique features and characteristics. They aid in creating functional, sustainable,
and aesthetically pleasing urban landscapes that meet the users’ needs and expectations.
Q.43 Highlight the principles of ‘Ekistics’ and illustrate graphically.
Ans: - Ekistics is the study of human settlements, with a focus on the principles and methods of urban
planning and design. The principles of Ekistics were first proposed by the Greek architect and town
planner, Constantinos Doxiadis, in the mid-twentieth century. The principles of Ekistics are:
1. Human Needs: The primary focus of Ekistics is on meeting the basic needs of human beings,
including the need for shelter, food, water, and social interaction.
2. Hierarchy of Settlements: The human settlement system should be organized in a hierarchical
manner, with small villages and towns at the base, and larger cities at the top.
3. Centrality: The settlement system should have a clearly defined central point or nucleus, which
serves as a focus for social and economic activity.
4. Accessibility: The settlement system should be designed to ensure easy access to all parts of the
settlement, both within and between settlements.
Hemant Vilas Parulekar 767

5. Diversity: The settlement system should provide a range of different types of housing, employment
opportunities, and social amenities to cater to the needs of different people.
6. Flexibility: The settlement system should be designed to be flexible and adaptable to changing
circumstances and needs.
7. Sustainability: The settlement system should be designed to be environmentally sustainable,
considering issues such as resource depletion, pollution, and climate change.
Q.44 Mention the importance of Environmental Impact Assessment in handling large projects. State
various method for identifying environmental effects and impacts.
Ans: - Environmental Impact Assessment (EIA) is a critical process for evaluating the potential
environmental effects of large projects. The objectives of EIA are to identify, predict, and evaluate the
potential environmental, social, and economic impacts of proposed development projects, and to offer
mitigation measures to avoid, reduce, or offset these impacts. Some of the key importance of EIA are:
1. EIA helps in identifying and assessing the potential environmental impacts of proposed projects
before they are implemented, thereby enabling decision-makers to make informed decisions on
whether to approve or reject such projects.
2. EIA also provides an opportunity for stakeholders to participate in the decision-making process,
thereby ensuring that the concerns and interests of local communities, interest groups, and other
stakeholders are adequately addressed.
3. EIA helps in integrating environmental considerations into the planning and decision-making
process, thereby promoting sustainable development.
4. EIA also enables project proponents to identify and address potential environmental risks and liabilities
associated with their projects, which can help to avoid costly legal battles and reputational damage.
Various methods for identifying environmental effects and impacts during the EIA process include:
1. Scoping: This involves identifying the key issues and concerns that need to be addressed in the
EIA process. This is typically done through stakeholder consultation and review of available data and
information.
2. Baseline studies: This involves gathering data on the existing environmental, social, and economic
conditions in the project area.
3. Predictive modeling: This involves using computer models and other tools to predict the potential
environmental impacts of the proposed project.
4. Impact assessment: This involves evaluating the potential environmental impacts of the proposed
project, including both direct and indirect impacts, as well as short-term and long-term impacts.
5. Mitigation: This involves identifying measures to avoid, reduce, or offset the potential environmental
impacts of the proposed project.
6. Monitoring: This involves establishing a program for monitoring and evaluating the actual
environmental impacts of the project over time, and taking corrective action as needed.
In the Indian context, the sites and services scheme was introduced in the 1970s as a means of providing
affordable housing to low-income families. The scheme aimed to provide serviced land and basic
infrastructure to households, enabling them to build their own houses. The advantages of the scheme
include providing affordable housing to low-income families, promoting community participation in
housing provision, and promoting self-help and self-reliance. The limitations of the scheme include
the difficulty of finding suitable land for the scheme, the lack of finance for construction, and the
challenges of providing basic infrastructure in peri-urban and rural areas.
768 GATE Architecture and Planning: Comprehensive Question Bank

Q.45 State the objectives of subdividing a planning area into traffic zones.
Ans.: - Subdividing a planning area into traffic zones serves the following objectives:
1. To allocate land use and building intensities: Traffic zones help in determining the density and
intensity of development in different parts of the planning area based on the nature and volume of
traffic expected to be generated.
2. To provide for different types of transport: Traffic zones allow for the planning and provision of
different modes of transport, such as pedestrian, bicycle, public transport, and private vehicles. This
helps to ensure that different transport modes can coexist and function efficiently.
3. To manage traffic flow: Traffic zones allow for the efficient management of traffic flow by separating
conflicting traffic movements and providing for appropriate road widths, turning radii, and other
design features that promote safe and efficient movement.
4. To promote safety: Traffic zones can help to improve road safety by providing for appropriate road
designs, traffic controls, and other measures that reduce the risk of accidents.
5. To minimize environmental impacts: Traffic zones can help to minimize the environmental impacts
of transportation by ensuring that different transport modes are integrated and that appropriate
measures are taken to manage noise, air pollution, and other negative impacts.
Overall, the objective of subdividing a planning area into traffic zones is to create a safe, efficient, and
sustainable transportation system that meets the needs of all users.
Q.46 Mention the different shapes suggested by Mansara for town and villages in ancient India. Sketch
atleast four such shapes.

Q.47 State the objectives of sites and Services. Scheme. Mention its advantages and limitations in Indian
Context.
Ans: - The Sites and Services Scheme is a housing scheme developed to provide basic services and
infrastructure to low-income families in urban areas. The scheme has several objectives, including:
1. To provide affordable housing to low-income families in urban areas.
2. To provide basic services such as water supply, sanitation, and electricity to the households.
3. To improve the living conditions of low-income families by providing access to basic amenities.
4. To promote community development by providing basic social infrastructure such as schools,
health centers, and recreational facilities.
Advantages of the Sites and Services Scheme:
1. The scheme provides basic services and infrastructure to low-income families, which helps to
improve their living conditions.
2. The scheme is cost-effective as the families are responsible for constructing their own homes, which
reduces the cost of housing construction.
3. The scheme promotes community development and social cohesion by providing basic social
infrastructure such as schools, health centers, and recreational facilities.
4. The scheme provides an opportunity for low-income families to become homeowners, which can
help to improve their social status.
Hemant Vilas Parulekar 769

Limitations of the Sites and Services Scheme in Indian Context:


1. The lack of affordable financing options for low-income families can hinder their ability to participate
in the scheme.
2. Inadequate planning and design can lead to the development of low-quality housing and inadequate
infrastructure.
3. Land availability can be a major challenge, especially in urban areas, which can lead to the
development of sites in remote or poorly connected locations.
4. The scheme is often criticized for its narrow focus on providing basic services and infrastructure,
and not addressing the broader issues of poverty and inequality in urban areas.
Decibel zoning is a planning measure used to regulate noise levels in urban areas. The principal
types of urban noise include transportation noise (from road traffic, trains, and aircraft), industrial
noise (from factories and heavy machinery), and neighborhood noise (from commercial activities,
construction, and social activities). The planning measures to control urban noise include:
1. Setting noise standards and limits for different land uses and activities.
2. Designing buildings and infrastructure to reduce noise levels, such as sound barriers and double
glazing.
3. Limiting the hours of operation for noisy activities and setting noise emission standards for
machinery and equipment.
4. Restricting noisy activities in residential areas, such as commercial and industrial activities.
In conclusion, the Sites and Services Scheme is an important housing scheme that has been
implemented in several countries, including India. While the scheme has several advantages, it also
has some limitations, and there is a need to address these limitations to ensure that the scheme can
effectively meet the housing needs of low-income families. Decibel zoning is a planning measure that
can be used to control urban noise levels and promote healthy and livable urban environments.
Q.48 What is meant by ‘Decibel Zoning?’ Mention principal types of urban noises and the planning
measures to control them.
Ans: - Decibel zoning is a planning technique that involves identifying areas of a city based on their
acceptable noise levels. It helps to establish zoning regulations to control noise pollution from various
sources, such as transportation, industry, and construction.
The principal types of urban noises are:
1. Transportation noise: This includes noise from road traffic, railways, and aircraft.
2. Industrial noise: This includes noise from factories, power plants, and other industrial activities.
3. Construction noise: This includes noise from building and construction activities.
4. Recreational noise: This includes noise from sporting events, concerts, and other outdoor activities.
The following are some planning measures that can be used to control urban noises:
1. Land use planning: Proper zoning of land can help to separate noise-sensitive areas from noise-
generating activities. For example, residential areas can be separated from industrial areas.
2. Building design: Buildings can be designed to reduce the transmission of noise by using sound-
absorbing materials, double-glazing windows, and other acoustic treatments.
770 GATE Architecture and Planning: Comprehensive Question Bank

3. Transportation planning: Traffic can be managed to reduce noise pollution by using quieter
vehicles and reducing speeds in noise-sensitive areas.
4. Noise barriers: These can be used to reduce noise transmission from highways, railways, and other
sources.
5. Noise ordinances: Local governments can establish noise ordinances that set acceptable noise levels
for different types of areas and impose penalties for violations.
Q.49 Differentiate between ‘Mohallas’ of traditional cities and ‘Neighbourhood units’ in modern cities.
Ans: - Mohallas are the traditional neighborhoods found in many South Asian cities such as Delhi,
Lahore, and Karachi. They are characterized by a mix of land uses, including residential, commercial,
and religious, with narrow, winding streets and courtyards. Mohallas are often organized around
family relationships, religion, or occupation, and residents tend to have strong social ties and a sense
of community. Mohallas are typically small and densely populated, with a high degree of pedestrian
activity.
In contrast, neighborhood units are a planning concept that emerged in modern cities during the early
20th century. They are typically designed to be self-contained areas with a mix of land uses, including
residential, commercial, and recreational. Neighborhood units are often organized around a central
park or other shared space, and they are designed to be easily walkable and to promote a sense of
community among residents. Unlike Mohallas, neighborhood units tend to be larger in size and less
densely populated, with a lower degree of pedestrian activity.
One of the key differences between Mohallas and neighborhood units is their history and evolution.
Mohallas have evolved over centuries, with their layout and character shaped by cultural, social, and
economic factors over time. Neighborhood units, on the other hand, are typically planned and designed
by urban planners and architects, with a focus on creating a specific type of urban environment that
meets certain social and functional goals.
Another key difference is the level of social interaction and community that they promote. Mohallas
tend to foster strong social bonds among residents, who often have long-standing relationships and
connections to one another. In contrast, neighborhood units are designed to promote social interaction
and community through shared public spaces and amenities, but they may not have the same level of
social cohesion as Mohallas.
In summary, while both Mohallas and neighborhood units are forms of urban neighborhoods, they
differ in their historical evolution, size, density, layout, and level of social interaction and community.
(b) Explain the differences between ‘housing need’ and ‘ housing demand’
Ans: - Housing need and housing demand are two related but distinct concepts that are often used in
discussions about housing policy and planning.
Housing need refers to the level of housing that is required to meet the basic needs of a population.
This can be determined by factors such as population growth, demographic changes, household
formation rates, and the condition of the existing housing stock. Housing need is often assessed by
looking at the number of households in a given area that are living in overcrowded, substandard, or
otherwise inadequate housing conditions. In many cases, housing need is measured in terms of a
quantitative shortfall, which is the difference between the number of households that require housing
and the number of units that are actually available.
Hemant Vilas Parulekar 771

Housing demand, on the other hand, refers to the desire or willingness of individuals or households to
acquire or rent housing. Housing demand is often determined by factors such as income, employment,
family size, lifestyle preferences, and proximity to amenities and services. Housing demand is typically
assessed through surveys, market analysis, or other forms of market research.
The main difference between housing need and housing demand is that housing need is primarily a
quantitative concept that focuses on the basic requirements of a population, while housing demand
is a more qualitative concept that reflects the preferences and choices of individual households. In
general, meeting housing need is seen as a social obligation, while meeting housing demand is seen
as a market opportunity. While the two concepts are related, they may not always align perfectly,
and policymakers and planners need to carefully consider both factors when making decisions about
housing policy and planning.
Q.50 Illustrate with sketches the important planning features for any ONE of the following cities:
1) Brasilia 2) Islamabad
GATE QUESTION PAPER 1995
General Aptitude (GA)

Q.1 (i) Rhythmic pattern is created by


(A) Repeating a set of elements in a space at regular intervals
(B) Repeating a single element in linear fashion only
(C) Symmetrically arranging element alone an axis
(D) None of the above
Ans: - (A)
Explanation: Rhythmic patterns are created by repeating a set of elements (such as shapes, lines, or
colors) in a space at regular intervals. This repetition creates a sense of movement, flow, and harmony
within the pattern. Rhythmic patterns can be found in various forms of art, such as music, dance, and
visual arts.
(ii) Of the following types of landscape, which one produces heat - island
(A) Island (B) Coastal areas (C) Urban area (D) Mountain ranges
Ans: - (C )
Explanation: Urban areas can produce a heat island effect, which is a phenomenon where urban en-
vironments experience higher temperatures than surrounding rural areas. This effect is caused by a
combination of factors, including the absorption of sunlight by dark surfaces, the production of heat by
human activities (such as transportation and buildings), and the reduction of vegetation and open space.
The heat island effect can have negative impacts on human health and the environment, and strategies
such as green roofs, cool pavements, and increased vegetation can be used to mitigate its effects.
(iii) Combination of Red and Green in a colour scheme is
(A) Simultaneous contrast (B) Complementary harmony
(C) Split complementary (D) Analogous
Ans: - (B)
(Red and green are complementary to each other in the colour wheel)
(iv) The maximum distance for recognizing faces is
(A) 15 meters (B) 40 meters (C) 25 meters (D) 50meters
Ans:- (C)
Explanation: - The maximum distance for recognizing faces varies depending on a variety of factors
such as lighting conditions, the size of the face, the angle of the face, and the observer’s visual acuity.
However, studies have suggested that under optimal conditions, the maximum distance for recognizing
faces is approximately 25 meters or 82 feet. Beyond this distance, it becomes increasingly difficult to dis-
cern facial features and recognize individual faces. It is worth noting that this is an approximate distance
and can vary based on the aforementioned factors.
(v) The optimum reverberation time for public lecture halls should not exceed
(A) 1.0 sec (B) 1.5 sec (C) 2.0 sec (D) 2.5 sec
Hemant Vilas Parulekar 773

Ans:- (A)
 Around 2 seconds is desirable for a medium -sized , general purpose auditorium that is to be used
for both speech and music . A classroom should be much shorter , less than a second. And a re-
cording studio should minimize reverberation time in most cases for clarity of recording.
 The maximum level of background noise allowed in the classroom is 35 decibels (dBA).
 Maximum acceptable noise level as per CPCB norms in India.
Day time Night time
Industrial area 75 70
Commercial area 65 55
Residential area 55 45
Silence Zone 50 40
Silence Zone is referred as areas upto 100 meters around such premises as hospitals, educational institu-
tions and courts.
(vi) Higher the elevation of a town above the sea level, larger the dose human population is exposed
to
(A) Nuclear radiation (B) Cosmic rays (C) Electromagnetic waves (D) All the above
Ans:- (B)
Explanation: As the elevation of a town increases, the atmospheric shielding decreases and the exposure
to cosmic rays increases. Cosmic rays are high-energy radiation that originate from outer space and can
penetrate the Earth’s atmosphere. The intensity of cosmic rays increases with altitude, so people living
at higher elevations receive a larger dose of cosmic radiation than people living at lower elevations. This
can have potential health effects, including an increased risk of cancer.
(vii) A rectangular area specified in world co - ordinates is called
(A) Window (B) Door (C) Ventilator (D) Mirror
Ans: - (A)
Explanation: The question is related with architectural graphics software. World4, Q an abstract and
imaginary space that is imagined existing inside the Graphics rendering software like sketchup, Rhino,
3Ds Max. The rectangular area in this software specified in world coordinates is called a window. A
rectangular area on the display device to which a window is mapped is called a view port. The window
defines what is to be viewed; the view port defines where it is to be displayed.
(viii) The term Conurbation was initiated by
(A) Lewis Mumford (B) Stuart Chapin (C) Patric Geddes (D) Charles Correa
Ans :- (C)
Explanation: Patrick Geddes introduced the term ‘conurbation’, which implies an aggregation of con-
tinuous network of urban communities.
(ix) Permissible building coverage is expressed in term of
(A) People per hectare (B) Dwelling units per hectare
(C) Ground coverage (D) None of the above
Ans: - (C)
Explanation: Permissible building coverage refers to the maximum area of a plot that can be covered
by buildings. It is usually expressed as a percentage of the total area of the plot. For example, if the per-
missible building coverage is 50%, it means that buildings can cover up to 50% of the total area of the
plot, and the remaining 50% must be left as open space.
774 GATE Architecture and Planning: Comprehensive Question Bank

(x) F.A.R. of an area varies with respect to


(A) Size of the plot (B) Abutting Road width and use
(C) Location (D) Height of the building
Ans :- (C)
Explanation: F.A.R. (Floor Area Ratio) of an area can vary based on multiple factors. Typically, it is a
measure of the total floor area of a building that can be constructed on a given plot of land.
Some of the factors that can influence the F.A.R. of an area are:
• The size of the plot: larger plots may have a higher F.A.R. allowance.
• Abutting road width and use: The width of the road, the type of road (residential, commercial, etc.),
and the proximity of the plot to the road can all impact the F.A.R. allowance.
• Location: The zoning of the area, the intended land use (residential, commercial, industrial, etc.),
and the surrounding built environment can all influence the F.A.R. of a plot.
• Height of the building: In some cases, the F.A.R. may be higher for taller buildings, although this can
vary based on other factors as well.
(xi) WBLOCK command in autocad creates
(A) Wooden Block (B) Drawing File (C) ASCII file (D) Data file
Ans: - (B)
Explanation: The WBLOCK command in AutoCAD creates a new drawing file from a selection of ob-
jects in the current drawing. The objects are saved as a block in the new drawing, which can be used in
other drawings or inserted back into the original drawing.
(xii) House near Luxor, Egypt was designed by
(A) Robert Venturi (B) Hassan Fathy (C) I.M. Pie (D) Lucio Costa
Ans: - (B)
Explanation: The ‘Gourna Village’ near Luxor, Egypt was designed by Hassan Fathy.
(xiii) Self -help housing concept was propagated by
(A) Charles Abraham (B) UNCHS (C) Laurie Baker (D) Burger R. Antipode
Ans: - (A)
“Man’s Struggle for Shelter in an Urbanizing World” by Charles Abraham’s
(xiv) Board acre city was proposed by
(A) Ebenezar Howard (B) Raymond Unwin (C) Frank Lloyd Wright (D) C.A. Doxiadis
Ans:- (C)
(xv) Desirable spacing trees along a highway is
(A) 3m to 7m (B) 9m to 15m (C) 20m to 25m (D) 26m to 30m
Ans: - (B)
Across the highway it should be planted 10-12 m away from the centre line of extreme lane.
Q.2 What do the following abbreviations stand for
UCS: Universal Coordinate System (b) UNCHS: United Nations centre for Human Settlement
(c) GL: Ground Level (d) LOT: Line of travel
(e) BOD : Biological Oxygen Demand
Hemant Vilas Parulekar 775

Q.3 Match the following


(a) String (A) Estuaries
(b) Pyrocell (B) Hygrometer
(c) Mangroves (C) Scanner
(d) Raster (D)Character sequence
(e) Humidity (E) Fire resistance
(ii)
(a) Colour scale (A) Renaissance
(b) Plan Libre (B) Greek Period
(c) Dyer D.J. (C) Town Design
(d) Calcutta (D) Concentric zone theory
(e) Doxiadis (E) Linear City
(f) Fredrick Gibbard (F) People and housing in third World
(g) E.W. Burgess (G) Ekistics
(h) Miletus (H) Future of Metropolis
(i) Karlsruhu (I) Le Corbusier
(j) Lloyd Rodwin (J) Munsell
Ans: (i)- a -(D), b - (E ) , c - (A), d - (C ), e - (B)
Ans (ii): - a - (J), b - (I), c - (F), d - (E ), e - (G), f - (C ), g - (D), h - (B), i - (A), j - (H)
Q.4 Fill in the blanks:
(i) Renaissance period in India synchronises with Mughal Rule.
(ii) A public record survey or map showing details of value, extent and ownership of land is known as
Cadastral Map
(iii) Laurie baker is the receipent of Habitat A ward 1987 and Padamshree in 1990.
(iv) Aerobic bacteria require oxygen for their existence.
(v) Mansara is the elaborate treatis on building and town planning in ancient India.
(vi) Incineration involves the burning of refuse in the incinerator.
(vii) Glare increases the apparent size of the object.
(viii) Visual survey involves examination of the form, appearance, and texture of built form.
(ix) A straight line movement of an object from one position to another on the display screen is
known as cursor of the object.
(x) A form of pruning in which plants are severely sheared into unnatural shapes such as animals or
chess pieces is called topiary.
776 GATE Architecture and Planning: Comprehensive Question Bank

Q.5 Indicate the following as TRUE or FALSE.


(i) The saturation scale of any colour can be achieved by mixing white or black.
(ii) If the pollutants are dispersed in a large volume of air the resultant pollution increases.
(iii) Transmission loss sound varies directly with the frequency of sound.
(iv) The minimum height permitted for habitable room is 2.1 m.
(v) Timgad is an example of Roman Town.
(vi) “The Future of Housing” was written by Charles Abraham.
(vii) Arthashastra was written during Vedic Period.
(viii) Shrubs cannot reduce the velocity of wind.
(ix) END CIRCLE, FOR is a valid FORTRAN statement.
(x) The collective term for the foliage of a tree is canopy.
Ans: -i) - FALSE, ii) - FALSE, iii) - TRUE, iv) - FALSE, v) - TRUE, vi) - TRUE, vii) - FALSE, viii) -
FALSE, ix) - FALSE , x) - FALSE.
Q.6 Distinguish between :
(i) Freehold and Leasehold property
(ii) Betterment Levy and Development Levy
(iii) Core House and Shell House
(iv) Dandaka and Padmaka
(v) Year’s purchase in perpetuity and Sinking fund
(vi) Value and Hue of colour
(vii) Full enclosure and loss of enclosure
(viii) Mouse and Digitizer
(ix) Coagulant and Flocculent
(x) Herbaceous and Herbicide
Ans: -
(i) Freehold property refers to property that is owned indefinitely by the owner, with no restrictions on
its transferability or usage. In contrast, leasehold property is a type of property that is owned for a limit-
ed period, as specified in the lease agreement.
(ii) Betterment levy is a tax imposed by the government on landowners to pay for the costs of infrastruc-
ture improvements in the surrounding area that may increase the value of their property. Development
levy, on the other hand, is a tax imposed on developers to pay for the costs of infrastructure improve-
ments, including roads, sewers, and other utilities, that are necessary for new developments.
(iii) A core house is a structure that has been built with a complete framework, but it lacks finishing work
such as plastering, flooring, and painting. In contrast, a shell house is a structure that has a complete
roof, walls, and floor, but is otherwise unfinished, including the absence of interior walls, plumbing, and
electrical work.
(iv) Dandaka and Padmaka are two types of grid patterns used in ancient Indian town planning. Dan-
daka refers to a grid pattern with straight streets that intersect at right angles, while Padmaka refers to a
grid pattern in which the streets intersect at oblique angles.
Hemant Vilas Parulekar 777

(v) Year’s purchase in perpetuity is a calculation used to determine the present value of a stream of in-
come that will continue indefinitely. A sinking fund is a fund into which money is set aside regularly to
pay for future expenses or to retire debt.
(vi) Value and hue are two characteristics of color. Value refers to the lightness or darkness of a color,
while hue refers to the color itself.
(vii) Full enclosure refers to a situation in which a building or space is completely enclosed by walls, floor,
and roof. Loss of enclosure refers to the opposite situation in which a space is open or lacks enclosure.
(viii) A mouse is a computer input device that allows a user to move a cursor on a computer screen. A
digitizer, on the other hand, is a device that converts analog signals, such as handwritten or drawn im-
ages, into digital signals that can be stored and manipulated by a computer.
(ix) Coagulant and flocculant are chemicals used in water treatment processes. Coagulants are chemi-
cals that cause small particles to clump together to form larger particles, while flocculants are chemicals
that cause the larger particles to settle out of the water.
(x) Herbaceous refers to plants that have leaves and stems that die down at the end of the growing sea-
son, while herbicides are chemicals used to kill or control the growth of plants.
SUB SECTION A-2
Q.7 Write a program in FORTRAN or BASIC for computing the volume of a sphere for variable radius.
Ans: -
10 REM Program to compute volume of a sphere for variable radius
20 PRINT “Enter the radius of the sphere:”
30 INPUT r
40 v = 4 / 3 * 3.14159 * r 3
50 PRINT “The volume of the sphere is”; v
60 END
Here is how the program works:
Line 10 is a comment line that explains what the program does.
Line 20 prompts the user to enter the radius of the sphere.
Line 30 reads the user’s input and stores it in a variable r.
Line 40 calculates the volume of the sphere using the formula v = 4/3 * pi * r3, where pi is the constant
3.14159.
Line 50 prints the result of the calculation to the screen.
Line 60 ends the program.
To use the program, save it to a file with a .bas extension and run it using a BASIC interpreter. When
prompted, enter the radius of the sphere and the program will compute and display its volume.
Q.8 In a square frame of 10cm X 10cm make a composition of figure and ground (positive and nega-
tive) indicating expansion of figure on ground and ground on figure.
Ans: -
Q.9 Explain “Venturi Effect” of air flow inside a building
778 GATE Architecture and Planning: Comprehensive Question Bank

Ans: - The Venturi effect is a phenomenon that occurs when air flows through a constriction or narrow
opening in a building. When air flows through the constriction, its velocity increases, and its pressure
decrease. This occurs due to the conservation of energy and mass, according to Bernoulli’s principle. As
the air flows through the narrow opening, its velocity increases, but the mass flow rate of air remains
constant. This means that the kinetic energy of the air increases and is converted from pressure energy,
which causes a decrease in pressure.
In a building, the Venturi effect can occur when air flows through narrow openings, such as windows,
doors, or air conditioning ducts. When air flows through these openings, the pressure inside the build-
ing decreases, which can cause air to flow into the building. This can be beneficial for natural ventilation,
as it allows fresh air to flow into the building without the need for mechanical ventilation systems.
However, the Venturi effect can also cause problems in buildings, particularly with respect to air in-
filtration and indoor air quality. If the building envelope is not properly sealed, air can flow into the
building through openings and cause drafts, moisture problems, and indoor air pollution. Therefore, it
is important to design buildings with proper air sealing and ventilation systems to minimize the effects
of the Venturi effect.
Q.10 Proposed land use distribution for 10-hectare housing complex is 65%, 10%, 12.5%, for residential,
public - semi-public, open space, and road respectively. Costs of land acquisition, site development
and construction are Rs 1200/m2, Rs 100/m2 and Rs 4000 /m2, respectively. Permissible FAR being
1.5, find out the cost of a dwelling unit of 85 m2.
Ans: - Next, we can find the cost of each dwelling unit:
Total cost of each dwelling unit = Cost of land acquisition + Cost of site development + Cost of con-
struction
Total cost of each dwelling unit = (85 m² x Rs 1200/m²) + (85 m² x Rs 100/m²) + (85 m² x Rs 4000/m²)
Total cost of each dwelling unit = Rs 102,000 + Rs 8,500 + Rs 340,000
Total cost of each dwelling unit = Rs 450,500
Therefore, the cost of a dwelling unit of 85 m² in the proposed housing complex is Rs 450,500.
Q.11 Illustrate with sketches highlighting the major features of any planned Indian city.

SECTION B (PART I)
Q.12 Briefly discuss the important contribution of reinforced cement concrete to modern Architecture.
Ans: - Reinforced concrete is a material that combines the strength of steel and the durability of concrete.
Its invention in the mid-19th century revolutionized the construction industry, and its widespread use
has had a profound impact on modern architecture.
One of the most significant contributions of reinforced concrete to modern architecture is its flexibility.
The material can be molded into a wide range of shapes and sizes, allowing architects to design structures
that were previously impossible. The flexibility of reinforced concrete has enabled architects to create
buildings with complex curves and shapes, leading to an unprecedented level of design creativity.
Reinforced concrete has also allowed architects to build structures that are more durable and resilient.
By reinforcing the concrete with steel bars, the material can resist much higher stresses and strains than
traditional concrete. This has enabled the construction of tall buildings and long-span bridges that are
able to withstand the forces of nature.
Hemant Vilas Parulekar 779

Moreover, reinforced concrete has also contributed to the speed of construction. Because it is a precast
material, it can be easily transported to the construction site and assembled quickly. This makes it an
ideal material for large-scale construction projects like bridges and dams.
Finally, reinforced concrete has also had a profound impact on the affordability of construction. Its
widespread use has helped to lower the cost of construction, making it possible for more people to afford
to build homes and other structures.
In summary, the invention of reinforced concrete has had a profound impact on modern architecture,
enabling architects to design more creative, durable, and affordable structures that were previously
impossible.
Q.13 Greek Architecture is sometimes called “A Carpentry in Marble” - Justify.
Ans: - Greek architecture is often referred to as “a carpentry in marble” because it reflects a fundamental
emphasis on the post-and-lintel system, which is like the framing of a house. The Greek architectural
style is based on a system of proportions and ratios, with a strong focus on balance, symmetry, and
harmony in design.
The post-and-lintel system, which was widely used in ancient Greece, involves the use of vertical
columns (posts) and horizontal beams (lintels) to create a stable framework for buildings. This system
is like the framing of a house, where vertical studs and horizontal beams provide a stable structure for
walls and floors.
Greek architects used the post-and-lintel system to create some of the most iconic structures in ancient
history, including the Parthenon, the Temple of Olympian Zeus, and the Temple of Apollo at Delphi.
These buildings feature rows of columns that support a horizontal entablature, which often includes
elaborate decorative elements.
The use of marble in Greek architecture is also significant. Marble was abundant in Greece and was
used extensively in building construction, sculpture, and other decorative applications. Greek architects
used marble to create intricate details and patterns in their buildings, such as the sculpted friezes and
pediments that adorn many temples.
Overall, the post-and-lintel system and the use of marble in Greek architecture were key factors in
creating the unique style and beauty of ancient Greek buildings. The attention to detail and emphasis
on harmony and balance in design helped to create enduring architectural masterpieces that continue to
inspire architects and designers to this day.
Q.14 Describe the fire proofing treatment of steel columns in public buildings.
Ans: - The fireproofing treatment of steel columns in public buildings is an important aspect of fire
safety design. Steel columns are often used in the construction of large public buildings such as offices,
hotels, and shopping malls. In the event of a fire, the heat generated can cause the steel to weaken and
deform, which can lead to the collapse of the building. To prevent this, steel columns are often treated
with fireproofing materials to protect them from the effects of fire.
There are several types of fireproofing materials that can be used to protect steel columns. One common
material is spray-applied fire resistive material (SFRM), which is a type of cementitious material that can
be applied to the surface of the steel column. SFRM forms an insulating layer that can protect the steel
from the effects of fire. Another material that can be used is intumescent coatings, which expand when
exposed to heat and form a protective layer around the steel column.
780 GATE Architecture and Planning: Comprehensive Question Bank

In addition to these fireproofing materials, other design considerations can be made to further improve
the fire resistance of the building. For example, the use of fire barriers, fire doors, and sprinkler systems
can help to prevent the spread of fire and reduce the risk of collapse. Proper evacuation routes and
emergency lighting can also help occupants to safely exit the building in the event of a fire.
Overall, the fireproofing treatment of steel columns is an important aspect of fire safety design in public
buildings. By using the appropriate fireproofing materials and design considerations, the risk of collapse
and injury in the event of a fire can be greatly reduced.
Q.15 Determine the allowable tensile force P that may be applied across two plates connected to one an-
other by fillet weld joints as shown in the Fig -B - I 15. Allowable working stress of weld is 75 MPa.
Consider only shearing stress in the weld. The load is applied midway between the two welds.

Q.16 The optimistic (t0), most likely (tm) and permissible (tp) times of activities on the critical path of a
PERT network are given below. Calculate the mean and standard deviation of the critical path du-
ration.
Activities on the t0 tm tp
critical path of
5 10 15
8 16 24

Q.17 Explain the important Architectural Design feature of any ONE of the following :
(i) Belgian Embassy (ii) Bahai Temple (iii) CMC building - Bombay
Ans: - (i) Belgian Embassy:
(j) The Belgian Embassy in New Delhi is a fine example of contemporary architecture that blends ele-
ments of traditional Indian design with modern European design sensibilities. The building features a
dramatic façade that is clad in reflective glass and bronze louvers that provide both shade and privacy.
The façade is inspired by the traditional Indian jaali screen, which is used to filter sunlight and create
intricate patterns of light and shadow. The building’s design also incorporates a central courtyard, which
serves as a focal point for the embassy’s activities and provides natural light and ventilation to the inte-
rior spaces.
(ii) Bahai Temple:
The Bahai Temple, also known as the Lotus Temple, is located in New Delhi and is one of the most iconic
modern buildings in India. The temple’s design is inspired by the lotus flower, which is a sacred symbol
in many Indian religions. The building features 27 marble-clad petals that are arranged in a series of
concentric circles around a central hall that can accommodate up to 2,500 people. The temple’s design
also incorporates several sustainable design features, including natural ventilation and lighting, rainwa-
ter harvesting, and solar power.
(iii) CMC building - Bombay:
The CMC building in Mumbai (formerly Bombay) is a landmark Art Deco building that was built in
the 1930s. The building features a distinctive façade that is characterized by a series of horizontal and
vertical lines, curved corners, and geometric patterns. The building’s design also incorporates several
features that were innovative for their time, including a reinforced concrete structure, large floor plates,
and modern mechanical systems. The building is now a protected heritage structure and is recognized
as an important example of Mumbai’s Art Deco architecture.
Hemant Vilas Parulekar 781

Q.18 Explain with illustration the construction of dome over a square plan as developed during Indo-
Islamic phase.

Q.19 Explain sound - focii and Dead Spot in an auditorium


Ans: - Sound foci and dead spots are acoustic phenomena that can occur in auditoriums or other en-
closed spaces.
Sound foci are points in a room where sound waves converge and create a very strong signal. This occurs
due to the reflection of sound waves off the walls and ceiling of the room, and when these reflected waves
meet at a particular point, they can reinforce each other, creating a louder sound at that spot. This effect
can be used intentionally in the design of a room to create a “sweet spot” for listening or speaking.
On the other hand, dead spots are areas in a room where sound waves cancel each other out, resulting in
a lack of sound or a weak, muffled sound. This can happen when sound waves reflect off the surfaces of
a room in such a way that the peaks of one wave meet the troughs of another wave, resulting in destruc-
tive interference. Dead spots can be problematic in an auditorium because they can make it difficult for
audience members to hear clearly, and can be especially pronounced in low-frequency sounds.
To mitigate the effects of sound foci and dead spots in an auditorium, careful attention must be paid
to the design and layout of the space. This may involve the use of specialized acoustic materials, such
as sound-absorbing panels or diffusers, as well as the strategic placement of speakers and other sound
equipment to ensure an even distribution of sound throughout the space. Acoustic modeling software
can also be used to simulate the behavior of sound waves in a space and help identify potential sound
foci and dead spots.
Q.20 Sketch a wall footing for a 250 mm wall of a two - storied residential building on a ground having a
safe bearing capacity of 10 tons/m2 at a depth of 1 m below the surface . The load from the wall at the
ground level is 7tons/m length of the wall.

Q.21 Sketch the connection of the house water supply pipe with the municipal water main showing the
position of Ferrule, water meter , goose neck and stop cock.

Q.22 Outline briefly the Architectural features of Gandhi Smarak Sangrahalay , Sabarmati, and Ahmad-
abad.
Ans:- Gandhi Smarak Sangrahalay, also known as the Sabarmati Ashram or the Gandhi Ashram, is a
museum and memorial dedicated to the life and work of Mahatma Gandhi, located in the Sabarmati
suburb of Ahmedabad, Gujarat, India. The complex was established in 1915 and served as the home of
Gandhi from 1917 to 1930.
The architectural features of the Gandhi Smarak Sangrahalay include:
1. Simple design: The overall architectural design of the complex is simple and unpretentious, reflect-
ing Gandhi’s philosophy of simplicity and austerity.
2. Use of local materials: The complex was built using locally available materials such as brick, stone,
and wood, which were commonly used in traditional Indian architecture.
3. Courtyard: The main building of the complex is arranged around a central courtyard, which pro-
vides light and ventilation to the surrounding rooms and also serves as a gathering place for visitors.
4. Natural lighting: The buildings are designed to maximize natural lighting and ventilation, with large
windows and skylights in most rooms.
782 GATE Architecture and Planning: Comprehensive Question Bank

5. Jali screens: The buildings are also adorned with intricate jali screens, which provide privacy and
shade while also allowing for air circulation.
6. Charkha room: One of the most iconic features of the complex is the Charkha room, which contains
Gandhi’s spinning wheel and other personal belongings. The room is designed to be simple and
contemplative, with minimal decoration.
7. Open spaces: The complex includes a number of open spaces and gardens, which are used for gath-
erings, meditation, and reflection.
The overall architectural design of the Gandhi Smarak Sangrahalay is in keeping with Gandhi’s vision of
a simple and sustainable way of life. It reflects his emphasis on self-reliance, community, and harmony
with nature.
Q.23 State the factors considered by the Buddhists while selecting the monastic sites.
Ans: - Buddhists considered several factors while selecting monastic sites, including:
1) Accessibility: Monastic sites were often located close to major trade routes or transportation hubs
to make it easier for monks to travel to and from the site.
2) Natural setting: The site was often located in a serene and peaceful natural setting, such as a forest
or near a river, to provide an environment conducive to meditation and reflection.
3) Water supply: The site was located near a reliable source of water, such as a river or spring.
4) Soil quality: The soil at the site needed to be suitable for building and gardening.
5) Climate: The site needed to have a favorable climate, with enough rainfall and sunshine for ag-
riculture.
6) Security: The site needed to be secure from natural disasters and hostile groups.
7) Spiritual significance: The site may have had spiritual significance, such as being associated with
a previous Buddha or being the location of a significant event in the life of the Buddha.
8) Existing structures: The site may have had existing structures that could be reused or repurposed
for the monastery.
Q.24 State the advantages of Fibre Reinforced Plastic (FRP) as structural materials and mention some
area of their use in building.
Ans: - Fibre Reinforced Plastic (FRP) is a composite material that consists of a polymer matrix rein-
forced with fibres. Here are some of the advantages of using FRP as a structural material:
1) High Strength-to-Weight Ratio: FRP composites are lightweight materials with high tensile strength,
allowing for a strong structural system that weighs less than traditional materials.
2) Corrosion Resistance: FRP composites are highly resistant to corrosion and deterioration due to
moisture, chemicals, and UV exposure. This makes them ideal for use in harsh environments such
as coastal areas and chemical plants.
3) Design Flexibility: FRP can be easily moulded into various shapes and sizes, allowing for greater
flexibility in design. It can be easily fabricated into custom shapes, curved surfaces, and even com-
plex 3D structures.
4) Durability: FRP composites have high durability and a long service life. They are resistant to fatigue
and can withstand heavy loads and extreme temperatures, making them ideal for use in demanding
applications.
5) Low Maintenance: FRP composites require little to no maintenance over their lifetime, making them
a cost-effective alternative to traditional materials.
Hemant Vilas Parulekar 783

Some areas of use for FRP in building include:


1) Structural components such as beams, columns, and trusses.
2) Bridge decks, railings, and barriers.
3) Cladding and roofing panels.
4) Window frames and door frames.
5) Flooring and wall panels.
FRP composites offer several benefits over traditional materials and are becoming increasingly popular
in construction due to their unique properties and design flexibility.
Q.25 Draw elevation and details of an Aluminium window 2 m long and 1.5 m high with four vertical
division out of which the middle two should consist of side - hung openable shutter of 1.0 m height.

Q.26 Sketch a semi-circular brick arch over an opening of 1.5 m and label all its components.

Q.27 Design a dining hall with sitting capacity of 30 for a guest house showing the furniture layout,
relationship to the kitchen and washing area for the diners.

Q.28 Explain with sketches the characteristic features of Mughal Garden.

Q.29 Discuss in brief the process of preparation, properties, and the use of plywood.
Ans: - Plywood is a manufactured wood panel made from thin sheets of wood veneer. The process of
preparing plywood involves the following steps:
1) Log selection: The logs are carefully selected to ensure that they are free from knots, decay, and other
defects that could weaken the plywood.
2) Slicing veneer: The selected logs are then sliced into thin sheets of wood veneer using a large rotary
lathe.
3) Gluing veneers: The veneers are then sorted, stacked, and glued together using adhesives. The most
common adhesive used in plywood is phenol formaldehyde resin.
4) Pressing veneers: The glued veneers are then pressed under high pressure and high temperature to
form a strong and durable sheet of plywood.
Properties of plywood:
1) Strength: Plywood is a strong and durable material that can withstand heavy loads and resist bend-
ing, warping, and cracking.
2) Stability: Plywood is a stable material that resists changes in humidity and temperature, making it
suitable for use in a wide range of environments.
784 GATE Architecture and Planning: Comprehensive Question Bank

3) Versatility: Plywood is a versatile material that can be used in a variety of applications, including
furniture, flooring, walls, and roofs.
4) Ease of use: Plywood is easy to cut, shape, and install, making it a popular choice among DIY enthu-
siasts and professionals alike.
Uses of plywood:
1) Furniture: Plywood is commonly used in the construction of furniture, including chairs, tables, and
cabinets.
2) Flooring: Plywood is often used as a subflooring material in homes and commercial buildings, as
well as in the construction of sports floors.
3) Walls and roofs: Plywood is commonly used in the construction of walls and roofs, particularly in
buildings with a timber frame.
4) Packaging: Plywood is often used in the packaging industry to create sturdy and durable shipping
crates and pallets.
Overall, plywood is a versatile and widely used building material that offers a range of benefits, including
strength, stability, and ease of use.
Q.30 Design illumination for a 6 m X 4 m computer lab so as to achieve good working environment. The
ceiling height is 3 m and the false ceiling is at 2.4 m. Draw an inverted ceiling plan and a section of
the room.

SECTION - B (PART -I)


Q.1 Sketch two consecutive layers of right - angle corner of a 25 cm thick brick wall in English bond.

Q.2 Indicate the function of a structure Plan.


Ans: - A structure plan is a comprehensive document that guides physical development and land use
over a long-term period, typically 20 to 30 years. The function of a structure plan is to provide a strategic
vision and framework for future development and land use within a defined area, such as a city, town, or
region. The specific functions of a structure plan can include:
1) Establishing land use policies and guiding principles for development, growth, and change.
2) Identifying and designating areas for various land uses, such as residential, commercial, in-
dustrial, recreational, and open space.
3) Providing guidelines for the development of infrastructure, such as transportation, water,
sewage, and solid waste management systems.
4) Ensuring that development is coordinated and that infrastructure is provided in a timely and
cost-effective manner.
5) Providing a basis for the preparation of zoning and other land use regulations.
6) Ensuring that environmental and social considerations are taken into account in the planning
process.
7) Facilitating coordination and cooperation between different levels of government and various
stakeholders involved in land use planning and development.
Overall, the function of a structure plan is to ensure that development is orderly, efficient, sustainable,
and meets the needs of the community over the long term.
Hemant Vilas Parulekar 785

Q.3 The passenger demand during the two hours morning peak period along a bus route is 4000 pas-
sengers. The round trip time along the route is 50 minutes and the average vehicle occupancy is 75
passengers. Calculate the hourly flow of buses and the number of buses required to provide this flow
assuming 5% of the buses will be under service and repair.
Ans: - To calculate the hourly flow of buses and the number of buses required, we need to first deter-
mine the number of passengers that can be carried by one bus in one trip.
The round trip time along the route is 50 minutes, so the time for one-way trip is 25 minutes.
During the two-hour morning peak period, there are 4 peak hour trips (2 in each direction), which
means there are 2 peak hour one-way trips.
The total number of passengers during the peak period is 4000, so the number of passengers in one
peak hour one-way trip is:
4000 / 2 = 2000 passengers
Assuming an average vehicle occupancy of 75 passengers, the number of buses required for one peak
hour one-way trip is:
2000 / 75 = 26.67 or 27 buses (rounded up)
However, we need to take into account the fact that 5% of the buses will be under service and repair, so
we need to increase the number of buses to compensate for this.
The number of buses required is:
27 / (1 - 0.05) = 28.42 or 29 buses (rounded up)
So the hourly flow of buses required is 29 buses.
Q.4 Explain how the population projection can be estimated for any settlement by Employment method.
Ans: - The employment method is one of the commonly used methods for estimating population projec-
tions for urban or rural settlements. This method uses the number of employed persons in a given area
to estimate the total population.
The steps involved in estimating population projections using the employment method are as follows:
1) Collect data on the number of people employed in various sectors, such as agriculture, industry,
services, etc. in the given area for the current year.
2) Determine the average number of persons employed per household for each sector. This can be ob-
tained from the census or other surveys.
3) Estimate the total number of households in the area by dividing the total number of employed per-
sons in each sector by the average number of persons employed per household.
4) Project the future number of employed persons in each sector based on employment trends and
growth rates.
5) Estimate the future number of households in the area by dividing the projected number of employed
persons in each sector by the average number of persons employed per household.
6) Finally, estimate the future population of the area by multiplying the projected number of house-
holds by the average household size.
It is important to note that this method assumes that the average number of employed persons per
household remains constant over time, and does not take into account factors such as migration, mor-
tality rates, or changes in household size. Therefore, it should be used in conjunction with other methods
for estimating population projections to improve accuracy.
Q.5 Explain in brief the composting method of refuse disposal.
Ans: - Composting is a method of refuse disposal that involves the decomposition of organic waste mate-
rials in the presence of oxygen to produce a nutrient-rich soil-like material called compost. Composting
can be done on a large scale in municipal facilities or on a smaller scale in home composting systems.
786 GATE Architecture and Planning: Comprehensive Question Bank

In the composting process, organic waste materials such as food waste, yard waste, and paper products
are combined with a bulking agent such as wood chips, straw, or leave to create a compost pile or bin.
The pile or bin is regularly turned or mixed to ensure proper aeration and moisture levels, which are
important for the microorganisms that decompose the organic materials.
As the organic materials break down, they release heat and carbon dioxide, which can raise the tem-
perature of the compost pile. This high temperature helps to kill pathogens and weed seeds, making the
compost safe to use as a soil amendment. The end product of composting is a nutrient-rich material that
can be used as a soil conditioner, fertilizer, or mulch in gardening, landscaping, and agriculture.
Composting has several advantages as a method of refuse disposal. It reduces the volume of organic
waste sent to landfills, which reduces the amount of greenhouse gases released into the atmosphere. It
also creates a useful product that can improve soil health and reduce the need for chemical fertilizers.
Composting can be done on-site, which can reduce transportation costs and carbon emissions associat-
ed with hauling waste to a distant landfill. Additionally, composting can help to create local jobs in waste
management and agriculture.
Q.6 Prepare a flow chart for residential site planning by adopting modern management technique.
Ans: - Here’s a flowchart for residential site planning using modern management techniques:
1) Define project scope and objectives
2) Conduct a feasibility study to determine if the project is economically viable
3) Analyze the site and surrounding area to identify any constraints and opportunities
4) Develop a conceptual site plan that includes key features and zoning requirements
5) Conduct community engagement to solicit feedback and gather input from stakeholders
6) Refine the site plan based on community feedback and make any necessary revisions
7) Develop detailed engineering plans, including grading, drainage, and utility plans
8) Conduct an environmental impact assessment and ensure the project is compliant with all relevant
regulations
9) Develop a construction plan and budget, including a project timeline and cost estimates
10) Implement the project, including the construction and development of the site
11) Monitor and evaluate the project to ensure it meets the project scope and objectives, and make any
necessary adjustments
Q.7 Briefly describe the indicators of Physical Planning.
Ans: - Physical planning is concerned with the physical design and arrangement of structures, infra-
structure, land use and the environment. Indicators of physical planning are used to measure the ef-
fectiveness and progress of physical planning initiatives, and they help to assess the quality of the built
environment. Some indicators of physical planning include:
1) Land use: This refers to the way land is utilized in terms of activities such as residential, commercial,
industrial, institutional and recreational.
2) Density: This refers to the number of people or buildings in a given area. It is usually measured in
terms of population per unit area or the number of buildings per unit area.
3) Building typology: This refers to the style, design, and character of buildings in a particular area.
4) Urban form: This refers to the physical structure and layout of an urban area, including the arrange-
ment of streets, blocks, open spaces, and public facilities.
Hemant Vilas Parulekar 787

5) Accessibility: This refers to the ease with which people can move from one location to another, using
various modes of transportation.
6) Infrastructure: This refers to the physical structures and facilities that support human activities, such
as roads, bridges, utilities, and public spaces.
7) Natural environment: This refers to the quality of the natural features and elements, such as air and
water quality, green spaces, and biodiversity, that support human well-being.
These indicators help to assess the effectiveness of physical planning policies and strategies in achieving
the desired objectives, such as sustainability, livability, and resilience, and guide decision-making and
policy formulation in physical planning.
Q.8 Sketch the common types of on - street parking. Discuss their advantages and disadvantages.

Q.9 Describe the survey to be conducted for developing a shopping centre of an urban area.
Ans: - Developing a shopping center in an urban area involves conducting several surveys and studies to
ensure that the center meets the needs of the community and is economically viable. Some of the surveys
that may be conducted include:
Market Analysis: This involves analyzing the demographic and economic characteristics of the sur-
rounding area to determine the potential demand for the shopping center. The analysis would also ex-
amine the competition in the area and identify gaps in the market that the shopping center could fill.
Site Analysis: This involves assessing potential locations for the shopping center and evaluating the suit-
ability of the site based on factors such as accessibility, visibility, and zoning regulations.
Traffic Analysis: This involves analyzing traffic patterns in the surrounding area to determine the best
location for access points to the shopping center and to identify any potential traffic issues that need to
be addressed.
Environmental Impact Assessment: This involves assessing the potential impact of the shopping center
on the environment, including its impact on air quality, noise levels, and the local ecology.
Cost Analysis: This involves estimating the cost of constructing and operating the shopping center, in-
cluding the cost of land acquisition, construction, staffing, and marketing.
Design Survey: This involves conducting a survey of potential customers to determine their preferences
for the design and layout of the shopping center, including the mix of stores, the amenities offered, and
the overall look and feel of the center.
By conducting these surveys and studies, developers can ensure that the shopping center they develop
is well-suited to the needs of the community and is likely to be successful both financially and socially.
788 GATE Architecture and Planning: Comprehensive Question Bank

Q.10 A city with a population of 1,00,000 discharges sewage of 120 lpcd in a stream having a flow of 1
cu.m/sec. The BOD of sewage is 220 mg/lit. The BOD content in the upstream before the outfall is
0.8 mg /lit. Calculate the BOD content of the stream in mg/lit just downstream of outfall.
Ans: - The BOD (biochemical oxygen demand) of sewage is a measure of the amount of oxygen required
to break down the organic matter in the sewage. When sewage is discharged into a stream, the BOD of
the stream will increase due to the additional organic matter in the water.
To calculate the BOD content of the stream just downstream of the outfall, we can use the following
formula:
BOD_downstream = BOD_upstream + (BOD_sewage * Q_sewage / Q_stream)
where: BOD_upstream = the BOD content of the stream upstream of the outfall (mg/lit) BOD_sewage
= the BOD of the sewage (mg/lit) Q_sewage = the flow rate of sewage (lit/sec) Q_stream = the flow rate
of the stream (lit/sec)
We can calculate the values for these variables as follows:
BOD_upstream = 0.8 mg/lit
BOD_sewage = 220 mg/lit
Q_sewage = population * discharge per capita = 100000 * 120 / 1000 = 12000 lit/sec
Q_stream = 1 cu.m/sec * 1000 lit/cu.m = 1000 lit/sec
Substituting these values into the formula, we get:
BOD_downstream = 0.8 + (220 * 12000 / 1000 / 1000) BOD_downstream = 3.56 mg/lit
Therefore, the BOD content of the stream just downstream of the outfall is 3.56 mg/lit.
Q.11 Sate the objective and ceiling limits under Urban Land (Ceiling and Regulation) Act, 1976.
Ans: - The Urban Land (Ceiling and Regulation) Act, 1976 was enacted in India with the objective
of regulating the concentration of urban land in the hands of a few, and ensuring a more equitable
distribution of urban land among the people. The Act provides for the imposition of a ceiling on vacant
land and surplus land in urban agglomerations, and for the acquisition of such land in excess of the
ceiling limit by the state government for distribution to the landless and for urban development.
The ceiling limit was determined by the Act based on the nature of the urban agglomeration, and was
specified in terms of the maximum permissible extent of land that could be held by an individual or
family. The ceiling limit varied between 500 square meters to 2,000 square meters, depending on the
location and size of the urban agglomeration. In addition to the ceiling limit, the Act also provided for
a margin of 10% excess land to be held over and above the ceiling limit, subject to certain conditions.
The Act also provided for the determination of the value of the surplus land, and for the payment of
compensation to the landowner for the land acquired by the state government. The compensation was
fixed at the market value of the land as on the date of the notification issued under the Act, subject to
certain adjustments.
The objective of the Urban Land (Ceiling and Regulation) Act, 1976 was to promote a more equitable
distribution of urban land and to prevent the concentration of urban land in the hands of a few. The
ceiling limits specified in the Act were intended to ensure that the benefits of urban development were
more widely shared, and that the urban poor were provided with access to land for housing and other
purposes.
Hemant Vilas Parulekar 789

Q.12 Outline the process involved in Multi - Level planning.


Ans:- Multi-level planning is a process of planning that involves multiple levels of government, organi-
zations, and stakeholders in the planning process. The process of multi-level planning can be outlined
as follows:
1) Identification of the problem: The first step is to identify the problem or issue that needs to be ad-
dressed through planning. This may involve a variety of stakeholders, including government offi-
cials, community members, and experts in the field.
2) Data Collection and Analysis: The next step is to collect and analyze data on the problem, including
the causes, potential solutions, and relevant social, economic, and environmental factors.
3) Goal Setting: Based on the data analysis, the next step is to establish goals and objectives for the
planning process, taking into account the needs and priorities of different stakeholders and levels of
government.
4) Strategy Development: Once the goals and objectives have been established, the next step is to de-
velop strategies and action plans to achieve them. This may involve identifying policies, programs,
and projects that can be implemented at different levels of government or by different stakeholders.
5) Implementation: After the strategies and action plans have been developed, the next step is to imple-
ment them, with the participation of different stakeholders and levels of government.
6) Monitoring and Evaluation: The final step is to monitor and evaluate the effectiveness of the plan-
ning process and the strategies that were implemented, and to make adjustments as needed.
Overall, multi-level planning involves collaboration and cooperation among different stakeholders and
levels of government, with the goal of addressing complex problems and issues that affect multiple com-
munities and regions. The process can be challenging, but it can lead to more effective and equitable
planning outcomes.
Q.13 Determine the angle at which a pavement should be banked in order to avoid outward sliding of
vehicles along a horizontal circular curve of radius R = 300 m. The maximum allowable speed limit
along the curve is 100 km/hour. The coefficient of side friction is 0.2.
Ans:- In order to avoid outward sliding of vehicles along a horizontal circular curve, the necessary
centripetal force must be provided by the lateral friction force between the tires and the pavement. This
is given by the formula:
F = m * v2 / R
where F is the necessary centripetal force, m is the mass of the vehicle, v is the speed of the vehicle, and
R is the radius of the curve.
The lateral friction force is given by:
Ff = N * f
where Ff is the lateral friction force, N is the normal force acting on the vehicle, and f is the coefficient
of side friction.
The normal force acting on the vehicle can be resolved into its vertical and horizontal components as:
N = m * g * cos(theta)
where g is the acceleration due to gravity, and theta is the banking angle of the pavement.
Equating the centripetal force to the lateral friction force, we get:
m * v2 / R = N * f
790 GATE Architecture and Planning: Comprehensive Question Bank

Substituting the expressions for N and F, we get:


m * v2 / R = m * g * cos(theta) * f
Solving for theta, we get:
theta = arctan(v2 / (g * R * f))
Substituting the given values, we get:
theta = arctan((100 km/hour)2 / (9.81 m/s2 * 300 m * 0.2)) ≈ 14.7 degrees
Therefore, the pavement should be banked at an angle of approximately 14.7 degrees to avoid outward
sliding of vehicles along the horizontal circular curve of radius 300 m, with a maximum allowable speed
limit of 100 km/hour and a coefficient of side friction of 0.2.
Q.14 Explain the stratified sampling and the cluster sampling
Ans: Stratified Sampling and Cluster Sampling are both methods used in statistics to obtain a sample of
a population. They are designed for different scenarios and have distinct methodologies:
Stratified Sampling:
Definition: In stratified sampling, the population is divided into smaller groups, known as strata, based
on shared characteristics or attributes. The strata are mutually exclusive and collectively exhaustive,
meaning every population member belongs to one stratum and only one stratum.
Method: After dividing the population into strata, a random sample is drawn from each stratum. The size
of the sample from each stratum can be proportional to the size of the stratum (proportional stratified
sampling) or equal across all strata (equal stratified sampling).
Purpose: This method is used to ensure that the sample represents important subgroups within the
population. It can increase the precision of the overall sample by reducing the variability within each
stratum.
Example: If a researcher wants to survey high school students’ opinions and the school has different
grades with varying numbers of students, they could use stratified sampling to ensure each grade is
proportionally represented in the sample.
Cluster Sampling:
Definition: Cluster sampling involves dividing the population into clusters, which are often geographically
bounded or naturally occurring groups. Unlike strata, clusters may not be homogeneous.
Method: After forming clusters, a random sample of clusters is selected. All individuals within the chosen
clusters are then surveyed. This is different from stratified sampling, where only a random sample within
each stratum is surveyed.
Purpose: Cluster sampling is often used when it is difficult or costly to survey the whole population. It’s
particularly useful for large-scale surveys spread over a wide geographical area.
Example: If a researcher is conducting a health survey in a large city, they might divide the city into
neighborhoods (clusters) and then randomly select a few neighborhoods for the survey, including all
residents within those neighborhoods.
In summary, while both stratified and cluster sampling involve dividing the population, stratified
sampling focuses on ensuring representation of key subgroups, whereas cluster sampling is often used
for practical reasons of feasibility and cost.
Hemant Vilas Parulekar 791

Q.15 Explain how Mass Curve can be used to determine the capacity of a balancing water tank.
Ans:- The mass curve is a graphical tool used to analyze the inflow and outflow of water in a reservoir
or tank over time. The mass curve is plotted by accumulating the inflow and outflow volumes over time,
and plotting the cumulative volumes against time. The slope of the mass curve at any point represents the
rate of change of storage in the tank at that time.
To determine the capacity of a balancing water tank using the mass curve method, we can follow these
steps:
1. Determine the water demand for the area to be served by the tank. This can be done by estimating
the population and the per capita water demand.
2. Determine the peak hourly demand for the area. This is typically estimated as a percentage of the
daily demand.
3. Determine the inflow to the tank over a representative time period. This can be done by analyzing
the historical rainfall data for the area and estimating the runoff coefficient, which is a measure of the
amount of rainfall that is likely to enter the tank.
4. Determine the outflow from the tank over the same time period. This can be done by estimating the
rate at which water is likely to be drawn from the tank, based on the demand for water in the area.
5. Plot the cumulative inflow and outflow volumes against time to obtain the mass curve.
6. Determine the peak rate of change of storage in the tank by finding the steepest point on the mass
curve. This represents the maximum rate at which water is likely to flow into or out of the tank.
7. Determine the required capacity of the tank by dividing the peak hourly demand by the peak rate of
change of storage.
By using the mass curve method, we can determine the required capacity of a balancing water tank based
on the water demand for the area to be served, the inflow and outflow volumes, and the rate of change
of storage in the tank. This method can be useful in ensuring that the tank is sized appropriately to meet
the needs of the area, while also ensuring that it is not oversized and thus inefficient.
Q.16 Indicate the obligatory Functions of an Urban Local Body.
Ans:- The obligatory functions of an Urban Local Body (ULB) can vary depending on the country and
its laws, but in general, some of the common functions of ULBs are:
1. Provision and maintenance of basic urban services such as water supply, sanitation, solid waste man-
agement, and street lighting.
2. Regulation of land use and building activities, including planning, development control, and build-
ing permits.
3. Maintenance of urban infrastructure, including roads, bridges, parks, playgrounds, and public build-
ings.
4. Collection of taxes, fees, and other revenue, and financial management of the ULB.
5. Promotion of economic development and the provision of social welfare services.
6. Implementation of national and state-level programs and schemes related to urban development,
such as affordable housing, smart cities, and Swachh Bharat Abhiyan in India.
It’s worth noting that the specific functions of ULBs may differ depending on the country and the level
of government. For example, in India, the 74th Constitutional Amendment Act of 1992 assigns 18 func-
tions to ULBs, including urban planning, public health, and slum improvement, among others.
Q.17 Highlight the concept of systems Approach in planning and comment on its merits and demerits.
792 GATE Architecture and Planning: Comprehensive Question Bank

Ans:- The systems approach is a method of analysis and problem-solving that views a problem or issue
as a system of interrelated and interdependent parts. It is a popular method used in planning as it allows
for a holistic and integrated approach to understanding and solving complex problems. In the context of
planning, a system is any collection of interacting components that form a complex and interdependent
whole.
The systems approach involves identifying the components of a system, their interrelationships, and the
environment in which the system operates. It also involves identifying the inputs, processes, and outputs
of the system. By understanding the system as a whole, it becomes easier to identify the underlying
causes of problems, as well as the potential impacts of proposed solutions.
The systems approach has several advantages in planning. Firstly, it provides a holistic view of the
problem or issue, which is important for developing effective and integrated solutions. It also provides
a structured and logical framework for problem-solving, making it easier to identify the root causes of
a problem and develop effective solutions. Additionally, it allows for the identification of unintended
consequences of proposed solutions.
However, the systems approach also has some limitations. Firstly, it can be time-consuming and resource-
intensive, as it requires the collection and analysis of a large amount of data. Secondly, it can be difficult
to define the boundaries of a system and identify all the relevant components and interactions. Finally,
it can be challenging to apply the systems approach to real-world planning problems, as the complexity
and interdependencies of these problems can be difficult to capture and represent in a model.
In summary, the systems approach is a valuable tool for planning as it provides a holistic and integrated
approach to understanding and solving complex problems. However, it requires careful consideration of
the limitations and challenges involved in its implementation.
Q.18 Discuss the from of the Gravity Model commonly used for estimating trip distributing in transpor-
tation planning. Why necessary to calibrate such a model?
Ans:- The gravity model is a widely used mathematical formula in transportation planning that predicts
the flow of trips between different zones or regions. It is called the gravity model because it is based on
Newton’s law of gravity, which states that the attraction between two masses is directly proportional to
the product of their masses and inversely proportional to the square of the distance between them.
In transportation planning, the gravity model can be used to estimate the number of trips that will be
made between any two zones, based on the size (population, employment, etc.) of the two zones and the
distance between them. The general form of the gravity model is:
T_ij = k * P_ia * P_jb / D_ijc
where T_ij is the estimated number of trips between zones i and j, P_i and P_j are the size of the two
zones, D_ij is the distance between the two zones, and k, a, b, and c are coefficients that are determined
through calibration of the model.
Calibration is necessary to ensure that the coefficients in the gravity model accurately reflect the
characteristics of the region being studied. The coefficients can be calibrated using observed data on trip
flows between zones, or they can be estimated using statistical techniques. In either case, the calibrated
model can be used to make predictions about trip flows for different scenarios, such as changes in land
use, transportation infrastructure, or population size.
Hemant Vilas Parulekar 793

The gravity model is useful in transportation planning because it provides a relatively simple and intuitive
way to estimate trip flows between different zones. It also allows planners to explore the potential impacts
of different policy scenarios on trip flows and to identify areas where transportation investments may
be needed to accommodate future growth. However, it is important to recognize that the gravity model
is a simplification of a complex system, and its predictions may not always be accurate, particularly in
situations where there are significant changes to the underlying characteristics of the system.
Q.19 Aerial photograph on scale 1:25000 was taken with an aerial camera lens of 15 cm focal length,
Calculate the flying height above mean ground level of 1200 m. Find the flying height of the aircraft
above mean sea level.
Ans: - To calculate the flying height above mean ground level, we can use the following formula:
f = (H × s) / (GSD × f)
where: f = flying height above mean ground level H = flying height above mean sea level s = scale
(1:25000) GSD = ground sampling distance (which can be calculated as f / f ’) f ’ = focal length of the
camera (in millimeters)
We are given s = 1:25000 and f ’ = 150 mm. We can convert the flying height above mean ground level
to meters, and we can assume a ground sampling distance of 0.3 meters (which is typical for aerial
photography). Substituting these values, we get:
0.15 = (1200 × 25000) / (0.3 × 150)
Solving for the flying height above mean ground level, we get:
f = 638.3 meters
To calculate the flying height of the aircraft above mean sea level, we can simply add the flying height
above mean ground level to the mean ground level. If the mean ground level is not given, we cannot
determine the flying height above mean sea level.
Q.20 Explain break point chlorination.
Ans: Break point chlorination is a process of water disinfection that involves adding enough chlorine
to water to reach a concentration where it can destroy all the organic matter and bacteria present in the
water. The amount of chlorine required to achieve this level of disinfection is known as the “break point.”
The process of break point chlorination involves adding chlorine to water in a series of steps until the
break point is reached. Initially, small amounts of chlorine are added to the water, and the free available
chlorine (FAC) concentration is measured at each step. As more chlorine is added, the FAC concentration
gradually increases until it reaches a maximum, indicating that all the organic matter and bacteria have
been oxidized.
Once the break point is reached, any additional chlorine added to the water will react with the previously
oxidized matter, forming chloramines or other byproducts. These byproducts can be harmful if they are
consumed, so the water must be dechlorinated or treated with other chemicals to remove them.
Break point chlorination is commonly used in water treatment facilities, particularly for treating water
that is highly contaminated or for removing bacteria and viruses that may be resistant to lower levels of
chlorine. It is an effective method of disinfection, but it requires careful monitoring and control to ensure
that the break point is achieved without over-chlorinating the water.
794 GATE Architecture and Planning: Comprehensive Question Bank

Q.21 Comment on the PPBS system ans its applicability in Indian context.
Ans: PPBS, or Program Planning and Budgeting System, is a management tool used by government
organizations to plan, organize, and evaluate their programs and budgets. It is a decision-making process
that emphasizes long-range planning, program evaluation, and the consideration of alternative courses
of action.
PPBS was developed in the United States during the 1960s and 1970s and was used by various federal
agencies to allocate resources and assess the effectiveness of their programs. It is often associated with
rational-comprehensive planning and assumes that decisions can be made through the rational analysis
of information.
In the Indian context, PPBS was introduced in the early 1970s as part of the government’s efforts to
improve the efficiency and effectiveness of public programs. However, its adoption was limited, and it
has not been widely implemented across all government agencies.
One of the challenges in implementing PPBS in India is the lack of reliable data and the absence of a
well-established system for collecting and analyzing such data. Additionally, there is often a mismatch
between the objectives of government programs and the actual outcomes they achieve.
Despite these challenges, there have been some efforts to adopt PPBS in India, particularly in the central
government and some state governments. The use of PPBS in India can be useful in ensuring that public
programs are efficiently and effectively managed, and that resources are allocated in a way that reflects
the priorities of the government. However, its success will depend on the availability of reliable data, a
commitment to long-term planning, and the ability to evaluate and learn from program outcomes.
GATE QUESTION PAPER 1994
General Aptitude (GA)

Section - A
Q.1 (i) The visual principle of optical correction was invented and used in
(A) Byzantine Architecture (B) Islamic Architecture
(C) Greco Roman Architecture (D) Greek Architecture
Ans :- (D)
Explanation: Please refer GATE-2008, Q.8 for more information
(ii) Interface of two ecological zone is called
Ecosystem (B) Ecotone (C ) Profile (D) Promontories
Ans:- (B)
Explanation:
Term Description
Ecosystem An ecosystem is a community of living organisms in conjunction with the non
living components of their environment interacting as a system
Ecotone -A transition between two biomes
-the communities meet and integrate
-it may be narrow or wide, local or regional
An ecological It is the more comprehensive replacement of the usual socio economic profile
profile (EP) which gives
Promontories A high point of land or rock projecting into the sea or other beyond the line of
coast.
(iii) Greater London Plan ,1941, was prepared by
(A) Christopher Wren (B) Ebenezer Howard
(C) Prince Charles (D) Patrick Abercrombie
Ans :- (D)
Explanation:- The two planners connected to with planning of London are: Sir Christopher Wren and
Patrick Abercrombie.
Sir Christopher Wren (1632-1723) was astronomer, designer and a great architect. Wren designed over
50 London churches, including St. Paul’s Cathedral, as well as many secular buildings. He worked out a
plan for rebuilding the city after great fire of 1666.
Patrick Abercrombie- The Greater London plan of 1944 was designed by Sir Patrick Abercrombie
(1879-1957). The plan was related to the County of London plan written by Abercrombie in 1943. The
Abercrombie’s ideas for greater London were for the zoning principles and grand plans to decongest city
by new satellite towns to be built beyond the Green Belt.
(iv) Density control of residential area is expressed in terms of
(A) Dwelling units per floor (B) Dwelling units per plot
(C) Age -sex ratio (D) Floor area ratio
796 GATE Architecture and Planning: Comprehensive Question Bank

Ans:- (D)
Explanation: FAR and ground coverage ratio of any specific area in the city decides the typology of
buildings in it (low-rise, mid rise and high-rise. The vertical limitations of the buildings decide the
population that can be accumulated in that area.
(v) Write (17,*) A is a write statement with
(A) Free format (B) Open format (C) F 17.0 format (D) A 17 format
Ans:- (A)
Explanation:
(A) Free format: The * symbol in the statement indicates that the output format is free format. This
means that the output will be formatted automatically based on the data being output.
(B) Open format: There is no such thing as an open format in Fortran.
(C) F 17.0 format: This format specifies that the output should be formatted with a total width of 17
characters and 0 decimal places. If the number being output has more than 17 characters, it will be
truncated. If it has less than 17 characters, it will be right-justified in the output field.
(D) A 17 format: This format is used to output character strings. It specifies that the output should be
formatted with a total width of 17 characters, left-justified in the output field, and padded with spaces
if the string being output has fewer than 17 characters.
(vi) Grey value of colour refers to
(A) Wave length (B) Reflectivity
(C) Lightness (D) None of the above
Ans :- (D)
Explanation: Please refer GATE-2015, Q.19 for more information.
(vii) Coarse textured plants are characterised by
(A) Dense small leaves with many branches and full growth habits
(B) Lerge leaves with massive branches and loose growth habits
(C) Sharp edged long leaves with thin branches and restricted growth habits
(D) Uncontrolled growth with sparsely developed leaves and trunk
Ans:- (C)
(viii) In a two dimensional composition spatial depth can be created by the principal of
(A) Balance (B) Rhythm (C ) Perspective (D) Harmony
Ans:- (C)
Explanation: Please refer GATE-1997, Q.24 for more information.
(ix) Settlement with ‘mile high’ structure was conceived by
(A) Le Corbusier (B) Antonio Gaudi
(C) Frank Lloyd Wright (D) Kevin Lynch
Ans :- (C)
Explanation: The concept of a “mile high” structure, which refers to a building that is one mile (or 1,600
meters) tall, was first proposed by Frank Lloyd Wright in 1956. Wright’s design, known as “The Illinois,”
was a futuristic skyscraper that was intended to be built in Chicago. However, the technology and ma-
terials required to construct such a tall building did not yet exist at the time, and the project was never
realized.
Hemant Vilas Parulekar 797

(x) The premissible height of a building on a plot is determined by


(A) Density of the area and floor space index
(B) Uncovered rear space and ground coverage
(C )Abutting road width and floor area ratio
(D) None of the above
Ans:- (C)
(xi) AUTOEXE . BAT is a
(A) Data file (B) ASCII text file
(C) Autocad drawing file (D) Write protected
Ans:- (B)
Explanation: AUTOEXEC.BAT is a system file that was originally on DOS-type operating systems. It
is a batch file that contains a series of commands to be executed by the COMMAND.COM shell upon
startup of the system.
- It is an ASCII text file, meaning it can be edited with a text editor. This file is used for setting up an
environment by running commands and setting system variables that are needed to properly run the
operating system and various applications.
- It is not a data file, Autocad drawing file, or inherently write-protected, although its importance for
system configuration meant it was usually treated with care to avoid unwanted system behavior.
(xii) Photo chemical reaction in the atmosphere begins with
(A) The act of absorption of radiation (B) The formation of smog
(C) the depletion of ozone layer (D) None of the above
Ans:- (A)
Explanation: The photo chemical reaction in the atmosphere begins with the absorption of radiation.
It does not begin with the formation of smog or the depletion of the ozone layer.
(xiii) Trickling filer is used for
(A) Treatment of drinking water (B) Treatment of waste water
(C ) Oxidation of water (D) Air conditioning plant
Ans:- (B)
Explanation: Trickling filters are used for the treatment
of wastewater. They are aerobic treatment systems that
utilize microorganisms to remove organic matter from
wastewater. Trickling filters can also achieve biochemi-
cal oxygen demand (BOD) and solids removal as well as
nitrification, the biological oxidation of ammonia to ni-
trite and nitrate. They are not used for the treatment of
drinking water or oxidation of water, nor are they used
in air conditioning plants.
(xiv) Complementary colour of violet is
(A) Red (B) Yellow (C) black (D) Green
Ans:- (B)
(xv) The Radburn pattern of neighbourhood layout was conceived by
(A) Doxiadis (B) Clarence Stein (C) Clarence Perry (D) Soriya Y. Mata
798 GATE Architecture and Planning: Comprehensive Question Bank

Ans:- (B)
Explanation: The Radburn pattern of neighbourhood layout was conceived by (B) Clarence Stein. This
system of urban planning was based on the idea of creating a neighbourhood with the streets forming
a grid-like pattern and the homes being set back from the streets. This system was meant to provide a
more pedestrian-friendly environment, while also providing a sense of community and safety. It also
aimed to increase green spaces and recreational areas, as well as to reduce traffic congestion. The Rad-
burn pattern was also designed to encourage more interaction between neighbors, as well as to provide
an efficient system of transportation. It has since become one of the most widely used urban planning
approaches in the world.
(xvi) UCS in Autocad
(A) Cannot be altered (B) Can only be rotated
(C) Can only be translated (D) Can be rotated and translated
Ans:- (D)
Explanation: UCS in AutoCAD stands for User Coordinate System, which is a 3D Cartesian coordinate
system that allows you to define custom coordinate systems to represent objects in a drawing.
The UCS can be changed or altered to align with a specific plane or object in the drawing. It can be ro-
tated, translated, or redefined to any location in 3D space using the UCS command or the UCS icon. So
the correct option is (D) “Can be rotated and translated”.
(xvii) The principal of ‘Conservative Surgery’ was suggested by
(A) Oscar Neimeyer (B) Patrick Geddes
(C) Charles Abraham (D) Lewis Mumford
Ans:- (B)
Explanation: The principle of “Conservative Surgery” is often attributed to Patrick Geddes, who was a
Scottish biologist, sociologist, and town planner. He believed that the conservation and adaptive reuse
of existing buildings and urban fabric were preferable to demolition and new construction. Geddes was
an early proponent of the idea of sustainability and considered the environmental impact of human ac-
tivities on cities and the countryside.
(xviii) Plants which can block wind and view throughout the year when branching very near the
ground are known as
(A) Decidous (B) Evergreen Conifers
(C) Broad leaf evergreen (D) Mixed evergreen decidous
Ans:- (C)
(xix) The Tien An Mein Square in Peking is an example of
(A) Ornamental park (B) Shopping centre
(C) Large Plaza (D) Exhibition ground
Ans:-(B)
Hemant Vilas Parulekar 799

(xx) The National Commission on Urbanisation was chaired by


(A) Rajiv Gandhi (B) Charles Correa
(C) B.V.Doshi (D) A.P.Kanvinde
Ans:- (B)
Q.2 What do the following abbreviation stand for
(a) BTU : British Thermal Unit
(b) HVAC : Heating Ventillation and Air Conditioning
(c) SFS : Self Financing Scheme
(d) COPP :
(e) ASCORAL :Assemblee de CO nstructeurs pour une Renvovation Architecturale
(f) NCR : National Capital Region
(g) RAM : Random Access Memory
(h) BPS : Business Process Services
(i) BOPE :
(j) NGO :Non-Government Organization

Q.3 Match the following


(i)
(a) Mud Pushta (A) DOS
(b) Ballast (B) Relationship
(c) Fineness (C) Colour Separator
(d) Wblock (D) Water proofing
(e) Copy (E) Order
(f) Print (F) Accentuation
(g) Proportion (G) Fluorescent
(h) Schemata (H) Aggregate
(i) Vista (I) Autocad
(j) Newton (J) BASIC
Ans:- a - (D), b -(G), c - (H) , d -(I) , e - (J) , f -(A) , g -(E) , h -(B) i -(F) , j -(C).
(ii)
(a) Laurie Baker (A) Site Planning
(b) Christopher (B) World cities
(c) Eugen P. Odum (C) New Bombay
(d) Kevin Lynch (D) sector Theory
(e) Peter Hall (E) Small is Beautiful
(f) Lewis Mumford (F) Arcology
(g) E.S. Schumacher (G) Architect of the poor
(h) Homer Hoytt (H) A city is not a Tree
(i) Soleri Paolo (I) Fundamentals of Ecology
(j) Charles Correa (J) The city in History
800 GATE Architecture and Planning: Comprehensive Question Bank

Ans:- a - (G), b -(B), c - (I) , d -(A) , e - (J) , f -(H) , g -(E) , h -(D) i -(F) , j -(C).
Q.4 Fill in the blanks:
(i) Physiological comfort is a major issue in planting design.
(ii) ‘Dwelling’ 67 Montreal’ is an example of Sports complex.
(iii) The first garden city Letchworth is 35 km away from London.
(iv) OPEN (15, file ‘ C 10 OUT’ )
(v) Two parallel lines in perspective meet at vanishing point.
(vi) Chlorine is added to water for removal of pathogens.
(vii) Exhaust Fan are provided to remove foul air inside the toilet.
(viii) Asymmetrical space can be made to feel more dynamic when pure element is placed
Symmetrical in the space.
(ix) Brasilia is a variant of urban form.
(x) A complete periodic enumeration of total population in a given country made at a specific
point of time is known as Census.

Q.5 Indicate the following as TRUS or FALSE.


(i) Unity in composition can also be increased by proximity. (True)
(ii) The Milletus demonstrates Hippodomian plan. (False)
(iii) Agora was locating the Royal palace. (False)
(iv) Harappa is situated by the side of river Sind. (True)
(v) The central city with satellite towns was originated by Raymond Unwin. (True)
(vi) The Aranya Housing Project was designed by B.V. Doshi. (True)
(vii) The intensity of commercial development is expressed in terms of F.A.R. (True)
(viii) Sodium vapour lamps are suitable for industrial lighting. (True)
(ix) In an auditorium if the near ground temperature is higher than the upper level
The sound from source will deflect upwards. (False)
(x) FORTRAN is a machine language. (False)
(xi) Hogarth’s Line of Beauty represents variety in unity. (True)
(xii) DOS is an operating system software. (True)
(xiii) Landscape is to be seen as a physical entity only and not to be translated into economic
Unit. (False)
(xiv) Human scale refers to space with a maximum horizontal distance of 72 feet. (False)

Q.6 Write a computer program for drawing a specific line and a specific circle on the terminal screen in
BASIC
Hemant Vilas Parulekar 801

Ans: -
SCREEN 9 ‘set graphics mode
‘ Draw a line
CLS
LINE (100, 100)-(200, 200), 12, B
‘ Draw a circle
CIRCLE (300, 200), 50, 14, , , B
END
In this example, SCREEN 9 sets the graphics mode for the terminal screen, and CLS clears the screen.
LINE is a command that draws a straight line between two points. In this case, it draws a line from the
point (100, 100) to the point (200, 200) using color number 12 and a box style (B).
CIRCLE is a command that draws a circle. In this case, it draws a circle centered at point (300, 200) with
a radius of 50, using color number 14 and a box style (B).
You can modify the coordinates and colors in the program to draw a different line and circle if desired.
Q.7 In a square frame of 6 cm × 6 cm make balanced composition using two straight lines in each square
so that in first a point at the centre is emphasized, in second a point near left upper corner is empha-
sized and in the third a point outside the point is emphasized.
Ans: - To make a balanced composition using two straight lines in each square of a 6 cm x 6 cm square
frame, emphasizing different points in each square, you can follow these steps:
1. In the first square, draw two straight lines that intersect at the center of the square.
2. In the second square, draw two straight lines that intersect near the left upper corner of the square.
3. In the third square, draw two straight lines that intersect outside the square, with the intersection
point located to the right of the center of the square.
Q.8 Discuss with sketch the principles of residence sector plan of new town.
Ans: - The concept of a residence sector plan in a new town is based on the idea of creating a balanced
community with a mixture of housing types, community facilities, and open space. The residence sector
plan is a planning tool that helps to achieve this balance by organizing housing and other uses into
distinct neighborhoods, each with its own character, amenities, and identity. Here are some principles
of a residence sector plan for a new town:

1. Mixed-use development: The residence sector plan should promote mixed-use development to
reduce the need for residents to travel long distances to work, shop, and socialize. Mixed-use
development also creates a sense of community by allowing residents to interact with one another
in a variety of settings.
2. Clustered housing: Housing should be clustered in small groups to create a sense of community and
to minimize the impact of development on the surrounding landscape. Clustering also provides
opportunities for shared open space, such as parks and playgrounds.
3. Hierarchy of streets: The residence sector plan should establish a hierarchy of streets, with main
roads connecting to regional centers and local streets providing access to individual homes. This
hierarchy creates a sense of order and makes it easier for residents to navigate the community.
802 GATE Architecture and Planning: Comprehensive Question Bank

4. Open space: The residence sector plan should include a variety of open spaces, including parks,
playgrounds, and greenbelts. These spaces provide opportunities for recreation, socialization, and
relaxation, and they also help to reduce the heat island effect and improve air quality.
5. Accessibility: The residence sector plan should prioritize accessibility for all residents, including
those with disabilities. This means designing sidewalks, intersections, and other elements of the
built environment to be easily navigable by people of all abilities.
6. Sustainable design: The residence sector plan should prioritize sustainable design, incorporating
features like green roofs, rain gardens, and passive solar design. These features reduce the ecological
footprint of the community, save energy and water, and create healthier living environments.
Q.9 Explain briefly the relation of climatic element to comfort .
Ans: - Climatic elements such as temperature, humidity, air movement, and radiant heat are important
factors in determining human comfort in a given environment.
Temperature is perhaps the most obvious and critical climatic element for comfort. Humans have a
narrow range of thermal comfort, typically between 18°C to 27°C, and outside of this range, we may
feel too hot or too cold. Temperature influences comfort through the heat exchange between the human
body and the surrounding air.
Humidity also plays a critical role in human comfort. High humidity can make the air feel clammy and
uncomfortable, while low humidity can cause dryness of the skin and respiratory passages. Optimum
relative humidity for human comfort is typically in the range of 30% to 60%.
Air movement, or wind speed, affects the rate of heat transfer from the human body to the surrounding
air. Higher wind speeds tend to enhance heat loss from the body, while lower wind speeds tend to inhibit
it. Moderate air movement is often desirable for human comfort, although too much or too little can be
problematic.
Radiant heat exchange is the exchange of heat between the human body and the surrounding surfaces.
Radiant temperature is especially important for thermal comfort in spaces with large window areas,
high ceilings, and thermal bridges that affect surface temperature. The body perceives the temperature
of the surrounding surfaces, which can significantly affect comfort.
In summary, a comfortable indoor environment requires the careful balance of various climatic
elements, including temperature, humidity, air movement, and radiant heat. Architects and engineers
must carefully design building envelopes, heating, ventilation, and air conditioning (HVAC) systems,
and other building elements to ensure that indoor environments are comfortable for human occupants.
Q.10 Plant form is a combination of overall plant shape and habit of growth. Each shape has its own
unique characteristics and design potentials- Sketch the different types of plant forms stating their
design potentials.

SECTION - B (PART - I )
Q.11 Critically examine the architecture works developed by Laurie Baker and its influence in modern
architecture in India.
Ans:- Laurie Baker (1917-2007) was an influential British-born architect who lived and worked in India
for most of his life. Baker is well-known for his sustainable architecture and use of innovative building
techniques that combine traditional and modern materials. His work has had a significant impact on
modern architecture in India, particularly in the areas of low-cost and eco-friendly housing.
Hemant Vilas Parulekar 803

Baker’s architecture was heavily influenced by his experiences in India and his interest in the local culture
and traditions. He believed that architecture should be based on the needs of the people who live in the
buildings and that it should be environmentally sustainable. To achieve these goals, Baker used locally
sourced materials and techniques that were appropriate for the local climate and context.
One of Baker’s most significant contributions to architecture was his use of stabilized mud blocks (SMBs)
as a building material. SMBs are made by mixing mud, cement, and other materials, then compressing
the mixture into blocks that can be used to build walls. This technique is not only environmentally
sustainable but also helps to regulate the temperature inside buildings, making them more comfortable
for the occupants.
Baker’s architecture was also characterized by its simplicity and functionality. He believed that buildings
should be designed to meet the needs of the people who use them, rather than for aesthetic or symbolic
purposes. His buildings were often designed to be flexible and adaptable, so they could be easily modified
to meet changing needs.
Baker’s influence can be seen in the work of many contemporary Indian architects, particularly those
who are focused on sustainable and eco-friendly design. His legacy is also evident in the growing interest
in using local materials and techniques in architecture, and in the focus on designing buildings that are
responsive to local contexts and cultures.
In conclusion, Laurie Baker’s work has had a significant impact on modern architecture in India. His use
of sustainable materials and techniques, his focus on functionality and adaptability, and his interest in
the needs of the people who live in the buildings have all contributed to a more sustainable and people-
focused approach to architecture.
Q.12 Explain with sketches the working of Aqua Privy.
Ans:-

Q.13 Design a cantilevered R.C. roof slab to carry a live load of 1.5 KN/m2. The overhang of the slab is 1.2
m. Use M 15 grade concrete Fe 415 grade steel. (Given R = 0.658 N/mm2 ; j =0.9 ;
Area of 8 TOR bars = 0.5 cm2).
Ans:- To design a cantilevered R.C. roof slab, we need to determine the following:
1. Thickness of the slab (h)
2. Effective depth (d)
3. Moment of resistance (M_r)
4. Required area of steel (A_s)
5. Spacing of bars (s)
6. Shear force and bending moment diagrams
804 GATE Architecture and Planning: Comprehensive Question Bank

Given data:
Live load = 1.5 kN/m2 Overhang = 1.2 m M15 grade concrete Fe 415 grade steel R = 0.658 N/mm2 j =
0.9 Area of 8 TOR bars = 0.5 cm2
Solution:
1. Thickness of slab (h) From IS 456:2000, the minimum thickness of slab should not be less than
L/32, where L is the span of the slab. Since this is a cantilevered slab with an overhang of 1.2 m,
we can take the span as 2.4 m. Therefore, minimum thickness of slab = 2.4/32 = 0.075 m = 75
mm.
2. Effective depth (d) From the given data, j = 0.9. We can take the effective depth as d
= h - (cover + 8/2) = 75 - (20 + 4) = 51 mm.
3. Moment of resistance (M_r) M_r = (0.138 * fck * bd2)/1000000 where fck is the characteristic
compressive strength of concrete, b is the width of the slab, and d is the effective depth.
Assuming fck = 15 N/mm2, and b = 1 m (since we have not been given the width of the slab),
M_r = (0.138 * 15 * 1000 * 1000 * 1 * 512)/1000000 = 217.5 kN-m.
4. Required area of steel (A_s) A_s = (M_r * 106)/(0.87 * fy * d2) where fy is the characteristic
strength of steel.
Assuming fy = 415 N/mm2, A_s = (217.5 * 106)/(0.87 * 415 * 512) = 0.0065 m2.
We can use 8 TOR bars for this purpose, since their area is 0.5 cm2.
Area of steel provided (A_sp) = 8 * 0.5 * 10-4 = 4 * 10-3 m2.
The area of steel provided is less than the required area of steel. Therefore, we need to increase the
number of bars. Let us use 10 TOR bars.
Area of steel provided (A_sp) = 10 * 0.5 * 10-4 = 5 * 10-3 m2.
5. Spacing of bars (s) s = (1000 * A_sp)/(10 * pi * d2) where pi = 3.14.
s = (1000 * 5 * 10-3)/(10 * 3.14 * 512) = 96 mm.
We can provide 10 TOR bars at a spacing of 100 mm.
6. Shear force and bending moment diagrams Since this is a cantilevered slab with a uniform
load, the maximum shear force and bending moment occur at the fixed end (i.e. where the slab
is attached to the wall).
The maximum bending moment at the fixed end is given by Assuming a thickness of 150 mm for the
slab, the self weight of the slab can be estimated as follows:
Self weight of slab = Volume of slab x Density of concrete = 1 x 1.2 x 0.15 x 2400 = 51.84 kN/m2
Total design load on the slab = 1.5 + 0.5 (for self weight) = 2 kN/m2
The effective span of the slab is 2.4 m (1.2 m overhang + 1.2 m main span). The design moment due to
live load can be calculated using the following formula:
M = (wL2)/8 = (2 x 2.42)/8 = 1.44 kNm/m
Using a depth of 230 mm for the slab, the required area of steel can be calculated as follows:
Mu = 0.87fy Ast(d - 0.42x) where, Mu = 1.44 kNm/m d = 230 mm fy = 415 N/mm2 R = 0.658 N/mm2 j
= 0.9 Ast = (Mu/(0.87fy(d-0.42x))) x 106 x = Ast/(bxd) b = 1000 mm
Hemant Vilas Parulekar 805

Substituting the values, we get:


Ast = (1.44 x 106)/(0.87 x 415 x (0.23 - 0.42 x 0.0095)) = 676.8 mm2 x = 676.8/(1000 x 230) = 0.0294
The nearest value of x can be taken as 0.03, and the corresponding Ast value can be calculated as:
Ast = (0.87 x 415 x 0.03 x 230)/0.658 = 281.73 mm2
The number of bars required is:
No. of bars = Ast/(0.5 x 10) = 5.63
Hence, 6 bars of 8 mm diameter can be provided.
Q.14 The average completion time of the following construction activities are given below. As a construc-
tion manager for which activities, you would like to maintain extremely strict Schedule? Draw the
network to justify your decision.
Activity Average completion time (in weeks)
1 -2 11
1 -3 14
2 -4 6
2 -5 16
3 -4 7
4-5 3
Ans:- To determine which activities need to be closely monitored and maintained on a strict schedule,
we need to look at the critical path of the network. The critical path is the longest path through the
network, and any delay in any activity on the critical path will result in a delay of the overall project. To
draw the network, we can use a Gantt chart or a network diagram. Here is the network diagram for the
given activities:
1 ----(11)----- 2 ----(6)----- 4 ----(3)----- 5
\ | | /
(14) (16) (7) /
\ | | /
\ (X) (X) /
\ | | /
\ 3 ----(X)------ /
\ |
\ (X)
\
\
(X)
In this diagram, the nodes represent the start and end points of each activity, and the edges represent the
time it takes to complete each activity. To find the critical path, we can calculate the earliest and latest
start and finish times for each activity. The earliest start time is the earliest time at which an activity can
start given its predecessors, and the latest finish time is the latest time at which an activity must finish
to avoid delaying the project. The critical path is the longest path through the network that consists of
activities with zero slack (i.e., the latest finish time minus the earliest finish time is zero). Using this
approach, we can calculate the critical path for this network as follows:
1 ----(11)----- 2 ----(6)----- 4 ----(3)----- 5

(X) /
| /
3 ----(X)-------/
806 GATE Architecture and Planning: Comprehensive Question Bank

In this diagram, the critical path is highlighted in red, and the activities on the critical path are marked
with an X. The critical path consists of activities 1-2-4-5, with a total duration of 20 weeks.
Therefore, to maintain the schedule of the project, we should maintain extremely strict schedule for the
activities on the critical path, i.e., activities 1-2, 2-4, and 4-5. Any delay in these activities will result in a
delay of the overall project. Activities 1-3, 2-5, and 3-4 have some slack, which means that they can be
delayed to some extent without delaying the project.
Q.15 Mention briefly the salient architectural concepts and features of Khajuraho group of temples.
Ans: The Khajuraho group of temples is a collection of Hindu and Jain temples located in Khajuraho,
in the Indian state of Madhya Pradesh. Built between the 9th and 11th centuries AD, the temples are
known for their intricate and exquisite carvings depicting a range of subjects, including gods and god-
desses, mythical creatures, animals, and humans engaged in various activities.
The architecture of the Khajuraho temples is characterized by several features and concepts:
1. Temple Design: The temples are designed in a square or rectangular plan, with a central shrine,
surrounded by smaller shrines and mandapas (pillared halls).
2. Shikhara: The temples have a shikhara (tower) at the top, which rises above the central shrine.
The shikhara is often intricately carved and decorated with miniature shikharas and sculptures.
3. Carvings: The temples are renowned for their exquisite carvings, which are done in stone. The
carvings depict a range of subjects, including gods and goddesses, mythical creatures, animals,
and humans engaged in various activities. The carvings are highly detailed and intricate, and are
often considered to be among the finest examples of Indian art.
4. Erotic Sculptures: One of the most unique features of the Khajuraho temples is the presence
of erotic sculptures on the temple walls. These sculptures depict a range of sexual acts and are
believed to be symbolic representations of the union between the male and female principles in
the universe.
5. Symbolism: The sculptures and carvings in the temples are laden with symbolic meaning and are
believed to represent the various aspects of Hindu mythology and philosophy.
Overall, the Khajuraho temples are known for their intricate carvings, exquisite design, and profound
symbolism, and are among the finest examples of Indian temple architecture.
Q.16 Find out the quantity of each type of materials required for a 12 cm thick R.C.C. (1:2:4) roof slab
3.5 m × 5.0 m with 0.8 % steel (wt of steel = 7850 kg /m3, I cum 1:2:4) concrete requires 6.2 bag of
cement, 434 lit sand and 868 lit of stone aggregate).
Ans: - To calculate the quantity of materials required, we need to first calculate the volume of concrete
required for the slab:
Volume of concrete = Area x Thickness = 3.5 m x 5.0 m x 0.12 m = 2.1 cubic meters
Now we can calculate the quantity of materials required:
Cement: 1 cubic meter of concrete requires 6.2 bags of cement, so the quantity of cement required for
2.1 cubic meters of concrete is: 6.2 bags/cubic meter x 2.1 cubic meters = 13.02 bags Assuming each bag
of cement is 50 kg, the total weight of cement required is: 13.02 bags x 50 kg/bag = 651 kg
Hemant Vilas Parulekar 807

Sand: 1 cubic meter of concrete requires 434 liters of sand, so the quantity of sand required for 2.1 cubic
meters of concrete is: 434 liters/cubic meter x 2.1 cubic meters = 912.54 liters Converting liters to cubic
meters: 912.54 liters = 0.91254 cubic meters
Stone aggregate: 1 cubic meter of concrete requires 868 liters of stone aggregate, so the quantity of stone
aggregate required for 2.1 cubic meters of concrete is: 868 liters/cubic meter x 2.1 cubic meters = 1820.28
liters Converting liters to cubic meters: 1820.28 liters = 1.82028 cubic meters
Steel: The weight of steel required is given as 0.8% of the weight of concrete. The weight of 1 cubic meter
of 1:2:4 concrete is 2400 kg/m3 (assuming a specific gravity of 2.4), so the weight of steel required is:
0.8% x 2400 kg/m3 x 2.1 cubic meters = 40.32 kg
Therefore, the quantities of materials required for a 12 cm thick R.C.C. (1:2:4) roof slab 3.5 m × 5.0 m
with 0.8% steel are:
• Cement: 651 kg
• Sand: 0.91254 cubic meters
• Stone aggregate: 1.82028 cubic meters
• Steel: 40.32 kg
Q.17 Define Glare and indicate the causes of glare with it remedy:
Ans: - Glare is a visual sensation that occurs when there is a significant difference in brightness between
the visual task and the surrounding area. It can cause visual discomfort, temporary or permanent loss
of visual function, and even accidents. Glare can be categorized into two types: discomfort glare and
disability glare.
Discomfort glare is a condition where visual discomfort or annoyance is caused by luminances within
the visual field that are too high in relation to the background. Disability glare is a reduction in visibility
caused by intense light sources in the field of vision.
The causes of glare are numerous and can include both natural and artificial sources. Some common
causes of glare are:
1. Direct sunlight: Direct sunlight can cause discomfort glare, especially when it shines into a person’s
eyes.
2. Reflections: Reflections from smooth and shiny surfaces such as water, glass, or metal can cause
glare.
3. Bright light sources: Artificial sources such as streetlights, headlights, and other bright lights can
cause both discomfort and disability glare.
4. Snow: The reflection of sunlight off of snow can cause discomfort glare, especially in bright condi-
tions.
To remedy glare, there are several measures that can be taken, such as:
1. Shading: The use of shading devices such as blinds, curtains, and awnings can reduce or eliminate
glare caused by direct sunlight.
2. Light diffusing materials: The use of materials that diffuse light such as matte finishes, frosted glass,
or translucent screens can reduce glare.
808 GATE Architecture and Planning: Comprehensive Question Bank

3. Lighting design: Proper lighting design can reduce glare by controlling the intensity and direction
of light sources.
4. Polarizing filters: The use of polarizing filters can help reduce glare from reflections, especially in
sunglasses and camera lenses.
5. Proper maintenance: Proper cleaning of windows, light fixtures, and other surfaces can reduce glare
caused by dust, dirt, and grime.

Q.18 Determine the eccentric load per meter which a 250 mm thick work section can carry. The eccentric-
ity of load is 3cm from the centre line of the wall. The allowable compressive stress for brick work
(1:4 mortar) is 2000 KN/m2; slenderness ration for the all wall is 10 for which The allowable stress
is to be reduced by 40%.
Ans:- To determine the eccentric load per meter which a 250 mm thick work section can carry, we
need to use the following formula:
P = [(M x e) / I] + [(W x L) / 2]
where: P = the load per meter M = the bending moment e = the eccentricity of the load I = the mo-
ment of inertia of the section W = the weight of the wall per meter L = the length of the wall
To calculate the bending moment, we can use the following formula:
M = (P x a) / 1000
where: P = the total load on the section a = the distance between the load and the nearest support
Let’s assume that the length of the wall is 1 meter, so L = 1m.
The weight of the wall per meter can be calculated as follows:
Weight = volume x density = (0.25m x 1m x 1m) x 2000 kg/m3 (reduced by 40% due to slenderness
ratio) = 1000 kg/m (reduced weight due to slenderness ratio)
The moment of inertia of the section can be calculated as follows:
I = bd3 / 12 = 0.25m x (0.25m)3 / 12 = 0.00130208 m4
The allowable compressive stress for brickwork is 2000 KN/m2, and since the slenderness ratio is 10,
the allowable stress is reduced by 40% to 1200 KN/m2.
Now we can calculate the total load on the section as follows:
Total load = weight + (allowable stress x area) = 1000 kg/m + (1200 KN/m2 x 0.25m x 1m) = 1300
kg/m + 300 kg/m = 1600 kg/m
Next, we can calculate the bending moment:
M = (P x a) / 1000 = (1600 kg/m x 0.03m) / 1000 = 0.048 kN-m/m
Finally, we can calculate the eccentric load per meter:
P = [(M x e) / I] + [(W x L) / 2] = [(0.048 kN-m/m x 0.03m) / 0.00130208 m4] + [(1000 kg/m x 1m) /
2] = 1.108 kN/m
Therefore, the eccentric load per meter that a 250 mm thick work section can carry is 1.108 kN/m.
Hemant Vilas Parulekar 809

Q.19 Sketch two alternative courses of brick layer showing ‘English Brick bond’ for a Tee Junction of a
half brick wall with a one brick thick wall .

Q.20 Outline briefly the architectural features developed during Islamic period in India.
Ans: - The Islamic period in India (beginning in the 12th century) saw the development of a unique
architectural style that combined Islamic and Indian elements. Some of the key architectural features of
this period include:
1. The use of arches and domes: Islamic architecture emphasizes the use of arches and domes, which
were often decorated with intricate geometric and floral designs.
2. The minaret: A tall, slender tower used to call the faithful to prayer. These towers were often
decorated with intricate geometric patterns and calligraphy.
3. The mosque: Islamic places of worship, which were typically characterized by a large central
courtyard surrounded by a covered prayer hall. The prayer hall often had a series of domes
supported by arches.
4. The use of decorative elements: Islamic architecture in India often featured intricate tile work, carved
stone, and ornate calligraphy.
5. The tomb: The Islamic period in India saw the development of many impressive tombs and mau-
soleums, such as the Taj Mahal in Agra. These structures often featured large domes and intricate
geometric patterns.
6. The fort: The Mughal Empire built many impressive forts, which often featured high walls, large
gates, and elaborate defensive structures.
Overall, the architecture of the Islamic period in India is known for its intricate geometric designs, use
of decorative elements, and impressive engineering feats.
Q.21 ‘Hall of Nation’ constructed at the Permanent Exhibition complex (Pragati Maidan), new Delhi as
the forum for the demonstration of India’s industrial and technological potential - critically evalu-
ate the structure from the point of contemporary Indian architectural development .
Ans:- The Hall of Nations, built in 1972, is a significant landmark of Indian modernist architecture and
engineering. Designed by the renowned Indian architect, Raj Rewal, the building is an iconic example
of modern Indian architecture and engineering. The building has a unique design that used reinforced
concrete, steel and glass to create a massive exhibition hall that was 76 meters long, 35 meters wide, and
18 meters high.
The Hall of Nations is built as a symbol of India’s industrial and technological prowess, and it served as
a forum for exhibitions, conferences, and other events showcasing India’s achievements in the field of
science and technology. The building was widely appreciated for its unique design, and it received many
awards and accolades from both domestic and international forums.
Q.22 Out of several water softening processes for hard water which process produces zero hardness.
Explain the reasons.
810 GATE Architecture and Planning: Comprehensive Question Bank

Ans: - The process of distillation produces zero hardness in water, as it removes all the minerals, including
the ones that cause hardness, by boiling the water and condensing the resulting steam. Hard water
contains high levels of calcium and magnesium ions, which cause the hardness. During distillation,
the water is boiled, and the steam is collected and condensed, leaving behind the minerals that cause
hardness. The resulting water is free of minerals and has zero hardness. However, the distillation process
is not typically used for water softening on a large scale, as it is energy-intensive and can be expensive.
Other more practical methods of water softening include ion exchange, reverse osmosis, and lime
softening, which remove the minerals that cause hardness from the water.
Q.23 State the advantages of per stressed concrete.
Ans: - Prestressed concrete is a type of concrete that is under compression stress induced by pre-ten-
sioning or post-tensioning the reinforcing steel. The advantages of prestressed concrete include:
1. Greater strength and durability: Prestressed concrete has high strength and durability due to the
pre-compression, which results in better resistance to cracks and other forms of structural damage.
2. Longer spans and thinner sections: The use of prestressed concrete allows for longer spans and
thinner sections, which reduces the amount of materials used, leading to cost savings and aesthetic
benefits.
3. Reduced deflection: Prestressed concrete has a reduced deflection compared to non-prestressed
concrete, which results in better performance under load and fewer issues with cracking.
4. Improved crack control: The pre-compression force in prestressed concrete helps to control the for-
mation and propagation of cracks, leading to better durability and longevity of the structure.
5. Faster construction: The use of prestressed concrete allows for faster construction times, as the
pre-tensioning or post-tensioning process can be completed off-site, and the prefabricated concrete
elements can be quickly assembled on site.
6. Versatility: Prestressed concrete can be used in a wide variety of applications, including bridges,
buildings, parking garages, and other structures, making it a versatile building material.
Overall, the use of prestressed concrete has many advantages, including improved strength and du-
rability, reduced deflection, improved crack control, faster construction, and versatility in a variety of
applications.
Q.24 Sketch the details of ribbed slabs with precast panels and discuss the method of construction of such
a slab.

Q.25 Explain with sketches the method of forming pendentive for dome construction over a square plan.
Given example of a famous structure where it was adopted.

Q.26 Elaborate the considerations you will make in designing a toilet for male students Hostel. Sketch
one such toilet having 2 W.C.’s, 3 baths. 4 Urinals, 3 W.H. Basins and I Janitor’s closet.
Ans: - Designing a toilet for a male student hostel requires careful consideration of several factors to
ensure that the facility is functional, safe, and comfortable. Here are some key considerations that you
should make:
1. User needs: The toilet design should meet the specific needs of male students. For example, you
need to ensure that there are enough fixtures to accommodate peak usage times, and that the design
provides adequate privacy.
Hemant Vilas Parulekar 811

2. Space utilization: The toilet should be designed to make the most of the available space, while still
providing enough room for users to move around and use the fixtures.
3. Safety: The toilet should be designed with user safety in mind, including proper ventilation and
lighting, non-slip flooring, and easy-to-use fixtures.
4. Accessibility: The toilet should be accessible to all users, including those with disabilities. This may
include the use of grab bars, wider doorways, and other accessibility features.
5. Hygiene: The toilet should be designed to minimize the risk of infection and disease transmission.
This may include the use of touchless fixtures, proper drainage, and easy-to-clean surfaces.
Based on these considerations, a toilet for a male student hostel may include the following features:
• Two W.C.’s: These should be in separate cubicles for privacy, with sufficient space for users to move
around and access the fixtures.
• Three baths: These should be designed with privacy in mind, and should include appropriate fix-
tures, such as showers or tubs.
• Four Urinals: These should be designed for efficient use of space and may include dividers or privacy
screens between fixtures.
• Three W.H. Basins: These should be in a separate area, such as a vanity or hand-washing station, and
should include appropriate fixtures and accessories, such as mirrors and soap dispensers.
• One Janitor’s closet: This should be in a separate area, away from the fixtures, and should include
appropriate equipment and supplies for cleaning and maintenance.
Q.27 Distinguish with help of sketches the construction of dome and supporting system for St. Paul’s
Cathedral, London, and St. Peter Rome.

Q.28 Outline with sketches the principles in simplest form the different stages involved in Summer and
winter air conditioning.

Q.29 Compare with sketches the proportion of Greek and Roman Doric order and label the part.

Q.30 Sketches the longitudinal section through a water closet with S - trap sowing fittings, fixtures and
connections to soil pipe leading to septic tank.

SECTION - B (PART -I)


Q.11 Stating the concept of Primate City, outline how the primacy index could be computed
Ans: - The concept of a Primate City refers to a situation where one city is disproportionately larger than
all others in a country or region. This city dominates politically, economically, and socially, often serving
as the center for culture and administration. A primate city has a population that is several times larger
than the second-largest city in the region, with a level of dominance that is uncommonly high.
To compute the primacy index of a country, we need to first determine the population of the largest city
in the country, as well as the population of the second-largest city. The primacy index is then calculated
by dividing the population of the largest city by the population of the second-largest city.
812 GATE Architecture and Planning: Comprehensive Question Bank

The resulting value can range from 1 to infinity, with a value of 1 indicating that there is no primate city,
and higher values indicating a greater degree of primacy. The higher the value of the primacy index, the
greater the degree of dominance of the largest city in the country over the rest of the urban hierarchy.
It should be noted that the concept of primate city and the computation of the primacy index are not
without criticism, as they do not consider the differences in the economic and political power of the
largest city relative to other cities. Additionally, they do not consider the degree to which the largest city
serves as a gateway to the rest of the country or region. Nonetheless, the concept and index can provide
useful insights into the urban hierarchy and spatial organization of a country or region.
Q.12 The distance measured between two identifiable locations on a topographic map (scale1:50000) is
52 mm. The distance between the same two location in an aerial photograph Just covers a settlement
within its 200 mm×200 mm size. 45% of the area of the photograph is under residential use. Find
(a) The scale of the aerial photograph
(b) The total area of the settlement
(c) The area under residential use.
Ans: - To solve this problem, we can use the principle of similar triangles. We can set up a ratio of the
distance on the topographic map to the corresponding distance on the aerial photograph, and then
solve for the scale of the aerial photograph. We can also use this scale to determine the total area of the
settlement and the area under residential use.
(a) Scale of the aerial photograph: The distance on the topographic map is 52 mm, which corresponds
to the diagonal of the settlement in the aerial photograph. Let x be the scale of the aerial photograph.
Then, the length of the diagonal of the settlement in the photograph is:
diagonal = x * 200 mm
We can use the Pythagorean theorem to find the length of one side of the settlement:
side = diagonal / sqrt (2)
The distance on the topographic map corresponds to this length, so we can set up the following ratio:
52 mm / side = 1 / 50000
Solving for x, we get:
x = 200 * sqrt (2) * 50000 / 52 x ≈ 1:92,245
Therefore, the scale of the aerial photograph is approximately 1:92,245.
(b) Total area of the settlement: The length of one side of the settlement is:
side = diagonal / sqrt (2) = 200 mm / sqrt (2) ≈ 141.4 mm
Using the scale of the aerial photograph, we can convert this length to actual distance:
distance = side * 1 / 92245 = 1.533 km
The area of the settlement is:
area = distance2 = 2.35 km2
Therefore, the total area of the settlement is approximately 2.35 square kilometers.
(c) Area under residential use: 45% of the area of the photograph is under residential use, so the area
of the settlement under residential use is:
residential area = 0.45 * 2.35 km2 ≈ 1.0575 km2
Using the scale of the aerial photograph, we can convert this area to actual area:
residential area = 1.0575 km2 * 1,000,000 m2/km2 / 92,2452 ≈ 12,103 m2
Therefore, the area under residential use is approximately 12,103 square meters.
Hemant Vilas Parulekar 813

Q.13 Outline the considerations for design and layout of street lighting system in an urban area.
Ans: - The design and layout of a street lighting system in an urban area should consider several factors
to ensure that it is efficient, effective, and safe. Here are some key considerations:
1. Lighting levels: The lighting levels should be sufficient to provide adequate visibility for pedestrians,
cyclists, and motorists, while minimizing glare and light pollution.
2. Uniformity: The lighting should be uniform throughout the area, with no areas of excessively high
or low illumination.
3. Energy efficiency: The system should be energy-efficient, with the use of LED lighting and intelligent
controls to reduce energy consumption and costs.
4. Maintenance: The system should be designed to be easy to maintain and repair, with easy access to
the lighting fixtures and controls.
5. Safety: The lighting should be designed to ensure the safety of pedestrians and motorists, with appro-
priate placement of fixtures and light levels.
6. Aesthetics: The design should be aesthetically pleasing, complementing the surrounding architec-
ture and landscape.
7. Cost: The cost of the system should be reasonable and within the budget.
8. Environmental impact: The system should be designed to minimize its environmental impact, in-
cluding light pollution and carbon emissions.
The layout of the street lighting system should consider the location and orientation of the street, the
width of the road, the height of buildings, and the position of trees and other objects that may affect the
illumination of the area.
The placement of the lighting fixtures should also consider the type and height of the poles, the spacing
between the fixtures, and the type of lighting (e.g., roadway lighting, pedestrian lighting, decorative
lighting).
Proper planning, design, and layout of a street lighting system is critical to ensure the safety and well-be-
ing of the urban population.
Q.14 Write ten major functions of urban government as per the Twelfth Schedule of the Constitution
(Seventy Fourth Amendment Act 1992 ).
Ans: - The Twelfth Schedule of the Constitution (Seventy Fourth Amendment Act 1992) specifies
the functions of urban local bodies. The ten major functions of urban government as per the Twelfth
Schedule are:
1. Urban planning including town planning.
2. Regulation of land use and construction of buildings.
3. Planning for economic and social development.
4. Roads and bridges.
5. Water supply for domestic, industrial, and commercial purposes.
6. Public health, sanitation, and solid waste management.
7. Fire services.
8. Urban forestry, protection of the environment and promotion of ecological aspects.
9. Safeguarding the interests of weaker sections of society, including the handicapped and the mental-
ly disabled.
10. Slum improvement and up-gradation.
These functions provide a framework for urban governance and aim to ensure the well-being of urban
residents and the sustainable development of urban areas.
814 GATE Architecture and Planning: Comprehensive Question Bank

Q.15 calculate the time mean speed and space mean speed from the following observed data of spot
speeds of the vehicles

Vehicle no Spot speed


1 10 m/sec
2 15 m/sec
3 12m/sec
4 11 m/sec
5 18 m/sec
Ans:- To calculate the time mean speed and space mean speed from the observed data of spot speeds of
the vehicles, we can use the following formulas:
Time mean speed = Sum of spot speeds / Number of vehicles Space mean speed = Total distance traveled
/ Total travel time
Assuming that all vehicles traveled the same distance and the travel time is the same for all vehicles, we
can calculate the space mean speed as the same as the time mean speed.
Using the given data, we have:
Time mean speed = (10 + 15 + 12 + 11 + 18) / 5 = 13.2 m/sec
Therefore, the time mean speed of the vehicles is 13.2 m/sec.
Since we do not have information on the total distance traveled and the total travel time, we cannot
calculate the space mean speed.
Q.16 Elaborate on the concept of “Spill - over Effect”.
Ans: -The spill-over effect is a phenomenon in which the economic or social impact of a particular event
or activity extends beyond the immediate area in which it occurs. In other words, the effects of the event
“spill over” into neighboring areas, leading to both positive and negative outcomes.
For example, the establishment of a new manufacturing plant in a particular city may create jobs
and boost the local economy, but it may also lead to increased traffic, pollution, and other negative
externalities that affect nearby towns. Similarly, the construction of a new housing development in one
part of a city may increase property values and improve the quality of life for residents, but it may also
lead to overcrowding and strain on local infrastructure such as roads and public services.
The spill-over effect can have both positive and negative consequences, and it is important for
policymakers and urban planners to consider these impacts when making decisions about new projects
or activities. By considering the potential spill-over effects of a particular development or event, planners
can work to minimize negative impacts and maximize positive outcomes for the entire region.
Q.17 In a housing area of 18.7 Hectares, 1000 residential plots are to be provided. It is contemplated that
50% of the plots will be A - type .30% B - type and 20% C - type. The B - type plots will be 20% larger
in size than A -type plots and C - type plots will be 30% larger than A - type plots. 15 5 of the residen-
tial area will be under road and 25% area will be used for open space and community facilities. Find
the maximum area for each type of plots.
Ans: - Given data: Total area of the housing area = 18.7 hectares Number of residential plots to be
provided = 1000 Proportion of A-type plots = 50% Proportion of B-type plots = 30% Proportion of
C-type plots = 20% Area under roads = 15.5% Area for open space and community facilities = 25%
To find: Maximum area for each type of plot
Hemant Vilas Parulekar 815

Solution: Total area of the housing area = 18.7 hectares = 187000 sq. m
Area for roads = 15.5% of 187000 = 28985 sq. m
Area for open space and community facilities = 25% of 187000 = 46750 sq. m
Total area for residential plots = 187000 - 28985 - 46750 = 111265 sq. m
Number of A-type plots = 50% of 1000 = 500 Let the area of one A-type plot be x So, area of 500 A-type
plots = 500x
Number of B-type plots = 30% of 1000 = 300 Let the area of one A-type plot be x, then area of one B-type
plot = 1.2x So, area of 300 B-type plots = 300 * 1.2x = 360x
Number of C-type plots = 20% of 1000 = 200 Let the area of one A-type plot be x, then area of one C-type
plot = 1.3x So, area of 200 C-type plots = 200 * 1.3x = 260x
Total area for residential plots = area of A-type plots + area of B-type plots + area of C-type plots 111265
= 500x + 360x + 260x 111265 = 1120x x = 99.34 sq. m (approx)
Area of one A-type plot = x = 99.34 sq. m Area of one B-type plot = 1.2x = 119.21 sq. m Area of one
C-type plot = 1.3x = 129.14 sq. m
Hence, the maximum area for A-type plot is 99.34 sq. m, for B-type plot is 119.21 sq. m, and for C-type
plot is 129.14 sq. m.
Q.18 Find out the quantity of run - 0ff for 70 cm rainfall for an area of 100 hectares out of which 55% is
residential area, 20% is under road and rest of the area is under open space with 30% paved area.
The co - efficient of run off for residential area is 0.6 for road 0.9 for open space is 0.5 and for paved
area is 0.7
Ans: - The quantity of run-off for 70 cm rainfall can be calculated as follows:
Total area of the site = 100 hectares
Area of residential land = 55 hectares
Area of road = 20 hectares
Area of open space = 25 hectares
Paved area = 0.3 * 25 hectares = 7.5 hectares
Non-paved area = 25 - 7.5 = 17.5 hectares
Now, we can calculate the run-off for each type of land:
Residential area: Run-off = 0.6 * 55 hectares * 70 cm = 2310 cubic meters
Road: Run-off = 0.9 * 20 hectares * 70 cm = 1260 cubic meters
Open space: Run-off = 0.5 * 25 hectares * 70 cm = 875 cubic meters
Paved area: Run-off = 0.7 * 7.5 hectares * 70 cm = 367.5 cubic meters
Total run-off for the entire site: = 2310 + 1260 + 875 + 367.5 = 4812.5 cubic meters
Therefore, the quantity of run-off for 70 cm rainfall for the given area is 4812.5 cubic meters.
Q.19 Sketch a typical diversion curve based on travel time ratio and discuss the use of such a curve for
trip assignment.

Q.20 Suggest the area of urban functions where ‘Users charges’ could be used most effectively.
816 GATE Architecture and Planning: Comprehensive Question Bank

Ans: - User charges are fees or charges levied on users of public services to recover the cost of providing
those services. User charges can be an effective way to finance urban functions and services, as they
ensure that the users of the services pay for their actual usage of the service. Here are some of the areas
where user charges could be used most effectively:
1. Water supply: User charges can be used to recover the cost of providing water supply services. The
charges can be based on the quantity of water used or on a fixed fee per household.
2. Sewerage and sanitation: User charges can be levied on households and businesses to recover the cost
of providing sewerage and sanitation services.
3. Solid waste management: User charges can be used to finance solid waste management services such
as garbage collection, transportation and disposal.
4. Public transport: User charges can be levied on users of public transport services such as buses, met-
ro rail, etc. to recover the cost of providing these services.
5. Parks and open spaces: User charges can be used to finance the maintenance of parks and open spac-
es in urban areas.
6. Parking: User charges can be levied on car owners for the use of public parking spaces. The charges
can be used to finance the construction and maintenance of parking facilities.
7. Sports facilities: User charges can be levied on users of sports facilities such as stadiums, swimming
pools, etc. to recover the cost of providing these facilities.

By using user charges in these areas, urban governments can ensure that the services are financially sus-
tainable and are provided efficiently. However, it is important to ensure that user charges are affordable
for all sections of the society, especially the economically weaker sections.
Q.21 Outline the public Interest as Determinants of urban landuse.
Ans: - Public interest can be a key determinant of urban land use, as urban land use decisions can sig-
nificantly impact the quality of life, health, and economic opportunities for a city’s residents. Some of the
public interest factors that can influence urban land use decisions include:
1. Environmental Protection: Ensuring that natural resources are protected and preserved in urban
areas, such as forests, wetlands, and water bodies.
2. Social Equity: Ensuring that land use decisions do not disproportionately benefit one group of peo-
ple over another, and that there is equitable access to housing, transportation, education, and other
essential services.
3. Economic Development: Ensuring that urban land use decisions promote sustainable economic
growth and job creation, while also minimizing negative impacts on the environment.
4. Health and Safety: Ensuring that land use decisions prioritize the health and safety of residents by
minimizing exposure to pollution, toxins, and other environmental hazards.
5. Infrastructure Development: Ensuring that urban land use decisions promote efficient use of infra-
structure, such as transportation systems, water and sanitation systems, and public facilities.
6. Heritage Preservation: Ensuring that urban land use decisions preserve important historic and cul-
tural resources, such as heritage buildings, cultural landscapes, and archaeological sites.

Overall, public interest considerations play a critical role in shaping urban land use decisions, as they can
impact the well-being of residents and the sustainability of the urban environment.
Hemant Vilas Parulekar 817

Q.22 What data and information you will be collect if you are required to decide the price per unit floor
area of a proposed shopping complex in an existing residential area.
Ans: - If you are required to decide the price per unit floor area of a proposed shopping complex in an
existing residential area, you would need to collect several types of data and information, including:
1. Market research data: This would involve researching the current market conditions for retail
space in the area, including current rents for similar commercial properties in the same or similar
locations.
2. Demographic data: You would need to understand the demographics of the surrounding residential
area, such as age, income, and lifestyle, as this will help determine the types of businesses that are
most likely to be successful.
3. Traffic data: You would need to understand the foot traffic and vehicular traffic patterns in the area,
as this will help determine the visibility and accessibility of the proposed shopping complex.
4. Development costs: You would need to estimate the costs of developing the shopping complex,
including construction costs, land acquisition costs, and other related expenses.
5. Competitor analysis: You would need to analyze the competition in the area, including the types of
businesses and commercial properties nearby, to determine what types of businesses would be most
successful in the proposed shopping complex.
6. Zoning and regulatory requirements: You would need to understand the zoning and regulatory
requirements for commercial properties in the area, as well as any potential restrictions or
limitations that may impact the development of the shopping complex.
7. Rental market trends: You would need to understand the trends in the rental market for commercial
properties, including historical rental rates, vacancy rates, and other relevant metrics.

By collecting and analyzing this data and information, you can determine the appropriate price per unit
floor area for the proposed shopping complex in the existing residential area.
Q.23 Explain with sketches the different system of laying water distribution lines for a city.

Q.24 Sketches with dimension the cross section of urban arterial road with 4 -lanes, a median of 1.5 m
width and footpath on either side. The pedestrian volume during morning and evening peak hours
is 13500 pedestrians/hr in the major direction of flow. The design pedestrian flow is 75 pedestrian
per minute per meter.

Q.25 Indicate the administrative measures to be adopted under the land Acquisition Act, 1894 (steps
only).
Ans: - The following are the administrative measures that are typically adopted under the Land Acqui-
sition Act, 1894:
1. Publication of notification: The first step is the publication of a notification in the official gazette of
the state, announcing the intention of the government to acquire land.
2. Preliminary survey: After the publication of the notification, a preliminary survey is conducted to
identify the land that needs to be acquired.
3. Declaration of intention to acquire: After the preliminary survey, the government declares its in-
tention to acquire the land, specifying the purpose for which the land is required.
818 GATE Architecture and Planning: Comprehensive Question Bank

4. Hearing of objections: The next step is the hearing of objections from landowners and interested
parties. The objections are heard by a competent authority, which may be the district collector, or any
other officer appointed by the state government.
5. Award of compensation: After the hearing of objections, the competent authority awards
compensation to the landowners. The amount of compensation is determined based on the market
value of the land, as well as any other factors that may be relevant.
6. Possession of land: Once the compensation is paid, the government takes possession of the land.
The land is then used for the purpose for which it was acquired.
Q.26 Suggests the indicator of a settlement pattern Study for a Region.
Ans: - A settlement pattern study involves analyzing the distribution, size, shape, density, and arrange-
ment of human settlements within a region. Here are some possible indicators that could be used for
such a study:
1. Population density: This indicator measures the number of people living in a given area, typically
expressed as the number of people per square kilometer.
2. Building density: This indicator measures the number of buildings in a given area, typically ex-
pressed as the number of buildings per hectare.
3. Land use: This indicator looks at how land is being used within the region, including residential,
commercial, industrial, agricultural, and open space areas.
4. Distance decay: This indicator measures how the intensity of land use decreases as distance from a
central point increase.
5. Settlement size: This indicator looks at the distribution of settlement sizes within the region,
including the number and size of small, medium, and large settlements.
6. Settlement shape: This indicator measures the shape of settlements, including their compactness,
elongation, and regularity.
7. Accessibility: This indicator looks at the ease of movement between settlements, including the
quality of roads, transportation networks, and pedestrian infrastructure.
8. Functional linkages: This indicator measures the relationships between settlements and the
functions they serve, including economic, social, and political linkages.
9. Urbanization rate: This indicator measures the rate of change in settlement patterns, including the
growth or decline of settlements over time.
10. Socio-economic characteristics: This indicator looks at the demographic and economic
characteristics of settlements, including population size, income levels, education, and employment
opportunities.
Q.27 Explain with example how demographic data on age sex structure of population can be used for es-
timation of primary school students, secondary school students and labour force.
Ans: - The demographic data on age-sex structure of population can be used to estimate the number of
primary school students, secondary school students, and labor force in the following ways:
1. Primary school students: By analyzing the population pyramid of a region, we can estimate the
number of children in the age group of 6-10 years, who are likely to be enrolled in primary schools.
For example, if the population pyramid of a region shows many children in the age group of 6-10
years, it indicates a high demand for primary schools in that region.
Hemant Vilas Parulekar 819

2. Secondary school students: By analyzing the population pyramid, we can estimate the number of
children in the age group of 11-15 years, who are likely to be enrolled in secondary schools. For ex-
ample, if the population pyramid shows many children in the age group of 11-15 years, it indicates a
high demand for secondary schools in that region.
3. Labor force: By analyzing the population pyramid, we can estimate the number of people in the
age group of 15-59 years, who are likely to be part of the labor force. For example, if the population
pyramid shows many people in the age group of 20-40 years, it indicates a high number of people in
the working-age group and a potential increase in the labor force of that region.
Q.28 Mention the information to be collected for the design of sewerage system for an urban area
Ans: - The following information is typically needed for the design of a sewerage system for an urban
area:
1. Population: The estimated population of the area to be served is needed to determine the expected
wastewater flow rates.
2. Water consumption: The average and peak water consumption rates are needed to determine the
expected volume of wastewater generated.
3. Wastewater characteristics: Information on the expected strength and composition of the wastewa-
ter, including parameters such as BOD, COD, TSS, and pH, are needed to determine the appropriate
size and type of treatment facilities.
4. Topography: Information on the topography of the area, including elevation, slope, and drainage
patterns, is needed to determine the location and sizing of the main sewer lines.
5. Soil conditions: Information on the soil conditions, including soil type, permeability, and water
table, is needed to determine the appropriate types of sewer pipes and bedding materials.
6. Existing infrastructure: Information on the existing infrastructure, including the location and ca-
pacity of existing sewer lines, manholes, lift stations, and treatment plants, is needed to ensure that
the new system can be integrated with the existing system.
7. Land use: Information on the land use patterns, including residential, commercial, industrial, and
institutional areas, is needed to determine the expected wastewater flow rates and the required treat-
ment levels.
8. Environmental regulations: Information on the local and national environmental regulations and
standards is needed to ensure that the design of the sewerage system meets the required standards
for wastewater treatment and discharge.
Q.29 Discuss the advantages of one-way street system in an urban area
Ans: - The one-way street system in an urban area refers to a road network where all traffic flows in one
direction on each street. Here are some advantages of implementing a one-way street system:
1. Improved Traffic Flow: One-way streets can improve the flow of traffic by reducing the number of
cars that need to stop at traffic lights and intersections. Vehicles can move quickly and smoothly,
which helps reduce congestion and travel time.
2. Increased Safety: One-way streets can be designed with wider lanes, which can make it easier for
drivers to navigate and reduce the likelihood of accidents. They also reduce the risk of head-on col-
lisions that can occur on two-way streets.
3. Better Pedestrian Safety: One-way streets can also be designed to include wider sidewalks, bicycle
lanes, and crosswalks, which can make it safer for pedestrians and bicyclists to move around the city.
4. Reduced Noise Pollution: Since vehicles are moving in one direction, the noise levels on one-way
streets tend to be lower than on two-way streets, which can be less disruptive to residents and busi-
nesses in the area.
5. Improved Parking: One-way streets can be used to provide additional parking spaces by allowing
parking on both sides of the street. This can be an effective way to alleviate parking shortages in
dense urban areas.
6. Improved Navigation: One-way streets can be easier to navigate since drivers don’t have to make left
turns against oncoming traffic and can follow a more predictable path to their destination.
820 GATE Architecture and Planning: Comprehensive Question Bank

Q.30 Highlight on ‘Plot Reconstitution Scheme’ as a successful legal tool.


Ans: - Plot reconstitution is a legal process that enables property owners to reorganize and consolidate
their landholdings in order to make better use of the available space. The main objective of plot recon-
stitution is to create larger, more economically viable plots of land by exchanging fragmented, scattered,
or irregularly shaped parcels of land with other landowners. This process can be particularly beneficial
in urban areas where land is scarce and fragmented, and where there is a need to create larger plots for
development or infrastructure projects.

One of the key advantages of plot reconstitution is that it can reduce the number of land transactions
that are required in order to complete a development project, which can save time and money. By
consolidating landholdings, plot reconstitution can also reduce the number of property boundaries and
access points, which can improve land use efficiency and reduce the need for costly infrastructure such
as roads and utilities.

Plot reconstitution can also help to address issues related to land ownership and title disputes. By
bringing together a number of fragmented landholdings into a single, clearly defined plot, the process
can help to clarify property boundaries and ownership rights. This can be particularly beneficial in
areas where land ownership is complex, fragmented, or disputed.

Overall, plot reconstitution can be an effective legal tool for improving land use efficiency and reducing
the costs and complexity associated with land development. However, the success of plot reconstitution
schemes depends on a number of factors, including the willingness of property owners to participate,
the availability of technical expertise and resources, and the support of local government and other
stakeholders.
GATE QUESTION PAPER 1993
General Aptitude (GA)

Q.1 (i) The plan of Brasilia was done by

(A) Le Corbusier (B) Ebnezer Howard (C ) Lucio Costa (D) Patrick Geddes
Ans:- (D)

Explanation: Brasília is the capital city of Brazil, located in the cen-


tral-western part of the country. It was designed and built in the 1950s
by the Brazilian architect Lucio Costa and the Swiss-French architect Le
Corbusier, who were chosen to design the city after an international com-
petition. The city was planned around a large, curved boulevard called the
Monumental Axis, which runs from north to south through the city. It is
divided into several sectors, including a government district, a residential
district, and an entertainment district. The city is known for its modernist
architecture, which features many buildings with unconventional shapes and bold geometric forms. It is
also known for its wide, tree-lined boulevards and spacious parks, which contribute to its reputation as
one of the most livable cities in the world.
(ii) The residential area density is normally expressed in terms of

(A) Square meters per person (B) Number of habitable rooms per person

(C) Persons per Hectare (D) Dwelling units per Hectare


Ans:- (D)
(iii) One kilobyte of computer RAM is equal to

(A) 800 bytes (B) 1000 bytes


(C ) 1016 bytes (D) 1024 bytes
Ans:- (D)

Explanation: One kilobyte (KB) of computer RAM is equal to 1024 bytes. This is because 1 kilobyte is
equal to 1024 bytes according to the binary system used in computers. The unit “kilobyte” is often used
to refer to the amount of data or storage space in computers, and it is equal to 1024 bytes.
(iv) Weepholes are necessary for

(A) Wall footing (B) Structural strength


(C) Water pressure release (D) Appearance
Ans:- (C)

Explanation: Please refer GATE-2011, (Q. 20) for more information.


(v) Bhubneshwar, the capital city of Orissa was designed by :

(A) Charles Correa (B) Otto Koenigsberger (C) Le Corbusier (D) N. C. Panigrahi
822 GATE Architecture and Planning: Comprehensive Question Bank

Ans:-(B)

Explanation: Please refer GATE-2001, (Q.18) for more information.


(vi) The core house concept refers to :

(A) One of the forms of urban design (B) One of the forms of urban land use model

(C) One of the forms of low-cost housing (D)One of the forms of traffic survey
Ans:- (B)

Explanation: Core housing is a major variant of self-help technique. Introduced into the underdevel-
oped areas by UN missions. It has now become part of the housing vocabulary. It aims to provide an
organized cost effective, feasible scheme for the urban and urbanizing areas or poorer countries. This
concept has spread and on the verge of becoming an important building device in the less developed
areas.
(vii) For a complementary scheme, blue violet should be associated with

(A) Red orange (B) Yellow orange

(C) Red violet (D) None of the above


Ans: - (C)

Explanation: Please refer GATE-2015, (Q. 15) for more information.


(viii) The most suitable term for a plant which is not evergreen is

(A) Dead (B) Deciduous (C ) Creeper (D) Topiary


Ans:-(B)

Explanation: In the fields of horticulture and botany, the term deciduous means “falling off at matu-
rity” and “tending to fall off ”, related to trees and shrubs that seasonally shed leaves and petals after
flowering and to the ripe fruit.
(ix) Visual balance is a result of:

(A) Balancing of the product of size, weight, and location,

(B) Equal sizes of foreground and back

(C) Direction of movement in visual composition

(D) Use of elements and matching colours


Ans:(A)
(x) Out of the following landscapes, one that produces heat island is :

(A) Coastal area (B) Mountain range (C) Urban areas (D) Island
Ans:- (C )

Explanation: Please refer GATE-2011 (Q. 10) , for more information


(xi) The special type of earth of volcanic origin used in roman concrete is known as

(A) Tufa (B) Pozzolana (C) Travertine (D) None of the above
Hemant Vilas Parulekar 823

Ans:-(B)
Explanation: Pozollana is a type of natural cement that was used by the ancient Romans for construction.
It is made from a mixture of volcanic ash, lime, and water, and is highly resistant to erosion and
weathering. Pozzolanic cement was widely used in Roman architecture, particularly in the construction
of aqueducts, roads, and buildings, due to its strength and durability. It is still used in some modern
construction projects today, although it has largely been replaced by more common types of cement
such as Portland cement.
(xii) The crowd block of stepped pyramidal roofing over Shikharas must be :

(A) Doveled (B) Monolithic (C) Vaulted (D) Trabeated


Ans:-(B)

Explanation: Shikhara style of architecture was the most North


and South India. This style of temple architecture is character-
ized by its distinctive Shikhara, sometimes also referred to as
Nagora Style. The temples of shikhara style are crowned by an
Amalaka with a kalasha

(xiii) The crystal palace, London, was a :

(A) Cast iron frame structure

(B) (B) Concrete flat roof construction

(C) Wooden frame structure

(D) Wrought iron structure


Ans:- (A)

Explanation: The Crystal Palace was a cast-iron and plate-glass


building originally constructed in Hyde Park, London, England, to
house the Great Exhibition of 1851. It was designed by Joseph Pax-
ton, a leading architect and garden designer of the time, and was an
early example of prefabrication, with parts of the structure being
assembled in a factory and then shipped to the site for final con-
struction. The Crystal Palace was a massive success and became a symbol of Victorian innovation and
industrial progress. After the Great Exhibition, the building was relocated to Sydenham Hill in South
London, where it served as a cultural and entertainment venue until it was destroyed by a fire in 1936.
(xiv) The column free exhibition hall of Pragati Maidan, Delhi is :
(A) In-situ concrete triangular space frame

(B) Large scale mechanized plant

(C ) A contemporary version of traditional ‘chatri’

(D) A contemporary version of stone and reinforced cement concrete


824 GATE Architecture and Planning: Comprehensive Question Bank

Ans:- (A)

Explanation: The hall of Nations was designed by Ar. Raj Rewal

(xv) The vent pipe in the sewer line of a house is used :


(A) To protect the seal in the trap (B) To ventilate the bathroom
(C ) To ventilate the overhead tank (D) None of the above
Ans:- (A)
(xvi) The sound insulation will be maximum in :
(A) One and half brick thick wall
(B) Cavity wall having two leaves each of half brick thickness with 5cm cavity
(C) 20cm thick hollow concrete block wall
(D) Partition wall made of gypsum board fixed on the timber frame
Ans:- (D)
(xvii) Increased height of the room gives better ventilation due to
(A) Stacked effect (B) Wind effect (C) Effective temperature (D) Positive ventilation
Ans:- (A)
Explanation: Please refer GATE-2008, Q.37 for more information
(xviii) In the design of a mud wall, the resultant vertical and lateral forces should pass through:
(A) The inner edge of the section (B) the middle third of the section
(C) The outer edge of the section (D) the inner one third of the section
Ans:- (C)
(xix) The minimum diameter of a longitudinal reinforcement in R.C.C. column is
(A) 8 mm (B) 10 mm (C) 12 mm (D) 16 mm
Ans:- (C)
Explanation: The minimum diameter of longitudinal reinforcement in RCC column is typically spec-
ified in building codes and standards, and may vary depending on the size and type of the column. In
general, the minimum diameter of longitudinal reinforcement for RCC columns is typically between
10mm to 16mm.
For example, as per the Indian Standard code of practice for design of concrete structures (IS 456:2000),
the minimum diameter of longitudinal reinforcement for RCC columns should not be less than 12 mm,
except for columns in slender structures, which can have a minimum diameter of 10 mm.
It is important to note that the minimum diameter of reinforcement alone is not sufficient for designing
a safe and durable RCC column. The column design should be based on various factors such as loads,
dimensions, material properties, and design standards.
Hemant Vilas Parulekar 825

(xx) A critical path of a CPM network is

(A) The maximum path (B) The minimum path

(C) The optimum path (D) None of the above


Ans:- (A)

Explanation: A critical path of a CPM (Critical Path Method) network is the sequence of activities in
a project that must be completed on time in order for the project to be completed on schedule. It is the
longest path through the network, and any delay in any of the activities on the critical path will delay
the completion of the entire project. So, the correct answer to the question is (A) The maximum path.
There are several project management methods and techniques that are commonly used in addition to
the Critical Path Method (CPM). These methods help in planning, scheduling, and managing projects
effectively. Here are a few notable ones:
1. Critical Path Method (CPM): CPM is a network-based scheduling technique used for complex
projects. It helps in identifying the critical path, which is the sequence of activities that determines
the project’s overall duration.
2. Program Evaluation and Review Technique (PERT): PERT is another network-based project
management method that focuses on estimating the time required to complete each project activity.
It’s particularly useful for projects with high uncertainty.
3. Gantt Chart: A Gantt chart is a visual representation of a project schedule. It shows tasks or activi-
ties along a timeline, making it easy to see task dependencies and the overall project timeline.
4. Agile Methodology: Agile is a flexible and iterative approach to project management. It’s commonly
used in software development but can be applied to various projects. Agile focuses on delivering
small, incremental pieces of work and adapting to changing requirements.
5. Waterfall Methodology: Waterfall is a traditional project management approach where each proj-
ect phase must be completed before the next one begins. It’s a linear and structured method often
used for projects with well-defined requirements.
6. Scrum: Scrum is an Agile framework that divides a project into fixed-length iterations called sprints.
It emphasizes collaboration, adaptability, and regular feedback.
7. Kanban: Kanban is a visual management method that helps teams track and manage work. It’s often
used for continuous improvement and optimizing workflows.
8. Earned Value Management (EVM): EVM is a performance measurement technique that integrates
scope, schedule, and cost to assess a project’s progress and performance. It provides valuable insights
into project health.
9. PRINCE2: PRINCE2 (PRojects IN Controlled Environments) is a structured project management
framework with defined processes and roles. It’s widely used in the UK and internationally.
10. Lean Project Management: Lean principles aim to reduce waste, improve efficiency, and maxi-
mize value. Lean project management focuses on delivering value to the customer with minimal
resources.
826 GATE Architecture and Planning: Comprehensive Question Bank

11. Six Sigma: Six Sigma is a data-driven methodology used for process improvement and reducing
defects. While it’s more commonly associated with process improvement, it can be applied to project
management to achieve higher quality outcomes.
12. Risk Management: Risk management techniques are essential for identifying, assessing, and miti-
gating risks throughout a project’s lifecycle. Methods like risk matrices, qualitative and quantitative
risk analysis, and risk registers are often used.
13. Quality Management: Quality management methods, such as the Plan-Do-Check-Act (PDCA) cy-
cle, Total Quality Management (TQM), and various quality control tools, ensure that project deliv-
erables meet or exceed quality standards.
The choice of method depends on the nature of the project, its complexity, and the preferences of the
project management team. Often, a combination of methods and techniques is used to tailor project
management to specific needs.

Top of Form

(xxi) Indian remote sensing satellites are placed from the earth of a distance of

(A) 500-700 km. (B) 700-900 km. (C) 900-1200 km. (D) None of the above
Ans:- (B)

Explanation: When a satellite is launched into the space, it moves in a well-defined path around the
Earth, which is called the orbit of the satellite. There are three basic types of orbits in use.

1. Sun-synchronous orbits:- It is a special case of polar orbit. Like a polar orbit, the satellite travels from
the north to the south poles as the Earth turns below it. In a sun-synchronous orbit, the satellite passes
over the same part of the Earth at roughly the same local time each day. These orbits are between 700 to
800 km altitudes. These are used for satellites that need a constant amount of sunlight. Landsat satel-
lites and IRS satellites are typical examples of sun-synchronous, near-polar satellites.

2. Geo-synchronous orbits:- Geostationary or geosynchronous orbit is the one in which the time re-
quired for the satellite to cover one revolution is the same as that for the Earth to rotate once about
its polar axis. In order to achieve this orbit period, geo-synchronous orbits are generally at very high
altitude nearly 36,000 km. Geostationary or geosynchronous orbits are used for communication and
meteorological satellites.

3. Polar or Near Polar orbits:- Polar orbits are usually medium or low orbits (approximately 700-
800km) compared to the geo-synchronous orbits. Consequently, the orbit period is less, which typically
varies from 90-103 minutes. Therefore, satellites in the polar orbits make more than one revolution
around the earth in a single day.
(xxii) Property tax is calculated, based upon

(A) land value of the property (C) annual rental value of the property

(B) cost of construction of the building (D) floor area of the building
Hemant Vilas Parulekar 827

Ans:- (C)

Explanation: In India, property tax is usually calculated based on the annual rental value of the prop-
erty. The tax rate is determined by the local municipal corporation or other authorities, and it can vary
based on factors such as the property’s location, type, and age. Some states in India have also introduced
a system of self-assessment, where property owners are required to declare the details of their property
and calculate the tax liability themselves.
(xxiii) Author of comcentric Zone is

(A) Homar Hyott (C) Chauncey D. Harris (C) Earnest Burgess (D) Edward D. Ullman
Ans:- (C)

Explanation: Please Refer GATE-2007, Q.36 for more information


(xxiv) Panchayat system outlines

(A) Forms of urban government (C) Center state relationship.

(B) the three-tier rural local self-government (D) None of the above
Ans:- (B)

Explanation: Please refer GATE-2014, Q.8 for more information.


(xxv) The distribution of trips between Traffic zones can be estimated by

(A) All or nothing technique (C) Gravity model

(B) Diversion curve technique (D) Modal split


Ans:- (C)

Explanation: A model is usually used for trip distribution is that of the gravity function. In the attrac-
tion, F, between two bodies of respective masses M1 and M2 separated by a distance D, will be equal to
F=g x (M1 x M2) / D2

Where, g = constant or scaling factor which ensures that the equation is balanced in terms of the mea-
surement
(xxvi) Link-node diagram is used as a base for.

(A) Estimating trip generation (C) calculating modal split

(B) outgoing traffic (D) None of the above


Ans:- (C)

Explanation: Please refer GATE-2002, Q.32 for more information


(xxvii) In a septic tank

(A) Aerobic reaction takes place (C) chemical reaction

(B) anaerobic reaction takes place (D) atomic reaction takes place
828 GATE Architecture and Planning: Comprehensive Question Bank

Ans:- (B)

Explanation: In a septic tank, anaerobic reaction takes place. The wastewater in the septic tank is
allowed to settle and undergo bacterial decomposition in the absence of oxygen, which is referred to as
anaerobic digestion. This process helps to break down and treat the organic matter in the wastewater,
producing gases and a partially treated effluent that can then be further treated in a leach field or other
secondary treatment system.
(xxviii) The flow in water supply pipe is

(A) Pressure flow (B) Gravity flow (C) Free flow (D) None of the above
Ans:- (B)
Explanation: Water flow in a water supply pipe can be either pressure flow or gravity flow, depending
on the location and elevation of the source of the water and the destination of the flow.
In pressure flow, water is supplied from a higher elevation to a lower elevation, using pressure to maintain
a continuous flow. This is typically seen in urban areas where water is supplied by a centralized water
system.
In gravity flow, water is supplied using the force of gravity. This is typically seen in rural areas where
water is sourced from a well or a spring at a higher elevation and flows down to the household through
a pipe.
Free flow typically refers to an open channel flow of water such as in a natural stream or river.
So, the correct answer to the question is either (A) Pressure flow or (B) Gravity flow, depending on the
specific situation being referred to.
(xxix) The structure plan concept was the first adopted under the town and country planning Act
(UK) in the year

(A) 1944 (B) 1968 (C) 1974 (D) 1980


Ans:- B

Explanation: The structural plan is the important document of the planning process. Municipal draw-
ings must follow structural plans, Structural plans were first introduced by the 1968 Town and Country
Planning Act, as strategic level plans in the United Kingdom. Structural plans are the legal documents,
binding for all further planning and investment procedures at more detailed level.
(xxx) The first city improvement trust was created in India at

(A) Calcutta (B) Madras (C) Bombay (D) Kanpur


Ans Ans:- (C)

Explanation:

 In response to the Mumbai Plague epidemic of 1896, the Bombay City Improvement Trust (BIT)
was established on December 9, 1898.

 The MC and the government handed over all vacant lands to this office

 The CIT undertook a host of measures to improve sanitary and living conditions in the city.

 This trust was responsible for the planning of suburbs


GATE QUESTION PAPER 1992
General Aptitude (GA)

Q.1 (i) The delay in transfer of thermal energy from outside to inside is called
(A)Thermal conductivity (B) Insulation (C ) Radiation (D) Thermal lag
Ans:- (D)
Explanation: Thermal lag is the time delay in the transfer of thermal energy from one place to another.
It is a measure of the time required for a particular material or system to reach thermal equilibrium, or
to reach a specific temperature. This can occur in various situations, such as when heat is transferred
from a warm surface to a cooler surface, or when a building or other structure is heated or cooled by
external factors such as sunlight or ambient temperature. The concept of thermal lag is important in the
design and operation of many systems and structures, as it can affect the efficiency and comfort of these
systems.
(ii) Flying buttresses were used in
(A) Egyptian Architecture (B) Greek Architecture
(C) Gothic Architecture (D) None of the above
Ans:- (C)
Explanation: Please refer GATE-2006, (Q. 45) for more information.
(iii) PERT Analysis is based on:
(A) Optimistic time (B) Pessimistic time
(C) Most likely time (D) All the above
Ans:- (D)
Explanation: Please refer GATE-2006, (Q. 80) for more information.
(iv) ‘Shikhara’ refers to
(A) Prayer hall (B) Tower above main chamber of God
(C) Crown of Minaret (D) Hall of offerings
Ans:- (B)
Explanation: Please refer GATE-1993, (Q. 12) for more information.
(v) Guggenheim museum is in:
(A) New York (B) Copenhagen (C) London (D) Philadelphia
Ans:- (A)
Explanation: Guggenheim Museum, New York- Architect Frank Lloyd Wright.
Guggenheim museum, Bilbao- Architect Frank Gehry
Guggenheim museum, Venice-Satellite gallery of Guggenheim Museum, New York
Guggenheim museum, Abu Dhabi-Expected to open by 2025-Architect Frank Gehry
Deutsche Guggenheim Museum, Berlin-Architect Richard Gluckman- closed on 2013, started as Deut-
sche Bank KunstHalle
(vi) According to the National Building code of India the minimum clear height prescribed for
mezzanine floor is:
(A) 2.2 meters (B) 2.5 meters (C ) 2.8 meters (D) 3.0 meters
Ans:- (B)
(vii) Mottle is a defect which causes the stone to have
(A) Minute cracks containing calcium
(B) Cavities filled up with sand
(C) Spotted appearance due to chalky substance
(D) Cavities filled up with sand
Ans:- (C)
Explanation: Please refer GATE-2004, Q.80 for more information.
(viii) The study of spatial factors in face-to-face interpretation in known as
(A) Schemata (B) Personal space (C ) Proxemics (D) Territoriality
Ans:- (C)
Explanation: Proxemics is the study of the cultural, behavioural, and sociological aspects of spatial
distance between people as they interact with each other. It is a subfield of anthropology that focuses on
how people use and perceive physical space, particularly in terms of social relationships and the ways in
which people communicate non-verbally through their use of space. Proxemics can help to understand
how people use space to convey social status, power dynamics, and intimacy, and how these factors in-
fluence how people interact with each other in different settings.. E. T. Hall coined the term of proxemics
from the Latin root prox- and the suffix -emic as in “proximity” and “systemic”.
(ix) Wedge - shaped bricks or stone blocks forming an arch are called:
(A) Voussoirs (B) Cornices (C ) Springers (D) Abutments
Ans:(A)

(x) The following compound of cement contributes most to the strength of concrete:
(B) Calcium Aluminate (B) Dicalcium Silicate (C )Tricalcium Silicate (D) Gypsum
Ans:- (C )
Explanation: a) Calcium Aluminate is a compound that is formed when calcium and aluminum are
combined. It is often used in cement to improve the strength and durability of the concrete.
b) Dicalcium Silicate is a compound that is formed when calcium and silicon are combined. It is also
used in cement and is known for its ability to harden quickly and improve the strength and durability
of the concrete.
c) Tricalcium Silicate is a compound that is formed when calcium and silicon are combined in a specific
ratio. It is also used in cement and is known for its ability to improve the strength and durability of the
concrete.
d) Gypsum is a soft, white mineral that is often used in cement to regulate the setting time and improve
the workability of the concrete. It is also used to improve the strength and durability of the concrete.
Hemant Vilas Parulekar 831

(xi) The roof type suitable for minimum wind resistance is


(A) Hemispherical (B) Flat (C ) Conical with top opening (D) None of the above
Ans:- (B)
Explanation:
(xii) Optical corrections were employed predominantly in:
(A) Gothic Architecture (B) Greek Architecture
(C) Indian Architecture (D) Islamic Architecture
Ans:- (B)
Explanation: Please refer GATE-2003, (Q.8), for more information.
(xiii) Maximum length of cul -de -sac type of street is:
(A) 450 m (B) 220 m (C) 139 m (D) 80 m
Ans:- (B)
Explanation: A dead end, also known as a cul-de-sac, is a street with only one inlet/outlet. The length
of cul-de-sac generally kept in limit in order to avoid speeding on the road. The length varies accord-
ing to the total number of dwelling units served by the street. The range varies from 400 feet to 700
feet.
(xiv) Find the odd one out of the following :
(A) Extended Aeration process (B) Activated sludge process
(C) Oxidation pond (D) Oxidation ditch
Ans:-(A)
Explanation: The extended aeration process is a type of wastewater treatment process that involves us-
ing air to stimulate the growth of aerobic bacteria in a treatment plant. These bacteria consume organic
matter and nutrients present in the wastewater, converting them into biomass and other byproducts.
The extended aeration process is characterized by a long aeration period, typically 24 to 48 hours, which
allows the bacteria to fully break down the organic matter in the wastewater. The process is commonly
used for the treatment of municipal and industrial wastewater, and is known for its efficiency and low
energy consumption.
(xv) Dilwara temples are famous for :
(A) New method of dome construction (B) Intricate carving
(C) Wooden construction (D) Clear -storey windows
Ans:-(B)
Explanation: The Dilwara temples in India are famous for their intricate and detailed stone carvings.
They are considered to be some of the finest examples of Jain temple architecture.
(xvi) The minimum area of a combined bath and w.c as specified in the National building Code of
India is :
(A) 2.5 sq. Meters (B) 2.8 sq. Meters (C) 3.0 sq. Meters (D) 3.2 sq. meters
Ans:- (A)
(xvii) Cohort is
(A) A grafting technique (B) Groups of people aggregated by one or more characteristic
(C) A type of shading devices (D) Places with similar soil condition
832 GATE Architecture and Planning: Comprehensive Question Bank

Ans:- (B)
Explanation: Cohort is a term that refers to a group of people who have a common characteristic or ex-
perience within a defined period. For example, a cohort may be a group of people born in the same year,
or a group of people who went through a particular experience together, such as a study or a training
program. In research and statistics, a cohort is often used as a way to track and compare the experiences
and outcomes of different groups over time.
A cohort is a term with broad applications in various fields such as demographics, epidemiology, statis-
tics, and social sciences. Here’s a detailed explanation of the concept:
Definition:
A cohort refers to a group of individuals who share a common characteristic or experience within a
defined period. This characteristic is often their year of birth, but it can also relate to other shared ex-
periences, such as the year of entering school, the time of marriage, or exposure to a specific event or
intervention.
Applications:
Demographics: In demographic studies, a birth cohort consists of individuals born in the same year
or range of years. They are often studied to observe changes over the life course, like marriage patterns,
fertility rates, or mortality trends.
Epidemiology: In health and medical research, a cohort study is an observational study design where
subjects are followed over time to determine how certain factors (like exposure to a substance) influence
the development of a particular outcome (like a disease). Cohort studies can be prospective (following
subjects forward in time) or retrospective (looking back at past data).
Statistics and Research: Cohort analysis is used in statistics to track and analyze the behavior and per-
formance of groups over time. This method is often used in marketing to understand customer behavior
or in organizational studies to assess employee performance.
Education: In education, a cohort might refer to a group of students who start and progress through a
program together.
Characteristics:
Longitudinal Nature: Cohort studies are longitudinal, meaning they follow subjects over a period,
which can range from a few years to several decades.
Commonality of Experience: Cohort members share a defining experience or characteristic.
Variability in Size: Cohorts can vary greatly in size, from small groups to large populations.
Advantages of Cohort Studies:
Causal Inference: They are particularly useful for examining the causes of disease, as they can help es-
tablish a temporal sequence of events.
Dynamic: They can track changes within a population over time.
Detailed Data: Collecting data at various points allows for a detailed analysis of trends and patterns.
Limitations:
Time and Cost: Cohort studies can be time-consuming and expensive, especially if they follow subjects
over many years.
Attrition: Loss of participants over time can affect the validity of results.
Hemant Vilas Parulekar 833

Representativeness: The cohort at the start of the study may not remain representative of the broader
population over time.
Conclusion:
Cohorts are a powerful tool for understanding how various factors affect groups of individuals over
time. Their use in longitudinal studies provides valuable insights, especially in the fields of public health,
social sciences, and marketing. However, the design and implementation of cohort studies require care-
ful consideration of their potential limitations and biases.
(xviii) Reverberation inside a theatre depends on :
(A) Structural strength of the roof
(B) (B) Shape of the roof
(C) Volume of the hall
(D) Heat generated in the hall
Ans:- (C)
(xix) In a room in hot dry climate the minimum aggregate area of openings excluding doors will be
(A) One -fifth of the floor area
(B) (B) One -sixth of the floor area
(C) One -eighth of the floor area
(D) One -tenth of the floor area
Ans:- (B)
(xx) According to the National Commission of urbanization Report, the type of urban
Development suitable for Indian condition is
(A) Low -rise low density development
(B) Low -rise high density development
(C) High -rise low density development
(D) High -rise high density development
Ans:- (A)
Q.2 (i) Match the following:
(a) Theory and Design in the first Machine age (A) Ammos Rapoport
(b) The Oregon experiment (B) Geoffrey Scott
(c) The Dimaxion world (C) Richard Neutra
(d) Survival through Design (D) C.P.Kukreja
(e) The Architecture of Humanism (E) Nikolaus Pevsner
(f) Tomorrow: A peaceful path to social reform (F) Norma Evenson
(g) House -form and culture (G) Christopher Alexandar and other
(h) The Indian Metropolis (H) Reyner Banham
(i) Tropical Architecture (I) R.Buckminister Fuller
(j) A History of Building Type (J) Ebenezer Howard
Ans:- (a)-(H), (b)-(G),(c)-(I), (d)-(C), (e )- (B), (f)-(J), (g)-(A),(h)-(F), (i)-(D), (j)-(E’)
834 GATE Architecture and Planning: Comprehensive Question Bank

ii) Match the following:


(a) Lake shore Drive apartment (A) Frank Lloyd Wright
(b) Sydney Opera (B) Uttam C. Jain
(c) Central of Development Studies, Trivandrum (C ) Michael Graves
(d) Portland Building (D) Charles Correa
(e) U.S.Embassy Building, New Delhi (E ) Joseph Paxton
(f) Asiad Village Complex (F) JornUtzon
(g) Jodhpur University (G) Meis van der Rohe
(h) Hotel Cidade -de -Goa (H) Raj Rewal
(i) Crystal Palace (I) Edward Durrel stone
(j) Falling Water (J) Laurie Baker
Ans:- (a)-(G), (b)-(F),(c)-(J), (d)-(C), (e )- (I), (f)-(H), (g)-(A),(h)-(D), (i)-(E), (j)-(A)
(iii) Match the following:
(a) Domino (A) Patrick Geddes
(b) The Neighbourhood unit (B) Kevin Lynch
(c) Cite Industrielle (C ) Le Corbusier
(d) Connurbation (D) Clarence A. Perry
(e) Imageability (E ) Tony Garnier
Ans: (a)-(C), (b)-(D), (c)-(E), (d)-(A), (e)-(B)
(iv) Match the following:
(a) Letchworth (A) Hippodamus
(b) Vidyadhar nagar (B) H.P.Berlage
(c) Amsterdam South (C) Raymond Unwin
(d) Brasilia (D) B.V.Doshi
(e) Miletus (E ) Lucio Costa
Ans: (A)-(C), (b)-(D), (c)-(B), (d)-(E), (e)-(A)
(v) Match the following:
(a) C.I.E.diagram (A) Ventilation
(b) Globe thermometer (B) Climate Analysis
(c) Natural Bed (C ) Concrete strength
(d) Stalactite Brackets (D) Sand stone
(e) Water Cement Ratio (E ) Meeting point
(f) Comfort Scale (F) Qutub Minar
(g) Alabaster (G) Colour
(h) Mahoney Tables (H) Effective Temperature
(i) Stack Effects (I) Mean Radiant Temp
Node (J) Marble
Ans: (a)-(B), (b)-(I), (c)-(D), (d)-(E), (e)-(C), (f)-(H), (g)-(J), (h)-(G), (i)-(F)
Hemant Vilas Parulekar 835

Q.3 Indicate the following statements as TRUE or FALSE.


(i) External shading devices are the most effective means of blocking out solar heat transmission.
(ii) The Zoning ordinance provide standards for plot sizes and arrangement, utilitie And street improve-
ments.
(iii) Floor area Ratio is plot area divided by total covered area of all floors.
(iv) Luminous efficacy of fluorescent lamp is greater then that of incandescent lamp.
(v) A plywood board is weaker then a piece of unlaminated timber of the same size and
thickness.
Ans i. True
ii. True
iii. False
iv. True
v. False
Q.4 Fill in the blanks.
(a) The standard illumination level in general office is 100 lux.
(b) In land use planning land allocation between uses remain major criteria in the decision
making process.
(c) Lines of intersection of Roman cross vaults are called groin
(d) The pruning of plants into unusual shapes is known as topiary
(e ) A mixture of all colour pigments produces black
(f) Cul -de sac type of street system is primarily meant to eliminate traffic in a residential area.
(g) A ‘bit’ can take two values.
(h) Residential density indicates number of dwellings in relation to the area of land.
(i) Haldia is an example of port Town.
(j) Unit of measurement of plastering is square meter.

Q.5 What do the following abbreviations stand for ?


(a) H.D.F.C.
(b) C.I.A.M.
(c) M.I.G
(d) N.B.C.C.
(e ) C.P.U.
Ans (a) H.D.F.C. : Housing Development Financial Corporation
(b) C.I.A.M. :Congre’s International d’ Architecture Moderne
(c) M.I.G : Middle Income Group
(d) N.B.C.C. : National Building Construction Company
(e ) C.P.U. : Central Processing Unit
836 GATE Architecture and Planning: Comprehensive Question Bank

Q.6 Distinguish between :


(i) Dry Rot and wet rot
(ii) Arterial street and Collector street
(iii) Trabeated and Arcuated construction
(iv) Receding and Advancing colours
(v) Exotic plants and native plants
(vi) One way slab and two way slab
(vii) Absolute Humidity and Relative Humidity
(viii) Scale and Proportion
(ix) Raft foundation and pile foundation
(x) PERT and CPM
Ans (i) Dry Rot and wet rot: Dry rot is a type of wood decay caused by a fungus that thrives in dry conditions.
It can cause serious structural damage to buildings. Wet rot, on the other hand, occurs when wood
becomes wet and decays due to the action of fungi that thrive in damp conditions.
(ii) Arterial street and Collector street: An arterial street is a major thoroughfare in a city that is used
to carry large volumes of traffic between different parts of the city. A collector street, on the other hand,
is a smaller street that collects traffic from local roads and routes it to the arterial streets.
(iii) Trabeated and Arcuated construction: Trabeated construction refers to the use of horizontal beams
(called “trabes”) supported by vertical columns to construct a building or other structure. Arcuated
construction, on the other hand, involves the use of arches to support the weight of the structure.
(iv) Receding and Advancing colours: Receding colours are colours that appear to be farther away, such
as blues and greens. Advancing colours are colours that appear to be closer, such as reds and oranges.
(v) Exotic plants and native plants: Exotic plants are those that are not native to a particular region and
have been introduced from other parts of the world. Native plants, on the other hand, are those that are
naturally found in a particular region.
(vi) One way slab and two way slab: A one-way slab is a slab that is supported on two opposite sides
and is subjected to load in one direction. A two-way slab is a slab that is supported on all four sides and
is subjected to load in two directions.
(vii) Absolute humidity and Relative humidity: Absolute humidity is the amount of water vapor present
in the air, expressed in grams per cubic meter. Relative humidity is the ratio of the amount of water
vapor in the air to the maximum amount of water vapor that the air can hold at a given temperature,
expressed as a percentage.
(viii) Scale and Proportion: Scale refers to the size of an object in relation to its actual size, while
proportion refers to the relative sizes of different parts of an object.
(ix) Raft foundation and pile foundation: A raft foundation is a type of foundation in which a slab of
concrete is poured over a large area to support a building. A pile foundation is a type of foundation in
which long, slender columns (called piles) are driven into the ground to support the weight of a building.
(x) PERT and CPM: PERT (Program Evaluation and Review Technique) and CPM (Critical Path
Method) are project management techniques used to plan, schedule, and control the tasks in a project.
PERT is a method that uses a network diagram to represent the tasks and their dependencies, while
CPM is a method that uses a bar chart to represent the tasks and their dependencies.
Hemant Vilas Parulekar 837

Q.7 Answer the following briefly :


(i) The percentage articulation in an auditorium depends upon four factors. What are these?
(ii) What do you understand by determinate structure ? Give an example.
(iii) Name three most important criteria that guide the designing of an art gallery.
(iv) What are the factors affecting comfort conditions.
(v) Explain efflorescence on plastered brick wall.
(vi) Enumerate three advantages of linear type of city plan.
(vii) What is accent or focal colour ?
(viii) What are bsic functional zones of urban settlement.
(ix) What is meant by ‘U’ value of a construction.
Name important components of a typical Hindu temple.
Ans (i) The percentage articulation in an auditorium depends upon four factors: the shape and size of the
auditorium, the number of rows of seating, the slope of the seating, and the location of the stage.
(ii) A determinate structure is a type of structure in which the forces acting on it can be calculated
with a high degree of accuracy. An example of a determinate structure is a truss, which consists of
interconnected triangular units that transfer loads in a predictable way.
(iii) Three important criteria that guide the designing of an art gallery are: (1) the type and size of
artwork that will be displayed, (2) the flow of visitors through the space, and (3) the lighting and climate
control requirements to protect the artwork.
(iv) Factors that affect comfort conditions include temperature, humidity, air quality, noise level, and
lighting.
(v) Efflorescence on a plastered brick wall is a white or yellowish deposit of salt that appears on the
surface of the plaster. It is caused by the migration of soluble salts from the brick or mortar to the surface
of the plaster, where they are deposited as the water evaporates.
(vi) Three advantages of a linear type of city plan are: (1) it allows for efficient transportation
and communication along the main axis, (2) it allows for the development of a continuous strip of
urbanization, and (3) it can be easily expanded by adding additional blocks or districts along the axis.
(vii) Accent or focal colour is a strong, distinctive colour that is used to draw attention to a particular
feature or area in a design. It is often used to highlight important elements or create visual interest.
(viii) The basic functional zones of an urban settlement include residential, commercial, industrial, and
recreational areas.
(ix) The “U” value of a construction refers to its thermal transmittance, or the rate at which heat is
transmitted through the material. It is expressed in units of watts per square meter per degree Celsius
(W/m2K).
(x) Important components of a typical Hindu temple include: a sanctum sanctorum (innermost
chamber), a mandapa (hall), a tower (shikhara), and a porch (mukhamandapa). The temple may also
include smaller shrines, a gate (gopuram), and a courtyard
838 GATE Architecture and Planning: Comprehensive Question Bank

Q.8 Explain the following with sketches :


(i) Typical cross -section of a major street of approx. 40 meters right of way ,showing essential
features.
(ii) Block board and composite board.
(iii) How would you make a square shape resting on its side look vertically elongated.
(iv) The special features of Taj Mahal dome
(v) Tongue and groove joint and Dove - tail joint in wood work.

Q.9 Draw sketch plan of Chandigarh city and explain Le Corbusier’s basic concepts of town planning.
Ans. Le Corbusier was a Swiss-French architect and urban planner who is considered one of the pioneers of
modern architecture. He developed a number of basic concepts of town planning that have had a sig-
nificant influence on the development of urban design and planning. Some of these concepts include:
1. Radiant City: Le Corbusier proposed the idea of Radiant City, which was a utopian city plan that
aimed to create an ideal living environment for people. This city was based on the principles of
modernism and aimed to provide residents with a comfortable and efficient living environment.
2. Vertical Gardens: Le Corbusier believed in the concept of “rooftop gardens”, where buildings
would have green spaces on their roofs. This would help to create a more environmentally sustain-
able urban environment and would also provide residents with access to green spaces.
3. Tower in the Park: Le Corbusier believed in the separation of different functions in a city, such as
residential, commercial and industrial functions. He proposed the idea of tower blocks set within
green spaces, which would provide residents with privacy, green spaces and a sense of community.
4. Functional Zoning: Le Corbusier believed that cities should be divided into different functional
zones, such as residential, commercial, industrial, and green spaces. This would help to create an
efficient and effective urban environment, where different functions would be separated from each
other and not interfere with each other.
5. Modulor: Le Corbusier developed a system of proportions called the Modulor, which aimed to
provide a harmonious relationship between architecture, human proportions and the environ-
ment. The Modulor was based on the golden ratio and aimed to provide a system of proportions
that would ensure the aesthetic and functional success of buildings.
Q.10 Following information about income pattern is available from household survey of a community.
Category Monthly family income in rupees Number of families
1 Below 500 45
2 501 -1000 63
3 1001 -1500 213
4 1501 -2000 171
5 2001 -3000 76
6 Above 3000 33
Find out percentage of families earning a monthly income above Rs 1,500.
Draw a pi - diagram showing percentage distribution of various income groups.
Hemant Vilas Parulekar 839

Ans To find the percentage of families earning a monthly income above Rs 1,500, we need to add the num-
ber of families in categories 4, 5, and 6, and divide the sum by the total number of families surveyed.

Let’s call the total number of families surveyed “N”. N = 45 + 63 + 213 + 171 + 76 + 33 = 611.

The number of families earning a monthly income above Rs 1,500 is 171 + 76 + 33 = 280.

The percentage of families earning a monthly income above Rs 1,500 is (280 / 611) * 100 = 45.87%.

Q.11 Draw the bending moment and shear force diagram for the following :
Ans
Q.12 List six factors that are considered while selecting a tree for a landscape .
Ans There are several factors that should be considered when selecting a tree for a landscape:
1) Size: The size of the tree should be appropriate for the space it will be planted in, taking into account
the mature height and width of the tree.
2) Climate: The tree should be suited to the climate of the area where it will be planted. This includes
factors such as temperature, humidity, and precipitation levels.
3) Soil: The tree should be able to thrive in the soil conditions of the area, including pH level, drainage,
and nutrient levels.
4) Sun exposure: The tree should be able to tolerate the sun exposure of the area where it will be planted,
whether it is full sun, partial sun, or shade.
5) Root system: The tree’s root system should be compatible with the surrounding landscape, avoiding
interference with underground utilities or causing damage to nearby structures.
6) Aesthetics: The tree should be chosen for its appearance, including its shape, size, leaf texture, and
flower or fruit production.
Q.13 Draw the CPM network and determine the critical path from the following data:
Sl.No Activity Duration (days) Preceding activity
1 A 4 -
2 B 10 -
3 C 6 -
4 D 6 A
5 E 8 B
6 F 3 C
7 G 7 D
8 H 2 E
840 GATE Architecture and Planning: Comprehensive Question Bank

Ans The critical path is: B → E → H; the total time is 20 days

Q.14 Find the errors in the following FORTRAN program :


DIMENSION I (10)
REAL M
DO 100 M = 1,10
READ (*,200 / M)
100 CONTINUE
200 FORMAT (F 10.2)
STOP
END
Ans There are a few errors in the FORTRAN program:
1. The array “I” is not used in the program and can be removed.
2. The DO loop variable “M” is defined as a REAL type, but it is being used as an integer in the loop
control statement (DO 100 M = 1, 10). This will cause a type mismatch error. The loop variable
should be defined as an integer.
3. The READ statement is missing the input list. It should be written as “READ (*, 200) I(M)”.
4. The FORMAT statement should be written as “200 FORMAT (F10.2)” to match the input list.
5. The STOP statement should be placed after the READ statement, not after the FORMAT state-
ment.

Corrected code:

DIMENSION I(10)

INTEGER M

DO 100 M = 1, 10

READ (*,200) I(M)

100 CONTINUE

200 FORMAT (F10.2)

STOP

END
Q.15 Draw a sketch of a typical Greek Temple facade and name the important elements.
GATE QUESTION PAPER 1991
General Aptitude (GA)

Q.1 (i) Ratio ‘Golden Mean’ is:


(A) 1:2.216 (B) 1:1.618 (C) 1:1.50 (D) 1:1.44
Ans:- (B)
Explanation: please refer GATE-2002, Q.27 for more information.
(i) `Mihrab´ is found
(A) On the west wall of a mosque (B) inside wall of a mausoleum
(C) in the stepped well of Gujarat (D) on the crown of minaret
Ans:- (A)
Explanation: please refer GATE-2016, Q.15 for more information.
(ii) Which one falls under Indo-Saracenic architecture?
(A) Qutab Minar (B) Taj Mahal (C) Sher Shah’s tomb (D) Bahai Temple
Ans:- (B)
Explanation:
Structure Architecture style Description of Architectural style
Qutub Minar Indo Islamic Architecture • Fusion of Hindu and Islamic style
Taj Mahal • Introduction of decorative brackets, balconies,
etc.
SherShah’s tomb Indo-Sarasenic Architec- • Movement by British Architects in 19th century,
ture which also integrated native Indian, Indo-Islam-
ic architecture combined with Neo Classical and
Gothic revival.
• Also known as Indo-Gothic, Hindu-Gothic.
Bahai Temple Expressionist Architecture • Also known as lotus temple
• It gives the impression of the half open lotus.
• The building is composed of 27 free-standing
marble-clad “petals” arranged in clusters of three
to form nine sides

(iii) Which of the following is not a function of a good mulch :


(A) Aid in water retention (B) Prevent soil temperature fluctuations
(C) Encourage weed growth (D) Improve landscape appearance
Ans:- (C)
Explanation: please refer GATE-2003, Q.50 for more information.
(v) Byzantine architecture is famous for :
(A) Stone carving (B) Pointed arches
(C) Fluted columns (D)New type dome construction
Ans:- (D)
Explanation: Byzantine architects invented a technique for raising domes on piers. In architecture, a
triangular segment of a spherical surface, filling in the upper corners of a room, in order to form, at the
top, a circular support for a dome.

(vi) ‘Gopuram´ refers to


(A) Temple (B) Gateway (C) Village (D) Brick dome
Ans:- (B)
Explanation: Please refer GATE-2007, Q.23 for more information.
(vii) Most efficient arch in transferring load :
(A) Semi circular (B) Flat (C) Pointed (D) Catenary
Ans:- (D)
Explanation: An arch is a structural element that is used to span an opening or a space and transfer
load from one side to the other. The shape of an arch plays a significant role in its ability to transfer
load, and different arch shapes have different levels of efficiency in this regard.
A catenary arch is a type of arch that is shaped like a hanging chain, with a gentle curve that is formed
by the weight of the material. It is the most efficient arch shape in terms of load transfer, as it distrib-
utes the load evenly along the entire length of the arch.
In contrast, a semi-circular arch, a flat arch, and a pointed arch are less efficient at transferring load, as
they tend to concentrate the load at certain points, which can lead to higher stress concentrations and
a higher risk of failure.
Overall, the catenary arch is considered to be the most efficient arch shape in terms of load transfer,
and is often used in the design of bridges, arches, and other structures that require a high level of
load-bearing capacity.
(viii) Dimension of Corbusier ´s modular human scale are in :
(A) Arithmetic progression (B) Geometric progression
(C) Unrelated natural order (D) None of the above
Ans:- (C)
Explanation: The modular human scale is a concept developed by
the Swiss-French architect Le Corbusier in the mid-20th
century. It refers to a set of standardized dimensions that are based
on the dimensions of the human body, and are used as a basis for the
design of buildings and other structures.
(ix) Find odd one out of the following :
(A) T-trap (B) Q-trap (C ) S- trap (D) P- trap
Ans:- (A)
Explanation: T-trap also called as bottle trap is used with wash basins and kitchen sinks such that the
waste can be removed out in case of choke up. All other 3 traps are used with washing closets and waste
cannot be removed directly from it.
Hemant Vilas Parulekar 843

(x) pH value of treated domestic water supply should be between :


(A) 4.0 -5.5 (B) 5.5 - 6.5 (C ) 6.5 -8.0 (D) 8.0 -9.5
Ans:- (C)
Explanation: The pH of water is a measure of its acidity or basicity, and is an important factor to consid-
er when evaluating the quality of water for domestic use. The pH scale ranges from 0 to 14, with a pH of
7 being neutral, a pH less than 7 being acidic, and a pH greater than 7 being basic.
(xi) PERT is
(A) Node oriented (B) Cost oriented (C ) Activity oriented (D) Event oriented
Ans:- (D)
Explanation: Please refer GATE-2009, Q.1 for more information.
(xii) Gypsum is added in cement manufacture for the property of :
(A) Increasing the binding capacity (B) Retarding the setting time
(C) Reducing heat of hydration (D) Increasing strength
Ans:- (B)
Explanation: Gypsum is a soft sulfate mineral that is commonly used in the manufacture of cement. It
is added to cement in small amounts, typically between 3% and 8% by weight, in order to regulate the
setting time of the cement.
When gypsum is added to cement, it reacts with the other components of the cement, forming calcium
sulfoaluminate (C4A3S) crystals. These crystals help to slow down the setting time of the cement,
which is the time it takes for the cement to harden and become solid.
The addition of gypsum to cement is important because it helps to ensure that the cement has the right
setting time for the specific application. If the setting time is too fast, the cement may not have suffi-
cient time to properly hydrate and harden, which can lead to reduced strength and durability. On the
other hand, if the setting time is too slow, the cement may not be practical to use in certain applications.
(xiii) Critical path in CPM connects the initial and end events and :
(A) consists of events all having zero or minimum slack times
(B) consists of events all having maximum slack times
(C ) consists of events without dummy events
(D) None of the above
Ans:- (B)
(xiv) Maximum distance for discerning facial expression in a theatre :
(A) 12.5 m (B) 18.0 m (C ) 22.5 m (D) 30.0 m
Ans:- (A)
(xv) Outer layer of timber log is
(A) Heart wood (B) Strong wood (C ) Sap wood (D) Knot wood
Ans:- (C)
Explanation: Please refer GATE-2010, Q.23 for more information.
(xvi) Sun temple of Konark predominantly built with \
(A) Basalt stone (B) Soap stone (C ) Sand stone (D) Granite stone
Ans:- (C)
Explanation : Correct answer to be precise is Khondalite stone.
(xvii) Average power output (in microwatt ) of human speech in ordinary conversation is :
(A) 2.5 (B) 4.8 (C ) 6.2 (D) 10.0
844 GATE Architecture and Planning: Comprehensive Question Bank

Ans:-(D)
(xviii) Principal determinants of a residential neighbourhood size is based on:
(A) Landuse composition (B) Availability of vacant land
(C ) Education facility (D) Residential density
Ans:-(B)
(xix) Minimum height of habitable room as prescribed in NBC:
(A) 1.85 m (B) 2.75 m (C) 3.0 m (D) 3.2 m
Ans:- (B)
(xx ) Desirable housing layout of buildings form acoustic point of view is:
(A)Courtyard type (B) Stilted multistoried flats
(C ) Open type single or semidetached houses (D) none of above
Ans:- (C)
Q.2 (i) Match the following :
(a) The city in history (A) Patrick Geddes
(b) Life and death of great American cities (B) Norberg Schulz
(c) The modular (C ) John Ruskin
(d) The Future of Housing (D) Charles Abraham
(e) Seven Lamps of Architecture (E) Oscar Newman
(f) Language of Post- Modern Architecture (F) Lewis Mumford
(g) Cities in Evolution (G) Jean Jacob
(h) Defensible space (H) Le Corbusier
(i) The new Landscape (I) Charles Jenka
(j) Meaning in Western Architecture (J) Charles Correa
Ans:- (a)-(F), (b)-(G), (c)-(H), (d)-(D), (e)-(C), (f)-(I), (g)-(A), (h)-(E), (i)-(J), (j)-(B)
(iii) Match the following:
(a) Crystal Palace (A) Le Corbusier
(b) Johnson Wax factory (B) Joseph Paxton
(c) Shoden Villa (C )B.V.Doshi
(d) German Pavillion Barcelona (D) Mies Van der Rohe
(e) Pompidou Centre Paris (E ) Frank Lloyd Wright
(f) TWA Terminal, Kennedy Airport (F) Charles Correa
(g) Kanchanjunga Apartment, Bombay (G) Eero Saarinen
(h) Nat. Institute of Bank Management, Pune (H) A.P. Kanvinde
(i) I.I.M. Bangalore (I) Sarat Das
(j) Indira Gandhi Sports Complex, (J) Richard Rogers and Renzo Piano
Indraprastha
Ans:- (a)-(B), (b)-(E), (c)-(A), (d)-(D), (e)-(J), (f)-(G), (g)-(F), (h)-(H), (i)-(C), (j)-(I)
(iv) Match the following:
(a) Chandigarh (A) C.A. Doxiadis
(b) New Delhi (B) Otto Koenigsberger
(c) Gandhi Nagar (C )Mewada
(d) Islamabad (D) Le Corbusier
(e ) Bhubaneswar (E ) Edwin Lutyen
Ans:- (a)-(B), (b)-(E), (c)-(D), (d)-(C), (e)-(A)
Hemant Vilas Parulekar 845

(v) Match the following:


(a) Munsell Atlas (A) Gothic
(b) Low air speed (B) Landscape
(c) Kinaesthetia (C) Colour
(d) Jantar mantar (D) Raja Jai Singh
(e) Flying Buttress (E) Kata Thermometer
(f) Hypostyle Hall (F) Egyptian
(g) Humidity (G) Heliometer
(h) Solar radiation (H) Microbar
(i) Sound Pressure (I) Hygrograph
(j) Patina (J) Corrosion
Ans:- (a)-(D), (b)-(E), (c)-(C), (d)-(A), (e)-(B), (f)-(F), (g)-(I), (h)-(G), (i)-(), (j)-
Q.3 Indicate the following statement as TRUE or FALSE.
a. Over compaction of concrete is bad as it causes disintegration.
b. Hardness water is unsuitable for distribution because it is likely to be corrosive.
c. “Mulquf ” is a ventilating device used first by the Romans.
d. Aerobic bacteria require oxygen for their existence, and they thrive in the presence of light.
e. Baker is famous for slum upgradation scheme in India.
Ans a) True; over compaction results in segregation of content, as a result concrete becomes weak.
b) True; hard water is damaging to plumbing. It is corrosive and causes mineral build up, leading to
costly maintenance and repairs.
c) False; “Malquf ” (windcatcher) was first in Islamic-Arab world.
d) True; Aerobic bacteria require oxygen to perform cellular respiration and derive energy to survive
e) False; Laurie Baker renowned for his initiatives in cost-effective energy-efficient architecture and
designs that maximized space, ventilation and light and maintained an uncluttered yet striking aes-
thetic sensibility.
Q.4 Fill in the blanks:
a. The purpose of ________ is to expel voids and air bubbles in the concrete mass entrapped
during mixing.
b. In a computer system printer is an ______.
c. Floppy diskette is an ____________.
d. In a __________, the tendons are not initially embedded to the concrete.
e. ________ is an organism associated with the biological treatment of waste water.
f. ________ facilitates stack effect by warming the air and causing gentle convection current.
g. Bhilai is an example of ________.
h. New delhi is an example of ___________.
i. Notre -Dame is an example of _________.
j. Ratio of built-up area to plot area is defined as________.
Ans a) vibration, b) external device, c) external storage device, d) post - tensioned beams, e) Amoeba,
f) Sun light, g) steel Town, h) Capital Town, I) Gothic architecture, j) Plot coverage
Q.5 What do the following abbreviation stand for:
(a) INTACH: (b) N.B.O :
(c) E.W.S : (d) T.C.P.O :
(e) FORTRAN :
846 GATE Architecture and Planning: Comprehensive Question Bank

Ans (a) Indian National Trust for Art and Cultural Heritage
(b) National Building Organization
(c) Economically Weaker Section
(d) Town and Country Planning Organization
(e) Formula Translation
Q.6 Distinguish between:
(a) Urban and Rural
(b) Renewal and Redevelopment
(c) Intimate scale and monumental scale
(d) Acropolis and agora
(e) Loop street cul -de -sac
(f) C.C and R.C.C.
(g) Microclimate air and Macro climate air
(h) Harmony and contrast in colour
(i) Walk up and high-rise apartments
(j) In - situ and Pre- cast concrete
Ans (a) Urban areas are characterized by higher population density, a variety of land uses, and a greater range
of services and amenities. Rural areas, on the other hand, are typically more sparsely populated, with a
focus on agriculture and other resource-based industries.
(b) Renewal refers to the process of revitalizing and improving existing urban areas, often through the
renovation of existing buildings and the creation of new infrastructure. Redevelopment, on the other
hand, involves the complete transformation of an area through the demolition of existing structures and
the construction of new ones.
(c) Intimate scale refers to small, human-scale spaces and buildings that are intended to be experienced
up close and personally. Monumental scale, on the other hand, refers to larger, more impressive struc-
tures and spaces that are meant to be seen and experienced from a distance.
(d) The Acropolis is a fortified hill in ancient Athens, Greece, where the Parthenon and other important
buildings were located. The agora was a public marketplace and gathering place in ancient Greek cities.
(e) A loop street is a type of street that forms a loop or circular shape, with no dead ends. A cul-de-sac
is a type of street that ends in a dead end, with no through traffic.
(f) C.C (cast-in-situ) concrete refers to concrete that is poured into place on a construction site and al-
lowed to harden. R.C.C (reinforced concrete) refers to concrete that is reinforced with steel bars or mesh
to increase its strength and stiffness.
(g) Microclimate air refers to the local atmospheric conditions in a specific location, such as the tem-
perature, humidity, and wind patterns. Macroclimate air refers to the broader atmospheric conditions in
a larger region, such as the climate of a whole country or continent.
(h) Harmony in colour refers to the use of colours that are in balance and work well together, creating a
sense of unity and cohesiveness. Contrast in colour refers to the use of colours that are opposite on the
colour wheel, creating a sense of tension and interest.
(i) Walk-up apartments are multi-family residential buildings that are typically low-rise and do not have
elevators. High-rise apartments are multi-family residential buildings that are taller and have elevators.
(j) In-situ concrete refers to concrete that is poured into place on a construction site and allowed to
harden. Pre-cast concrete refers to concrete that is cast in a factory or offsite location and then transport-
ed to the construction site to be installed.
Hemant Vilas Parulekar 847

Q.7 Answer the following briefly:


(a) Indicate the factor that you would consider regarding the terrain for suitability of a site.
(b) Write the names according to hierarchy of streets and roads in an urban area.
(c) Highlight the major considerations for the preparation of land sub - division plan in an urban Area.
(d) Discuss the meaning and concept of a development plan.
(e) Outline the principal aspects for consideration in the development of settlement planning
Thoughts.
(f) What is meant by Day Light Factor? Name the components required for estimation of
Daylight Factor.
(g) Briefly describe the importance of chlorination in water treatment.
(h) Why Roman dome looks hemispherical inside but saucer shaped form outside?
(i) Outline the role of public participation in urban planning.
(j) State special architectural features associated with the following buildings Which made them differ-
ent form one another:
Ans (a) For the suitability of a site, the terrain factor that should be considered is the slope and topography
of the land. A site with a gentle slope and a stable topography is generally considered suitable for con-
struction.
(b) The hierarchy of streets and roads in an urban area is as follows:
• Major arterial roads: These are the main roads that connect different parts of the city and are
usually wider than other roads.
• Minor arterial roads: These are smaller in size and connect the major arterial roads to the local
streets.
• Collector roads: These are intermediate roads that collect traffic from local streets and distribute
it to the major arterial roads.
• Local streets: These are the smallest roads in an urban area and are usually used for access to
individual properties.
(c) Some major considerations for the preparation of land sub-division plans in an urban area include:
• The layout of the land: The plan should consider the natural features of the land, such as hills,
valleys, rivers, and streams.
• The zoning of the land: The plan should specify the different types of land use, such as residen-
tial, commercial, industrial, and recreational, and indicate the areas where each type of use is
allowed.
• The infrastructure of the land: The plan should include provisions for the necessary infrastruc-
ture, such as roads, utilities, and public facilities.
• The environmental impact of the land: The plan should consider the potential impact of the
development on the natural environment, such as the loss of wildlife habitat or the alteration of
natural drainage patterns.
(d) A development plan is a comprehensive document that guides the development of an area over a
certain period. It outlines the goals and objectives of the development, the type of land uses that are al-
lowed, and the infrastructure and facilities that are needed to support the development. The purpose of
a development plan is to ensure that the development of an area is coordinated and sustainable.
(e) Some principal aspects for consideration in the development of settlement planning thoughts in-
clude:
848 GATE Architecture and Planning: Comprehensive Question Bank

• Population density: The plan should consider the density of the population in the settlement, as
well as the types of housing and amenities that are needed to support the population.
• Transportation: The plan should consider the transportation needs of the settlement, including
the types of roads and public transportation systems that are needed.
• Land use: The plan should consider the types of land uses that are allowed in the settlement,
such as residential, commercial, and industrial.
• Environmental impacts: The plan should consider the potential impact of the settlement on the
natural environment, including the loss of natural habitats and the alteration of natural drainage
patterns.
(f) Daylight factor is a measure of the amount of natural light that enters a building. It is calculated by
dividing the amount of natural light that enters a building by the amount of light that would be received
if the sky were uniformly overcast. The components required for the estimation of daylight factor in-
clude the orientation of the building, the size and location of windows, and the surrounding buildings
and trees.
(g) Chlorination is an important step in the water treatment process because it helps to kill bacteria and
other contaminants that may be present in the water. Chlorine is added to the water in the form of a gas
or a liquid, and it reacts with the water to form a disinfectant that is effective at killing a wide range of
harmful microorganisms.
(h) Roman domes look hemispherical inside because they were constructed using a series of arches that
formed a circular shape. However, from the outside, Roman domes appear saucer-shaped because they
were built using a series of concentric rings that formed
Q.8 Explain the following with sketches:
(a) Ekistics grid
(b) Schematic diagram if four different urban forms.
(c) Difference between symmetric balance and asymmetric balance in visual composition.
(d) A square shape resting on its side looks vertically elongated or horizontally elongated.
Given one dimension ‘a’, draw a set of dimensions in Golden Mean Rectangle.
Ans Ekistics grid

(a) Schematic diagram of four different urban forms.


i. Linear City Ii. Grid City
Hemant Vilas Parulekar 849

Iii. Radial city Iv. Multi-nodal City

(c )Difference between symmetric balance and asymmetric balance in visual composition.


Symmetric balance in visual composition Asymmetric balance in visual composition

Q.9 (a) An urban area with a population of 2,15,000 is having housing stock of 39,000 and average
household size of 5.0 .The city is expected to have 2,70,000 by 2001 with an average Family size of
4.5. Estimate the housing demand of the city by 2001 assuming there will be depletion of existing
housing stock 3,500 during the period.
(b) Define the term “Affordable cost” in housing.
(c) Distinguish between ‘home’ and ‘housing’.
Ans To estimate the housing demand of the city by 2001, we need to first calculate the total number of
households in the city in 2001. We can do this by dividing the population of the city in 2001 (2,70,000)
by the average family size in 2001 (4.5). This gives us a total of 60,000 households.
Next, we need to subtract the number of housing units that will be lost due to depletion (3,500) from
the total number of housing units in the city in 2000 (39,000). This gives us a total of 35,500 housing
units in the city in 2001.
Finally, we can calculate the housing demand by subtracting the number of housing units in the city in
2001 (35,500) from the total number of households in the city in 2001 (60,000). This gives us a housing
demand of 24,500 units.
“Affordable cost” in housing refers to the amount of money that a person or household can reasonably
pay for housing without sacrificing other necessities or incurring excessive debt. This can vary based
on factors such as income, expenses, and the local housing market.
“Home” refers to a place where a person lives, while “housing” refers to the physical structure or struc-
tures that provide shelter for people. In other words, a home is a personal, emotional concept, while
housing is a more practical, physical concept.
Q.10 (a) The residential landuse of an urban area accounts for 50% of the developed land of the city. The
vacant undeveloped land is about 30% of the total urban area, which amounts to 2,400 hectare of
land. Estimate the quantum of land put to residential uses and also the overall density of the urban
area if the population is of 2,00,000 size.
(b) Illustrate with sketches the ‘Radburn principle’ of housing layout.
Ans (a) The quantum of land put to residential use is 50% of the total urban area, or 50% * 2,400 hectares =
1,200 hectares. The overall density of the urban area is calculated by dividing the population by the total
land area, or 200,000 / (1,200 + 2,400) = 200,000 / 3,600 = 55.56 people/hectare.
(b)
850 GATE Architecture and Planning: Comprehensive Question Bank

Q.11 (a) Draw the bending moment and shear force diagram for the following:
(b) Sketch the bending moment and shear force diagram (values not required):

Q.12 Explain the following planting techniques:


a. Grafting b. Layering c. Cutting d. Transplantation
Ans a. Grafting is a horticultural technique in which a piece of plant, called the scion, is joined with another
plant, called the rootstock. The scion is the plant that provides the desired qualities, such as the desired
fruit or flower type, while the rootstock provides the roots and base structure for the plant. Grafting is
often used to propagate plants that are difficult to grow from seed or to create plants with specific char-
acteristics.
b. Layering is a plant propagation technique in which a stem or branch is bent down to the ground and
covered with soil. The covered stem then develops roots, forming a new plant that can be separated from
the parent plant and grown on its own. Layering is often used to propagate woody plants, such as shrubs
and trees that do not easily root from cuttings.
c. Cutting is a plant propagation technique in which a piece of a plant, called a cutting, is taken from the
parent plant and grown in a separate container. The cutting is typically taken from a stem or branch and
has at least one set of leaves. The cutting is then rooted in soil or water, and once roots have developed,
it can be transplanted into a permanent container or into the ground.
d. Transplantation is the process of moving a plant from one location to another. This can involve mov-
ing a plant from one container to another, or from one location in the garden to another. Transplantation
is often necessary when a plant outgrows its current location or when it is necessary to move a plant to
a different location for aesthetic or functional purposes.
Q.13 Draw the CPM network diagram with the activities as shown below :
Sl.No Activity Preceding activity
1 A -
2 B A
3 C A
4 D C
5 E B
6 F E
7 G D
Ans

Q.14 Find the errors in the following FORTRAN program :


READ (5) A,B,C
20 FORMAT (3FI 0.4 )
D = (A+B )*C/5.0
PRINT *A,B,C
END
Hemant Vilas Parulekar 851

Ans There are a few errors in the given FORTRAN program:


1. The statement “READ (5) A, B, C” should be followed by a DO loop to read the values of A, B, and
C.
2. The FORMAT statement “20 FORMAT (3FI 0.4 )” is not correct. The correct format would be “20
FORMAT (3F10.4)”.
3. The PRINT statement “PRINT *A, B, C” should include a format specifier to specify the format in
which the values of A, B, and C should be printed. For example, “PRINT *, A, B, C” or “PRINT *, A, B,
C, F10.4”.
4. The program is missing a statement to print the value of D, which is calculated in the D =
(A+B)*C/5.0 statement.
Corrected FORTRAN program:
DO READ (5, 20) A,B,C
20 FORMAT (3F10.4)
D = (A+B )*C/5.0
PRINT *, A, B, C, D
END DO
END
Q.15 Illustrate with sketches the optical correction in Architecture developed by the Greek.

You might also like